You are on page 1of 530

EBD_7036

• Corp. Office : 45, 2nd Floor, Maharishi Dayanand Marg, Corner Market, Malviya Nagar, New Delhi-110017
Tel. : 011-26691021 / 26691713

How to access the ebook(s)?


Educore

INSTRUCTIONS
1. Mail your Order ID at ebooks.support@aiets.co.in to get the
16 digit Access Code.
2. Go to www.educoreonline.com/register.htm
3. Enter your details along with your 16 digit Access Code.
4. Click Register & you would be successfully redirected to the
Login Page.
Note: If you are already registered with us, you just have to
login (http://www.educoreonline.com/login.htm) & enter your
new 16 Character Unique Code under your Account Section.
Typeset by Disha DTP Team
5. Login with your registered email ID & password.
6. You can now view you e-book(s) under your Library.
7. You can read your e-books either Online or Offline. For offline,
simply download our Educore App once & download the e-
books inside the app. Educore App is available for Windows
Desktop, IOS & Android.
8. Educore works best in Desktop, Laptop, 7" & 10" tablets.
9. Contact us at support@educoreonline.com for any further
assistance.
Note: This app is not accessible on Mobiles.

DISHA PUBLICATION
ALL RIGHTS RESERVED

© Reserved

No part of this publication may be reproduced in any form without prior permission of the publisher. The author and the
publisher do not take any legal responsibility for any errors or misrepresentations that might have crept in. We have tried
and made our best efforts to provide accurate up-to-date information in this book.

For further information about the books from DISHA,


Log on to www.dishapublication.com or email to info@dishapublication.com
• JEE Advanced 2017 Solved Paper 1 - 15

• JEE Main 2017 Solved Paper 2017- 1 - 2017- 7

P-1 – P-204

CHAPTERS BASED ON CLASS 11th SYLLABUS

1. Units and Measurements 1-6

2. Motion 7 - 12

3. Laws of Motion 13 - 20

4. Work, Energy and Power 21 - 28

5. Momentum and Impulse 29 - 32

6. Rotational Motion 33 - 46

7. Gravitation 47 - 50

8. Mechanical Properties of Solids and Fluids 51 - 58

9. Heat & Thermodynamics and Gases 59 - 78

10. Simple Harmonic Motion (Oscillations) 79 - 86

11. Waves 87 - 96

CHAPTERS BASED ON CLASS 12th SYLLABUS

12. Electrostatics 97 - 114

13. Current Electricity 115 - 128

14. Moving Charges and Magnetism 129 - 144

15. Electromagnetic Induction and Alternating Current 145 - 156

16. Ray and Wave Optics 157 - 180

17. Modern Physics 181 - 204


EBD_7036
P-S-1 – P-S-300

1. Units and Measurements 1-6

2. Motion 7 - 15

3. Laws of Motion 16 - 26

4. Work, Energy and Power 27 - 36

5. Momentum and Impulse 37 - 43

6. Rotational Motion 44 - 68

7. Gravitation 69 - 75

8. Mechanical Properties of Solids and Fluids 76 - 86

9. Heat & Thermodynamics and Gases 87 - 117

10. Simple Harmonic Motion (Oscillations) 118 - 127

11. Waves 128 - 144

12. Electrostatics 145 - 173

13. Current Electricity 174 - 193

14. Moving Charges and Magnetism 194 - 216

15. Electromagnetic Induction and Alternating Current 217 - 235

16. Ray and Wave Optics 236 - 269

17. Modern Physics 270 - 300

* The chapters have been divided as per the Class 11th & 12th syllabus followed by the NCERT books. Some of the chapters which
are split in the class 11th & 12th syllabus in NCERT have been combined. There might be certain topics/ chapters which are not
covered in NCERT but are a part of JEE Advanced/IIT-JEE syllabus.
JEE ADVANCED 2017 - PHYSICS
PAPER - 1
SECTION - I mR
of the block M is: -
This section contains 7 questions. Each question has 4 options M+m
(A), (B), (C) and (D). ONE or MORE THAN ONE of these four
m
options is (are) correct. (D) The velocity of the block M is: V = - 2gR
M
1. A flat plate is moving normal to its plane through a gas 3. A block M hangs vertically at the bottom end of a uniform
under the action of a constant force F. The gas is kept at a rope of constant mass per unit length. The top end of the
very low pressure. The speed of the plate v is much less rope is attached to a fixed rigid support at O. A transverse
than the average speed u of the gas molecules. wave pulse (Pulse 1) of wavelength l0 is produced at point
Which of the following options is/are true? O on the rope. The pulse takes time TOA to reach point A. If
(A) The pressure difference between the leading and the wave pulse of wavelength l0 is produced at point A
trailing faces of the plate is proportional to uv (Pulse 2) without disturbing the position of M it takes time
(B) The resistive force experienced by the plate is TAO to reach point O. Which of the following options is/are
proportional to v correct?
(C) The plate will continue to move with constant non-
zero acceleration, at all times
(D) At a later time the external force F balances the resistive Pulse 1
O
force
2. A block of mass M has a circular cut with a frictionless
surface as shown. The block rests on the horizontal
frictionless surface of a fixed table. Initially the right edge of
the block is at x = 0, in a co-ordinate system fixed to the Pulse 2
table. A point mass m is released from rest at the topmost A M
point of the path as shown and it slides down.
When the mass loses contact with the block, its position is (A) The time TAO = TOA
x and the velocity is v. At that instant, which of the following (B) The velocities of the two pulses (Pulse 1 and Pulse 2)
options is/are correct? are the same at the midpoint of rope
(C) The wavelength of Pulse 1 becomes longer when it
R reaches point A
(D) The velocity of any pulse along the rope is
m independent of its frequency and wavelength
R
y 4. A human body has a surface area of approximately 1 m2.
The normal body temperature is 10 K above the surrounding
room temperature T0. Take the room temperature to be
x M
T0 = 300 K. For T0 = 300 K, the value of sT04 = 460 Wm-2
x=0 (where s is the Stefan-Boltzmann constant). Which of the
following options is/are correct?
(A) The amount of energy radiated by the body in 1 second
is close to 60 joules
mR (B) If the surrounding temperature reduces by a small
(A) The position of the point mass m is: x = - 2
M+m amount DT0 << T0, then to maintain the same body
temperature the same (living) human being needs to
2gR
radiate DW = 4sT03 DT0 more energy per unit time
(B) The velocity of the point mass m is: v = m
1+ (C) Reducing the exposed surface area of the body (e.g.
M
by curling up) allows humans to maintain the same
(C) The x component of displacement of the center of mass body temperature while reducing the energy lost by
radiation
EBD_7036
2 JEE Advanced 2017 Solved Paper
(D) If the body temperature rises significantly then the 7. For an isosceles prism of angle A and refractive index m, it is
peak in the spectrum of electromagnetic radiation found that the angle of minimum deviation dm= A.
emitted by the body would shift to longer wavelengths Which of the following options is/are correct?
5. A circular insulated copper wire loop is twisted to form two (A) For the angle of incidence i 1 = A, the ray inside the
loops of area A and 2A as shown in the figure. At the point prism is parallel to the base of the prism
of crossing the wires remain electrically insulated from each (B) For this prism, the refractive index µ and the angle of
other. The entire loop lies in the plane (of the paper). A
1 -1 æ m ö
® prism A are related as A = cos ç ÷
uniform magnetic field B points into the plane of the paper.. 2 è2ø
At t = 0, the loop starts rotating about the common diameter (C) At minimum deviation, the incident angle i1 and the
as axis with a constant angular velocity w in the magnetic refracting angle r 1 at the first refracting surface are
field. Which of the following options is/are correct? related by r1 = (i1/2)
(D) For this prism, the emergent ray at the second surface
will be tangential to the surface when the angle of
× × × × × × × × incidence at the first surface is
B
× × × × × × × ×
× × × × × × × × -1 é 2 A ù
i=
1 sin êsin A 4 cos - 1 - cos A ú
× × × ×area
× A× × × ë 2 û
× × × × × × × ×
× × × × × × × × SECTION - II
× × × × × × × × This section contains 5 questions. The answer to each question
× × × × × × × × is a SINGLE DIGIT INTEGER ranging from 0 to 9, both inclusive.
area 2A
× × × × × × × ×
× × × × × × × × 8. A drop of liquid of radius R = 10–2 m having surface tension
× × × × × × × × 0.1
× × × × × × × × S= Nm-1 divides itself into K identical drops. In this
w 4p
process the total change in the surface energy DU = 10–3 J.
If K = 10a then the value of a is
9. An electron in a hydrogen atom undergoes a transition from
(A) The emf induced in the loop is proportional to the sum an orbit with quantum number ni to another with quantum
of the areas of the two loops number n f. Vi and Vf are respectively the initial and final
(B) The amplitude of the maximum net emf induced due to
both the loops is equal to the amplitude of maximum Vi
emf induced in the smaller loop alone potential energies of the electron. If = 6.25, then the
Vf
(C) The net emf induced due to both the loops is smallest possible nf is
proportional to cos wt 10. A monochromatic light is travelling in a medium of refractive
(D) The rate of change of the flux is maximum when the index n = 1.6. It enters a stack of glass layers from the bottom
plane of the loops is perpendicular to plane of the side at an angle q = 30°. The interfaces of the glass layers
paper are parallel to each other. The refractive indices of different
6. In the circuit shown, L = 1 mH, C = 1 mF and R = 1 kW. They glass layers are monotonically decreasing as n m= n – mDn,
are connected in series with an a.c. source V = V0 sin wt as where nm is the refractive index of the mth slab and Dn = 0.1
shown. Which of the following options is/are correct? (see the figure). The ray is refracted out parallel to the
L = 1mH C = 1mF R = 1kW interface between the (m – 1)th and mth slabs from the right
side of the stack. What is the value of m?

~ V0 sin wt m n – mD n
m–1 n – (m – 1)Dn

(A) The current will be in phase with the voltage if w = 104


rad.s-1
(B) The frequency at which the current will be in phase 3 n – 3Dn
with the voltage is independent of R n – 2Dn
2
(C) At w ~ 0 the current flowing through the circuit n – Dn
1
becomes nearly zero
n
(D) At w >> 106 rad. s-1, the circuit behaves like a capacitor q
PHYSICS 3
11. A stationary source emits sound of frequency f0 = 492 Hz. labelled with 131I is injected into the blood of a person. The
The sound is reflected by a large car approaching the source activity of the amount of 131I injected was 2.4 × 105 Becquerel
with a speed of 2 ms–1. The reflected signal is received by (Bq). It is known that the injected serum will get distributed
the source and superposed with the original. uniformly in the blood stream in less than half an hour. After
What will be the beat frequency of the resulting signal in 11.5 hours, 2.5 ml of blood is drawn from person's body, and
Hz? (Given that the speed of sound in air is 330 ms–1 and gives an activity of 115 Bq. The total volume of blood in the
the car reflects the sound at the frequency it has received). person¡¦s body, in liters is approximately
12. 131I is an isotope of Iodine that B decays to an isotope of (you may use ex » 1 + x for |x| < < 1 and ln2 » 0.7).
Xenon with a half-life of 8 days. A small amount of a serum

SECTION - III
This section contains 6 questions of MATCHING TYPE, contains two tables each having 3 columns and 4 rows. Based on each table,
there are three questions. Each question has four options (A), (B), (C) and (D) ONLY ONE of these four options is correct.
Answer (Qs. 13-15) : By appropriately matching the information given in the three columns of the following table.
®
A charged particle (electron or proton) is introduced at the origin (x = 0, y = 0, z = 0) with a given initial velocity v . A uniform
® ® ® ® ®
electric field E and a uniform magnetic field B exist everywhere. The velocity v, electric field E and magnetic field B are given in
columns 1, 2 and 3, respectively. The quantities E0, B0 are positive in magnitude.
Column 1 Column 2 Column 3
® E0 ® ®
(I) Electron with v = 2 xˆ (i) E = E 0 zˆ (P) B = - B xˆ
B0 0

® E0 ® ®
(II) Electron with v = yˆ (ii) E = - E 0 yˆ (Q) B = B0 xˆ
B0

® ® ®
(III) Proton with v = 0 (iii) E = - E 0 xˆ (R) B = B0 yˆ

® E0 ® ®
(IV) Proton with v = 2 xˆ (iv) E = E 0 xˆ (S) B = B0 zˆ
B0

13. In which case will the particle move in a straight line with constant velocity?
(A) (III) (ii) (R) (B) (IV) (i) (S) (C) (III) (iii) (P) (D) (II) (iii) (S)
14. In which case will the particle describe a helical path with axis along the positive z direction?
(A) (IV) (i) (S) (B) (II) (ii) (R) (C) (III) (iii) (P) (D) (IV) (ii) (R)
15. In which case would the particle move in a straight line along the negative direction of y-axis (i.e., move along – ŷ )?
(A) (II) (iii) (Q) (B) (III) (ii) (R) (C) (IV) (ii) (S) (D) (III) (ii) (P)

Answer (Qs. 16-18) : By appropriately matching the information given in the three columns of the following table.
An ideal gas is undergoing a cyclic thermodynamic process in different ways as shown in the corresponding P–V diagrams in
column 3 of the table. Consider only the path from state 1 to state 2. W denotes the corresponding work done on the system. The
equations and plots in the table have standard notations as used in thermodynamic processes. Here Y is the ratio of heat capacities at
constant pressure and constant volume. The number of moles in the gas is n.

Column 1 Column 2 Column 3


1
(I) W1®2 = (P2 V2 - P1V1 ) (i) Isothermal (P) P 1
g -1 2

V
EBD_7036
4 JEE Advanced 2017 Solved Paper

(II) W1®2 = -PV2 + PV1 (ii) Isochoric (Q) P


1

P 1 2

(III) W1®2 = 0 (iii) Isobaric (R)

P
1

æV ö
(IV) W1®2 = -nRT ln ç 2 ÷ (iv) Adiabatic (S)
è V1 ø 2

16. Which of the following options is the only correct representation of a process in which DU = DQ – PDV?
(A) (II) (iv) (R) (B) (III) (iii) (P) (C) (II) (iii) (S) (D) (II) (iii) (P)
17. Which one of the following options is the correct combination?
(A) (IV) (ii) (S) (B) (III) (ii) (S) (C) (II) (iv) (P) (D) (II) (iv) (R)
18. Which one of the following options correctly represents a thermodynamic process that is used as a correction in the determination
of the speed of sound in an ideal gas?
(A) (I) (ii) (Q) (B) (IV) (ii) (R) (C) (III) (iv) (R) (D) (I) (iv) (Q)
PHYSICS 5

PAPER - 2
SECTION - I (A) ld/l (B) ld2 / l2
This section contains 7 questions. Each question has 4 options
(A), (B), (C) and (D). ONLY ONE of these four options is correct. (C) l3d / l (D) l3d / l2
4. A symmetric star shaped conducting wire loop is carrying a
1. Consider an expanding sphere of instantaneous radius R
steady state current I as shown in the figure. The distance
whose total mass remains constant. The expansion is such
between the diametrically opposite vertices of the star is 4a.
that the instantaneous density r remains uniform throughout
The magnitude of the magnetic field at the center of the
æ 1 drö loop is
the volume. The rate of fractional change in density ç
è r dt ÷ø
is constant. The velocity v of any point on the surface of
I
the expanding sphere is proportional to
(A) R (B) R3
1 4a
(C) (D) R2/3
R
2. Consider regular polygons with number of sides n = 3, 4,
5.... as shown in the figure. The center of mass of all the
polygons is at height h from the ground. They roll on a
horizontal surface about the leading vertex without slipping
and sliding as depicted. The maximum increase in height of m0 l m0 l
the locus of the center of mass for each polygon is D. Then (A) 6 [ 3 - 1] (B) 6 [ 3 + 1]
4 pa 4 pa
D depends on n and h as
m0 l m0l
(C) 3 [ 3 - 1] (D) 3 [2 - 3]
4 pa 4pa
r r r
5. Three vectors P, Q and R are shown in the figure. Let S be
r
any point on the vector R . The distance between the points
r r r
h h P and S is b | R |. The general relation among vectors P, Q
r
and S is

Y P b |R|

P R = Q- P
h S S
Q Q

æ ö O
ç ÷ X
1
æpö D = hç - 1÷ r r r
(A) D = h sin 2 ç ÷ (B) ç æpö ÷ (A) S= (1 - b)P + bQ
ènø ç cos ç n ÷ ÷
è è ø ø r r r
(B) S = (b - 1)P + bQ
r r r
æ 2pö 2æ p ö (C) S = (1 - b2 )P + bQ
(C) D = hsin ç ÷ (D) D = h tan ç ÷
è nø è 2n ø r r r
3. A photoelectric material having work–function f0 is (D) S = (1 - b)P + b 2 Q
6. A rocket is launched normal to the surface of the Earth,
æ hc ö
illuminated with light of wavelength l ç l < f ÷ . The away from the Sun, along the line joining the Sun and the
è 0ø Earth. The Sun is 3 × 105 times heavier than the Earth and is
fastest photoelectron has a de–Broglie wavelength ld. A at a distance 2.5 × 104 times larger than the radius of the
change in wavelength of the incident light by Dl result in a Earth. The escape velocity from Earth's gravitational field is
change Dld in ld. Then the ratio Dld/Dl is proportional to ve = 11.2 km s–1. The minimum initial velocity (vs) required
EBD_7036
6 JEE Advanced 2017 Solved Paper
for the rocket to be able to leave the Sun–Earth system is (D) For a fixed B, particles of same charge Q and same
closest to (Ignore the rotation and revolution of the Earth velocity v, the distance between the point P1 and the
and the presence of any other planet) point of re–entry into region 1 is inversely proportional
(A) vs= 22 km s–1 (B) vs= 42 km s–1 to the mass of the particle
(C) vs= 62 km s –1 (D) vs= 72 km s–1 9. The instantaneous voltages at three terminals marked X, Y
7. A person measures the depth of a well by measuring the and Z are given by
time interval between dropping a stone and receiving the Vx = V0 sin wt,
sound of impact with the bottom of the well. The error in his
æ 2p ö
measurement of time is dT = 0.01 seconds and he measures VY = V0 sin ç wt + ÷ and
the depth of the well to be L = 20 meters. Take the è 3 ø
acceleration due to gravity g = 10 ms–2 and the velocity of
æ 4p ö
sound is 300 ms–1 . Then the fractional error in the VZ = V0 sin ç wt + ÷
measurement, dL/L, is closest to è 3 ø
(A) 0.2% (B) 1% An ideal voltmeter is configured to read rms value of the
(C) 3% (D) 5% potential difference between its terminals. It is connected
between points X and Y and then between Y and Z. The
SECTION - II reading(s) of the voltmeter will be

This section contains 7 questions. Each question has 4 options rms 3


(A) VXY = V0
(A), (B), (C) and (D). ONE or MORE THAN ONE of these four 2
options is (are) correct.
rms 1
8. A uniform magnetic field B exists in the region between x = 0 (B) VYZ = V0
2
3R rms
and x = (region 2 in the figure) pointing normally into (C) VXY = V0
2
the plane of the paper. A particle with charge +Q and (D) Independent of the choice of the two terminals
momentum p directed along x–axis enters region 2 from region 10. A point charge +Q is placed just outside an imaginary
1 at point P1 (y = –R). Which of the following option(s) hemispherical surface of radius R as shown in the figure.
is/are correct? Which of the following statements is/are correct?

Q
Region 1 y Region 2 Region 3
× × ×
× × B × R
× × ×
× × ×
O × × × (A) The electric flux passing through the curved surface
× × × P2 x
+Q P1 æ
Q 1 ö
× × ×
of the hemisphere is - ç1 - ÷
(y = –R) × × × 2e0
è 2ø
× × × (B) Total flux through the curved and the flat surfaces is
3 R/2 Q
e0
2 p (C) The component of the electric field normal to the flat
(A) For B > , the particle will re–enter region 1 surface is constant over the surface
3 QR
(D) The circumference of the flat surface is an equipotential
8 p 11. Two coherent monochromatic point sources S1 and S2 of
(B) For B = , the particle will enter region 3 wavelength l = 600 nm are placed symmetrically on either
13 QR
side of the centre of the circle as shown. The sources are
through the point P2 on x–axis separated by a distance d = 1.8 mm. This arrangement
(C) When the particle re–enters region 1 through the produces interference fringes visible as alternate bright and
longest possible path in region 2, the magnitude of dark spots on the circumference of the circle. The angular
the change in its linear momentum between point P1 separation between two consecutive bright spots is Dq.
and the farthest point from y–axis is p/ 2 Which of the following options is/are correct?
PHYSICS 7
(A) The midpoint of the bar will fall vertically downward
P1 (B) The trajectory of the point A is a parabola
(C) Instantaneous torque about the point in contact with
the floor is proportional to sinq
Dq
(D) When the bar makes an angle q with the vertical, the
displacement of its midpoint from the initial position
P2 is proportional to (1 – cosq)
S1 S2
14. A wheel of radius R and mass M is placed at the bottom of
d a fixed step of height R as shown in the figure. A constant
force is continuously applied on the surface of the wheel so
that it just climbs the step without slipping. Consider the
torque t about an axis normal to the plane of the paper
(A) A dark spot will be formed at the point P2
passing through the point Q. Which of the following options
(B) At P2 the order of the fringe will be maximum
(C) The total number of fringes produced between P1 and is/are correct?
P2 in the first quadrant is close to 3000
(D) The angular separation between two consecutive S
bright spots decreases as we move from P1 to P2 along
the first quadrant
12. A source of constant voltage V is connected to a resistance Q
R and two ideal inductors L1 and L2 through a switch S as P
shown. There is no mutual inductance between the two R
inductors. The switch S is initially open. At t = 0, the switch X
is closed and current begins to flow. Which of the following
options is/are correct?
S
R
(A) If the force is applied at point P tangentially then
+ V L1 L2 decreases continuously as the wheel climbs

(B) If the force is applied normal to the circumference at
point X then t is constant
(A) After a long time, the current through L1 will be (C) If the force is applied normal to the circumference at
point P then t is zero
V L2 (D) If the force is applied tangentially at point S then t ¹ 0
R L1 + L 2 but the wheel never climbs the step
(B) After a long time, the current through L2 will be
SECTION - III
V L1
This section contains 2 paragraphs, each describing theory,
R L1 + L 2 experiments, data etc. four questions related to the two paragraphs
(C) The ratio of the currents through L1 and L2 is fixed at with two questions on each paragraph. Each question has only
all times (t > 0) one correct answer among the four given options (A), (B), (C) and
(D).
V
(D) At t = 0, the current through the resistance R is PARAGRAPH 1
R
13. A rigid uniform bar AB of length L is slipping from its vertical Consider a simple RC circuit as shown in Figure 1.
position on a frictionless floor (as shown in the figure). Process 1: In the circuit the switch S is closed at t = 0 and the
At some instant of time, the angle made by the bar with the capacitor is fully charged to voltage V0 (i.e., charging continues
vertical is q. Which of the following statements about its for time T >> RC). In the process some dissipation (ED) occurs
motion is/are correct? across the resistance R. The amount of energy finally stored in
the fully charged capacitor is EC.
V0
A Process 2: In a different process the voltage is first set to and
3
q
maintained for a charging time T >> RC. Then the voltage is raised
L
2V0
to without discharging the capacitor and again maintained
3
B for a time T >> RC. The process is repeated one more time by
O
EBD_7036
8 JEE Advanced 2017 Solved Paper
raising the voltage to V0 and the capacitor is charged to the same PARAGRAPH 2
final voltage V0 as in Process 1.
One twirls a circular ring (of mass M and radius R) near the tip of
These two processes are depicted in Figure 2.
one's finger as shown in Figure 1. In the process the finger never
S loses contact with the inner rim of the ring. The finger traces out
the surface of a cone, shown by the dotted line. The radius of the
R path traced out by the point where the ring and the finger is in
contact is r. The finger rotates with an angular velocity w0. The
V + C rotating ring rolls without slipping on the outside of a smaller

circle described by the point where the ring and the finger is in
contact (Figure 2). The coefficient of friction between the ring and
the finger is m and the acceleration due to gravity is g.
Figure 1

V
V0 Process 1
R

2V0 /3 r
Process 2 R
V0 /3 T > > RC Figure 1
Figure 2

T 2T t 17. The total kinetic energy of the ring is


Figure 2 1
(A) Mw02 R 2 (B) Mw20 (R - r)2
15. In Process 1, the energy stored in the capacitor EC and heat 2
dissipated across resistance ED are related by :
3
(A) EC = ED (B) EC = ED In 2 (C) Mw20 (R - r) 2 Mw20 (R - r)2
(D)
2
1 18. The minimum value of w0 below which the ring will drop
(C) EC = ED (D) EC = 2ED
2 down is
16. In Process 2, total energy dissipated across the resistance
ED is : g 2g
(A) (B)
m(R - r) m(R - r)
1 æ1 2ö
(A) ED = CV02 (B) ED = 3 ç CV0 ÷
2 è2 ø 3g g
(C) (D)
1æ 1 2m(R - r) 2m(R - r)

(C) ED = ç CV0 ÷ (D) ED = 3CV02
3è 2 ø
PHYSICS 9

SOLUTIONS
Paper - 1
1. (A, B, D) 2 gR
On solving we get, v = m
Flat plate Flat plate 1+
+ – v v M

m 2 gR
\V=
M m
u u v1 v2 1+
M
\ (C) is the correct option.
v 1 T
Before collision Just after collision 3. (A, D) We know that l = =
f f m
v1 - v v + v2 Where T = tension of string.
1= 1= Here To > TA \ lo > lA so option (C) is wrong.
v +u u–v
Velocity being a vector quantity has direction. The
\ v1 = u + 2v \ v2 = u - 2v velocities of the two pulses cannot be same at
\ Dv1 = 2u + 2v and Dv2 = 2u - 2v midpoint.
TAo = T oA because speed (or velocity) of wave
dp1 depends on mediums (and not on the wavelength or
Now F1 = = rA(u + v )(2u + 2v )
dt frequency of wave)
(A), (D) are the correct options
dp2 4. (A, B, C)
and F2 = = rA(u - v)(2 u - 2 v)
dt
Energy radiated = sA(T 4 - T0 4 ) t
\ F1 = 2rA(u + v) 2 and F2 = 2rA(u - v ) 2
[For a black body e = 1]
= sA[(T0 + 10)4 - T0 4 ]t
F2 DF é 4 ù
P= = 8ruv
F1 DF = F1 - F2 = 8rAuv \ A sAT 4 æ
ê
0 ç 1 +
10 ö
- 1ú t
= ÷
êëè T0 ø úû
The net force Fnet = F - DF = ma
4 é 40 ù 40
\ F - 8rAuv = ma = sAT0 ê T ú ´ t = 460 ´1´ 300 ´1 = 61.33J
ë 0û
2. (A, C) Let the block be displaced by x . If initially the centre
\ 'A' is a correct option
of mass of the system is at origin then
Energy radiated
M ´ x + m( x + R ) P= = sAT 4 - sAT0 4
O= time
M +m
- mR dp 3
O = Mx + mx + mR \ x= \ dT = sA(4T0 ) \ dp = sA(4T03 )dT0
m+M 0

\ 'A' is the correct option


\ DP = 4sAT03
If v is the velocity of mass 'm' as it leaves the block
and V is the velocity of block at that instant then 'B' is also a correct option
according to conservation of linear momentum Energy radiated µ A where A is the surface area of
mv = MV the body
By energy conservation \ 'C' is the correct option
5. (A, D) For smaller loop f = BA cos wt
1 2 1 2 The rate of change of flux
mgR = mv + MV
2 2
EBD_7036
10 JEE Advanced 2017 Solved Paper

df A
= – BAw sin wt
dt
df i1 P Q
For to be maximum, sin wt should be maximum r2
dt
and this will happen when wt = 90° i.e., the plane of
loop is perpendicular to the plane of paper. B C
Option (A) is correct. Emergent ray tangential
The emf produced will oppose each other. The net to the surface
emf will also be proportional to sin wt.
enet = B(2A)wsin wt – BAw sin wt = BAw sin wt But r1 + r2 = A
\ option (D) is also correct. \ r1 = A – r2
6. (A, B) The angular frequency at which the current and -1 æ 1 ö
voltage will be at same phase is \ r1 = A – sin ç ÷
èmø
1 1 Applying Snell's law at 'P' we get
wr = = -6 -6 1/2
= 106 rad s –1
LC (10 ´10 )
sin i1 -1 é -1 1 ù
m= \ i1 = sin êm sin( A - sin )
This value is independent of 'R' So (A) is correct sin r1 ë m úû
option.
For minimum deviation PQ || BC
V \ (D) is also a correct option.
At w » 0 , the current i =
1 2
R 2 + (w L - ) 8. (6) DU = S[k ´ 4pr 2 - 4pR 2 ]
wC
é 4 3 4 3ù
i = 0 (The circuit behaves as d.c circuit) êë where 3 pR = k ´ 3 pr úû
\ B is a correct option 1
If w >> w0, circuit behaves as an inductor. \ R=K 3r

7. (A, B, D)
For minimum deviation (when i1 = A) é R2 ù
\ DU = 4ps ê k ´ 2/3 - R ú = 4pSR éëk - 1ùû
2 2 1/3
i1 = e ë k û
A
r1 = r2 = r (say) = \ DU = 4pSR 2 éë10a /3 - 1ùû
2
0.1
A \ 10–3 = 4p ´ ´ (10-2 ) 2 ëé10a /3 - 1ûù
4p

Q \ 10 2 = 10 a /3 - 1
i1 P e
r1 r2 a
Neglecting 1 we get 102 = 10a/3 \ =2 \ a=6
3
B C
n 2 n
dm = 2i1 - A , 9. (5) Here U i = f = 6.25 \ f = 2.5
Uf 2 ni
ni
Here dm = A \ i1 = A \ e=A
If ni = 2 then n f = 5
i 10. (8) Here n × sin 30° = [n – m × 0.1] sin 90°
\ r1 = 1 \ 1.8 × sin 30° = 1.8 – m × 0.1 \ m = 8
2
option (A) is correct é 332 ù
11. (6) Frequency perceived by reflector = f1 = 492 ê
sin i1 sin A ë 330 úû
m= =
sin r1 sin A / 2 Frequency perceived by the source f2
2sin A / 2 cos A / 2 é 332 ù é 330 ù
= = 492 ê ´ = 498Hz
sin A / 2
= 2 cos A / 2 ë 330 úû êë 328 úû
\ option (B) is correct \ Beat frequency = 498 – 492 = 6Hz
Applying Snell's law at Q 12. (5) A = A0e–lt \ A0 = Aelt = 115 (1 + lt)
sin 90° 1 -1 æ 1 ö é ln 2 ù
m= = Þ r2 = sin ç ÷ \ A0 = 115 ê1 + ´ 11.5ú
sin r2 sin r2 èmø ë t1/ 2 û
PHYSICS 11
The force due to magnetic field FB will provide the
é 0.7 ù necessary centripetal force for circular motion which
= 115 ê1 + ´ 11.5ú
ë 8 ´ 24 û will be in X-Y plane. The force due to electric field
A0 » 120 Bq will accelerate proton in Z-direction. Thus the path
will be helical with increasing pitch.
120 Bq activity level is in 2.5 ml
\ 2.4 × 10 5 Bq activity level will be in 15. (D) x
5
2.5 ´ 2.4 ´10
ml
120 E = - E0 ˆj

2.5 ´ 2.4 ´ 105 vel = 0


l = 5l
120 ´1000
13. (A) For the particle to move in straight line , electric force Z
should be equal and opposite to the magnetic force.
(A) is the correct option.
r r B = B0 ˆj
FE = -eE = -e(- E0 xˆ ) = eE0 xˆ
Y
r r éE ù
FB = q(v ´ B) = -e ê 0 yˆ ´ B0 zˆ ú The electric field will apply a force on –Y axis thereby
ë B0 û accelerating the change along –Y axis.
r FB = qv B sin q
FB = -eE0 xˆ
Here q = 180° therefore , FB = 0
16. (B) DU = DQ – PDV
x
14. (C) show that work done = PDV
which is the formula for isobaric process.
2E0 17. (A) Work done in isochoric process is zero for which we
xˆ FB
B0 get a vertical line in P-V graph.
18. (D) Laplace's correction of the speed of sound in ideal
gas is related to adiabatic process.
B
Z
E

Y
EBD_7036
12 JEE Advanced 2017 Solved Paper

Paper - 2
1 dr OC
1. (A) = constant In D OAC cos 60° =
r dt OA

4pR 3 d é m ù 1
\ OC = 2a ´ =a
\ 3m dt ê 4 ú = constant 2
ê pR3 ú The magnetic field at 'O' due to
ë3 û
m0 I
d AB = [ sin 60° - sin 30°]
\ R
3
( R -3 ) = constant 4p a
dt
m0 I é 3 1 ù m0 I 1
3 -4 dR = ê - ú= ´ ( 3 - 1)
\ R ( -3 R ) = constant 4 p a ë 2 2 û 4pa 2
dt
The total magnetic field due to all the straight segments
dR of the star is
\ µR
dt é m0 I 1 ù m I
=ê ´ ( 3 - 1) ú ´12 = 0 ´ 6( 3 - 1)
O ë 4p a 2 û 4p a
O
r r
h r r r r S -P
p/n p/n 5. (A) Here P + bR = S \ R =
h A b
2. (B) r r r
Also R = Q - P
A B B
r r
S-P r r r r r r
\ = Q - P \ S - P = bQ - bP
p OA h b
In DOAB cos = \ OB = p
r r r
n OB cos \ S = bQ + (1 - b) P
n
1 GM e m GM e m ´ 3 ´105
h é 1 ù 6. (B) mVe2 - - =0
D= -h = hê -1 2 Re 2.5 ´104 Re
p p ú
cos ê cos ú
n ë n û Ve2 GM e é 3 ´105 ù
= ê1 + ú
hC p2 h2 2 Re ë 2.5 ´104 û
3. (D) - f 0 = K .E = =
l 2me 2me l d 2
æ 2GM e ö
Differentiating on both sides Ve = 13 ç ÷ = 13 ´11.2 » 42
è Re ø
-hC h2 æ -2 ö
dl = ç 3 ´ d ld ÷ 2L L
l2 2me l
è d ø 7. (B) T = +
g v
d ld l 3 with error limits
\ µ d2
dl l 2( L + dL) L + dL
T + dT = +
A
g v
A
4. (B)
2 L æ dL ö L æ dL ö
30° \ T + dT = ç1 + ÷ + ç1 + ÷
B I B g è L ø vè L ø
2a
60°
30° 30° 2L æ dL ö L æ dL ö
C 60°
\ T + dT = ´ ç1 + ÷ + ç1 + ÷
4a g è 2L ø v è L ø
O
O
2L 2 L dL L L dL
\ T + dT = + + +
g g 2L v v L
PHYSICS 13

2 L dL L dL Also CP2 = CO 2 + OP2 2


T + dT = T + +
g 2L v L
2 2
æ 5 R ö æ 3R ö
dL é 1 2L L ù = ç ÷ +ç ÷
è 8 ø è 2 ø
dT = L ê 2 g + v ú
êë úû
13 R
Substituting dT = 0.015, L = 20 m, g = 10 ms–2, CP2 =
8
v = 300 ms–1
We get
dL 15
=
L 1600
dL 15 15 p
´ 100 = ´ 100 = % » 1%
\ L 1600 16
8. (A, B) For the charge +Q to return region 1, the radius of
3R
the circular path taken by charge should by
2
mv 2 2p p
= QvB \ = QB
(3R / 2) 3R

2p
\ B = 3QR
Thus the particle will enter region 3 through the point P1 on
2p X-axis
i.e., B should be equal or greater than 'B' is the correct option.
2QR
Change in momentum = 2p
'A' is the correct option.
Thus 'C' is incorrect
8p
When B = mv
13QR mv 2
Further = qvB \ r = \ rµm
r qB
mv 2 æ 8p ö 13 R \ 'D' is incorrect.
= Qv ç ÷ \ r=
r è 13QR ø 8
9. (A, C) The potential difference between X and Y is
Thus 'C' is the of the centre of circular path of radius
VXY = V X - VY
13 R
8 VXY = (VXY )0 sin(wt + q1 )

2 2 2 2p
Y where (V XY ) 0 = V0 + V0 - 2V0 cos = 3V0
3

(VXY ) 0 3
and (V XY )rms = = V0
2 2
(A) is the correct option
C Now the potential difference between Y and Z is
5R
VYZ = VY - VZ
8
O X VYZ = (VYZ )0 sin(wt + q2 )
R P2
3R / 2 2p
2 2 2
Where (VYZ )0 = V0 + V0 - 2V0 cos = 3V0
3
P1
+Q (VYZ ) 03
and (VYZ )rms = =
V0
2 2
Thus (C) is the correct option.
EBD_7036
14 JEE Advanced 2017 Solved Paper
Now path difference
10. (A, D) P +Q p = dcos q = nl (for bright fringe)
nl
45° q \ cos q =
d
l
\ - sin q Dq = (Dn)
d
Dnl
or Dq = -
d sin q
E
Ecos q As we move from p1 to p2, q decreases and therefore
The circumference of the flat surface is an Dq increases. Therefore (D) is incorrect.
equipotential because the distance of each point on 12. (A, B, C)
the circumference is equal from + Q After a long time the current through the resistor is
(D) is the correct option. constant I will divide into two parts L1 and L2 which
The component of electric field normal to the flat are in parallel
surface is Ecosq. \ I1L1 = I2L2
Here E as well as q changes for different point on the V é L2 ù
flat surface. Therefore (C) is incorrect. Further I1 = R ê L + L ú
The total flux through the curved and flat surface ë 1 û
Q V é L1 ù
should be less than e . Therefore (B) is incorrect. I2 =
0
and R êë L1 + L2 úû
The solid angle subtended by the flat surface at Also the ratio of currents through L1 and L2 is fixed
æ 1 ö at all times At t = 0, I » 0
P = 2p ç 1 - ÷ 13. (A, C, D)
è 2ø
As Fx = 0, ax = 0. Therefore the force acting in vertical
\ Flux passing through curved surface direction will move the mid point of the bar fall
æ 1 ö vertically downwards. (A) is correct option.
2p ç 1 - ÷
Q' è 2ø Q æ 1 ö.
=- =- ç 1- ÷
e0 4p 2e 0 è 2ø Y
A (0, L)
(A) is the correct option.
11. (B, C)
Path difference at P2 is
p = S1P2 – S2P2 = d = 1.8 mm = 1.8 × 10–3m
= 3000 × 600 × 10–9 m
p = 3000 l. (x, y)
As the path difference is an integral multiple of l, P2 90 – q
B
should be a bright fringe with 300th maxima. (A) is
incorrect.
q
Further at P1, path difference = 0. Therefore a bright
fringe will be present at P1 also. Therefore total
number of fringes between P1 and P2 is 3000. (C) is a
correct option. L/2 L/2
Obviously at P2 the order of the fringe will be q L
mg cos q
maximum. Thus (B) is a correct option. 2

P1 X
P L
sin q
Rdq 2

q When the bar makes an angle q with the vertical, the


os
dc displacement of its mid point from the initial position
q P2
S1 S2 L L
d is - cos q
2 2
(D) is a correct option.
PHYSICS 15
Instantaneous torque about the point of contact P is 15. (A) Work done by battery = q × V
\ W = CVo × Vo = CVo2
L
t = mg ´ sin q
2 1
Energy stored in the battery = CV02
(C) is a correct option. 2
\ Energy dissipated
L
Now x = sin q , y = L sin(90 - q) = L cos q 1 2 1 2
2 ED = W – EC = CV02 - 2 CV0 = 2 CV0
2 2
æ 2x ö æ y ö 4x2 y2 \ EC = ED
\ ç ÷ + ç ÷ = 1 or 2 + 2 = 1
è L ø è Lø L L 16. (C) Let Vi and Vf be the initial and final voltage in each
This is equation of ellipse. Therefore B is incorrect process. Then
14. (C) If the force is applied at P tangential than the I remains Energy dissipated = Wbattery – DU
constant and is equal to F × 2R where F is the applied 1
force (A) is incorrect. = C(Vf –Vi )Vf – C(Vf – Vi ) 2
If force is applied normal to X, then as the wheels 2
climbs, then the perpendicular distance of force from 1
Q will go on changing initially the perpendicular is = C(Vf – Vi ) 2
2
QM, later it becomes QM'. (B) is incorrect \ Total heat dissipated

1 éæ Vo 2V ö ù
2 2 2
ö æ 2V V ö æ
F ED = C êç - 0÷ + ç o - o ÷ + çVo - o ÷ ú
2 ëè 3 ø è 3 3ø è 3 ø û

x F 1
M
= CVo 2
Q
6
æ R - rö
17. (C) Here w0 (R - r ) = wR \ w = wo çè ÷
R ø
x Now total kinetic energy of the ring (Kinetic rotational
+ kinetic translational)
If the force is applied normal to the circumference at
point P then I is zero. So (C) is correct. 1
K .Etotal = (2 MR 2 )w2 = M w0 2 ( R - r ) 2
If the force is applied tangentially at point S then 2
t = F ´ R and the wheel will climb. 18. (A) mM w2min ( R - r ) = Mg
(D) is incorrect
\ wmin = g
m( R - r )
EBD_7036
JEE MAIN 2017 - PHYSICS
Held On : 2nd April, 2017
1. A particle is executing simple harmonic motion with a time (2) real and at a distance of 6 cm from the convergent lens
period T. At time t = 0, it is at its position of equilibrium. The (3) real and at a distance of 40 cm from convergent lens
(4) virtual and at a distance of 40 cm from convergent
kinetic energy-time graph of the particle will look like: lens.
7. The moment of inertia of a uniform cylinder of length l and
radius R about its perpendicular bisector is I. What is the
(1) (2) ratio l/R such that the moment of inertia is minimum ?
3
(1) 1 (2)
2
(3) (4) 3 3
(3) (4)
2 2
2. The temperature of an open room of volume 30 m3 increases 8. An electron beam is accelerated by a potential difference V
from 17°C to 27°C due to sunshine. The atmospheric pressure to hit a metallic target to produce X-rays. It produces
in the room remains 1 × 105 Pa. If ni and nf are the number of continuous as well as characteristic X-rays.If lmin is the
molecules in the room before and after heating, then nf – ni will smallest possible wavelength of X-ray in the spectrum, the
be : variation of log lmin with log V is correctly represented in :
(1) 25 (2) -2.5 ´ 1025
2.5 ´ 10
(1) (2)
(3) 23 (4) 1.38 ´ 1023
-1.61 ´ 10
3. Which of the following statements is false ?
(1) A rheostat can be used as a potential divider (3) (4)
(2) Kirchhoff's second law represents energy conservation
(3) Wheatstone bridge is the most sensitive when all the 9. A radioactive nucleus A with a half life T, decays into a
four resistances are of the same order of magnitude nucleus B. At t = 0, there is no nucleus B. At sometime t, the
(4) In a balanced wheatstone bridge if the cell and the ratio of the number of B to that of A is 0.3. Then, t is given by
galvanometer are exchanged, the null point is T
disturbed. (1) t = T log (1.3) (2) t =
log(1.3)
4. The following observations were taken for determining
log 2 T log1.3
surface tensiton T of water by capillary method : (3) t = T (4) t = T
Diameter of capilary, D = 1.25 × 10–2 m log1.3 log 2
ur
rise of water, h = 1.45 × 10–2 m 10. An electric dipole has a fixed dipole moment p , which makes
Using g = 9.80 m/s2 and the simplified relation angle q with respect to x-axis. When subjected to an electric
uur ur
rhg field E1 = Eiˆ , it experiences a torque T1 = t iˆ . When
T= ´ 10 3 N/m, the possible error in surface tension is uur
2 subjected to another electric field E2 = 3E1 ˆj it
closest to : uur ur
(1) 2. 4 % (2) 10 % experiences torque T2 = -T1 . The angle q is :
(1) 60° (2) 90°
(3) 0.15% (4) 1.5%
(3) 30° (4) 45°
5. In amplitude modulation, sinusoidal carrier frequency used 11. In a common emitter amplifier circuit using an n-p-n
is denoted by ωc and the signal frequency is denoted by ω m . transistor, the phase difference between the input and the
output voltages will be :
The bandwidth ( Dωm ) of the signal is such that Dω m < ωc . (1) 135° (2) 180°
Which of the following frequencies is not contained in the (3) 45° (4) 90°
modulated wave ? 12. Cp and C v are specific heats at constant pressure and
(1) ωm + ωc (2) ωc - ωm constant volume respectively. It is observed that
Cp – Cv = a for hydrogen gas
(3) ωm (4) ωc Cp – Cv = b for nitrogen gas
6. A diverging lens with magnitude of focal length 25 cm is The correct relation between a and b is :
placed at a distance of 15 cm from a converging lens of
magnitude of focal length 20 cm. A beam of parallel light (1) a = 14 b (2) a = 28 b
falls on the diverging lens. The final image formed is : 1
(1) real and at a distance of 40 cm from the divergent lens (3) a = b (4) a = b
14
2017-2 JEE MAIN 2017 Solved Paper
13. A copper ball of mass 100 gm is at a temperature T. It is 19. In a coil of resistance 100 W , a current is induced by
dropped in a copper calorimeter of mass 100 gm, filled with changing the magnetic flux through it as shown in the figure.
170 gm of water at room temperature. Subsequently, the The magnitude of change in flux through the coil is
temperature of the system is found to be 75°C. T is given by
(Given : room temperature = 30° C, specific heat of copper =
0.1 cal/gm°C
(1) 1250°C (2) 825°C
(3) 800°C (4) 885° C
14. –2
A body of mass m = 10 kg is moving in a medium and
experiences a frictional force F = –kv2. Its intial speed is v0 (1) 250 Wb (2) 275 Wb
1 2 (3) 200 Wb (4) 225 Wb
= 10 ms–1. If, after 10 s, its energy is mv0 , the value of k
8 20. In a Young's double slit experiment, slits are separated by
will be: 0.5 mm, and the screen is placed 150 cm away. A beam of
(1) 10–4 kg m–1 (2) 10–1 kg m–1 s–1 light consisting of two wavelengths, 650 nm and 520 nm, is
–3
(3) 10 kg m –1 (4) 10–3 kg s–1 used to obtain interference fringes on the screen. The least
15. When a current of 5 mA is passed through a galvanometer distance from the common central maximum to the point
having a coil of resistance 15 W , it shows full scale where the bright fringes due to both the wavelengths
deflection. The value of the resistance to be put in series coincide is :
with the galvanometer to convert it into to voltmeter of range (1) 9.75 mm (2) 15. 6 mm
0 - 10 V is (3) 1.56 mm (4) 7.8 mm
(1) 2.535 ´ 103 W (2) 4.005 ´ 103 W 21. A magnetic needle of magnetic moment 6.7 × 10–2 Am2 and
(3) 1.985 ´ 10 W3 (4) 2.045 ´ 103 W moment of inertia 7.5 × 10–6 kg m2 is performing simple
harmonic oscillations in a magnetic field of 0.01 T. Time
16. A slender uniform rod of mass M and length l is pivoted at taken for 10 complete oscillations is :
one end so that it can rotate in a vertical plane (see figure). (1) 6.98 s (2) 8.76 s
There is negligible friction at the pivot. The free end is held (3) 6.65 s (4) 8. 89 s
vertically above the pivot and then released. The angular 22. The variation of acceleration due to gravity g with distance
acceleration of the rod when it makes an angle q with the d from centre of the earth is best represented by (R = Earth's
vertical is radius):
g

(1) (2)
d
O R
3g 2g g
(1) cos q (2) cos q
2l 3l
3g 2g
(3) sin q (4) sin q (3) (4)
2l 2l d
17. Some energy levels of a molecule are shown in the figure.
O R
The ratio of the wavelengths r = l1 / l2 , is given by
23.

In the above circuit the current in each resistance is


(1) 0.5A (2) 0 A
3 1 (3) 1 A (4) 0.25 A
(1) r= (2) r =
4 3 24. A particle A of mass m and initial velocity v collides with a
4 2 m
(3) r = (4) r = particle B of mass which is at rest. The collision is head
3 3 2
18. A man grows into a giant such that his linear dimensions on, and elastic. The ratio of the de-Broglie wavelengths
increase by a factor of 9. Assuming that his density remains lA to lB after the collision is
same, the stress in the leg will change by a factor of
1 lA 2 lA 1
(1) 81 (2) (1) l = (2) l = 2
81 B 3 B

1 lA 1 lA
(3) 9 (4) (3) = (4) =2
9 lB 3 lB
EBD_7036
PHYSICS 2017-3
25. An external pressure P is applied on a cube at 0 oC so that it r2 r1
is equally compressed from all sides. K is the bulk modulus (1) CE (2) CE
of the material of the cube and a is its coefficient of linear (r + r2 ) (r1 + r)
expansion. Suppose we want to bring the cube to its original r1
size by heating. The temperature should be raised by : (3) CE (4) CE
(r2 + r)
3a
(1) (2) 3PKa 29. A capacitance of 2m F is required in an electrical circuit
PK
P P across a potential difference of 1.0 kV. A large number of
(3) (4)
3a K aK 1m F capacitors are available which can withstand a potential
26. A time dependent force F = 6t acts on a particle of mass 1 kg. difference of not more than 300 V. The minimum number of
If the particle starts from rest, the work done by the force
during the first 1 secand will be capacitors required to achieve this is
(1) 9 J (2) 18 J (1) 24 (2) 32
(3) 4.5 J (4) 22 J (3) 2 (4) 16
27. An observer is moving with half the speed of light towards 30. A body is thrown vertically upwards. Which one of the
a stationary microwave source emitting waves at frequency
10 GHz. What is the frequency of the microwave measured following graphs correctly represent the velocity vs time?
by the observer? (speed of light = 3 × 108 ms–1)
(1) 17.3 GHz (2) 15.3 GHz
(3) 10.1 GHz (4) 12.1 GHz (1) (2)
28. In the given circuit diagram when the current reaches steady
state in the circuit, the charge on the capacitor of capacitance
C will be :

(3) (4)
2017-4 JEE MAIN 2017 Solved Paper

SOLUTIONS
1. (2) For a particle executing SHM
R1 R3
At mean position; t = 0, wt = 0, y = 0, V = Vmax = aw On balancing condition =
R2 R
1
\ K.E. = KEmax = mw2a2 rhg
2 4. (4) Surface tension, T = ´ 103
T p 2
At extreme position : t = , wt = , y = A, V = Vmin = 0
4 2 DT Dr Dh
\ K.E. = KEmin = 0 Relative error in surface tension, = + +0
1 T r h
Kinetic energy in SHM, KE = mw2(a2 – y2) (Q g, 2 and 103 are constant)
2
1 Percentage error
= mw2a2cos2wt
2
Hence graph (2) correctly depicts kinetic energy time graph. DT æ 10 –2 ´ 0.01 10 –2 ´ 0.01ö
100 ´ =ç + ÷ 100
2. (2) Given: Temperature Ti = 17 + 273 = 290 K T è 1.25 ´ 10 –2 1.45 ´ 10 –2 ø
Temperature Tf = 27 + 273 = 300 K = (0.8 + 0.689)
Atmospheric pressure, P0 = 1 × 105 Pa = (1.489) = 1.489% @ 1.5%
Volume of room, V0 = 30 m3 5. (3) Modulated carrier wave contains frequency wc and
Difference in number of molecules, Nf – Ni = ? wc ± wm
The number of molecules 6. (3) As parallel beam incident on diverging lens will form
PV image at focus.
Þ N= (N )
RT 0 \ v = –25 cm
P0V0 æ 1 1 ö 15 cm
\ Nf – Ni = ç - ÷N
R è T f Ti ø 0
1 ´ 105 ´ 30 æ 1 1 ö
= ´ 6.023 ´ 10 23 ç - ÷
8.314 è 300 290 ø
= – 2.5 × 1025
3. (4) There is no change in null point, if the cell and the
galvanometer are exchanged in a balanced wheatstone
bridge.

f = –25 cm f = 20 cm
The image formed by diverging lens is used as an object for
converging lens,
So for converging lens u = –25 – 15 = –40 cm, f = 20 cm
On balancing condition \ Final image formed by converging lens
R1 R2 1 1 1
= - =
R3 R4 V -40 20
or, V = 40 cm from converging lens real and inverted.
After exchange
7. (3) As we know, moment of inertia of a solid cylinder about
an axis which is perpendicular bisector
mR 2 ml 2
I= +
4 12

m é 2 l2 ù
I= êR + ú
4ë 3û
l
EBD_7036
PHYSICS 2017-5
Let V = Volume of cylinder = pR2l From eqns. (i) and (ii)

m é V l2 ù dI m é -V 2l ù pE sinq = 3 pE cosq
= ê + ú Þ = + =0
4 ë pl 3 û dl 4 êë pl 2 3 úû tanq = 3 \ q = 60°
V 2l 2pl 3 11. (2) In common emitter configuration for n-p-n transistor
= ÞV = input and output signals are 180° out of phase i.e., phase
pl 2 3 3
difference between output and input voltage is 180°.
2pl 3 l2 3 l 3
pR 2 l = Þ 2 = or, = 12. (1) As we know, Cp – Cv = R where Cp and Cv are molar
3 R 2 R 2
specific heat capacities
hc
8. (3) In X-ray tube, l min = R
eV or, Cp – Cv =
M
æ hc ö
In l min = In ç ÷ - InV R
è eø For hydrogen (M = 2) Cp – Cv = a =
2
Clearly, log lmin versus log V graph
R
slope is negative hence option (3) correctly depicts. For nitrogen (M = 28) Cp – Cv = b =
28
9. (4) Let initially there are total N0 number of nuclei
a
NB \ = 14 or, a = 14b
At time t N = 0.3(given) b
A 13. (4) According to principle of calorimetry,
Þ NB = 0.3NA Heat lost = Heat gain
N0 = NA + NB = NA + 0.3NA 100 × 0.1( – 75) = 100 × 0.1 × 45 + 170 × 1 × 45
10 – 750 = 450 + 7650
N0
\ NA = 10 = 1200 + 7650 = 8850
1.3
As we know Nt = N0 e– lt T = 885°C
14. (1) Let Vf is the final speed of the body.
N0
or, = N0 e– lt
1.3 From questions,
1 1 1 V0
= e–lt Þ ln(1.3) = lt mV f2 = mV02 Þ Vf = = 5 m/ s
1.3 2 8 2
ln(1.3) ln (1.3) ln(1.3) T æ dV ö 2 dV
or, t = Þ t= = ln F = mç ÷ = -kV \ (10–2) = –kV2
l ln(2) (2) è dt ø dt
T 5 10
dV
10. (1) T = PE sinq Torque experienced by the dipole in an
ur ur ur ò V2 = -100 K ò dt
electric field, T = P ´ E 10 0

ur 1 1
p = p cosq iˆ + psinq ĵ - = 100 K (10) or, K = 10–4 kgm–1
ur r 5 10
E1 = Ei 15. (3) Given : Current through the galvanometer,
ur ur ur ig = 5 × 10–3 A
T 1 = p ´ E1 = (p cos q iˆ + p sin q ĵ ) × E ( iˆ )
Galvanometer resistance, G = 15 W
t k̂ = pE sinq (– k̂ ) ...(i) Let resistance R to be put in series with the galvanometer to
ur convert it into a voltmeter.
E 2 = 3 E1 ˆj
V = ig (R + G)
ur
T 2 = p cos qiˆ + p sin qˆj ) ´ 3 E1 ˆj 10 = 5 × 10–3 (R + 15)
\ R = 2000 – 15 = 1985
tkˆ = 3 pE1 cos qkˆ ...(ii)
= 1.985 × 103 W
2017-6 JEE MAIN 2017 Solved Paper
16. (3) Torque at angle q 20. (4) For common maxima, n1l1 = n2l2
n1 l 2 520 ´10-9 4
w,a
Þ = = =
n2 l1 650 ´ 10-9 5
l For l1
t = Mg sin q. Q
Q
2 n1l1 D
y= , l1 = 650 nm
d
Also t = la
4 ´ 650 ´ 10-9 ´ 1.5
y= or, y = 7.8 mn
l a = Mg sin q
l 0.5 ´ 10-3
\
2 21. (3) Given : Magnetic moment, M = 6.7 × 10–2 Am2
Magnetic field, B = 0.01 T
Ml 2 l é Ml 2 ù
.a = Mg sin q êQ I rod = ú Moment of inertia, I = 7.5 × 10–6 Kgm2
3 2 ë 3 û
I
Using, T = 2p
la sin q 3 g sin q MB
Þ =g \ a=
3 2 2l
hc 7.5 ´ 10-6
2p
17. (2) From energy level diagram, using DE = = 2p = ´ 1.06 s
l 6.7 ´ 10-2 ´ 0.01 10
Time taken for 10 complete oscillations
hc
For wavelength l1 DE = – E – (–2E) = t = 10T = 2p × 1.06
l1 = 6.6568 » 6.65 s
hc 22. (2) Variation of acceleration due to gravity, g with distance
\ l1 = 'd ' from centre of the earth
E
æ 4E ö hc Gm
For wavelength l2 DE = – E – ç - ÷ = If d < R, g = .d i.e., g µ d (straight line)
è 3 ø l2 R2
Gm
hc l1 1 If d = R, gs =
\ l2 = \ r= = R2
æ Eö l2 3
çè ÷ø Gm 1
3 If d > R, g = 2 i.e., g µ
d d2
18. (3) As linear dimension increases by a factor of 9 23. (2) The potential difference in each loop is zero.
vf \ No current will flow or current in each resistance is
\ = 93 Zero.
vi
Q Density remains same m
24. (4) From question, mA = M; mB =
So, mass µ Volume 2
mf ( Area ) f uA = V uB = 0
= 93 Þ = 92 Let after collision velocity of A = V1 and
mi ( Area ) i velocity of B = V2
force (mass ) ´ g Applying law of conservation of momentum,
Stress (s) = =
area area
æ mö
mu = mv1 + ç ÷ v2
s2 æ m f ö æ Ai ö 93 è2ø
= ÷ø ç A ÷ = 92 = 9
s1 çè mi è fø
or, 24= 2v1 + v2 ....(i)
By law of collision
19. (1) According to Faraday's law of electromagnetic
v2 - v1
df e=
induction, e = u-0
dt
Also, e = iR or, u = v2 – v1 ....(ii)
[Q collision is elastic, e = 1]
df
\ iR = Þ ò d f = R ò idt using eqns (i) and (ii)
dt
Magnitude of change in flux (df) = R × area under current vs 4 4
v1 = and v 2 = u
time graph 3 3
h
1 1 de-Broglie wavelength l =
or, df = 100 ´ ´ ´ 10 = 250 Wb p
2 2
EBD_7036
PHYSICS 2017-7

28. (1) In steady state, flow fo current through capacitor will


m 4
´ u be zero.
l A PB 2 3 =2 Current through the circuit,
\ = =
l B PA 4

3
25. (3) As we know, Bulk modulus E
i=
r + r2
DP DV P
K= Þ =
æ -DV ö V K
çè ÷ø Potential difference through capacitor
V a
V = V0 (1 + gDt) Q æ E ö
Vc = = E - ir = E - ç r
DV C è r + r2 ÷ø
= gDt
V0
r2
\ Q = CE
P P P r + r2
\ = gDt Þ Dt = =
K gK 3aK 29. (2) To get a capacitance of 2 mF arrangement of capacitors
of capacitance 1mF as shown in figure 8 capacitors of 1mF in
dV
26. (3) Using, F = ma = m parallel with four such branches in series i.e., 32 such
dt capacitors are required.
dV
6t = 1. [Q m = 1 kg given]
dt
v

ò dV = ò 6t dt
0

ét 2 ù
1 m m m m
V = 6 ê ú = 3 ms –1 [Q t = 1 sec given]
ë 2 û0
From work-energy theorem,

W = DKE =
1
2
( 1
2
)
m V 2 - u 2 = ´ 1 ´ 9 = 4.5 J 1 1 1 1 1
= + + +
Ceq 8 8 8 8 \ Ceq = 2 mF
27. (1) Use relativistic doppler's effect as velocity of observer
is not small as compared to light
30. (1) For a body thrown vertically upwards acceleration
c+v remains constant (a = – g) and velocity at anytime t is given
f = f0 ; V = relative speed of approach
c+v by V = u – gt
f0 = 10 GHz During rise velocity decreases linearly and during fall
velocity increases linearly and direction is opposite to each
c other.
c+ Hence graph (1) correctly depicts velocity versus time.
\ f = 10 2 = 10 3 = 17.3 GHz
c
c-
2
CHAPTER

1 Units and Measurement s

Section-A JEE Advanced/ IIT-JEE


A Fill in the Blanks 6. Which of the following set have different dimensions?
(a) Pressure, Young’s modulus , Stress (2005S)
1. Planck’s constant has dimension ________.
(1985 - 2 Marks) (b) EMF, Potential difference, Electric potential
2. In the formula X = 3YZ 2, X and Z have dimensions of (c) Heat, Work done, Energy
capacitance and magnetic induction respectively. The (d) Dipole moment, Electric flux, Electric field
dimensions of Y in MKSQ sytem are _________, ________. 7. In a screw gauge, the zero of mainscale coincides with fifth
(1988 - 2 Marks) division of circular scale in figure (i). The circular division of
3. The equation of state for real gas is given by screw gauge are 50. It moves 0.5 mm on main scale in one
æ a ö rotation. The diameter of the ball in figure (ii) is
çè P + 2 ÷ø (V - b) = RT . The dimensions of the constant (2006 - 3M, –1)
V
a is _________. (1997 - 2 Marks) 0 10
5
0
C MCQs with One Correct Answer

The dimension of çæ ÷ö e 0 E 2 ( e 0 : permittivity of free space,


1
1.
è 2ø Figure (i)
E electric field) (2000S) 0 30
(a) MLT –1 (b) ML T 2 –2 25
20
(c) ML–1T–2 (d) ML2T–1
DV
2. A quantity X is given by e 0 L where Î0 is the
Dt
permittivity of the free space, L is a length, DV is a potential Figure (ii)
difference and Dt is a time interval. The dimensional formula (a) 2.25 mm (b) 2.20 mm
for X is the same as that of (2001S)
(c) 1.20 mm (d) 1.25 mm
(a) resistance (b) charge
8. A student performs an experiment for determination of
(c) voltage (d) current
3. A cube has a side of length 1.2 × 10–2m. Calculate its volume. æ 4p 2 l ö
gç= ÷ . The error in length l is Dl and in time T is DT
(a) 1.7 × 10–6 m3 (b) 1.73 × 10–6 m3 (2003S) è T2 ø
(c) 1.70 × 10 m–6 3 (d) 1.732 × 10–6 m3
and n is number of times the reading is taken. The
a az
4. Pressure depends on distance as, P = exp æç - ö÷ , where measurement of g is most accurate for (2006 - 3M, –1)
b è kq ø Dl DT n
a, b are constants, z is distance, k is Boltzman’s constant (a) 5 mm 0.2 sec 10
and q is temperature. The dimension of b are (2004S) (b) 5mm 0.2 sec 20
(a) M0L0T0 (b) M–1L–1T –1
(c) 5 mm 0.1 sec. 10
(c) M0L2T0 (d) M–1L1T 2
(d) 1mm 0.1 sec 50
5. A wire of length l = 6 ± 0.06 cm and radius r = 0.5 ± 0.005 cm
9. A student performs an experiment to determine the Young's
and mass m = 0.3 ± 0.003 gm. Maximum percentage error in
modulus of a wire, exactly 2 m long, by Searle's method. In a
density is (2004S)
particular reading, the student measures the extension in
(a) 4 (b) 2
the length of the wire to be 0.8 mm with an uncertainty
(c) 1 (d) 6.8 of ± 0.05 mm at a load of exactly 1.0 kg. The student also
EBD_7036
P-2 Topic-wise Solved Papers - PHYSICS
measures the diameter of the wire to be 0.4 mm with an (c) due to the error in the measurement of l is twice that
uncertainty of ± 0.01 mm. Take g = 9.8 m/s2 (exact). The due to the error in the measurement of d.
Young's modulus obtained from the reading is (2007) (d) due to the error in the measurement of d is four times
(a) (2.0 ± 0.3) × 10 N/m
11 2 (b) (2.0 ± 0.2) × 10 N/m2
11 that due to the error in the measurement of l.
(c) (2.0 ± 0.1) × 10 N/m
11 2 (d) (2.0 ± 0.05) × 1011 N/m2 14. The diameter of a cylinder is measured using a Vernier
10. Students I, II and III perform an experiment for measuring callipers with no zero error. It is found that the zero of the
the acceleration due to gravity (g) using a simple pendulum. Vernier scale lies between 5.10 cm and 5.15 cm of the main
They use different lengths of the pendulum and /or record scale. The Vernier scale has 50 divisions equivalent to 2.45
time for different number of oscillations. The observations cm. The 24th division of the Vernier scale exactly coincides
are shown in the table. (2008) with one of the main scale divisions. The diameter of the
Least count for length = 0.1 cm cylinder is (JEE Adv. 2013)
Least count for time = 0.1 s (a) 5.112 cm (b) 5.124 cm
(c) 5.136 cm (d) 5.148 cm
Student Length of the No. of Total time Time
15. There are two Vernier calipers both of which have 1 cm di-
pendulum oscillations for (n) period
vided into 10 equal divisions on the main scale. The Vernier
(cm) (n) oscillations (s) scale of one of the calipers (C1) has 10 equal divisions that
(s) correspond to 9 main scale divisions. The Vernier scale of
I 64.0 8 128.0 16.0 the other caliper (C2) has 10 equal divisions that correspond
II 64.0 4 64.0 16.0 to 11 main scale divisions. The readings of the two calipers
III 20.0 4 36.0 9.0 are shown in the figure. The measured values (in cm) by
If EI, EII and EIII are the percentage errors in g, i.e., calipers C1 and C2, respectively, are (JEE Adv. 2016)
2 3 4
æ Dg ö
çè g ´ 100÷ø for students I, II and III, respectively, then C1

(a) EI = 0 (b) EI is minimum 0 5 10


(c) EI = EII (d) EII is maximum
2 3 4
11. A vernier calipers has 1 mm marks on the main scale. It has 20
equal divisions on the Vernier scale which match with 16 main C2
scale divisions. For this Vernier calipers, the least count is
(a) 0.02 mm (b) 0.05 mm (2010) 0 5 10
(a) 2.85 and 2.82 (b) 2.87 and 2.83
(c) 0.1 mm (d) 0.2 mm
(c) 2.87 and 2.86 (d) 2.87 and 2.87
12. The density of a solid ball is to be determined in an
experiment. The diameter of the ball is measured with a screw
gauge, whose pitch is 0.5 mm and there are 50 divisions on
D MCQs with One or More than One Correct
the circular scale. The reading on the main scale is 1. The dimensions of the quantities in one (or more) of the
2.5 mm and that on the circular scale is 20 divisions. If the following pairs are the same. Identify the pair (s)
measured mass of the ball has a relative error of 2 %, the
(a) Torque and Work (1986 - 2 Marks)
relative percentage error in the density is (2011)
(b) Angular momentum and Work
(a) 0.9 % (b) 2.4 %
(c) Energy and Young’s modulus
(c) 3.1 % (d) 4.2 %
(d) Light year and Wavelength
æ 4 MLg ö 2. The pairs of physical quantities that have the same
13. In the determination of Young’s modulus ç Y = ÷ by
è pld 2 ø dimensions is (are) : (1995S)
using Searle’s method, a wire of length L = 2 m and diameter (a) Reynolds number and coefficient of friction
d = 0.5 mm is used. For a load M = 2.5 kg, an extension l = (b) Curie and frequency of a light wave
0.25 mm in the length of the wire is observed. Quantities d (c) Latent heat and gravitational potential
and l are measured using a screw gauge and a micrometer, (d) Planck’s constant and torque
respectively. They have the same pitch of 0.5 mm. The 3. The SI unit of inductance, the henry can be written as
number of divisions on their circular scale is 100. The (1998 - 2 Marks)
contributions to the maximum probable error of the Y (a) weber/ampere (b) volt-sec/amp
measurement (2012) (c) Joule/(ampere)2 (d) ohm-second
(a) due to the errors in the measurements of d 4. Let [e 0 ] denote the dimensional formula of the permittivity
and l are the same.
of the vacuum, and [m 0 ] that of the permeability of the
(b) due to the error in the measurement of d is twice that vacuum. If M = mass, L = length, T = time and I = electric
due to the error in the measurement of l . current, (1998 - 2 Marks)
Units and Measurements P-3

charged particles, and the charge (q) carried by each of the


(a) [e 0 ] = M -1 L-3 T 2 Ι (b) [e 0 ] = M -1 L-3 T 4 Ι 2
particles. Which of the following expression(s) for I is(are)
dimensionally correct? (JEE Adv. 2016)
(c) [m 0 ] = M LT -2 Ι -2 (d) [m 0 ] = M L2T -1 Ι
5. A student uses a simple pendulum of exactly 1m length to æ nq2 ö æ ek T ö
determine g, the acceleration due to gravity. He uses a stop (a) l= ç ÷ (b) l = ç B2 ÷
ç ek BT ÷ ç nq ÷
watch with the least count of 1 sec for this and records 40 è ø è ø
seconds for 20 oscillations. For this observation, which of
the following statement(s) is (are) true? (2010) æ ö æ ö
q2 q2
(a) Error DT in measuring T, the time period, is 0.05 seconds (c) l= ç ÷ (d) l= ç ÷
ç en k T ÷
2/3 ç en k T ÷
1/3
(b) Error DT in measuring T, the time period, is 1 second è B ø è B ø
(c) Percentage error in the determination of g is 5% 11. In an experiment to determine the acceleration due to gravity
(d) Percentage error in the determination of g is 2.5% g, the formula used for the time period of a periodic motion is
6. Using the expression 2d sin q = l, one calculates the values
of d by measuring the corresponding angles q in the range 7 (R - r)
T = 2p . The values of R and r are measured to be
0 to 90°. The wavelength l is exactly known and the error in 5g
q is constant for all values of q. As q increases from 0° (60 ± 1) mm and (10 ± 1) mm, respectively. In five successive
(JEE Adv. 2013) measurements, the time period is found to be 0.52s, 0.56s,
(a) The absolute error in d remains constant 0.57s, 0.54s and 0.59s. The least count of the watch used for
(b) The absolute error in d increases the measurement of time period is 0.01s. Which of the fol-
(c) The fractional error in d remains constant lowing statement(s) is (are) true? (JEE Adv. 2016)
(d) The fractional error in d decreases (a) The error in the measurement of r is 10%
7. Planck’s constant h, speed of light c and gravitational (b) The error in the measurement of T is 3.75%
constant G are used to form a unit of length L and a unit of (c) The error in the measurement of T is 2%
mass M. Then the correct option(s) is(are) (JEE Adv. 2015) (d) The error in the determined value of g is 11%
(a) Mµ c (b) M µ G
(c) L µ h (d) L µ G
E Subjective Problems
8. Consider a Vernier callipers in which each 1 cm on the main 1. Give the MKS units for each of the following quantities.
scale is divided into 8 equal divisions and a screw gauge (i) Young’s modulus (1980)
with 100 divisions on its circular scale. In the Vernier callipers,
(ii) Magnetic Induction
5 divisions of the Vernier scale coincide with 4 divisions on
the main scale and in the screw gauge, one complete rotation (iii) Power of a lens
of the circular scale moves it by two divisions on the linear 2. A gas bubble, from an explosion under water, oscillates with
scale. Then : (JEE Adv. 2015) a period T proportional to p a d b E c . Where ‘P’ is the static
(a) If the pitch of the screw gauge is twice the least count pressure, ‘d’ is the density of water and ‘E’ is the total energy
of the Vernier callipers, the least count of the screw of the explosion. Find the values of a, b and c.
gauge is 0.01 mm (1981- 3 Marks)
(b) If the pitch of the screw gauge is twice the least count 3. Write the dimensions of the following in terms of mass,
of the Vernier callipers, the least count of the screw time, length and charge (1982 - 2 Marks)
gauge is 0.005 mm (i) magnetic flux
(c) If the least count of the linear scale of the screw gauge (ii) rigidity modulus
is twice the least count of the Vernier callipers, the
4. Match the physical quantities given in column I with
least count of the screw gauge is 0.01 mm
dimensions expressed in terms of mass (M), length (L), time
(d) If the least count of the linear scale of the screw gauge (T), and charge (Q) given in column II and write the correct
is twice the least count of the Vernier callipers, the
answer against the matched quantity in a tabular form in
least count of the screw gauge is 0.005 mm
your answer book. (1983 - 6 Marks)
9. In terms of potential difference V, electric current I, permittivity
Column I Column II
e0, permeability m0 and speed of light c, the dimensionally
correct equation(s) is(are) (JEE Adv. 2015) Angular momentum ML2T–2
(a) m0I2 = e0V2 (b) m0I = m0V Latent heat ML2Q–2
(c) I = e0cV (d) m0cI = e0V Torque ML2T–1
10. A length–scale (l) depends on the permittivity (e) of a Capacitance ML3T–1Q–2
dielectric material. Boltzmann constant (kB), the absolute Inductance M –1 L –2 T 2 Q2
temperature (T), the number per unit volume (n) of certain Resistivity L2T –2
EBD_7036
P-4 Topic-wise Solved Papers - PHYSICS
5. Column-I gives three physical quantities. Select the surface area of wire in cm2 to appropriate significant figure.
appropriate units for the choices given in Column-II. Some 22
(use p = ). (2004 - 2 Marks)
of the physical quantities may have more than one choice 7
correct : (1990 - 3 Marks) 8. In Searle’s experiment, which is used to find Young’s
Column I Column II Modulus of elasticity, the diameter of experimental wire is D
Capacitance (i) ohm-second = 0.05 cm (measured by a scale of least count 0.001 cm) and
Inductance (ii) coulomb2–joule–1 length is L = 110 cm (measured by a scale of least count 0.1
Magnetic Induction (iii) coulomb (volt)–1 cm). A weight of 50 N causes an extension of X = 0.125 cm
(iv) newton (amp-metre)–1 (measured by a micrometer of least count 0.001cm). Find
maximum possible error in the values of Young’s modulus.
(v) volt-second (ampere)–1
Screw gauge and meter scale are free from error.
6. If nth division of main scale coincides with (n+1)th divisions (2004 - 2 Marks)
of vernier scale. Given one main scale division is equal to ‘a’ 9. The side of a cube is measured by vernier callipers (10
units. Find the least count of the vernier. (2003 - 2 Marks) divisions of a vernier scale coincide with 9 divisions of main
7. A screw gauge having 100 equal divisions and a pitch of scale, where 1 division of main scale is 1 mm). The main
length 1 mm is used to measure the diameter of a wire of scale reads 10 mm and first division of vernier scale coincides
length 5.6 cm. The main scale reading is 1 mm and 47th circular with the main scale. Mass of the cube is 2.736 g. Find the
division coincides with the main scale. Find the curved density of the cube in appropriate significant figures.
(2005 - 2 Marks)

F Match the Following

DIRECTIONS (Q. No. 1) : Each question contains statements given in two columns, which have to be p q r s t
matched. The statements in Column-I are labelled A, B, C and D, while the statements in Column-II are
A p q r s t
labelled p, q, r and s. Any given statement in Column-I can have correct matching with ONE OR MORE
B p q r s t
statement(s) in Column-II. The appropriate bubbles corresponding to the answers to these questions
have to be darkened as illustrated in the following example : C p q r s t
If the correct matches are A-p, s and t; B-q and r; C-p and q; and D-s then the correct darkening of D p q r s t
bubbles will look like the given.
1. Some physical quantities are given in Column I and some possible SI units in which these quantities may be expressed are given
in Column II. Match the physical quantities in Column I with the units in Column II and indicate your answer by darkening
appropriate bubbles in the 4 × 4 matrix given in the ORS. (2007)
Column I Column II
(A) GMeMs , G – universal gravitational constant, (p) (volt) (coulomb)(metre)
Me – mass of the earth, Ms – mass of the Sun
3RT
(B) , R – universal gas constant, (q) (kilogram) (metre)3(second)–2
M
T – absolute temperature, M – molar mass
F2
(C) , F – Force, q – charge, B – magnetic field (r) (metre)2 (second)–2
q2 B 2
GM e
(D) , G – universal gravitational constant, (s) (farad) (volt)2 (kg)–1
Re
Me – mass of the earth, Re – radius of the earth

DIRECTIONS (Q. No. 2) : Following question has matching lists. The codes for the lists have choices (a), (b), (c) and (d) out of
which ONLY ONE is correct.
2. Match List I with List II and select the correct answer using the codes given below the lists: (JEE Adv. 2013)
List I List II Codes:
P. Boltzmann constant 1. [ML2T-1] P Q R S
Q. Coefficient of viscosity 2. [ML T ]–1 –1 (a) 3 1 2 4
R. Planck constant 3. [MLT–3K–1] (b) 3 2 1 4
S. Thermal conductivity 4. [ML2T–2K–1] (c) 4 2 1 3
(d) 4 1 2 3
Units and Measurements P-5

where these quantities are in proper SI units.


G Comprehension Based Questions
(a) 800 nm (b) 600 nm
PASSAGE (c) 300 nm (d) 200 nm
A dense collection of equal number of electrons and positive ions
is called neutral plasma. Certain solids containing fixed positive I Integer Value Correct Type
ions surrounded by free electrons can be treated as neutral plasma.
1. To find the distance d over which a signal can be seen clearly
Let ‘N’ be the number density of free electrons, each of mass ‘m’. in foggy conditions, a railways-engineer uses dimensions
When the electrons are subjected to an electric field, they are and assumes that the distance depends on the mass density
displaced relatively away from the heavy positive ions. If the r of the fog, intensity (power/area) S of the light from the
electric field becomes zero, the electrons begin to oscillate about signal and its frequency f. The engineer finds that d is
proportional to S1/n. The value of n is (JEE Adv. 2014)
the positive ions with a natural angular frequency ‘wp’ which is
2. During Searle’s experiment, zero of the Vernier scale lies
called the plasma frequency. To sustain the oscillations, a time between 3.20 × 10–2 m and 3.25 × 10–2 m of the main scale.
varying electric field needs to be applied that has an angular The 20th division of the Vernier scale exactly coincides with
frequency w, where a part of the energy is absorbed and a part of one of the main scale divisions. When an additional load of
it is reflected. As w approaches wp all the free electrons are set to 2 kg is applied to the wire, the zero of the Vernier scale still
resonance together and all the energy is reflected. This is the lies between 3.20 × 10–2 m and 3.25 × 10–2 m of the main scale
but now the 45th division of Vernier scale coincides with one
explanation of high reflectivity of metals. (2011)
of the main scale divisions. The length of the thin metallic
1. Taking the electronic charge as ‘e’ and the permittivity as wire is 2 m and its cross-sectional area is 8 × 10–7 m2. The
‘e0’. Use dimensional analysis to determine the correct least count of the Vernier scale is 1.0 × 10–5 m. The maximum
expression for wp. percentage error in the Young’s modulus of the wire is
(JEE Adv. 2014)
Ne me0 Ne 2 Ne 2 3. The energy of a system as a function of time t is given as E(t)
(a) (b) (c) (d)
me 0 Ne me0 me0 = A2 exp(–at,) where a = 0.2 s–1. The measurement of A has
2. Estimate the wavelength at which plasma reflection will occur an error of 1.25%. If the error in the measurement of time is
1.50%, the percentage error in the value of E(t) at t = 5 s is
for a metal having the density of electrons
(JEE Adv. 2015)
N » 4 ´ 1027 m–3. Taking e0 = 10–11 and mass m » 10–30,

Section-B JEE Main / AIEEE


1. Identify the pair whose dimensions are equal [2002] 6. The dimension of magnetic field in M, L, T and C (coulomb)
(a) torque and work (b) stress and energy is given as [2008]
(c) force and stress (d) force and work (a) MLT–1 C–1 (b) MT2 C–2
(c) MT–1 C–1 (d) MT–2 C–1
2. Dimension of 1 , where symbols have their usual 7. A body of mass m = 3.513 kg is moving along the x-axis with
m0 e0 a speed of 5.00 ms–1. The magnitude of its momentum is
meaning, are [2003] recorded as [2008]
(a) 17.6 kg ms–1 (b) 17.565 kg ms–1
(a) [L-1T] (b) [L-2 T 2 ] (c) [L2 T -2 ] (d) [LT -1 ]
(c) 17.56 kg ms–1 (d) 17.57 kg ms–1
3. The physical quantities not having same dimensions are 8. Two full turns of the circular scale of a screw gauge cover a
(a) torque and work [2003] distance of 1mm on its main scale. The total number of
(b) momentum and planck’s constant divisions on the circular scale is 50. Further, it is found that
(c) stress and young’s modulus the screw gauge has a zero error of – 0.03 mm. While
measuring the diameter of a thin wire, a student notes the
(d) speed and (m o e o ) -
1/ 2
main scale reading of 3 mm and the number of circular scale
4. Which one of the following represents the correct divisions in line with the main scale as 35. The diameter of
dimensions of the coefficient of viscosity? [2004] the wire is [2008]
(a) 3.32 mm (b) 3.73 mm
(a) ML-1T -1 (b) MLT -1 (c) ML-1T -2 (d) ML-2T -2 (c) 3.67 mm (d) 3.38 mm
5. Out of the following pair , which one does NOT have identical 9. In an experiment the angles are required to be measured
dimensions is [2005] using an instrument, 29 divisions of the main scale exactly
(a) impulse and momentum coincide with the 30 divisions of the vernier scale. If the
(b) angular momentum and planck’s constant smallest division of the main scale is half- a degree (= 0.5°),
(c) work and torque then the least count of the instrument is : [2009]
(d) moment of inertia and moment of a force (a) half minute (b) one degree
(towards north-west) (c) half degree (d) one minute
EBD_7036
P-6 Topic-wise Solved Papers - PHYSICS
10. The respective number of significant figures for the numbers 15. A student measured the length of a rod and wrote it as 3.50
23.023, 0.0003 and 2.1 × 10–3 are [2010] cm. Which instrument did he use to measure it?
(a) 5, 1, 2 (b) 5, 1, 5 [JEE Main 2014]
(c) 5, 5, 2 (d) 4, 4, 2 (a) A meter scale.
11. A screw gauge gives the following reading when used to (b) A vernier calliper where the 10 divisions in vernier scale
measure the diameter of a wire. matches with 9 division in main scale and main scale has
Main scale reading : 0 mm 10 divisions in 1 cm.
Circular scale reading : 52 divisions (c) A screw gauge having 100 divisions in the circular scale
Given that 1mm on main scale corresponds to 100 divisions
and pitch as 1 mm.
of the circular scale. The diameter of wire from the above
(d) A screw gauge having 50 divisions in the circular scale
data is: [2011]
and pitch as 1 mm.
(a) 0.052 cm (b) 0.026 cm
(c) 0.005 cm (d) 0.52 cm L
16. The period of oscillation of a simple pendulum is T = 2p .
12. Resistance of a given wire is obtained by measuring the g
current flowing in it and the voltage difference applied across Measured value of L is 20.0 cm known to 1 mm accuracy and
it. If the percentage errors in the measurement of the current time for 100 oscillations of the pendulum is found to be 90 s
and the voltage difference are 3% each, then error in the using a wrist watch of 1s resolution. The accuracy in the
value of resistance of the wire is : [2012] determination of g is : [JEE Main 2015]
(a) 6% (b) zero (a) 1% (b) 5%
(c) 1% (d) 3% (c) 2% (d) 3%
13. A spectrometer gives the following reading when used to 17. A student measures the time period of 100 oscillations of a
measure the angle of a prism. [2012] simple pendulum four times. The data set is 90 s, 91 s, 95 s,
Main scale reading : 58.5 degree and 92 s. If the minimum division in the measuring clock is 1
Vernier scale reading : 09 divisions s, then the reported mean time should be: [JEE Main 2016]
Given that 1 division on main scale corresponds to 0.5 degree. (a) 92 ± 1.8 s (b) 92 ± 3s
Total divisions on the Vernier scale is 30 and match with 29 (c) 92 ± 2 s (d) 92 ± 5.0 s
divisions of the main scale. The angle of the prism from the 18. A screw gauge with a pitch of 0.5 mm and a circular scale
above data : with 50 divisions is used to measure the thickness of a thin
(a) 58.59 degree (b) 58.77 degree sheet of Aluminium. Before starting the measurement, it is
(c) 58.65 degree (d) 59 degree found that wen the two jaws of the screw gauge are brought
in contact, the 45th division coincides with the main scale
14. Let [ Î0 ] denote the dimensional formula of the permittivity line and the zero of the main scale is barely visible. What is
of vacuum. If M = mass, L = length, T = time and A = electric the thickness of the sheet if the main scale reading is 0.5 mm
current, then: [JEE Main 2013] and the 25th division coincides with the main scale line?
(a) Î0 = [M–1 L–3 T2 A] (b) Î0 = [M1 L3 T5 A2] [JEE Main 2016]
(a) 0.70 mm (b) 0.50 mm
(c) Î0 = [M1 L2 T1 A2] (d) Î0 = [M1 L2 T1 A] (c) 0.75 mm (d) 0.80 mm
CHAPTER

2 Motion
Section-A JEE Advanced/ IIT-JEE

A Fill in the Blanks (a) due north (b) 30° east of north
(c) 30° west of north (d) 60° east of north
1. A particle moves in a circle of radius R. In half the period of 2. A boat which has a speed of 5 km/hr in still water crosses a
revolution its displacement is ____________and distance river of width 1 km along the shortest possible path in 15
covered is ________. (1983 - 2 Marks) minutes. The velocity of the river water in km/hr is
2. Four persons K, L, M, N are initially at the four corners of a (a) 1 (b) 3 (1988 - 1 Mark)
square of side d. Each person now moves with a uniform (c) 4 (d)
speed v in such a way that K always moves directly towards 41
L, L directly towards M, M directly towards N, and N directly 3. In 1.0 s, a particle goes from point A to point B, moving in a
towards K. The four persons will meet at a time .............. semicircle of radius 1.0 m (see Figure). The magnitude of the
(1984- 2 Marks) average velocity A (1999S - 2 Marks)
3. Spotlight S rotates in a horizontal plane with constant angular (a) 3.14 m/s
velocity of 0.1 radian/second. The spot of light P moves
along the wall at a distance of 3 m. The velocity of the spot
(b) 2.0 m/s 1.0m
P when q = 45° (see fig.) is ................. m/s
(1987 - 2 Marks)
(c) 1.0 m/s

(d) Zero
B
4. A ball is dropped vertically from a height d above the ground.
It hits the ground and bounces up vertically to a height d/2.
Neglecting subsequent motion and air resistance, its velocity
v varies with the height h above the ground as (2000S )

B True/False
1. Two balls of different masses are thrown vertically upwards (a) (b)
with the same speed. They pass through the point of
projection in their downward motion with the same speed
(Neglect air resistance). (1983 - 2 Marks)
2. A projectile fired from the ground follows a parabolic path.
The speed of the projectile is minimum at the top of its path. (c) (d)
(1984 - 2 Marks)
3. Two identical trains are moving on rails along the equator
on the earth in opposite directions with the same speed. 5. A particle starts sliding down a frictionless inclined plane.
They will exert the same pressure on the rails. If Sn is the distance travelled by it from time t = n – 1 sec to
(1985 - 3 Marks) t = n sec, the ratio Sn/Sn+1 is (2004S)
2n - 1 2n + 1
(a) (b)
C MCQs with One Correct Answer 2n + 1 2n
2n
1. A river is flowing from west to east at a speed of 5 metres (c) (d) 2n + 1
per minute. A man on the south bank of the river, capable of 2n + 1 2n - 1
swimming at 10 metres per minute in still water, wants to 6. A body starts from rest at time t = 0, the acceleration time
swim across the river in the shortest time. He should swim graph is shown in the figure. The maximum velocity attained
in a direction (1983 - 1 Mark) by the body will be (2004S)
EBD_7036
P-8 Topic-wise Solved Papers - PHYSICS

(a) 110 m/s range is less than half the disc radius, and (iii) w remains
constant throughout. Then (2012)
(b) 55 m/s

(c) 650 m/s

(d) 550 m/s


7. The velocity-displacement graph of a particle moving (a) P lands in the shaded region and Q in the unshaded
along a straight line is shown (2005S) region.
(b) P lands in the unshaded region and Q in the shaded
region.
(c) Both P and Q land in the unshaded region.
(d) Both P and Q land in the shaded region.

D MCQs with One or More than One Correct


The most suitable acceleration-displacement graph will be 1. A particle is moving eastwards with a velocity of 5 m/s. In
10s the velocity changes to 5 m/s northwards. The average
acceleration in this time is (1982 - 3 Marks)
(a) zero
(a) (b) (b) 1/Ö2 m/s2 towards north-west
(c) 1 / 2 m/s2 towards north-east
1
(d) m / s 2 towards north-west
2
1
(e) m / s 2 towards north
2
(c) (d) 2. A particle of mass m moves on the x-axis as follows : it starts
from rest at t = 0 from the point x = 0, and comes to rest at
t = 1 at the point x = 1. NO other information is available about
8. Two identical discs of same radius R are rotating about their its motion at intermediate times (0 < t< 1). If a denotes the
axes in opposite directions with the same constant angular instantaneous acceleration of the particle, then:
speed w. The discs are in the same horizontal plane. At time (1993-2 Marks)
t = 0, the points P and Q are facing each other as shown in (a) a cannot remain positive for all t in the interval
the figure. The relative speed between the two points P and 0 £ t £ 1.
Q is vr. In one time period (T) of rotation of the discs, vr as a (b) |a| cannot exceed 2 at any point in its path.
function of time is best represented by (2012) (c) |a| must be ³ 4 at some point or points in its path.
(d) a must change sign during the motion, but no other
assertion can be made with the information given.
3. The coordinates of a particle moving in a plane are given by
x(t) = a cos (pt) and y (t) = b sin (pt) where a, b (< a) and p are
positive constants of appropriate dimensions. Then
(1999S - 3 Marks)
(a) the path of the particle is an ellipse
(a) (b) (b) the velocity and acceleration of the particle are normal
to each other at t = p/(2p)
(c) the acceleration of the particle is always directed
towards a focus
(d) the distance travelled by the particle in time interval
(c) (d)
t = 0 to t = p/(2p) is a
9. Consider a disc rotating in the horizontal plane with a E Subjective Problems
constant angular speed w about its centre O. The disc has a
shaded region on one side of the diameter and an unshaded 1. A car accelerates from rest at a constant rate a for some time
region on the other side as shown in the figure. When the after which it decelerates at a constant rate b to come to
disc is in the orientation as shown, two pebbles P and Q are rest. If the total time lapse is t seconds, evaluate. (1978)
simultaneously projected at an angle towards R. The velocity (i) maximum velocity reached, and
of projection is in the y-z plane and is same for both pebbles (ii) the total distance travelled.
with respect to the disc. Assume that (i) they land back on 2. The displacement x of particle moving in one dimension,
under the action of a constant force is related to the time t
the disc before the disc has completed 1/8 rotation, (ii) their
Motion P-9

by the equation t = x + 3 (1979) of projection makes an angle a with the bottom as shown in
where x is in meters and t in seconds. Find Figure. (1998 - 8 Marks)
(i) The displacement of the particle when its velocity is
zero, and
(ii) The work done by the force in the first 6 seconds.
3. Answer the following giving
a Q
reasons in brief : P
Is the time variation of position,
q
shown in the figure observed
(a) Find the distance along the bottom of the box between
in nature? (1979) the point of projection P and the point Q where the
4. Particles P and Q of mass 20 gm and 40 gm respectively are particle lands. (Assume that the particle does not hit
simultaneously projected from points A and B on the ground. any other surface of the box. Neglect air resistance.)
The initial velocities of P and Q make 45° and 135° angles (b) If the horizontal displacement of the particle as seen
respectively with the horizontal AB as shown in the figure. by an observer on the ground is zero, find the speed of
Each particle has an initial speed of 49 m/s. The separation the box with respect to the ground at the instant when
AB is 245 m. (1982 - 8 Marks) particle was projected.
8. An object A is kept fixed at the point x = 3 m and y = 1.25 m
on a plank P raised above the ground. At time t = 0 the plank
starts moving along the + x direction with an acceleration
1.5 m/s2. At the same instant a stone is projected from the
r
origin with a velocity u as shown. A stationary person on
the ground observes the stone hitting the object during its
Both particle travel in the same vertical plane and undergo a downward motion at an angle of 45° to the horizontal. All
collision After the collision, P retraces its path, Determine r
the motions are in the X–Y plane. Find u and the time after
the position of Q when it hits the ground. How much time which the stone hits the object. Take g = 10 m/s
after the collision does the particle Q take to reach the (2000 - 10 Marks)
ground? Take g = 9.8 m/s2.
5. Two towers AB and CD are situated a distance d apart as
shown in figure.
AB is 20 m high and CD is 30 m high from the ground. An
object of mass m is thrown from the top of AB horizontally
with a velocity of 10 m/s towards CD. (1994 - 6 Marks)
Simultaneously another object
of mass 2 m is thrown from the
top of CD at an angle of 60° to
the horizontal towards AB with
the same magnitude of initial
velocity as that of the first
object. The two objects move 9. On a frictionless horizontal surface, assumed to be the x-y
in the same vertical plane, plane, a small trolley A is moving along a straight line parallel
collide in mid-air and stick to
to the y-axis (see figure) with a constant velocity of
each other.
(i) Calculate the distance ‘d’ between the towers and, ( 3 - 1) m/s. At a particular instant, when the line OA makes
(ii) Find the position where the objects hit the ground. an angle of 45° with the x-axis, a ball is thrown along the
6. Two guns, situated on the top of a hill of height 10 m, fire surface from the origin O. Its velocity makes an angle f with
one shot each with the same speed 5 3 m s–1 at some the x-axis and it hits the trolley.
interval of time. One gun fires horizontally and other fires (a) The motion of the ball is y
upwards at an angle of 60° with the horizontal. The shots observed from the frame
collide in air at a point P. Find (i) the time-interval between of the trolley. Calculate A
the firings, and (ii) the coordinates of the point P. Take origin the angle q made by the
of the coordinate system at the foot of the hill right below velocity vector of the ball
the muzzle and trajectories in x-y plane. (1996 - 5 Marks) with the x-axis in this
7. A large, heavy box is sliding without friction down a smooth frame.
plane of inclination q. From a point P on the bottom of the (b) Find the speed of the ball
with r espect to the 45°
box, a particle is projected inside the box. The initial speed
surface, if f = 4q/4. O x
of the particle with respect to the box is u, and the direction (2002 - 5 Marks)
EBD_7036
P-10 Topic-wise Solved Papers - PHYSICS
2. Airplanes A and B are flying with constant velocity in the
H Assertion & Reson Type Questions same vertical plane at angles 30° and 60° with respect to the
horizontal respectively as shown in figure. The speed of A is
1. STATEMENT-1 : For an observer looking out through the
window of a fast moving train, the nearby objects appear to 100 3 m/s. At time t = 0 s, an observer in A finds B at a
move in the opposite direction to the train, while the distant distance of 500 m. The observer sees B moving with a
objects appear to be stationary. constant velocity perpendicular to the line of motion of A. If
STATEMENT-2 : If the observer and the object are moving at t = t0, A just escapes being hit by B, t0 in seconds is
r r (JEE Adv. 2014)
at velocities v 1 and v 2 respectively with reference to a A
laboratory frame, the velocity of the object with respect to
r r
the observer is v 2 – v 1 . (2008)
B
(a) Statement-1 is True, Statement-2 is True; Statement-2
is a correct explanation for Statement-1 30° 60°
(b) Statement1 is True, Statement-2 is True; Statement-2 is 3. A rocket is moving in a gravity free space with a constant
NOT a correct explanation for Statement-1 acceleration of 2 m/s2 along +x direction (see figure). The
(c) Statement -1 is True, Statement-2 is False length of a chamber inside the rocket is 4 m. A ball is thrown
(d) Statement -1 is False, Statement-2 is True from the left end of the chamber in +x direction with a speed
of 0.3 m/s relative to the rocket. At the same time, another
I Integer Value Correct Type ball is thrown in –x direction with a speed of 0.2 m/s from its
right end relative to the rocket. The time in seconds when
1. A train is moving along a straight line with a constant the two balls hit each other is (JEE Adv. 2014)
acceleration ‘a’. A boy standing in the train throws a ball
forward with a speed of 10 m/s, at an angle of 60° to the 2
0.3 m/s 0.2 m/s a = 2 m/s
horizontal. The boy has to move forward by 1.15 m inside x
the train to catch the ball back at the initial height. The
acceleration of the train, in m/s2, is (2011)
4m

Section-B JEE Main / AIEEE


1. A ball whose kinetic energy is E, is projected at an angle of 5. The co-ordinates of a moving particle at any time ‘t’are given
45° to the horizontal. The kinetic energy of the ball at the by x = a t 3 and y = b t 3 . The speed of the particle at time
highest point of its flight will be [2002] ‘t’ is given by [2003]
(a) E (b) E / 2 (c) E/2 (d) zero.
(a) 3t a 2 + b2 (b) 3t 2 a 2 + b2
2. From a building two balls A and B are thrown such that A is
thrown upwards and B downwards (both vertically with the
same speed). If vA and vB are their respective velocities on
(c) t 2 a 2 + b2 (d) a 2 + b2
reaching the ground, then [2002] 6. A ball is released from the top of a tower of height h meters.
(a) vB > vA (b) vA = vB (c) vA > vB It takes T seconds to reach the ground. What is the position
(d) their velocities depend on their masses.
T
3. A car, moving with a speed of 50 km/hr, can be stopped by of the ball at second [2004]
3
brakes after at least 6 m. If the same car is moving at a speed
of 100 km/hr, the minimum stopping distance is [2003] 8h
(a) meters from the ground
(a) 12 m (b) 18 m (c) 24 m (d) 6 m 9
4. A boy playing on the roof of a 10 m high building throws a 7h
ball with a speed of 10m/s at an angle of 30º with the (b) meters from the ground
9
horizontal. How far from the throwing point will the ball be
at the height of 10 m from the ground ? [2003] h
(c) meters from the ground
9
1 3
[ g = 10m/s2 , sin 30o = , cos 30o = ] 17 h
2 2 (d) meters from the ground
18
(a) 5.20m (b) 4.33m r r r r
7. If A ´ B = B ´ A, then the angle between A and B is [2004]
(c) 2.60m (d) 8.66m
p p p
(a) (b) (c) p (d)
2 3 4
Motion P-11

8. A projectile can have the same range ‘R’ for two angles of 16. A particle is projected at 60o to the horizontal with a kinetic
projection. If ‘T1’ and ‘T2’ to be time of flights in the two energy K. The kinetic energy at the highest point is [2007]
cases, then the product of the two time of flights is directly (a) K/2 (b) K (c) Zero (d) K/4
proportional to. [2004] 17. The velocity of a particle is v = v0 + gt + ft2. If its position is
x = 0 at t = 0, then its displacement after unit time (t = 1) is
1 1 (a) v0 + g/2 + f (b) v0 + 2g + 3f [2007]
(a) R (b) (c) (d) R 2
R R2 (c) v0 + g/2 + f/3 (d) v0 + g + f
9. Which of the following statements is FALSE for a particle 18. A body is at rest at x = 0. At t = 0, it starts moving in the
moving in a circle with a constant angular speed ? positive x-direction with a constant acceleration. At the same
[2004] instant another body passes through x = 0 moving in the
(a) The acceleration vector points to the centre of the circle positive x-direction with a constant speed. The position of
(b) The acceleration vector is tangent to the circle the first body is given by x1(t) after time ‘t’; and that of the
(c) The velocity vector is tangent to the circle second body by x2(t) after the same time interval. Which of
(d) The velocity and acceleration vectors are perpendicular the following graphs correctly describes (x1 – x 2) as a
function of time ‘t’? [2008]
to each other.
10. An automobile travelling with a speed of 60 km/h, can brake (x1 – x2) (x1 – x2)
to stop within a distance of 20m. If the car is going twice as
fast i.e., 120 km/h, the stopping distance will be [2004]
(a) 60 m (b) 40 m (c) 20 m (d) 80 m
(a) t (b) t
11. A ball is thrown from a point with a speed ' v0 ' at an O O
elevation angle of q. From the same point and at the same
(x1 – x2) (x1 – x2)
' v0 '
instant, a person starts running with a constant speed
2
to catch the ball. Will the person be able to catch the ball? If
yes, what should be the angle of projection q ? [2004] t
(c) O (d) t
(a) No (b) Yes, 30° (c) Yes, 60° (d) Yes, 45° O
12. A car, starting from rest, accelerates at the rate f through a
distance S, then continues at constant speed for time t and 19. Consider a rubber ball freely falling from a height h = 4.9 m
f onto a horizontal elastic plate. Assume that the duration of
then decelerates at the rate to come to rest. If the total collision is negligible and the collision with the plate is totally
2
distance traversed is 15 S , then [2005] elastic.
Then the velocity as a function of time and the height as a
1 2 function of time will be : [2009]
(a) S = ft (b) S = f t
6 v y

1 2 1 2 +v1
h
(c) S = ft (d) S = ft
4 72 (a) O t

13. A particle is moving eastwards with a velocity of 5 ms -1 . In –v1


t
v y
-1
10 seconds the velocity changes to 5 ms northwards. +v1
The average acceleration in this time is [2005] h
(b) O t
1 -2 t1 2t1 4t1
(a) ms towards north
2 –v1
t
1 y
(b) ms - 2 towards north - east
2 h
O t
1 (c) t1 2t1
(c) ms - 2 towards north - west
2 t
(d) zero v y
2 v1 h
14. The relation between time t and distance x is t = ax + bx
where a and b are constants. The acceleration is [2005]
(d) O t t
(a) 2bv 3 (b) -2abv 2 (c) 2 av 2 (d) -2av3
15. A particle located at x = 0 at time t = 0, starts moving along
with the positive x-direction with a velocity 'v' that varies as 20. A particle has an initial velocity of 3iˆ + 4 ˆj and an
v = a x . The displacement of the particle varies with time acceleration of 0.4iˆ + 0.3 ˆj . Its speed after 10 s is : [2009]
as [2006]
(a) t 2 (b) t (c) t 1/2 (d) t 3 (a) 7 2 units (b) 7 units (c) 8.5 units (d) 10 units
EBD_7036
P-12 Topic-wise Solved Papers - PHYSICS
r 28. Two cars of mass m1 and m2 are moving in circles of radii r1
21. A particle is moving with velocity n = k ( yiˆ + xjˆ) , where k
and r2, respectively. Their speeds are such that they make
is a constant. The general equation for its path is [2010] complete circles in the same time t. The ratio of their
(a) y = x2 + constant (b) y2 = x + constant centripetal acceleration is :
(c) xy = constant (d) y2 = x2 + constant (a) m1r1 : m2r2 (b) m1 : m2
22. A point P moves in counter-clockwise direction on a circular (c) r1 : r2 (d) 1 : 1
path as shown in the figure. The movement of 'P' is such 29. A particle of mass m is at rest at the origin at time
that it sweeps out a length s = t3 + 5, where s is in metres and t = 0. It is subjected to a force F(t) = F0e–bt in the x direction.
t is in seconds. The radius of the path is 20 m. The Its speed v(t) is depicted by which of the following curves?
acceleration of 'P' when t = 2 s is nearly. [2010]
y F0 F0
(a) 13m/s2
mb mb
B
(b) 12 m/s2 P(x,y) (a) (b) v(t)
v(t)
(c) 7.2 ms2 m t t
20
(d) 14m/s2 x F0 F0
O A
For a particle in uniform circular motion, the acceleration ar mb mb
23.
at a point P(R,q) on the circle of radius R is (Here q is measured (c) v(t) (d) v(t)
from the x-axis ) [2010] t t
n2 n2 n2 n2
(a) - cos q iˆ + sin q ˆj (b) - sin q iˆ + cos q ˆj 30. A projectile is given an initial velocity of (iˆ + 2 ˆj ) m/s, where
R R R R
n2 n2 n2 ˆ n2 ˆ iˆ is along the ground and ĵ is along the vertical. If g = 10
(c) - cos q iˆ - sin q ˆj (d) i+ j
R R R R m/s2 , the equation of its trajectory is : [JEE-Main 2013]
24. A small particle of mass m is projected at an angle q with the (a) y = x - 5 x2 (b) y = 2 x - 5 x2
x-axis with an initial velocity n0 in the x-y plane as shown in
n sin q (c) 4 y = 2 x - 5 x2 (d) 4 y = 2 x - 25 x 2
the figure. At a time t < 0 , the angular momentum of
g 31. From a tower of height H, a particle is thrown vertically
the particle is [2010] upwards with a speed u. The time taken by the particle, to hit
the ground, is n times that taken by it to reach the highest
(a) - mg n 0 t 2 cos q ˆj y point of its path. The relation between H, u and n is:
[JEE Main 2014]
(b) mg n0t cos q kˆ v0
(b) gH = ( n - 2 ) u 2
2
(a) 2gH = n 2 u 2
1
(c) - mg n 0 t 2 cos q kˆ (c) 2gH = nu 2 ( n - 2 ) (d) gH = ( n - 2 ) u 2
2
q 32. Two stones are thrown up simultaneously from the edge of a
1
(d) mgn 0t 2 cos q iˆ x cliff 240 m high with initial speed of 10 m/s and 40 m/s
2 respectively. Which of the following graph best represents
where iˆ, ˆj and k̂ are unit vectors along x, y and z-axis the time variation of relative position of the second stone
with respect to the first ?
respectively.
(Assume stones do not rebound after hitting the ground and
25. An object, moving with a speed of 6.25 m/s, is decelerated
neglect air resistance, take g = 10 m/ s2)
at a rate given by: [2011]
(The figures are schematic and not drawn to scale)
dv [JEE Main 2015]
= -2.5 v where v is the instantaneous speed. The time
dt
taken by the object, to come to rest, would be: (a) (y2 – y1) m (b) (y2 – y1 ) m
(a) 2 s (b) 4 s (c) 8 s (d) 1 s 240 240
26. A water fountain on the ground sprinkles water all around
it. If the speed of water coming out of the fountain is v, the
total area around the fountain that gets wet is : [2011] 8 12
t(s) t(s)
8 12
4 4 2 2
v pv v v
(a) p (b) (c) p (d) p (y2 – y1 ) m
g2 2 g2 g2 g (c) (y2 – y1 ) m (d) 240
240
27. A boy can throw a stone up to a maximum height of 10 m.
The maximum horizontal distance that the boy can throw
the same stone up to will be : [2012] t(s) 12
t(s)
t® 8 12
(a) 20 2 m (b) 10 m (c) 10 2 m (d) 20 m
CHAPTER

3 Laws of Motion
Section-A JEE Advanced/ IIT-JEE

A Fill in the Blanks C MCQs with One Correct Answer


1. A block of mass 1 kg lies on a horizontal surface in a truck. 1. A ship of mass 3 × 107 kg initially at rest, is pulled by a force
The coefficient of static friction between the block and the of 5 × 104 N through a distance of 3m. Assuming that the
surface is 0.6. If the acceleration of the truck is 5 m/s2, the resistance due to water is negligible, the speed of the ship is
frictional force acting on the block is ..... newtons. (1980)
(1984 - 2 Marks) (a) 1.5 m/sec. (b) 60 m/sec.
(c) 0.1 m/sec. (d) 5 m/sec.
2. A uniform rod of length L and density r is being pulled
2. A block of mass 2 kg rests on a rough inclined plane making
along a smooth floor with a horizontal acceleration a (see an angle of 30° with the horizontal. The coefficient of static
Fig.) The magnitude of the stress at the transverse cross- friction between the block and the plane is 0.7. The frictional
section through the mid- point of the rod is ......... force on the block is
(1993 - 1 Mark) (a) 9.8 N (b) 0.7 ´ 9.8 ´ 3N
L (c) 9.8 ´ 3N (d) 0.7 × 9.8 N (1980)
3. A block of mass 0.1 is held against a wall applying a horizontal
a
force of 5 N on the block. If the coefficient of friction between
the block and the wall is 0.5, the magnitude of the frictional
force acting on the block is : (1994 - 1 Mark)
(a) 2.5 N (b) 0.98 N
B True/False (c) 4.9 N (d) 0.49 N
4. A small block is shot into each of the four tracks as shown
1. A rocket moves forward by pushing the surrounding air below. Each of the tracks rises to the same height. The
backwards. (1980) speed with which the block enters the track is the same in all
2. When a person walks on a rough surface, the frictional force cases. At the highest point of the track, the normal reaction
exerted by the surface on the person is opposite to the is maximum in (2001S)
direction of his motion. (1981 - 2 Marks)
3. A simple pendulum with a bob of mass m swings with an
angular amplitude of 40°. When its angular displacement is (a) (b) v
v
20°, the tension in the string is greater than mg cos 20°.
(1984 - 2 Marks)
4. The pulley arrangements of Figs. (a) and (b) are identical.
The mass of the rope is negligible. In (a) the mass m is lifted (c) v (d) v
up by attaching a mass 2m to the other end of the rope. In
(b) , m is lifted up by pulling the other end of the rope with 5. An insect crawls up a hemispherical surface very slowly
(see fig.). The coefficient of friction between the insect and
a constant downward force F = 2 mg. The acceleration of m
the surface is 1/3. If the line joining the center of the
is the same in both cases (1984 - 2 Marks)
hemispherical surface to the insect makes an angle a with
the vertical, the maximum possible value of a is given by
(2001S)

m F = 2 mg
m 2m (a) cot a = 3 (b) tan a = 3
(a) (b) (c) sec a = 3 (d) cosec a = 3
EBD_7036
P-14 Topic-wise Solved Papers - PHYSICS
6. The pulleys and strings shown in the figure are smooth and move towards each other on the surface. The magnitude of
of negligible mass. For the system to remain in equilibrium, acceleration, when the separation between them becomes
the angle q should be (2001S) 2x, is (2007)
F a F
(a)
(a) 0o 2m a 2 - x 2
q
(b) 30o F x
(b) m m
2m a - x 2
2 P
Ö2m
(c) 45o a a
F x
(c)
(d) 60o m m 2m a
7. A string of negligible mass going over a F a2 - x2
clamped pulley of mass m supports a block m (d)
2m x
of mass M as shown in the figure. The 12. A particle moves in the X-Y plane under the influence of a
force on the pulley by the clamp is given force such that its linear momentum is
by (2001S) r
p(t )= A [iˆ cos(kt ) - ˆj sin(kt )], where A and k are constants.
M The angle between the force and the momentum is (2007)
(a) 2 Mg (b) 2 mg
(a) 0° (b) 30°
(c) ( M + m) 2 + m 2 g (d) (M + m)2 + M2 g (c) 45° (d) 90°
8. What is the maximum value of the force F such that the 13. A block of base 10 cm × 10 cm and height 15 cm is kept on an
block shown in the arrangement, does not move? inclined plane. The coefficient of friction between them is
F
(2003S) 3. The inclination q of this inclined plane from the
horizontal plane is gradually increased from 0°. Then
1 (2009)
m=
60º 2 3 (a) at q = 30°, the block will start sliding down the plane
m = 3kg (b) the block will remain at rest on the plane up to certain q
and then it will topple
(a) 20N (b) 10N (c) at q = 60°, the block will start sliding down the plane
(c) 12N (d) 15N and continue to do so at higher angles
9. A block P of mass m is placed on a horizontal frictionless (d) at q = 60°, the block will start sliding down the plane
and on further increasing q, it will topple at certain q.
plane. A second block of same mass m is placed on it and is
14. A block of mass m is on an inclined plane of angle q. The
connected to a spring of spring constant k, the two blocks
coefficient of friction between the block and the plane is m
are pulled by distance A. Block Q oscillates without slipping.
and tan q > m. The block is held stationary by applying a
What is the maximum value of frictional force between the
force P parallel to the plane. The direction of force pointing
two blocks. (2004S)
up the plane is taken to be positive. As P is varied from
P1 = mg(sinq – m cosq ) to P2 = mg(sinq + m cosq), the
k frictional force f versus P graph will look like (2010)
Q
ms f f
P

(a) kA/2 (b) kA


(c) ms mg (d) zero (a) (b)
10. The string between blocks of mass m
and 2m is massless and inextensible. f f
The system is suspended by a
massless spring as shown. If the string
is cut find the magnitudes of (c) (d)
accelerations of mass 2m and m
2m
(immediately after cutting)
(2006 - 3M, –1) 15. A ball of mass (m) 0.5 kg is attached to
m the end of a string having length (L)
0.5 m. The ball is rotated on a
g g g g horizontal circular path about vertical
(a) g, g (b) g, (c) , g (d) ,
2 2 2 2 axis. The maximum tension that the
11. Two particles of mass m each are tied at the ends of a light string can bear is 324 N. The maximum L
string of length 2a. The whole system is kept on a frictionless possible value of anguar velocity of
horizontal surface with the string held tight so that each mass
is at a distance 'a' from the centre P (as shown in the figure). ball (in radian/s) is (2011)
Now, the mid-point of the string is pulled vertically upwards (a) 9 (b) 18 m
with a small but constant force F. As a result, the particles (c) 27 (d) 36
Laws of Motion P-15

16. The image of an object, formed by a plano-convex lens at a 5. A small block of mass of 0.1 kg lies on a fixed inclined plane
distance of 8 m behind the lens, is real is one-third the size of PQ which makes an angle q with the horizontal. A horizontal
2 force of 1 N acts on the block through its centre of mass as
the object. The wavelength of light inside the lens is shown in the figure. (2012)
3
times the wavelength in free space. The radius of the curved The block remains stationary if (take g = 10 m/s2)
surface of the lens is (JEE Adv. 2013)
(a) 1 m (b) 2 m (a) q = 45°
(c) 3 m (d) 6 m (b) q > 45° and a frictional
force acts on the block
D MCQs with One or More than One Correct towards P.
(c) q > 45° and a frictional
1. In the arrangement shown in the Fig, the ends P and Q of an force acts on the block
unstretchable string move downwards with uniform speed towards Q.
U. Pulleys A and B are fixed. (1982 -3 Marks)
Mass M moves upwards with a speed (d) q < 45° and a frictional force acts on the block towards Q.
6. A wire, which passes through the hole A
(a) 2Ucos q A in a small bead, is bent in the form of
B
quarter of a circle. The wire is fixed
(b) U/ cos q vertically on ground as shown in the
qq
figure. The bead is released from near
(c) 2U / cos q the top of the wire and it slides along
P Q
M the wire without friction. As the bead 90°
(d) U cos q moves from A to B, the force it applies B
2. A reference frame attached to the earth (1986 - 2 Marks) on the wire is (JEE Adv. 2014)
(a) is an inertial frame by definition. (a) always radially outwards
(b) cannot be an inertial frame because the earth is (b) always radially inwards
revolving round the sun. (c) radially outwards initially and radially inwards later
(c) is an inertial frame because Newton’s laws are applicable (d) radially inwards initially and radially outwards later
in this frame.
(d) cannot be an inertial frame because the earth is rotating
about its own axis. E Subjective Problems
3. A simple pendulum of length L and mass (bob) M is
oscillating in a plane about a vertical line between angular 1. In the diagram shown, the
limit – f and + f . For an angular displacement q ( | q | < f ), blocks A, B and C weight, A
3 kg, 4 kg and 5 kg B
the tension in the string and the velocity of the bob are T
respectively. The coefficient f C
and V respectively. The following relations hold good under
the above conditions : (1986 - 2 Marks) of sliding friction between
(a) T cos q = Mg. any two surface is 0.25. A is held at rest by a massless rigid
MV 2 rod fixed to the wall while B and C are connected by a light
(b) T – Mg cos q = flexible cord passing around a frictionless pulley. Find the
L
(c) The magnitude of the tangenial acceleration of the bob force F necessary to drag C along the horizontal surface to
| aT | = g sin q the left at constant speed. Assume that the arrangement
shown in the diagram, B on C and A on B, is maintained all
(d) T = Mg cos q
through. (g = 9.8 m/s2) (1978)
4. A particle P is sliding down a frictionless hemispherical bowl.
2. Two cubes of masses m1 and
It passes the point A at t = 0. At this instant of time, the m2
m 2 be on two frictionless
horizontal component of its velocity is v. A bead Q of the
slopes of block A which rests
same mass as P is ejected from A at t = 0 along the horizontal b f
on a horizontal table. The
string AB, with the speed v. Friction between the bead and cubes are connected by a
the string may be neglected. Let tP and tQ be the respective string which passes over a m1
times taken by P and Q to reach the point B. Then : pulley as shown in the figure. A
a
(1993-2 Marks) To what horizontal acceleration
(a) t P < tQ Q f should the whole system (that is blocks and cubes) be
A B subjected so that the cubes do not slide down the planes.
(b) t P = tQ What is the tension of the string in this situation? (1978)
(c) t P > tQ 3. A horizontal uniform rope of length L, resting on a
P C frictionless horizontal surface, is pulled at one end by force
tP length of arc ACB F. What is the tension in the rope at a distance l from the
(d) =
tQ length of arc AB end where the force is applied? (1978)
EBD_7036
P-16 Topic-wise Solved Papers - PHYSICS
4. Masses M1, M2 and M3 are connected by strings of negligible 8. A particle of mass 10–2 kg is moving along the positive x
mass which pass over massless and friction less pulleys P1 axis under the influence of a force F(x) = – K/(2x2) where
and P2 as shown in fig The masses move such that the K = 10–2 N m2. At time t = 0 it is at x = 1.0 m and its velocity
portion of the string between P1 and P2 in parallel to the is v = 0. (1998 - 8 Marks)
inclined plane and the portion of the string between P2 and (a) Find its velocity when it reaches x = 0.50 m.
M3 is horizontal. The masses M2 and M3 are 4.0 kg each and (b) Find the time at which it reaches x = 0.25 m.
the coefficient of kinetic friction between the masses and 9. In the figure masses m1, m2 and M are 20 kg, 5 kg and 50 kg
the surfaces is 0.25. The inclined plane makes an angle of respectively. The coefficient of friction between M and
37° with the horizontal. (1981- 6 Marks) ground is zero. The coefficient of friction between m1 and M
p and that between m2 and ground is 0.3. The pulleys and the
1
strings are massless. The string is perfectly horizontal
2 p between P1 and m1 and also between P2 and m2. The string
2 is perfectly vertical between P1 and P2. An external horizontal
3
force F is applied to the mass M. Take g = 10 m/s2.
1 (2000 - 10 Marks)
37°
P1 m1

If the mass M1 moves downwards with a uniform velocity,


find M
P2 F
(i) the mass of M1
(ii) The tension in the horizontal portion of the string m2
(g = 9.8 m/sec2, sin 37° ~ - 3/5)
5. A particle of mass m rests on a horizontal floor with which it (a) Draw a free body diagram for mass M, clearly showing
has a coefficient of static friction m . It is desired to make the all the forces.
body move by applying the minimum possible force F. Find (b) Let the magnitude of the force of friction between m1
the maguitude of F and the direction in which it has to be and M be f1 and that between m2 and ground be f2. For
applied. (1987 - 7 Marks) a particular F it is found that f1 = 2f2. Find f1 and f2.
6. Two blocks of mass 2.9 kg and 1.9 kg are Write equations of motion of all the masses. Find F,
S tension in the string and acceleration of the masses.
suspended from a rigid support S by two
inextensible wires each of length 1 meter, see 10. Two block A and B of equal masses are placed on rough
fig. The upper wire has negligible mass and inclined plane as shown in figure. When and where will the
the lower wire has a uniform mass of 0.2 kg/m. two blocks come on the same line on the inclined plane
The whole system of blocks wires and 2.9 kg if they are released simultaneously? Initially the block A
support have an upward acceleration of 0.2
is 2 m behind the block B. Co-efficient of kinetic
m/s2. Acceleration due to gravity is 9.8 m/s2.
friction for the blocks A and B are 0.2 and 0.3 respectively
(1989 - 6 Marks) (g =10 m/s2). (2004 - Marks)
1.9 kg 2m
(i) Find the tension at the mid-point of the A
lower wire. B
A
(ii) Find the tension at the mid-point of the
upper wire. B
45º
7. A smooth semicircular wire-track of radius R is fixed in a 11. A circular disc with a groove along its diameter is placed
vertical plane. One end of a massless spring of natural length horizontally on a rough surface. A block of mass 1 kg is
3R/4 is attached to the lowest point O of the wire-track. A placed as shown. The co-efficient of friction between the
small ring of mass m, which can slide on the track, is block and all surfaces of groove and horizontal surface in
attached to the other end of the 2
spring. The ring is held contact is m = . The disc has an acceleration of 25 m/s2
stationary at point P such that 5
towards left. Find the acceleration of the block with respect
the spring makes an angle of
60° with the vertical. The 4 3
spring constant K= mg/R. to disc. Given cos q = , sin q = . (2006 - 6M)
5 5
Consider the instant when the
ring is released, and (i) draw
the free body diagram of the
ring, (ii) determine the 2
25 m/s
tangential acceleration of the
ring and the normal reaction. q
(1996 - 5 Marks)
Laws of Motion P-17

F Match the Following

DIRECTIONS (Q. No. 1) : Following question has matching lists. The codes for the lists have choices (a), (b), (c) and (d) out of
which ONLY ONE is correct.
1. A block of mass m1 = 1 kg another mass m2 = 2 kg, are placed together (see figure) on an inclined plane with
angle of inclination q. Various values of q are given in List-I. The coefficient of friction between the block m1 m1
and plane is always zero. The coefficient of static and dynamic friction between the block m2 and the plane m2
are equal to m = 0.3. In List-II expressions for the friction on block m2 are given. Match the correct expression
of the friction in List-II with the angles given in List-I, and choose the correct option. The acceleration due
to gravity is denoted by g. q
[Useful information: tan (5.5°) » 0.1; tan (11.5°) » 0.2; tan (16.5°) » 0.3] (JEE Adv. 2014)
List-I List-II
P. q = 5° 1. m2g sinq
Q. q = 10° 2. (m1 + m2)g sinq
R. q = 15° 3. mm2g cosq
S. q = 20° 4. m(m1 + m2)g cosq
Code:
(a) P-1, Q-1, R-1, S-3 (b) P-2, Q-2, R-2, S-3 (c) P-2, Q-2, R-2, S-4 (d) P-2, Q-2, R-3, S-3

2. The net reaction of the disc on the block is (JEE Adv. 2016)
G Comprehension Based Questions 1
mw2 R e2wt - e -2wt ˆj + mgkˆ
(a)
2 ( )
PARAGRAPH 1
mw2 R ewt - e-wt ˆj + mgkˆ
(
A frame of reference that is accelerated with respect to an inertial
frame of reference is called a non-inertial frame of reference. A
(b)
2 )
coordinate system fixed on a circular disc rotating about a fixed (c) - mw2 R coswt ˆj - mgkˆ
axis with a constant angular velocity w is an example of non- (d) mw2 R sinswt ˆj - mgkˆ
inertial frame of reference. The relationship between the force
r
Frot experienced by a particle of mass m moving on the rotating H Assertion & Reason Type Questions
r
disc and the force Fin experienced by the particle in an inertial 1. STATEMENT-1 : A cloth covers a table. Some dishes are
frame of reference is kept on it. The cloth can be pulled out without dislodging
r r r ur ur r ur the dishes from the table.
(
Frot = Fin + 2m v rot ´w + m w´ r ´w.
r
) ( ) STATEMENT-2 : For every action there is an equal and
where v rot is the velocity of the particle in the rotating frame of opposite reaction. (2007)
r (a) Statement-1 is True, Statement-2 is True; Statement-2
reference and r is the position vector of the particle with respect is a correct explanation for Statement-1
to the centre of the disc. (b) Statement-1 is True, Statement-2 is True; Statement-2
Now consider a smooth slot along a w is NOT a correct explanation for Statement-1
diameter of a disc of radius R rotating (c) Statement-1 is True, Statement-2 is False
counter-clockwise with a constant (d) Statement-1 is False, Statement-2 is True.
R
angular speed w about its vertical 2. STATEMENT-1 : It is easier to pull a heavy object than to
axis through its center. We assign a m push it on a level ground and
coordinate system with the origin R/2 STATEMENT-2 : The magnitude of frictional force depends
at the center of the disc, the x-axis on the nature of the two surfaces in contact. (2008)
along the slot, the y-axis (a) Statement-1 is True, Statement-2 is True;
perpendicular to the slot and the z-axis along the rotation axis Statement-2 is a correct explanation for Statement-1
ur (b) Statement-1 is True, Statement-2 is True; Statement-2
(r
)
w = wk̂ . A small block of mass m is gently placed in the slot at is NOT a correct explanation for Statement-1
(c) Statement -1 is True, Statement-2 is False
r (R / 2) ˆi at t = 0 and is constrained to move only along the slot. (d) Statement -1 is False, Statement-2 is True
1. The distance r of the block at time t is (JEE Adv. 2016)
I Integer Value Correct Type
R wt -wt R
(a)
4 (
e +e
) (b)
2
cos wt
1. A block is moving on an inclined plane making an angle 45°
with the horizontal and the coefficient of friction is m. The
R 2wt R force required to just push it up the inclined plane is 3 times
+ e -2wt
4( )
(c) e (d) cos 2wt
2 the force required to just prevent it from sliding down. If we
define N = 10 m, then N is (2011)
EBD_7036
P-18 Topic-wise Solved Papers - PHYSICS

Section-B JEE Main / AIEEE


1. If a body looses half of its velocity on penetrating 3 cm in a 9. A spring balance is attached to the ceiling of a lift. A man
wooden block, then how much will it penetrate more before hangs his bag on the spring and the spring reads 49 N,
coming to rest? [2002] when the lift is stationary. If the lift moves downward with
(a) 1 cm (b) 2 cm an acceleration of 5 m/s2, the reading of the spring balance
(c) 3 cm (d) 4 cm. will be [2003]
2. A lift is moving down with acceleration a. A man in the lift (a) 24 N (b) 74 N
drops a ball inside the lift. The acceleration of the ball as (c) 15 N (d) 49 N
observed by the man in the lift and a man standing stationary 10. Three forces start acting simultaneously C
r
on the ground are respectively [2002] on a particle moving with velocity, v .
(a) g, g (b) g – a, g – a These forces are r epresented in
(c) g – a, g (d) a, g magnitude and direction by the three
3. When forces F1, F2, F3 are acting on a particle of mass m sides of a triangle ABC. The particle
such that F2 and F3 are mutually perpendicular, then the will now move with velocity [2003]
A B
particle remains stationary. If the force F1 is now removed r
then the acceleration of the particle is [2002] (a) less than v
r
(a) F1/m (b) F2F3/mF1 (b) greater than v
(c) (F2 – F3)/m (d) F2/m. (c) v in the direction of the largest force BC
4. Two forces are such that the sum of their magnitudes is 18 N r
(d) v , remaining unchanged
and their resultant is 12 N which is perpendicular to the
11. A horizontal force of 10 N is
smaller force. Then the magnitudes of the forces are necessary to just hold a block
(a) 12 N, 6 N (b) 13 N, 5 N [2002] stationary against a wall. The
(c) 10 N, 8 N (d) 16N, 2N. coefficient of friction between
5. Speeds of two identical cars are u and 4u at the specific the block and the wall is 0.2. 10N
instant. The ratio of the respective distances in which the The weight of the block is
two cars are stopped from that instant is [2002]
(a) 1 : 1 (b) 1 : 4 (a) 20 N (b) 50 N [2003]
(c) 1 : 8 (d) 1 : 16. (c) 100 N (d) 2 N
6. A light string passing over a smooth light pulley connects 12. A marble block of mass 2 kg lying on ice when given a
two blocks of masses m 1 and m2 (vertically). If the velocity of 6 m/s is stopped by friction in 10 s. Then the
acceleration of the system is g/8, then the ratio of the masses coefficient of friction is [2003]
is [2002] (a) 0.02 (b) 0.03
(a) 8 : 1 (b) 9 : 7 (c) 0.04 (d) 0.06
(c) 4 : 3 (d) 5 : 3. 13. A block of mass M is pulled along a horizontal frictionless
7. Three identical blocks of masses m = 2 kg are drawn by a surface by a rope of mass m. If a force P is applied at the free
force F = 10. 2 N with an acceleration of 0. 6 ms–2 on a end of the rope, the force exerted by the rope on the block is
frictionless surface, then what is the tension (in N) in the Pm Pm
(a) (b) [2003]
string between the blocks B and C? [2002] M +m M -m
PM
C B A F (c) P (d)
M +m
14. A light spring balance hangs from the hook of the other
(a) 9.2 (b) 3.4
light spring balance and a block of mass M kg hangs from
(c) 4 (d) 9.8 the former one. Then the true statement about the scale
8. One end of a massless rope, which passes over a massless reading is [2003]
and frictionless pulley P is tied to a hook C while the other
(a) Both the scales read M kg each
end is free. Maximum tension that the rope can bear is 360
(b) The scale of the lower one reads M kg and of the upper
N. With what value of maximum safe acceleration (in ms–2)
one zero
can a man of 60 kg climb on the rope? [2002]
(c) The reading of the two scales can be anything but the
(a) 16 sum of the reading will be M kg
P (d) Both the scales read M/2 kg each
(b) 6 15. A rocket with a lift-off mass 3.5 × 104 kg is blasted upwards
C with an initial acceleration of 10m/s2. Then the initial thrust
(c) 4 of the blast is [2003]
(a) 3.5 ´ 10 N5 5
(b) 7.0 ´ 10 N
(d) 8.
(c) 14.0 ´ 10 5 N (d) 1.75 ´ 10 5 N
Laws of Motion P-19

16. Two masses m1 = 5kg and m2 = 4.8 kg tied to a string 23. A particle of mass 0.3 kg subject to a force F = – kx with
k = 15 N/m . What will be its initial acceleration if it is released
are hanging over a light frictionless pulley. What is the
from a point 20 cm away from the origin ? [2005]
acceleration of the masses when left free to move ?
(a) 15 m / s 2 (b) 3 m / s 2
( g = 9.8m / s 2 ) [2004] (c) 10 m / s
2
(d) 5 m / s 2
24. A block is kept on a frictionless inclined surface with angle
(a) 5 m / s2
of inclination ‘ a ’ . The incline is given an acceleration ‘a’ to
(b) 9.8 m / s 2 keep the block stationary. Then a is equal to [2005]

(c) 0.2 m / s 2
(d)4.8 m / s 2
17. A block rests on a rough inclined plane making an angle of
30° with the horizontal. The coefficient of static friction a
between the block and the plane is 0.8. If the frictional force a
on the block is 10 N, the mass of the block (in kg) is (a) g cosec a (b) g / tan a
(c) g tan a (d) g
(take g = 10 m / s 2 ) [2004]
25. Consider a car moving on a straight road with a speed of
(a) 1.6 (b) 4.0
(c) 2.0 (d) 2.5 100 m/s . The distance at which car can be stopped is
18. A smooth block is released at rest on a 45° incline and then [ m k = 0.5 ] [2005]
slides a distance ‘d’. The time taken to slide is ‘n’ times as
(a) 1000 m (b) 800 m
much to slide on rough incline than on a smooth incline.
The coefficient of friction is [2005] (c) 400 m (d) 100 m
26. A mass of M kg is suspended by a weightless string. The
1 1
(a) mk = 1 – 2 (b) mk = 1- horizontal force that is required to displace it until the string
n n2 makes an angle of 45° with the initial vertical direction is
1 1
(c) ms = 1 - ms = 1-
(d) (a) Mg ( 2 + 1) (b) Mg 2 [2006]
n 2
n2
19. A parachutist after bailing out falls 50 m without friction.
Mg
When parachute opens, it decelerates at 2 m / s . He 2 (c) (d) Mg ( 2 - 1)
2
reaches the ground with a speed of 3 m/s. At what height,
did he bail out ? [2005] 27. A ball of mass 0.2 kg is thrown vertically upwards by applying
(a) 182 m (b) 91 m a force by hand. If the hand moves 0.2 m while applying the
(c) 111 m (d) 293 m force and the ball goes upto 2 m height further, find the
20. A bullet fired into a fixed target loses half of its velocity after magnitude of the force. (Consider g = 10 m/s2).
penetrating 3 cm. How much further it will penetrate before (a) 4 N (b) 16 N [2006]
coming to rest assuming that it faces constant resistance to
motion ? [2005] (c) 20 N (d) 22 N
(a) 2.0 cm (b) 3.0 cm 28. A player caught a cricket ball of mass 150 g moving at a rate
(c) 1.0 cm (d) 1.5 cm of 20 m/s. If the catching process is completed in 0.1s, the
21. An annular ring with inner and outer radii R1 and R2 is force of the blow exerted by the ball on the hand of the
rolling without slipping with a uniform angular speed. The player is equal to [2006]
ratio of the forces experienced by the two particles situated (a) 150 N (b) 3 N
F1 (c) 30 N (d) 300 N
on the inner and outer parts of the ring , F is [2005] 29. A coin is placed on a horizontal platform which undergoes
2
vertical simple harmonic motion of angular frequency w. The
2 R2 R1
æ R1 ö amplitude of oscillation is gradually increased. The coin will
(a) çè R ÷ø (b) R (c) R2 (d) 1
2 1 leave contact with the platform for the first time
(a) at the mean position of the platform [2006]
22. The upper half of an inclined plane with inclination f is
perfectly smooth while the lower half is rough. A body g
(b) for an amplitude of
starting from rest at the top will again come to rest at the w2
bottom if the coefficient of friction for the lower half is given
by [2005] g2
(c) for an amplitude of
(a) 2 cos f (b) 2 sin f w2
(d) at the highest position of the platform
(c) tan f (d) 2 tan f
EBD_7036
P-20 Topic-wise Solved Papers - PHYSICS
30. A block of mass m is connected to another block of mass M 33. A block of mass m is placed on a surface with a vertical cross
by a spring (massless) of spring constant k. The block are
x3
kept on a smooth horizontal plane. Initially the blocks are at section given by y = . If the coefficient of friction is 0.5,
rest and the spring is unstretched. Then a constant force F 6
starts acting on the block of mass M to pull it. Find the force the maximum height above the ground at which the block can
of the block of mass m. [2007] be placed without slipping is: [JEE Main 2014]
1 2
MF mF (a) m (b) m
(a) (b) 6 3
(m + M ) M
1 1
( M + m) F mF (c) m (d) m
(c) (d) 3 2
m (m + M ) 34. Given in the figure are two blocks A and B of weight 20 N and
31. Two fixed frictionless inclined planes making an angle 30° 100 N, respectively. These are being pressed against a wall
and 60° with the vertical are shown in the figure. Two blocks by a force F as shown. If the coefficient of friction between
A and B are placed on the two planes. What is the relative the blocks is 0.1 and between block B and the wall is 0.15, the
vertical acceleration of A with respect to B ? [2010] frictional force applied by the wall on block B is:
[JEE Main 2015]
A
B F
A B

(a) 120 N (b) 150 N


30°
(c) 100 N (d) 80 N
60°
35. A point particle of mass m, moves long the uniformly rough
(a) 4.9 ms–2 in horizontal direction track PQR as shown in the figure. The coefficient of friction,
(b) 9.8 ms–2 in vertical direction between the particle and the rough track equals m. The
(c) Zero particle is released, from rest from the point P and it comes
(d) 4.9 ms–2 in vertical direction to rest at a point R. The energies, lost by the ball, over the
32. A mass ‘m’ is supported by a massless string wound around parts, PQ and QR, of the track, are equal to each other, and
a uniform hollow cylinder of mass m and radius R. If the no energy is lost when particle changes direction from PQ
string does not slip on the cylinder, with what acceleration to QR.
will the mass fall or release? [JEE Main 2014] The value of the coefficient of friction m and the distance x
2g (= QR), are, respectively close to : [JEE Main 2016]
(a) P
3
R h=2m
g
(b) m
2 30° R
5g Horizontal Q
(c) Surface
6
(a) 0.29 and 3.5 m (b) 0.29 and 6.5 m
(d) g m
(c) 0.2 and 6.5 m (d) 0.2 and 3.5 m
CHAPTER
Work, Energy and
4 Power
Section-A JEE Advanced/ IIT-JEE
7. An ideal spring with spring-constant k is hung from the
C MCQs with One Correct Answer ceiling and a block of mass M is attached to its lower end.
1. If a machine is lubricated with oil (1980) The mass is released with the spring initially unstretched.
(a) the mechanical advantage of the machine increases. Then the maximum extension in the spring is (2002S )
(b) the mechanical efficiency of the machine increases.
(c) both its mechanical advantage and efficiency increase. 4 Mg 2 Mg Mg Mg
(a) (b) (c) (d)
(d) its efficiency increases, but its mechanical advantage k k k 2k
decreases. 8. If W1, W2 and W3 represent the work done in moving a
2. Two masses of 1 gm and 4 gm are moving with equal kinetic particle from A to B along three different paths 1,2 and 3
energies. The ratio of the magnitudes of their linear momenta
respectively (as shown) in the gravitational field of a point
is (1980)
mass m, find the correct relation between W1, W2 and W3
(a) 4 : 1 (b) 2 :1 (c) 1 : 2 (d) 1 : 16
(2003S)
3. A particle of mass m is moving in a circular path of constant
radius r such that its centripetal acceleration ac is varying B
(a) W1> W2 > W3
with time t as ac = k2rt2 where k is a constant. The power
m
delivered to the particles by the force acting on it is:
(1994 - 1 Mark) (b) W1= W2 = W3
1 2
(a) 2p mk r t2 2 2
(b) mk r t 2
3
(mk 4 r 2 t 5 ) (c) W1< W2 < W3
(c) (d) zero
3
4. A spring of force-constant k is cut into two pieces such that (d) W2> W1 > W3 A
one piece is double the length of the other. Then the long
piece will have a force-constant of (1999S - 2 Marks) 9. A particle is acted by a force F = kx, where k is a +ve
(a) (2/3)k (b) (3/2)k (c) 3 k (d) 6 k constant. Its potential energy at x = 0 is zero. Which curve
5. A wind-powered generator converts wind energy into correctly represents the variation of potential energy of
electrical energy. Assume that the generator converts a fixed
the block with respect to x (2004S)
fraction of the wind energy intercepted by its blades into
electrical energy. For wind speed v, the electrical power U
output will be proportional to (2000S ) U
(a) v (b) v 2 (c) v 3 (d) v 4
6. A particle, which is constrained to move along the x-axis, (a) x (b) x
is subjected to a force in the same direction which
varies with the distance x of the particle from the origin as
F(x) = –kx + ax3. Here k and a are positive constants. For
x ³ 0 , the functional form of the potential energy U(x) of
the particle is (2002S ) U U

U(x) U(x) (c) x (d) x


(a) X (b) X

10. A block (B) is attached to two unstretched springs S1 and


U(x) U(x) S2 with spring constants k and 4k, respectively (see fig. I).
(c) X (d) X The other ends are attached to identical supports M1 and
M2 not attached to the walls. The springs and supports
EBD_7036
P-22 Topic-wise Solved Papers - PHYSICS
have negligible mass. There is no friction anywhere. The
14. The work done on a particle of mass m by a force,
block B is displaced towards wall 1 by a small distance x
(figure II) and released. The block returns and moves a é x y ù
Kê iˆ + ˆj ú
maximum distance y towards wall 2. Displacements x and y êë ( x 2 + y 2 )3 2 ( x 2 + y 2 )3 2 úû
are measured with respect to the equilibrium position of the (K being a constant of appropriate dimensions), when the
block B. The ratio y/x is – (2008) particle is taken from the point (a, 0) to the point (0, a) along
2 1 a circular path of radius a about the origin in the x – y plane
M2 S M1
2 S1 is (JEE Adv. 2013)
B I
2K p Kp Kp
x (a) (b) (c) (d) 0
2 1 a a 2a
M2 M1 15. A tennis ball is dropped on a horizontal smooth surface. It
S2 S1 bounces back to its original position after hitting the surface.
B II
The force on the ball during the collision is proportional to
x
the length of compression of the ball. Which one of the
(a) 4 (b) 2 (c) 1/2 (d) 1/4 following sketches describes the variation of its kinetic
11. Two small particles of equal masses start moving in opposite energy K with time t most appropriately? The figure are only
directions from a point A in a horizontal circular orbit. Their illustrative and not to the scale. (JEE Adv. 2014)
tangential velocities are v and 2v, respectively, as shown in
the figure. Between collisions, the particles move with K K
constant speeds. After making how many elastic collisions,
other than that at A, these two particles will again reach the (a) (b)
point A? (2009) t
A t
v 2v
K K

(c) (d)
t t

(a) 4 (b) 3 (c) 2 (d) 1 D MCQs with One or More than One Correct
12. A piece of wire is bent in the shape of a parabola y = kx2
(y-axis vertical) with a bead of mass m on it. The bead can 1. A body is moved along a straight line by a machine delivering
constant power. The distance moved by the body in time t
slide on the wire without friction. It stays at the lowest point
is proportional to (1984- 2 Marks)
of the parabola when the wire is at rest. The wire is now
(a) t 1/2 (b) t 3/4 (c) t 3/2 (d) t 2
accelerated parallel to the x-axis with a constant acceleration
2. A uniform chain of length L and mass M is lying on a smooth
a. The distance of the new equilibrium position of the bead,
table and one third of its length is hanging vertically down
where the bead can stay at rest with respect to the wire,
over the edge of the table. If g is acceleration due to gravity,
from the y-axis is (2009)
the work required to pull the hanging part on to the table is
a a 2a a (1985 - 2 Marks)
(a) (b) (c) (d) (a) MgL (b) MgL/3 (c) MgL/9 (d) MgL/18
gk 2 gk gk 4 gk
3. A particle is acted upon by a force of constant magnitude
13. A block of mass 2 kg is free to move along the x-axis. It is at
which is always perpendicular to the velocity of the particle.
rest and from t = 0 onwards it is subjected to a time-dependent
The motion of the particle takes place in a plane. It follows
force F(t) in the x direction. The force F(t) varies with t as
that : (1987 - 2 Marks)
shown in the figure. The kinetic energy of the block after 4.5
(a) its velocity is constant
seconds is (2010)
F(t) (b) its acceleration is constant
(a) 4.50 J (c) its kinetic energy is constant.
4N (d) it moves in a circular path.
(b) 7.50 J 4. A force F = - K ( yiˆ + xjˆ) (where K is a positive constant) acts
4.5s on a particle moving in the xy plane. Starting from the origin,
(c) 5.06 J O 3s t the particle is taken along the positive x axis to the point (a, 0),
and then parallel to the y axis to the point (a, a), The total work
(d) 14.06 J
done by the force F on the particle is (1998S - 2 Marks)
2
(a) – 2Ka (b) 2Ka 2 (c) – Ka 2 (d) Ka 2
Work, Energy and Power P-23
5. A stone tied to a string of length L is whirled in a vertical 6. Two blocks A an d B are
circle with the other end of the string at the centre. At a connected to each other by a
certain instant of time, the stone is at its lowest position, string and a spring ; the string
and has a speed u. The magnitude of the change in its B
velocity as it reaches a position where the string is horizontal passes over a frictionless pulley
is (1998S - 2 Marks) as shown in the figure. Block B
slides over the horizontal top C
(a) u 2 - 2 gL (b) 2gL
surface of a stationary block C A
(c) u 2 - gL (d) 2(u 2 - gL) and the block A slides along the
6. A small ball starts moving from A over a fixed track as shown vertical side of C, both with the
in the figure. Surface AB has friction. From A to B the ball rolls same uniform speed.
without slipping. Surface BC is frictionless. KA, KB and KC The coefficient of friction between the surfaces of blocks is
are kinetic energies of the ball at A, B and C, respectively.
0.2. Force constant of the spring is 1960 newtons/m. If mass
Then (2006 - 5M, –1)
of block A is 2 Kg., calculate the mass of block B and the
C
A energy stored in the spring. (1982 - 5 Marks)
7. A 0.5 kg block slides from the point A (see Fig) on a horizontal
track with an initial speed of 3 m/s towards a weightless
hC
hA horizontal spring of length 1 m and force constant 2 Newton/
m. The part AB of the track is frictionless and the part BC
has the coefficients of static and kinetic friction as 0.22 and
B 0 2 respectively. If the distances AB and BD are 2 m and 2.14
(a) hA > hC ; K B > KC (b) hA > hC ; KC > K A m respectively, find the total distance through which the
block moves before it comes to rest completely.
(c) hA = hC ; K B = KC (d) hA < hC ; K B > KC
(Take g = 10m / s2) (1983 - 7 Marks)
E Subjective Problems
1. A bullet is fired from a rifle. If the rifle recoils freely, determine
whether the kinetic energy of the rifle is greater than, equal
or less than that of the bullet. (1978) A B D C
2. A spring of force constant k is cut into three equal parts.
8. A string, with one end fixed on a rigid wall, passing over a
What is force constant of each part? (1978)
3. A 20 gm bullet pierces through a plate of mass M1 = 1 kg and fixed frictionless pulley at a distance of 2m from the wall,
then comes to rest inside a second plate of mass M2 = 2.98 has a point mass M = 2kg attached to it at a distance of 1m
kg. as shown. It is found that the two plates initially at rest, now from the wall. A mass m = 0.5 kg attached at the free end is
move with equal velocities. Find the held at rest so that the
percentage loss in the initial velocity string is horizontal between the wall
of the bullet when it is between M1 M
and the pulley and vertical beyond
and M2. Neglect any loss of material
of the plates due to the action of the the pulley. What will be the speed with
bullet. (1979) M1 M2 which the mass M will hit the wall
4. When a ball is thrown up, the magnitude of its momentum when the mass m is released ?
decreases and then increases. Does this violate the (1985 - 6 Marks) m
conservation of momentum principle? (1979) 9. A simple pendulum is suspended from a peg on a vertical
A D wall. The pendulum is pulled away from the wall to a
5.
horizontal position (see fig.) and released. The ball hits the
G 2
wall, the coefficient of restitution being .
5

B C E F l
(a) (b)
In the figures (a) and (b) AC, DG and GF are fixed inclined (1987 - 7 Marks)
planes, BC = EF = x and AB = DE = y. A small block of mass
M is released from the point A. It slides down AC and reaches
C with a speed Vc. The same block is released from rest from
the point D. It slides down DGF and reaches the point F
with speed VF. The coefficients of kinetic frictions between
the block and both the surface AC and DGF are m. (1980) What is the minimum number of collisions after which the
Calculate VC and VF. amplitude of oscillations becomes less than 60 degrees ?
EBD_7036
P-24 Topic-wise Solved Papers - PHYSICS
10. Two blocks of mass 2 kg and M are at rest on an inclined frictionless. The ball is given a gentle push (towards the
plane and are separated by a distance of 6.0 m as shown in right in the figure). The angle made by the radius vector of
Figure. The coefficient of friction between each of the blocks the ball with the upward vertical is denoted by q (shown in
and the inclined plane is 0.25. The 2 kg block is given a the figure). (2002 - 5 Marks)
velocity of 10.0 m/s up the inclined plane. It collides with M,
comes back and has a velocity of 1.0 m/s when it reaches its
initial position. The other block M after the collision moves Sphere B
0.5 m up and comes to rest. Calculate the coefficient of
q
restitution between the blocks and the mass of the block M.
d
[Take sin q » tan q = 0.05 and g = 10m / s 2 . ] O
R
(1999 - 10 Marks)
M
Sphere A

2kg (a) Express the total normal reaction force exerted by the
6.0 m
sphere on the ball as a function of angle q.
(b) Let NA and NB denote the magnitudes of the normal
q reaction forces on the ball exerted by the sphere A and
11. A spherical ball of mass m is kept at the highest point in the B, respectively. Sketch the variations of NA and NB as
space between two fixed, concentric spheres A and B (see functions of cos q in the range 0 £ q £ p by drawing
figure). The smaller sphere A has a radius R and the space two separate graphs in your answer book, taking cos
between the two spheres has a width d. The ball has a q on the horizontal axes.
diameter very slightly less than d. All surfaces are

F Match the Following

DIRECTIONS (Q. No. 1) : Each question contains statements given in two columns, which have to be p q r s t
matched. The statements in Column-I are labelled A, B, C and D, while the statements in Column-II are A p q r s t
labelled p, q, r and s. Any given statement in Column-I can have correct matching with ONE OR MORE p q r s t
B
statement(s) in Column-II. The appropriate bubbles corresponding to the answers to these questions p q
C r s t
have to be darkened as illustrated in the following example :
D p q r s t
If the correct matches are A-p, s and t; B-q and r; C-p and q; and D-s then the correct darkening of
bubbles will look like the given.
1. A particle of unit mass is moving along the x-axis under the influence of a force and its total energy is conserved. Four possible
forms of the potential energy of the particle are given in column I (a and U0 constants). Match the potential energies in column
I to the corresponding statement(s) in column II.
Column I Column II
2
U0 é æ x ö2 ù
(A) U1(x) = ê1 - ç ÷ ú (p) The force acting on the particle is zero at x = a
2 êë è a ø úû

2
U0 æxö
(B) U2(x) = ç ÷ (q) The force acting on the particle is zero at x = 0
2 èaø

U 0 æ x ö 2 exp éê- æ x ö úù
2
(C) U3(x) = ç ÷ ç ÷ (r) The force acting on the particle is zero at x = – a
2 èaø êë è a ø úû

é 3ù
U0 ê x – 1 æ x ö ú
(D) U4(x) = ç ÷ (s) The particle experiences an attractive force towards x = 0 in the region |x| < a
2 êë a 3 è a ø úû

U0
(t) The particle with total energy can oscillate about the point x = – a
4
Work, Energy and Power P-25

in the figure below, is 150 J.


G Comprehension Based Questions (Take the acceleration due to gravity, g = 10 ms–2)
PASSAGE-1 y
A small block of mass M moves on a frictionless surface of an (JEE Adv. 2013)
inclined plane, as shown in figure. The angle of the incline suddenly
changes from 60° to 30° at point B. The block is initially at rest at R P
30°
A. Assume that collisions between the block and the incline are
totally inelastic (g = 10 m/s2). (2008)
Q R

A M
v x
O
60° B
4. The magnitude of the normal reaction that acts on the block
at the point Q is
30° (a) 7.5 N (b) 8.6 N
C (c) 11.5 N (d) 22.5 N
3m 3 3m 5. The speed of the block when it reaches the point Q is
(a) 5 ms–1 (b) 10 ms–1
1. The speed of the block at point B immediately after it strikes
the second incline is – (c) 10 3 ms -1 (d) 20 ms–1

A H Assertion & Reason Type Questions


3m 1. STATEMENT–1 : A block of mass m starts moving on a
60° B
v rough horizontal surface with a velocity v. It stops due to
friction between the block and the surface after moving
3m 30°
through a certain distance. The surface is now tilted to an
30° C angle of 30° with the horizontal and the same block is made
3m 3 3m to go up on the surface with the same initial velocity v. The
decrease in the mechanical energy in the second situation
(a) 60 m/s (b) 45 m/s is smaller than that in the first situation.
STATEMENT–2 : The coefficient of friction between the
(c) 30 m/s (d) 15 m/s block and the surface decreases with the increase in the
2. The speed of the block at point C, immediately before it angle of inclination. (2007)
leaves the second incline is
(a) Statement–1 is True, Statement–2 is True; Statement–
(a) 120 m/s (b) 105 m/s 2 is a correct explanation for Statement–1
(c) 90 m/s (d) 75 m/s (b) Statement–1 is True, Statement–2 is True; Statement–
3. If collision between the block and the incline is completely 2 is NOT a correct explanation for Statement–1
elastic, then the vertical (upward) component of the velocity (c) Statement–1 is True, Statement–2 is False
of the block at point B, immediately after it strikes the second (d) Statement–1 is False, Statement–2 is True
incline is –
(a) 30 m/s (b) 15 m/s
I Integer Value Correct Type

(c) 0 (d) - 15 m/s 1. A light inextensible string that goes over a


smooth fixed pulley as shown in the figure
PASSAGE-2 connects two blocks of masses 0.36 kg and
A small block of mass 1 kg is released from rest at the top of a 0.72 kg. Taking g = 10 m/s2, find the work
rough track. The track is a circular arc of radius 40 m. The block done (in joules) by the string on the block
slides along the track without toppling and a frictional force acts of mass 0.36 kg during the first second after
on it in the direction opposite to the instantaneous velocity. The the system is released from rest. (2009)
work done in overcoming the friction up to the point Q, as shown
EBD_7036
P-26 Topic-wise Solved Papers - PHYSICS
2. Three objects A, B and C are kept in a straight line on a block in m/s is V = N/10. Then N is (2011)
frictionless horizontal surface. These have masses m, 2m
and m, respectively. The object A moves towards B with a
speed 9 m/s and makes an elastic collision with it. There
after, B makes completely inelastic collision with C. All 4. A particle of mass 0.2 kg is moving in one dimension under a
force that delivers a constant power 0.5 W to the particle. If
motions occur on the same straight line. Find the final speed
the initial speed (in ms–1) of the particle is zero, the speed (in
(in m/s) of the object C. (2009)
ms–1) after 5 s is (JEE Adv. 2013)
5. Consider an elliptical shaped rail PQ in the vertical plane
with OP = 3 m and OQ = 4 m. A block of mass 1 kg is pulled
m 2m m
along the rail from P to Q with a force of 18 N, which is
always parallel to line PQ (see the figure given). Assuming
A B C
no frictionless losses, the kinetic energy of the block when it
3. A block of mass 0.18 kg is attached to a spring of force- reaches Q is (n × 10) joules. The value of n is (take acceleration
constant 2 N/m. The coefficient of friction between the block due to gravity = 10 ms–2) (JEE Adv. 2014)
and the floor is 0.1. Initially the block is at rest and the Q
spring is un-stretched. An impulse is given to the block as
shown in the figure. The block slides a distance of 0.06 m
and comes to rest for the first time. The initial velocity of the 4m

90°
O 3m P

Section-B JEE Main / AIEEE


1. Consider the following two statements : [2003] 4. A body is moved along a straight line by a machine
A. Linear momentum of a system of particles is zero delivering a constant power. The distance moved by the
body in time ‘t’ is proportional to [2003]
B. Kinetic energy of a system of particles is zero.
(a) t 3/4 (b) t 3/2
Then
(c) t 1/4 (d) t 1/2
(a) A does not imply B and B does not imply A
5. A particle moves in a straight line with retardation
(b) A implies B but B does not imply A
proportional to its displacement. Its loss of kinetic energy
(c) A does not imply B but B implies A for any displacement x is proportional to [2004]
(d) A implies B and B implies A (a) x (b) ex
2. A wire suspended vertically from one of its ends is stretched (c) x2 (d) loge x
by attaching a weight of 200N to the lower end. The weight
6. A uniform chain of length 2 m is kept on a table such that a
stretches the wire by 1 mm. Then the elastic energy stored
length of 60 cm hangs freely from the edge of the table. The
in the wire is [2003]
total mass of the chain is 4 kg. What is the work done in
(a) 0.2 J (b) 10 J pulling the entire chain on the table ? [2004]
(c) 20 J (d) 0.1 J (a) 12 J (b) 3.6 J
3. A spring of spring constant 5 × 103 N/m is stretched initially (c) 7.2 J (d) 1200 J
by 5cm from the unstretched position. Then the work r r r r
7. A force F = (5i + 3 j + 2k ) N is applied over a particle
required to stretch it further by another 5 cm is [2003] r r r
which displaces it from its origin to the point r = (2i - j )m.
(a) 12.50 N-m (b) 18.75 N-m
The work done on the particle in joules is [2004]
(c) 25.00 N-m (d) 6.25 N-m
(a) +10 (b) +7
(c) –7 (d) +13
Work, Energy and Power P-27

8. A body of mass ‘m’, accelerates uniformly from rest to ‘v1’ The total mechanical energy of the particle is 2 J. Then, the
in time ‘t1’. The instantaneous power delivered to the body maximum speed (in m/s) is [2006]
as a function of time ‘t’ is [2004]
3 1
(a) (b) 2 (c) (d) 2
mv1t 2 mv12t mv1t mv12t 2 2
(a) (b) 2 (c) t1 (d)
t1 t1 t1 15. A 2 kg block slides on a horizontal floor with a speed of 4m/s.
It strikes a uncompressed spring, and compresses it till the
9. A Particle is acted upon by a force of constant magnitude block is motionless. The kinetic friction force is 15N and
which is always perpendicular to the velocity of the particle, spring constant is 10,000 N/m. The spring compresses by
the motion of the particles takes place in a plane. It follows (a) 8.5 cm (b) 5.5 cm [2007]
that [2004]
(c) 2.5 cm (d) 11.0 cm
(a) its kinetic energy is constant 16. An athlete in the olympic games covers a distance of 100 m
(b) its acceleration is constant in 10 s. His kinetic energy can be estimated to be in the
(c) its velocity is constant range [2008]
(d) it moves in a straight line (a) 200 J - 500 J (b) 2 × 105 J - 3 × 105 J
10. The block of mass M moving on the frictionless horizontal (c) 20, 000 J - 50,000 J (d) 2,000 J - 5, 000 J
surface collides with the spring of spring constant k and 17. A block of mass 0.50 kg is moving with a speed of 2.00 ms–1
compresses it by length L. The maximum momentum of on a smooth surface. It strikes another mass of 1.00 kg and
the block after collision is [2005] then they move together as a single body. The energy loss
during the collision is [2008]
M (a) 0.16 J (b) 1.00 J
(c) 0.67 J (d) 0.34 J
18. The potential energy function for the force between two
kL2 ML2 atoms in a diatomic molecule is approximately given by
(a) (b) Mk L (c) (d) zero
2M k
a b
11. A spherical ball of mass 20 kg is stationary at the top of a hill U(x) = 12 - 6 , where a and b are constants and x is the
x x
of height 100 m. It rolls down a smooth surface to the ground, distance between the atoms. If the dissociation energy of
then climbs up another hill of height 30 m and finally rolls
down to a horizontal base at a height of 20 m above the the molecule is D = éëU ( x = ¥) - U at equilibrium ùû , D is
ground. The velocity attained by the ball is [2005]
[2010]
(a) 20 m/s (b) 40 m/s
b2 b2 b2 b2
(c) 10 30 m/s (d) 10 m/s (a) (b) (c) (d)
2a 12a 4a 6a
12. A body of mass m is accelerated uniformly from rest to a 19. This question has Statement 1 and Statement 2. Of the four
speed v in a time T. The instantaneous power delivered to choices given after the Statements, choose the one that
the body as a function of time is given by [2005] best describes the two Statements.
If two springs S1 and S2 of force constants k1 and k2 ,
mv 2 2 mv 2 1 mv 2 2 1 mv 2
(a) .t (b) .t (c) .t (d) .t respectively, are stretched by the same force, it is found
T2 T2 2 T2 2 T2 that more work is done on spring S1 than on spring S2.
13. A particle of mass 100g is thrown vertically upwards with a STATEMENT 1 : If stretched by the same amount work
speed of 5 m/s. The work done by the force of gravity done on S1, Work done on S1 is more than S2
during the time the particle goes up is [2006] STATEMENT 2 : k1 < k2 [2012]
(a) –0.5 J (b) –1.25 J (a) Statement 1 is false, Statement 2 is true.
(c) 1.25 J (d) 0.5 J (b) Statement 1 is true, Statement 2 is false.
14. The potential energy of a 1 kg particle free to move along (c) Statement 1 is true, Statement 2 is true, Statement 2 is
the correct explanation for Statement 1
æ x 4 x2 ö (d) Statement 1 is true, Statement 2 is true, Statement 2 is
the x-axis is given by V ( x) = ç - ÷ J.
è 4 2ø not the correct explanation for Statement 1
EBD_7036
P-28 Topic-wise Solved Papers - PHYSICS
20. When a rubber-band is stretched by a distance x, it exerts 21. A person trying to lose weight by burning fat lifts a mass of
restoring force of magnitude F = ax + bx 2 where a and b are 10 kg upto a height of 1 m 1000 times. Assume that the
constants. The work done in stretching the unstretched potential energy lost each time he lowers the mass is
rubber-band by L is: [JEE Main 2014] dissipated. How much fat will he use up considering the
work done only when the weight is lifted up? Fat supplies
(a) aL2 + bL3 (b)
1
2
(aL2 + bL3 ) 3.8 × 107 J of energy per kg which is converted to mechanical
energy with a 20% efficiency rate. Take g = 9.8 ms–2 :
[JEE Main 2016]
2
aL 3
bL 1 æ aL2 bL3 ö
(c) + (d) 2 çç 2 + 3 ÷÷ (a) 9.89 × 10–3 kg (b) 12.89 × 10–3 kg
2 3 è ø (c) 2.45 × 10–3 kg (d) 6.45 × 10–3 kg
CHAPTER

5 Momentum and Impulse

Section-A JEE Advanced/IIT-JEE


A Fill in the Blanks horizontal direction, hits the centre of the ball. After the
collision, the ball and bullet travel independently. The ball
1. A particle of mass 4 m which is at rest explodes into three hits the ground at a distance of 20 m and the bullet at a
fragments. Two of the fragments each of mass m are found distance of 100 m from the foot of the post. The velocity V
to move with a speed v each in mutually perpendicular of the bullet is (2011)
directions. The total energy released in the process of
explosion is ............ (1987 - 2 Marks)
2. The magnitude of the force (in newtons) acting on a body
varies with time t (in micro seconds) as shown in the fig AB,
BC and CD are straight line segments. The magnitude of
the total impulse of the force on the body from t = 4 ms to
t = 16ms is ...............Ns. (1994 - 2 Marks) (a) 250 m/s (b) 250 2 m/s
C (c) 400 m/s (d) 500 m/s
800 4. A particle of mass m is projected from the ground with an
initial speed u0 at an angle a with the horizontal. At the
600 highest point of its trajectory, it makes a completely inelastic
Force (N)

collision with another identical particle, which was thrown


400 vertically upward from the ground with the same initial speed
A B u0. The angle that the composite system makes with the
200 horizontal immediately after the collision is
E F D (JEE Adv. 2013)
0 2 4 6 8 10 12 14 16 p p p p
(a) (b) + a (c) -a (d)
Time (m s) 4 4 2 2

C MCQs with One Correct Answer D MCQs with One or More than One Correct
1. Two particles of masses m1 and m2 in projectile motion have 1. A ball hits the floor and rebounds after an inelastic collision.
r r In this case (1986 - 2 Marks)
velocities v1 and v2 respectively at time t = 0. They collide
r r (a) the momentum of the ball just after the collision is the
at time to. Their velocities become v1 ' and v2 ' at time 2to same as that just before the collision.
while still moving in air. The value of (b) the mechanical energy of the ball remains the same in
r r r r the collision
( m1v1 '+ m2v2 ') - ( m1v1 + m2v2 ) is (2001S)
(c) the total momentum of the ball and the earth is
(a) zero (b) ( m1 + m2 ) gto conserved
1 (d) the total energy of the ball and the earth is conserved
(c) ( m1 + m2 ) gto (d) 2(m1 + m2)gto 2. A shell is fired from a cannon with a velocity v (m/sec.) at an
2
angle q with the horizontal direction. At the highest point
2. Two blocks of masses 10 kg and 4 kg are connected by a
in its path it explodes into two pieces of equal mass. One of
spring of negligible mass and placed on a frictionless
the pieces retraces its path to the cannon and the speed (in
horizontal surface. An impulse gives a velocity of 14 m/s to
the heavier block in the direction of the lighter block. The m/sec.) of the other piece immediately after the explosion is
velocity of the centre of mass is (2002S) (1986 - 2 Marks)
(a) 30 m/s (b) 20 m/s (a) 3v cos q (b) 2v cos q
(c) 10 m/s (d) 5 m/s 3 3
3. A ball of mass 0.2 kg rests on a vertical post of height 5 m. A (c) v cos q (d) v cos q
2 2
bullet of mass 0.01 kg, traveling with a velocity V m/s in a
EBD_7036
P-30 Topic-wise Solved Papers - PHYSICS
3. Two blocks A and B, each of mass m, are connected by a
massless spring of natural length L and spring constant K. E Subjective Problems
The blocks are initially resting on a smooth horizontal floor
with the spring at its natural length, as shown in fig.. A third 1. A body of mass m moving with velocity V in the X-direction
identical block C, also of mass m, moves on the floor with a collides with another body of mass M moving in Y-direction
speed v along the line joining A and B, and collides elastically with velocity v. They coalesce into one body during collision.
with A. Then (1993-2 Marks) Calculate : (1978)
v (i) the direction and magnitude of the momentum of the
L final body.
C A B (ii) the fraction of initial kinetic energy transformed into
heat during the collision in terms of the two masses.
(a) the kinetic energy of the A-B system, at maximum 2. Three particles A, B and C of equal mass move with equal
compression of the spring, is zero. speed V along the medians of an equilateral triangle as shown
(b) the kinetic energy of the A-B system, at maximum
in figure. They collide at the centroid G of the triangle. After
compression of the spring, is mv2/4.
the collision, A comes to rest, B retraces its path with the
(c) the maximum compression of the spring is v (m / K ) speed V. What is the velocity of C ? (1982 - 2 Marks)
A
(d) the maximum compression of the spring is v (m / 2K )
r r r r
4. The balls, having linear momenta p1 = pi and p2 = - pi ,
undergo a collision in free space. There is no external force
r r
acting on the balls. Let p1¢ and p¢2 be their final momenta.
The following option (s) is (are) NOT ALLOWED for any G
non-zero value of p, a1, a2, b1, b2, c1 and c2. (2008)
r ˆ ˆ ˆ r ˆ
(a) pr1¢ = a1i + b1 j + c1k (b) pr1¢ = c1k B C
ˆ ˆ
p¢2 = a 2 i + b 2 j p¢2 = c 2 kˆ 3. Two bodies A and B of masses m and 2 m respectively are
r r placed on a smooth floor. They are connected by a spring. A
(c) p1¢ = a1ˆi + b1ˆj + c1kˆ (d) p1¢ = a1ˆi + b1ˆj third body C of mass m moves with velocity v0 along the
r r
p¢2 = a 2 iˆ + b 2 ˆj - c1kˆ p¢2 = a 2ˆi + b1ˆj line joining A and B and collides elastically with A as shown
5. A point mass of 1 kg collides elastically with a stationary in Fig.
point mass of 5 kg. After their collision, the 1 kg mass reverses C A B
its direction and moves with a speed of 2 ms–1. Which of the
v0
following statement(s) is (are) correct for the system of these
two masses? (2010)
(a) Total momentum of the system is 3 kg ms–1 At a certain instant of time t0 after collision, it is found that
(b) Momentum of 5 kg mass after collision is 4 kg ms–1 the instantaneous velocities of A and B are the same. Further
(c) Kinetic energy of the centre of mass is 0.75 J at this instant the compression of the spring is found to be
(d) Total kinetic energy of the system is 4J x0. Determine (i) the common velocity of A and B at time t0;
6. A particle of mass m is attached to one end of a mass-less and (ii) the spring constant. (1984- 6 Marks)
spring of force constant k, lying on a frictionless horizontal 4. A ball of mass 100 gm is projected vertically upwards from
plane. The other end of the spring is fixed. The particle
the ground with a velocity of 49 m/sec. At the same time
starts moving horizontally from its equilibrium position at
another identical ball is dropped from a height of 98 m to
time t = 0 with an initial velocity u0. When the speed of the
fall freely along the same path as that followed by the first
particle is 0.5 u0, it collides elastically with a rigid wall. After
ball. After some time the two balls collide and stick together
this collision (JEE Adv. 2013)
and finally fall to the ground. Find the time of flight of the
(a) The speed of the particle when it returns to its
masses. (1985 - 8 Marks)
equilibrium position is u0
(b) The time at which the particle passes through the 5. A bullet of mass M is fired with a velocity 50 m/s at an angle
with the horizontal. At the highest point of its trajectory, it
m collides head-on with a bob of mass 3M suspended by a
equilibrium position for the first time is t = p
k massless string of length 10/3 metres and gets embedded in
(c) The time at which the maximum compression of the the bob. After the collision, the string moves through an
4p m angle of 120°. Find
spring occurs is t = (i) the angle q ;
3 k
(d) The time at which the particle passes through the (ii) the vertical and horizontal coordinates of the initial
position of the bob with respect to the point of firing of
5p m
equilibrium position for the second time is t = the bullet. Take g =10 m /s2
3 k
Momentum and Impulse P-31

6. A block ‘A’ of mass 2m is placed on another block ‘B’ of


mass 4m which in turn is placed on a fixed table. The two
blocks have a same length 4d and they are placed as shown C
P
in fig. The coefficient of friction (both static and kinetic)
between the block ‘B’ and table is m . There is no friction A
B
between the two blocks. A small object of mass m moving
horizontally along a line passing through the centre of mass At to, the second cannon ball is fired. Assume that the resistive
(cm.) of the block B and perpendicular to its face with a force between the rails and the carriage is constant and ignore
speed v collides elastically with the block B at a height d the vertical motion of the carriage throughout. If the second
above the table. (1991 - 4 + 4 Marks) ball also hits and sticks to the carriage, what will be the
horizontal velocity of the carriage just after the second impact?
9. A particle of mass m, moving in a circular path of radius R
A 2m with a constant speed v2 is located at point (2R, 0) at time
t = 0 and a man starts moving with a velocity v1 along the
+ve y-axis from origin at time t = 0. Calculate the linear
mm momentum of the particle w.r.t. the man as a function of time.
V B 4m 2d (2003 - 2 Marks)
dd y
v2
P
4d v1
(a) What is the minimum value of v (call it v0) required to
make the block A topple ? R x
(b) If v = 2v0, find the distance (from the point P in the (0, 0)
m
figure) at which the mass m falls on the table after
collision. (Ignore the role of friction during the
collision).
7. A cart is moving along + x direction with a velocity of 4 m/s.
A person on the cart throws a stone with a velocity of 6 m/s
H Assertion & Reason Type Questions
relative to himself. In the frame of reference of the cart the 1. STATEMENT–1 : In an elastic collision between two bod-
stone is thrown in y-z plane making an angle of 30° with ies, the relative speed of the bodies after collision is equal
vertical z-axis. At the highest point of its trajectory, the stone to the relative speed before the collision. (2007)
STATEMENT–2 : In an elastic collision, the linear
hits an object of equal mass hung vertically from the branch momentum of the system is conserved.
of a tree by means of a string of length L. A completely (a) Statement–1 is True, Statement–2 is True; State-
inelastic collision occurs, in which the stone gets embedded ment–2 is a correct explanation for Statement–1.
in the object. Determine : (1997 - 5 Marks) (b) Statement–1 is True, Statement–2 is True; State-
ment–2 is NOT a correct explanation for Statement–1
(i) The speed of the combined mass immediately after the (c) Statement–1 is True, Statement–2 is False.
collision with respect to an observer on the ground, (d) Statement–1 is False, Statement–2 is True.

(ii) The length L of the string such that the tension in


the string becomes zero when the string becomes
I Integer Value Correct Type
horizontal during the subsequent motion of the 1. A bob of mass m, suspended by a string of length l1, is given
combined mass. a minimum velocity required to complete a full circle in the
8. A car P is moving with a uniform speed of 5Ö3 m/s towards vertical plane. At the highest point, it collides elastically
a carriage of mass 9 kg at rest kept on the rails at a point B with another bob of mass m suspended by a string of length
as shown in figure. The height AC is 120 m. Cannon l2, which is initially at rest. Both the strings are mass-less
balls of 1 kg are fired from the car with an initial velocity and inextensible. If the second bob, after collision acquires
100 m/s at an angle 30o with the horizontal. The first cannon the minimum speed required to complete a full circle in the
ball hits the stationary carriage after a time to and sticks to l1
it. Determine to. (2001 - 10 Marks) vertical plane, the ratio is (JEE Adv. 2013)
l2
EBD_7036
P-32 Topic-wise Solved Papers - PHYSICS

Section-B JEE Main / AIEEE


1. A machine gun fires a bullet of mass 40 g with a velocity
1200 ms–1. The man holding it can exert a maximum force of
144 N on the gun. How many bullets can he fire per second 2
at the most? [2004]
(a) Two (b) Four x (m)
(c) One (d) Three
2. A mass ‘m’ moves with a velocity ‘v’ and collides inelastically
0 2 4 6 8 10 12 14 16
with another identical mass . After collision the lst mass
v t(s)
moves with velocity in a direction perpendicular to the
3 (a) 0.4 Ns (b) 0.8 Ns
initial direction of motion. Find the speed of the 2 nd mass
(c) 1.6 Ns (d) 0.2 Ns
after collision. [2005]
6. This question has statement I and statement II. Of the four
choices given after the statements, choose the one that
m m v
3 best describes the two statements. [JEE Main 2013]
A before Aafter
collision collision Statement - I: Apoint particle of mass m moving with speed
u collides with stationary point particle of mass M. If the
(a) 3v (b) v
æ 1 2ö
v 2 maximum energy loss possible is given as f çè mv ÷ø then
(c) (d) v 2
3 3
3. A bomb of mass 16kg at rest explodes into two pieces of æ m ö
f= ç ÷.
masses 4 kg and 12 kg. The velolcity of the 12 kg mass is èM + mø
4 ms–1. The kinetic energy of the other mass is Statement - II: Maximum energy loss occurs when the
(a) 144 J (b) 288 J [2006] particles get stuck together as a result of the collision.
(c) 192 J (d) 96 J
(a) Statement - I is true, Statment - II is true, Statement - II
4. Statement -1: Two particles moving in the same direction
is the correct explanation of Statement - I.
do not lose all their energy in a completely inelastic collision.
Statement -2 : Principle of conservation of momentum holds (b) Statement-I is true, Statment - II is true, Statement - II is
true for all kinds of collisions. [2010] not the correct explanation of Statement - II.
(a) Statement -1 is true, Statement -2 is true ; Statement -2 (c) Statement - I is true, Statment - II is false.
is the correct explanation of Statement -1.
(d) Statement - I is false, Statment - II is true.
(b) Statement -1 is true, Statement -2 is true; Statement -2
7. A particle of mass m moving in the x direction with speed 2v
is not the correct explanation of Statement -1
is hit by another particle of mass 2m moving in the y direction
(c) Statement -1 is false, Statement -2 is true.
with speed v. If the collision is perfectly inelastic, the
(d) Statement -1 is true, Statement -2 is false. percentage loss in the energy during the collision is close to :
5. The figure shows the position–time (x – t) graph of one- [JEE Main 2015]
dimensional motion of a body of mass 0.4 kg. The magnitude (a) 56% (b) 62%
of each impulse is [2010] (c) 44% (d) 50%
CHAPTER

6 Rotational Motion
Section-A JEE Advanced/ IIT-JEE
6. A symmetric lamina of A B
A Fill in the Blanks mass M consists of a
square shape with a
1. A uniform cube of side a and mass m rests on a rough semicircular section over
horizontal table. A horizontal force F is applied normal to of the edge of the square 2a
one of the faces at a point that is directly above the centre as shown in Fig. P-10. The
side of the square is 2a. O
of the face, at a height 3a/4 above the base. The minimum
value of F for which the cube begins to tip about the edge is The moment of inertia of
.... (Assume that the cube does not slide). (1984 - 2 Marks) the lamina about an axis
through its centre of mass
2. A smooth uniform rod of length L and mass M has two and perpendicular to the
identical beads of negligible size, each of mass m, which can plane is 1.6 Ma2. The moment of inertia of the lamina about
slide freely along the rod. Initially the two beads are at the the tangent AB in the plane of the lamina is....
centre of the rod and the system is rotating with an angular (1997 - 2 Marks)
velocity w0 about an axis perpenducular to the rod and
passing through the midpoint of the rod (see figure). There
B True / False
are no external forces. When the beads reach the ends of 1. A triangular plate of uniform
the rod, the angular velocity of the system is ......... thickness and density is made to
(1988 - 2 Marks) rotate about an axis perpendicular
to the plane of the paper and (a)
passing through A, (b) passing A B
C
through B, by the application of
the same force, F, at C (midpoint F
L L of AB) as shown in the figure. The angular acceleration in
2 2
w0 both the cases will be the same. (1985 - 3 Marks)
2. A thin uniform circular disc of mass M and radius R is rotating
in a horizontal plane about an axis passing through its centre
3. A cylinder of mass M and radius R is resting on a horizontal and perpendicular to its plane with an angular velocity w .
platform (which is parallel to the x–y plane) with its axis Another disc of the same dimensions but of mass M/4 is
fixed along the y-axis and free to rotate about its axis. The placed gently on the first disc coaxially. The angular velocity
platform is given a motion in the x-direction given by x = A of the system now is 2 w / 5 . (1986 - 3 Marks)
cos ( w t). There is no slipping between the cylinder and 3. A ring of mass 0.3 kg and radius 0.1 m and a solid cylinder
platform. The maximum torque acting on the cylinder during of mass 0.4 kg and of the same radius are given the same
its motion is .................. (1988 - 2 Marks) kinetic energy and released simultaneously on a flat
4. A stone of mass m, tied to the end of a string, is whirled horizontal surface such that they begin to roll as soon as
around in a horizontal circle. (Neglect the force due to released towards a wall which is at the same distance from
gravity). The length of the string is reduced gradually the ring and the cylinder. The rolling friction in both cases is
keeping the angular momentum of the stone about the centre negligible. The cylinder will reach the wall first.
of the circle constant. Then, the tension in the string is (1989 - 2 Marks)
given by T = Arn where A is a constant, r is the instantaneous 4. Two particles of mass 1 kg and 3 kg move towards each
radius of the circle and n = ..... (1993 - 1 Mark) other under their mutual force of attraction. No other force
5. A rod of weight w is supported by two parallel knife edges acts on them. When the relative velocity of approach of the
A and B and is in equilibrium in a horizontal position. The two particles is 2 m/s, their centre of mass has a velocity of
knives are at a distance d from each other. The centre of 0.5 m/s. When the relative velocity of approach becomes
mass of the rod is at distance x from A . The normal reaction 3 m/s, the velocity of the centre of mass is 0.75 m/s.
on A is... and on B is..... (1997 - 2 Marks) (1989 - 2 Marks)
EBD_7036
P-34 Topic-wise Solved Papers - PHYSICS
7. A cubical block of side L rests on a rough horizontal surface
C MCQs with One Correct Answer with coefficient of friction m . A horizontal force F is applied
1. A thin circular ring of mass ‘M and radius r is rotating about on the block as shown. If the coefficient of friction is
its axis with a constant angular velocity w, Two objects, sufficiently high so that the block does not slide before
each of mass m,, are attached gently to the opposite ends of toppling, the minimum force required to topple the block is
a diameter of the ring. The wheel now rotates with an angular (2000S)
F P
velocity (1983 - 1 Mark) (a) infinitesimal
wM w ( M – 2 m) (b) mg/4 M
(a) (b) L
( M + m) (M + 2m) (c) mg/2
wM w ( M + 2 m) (d) mg(1 - m)
(c) (d) 8. A thin wire of length L and X X'
( M + 2 m) M
2. Two point masses of 0.3 kg and 0.7 kg are fixed at the ends uniform linear mass density 90º
of a rod of length 1.4 m and of negligible mass. The rod is set r is bent into a circular
rotating about an axis perpendicular to its length with a loop with centre at O as
uniform angular speed. The point on the rod through which shown. The moment of
the axis should pass in order that the work required for inertia of the loop about the
rotation of the rod is minimum, is located at a distance of axis XX' is (2000S)
(1995S) rL3 rL3 5rL3 3rL3
(a) 0.42 m from mass of 0.3 kg (a) (b) (c) (d)
(b) 0.70 m from mass of 0.7 kg 8p 2 16p 2 16p 2 8p2
9. An equilateral triangle ABC formed from a uniform wire has
(c) 0.98 m from mass of 0.3 kg
two small identical beads initially located at A. The triangle
(d) 0.98 m from mass of 0.7 kg
is set rotating about the vertical axis AO. Then the beads are
3. A smooth sphere A is moving on a frictionless horizontal released from rest simultaneously and allowed to slide down,
plane with angular speed w and centre of mass velocity u. It one along AB and the other along AC as shown. Neglecting
collides elastically and head on with an identical sphere B at frictional effects, the quantities that are conserved as the
rest. Neglect friction everywhere. After the collision, their beads slide down, are (2000S)
angular speeds are wA and wB, respectively. Then
(1999S - 2 Marks) (a) angular velocity and total energy
(a) wA< wB (b) wA= wB (kinetic and potential) A
(b) Total angular momentum and total
(c) wA= w (d) wB= w
energy g
4. A disc of mass M and radius R is rolling with angular speed
(c) angular velocity and moment of
w on a horizontal plane as shown in Figure. The magnitude
inertia about the axis of rotation
of angular momentum of the disc about the origin O is
(d) total angular momentum and B C
(1999S - 2 Marks) O
Y moment of inertia about the axis of
(a) (1/2) MR w 2
rotation
(b) MR2w w 10. One quarter sector is cut from a
(c) (3/2) MR2w M uniform circular disc of radius R. This
(d) 2 MR2w sector has mass M. It is made to
X
O rotate about a line perpendicular to
5. A cubical block of side a is moving with velocity V on a its plane and passing through the
horizontal smooth plane as shown in Figure. It hits a ridge center of the original disc. Its
at point O. The angular speed of the block after it hits O is moment of inertia about the axis of
(1999S - 2 Marks) rotation is (2001S)
a
(a) 3V/(4a)
(b) 3V/(2a) 1 1
M V
(a) MR 2 (b) MR 2
2 4
(c) 3V / ( 2a )
1
(d) zero O (c) MR 2 (d) 2 MR 2
a 8
6. A long horizontal rod has a bead 11. A cylinder rolls up an inclined plane, reaches some height,
A B
which can slide along its length and then rolls down (without slipping throughout these
and initially placed at a L motions). The directions of the frictional force acting on the
distance L from one end A of the rod. The rod is set in cylinder are (2002S)
angular motion about A with constant angular acceleration (a) up the incline while ascending and down the incline
a . If the coefficient of friction between the rod and the descending
bead is m , and gravity is neglected, then the time after which (b) up the incline while ascending as well as descending
the bead starts slipping is (2000S) (c) down the incline while ascending and up the incline
1 while descending
(a) m / a (b) m / a (c) ma
(d) infinitesimal (d) down the incline while ascending as well as descending.
Rotational Motion P-35

12. A circular platform is free to rotate in a horizontal plane 18. From a circular disc of radius R and mass 9M, a small disc of
about a vertical axis passing through its centre. A tortoise is radius R/3 is removed from the disc. The m oment of inertia
sitting at the edge of the platform. Now, the platform is given of the remaining disc about an axis perpendicular to the
an angular velocity w0. When the tortoise move along a plane of the disc and passing through O is (2005S)
chord of the platform with a constant velocity (with respect 2
(a) 4MR
to the platform), the angular velocity of the platform w(t) will R
vary with time t as (2002S) 40
(b) MR 2 3 2R/3
w(t ) 9
w(t ) (c) 10 MR 2 R

w0 37
(a) w0 (b) (d) MR 2
9
t
19. A particle is confined to rotate in a circular path decreasing
linear speed, then which of the following is correct? (2005S)
r
t
w( t ) w( t ) (a) L (angular momentum) is conserved about the centre
r
(b) only direction of angular momentum L is conserved
w0 w0
(c) (d) (c) It spirals towards the centre
(d) its acceleration is towards the centre.
t t 20. A solid sphere of mass M and radius R having moment of
13. Consider a body, shown in figure, consisting of two identical inertia I about its diameter is recast into a solid disc of radius
balls, each of mass M connected by a light rigid rod. If an r and thickness t. The moment of inertia of the disc about an
impulse J=MV is imparted to the body at one of its ends, axis passing the edge and perpendicular to the plane remains
what would be its angular velocity? (2003S) I. Then R and r are related as (2006 - 3M, –1)
(a) V/L L
2 2
M M (a) r= R (b) r= R
(b) 2V/L 15 15
(c) V/3L 2 2
J = MV (c) r= R (d) r= R
(d) V/4L 15 15
14. A particle undergoes uniform circular motion. About which 21. A small object of uniform
point on the plane of the circle, will the angular momentum density rolls up a curved
of the particle remain conserved? (2003S) surface with an initial
(a) centre of the circle velocity v. It reaches up to
(b) on the circumference of the circle.
3v 2 v
(c) inside the circle a maximum height of
(d) outside the circle. 4g
15. A horizontal circular plate is rotating about a vertical axis with respect to the initial position. The object is (2007)
passing through its centre with an angular velocity wo. A (a) ring (b) solid sphere
man sitting at the centre having two blocks in his hands
(c) hollow sphere (d) disc
stretches out his hands so that the moment of inertia of the
system doubles. If the kinetic energy of the system is K 22. A bob of mass M is suspended by a massless string of
initially, its final kinetic energy will be (2004S) length L. The horizontal velocity v at position A is just
(a) 2 K (b) K/ 2 (c) K (d) K/ 4 sufficient to make it reach the point B. The angle q at which
16. A disc is rolling without slipping with angular velocity w. P the speed of the bob is half of that at A, satisfies – (2008)
and Q are two points equidistant from the centre C. The p B
(a) q=
order of magnitude of velocity is (2004S) 4
(a) vQ > vC > vP p p
(b) vP > vC > vQ (b) <q<
C P 4 2
(c) vP = vC , vQ = vC/2 Q
p 3p
(d) vP < vC > vQ (c) <q<
2 4 L
17. A block of mass m is at rest under the action of force F
against a wall as shown in figure. Which of the following 3p
(d) <q<p A
v
statement is incorrect? a (2005S) 4
(a) f = mg [f friction force] 23. Look at the drawing given in the figure which has been
(b) F = N [N normal force]
a drawn with ink of uniform line-thickness. The mass of ink
F used to draw each of the two inner circles, and each of the
(c) F will not produce torque two line segments is m. The mass of the ink used to draw the
(d) N will not produce torque outer circle is 6 m.
EBD_7036
P-36 Topic-wise Solved Papers - PHYSICS
The coordinates of the centres of the different parts are: The reaction of the wall on the stick is equal in magnitude to
outer circle (0, 0), left inner circle (– a, a), right inner circle the reaction of the floor on the stick. The ratio h/l and the
(a, a), vertical line (0, 0) and horizontal line (0, – a). The frictional force f at the bottom of the stick are
y-coordinate of the centre of mass of the ink in this drawing (g = 10 m s–2) (JEE Adv. 2016)
is (2009)
h 3 16 3 h 3 16 3
a y (a) = ,f = N (b) = ,f = N
(a) l 16 3 l 16 3
10
a h 3 3 8 3 h 3 3 16 3
(b) (c) = ,f = N (d) = ,f = N
8 l 16 3 l 16 3
a
(c)
12
x
D MCQ with One or More than One Correct
a 1. Two particles A and B initially at rest, move towards each
(d)
3 other under mutual force of attraction. At the instant when
24. A small mass m is attached to a z the speed of A is V and the speed of B is 2V, the speed of the
massless string whose other end is centre of mass of the system is (1982 - 3 Marks)
fixed at P as shown in the figure. The P (a) 3 V (b) V
mass is undergoing circular motion in (c) 1.5 V (d) zero
the x-y plane with centre at O and 2. A mass M moving with a constant velocity parallel to the
constant angular speed w. If the m X-axis. Its angular momentum with respect to the origin
O
angular momentum of the system, (1985 - 2 Marks)
calculated about O and P are denoted
r r w (a) is zero (b) remains constant
by LO and LP respectively, then (c) goes on increasing (d) goes on decreasing
r r 3. When a bicycle is in motion, the force of friction exerted by
(a) LO and LP do not vary with time
the ground on the two wheels is such that it acts
r r
(b) LO varies with time while LP remains constant (1990 - 2 Marks)
r r (a) in the backward direction on the front wheel and in the
(c) LO remains constant while LP varies with time forward direction on the rear wheel.
r r (b) in the forward direction on the front wheel and in the
(d) LO and LP both vary with time
z backward direction on the rear wheel.
25. A thin uniform rod, pivoted
(c) in the backward direction on both the front and the
at O, is rotating in the
horizontal plane with rear wheels.
w (d) in the forward direction on both the front the rear wheels.
constant angular speed w,
as shown in the figure. At 4. A particle of mass m is projected with a velocity v making an
time t = 0, a small insect v® angle of 45° with the horizontal. The magnitude of the angular
starts from O and moves momentum of the projectile about the point of projection
O
with constant speed v, with when the particle is at its maximum height h is
respect to the rod towards (1990 - 2 Marks)
the other end. It reaches the end of the rod at t = T and mv 3
stops. The angular speed of the system remains w (a) zero (b)
throughout. The magnitude of the torque ( rt ) about O, as a
4 2g
3
mv
function of time is best represented by which plot? (2012) (c) (d) m 2 gh3
2g
r r 5. A uniform bar of length 6a and mass 8m lies on a smooth
|t| |t | horizontal table. Two point masses m and 2m moving in the
(a) (b) same horizontal plane with speed 2v and v, respectively,
O t O t
T T strike the bar [as shown in the fig.] and stick to the bar after
collision. Denoting angular velocity (about the centre of
r r mass), total energy and centre of mass velocity by w, E and
|t| |t | vc respectively, we have after collision (1991 - 2 Mark)
(c) (d) (a) vc = 0
O t O t 2m
T T
3v
26. A uniform wooden stick of mass 1.6 kg and length l rests in (b) w= 3a
5a v C
an inclined manner on a smooth, vertical wall of height h(<
l) such that a small portion of the stick extends beyond the v
(c) w=
wall. The reaction force of the wall on the stick is 5a 2a 2a
perpendicular to the stick. The stick makes an angle of 30° 3mv 2 2v
with the wall and the bottom of the stick is on a rough floor. (d) E= mm
5
Rotational Motion P-37

6. The moment of inertia of a thin square plate ABCD, Fig., of (c) angular momentum of the system does not change in
uniform thickness about an axis passing through the centre time
O and perpendicular to the plane of the plate is (d) potential energy of the system does not change in time
(1992 - 2 Marks) 13. A sphere is rolling without slipping on a fixed horizontal
(a) I1 + I 2 4 1 plane surface. In the figure, A is the point of contact, B is the
A B centre of the sphere and C is its topmost point. Then,
(b) I3 + I 4 (2009)
O
3 r r r r C
(c) I1 + I3 (a) VC - VA = 2 (VB - VC )
(d) I1 + I 2 + I3 + I 4 D C r r r r
2
(b) VC - VB = VB - VA
where I1, I 2 , I3 and I 4 are respectively the moments of r r r r B
intertial about axis 1, 2, 3 and 4 which are in the plane of the (c) VC - VA = 2 VB - VC
plate. r r r
7. A tube of length L is filled completely with an incomressible (d) VC - VA = 4 VB A
liquid of mass M and closed at both the ends. The tube is 14. Two solid spheres A and B of equal volumes but of different
then rotated in a horizontal plane about one of its ends with densities dA and dBare connected by a string. They are fully
a uniform angular velocity w . The force exerted by the liquid immersed in a fluid of density dF. They get arranged into an
at the other end is (1992 - 2 Marks) equilibrium state as shown in the figure with a tension in the
string. The arrangement is possible only if (2011)
M w2 L
(a) (b) M w2 L (a) dA < dF
2 (b) dB > dF
A

M w2 L M w 2 L2 (c) dA > dF
(c) (d) B
4 2 (d) dA + dB = 2dF
8. A car is moving in a circular horizontal track of radius 10 m 15. A thin ring of mass 2 kg and radius 0.5 m is rolling without
with a constant speed of 10 m/s. A pendulum bob is on a horizontal plane with
suspended from the roof of the car by a light rigid rod of velocity 1 m/s. A small
length 1.00 m. The angle made by the rod with track is
(1992 - 2 Mark) ball of mass 0.1 kg,
(a) zero (b) 30° moving with velocity 20
(c) 45° (d) 60° m/s in the opposite
9. Let I be the moment of inertia of a uniform square plate direction hits the ring at
about an axis AB that passes through its centre and is parallel a height of 0.75 m and goes vertically up with velocity 10 m/
to two of its sides. CD is a line in the plane of the plate that s. Immediately after the collision (2011)
passes through the centre of the plate and makes an angle q (a) the ring has pure rotation about its stationary CM.
with AB. The moment of inertia of the plate about the axis (b) the ring comes to a complete stop.
CD is then equal to (1998S - 2 Marks) (c) friction between the ring and the ground is to the left.
(a) I (b) I sin2 q (d) there is no friction between the ring and the ground.
(c) I cos2 q (d) I cos2 (q/2) 16. The figure shows a system consisting of (i) a ring of outer
10. The torque t on a body about a given point is found to be radius 3R rolling clockwise
z
equal to A × L where A is a constant vector, and L is the without slipping on a w
angular momentum of the body about that point. From this horizontal surface with
it follows that (1998S - 2 Marks) angular speed w and (ii) an
dL inner disc of radius 2R w/ 2
rotating anti-clockwise with 3R
(a) is perpendicular to L at all instants of time.
dt angular speed w/2. The ring x
O 2R
(b) the component of L in the direction of A does not and disc are separated by
change with time. frictionless ball bearings. The
(c) the magnitude of L does not change with time. point P on the inner disc is at
(d) L does not change with time a distance R from the origin,
11. A solid cylinder is rolling down a rough inclined plane of where OP makes an angle of 30° with the horizontal. Then
inclination q. Then (2006 - 5M, –1) with respect to the horizontal surface, (2012)
(a) The friction force is dissipative
(b) The friction force is necessarily changing (a) the point O has linear velocity 3 Rw î
(c) The friction force will aid rotation but hinder translation 11 3
(d) The friction force is reduced if q is reduced (b) the point P has linear velocity Rwiˆ + Rwkˆ .
12. If the resultant of all the external forces acting on a system 4 4
of particles is zero, then from an inertial frame, one can surely 13 3
(c) the point P has linear velocity Rwiˆ - Rwkˆ
say that (2009) 4 4
(a) linear momentum of the system does not change in (d) the point P has linear velocity
time
(b) kinetic energy of the system does not change in time æ 3ö ˆ 1 ˆ
çè 3 - ÷ø Rwi + Rwk
4 4
EBD_7036
P-38 Topic-wise Solved Papers - PHYSICS
17. Two solid cylinders P and Q of same mass and same radius 2 1
start rolling down a fixed inclined plane from the same height (a)R (b) R
3 3
at the same time. Cylinder P has most of its mass 3 4
concentrated near its surface, while Q has most of its mass (c) R (d) R
5 5
concentrated near the axis. Which statement(s) is(are) 20. Two thin circular discs of mass m and 4m, having radii of a
correct? (2012) and 2a, respectively, are rigidly fixed by a massless, rigid
(a) Both cylinders P and Q reach the ground at the same rod of length l = 24a through their centres. This assembly
time.
is laid on a firm and flat surface, and set rolling without
(b) Cylinders P has larger linear acceleration than cylinder slipping on the surface so that the angular speed about the
Q. axis of the rod is w. The angular momentum of the entire
(c) Both cylinders reach the ground with same translational ur
assembly about the point ‘O’ is L (see the figure). Which
kinetic energy.
of the following statement(s) is (are) true? (JEE Adv. 2016)
(d) Cylinder Q reaches the ground with larger angular 4m
speed.
18. In the figure, a ladder of mass m is
shown leaning against a wall. It is m1
z m w
in static equilibrium making an l
angle q with the horizontal floor. 2a
The coefficient of friction between l a
the wall and the ladder is m1 and O
that between the floor and the
ladder is m2. The normal reaction of (a)
The centre of mass of the assembly rotates about the
the wall on the ladder is N1 and that m2 z-axis with an angular speed of w/5
of the floor is N2. If the ladder is about to slip, then (b) The magnitude of angular momentum of center of mass
(JEE Adv. 2014) of the assembly about the point O is 81 ma2w
mg (c) The magnitude of angular momentum of the assembly
(a) m1 = 0, m 2 ¹ 0 and N2 tanq = about its center of mass is 17 ma2w/2.
2 ur
(d) The magnitude of the z-component of L is 55 ma2w.
mg
(b) m1 ¹ 0, m2 = 0 and N1 tan q =
2 21. The position vector rr of a particle of mass m is given by the
following equation
mg
(c) m1 ¹ 0, m2 ¹ 0 and N 2 = 1 + m m r
1 2 r (t) = at 3ˆi + bt 2ˆj ,
mg where a = 10/3 ms–3, b = 5 ms–2 and m = 0.1 kg. At t = 1 s,
(d) m1 = 0, m 2 ¹ 0 and N1 tan q = which of the following statement(s) is(are) true about the
2
particle? (JEE Adv. 2016)
19. A ring of mass M and radius R is rotating with angular speed
r r
(a) The velocity v is given by v = (10iˆ + 10ˆj) ms–1
w about a fixed vertical axis passing through its centre O
r
M (b) The angular momentum L with respect to the origin is
with two point masses each of mass at rest at O. These r
8
given by L = – 5/3) k̂ N m s
masses can move radially outwards along two massless rods r r
fixed on the ring as shown in the figure. At some instant the (c) The force F is given by F = (iˆ + 2ˆj) N
r
8 (d) The torque t with respect to the origin is given by
angular speed of the system is w and one of the masses is r
9 t = – (20/3) k̂ N m
3
at a distance of R from O. At this instant the distance of E Subjective Problems
5
the other mass from O is (JEE Adv. 2015)
1. A 40 kg mass, hanging at the end of a rope of length l,
w
oscillates in a vertical plane with an angular amplitude q0.
What is the tension in the rope when it makes an engle q
with the vertical? If the breaking strength of the rope is
O 80 kg, what is the maximum amplitude with which the mass
can oscillate without the rope breaking? (1978)
Rotational Motion P-39
2. A large mass M and a small mass m hang stick to the bar on collision. Calculate the loss of the kinetic
at two ends of a string that passes over energy of the system in the above collision process.
l
a smooth tube as shown in the figure. (1989 - 8 Marks)
The mass m moves around a circular 8. A homogeneous rod AB of length L = 1.8 m and mass M is
path which lies in a horizontal plane. m pivoted at the centre O in such a way that it can rotate freely
The length of string from the mass m to in the vertical plane (Fig). The rod is initially in the horizontal
the top of the tube is l and q is the ‘angle’ position. An insect S of the same mass M falls vertically
this length makes with the vertical. What M with speed V on the point C, midway between the points O
should be the frequency of rotation of and B. Immediately after falling, the insect moves towards
mass m, so that the mass M remains stationary? (1978) the end B such that the rod rotates with a constant angular
3. A circular plate of uniform velocity w . (1992 - 8 Marks)
thickness has a diameter of 56 cm. (a) Determine the S
A circular portion of diameter 42 angular velocity w
cm is removed from one edge of in terms of V and L. V

the plate as shown in figure. (b) If the insect reaches


Find the position of the centre of the end B when the A O C B
rod has turned L L L
mass of the remaining portion. 42cm
2 4 4
(1980) 56cm through an angle of
4. A block of mass M with a 90°, determine V.
r
semicircular of radius R, R 9. A uniform thin rod of mass M and length L is standing
m
vertically along the y-axis on a smooth horizontal surface,
rests on a horizontal
B M with its lower end at the origin (0, 0). A slight disturbance at
frictionless surface. A t = 0 causes the lower end to slip on the smooth surface
uniform cylinder of radius r along the positive x-axis, and the rod starts falling.
and mass m is released from rest at the top point A (see Fig). (1993-1+5 Marks)
The cylinder slips on the semicircular frictionless track. (i) What is the path followed by the centre of mass of the
How far has the block moved when the cylinder reaches rod during its fall?
the bottom (point B) of the track ? How fast is the block (ii) Find the equation to the trajectory of a point on the rod
moving when the cylinder reaches the bottom of the track ? located at a distance r from the lower end. What is the
(1983 - 7 Marks) shape of the path of this point?
5. A particle is projected at time t= 0 from a point P on the 10. A block X of mass 0.5 kg is held by a long massless string
ground with a speed v0, at an angle of 45° to the horizontal. on a frictionless inclined plane of inclination 30° to the
Find the magnitude and direction of the angular momentum horizontal. The string is wound on a uniform solid cylindrical
drum Y of mass 2 kg and of radius 0.2 m as shown in Figure.
of the particle about P at time t = v0/g (1984- 6 Marks)
The drum is given an initial angular velocity such that the
6. A small sphere rolls block X starts moving up the plane. (1994 - 6 Marks)
down without slipping (i) Find the tension in the string Y
from the top of a track during the motion.
in a vertical plane. The (ii) At a certain instant of time the
2.4 m

track has an elevated magnitude of the angular velocity X


section and a horizontal 1.0 m
of Y is 10 rad s–1 calculate the
part, The horizontal B
distance travelled by X from that 30°
part is 1.0 metre above the ground level and the top of the instant of time until it comes to rest
track is 2.4 metres above the ground. Find the distance on 11. Two uniform thin rods A and B of length 0.6 m P
each and of masses 0.01 kg and 0.02 kg
the ground with respect to the point B (which is vertically
respectively are rigidly joined end to end. The A
below the end of the track as shown in fig.) where the sphere combination is pivoted at the lighter end, P as
lands. During its flight as a projectile, does the sphere continue shown in fig. Such that it can freely rotate
to rotate about its centre of mass? Explain. (1987 - 7 Marks) about point P in a vertical plane. A small object B
7. A thin uniform bar lies on a frictionless of mass 0.05 kg, moving horizontally, hits the
horizontal surface and is free to move in 10 m/s A lower end of the combination and sticks to it.
any way on the surface. Its mass is 0.16 What should be the velocity of the object so that the system
kg and length 3 meters. Two particles, could just be raised to the horizontal position.
6 m/s B (1994 - 6 Marks)
each of mass 0.08 kg, are moving on the
same surface and towards the bar in a 12. A rectangular rigid fixed block
direction perpendicular to the bar, one with a velocity of 10 has a long horizontal edge. A
m/s, and other with 6 m/s as shown in fig. The first particle solid homogeneous cylinder of
strikes the bar at point A and the other at point B. Points A radius R is placed horizontally R
and B are at a distance of 0.5m from the centre of the bar. at rest its length parallel to the
The particles strike the bar at the same instant of time and edge such that the axis of the
EBD_7036
P-40 Topic-wise Solved Papers - PHYSICS
cylinder and the edge of the block are in the same vertical (a) Find the force F and also the normal force N exerted by
plane as shown in the figure below. There is sufficient friction the table on the wedge during the time Dt.
present at the edge so that a very small displacement causes (b) Let h denote the perpendicular distance between the
the cylinder to roll off the edge without slipping. Determine: centre of mass of the wedge and the line of action of
(1995 - 10 Marks) F. Find the magnitude of the torque due to the normal
(a) the angle qc through which the cylinder rotates before force N about the centre of the wedge, during the
it leaves contact with the edge, interval Dt .
(b) the speed of the centre of mass of the cylinder before 16. A uniform circular disc A
leaving contact with the edge, and has radius R and mass
(c) the ratio of the translational to rotational kinetic m. A particle also of
R
energies of the cylinder when its centre of mass is in mass m, is fixed at a
horizontal line with the edge. point A on the edge of C
13. A small sphere of radius the disc as shown in R/4
R is held against the inner Figure. The disc can P Q
surface of a larger sphere rotate freely about a
of radius 6R (Fig. P-3). fixed horizontal chord
The masses of large and PQ that is at a distance R/4 from the centre C of the disc.
small spheres are 4M and The line AC is perpendicular to PQ.
M, respectively, This Initially, the disc is held vertical with the point A at its highest
arrangement is placed on position. It is then allowed to fall so that it starts rotating
a horizontal table. There is no friction between any surfaces about PQ. Find the linear speed of the particle as it reaches
of contact. The small sphere is now released. Find the its lowest position. (1998 - 8 Marks)
coordinates of the centre of the larger sphere when the smaller 17. A man pushes a cylinder of mass m1with the help of a plank
sphere reaches the other extreme position. (1996 - 3 Marks) of mass m2 as shown in Figure. There in no slipping at any
14. Two thin circular contact. The horizontal component of the force applied by
disks of mass 2 kg the man is F. (1999 - 10 Marks)
and radius 10 cm
Find
each are joined by O
a rigid massless (a) the accelerations of the F m2
rod of length 20 plank and the center of
cm. The axis of 20cm mass of the cylinder, and
(b) the magnitudes and M1
the rod is along the perpendicular to the planes of the disk
through their centres. This object is kept on a truck in such directions of frictional
a way that the axis of the object is horizontal and forces at contact points.
perpendicular to the direction of the motion of the truck. Its 18. Two heavy metallic A
friction with the floor of the truck is large enough so that the plates are joined together
object can roll on the truck without slipping. Take x axis as at 90o to each other. A
the direction of motion of the truck and z axis as the vertically laminar sheet of mass 30 Q
upwards direction. If the truck has an acceleration of 9 m/s2. kg is hinged at the line AB
Calculate: (1997 - 5 Marks) joining the two heavy P B
(i) The force of friction on each disk, metallic plates. The hinges
(ii) The magnitude and the direction of the frictional torque
are frictionless. The
acting on each disk about the centre of mass O of the
moment of inertia of the
object. Express the torque in the vector form in terms
laminar sheet about an axis
of unit vectors iˆ, ˆj and k̂ in the x, y, and z directions. parallel to AB and passing through its center of mass is 1.2 kg-
15. A wedge of mass m and triangular cross-section (AB = BC = m2. Two rubber obstacles P and Q are fixed, one on each
metallic plate at a distance 0.5 m from the line AB. This distance
CA = 2R) is moving with a constant velocity - viˆ towards a
sphere of radius R fixed on a smooth horizontal table as is chosen so that the reaction due to the hinges on the laminar
shown in Figure. The wedge makes an elastic collision with sheet is zero during the impact. (2001-10 Marks)
the fixed sphere and returns along the same path without Initially the laminar sheet hits one of the obstacles with an
any rotation. Neglect all friction and suppose that the wedge angular velocity 1 rad/s and turns back. If the impulse on
remains in contact with the sphere for a very short time. Dt, the sheet due to each obstacle is 6 N-s,
during which the sphere exerts a constant force F on the (a) Find the location of the center of mass of the laminar
wedge. (1998 - 8 Marks)
A sheet from AB.
z
(b) At what angular velocity does the laminar sheet come
y R
v back after the first impact?
(c) After how many impacts, does the laminar sheet come
to rest?
x B C
Rotational Motion P-41

19. Three particles A, B and C, each of mass m, are connected to 20. A wooden log of mass M and length L is O
each other by three massless rigid rods to form a rigid, hinged by a frictionless nail at O. A bullet
equilateral triangular body of side l . This body is placed on of mass m strikes with velocity v and sticks L
M
to it. Find angular velocity of the system
a horizontal frictioness table (x-y plane) and is hinged to it at immediately after the collision about O.
m
the point A so that it can move without friction about the v

vertical axis through A (see figure). The body is set into (2005 - 2 Marks)
21. A cylinder of mass m and radius R rolls down an inclined
rotational motion on the table about A with a constant angular
plane of inclination q. Calculate the linear acceleration of
velocity w. (2002 - 5 Marks ) the axis of cylinder. (2005 - 4 Marks)
y 22. Two identical ladders, each of P
mass M and length L are resting
on the rough horizontal surface L m
A
x as shown in the figure. A block q q
of mass m hangs from P. If the A
w B
system is in equilibrium, find the
magnitude and the direction of frictional force at A and B.
(2005 - 4 Marks)
23. A rectangular plate of b
mass M and dimension a
F × b is held in horizontal
B l C position by striking n small a
balls (each of mass m) per
(a) Find the magnitude of the horizontal force exerted by unit area per second. The
the hinge on the body. balls are striking in the v
(b) At time T, when the side BC is parallel to the x-axis, a shaded half region of the plate. The collision of the balls
force F is applied on B along BC (as shown). Obtain the with the plate is elastic. What is v? (2006 - 6M)
x-component and the y-component of the force exerted (Given n = 100, M = 3 kg, m = 0.01 kg; b = 2 m; a = 1 m;
by the hinge on the body, immediately after time T. g = 10 m/s2).

F Match the Following

DIRECTIONS (Q. No. 1) : Each question contains statements given in two columns, which have to be p q r s t
matched. The statements in Column-I are labelled A, B, C and D, while the statements in Column-II are
A p q r s t
labelled p, q, r and s. Any given statement in Column-I can have correct matching with ONE OR MORE
B p q r s t
statement(s) in Column-II. The appropriate bubbles corresponding to the answers to these questions
have to be darkened as illustrated in the following example : C p q r s t
If the correct matches are A-p, s and t; B-q and r; C-p and q; and D-s then the correct darkening of D p q r s t
bubbles will look like the given.

1. Column-II shows five systems in which two objects are labelled as X and Y. Also in each case a point P is shown. Column-I gives
some statements about X and/or Y. Match these statements to the appropriate system(s) from Column II. (2009)
Column-I Column II
(A) The force exerted by X on Y has a magnitude Mg. (p) Y
X

P
Block Y of mass M left on a fixed inclined plane X, slides on
it with a constant velocity.
(B) The gravitational potential energy of X is (q)
continuously increasing. P

Y
X
EBD_7036
P-42 Topic-wise Solved Papers - PHYSICS

Two ring magnets Y and Z, each of mass M, are kept in


frictionless vertical plastic stand so that they repel each other.
Y rests on the base X and Z hangs in air in equilibrium. P is
the topmost point of the stand on the common axis of the
two rings. The whole system is in a lift that is going up with
a constant velocity.

(C) Mechanical energy of the system X +Y is (r)


continuously decreasing.
P
Y

A pulley Y of mass m0 is fixed to a table through a clamp X. A


block of mass M hangs from a string that goes over the
pulley and is fixed at point P of the table. The whole system
is kept in a lift that is going down with a constant velocity.
(D) The torque of the weight of Y about point P is zero. (s)
Y

X
P
A sphere Y of mass M is put in a non-viscous liquid X kept in
a container at rest. The sphere is released and it moves down
in the liquid.
(t)
Y

X
P
A sphere Y of mass M is falling with its terminal velocity in a
viscous liquid X kept in a container.
2. When disc B is brought in contact with disc A, they acquire
G Comprehension Based Questions a common angular velocity in time t . The average frictional
torque on one disc by the other during this period is (2007)
PASSAGE - 1 2I w 9I w 9I w 3I w
(a) (b) (c) (d)
Two discs A and B are mounted coaxially on a vertical axle. The 3t 2t 4t 2t
discs have moments of inertia I and 2 I respectively about the 3. The loss of kinetic energy in the above process is (2007)
common axis. Disc A is imparted an initial angular velocity 2 w
I w2 2
using the entire potential energy of a spring compressed by a (a) (b) I w
distance xl. Disc B is imparted an angular velocity w by a spring 2 3
having the same spring constant and compressed by a distance
x2 .Both the discs rotate in the clockwise direction. I w2 I w2
(c) (d)
1. The ratio x1/x2 is (2007) 4 6
1 PASSAGE - 2
(a) 2 (b)
2
A uniform thin cylindrical disk of mass M and radius R is attached
1 to two identical massless springs of spring constant k which are
(c) 2 (d)
2 fixed to the wall as shown in the figure. The springs are attached
Rotational Motion P-43

to the axle of the disk symmetrically y 7. Which of the following statements about the instantaneous
on either side at a distance d from its axis (passing through the centre of mass) is correct?
centre. The axle is massless and both (a) It is vertical for both the cases (a) and (b)
the springs and the axle are in 2d
(b) It is vertical for case (a); and is at 45° to the x-z plane
horizontal plane.
The unstretched length of each and lies in the plane of the disc for case (b).
spring is L. The disk is initially at its (c) It is horizontal for case (a); and is at 45° to the x-z plane
equilibrium position with its centre of and is normal to the plane of the disc for case (b).
mass (CM) at a distance L from the d (d) It is vertical for case (a); and is 45° to the x-z plane and
d
wall. The disk rolls without slipping R is normal to the plane of the disc for case (b).
V0
r x
with velocity V = V ˆi . The
0 0
8. Which of the following statements regarding the angular
coefficient of friction is µ. (2008) speed about the instantaneous axis (passing through the
4. The net external force acting on the disk when its centre of centre of mass) is correct?
mass is at displacement x with respect to its equilibrium
(a) It is 2w for both the cases
position is
(a) – kx (b) – 2kx (c) – 2kx/3 (d) – 4kx/3 (b) It is w for case (a); and w/ 2 for case (b)
5. The centre of mass of the disk undergoes simple harmonic
motion with angular frequency w equal to – (c) It is w for case (a); and 2w for case (b)
k 2k 2k 4k (d) It is w for both the cases.
(a) (b) (c) (d)
M M 3M 3M
6. The maximum value of V0 for which the disk will roll without H Assertion & Reason Type Questions
slipping is –
1. STATEMENT-1: If there is no external torque on a body about
M M 3M 5M its center of mass, then the velocity of the center of mass
(a) mg (b) mg (c) mg (d) mg
k 2k k 2k remains constant.
PASSAGE-3 STATEMENT-2: The linear momentum of an isolated system
The general motion of a rigid body can be considered to be a remains constant. (2007)
combination of (i) a motion of its centre of mass about an axis, and (a) Statement-1 is True, Statement-2 is True; Statement-2
(ii) its motion about an instantaneous axis passing through the is a correct explanation for Statement-1
centre of mass. (b) Statement-1 is True, Statement-2 is True; Statement-2
These axes need not be stationary. Consider, for example, a thin is NOT a correct explanation for Statement-1
uniform disc welded (rigidly fixed) horizontally at its rim to a (c) Statement-1 is True, Statement-2 is False
massless, stick, as shown in the figure. When the disc-stick system (d) Statement-1 is False, Statement-2 is True
is rotated about the origin on a horizontal frictionless plane with 2. STATEMENT-1 : Two cylinders, one hollow (metal) and the
angular speed w, the motion at any instant can be taken as a other solid (wood) with the same mass and identical
combination of (i) a rotation of the centre of mass of the disc dimensions are simultaneously allowed to roll without
about the z-axis and (ii) a rotation of the disc through an
slipping down an inclined plane from the same height. The
instantaneous vertical axis passing through its centre of mass (as
is seen from the changed orientation of points P and Q). Both hollow cylinder will reach the bottom of the inclined plane
these motions have the same angular speed w in this case (2012) first.
STATEMENT-2 : By the principle of conservation of
energy, the total kinetic energies of both the cylinders are
identical when they reach the bottom of the incline. (2008)
(a) Statement-1 is True, Statement-2 is True; Statement-2
is a correct explanation for Statement-1
(b) Statement-1 is True, Statement-2 is True; Statement-2
is NOT a correct explanation for Statement-1
Now consider two similar systems as shown in the figure: Case (a)
(c) Statement -1 is True, Statement-2 is False
the disc with its face vertical and parallel to x-z plane; Case (b) the
disc with its face making an angle of 45° with x-y plane and its (d) Statement -1 is False, Statement-2 is True
horizontal diameter parallel to x-axis. In both the cases, the disc is
welded at point P, and the systems are rotated with constant I Integer Value Correct Type
angular speed w about the z-axis.
z Q z 1. A binary star consists of two stars A (mass 2.2Ms) and B
w w Q
(mass 11Ms), where Ms is the mass of the sun. They are
separated by distance d and are rotating about their centre
y 45° y of mass, which is stationary. The ratio of the total angular
P P
momentum of the binary star to the angular momentum of
x Case (a) x Case (b) star B about the centre of mass is (2010)
EBD_7036
P-44 Topic-wise Solved Papers - PHYSICS
2. A boy is pushing a ring of mass 9 ms–1 with respect to the ground. The rotational speed of
2 kg and radius 0.5 m with a stick the platform in rad s–1 after the balls leave the platform is
as shown in the figure. The stick (JEE Adv. 2014)
applies a force of 2N on the ring 7. A uniform circular disc of mass 1.5 kg F
and rolls it without slipping with and radius 0.5 m is initially at rest on a X
an acceleration of 0.3 m/s2. The horizontal frictionless surface. Three
coefficient of friction between forces of equal magnitude F = 0.5 N are
the ground and the ring is large applied simultaneously along the three O
enough that rolling always occurs and the coefficient of sides of an equilateral triangle XYZ with
friction between the stick and the ring is (P/10). The value of its vertices on the perimeter of the disc Y F
Z
P is (2011) (see figure). One second after applying F
the forces, the angular speed of the disc
3. Four solid spheres each of diameter 5 cm and mass 0.5 kg in rad s–1 is (JEE Adv. 2014)
are placed with their centers at the corners of a square of 8. Two identical uniform discs roll without slipping on two
side 4 cm. The moment of inertia of the system about the different surfaces AB and CD (see figure) starting at A and C
diagonal of the square is N ´ 10–4 kg– m2, then N is.(2011) with linear speeds v1 and v2, respectively, and always remain
4. A lamina is made by removing a small in contact with the surfaces. If they reach B and D with the
disc of diameter 2R from a bigger disc same linear speed and v1 = 3 m/s then v2 in m/s is (g = 10 m/s2)
of uniform mass density and radius
(JEE Adv. 2015)
2R, as shown in the figure. The moment
of inertia of this lamina about axes A v1 = 3m/s
passing though O and P is IO and IP
respectively. Both these axes are 30m
perpendicular to the plane of the lamina. The ratio IP / IO to B
the nearest integer is (2012)
5. A uniform circular disc of mass 50 kg and radius 0.4 m is
rotating with an angular velocity of 10 rad s–1 about its own v2
axis, which is vertical. Two uniform circular rings, each of C
mass 6.25 kg and radius 0.2 m, are gently placed symmetrically 27m
on the disc in such a manner that they are touching each
D
other along the axis of the disc and are horizontal. Assume
that the friction is large enough such that the rings are at
rest relative to the disc and the system rotates about the
original axis. The new angular velocity (in rad s–1) of the 9. The densities of two solid spheres A and B of the same radii
system is (JEE Adv. 2013)
R vary with radial distance r as rA(r) = k æç ö÷ and rB(r) =
r
6. A horizontal circular platform of
è Rø
radius 0.5 m and mass
0.45 kg is free to rotate about its 5
ærö
axis. Two massless spring toy- k ç ÷ , respectively, where k is a constant. The moments
guns, each carrying a steel ball of è Rø
mass 0.05 kg are attached to the platform at a distance 0.25 m of inertia of the individual spheres about axes passing
from the centre on its either sides along its diameter (see IB n
through their centres are IA and IB, respectively. If , = ,
figure). Each gun simultaneously fires the balls horizontally I A 10
and perpendicular to the diameter in opposite directions. the value of n is (JEE Adv. 2015)
After leaving the platform, the balls have horizontal speed of

Section-B JEE Main / AIEEE


1. Initial angular velocity of a circular disc of mass M is w 1. 2. The minimum velocity (in ms–1) with which a car driver must
Then two small spheres of mass m are attached gently to traverse a flat curve of radius 150 m and coefficient of friction
diametrically opposite points on the edge of the disc. What 0.6 to avoid skidding is [2002]
is the final angular velocity of the disc? [2002] (a) 60 (b) 30 (c) 15 (d) 25
3. A cylinder of height 20 m is completely filled with water. The
æ M + mö æ M + mö velocity of efflux of water (in ms–1) through a small hole on
(a) çè ÷w (b) çè ÷w
M ø 1 m ø 1 the side wall of the cylinder near its bottom is [2002]
(a) 10 (b) 20 (c) 25.5 (d) 5
æ M ö æ M ö 4. Two identical particles move towards each other with
(c) çè ÷w (d) çè ÷ w1. velocity 2v and v respectively. The velocity of centre of
M + 4m ø 1 M + 2mø mass is [2002]
(a) v (b) v/3 (c) v/2 (d) zero.
Rotational Motion P-45

5. A solid sphere, a hollow sphere and a ring are released from (a) does not shift
top of an inclined plane (frictionless) so that they slide down (b) depends on height of breaking
the plane. Then maximum acceleration down the plane is for (c) body B
(no rolling) [2002] (d) body C
(a) solid sphere (b) hollow sphere 14. The moment of inertia of a uniform semicircular disc of mass
(c) ring (d) all same. M and radius r about a line perpendicular to the plane of the
6. Moment of inertia of a circular wire of mass M and radius R disc through the centre is [2005]
about its diameter is [2002] 2 1 1
2
(a) MR /2 (b) MR 2 (c) 2MR 2 2
(d) MR /4. (a) Mr 2 (b) Mr (c) Mr 2 (d) Mr 2
5 4 2
7. A particle of mass m moves along line PC with velocity v as 15. A ‘T’ shaped object with l
shown. What is the angular momentum of the particle dimensions shown in the figure, is A B
about P? [2002] uur
lying on a smooth floor. A force ‘ F ’
(a) mvL C P
is applied at the point P parallel to F 2l
(b) mvl L
AB, such that the object has only
(c) mvr P the translational motion without
l
rotation. Find the location of P with
(d) zero. O respect to C. [2005] C

8. A circular disc X of radius R is made from an iron plate of 3 2 4


thickness t, and another disc Y of radius 4R is made from an (a) l (b) l (c) l (d) l
2 3 3
t 16. Consider a two particle system with particles having masses
iron plate of thickness . Then the relation between the m1 and m2. If the first particle is pushed towards the centre
4
of mass through a distance d, by what distance should the
moment of inertia I X and IY is [2003]
second particle is moved, so as to keep the centre of mass at
(a) ΙY = 32 Ι X (b) ΙY = 16 Ι X the same position? [2006]
(c) Ι Y = Ι X (d) ΙY = 64 Ι X m2 m1 m1
(a) d (b) d (c) d (d) d
9. A particle performing uniform circular motion has angular m1 m1 + m2 m2
frequency is doubled & its kinetic energy halved, then the
17. Four point masses, each of value m, are placed at the corners
new angular momentum is [2003]
of a square ABCD of side l. The moment of inertia of this
L
(a) L (b) 2 L (c) 4 L (d) system about an axis passing through A and parallel to BD
2 is [2006]
r4
10. Let F be the force acting on a particle having position (a) 2ml 2 (b) 3ml2 (c) 3ml 2 (d) ml 2
r r
vector r , and T be the torque of this force about the origin. 18. A force of – Fkˆ acts on O, the origin of the coordinate
Then [2003] system. The torque about the point (1, –1) is [2006]
r r r r r r r r Z
(a) r .T = 0 and F .T ¹ 0 (b) r .T ¹ 0 and F .T = 0 (a) F (iˆ - ˆj )
r r r r r r r r
(c) r .T ¹ 0 and F .T ¹ 0 (d) r .T = 0 and F .T = 0 (b) - F (iˆ + ˆj )
11. A solid sphere is rotating in free space. If the radius of the O
sphere is increased keeping mass same which one of the
(c) F (iˆ + ˆj ) Y

following will not be affected ? [2004] (d) - F (iˆ - ˆj ) X


(a) Angular velocity (b) Angular momentum
(c) Moment of inertia (d) Rotational kinetic energy 19. A thin circular ring of mass m and radius R is rotating about
12. One solid sphere A and another hollow sphere B are of same its axis with a constant angular velocity w. Two objects each
mass and same outer radii. Their moment of inertia about of mass M are attached gently to the opposite ends of a
diameter of the ring. The ring now rotates with an angular
their diameters are respectively I A and I B Such that velocity w' = [2006]
[2004] w ( m - 2 M )
w(m + 2 M )
(a) I A < I B (b) I A > I B (a) (b)
(m + 2 M )
m
IA dA wm wm
(c) IA = IB (d) = (c) (d)
I B dB (m + M ) (m + 2M )
where d A and d B are their densities. 20. A circular disc of radius R is removed from a bigger circular
13. A body A of mass M while falling vertically downwards disc of radius 2R such that the circumferences of the discs
coincide. The centre of mass of the new disc is a / R form
1
under gravity breaks into two parts; a body B of mass the centre of the bigger disc. The value of a is [2007]
3 (a) 1/4 (b) 1/3 (c) 1/2 (d) 1/6
2 21. A round uniform body of radius R, mass M and moment of
M and a body C of mass M. The centre of mass of
3 inertia I rolls down (without slipping) an inclined plane
bodies B and C taken together shifts compared to that of making an angle q with the horizontal. Then its acceleration
body A towards [2005] is [2007]
EBD_7036
P-46 Topic-wise Solved Papers - PHYSICS

g sin q g sin q a diameter of the disc to reach its other end. During the
(a) (b) journey of the insect, the angular speed of the disc.
1 - MR 2 / I 1 + I / MR 2 (a) continuously decreases [2011]
g sin q g sin q (b) continuously increases
(c) (d) (c) first increases and then decreases
1 + MR 2 / I 1 - I / MR 2
(d) remains unchanged
22. Angular momentum of the particle rotating with a central
29. A pulley of radius 2 m is rotated about its axis by a force
force is constant due to [2007]
F = (20t – 5t2) newton (where t is measured in seconds)
(a) constant torque
applied tangentially. If the moment of inertia of the pulley
(b) constant force
about its axis of rotation is 10 kg-m2 the number of rotations
(c) constant linear momentum
made by the pulley before its direction of motion is
(d) zero torque
reversed, is: [2011]
23. For the given uniform square lamina ABCD, whose centre
(a) more than 3 but less than 6
is O, [2007]
(b) more than 6 but less than 9
(a) I AC = 2 I EF D F C (c) more than 9
(d) less than 3
(b) 2 I AC = I EF 30 . A hoop of radius r and mass m rotating with an angular
O
(c) I AD = 3I EF velocity w0 is placed on a rough horizontal surface. The
initial velocity of the centre of the hoop is zero.What will
(d) I AC = I EF A E
B
be the velocity of the centre of the hoop when it ceases to
24. A thin rod of length ‘L’ is lying along the x-axis with its ends slip ? [JEE main 2013]
at x = 0 and x = L. Its linear density (mass/length) varies with
rw0 rw0 rw0
n (a) (b) (c) (d) rw0
æ xö 4 3 2
x as k ç ÷ , where n can be zero or any positive number. If
è Lø 31. A bob of mass m attached to an inextensible string of length
the position xCM of the centre of mass of the rod is plotted l is suspended from a vertical support. The bob rotates in a
against ‘n’, which of the following graphs best approximates horizontal circle with an angular speed w rad/s about the vertical.
the dependence of xCM on n? [2008] About the point of suspension: [JEE Main 2014]
xCM xCM (a) angular momentum is conserved.
(b) angular momentum changes in magnitude but not in
L direction.
L L
(a) (b) (c) angular momentum changes in direction but not in
2 2
n n magnitude.
O O (d) angular momentum changes both in direction and
xCM xCM
magnitude.
L L 32. Distance of the centre of mass of a solid uniform cone from
L L its vertex is z0. If the radius of its base is R and its height is h
(c) 2
(d) 2 then z0 is equal to : [JEE Main 2015]
n n
O O
25. Consider a uniform square plate of side ‘a’ and mass ‘m’.
5h 3h 2 h2 3h
(a) (b) (c) (d)
The moment of inertia of this plate about an axis 8 8R 4R 4
perpendicular to its plane and passing through one of its 33. From a solid sphere of mass M and radius R a cube of maximum
corners is [2008] possible volume is cut. Moment of inertia of cube about an
5 2 1 7 2 2 axis passing through its center and perpendicular to one of
(a) ma (b) ma 2 (c) ma 2 (d) ma its faces is : [JEE Main 2015]
6 12 12 3
26. A thin uniform rod of length l and mass m is swinging freely 4MR 2 4MR 2 MR 2 MR 2
about a horizontal axis passing through its end. Its maximum (a) (b) (c) (d)
9 3p 3 3p 32 2p 16 2p
angular speed is w. Its centre of mass rises to a maximum 34. A roller is made by joining B
height of : [2009] D
together two cones at their
1 lw 1 l 2w 2 1 l 2w 2 1 l 2w2 vertices O. It is kept on two rails
(a) (b) (c) (d) AB and CD, which are placed
6 g 2 g 6 g 3 g asymmetrically (see figure), with
27. A mass m hangs with the help of a string wrapped around its axis perpendicular to CD and O
a pulley on a frictionless bearing. The pulley has mass m its centre O at the centre of line
and radius R. Assuming pulley to be a perfect uniform joining AB and Cd (see figure).
circular disc, the acceleration of the mass m, if the string C
It is given a light push so that it A
does not slip on the pulley, is: [2011] starts rolling with its centre O moving parallel to CD in the
2
g
g 3 direction shown. As it moves, the roller will tend to :
(a) g (b) (c) (d) g
(a) go straight. [JEE Main 2016]
3 3 2
28. A thin horizontal circular disc is rotating about a vertical (b) turn left and right alternately.
axis passing through its centre. An insect is at rest at a (c) turn left.
point near the rim of the disc. The insect now moves along (d) turn right.
CHAPTER

7 Gravitation
Section-A JEE Advanced/ IIT-JEE
(a) 1/2 hr (b) 1 hr (c) 2 hr (d) 4 hr
A Fill in the Blanks 5. A simple pendulum is oscillating without damping. When
the displacement of the bob is less than maximum, its
acceleration vector ar is correctly shown in :
1. The numerical value of the angular velocity of rotation of
(2002S)
the earth should be ............rad/s in order to make the effective
acceleration due to gravity equal to zero. (1984 - 2 Marks)
2. A geostationary satellite is orbiting the earth at a height of
6 R above the surface of the earth, where R is the radius of
the earth. The time period of another satellite at a height of a
(a) (b)
2.5 R from the surface of the earth is ..........hours.
(1987 - 2 Marks) a
3. The masses and radii of the Earth and the Moon are M1, R1and
M2, R2 respectively. Their centres are at a distance d apart. The
minimum speed with which a particle of mass m should be
projected from a point midway between the two centres so as
to escape to infinity is ................. (1988 - 2 Marks) (c) (d)
4. A particle is projected vertically upwards from the surface a a
of earth (radius Re) with a kinetic energy equal to half of the
minimum value needed for it to escape. The height to which
6. A binary star system consists of two stars A and B which
it rises above the surface of earth is.... (1997 - 2 Marks)
have time period TA and TB, radius RA and RB and mass MA
B True/False and MB. Then (2006 - 3M, –1)
(a) if TA > TB then RA > RB (b) if TA > TB then MA > MB
1. It is possible to put an artificial satellite into orbit in such a 2 3
way that it will always remain directly over New Delhi. æ TA ö æ RA ö
(c) ç ÷ = ç (d) TA = TB
(1984 - 2 Marks) è TB ø è RB ÷ø
7. A spherically symmetric gravitational system of particles
C MCQs with One Correct Answer ìr0 for r £ R
has a mass density r = í
1. If the radius of the earth were to shrink by one percent, its î0 for r > R
mass remaining the same, the acceleration due to gravity on where r0 is a constant. A test mass can undergo circular
the earth’s surface would (1981 - 2 Marks) motion under the influence of the gravitational field of
(a) decrease (b) remain unchanged particles. Its speed v as a function of distance r (0 < r < ¥)
(c) increase (d) be zero from the centre of the system is represented by – (2008)
2. If g is the acceleration due to gravity on the earth’s surface, v v
the gain in the potential energy of an object of mass m
raised from the surface of the earth to a height equal to the
radius R of the earth, is (1983 - 1 Mark)
1 1 (a) (b)
(a) mg R (b) 2 mg R (c) mg R (d) mg R
2 4
3. If the distance between the earth and the sun were half its R r R r
present value, the number of days in a year would have v v
been (1996 - 2 Marks)
(a) 64.5 (b) 129 (c) 182.5 (d) 730
4. A geo-stationary satellite orbits around the earth in a circular
orbit of radius 36,000km. Then, the time period of a spy (c) (d)
satellite orbiting a few hundred km above the earth's surface
(Rearth = 6,400km) will approximately be (2002S) R r R r
EBD_7036
P-48 Topic-wise Solved Papers - PHYSICS
8. A thin uniform annular disc (see figure) of mass M has outer m are F1 and F2 respectively. Then: (1994 - 2 Marks)
radius 4R and inner radius 3R. The work required to take a
F1 r1
unit mass from point P on its axis to infinity is (2010) =
(a) F2 r2 if r1 < R and r2 < R
2GM
(a) (4 2 - 5)
7R F1 r22
=
2GM (b) F2 r12 if r1 > R and r2 > R
(b) - (4 2 - 5)
7R
F1 r1
=
GM 2GM
(c) F2 r2 if r1 > R and r2 > R
(c) (d) ( 2 - 1)
4R 5R F1 r12
9. A satellite is moving with a constant speed ‘V’ in a circular (d) F = 2 if r1 < R and r2 < R
orbit about the earth. An object of mass ‘m’ is ejected from 2 r2
the satellite such that it just escapes from the gravitational 4. A satellite S is moving in an elliptical orbit around the earth.
pull of the earth. At the time of its ejection, the kinetic energy The mass of the satellite is very small compared to the mass
of the object is (2011) of the earth. (1998S - 2 Marks)
1 3 (a) The acceleration of S is always directed towards the
(a) mV 2 (b) mV2 (c) mV 2 (d) 2mV 2 centre of the earth.
2 2
(b) The angular momentum of S about the centre of the
1
10. A planet of radius R = ´ ( radius of Earth ) has the same earth changes in direction, but its magnitude remains
10 constant.
R (c) The total mechanical energy of S varies periodically
mass density as Earth. Scientists dig a well of depth on
5 with time.
it and lower a wire of the same length and a linear mass (d) The linear momentum of S remains constant in
density 10–3 kg m–1 into it. If the wire is not touching magnitude.
anywhere, the force applied at the top of the wire by a person 5. Two spherical planets P and Q have the same uniform
holding it in place is (take the radius of Earth = 6 × 106 m and density r, masses MP and MQ and surface areas A and 4A
the acceleration due to gravity on Earth is 10 ms–2) respectively. A spherical planet R also has uniform density
(JEE Adv. 2014) r and its mass is (MP + MQ). The escape velocities from the
(a) 96 N (b) 108 N (c) 120 N (d) 150 N planets P, Q and R are VP, VQ and VR, respectively. Then
(2012)
(a) VQ > VR > VP (b) VR > VQ > VP
D MCQs with One or More than One Correct
1
1. Imagine a light planet revolving around a very massive star (c) VR/ VP = 3 (d) VP / VQ =
2
in a circular orbit of radius R with a period of revolution T. If 6. Two bodies, each of mass M, are kept fixed with a separation
the gravitational force of attraction between the planet and 2L. A particle of mass m is projected from the midpoint of the
the star is proportional to R–5/2 (1989 - 2 Mark) line joining their centres, perpendicular to the line. The
(a) T2 is proportional to R3 gravitational constant is G. The correct statement(s) is (are)
(b) T2 is proportional to R7/2 (JEE Adv. 2013)
(c) T2 is proportional to R3/2 (a) The minimum initial velocity of the mass m to escape
(d) T2 is proportional to R3/73
2. A solid sphere of uniform density and radius 4 units is located GM
with its centre at the origin O of coordinates. Two spheres the gravitational field of the two bodies is 4
L
of equal radii 1 unit, with their centres at A (–2, 0 ,0) and (b) The minimum initial velocity of the mass m to escape
B (2, 0, 0) respectively, are taken out of the solid leaving
behind spherical cavities as shown in fig (1993-2 Marks) GM
Then : y the gravitational field of the two bodies is 2
L
(a) The gravitational (c) The minimum initial velocity of the mass m to escape
force due to this
object at the origin is 2GM
zero. -m the gravitational field of the two bodies is
A B
x L
(b) the gravitational force O
(d) The energy of the mass m remains constant
at the point B (2, 0 ,0)
is zero. z E Subjective Problems
(c) the gravitational potential is the same at all points of
circle y2 + z2 = 36. 1. Two satellites S1 and S2 revolve round a planet in coplanar
(d) the gravitational potential is the same at all points on circular orbits in the same sense. Their periods of revolution
the circle y2 + z2 = 4. are 1 hour and 8 hours respectively. The radius of the orbit
3. The magnitudes of the gravitational field at distance r1 and of S1 is 104 km. When S2 is closest to S1, find
r2 from the centre of a uniform sphere of radius R and mass
Gravitation P-49

(i) the speed of S2 relative to S1, (a) Statement- 1 is True, Statement-2 is True, Statement-2
(ii) the angular speed of S2 as actually observed by an is a correct explanation for Statement -1
astronaut in S1. (1986 - 6 Marks) (b) Statement -1 is True, Statement -2 is True ; Statement-
2. Three particles, each of mass m, are situated at the vertices 2 is NOT a correct explanation for Statement - 1
of an equilateral triangle of side length a. The only forces (c) Statement - 1 is True, Statement- 2 is False
acting on the particles are their mutual gravitational forces. (d) Statement -1 is False, Statement -2 is True
It is desired that each particle moves in a circle while
maintaining the original mutual separation a. Find the intial I Integer Value Correct Type
velocity that should be given to each particle and also the
time period of the circular motion. (1988 - 5 Marks) 6
3. An artificial satellite is moving in a circular orbit around the 1. Gravitational acceleration on the surface of a planet is g.
earth with a speed equal to half the magnitude of escape 11
velocity from the earth. (1990 - 8 Mark) where g is the gravitational acceleration on the surface of
(i) Determine the height of the satellite above the earth’s 2
the earth. The average mass density of the planet is
surface. 3
(ii) If the satellite is stopped suddenly in its orbit and times that of the earth. If the escape speed on the surface of
allowed to fall freely onto the earth, find the speed with the earth is taken to be 11 kms–1, the escape speed on the
which it hits the surface of the earth. surface of the planet in kms–1 will be (2010)
4. Distance between the centres of two stars is 10a. The masses 2. A bullet is fired vertically upwards with velocity v from the
of these stars are M and 16M and their radii a and 2a, surface of a spherical planet. When it reaches its maximum
respectively. A body of mass m is fired straight from the 1 th
surface of the larger star towards the smaller star. What height, its acceleration due to the planet's gravity is of
4
should be its minimum initial speed to reach the surface of
its value of the surface of the planet. If the escape velocity
the smaller star? Obtain the expression in terms of G, M
and a. (1996 - 5 Marks) from the planet is vesc = v N , then the value of N is (ignore
5. A body is projected vertically upwards from the bottom of a energy loss due to atmosphere) (JEE Adv. 2015)
R 3. A large spherical mass M is fixed at one position and two
crater of moon of depth where R is the radius of moon identical point masses m are kept on a line passing through
100 the centre of M (see figure). The point masses are connected
with a velocity equal to the escape velocity on the surface by a rigid massless rod of length l and this assembly is free
of moon. Calculate maximum height attained by the body to move along the line connecting them. All three masses
from the surface of the moon. (2003 - 4 Marks) interact only through their mutual gravitational interaction.
When the point mass nearer to M is at a distance r = 3l from
H Assertion & Reason Type Questions
æ M ö
1. STATEMENT - 1 : An astronaut in an orbiting space station M, the tension in the rod is zero for m = k ç ÷ . The value
è 288 ø
above the earth experiences weightlessness.
of k is (JEE Adv. 2015)
because
STATEMENT - 2 : An object moving the earth under the
influence of Earth’s gravitational force is in a state of “free-
fall”. (2008)

Section-B JEE Main / AIEEE


1. The kinetic energy needed to project a body of mass m from 5. The time period of a satellite of earth is 5 hours. If the
the earth surface (radius R) to infinity is [2002] separation between the earth and the satellite is increased
(a) mgR/2 (b) 2mgR (c) mgR (d) mgR/4. to 4 times the previous value, the new time period will become
2. If suddenly the gravitational force of attraction between (a) 10 hours (b) 80 hours [2003]
Earth and a satellite revolving around it becomes zero, then (c) 40 hours (d) 20 hours
the satellite will [2002] 6. Two spherical bodies of mass M and 5M & radii R & 2R
(a) continue to move in its orbit with same velocity respectively are released in free space with initial separation
(b) move tangentially to the original orbit in the same between their centres equal to 12 R. If they attract each other
velocity due to gravitational force only, then the distance covered by
(c) become stationary in its orbit the smaller body just before collision is [2003]
(d) move towards the earth (a) 2.5 R (b) 4.5 R (c) 7.5 R (d) 1.5 R
3. Energy required to move a body of mass m from an orbit of 7. The escape velocity for a body projected vertically upwards
radius 2R to 3R is [2002] from the surface of earth is 11 km/s. If the body is projected
(a) GMm/12R2 (b) GMm/3R2 at an angle of 45°with the vertical, the escape velocity will be
(c) GMm/8R (d) GMm/6R. (a) 11 2 km / s (b) 22 km/s [2003]
4. The escape velocity of a body depends upon mass as 11
(a) m0 (b) m1 [2002] (c) 11 km/s (d) km / s
(c) m2 (d) m3. 2
EBD_7036
P-50 Topic-wise Solved Papers - PHYSICS
8. A satellite of mass m revolves around the earth of radius R the strength of the source enclosed by the surface and not
at a height x from its surface. If g is the acceleration due to on the size or shape of the surface.
gravity on the surface of the earth, the orbital speed of the (a) Statement -1 is false, Statement-2 is true
satellite is [2004] (b) Statement -1 is true, Statement-2 is true; Statement -2 is
æ gR ö 1/ 2
2 a correct explanation for Statement-1
gR 2 gR (c) Statement -1 is true, Statement-2 is true; Statement -2
(a) (b) (c) gx (d) ç ÷
R+x R-x è R + xø is not a correct explanation for Statement-1
9. The time period of an earth satellite in circular orbit is (d) Statement -1 is true, Statement-2 is false
independent of [2004] 17. The height at which the acceleration due to gravity becomes
(a) both the mass and radius of the orbit g
(b) radius of its orbit (where g = the acceleration due to gravity on the surface
9
(c) the mass of the satellite of the earth) in terms of R, the radius of the earth, is : [2009]
(d) neither the mass of the satellite nor the radius of its R
orbit. (a) (b) R / 2 (c) 2 R (d) 2 R
10. If ‘g’ is the acceleration due to gravity on the earth’s surface, 2
the gain in the potential energy of an object of mass ‘m’ 18. Two bodies of masses m and 4 m are placed at a distance r.
raised from the surface of the earth to a height equal to the The gravitational potential at a point on the line joining
radius ‘R' of the earth is [2004] them where the gravitational field is zero is: [2011]
1 1 4Gm 6Gm 9Gm
(a) mgR (b) mgR (c) 2 mgR (d) mgR (a) - (b) - (c) - (d) zero
4 2 r r r
11. Suppose the gravitational force varies inversely as the nth 19. The mass of a spaceship is 1000 kg. It is to be launched from
power of distance. Then the time period of a planet in circular the earth's surface out into free space. The value of g and R
orbit of radius ‘R’ around the sun will be proportional to (radius of earth) are 10 m/s2 and 6400 km respectively. The
[2004] required energy for this work will be : [2012]
(a) 6.4 × 1011Joules (b) 6.4 × 108 Joules
æ n -1ö æ n +1ö æ n- 2ö
çè ÷ çè ÷ (c) 6.4 × 109 Joules (d) 6.4 × 1010 Joules
(a) R n (b) R 2 ø (c) R èç 2 ø÷(d) R 2 ø
20. What is the minimum energy required to launch a satellite of
12. The change in the value of ‘g’ at a height ‘h’ above the mass m from the surface of a planet of mass M and radius R
surface of the earth is the same as at a depth ‘d’ below the in a circular orbit at an altitude of 2R? [JEE Main 2013]
surface of earth. When both ‘d’ and ‘h’ are much smaller
than the radius of earth, then which one of the following is 5GmM 2GmM GmM GmM
(a) (b) (c) (d)
correct ? [2005] 6R 3R 2R 2R
3h h 21. Four particles, each of mass M and equidistant from each
(a) d = (b) d = (c) d = h (d) d =2 h other, move along a circle of radius R under the action of
2 2
13. A particle of mass 10 g is kept on the surface of a uniform their mutual gravitational attraction. The speed of each particle
sphere of mass 100 kg and radius 10 cm. Find the work to is: [JEE Main 2014]
be done against the gravitational force between them to GM GM
take the particle far away from the sphere (you may take G (a) (b) 2 2
R R
= 6.67× 10-11 Nm 2 / kg 2 ) [2005]
(a) 3.33 × 10 -10 J (b) 13.34 × 10 -10 J (c)
GM
R
(
1+ 2 2 ) (d)
1 GM
2 R
1+ 2 2( )
(c) 6.67 × 10 -10 J (d) 6.67 × 10 -9 J 22. From a solid sphere of mass M and radius R, a spherical
14. Average density of the earth [2005] portion of radius R/2 is removed, as shown in the figure.
(a) is a complex function of g Taking gravitational potential V = 0 at r = ¥, the potential at
(b) does not depend on g the centre of the cavity thus formed is : [JEE Main 2015]
(c) is inversely proportional to g (G = gravitational constant)
(d) is directly proportional to g
15. A planet in a distant solar system is 10 times more massive
than the earth and its radius is 10 times smaller. Given that
the escape velocity from the earth is 11 km s–1, the escape
velocity from the surface of the planet would be [2008]
(a) 1.1 km s–1 (b) 11 km s–1 (c) 110 km s–1 (d) 0.11 km s–1
16. This question contains Statement-1 and Statement-2. Of the -2GM -2GM -GM -GM
four choices given after the statements, choose the one (a) (b) (c) (d)
3R R 2R R
that best describes the two statements. [2008]
23. A satellite is revolving in a circular orbit at a height 'h' from
Statement-1 :
the earth's surface (radius of earth R; h < < R). The minimum
For a mass M kept at the centre of a cube of side ‘a’, the flux
increase in its orbital velocity required, so that the satellite
of gravitational field passing through its sides 4 p GM.
could escape from the earth's gravitational field, is close to
Statement-2:
If the direction of a field due to a point source is radial and : (Neglect the effect of atmosphere.) [JEE Main 2016]
its dependence on the distance ‘r’ from the source is given
1
(a) gR / 2 (b) gR ( )
2 -1
as 2 , its flux through a closed surface depends only on (c) 2gR (d) gR
r
CHAPTER

8 Mechanical Properties
of Solids and Fluids
Section-A JEE Advanced/ IIT-JEE
A Fill in the Blanks 4. A block of ice with a lead shot embedded in it is floating on
water contained in a vessel. The temperature of the system
1. A wire of length L and cross sectional area A is made of a is maintained at 0°C as the ice melts. When the ice melts
material of Young’s modulus Y. If the wire is stretched by an completely the level of water in the vessel rises .
amount x, the work done is ................. (1987 - 2 Marks) (1986 - 3 Marks)
2. A solid sphere of radius R made of a material of bulk modulus
K is surrounded by a liquid in a cylindrical container. A
massless pistion of area A floats on the surface of the liquid.
C MCQs with One Correct Answer
When a mass M is placed on the piston to compress 1. A vessel containing water is given a constant acceleration
the liquid the fractional change in the radius of the sphere, ‘a’ towards the right along a straight horizontal path. Which
d R/R, is ........... (1988 - 2 Mark) of the following diagrams in Fig. represents the surface of
3. A piece of metal floats on mercury. The coefficients of
the liquid ? (1981- 2 Marks)
volume expansion of the metal and mercury are g1 and g 2
respectively. If the temperatures of both mercury and the a a a
metal are increased by an amount DT , the fraction of the
volume of the metal submerged in mercury changes by the (a) (b) (c)
factor ................... (1991 - 2 Mark) 2. The following four wires are made of the same material.
4. A horizontal pipeline carries water in a streamline flow. Which of these will have the largest extension when the
At a point along the pipe, where the cross- sectional
same tension is applied ? (1981- 2 Marks)
area is 10 cm 2, the water velocity is 1 ms–1 and the
(a) length = 50 cm , diameter = 0.5 mm
pressure is 2000 Pa. The pressure of water at another
point where the cross-sectional area is 5 cm 2, is... Pa. (b) length = 100 cm, diameter = 1 mm
(Density of water = 103 kg.m –3) (1994 - 2 Marks) (c) length = 200 cm, diameter = 2 mm
(d) length = 300 cm, diameter = 3 mm.
B True/False 3. A U-tube of uniform cross section (see Fig) is partially filled
with a liquid I. Another liquid II which does not mix with
1. A man is sitting in a boat which is floating in a pond. If the
liquid I is poured into one side. It is found that the liquid
man drinks some water from the pond, the level of the water
levels of the two sides of the tube are the same, while the
in the pond decreases. (1980) level of liquid I has risen by 2 cm. If the specific gravity of
2. A barometer made of a very narrow vacuum liquid I is 1.1, the specific gravity of liquid II must be
tube (see Fig) is placed at normal (1983 - 1 Mark)
temperature and pressure. The Hg
coefficient of volume expansion of (a) 1.12
II I
mercury is 0.00018 per C° and that (b) 1.1
of the tube is negligible. The
temperature of mercury in the barometer is now raised by (c) 1.05
1°C, but the temperature of the atmosphere does not change. (d) 1.0
Then the mercury height in the tube remains unchanged.
4. A homogeneous solid cylinder of length L (L < H/2), cross-
(1983 - 2 Marks) sectional area A/5 is immersed such that it floats with its axis
3. Water in a closed tube (see Fig) is heated vertical at the liquid-liquid interface with length L/4 in the
with one arm vertically placed above a lamp. denser liquid as shown in the figure. The lower density
Water will begin to circulate along the tube A B
liquid is open to atmosphere having pressure P0. Then
in counter-clockwise direction. density D of solid is given by (1995S)
(1983 - 2 Marks)
EBD_7036
P-52 Topic-wise Solved Papers - PHYSICS
of the container is A. If a/A = 0.1 then v2 is (where v is the
5
(a) d velocity of water coming out of the hole) (2005S)
4
(a) 50 (b) 51 (c) 48 (d) 51.5
4 3L/4 10. When temperature of a gas is 20ºC and pressure is changed
(b) d d
5 from p1 = 1.01 × 105 Pa to p2 =1.165 × 105 Pa then the
H
(c) 4d L/4 volume changed by 10%. The bulk modulus is (2005S)
(a) 1.55 × 105 Pa (b) 0.115 × 105 Pa
d 2d
(d) (c) 1.4 × 105 Pa (d) 1.01 × 105 Pa
5 11. A glass tube of uniform
5. A large open tank has two holes in the wall. One is a square internal radius (r) has a valve
hole of side L at a depth y from the top and the other is a separating the two identical
circular hole of radius R at a depth 4 y from the top. When ends. Initially, the valve is in
the tank is completely filled with water, the quantities of a tightly closed position. 1
2
water flowing out per second from both holes are the same.
End 1 has a hemispherical soap bubble of radius r. End 2 has
Then, R is equal to (2000S)
sub-hemispherical soap bubble as shown in figure. Just after
L L opening the valve, (2008)
(a) (b) 2pL (c) L (d)
2p 2p (a) air from end 1 flows towards end 2. No change in the
6. A hemispherical portion of radius R is removed from the volume of the soap bubbles
bottom of a cylinder of radius R. The volume of the remaining (b) air from end 1 flows towards end 2. Volume of the soap
cylinder is V and its mass M. It is suspended by a string in bubble at end 1 decreases
a liquid of density r where it stays vertical. The upper (c) no changes occurs
surface of the cylinder is at a depth h below the liquid surface. (d) air from end 2 flows towards end 1. volume of the soap
The force on the bottom of the cylinder by the liquid is bubble at end 1 increases
12. A thin uniform cylindrical shell, closed at both ends, is
(a) Mg (2001S) partially filled with water. It is floating vertically in water in
half-submerged state. If rc is the relative density of the
(b) Mg – Vrg h material of the shell with respect to water, then the correct
statement is that the shell is (2012- II)
(c) Mg + pR2hrg r
(a) more than half-filled if rc is less than 0.5.
(d) rg(V + pR2h) 2R
(b) more than half-filled if rc is more than 1.0.
coin (c) half-filled if rc is more than 0.5.
7. A wooden block, with a coin (d) less than half-filled if rc is less than 0.5.
placed on its top, floats in 13. One end of a horizontal thick copper wire of length 2L and
water as shown in figure. The l radius 2R is welded to an end of another horizontal thin
distance l and h are shown h copper wire of length L and radius R. When the arrangement
here. After some time the coin is stretched by applying forces at two ends, the ratio of the
falls into the water. Then elongation in the thin wire to that in the thick wire is
(a) l decreases and h increases (2002S ) (a) 0.25 (b) 0.50 (JEE Adv. 2013)
(b) l increases and h decreases (c) 2.00 (d) 4.00
(c) both l and h increase (d) both l and h decrease 14. A glass capillary tube is of the shape of a truncated cone
8. The adjacent graph shows the estension (Dl) of a wire of with an apex angle a so that its two ends have cross sections
of different radii. When dipped in water vertically, water rises
length 1 m suspended from the top of a roof at one end and
in it to a height h, where the radius of its cross section is b. If
with a load W connected to the other end. If the cross-
the surface tension of water is S, its density is r, and its
sectional area of the wire is 10–6 m2, calculate the Young’s
contact angle with glass is q, the value of h will be (g is the
modulus of the material of the wire. (2003S) acceleration due to gravity) (JEE Adv. 2014)
Dl (x10 -4 m)
(a) 2 × 1011 N/m 2S
(a) cos ( q - a )
4 brg
(b) 2 × 10–11 N/m
2S h
3
(b) cos ( q + a )
2 brg
(c) 3 × 10–12 N/m
1 2S
W (N) (c) cos ( q - a / 2)
(d) 2 × 10–13 N/m 20 40 60 80 brg
9. Water is filled in a container upto height 3m. A small hole 2S
of area ‘a’ is punched in the wall of the container at a (d) cos ( q + a / 2)
brg
height 52.5 cm from the bottom. The cross sectional area
Mechanical Properties of Solids & Fluids P-53

D MCQs with One or More than One Correct


(a) The net elongation of the spring is
4pR3rg
3k
1. A body floats in a liquid contained in a beaker. The whole
system as shown in Figure falls freely under gravity. The 8pR3rg
upthrust on the body is (1982 - 3 Marks) (b) The net elongation of the spring is
3k
(c) The light sphere is partially submerged
(d) The light sphere is completely submerged
7. In plotting stress versus strain curves for two materials P
and Q,a student by mistake puts strain on the y-axis and
(a) zero stress on the x-axis as shown in the figure. Then the correct
(b) equal to the weight of the liquid displaced statement(s) is (are) (JEE Adv. 2015)
(c) equal to the weight of the body in air
(d) equal to the weight of the immersed portion of the body
2. The spring balance A reads 2 kg with a block m suspended
from it. A balance B reads 5 kg when a beaker with liquid is
put on the pan of the balance. The two balances are now so
arranged that the hanging mass is inside the liquid in the
beaker as shown in the figure. In this situation:
(1985 - 2 Marks)
(a) the balance A will read more than 2 kg
(b) the balance B will read more than 5 kg A
(c) the balance A will read less than 2 kg
and B will read more than 5 kg m
B (a) P has more tensile strength than Q
(d) the balance A and B will read 2 kg and
(b) P is more ductile than Q
5 kg respectively
(c) P is more brittle than Q
3. A vessel contains oil (density = 0.8 gm/cm3) over mercury
(d) The Young's modulus of P is more than that of Q
(density = 13.6 gm cm3). A homogeneous sphere floats with
8. A spherical body of radius R consists of a fluid of constant
half its volume immersed in mercury and the other half in oil.
density and is in equilibrium under its own gravity. If P(r) is
The density of the material of the sphere in gm/cm3 is
the pressure at r(r < R), then the correct option(s) is (are)
(1988 - 2 Mark)
(JEE Adv. 2015)
(a) 3.3 (b) 6.4
(c) 7.2 (d) 12.8 P (r = 3R /4) 63
(a) P(r = 0) = 0 (b) =
4. Two rods of different materials having coefficients of thermal P (r = 2R /3) 80
expansion a1, a 2 and Young’s modulii Y1, Y2 respectively
P (r = 3R /5) 16 P (r = R /2) 20
are fixed between two rigid massive walls. The rods are (c) = (d) =
heated such that they undergo the same increase in P (r = 2R /5) 21 P (r = R /3) 27
temperature. There is no bending of the rods. If a1 : a 2 = 9. Two spheres P and Q of equal radii have densities r1 and
r2, respectively. The spheres are connected by a massless
2 : 3, the thermal stresses developed in the two rods are
string and placed in liquids L1 and L2 of densities s1 and s2
equal provided Y1 : Y2 is equal to (1989 - 2 Mark) and viscosities h1 and h2, respectively. They float in
(a) 2 : 3 (b) 1 : 1 equilibrium with the sphere P in L1 and sphere Q in L2 and
(c) 3 : 2 (d) 4 : 9 the string being taut (see figure). If sphere P alone in L2 has
5. Water from a tap emerges vertically downwards with an ur
terminal velocity VP and Q alone in L1 has terminal velocity
initial spped of 1.0 m s–1. The cross-sectional area of the tap ur
is 10–4 m2. Assume that the pressure is constant throughout V Q , then (JEE Adv. 2015)
the stream of water, and that the flow is steady. The cross- ur
sectional area of the stream 0.15 m below the tap is VP h
(1998S - 2 Marks) (a) ur = 1
VQ h 2 L1
(a) 5.0 × 10–4 m2 (b) 1.0 × 10–5 m2
(c) 5.0 × 10–5 m2 (d) 2.0 × 10–5 m2 ur
VP h
6. A solid sphere of radius R and density r is attached to one ur = 2 L2
(b) h1
end of a mass-less spring of force constant k. The other end VQ
of the spring is connected to another solid sphere of radius ur ur
R and density 3r. The complete arrangement is placed in a (c) V P .V Q > 0
liquid of density 2r and is allowed to reach equilibrium. The ur ur
(d) V P .V Q < 0
correct statement(s) is (are) (JEE Adv. 2013)
EBD_7036
P-54 Topic-wise Solved Papers - PHYSICS
(i) the density D of the solid and
E Subjective Problems (ii) the total pressure at the bottom of the container.
(b) The cylinder is removed and the original arrangement
1. A column of mercury of 10 cm length is contained in the
is restored. A tiny hole of area s(s << A) is punched on
middle of a narrow horizontal 1 m long tube which is closed the vertical side of the container at a height h(h < H/2).
at both the ends. Both the halves of the tube contain air at Determine :
a pressure of 76 cm of mercury. By what distance will the (i) the initial speed of efflux of the liquid at the hole,
column of mercury be displaced if the tube is held vertically? (ii) the horizontal distance x travelled by the liquid
(1978) initially, and
2. A point mass m is suspended at the end of a massless wire (iii) the height hm at which the hole should be punched
of length l and cross section A. If Y is the Young’s modulus so that the liquid travels the maximum distance xm
for the wire, obtain the frequency of oscillation for the simple initially. Also calculate xm.
harmonic motion along the vertical line. (1978) (Neglect the air resistance in these calculations.)
3. A cube of wood supporting 200 gm mass just floats in water. 8. A non-viscous liquid of constant density 1000 kg/m3 flows
When the mass is removed, the cube ruses by 2cm. What is in a streamline motion along a tube of variable cross section.
the size of the cube? (1978) The tube is kept inclined in the vertical plane as shown in
4. A boat floating in a water tank is carrying a number of large Figure. The area of cross section of the tube two points P
and Q at heights of 2 metres and 5 metres are respectively
stones. If the stones are unloaded into water, what will
4×10–3 m2 and 8×10–3 m2. The velocity of the liquid at point
happen to the water level? (1979) P is 1 m/s. Find the work done per unit volume by the
5. A wooden plank of length 1 m and uniform cross-section is pressure and the gravity forces as the fluid flows from point
hinged at one end to the bottom of a tank as shown in fig P to Q. (1997 - 5 Marks)
The tank is filled with water upto a height 0.5 m. The specific
gravity of the plank is 0.5. Find the angle q that the plank
Q
makes with the vertical in the equilibrium position. (Exclude
the case q = 0°) (1984- 8 Marks)
P 5m

2m
q
9. A uniform solid cylinder of
6. A ball of density d is dropped on to a horizontal solid surface. density 0.8 g/cm3 floats in
It bounces elastically from the surface and returns to its equilibrium in a combination of Air h
original position in a time t1. Next, the ball is released and it two non-mixing liquids A and
falls through the same height before striking the surface of B with its axis vertical.
a liquid of density of dL (1992 - 8 Marks) The densities of the liquids A hA
(a) If d < dL, obtain an expression (in terms of d, t1 and dL) and B are 0.7 g/cm3 and 1.2 g/ A
for the time t2 the ball takes to come back to the position cm3, respectively. The height
from which it was released. of liquid A is hA = 1.2 cm. The hB
length of the part of the B
(b) Is the motion of the ball simple harmonic?
(c) If d = dL, how does the speed of the ball depend on its cylinder immersed in liquid B
depth inside the liquid? Neglect all frictional and other is hB = 0.8 cm. (2002 - 5 Marks )
dissipative forces. Assume the depth of the liquid to (a) Find the total force exerted by liquid A on the cylinder.
be large. (b) Find h, the length of the part of the cylinder in air.
7. A container of large uniform cross-sectional area A resting (c) The cylinder is depressed in such a way that its top
on a horizontal surface, holds two immiscible, non-viscous surface is just below the upper surface of liquid A and is
and incompressible liquids of densities d and 2d, each of then released. Find the acceleration of the cylinder
height H/2 as shown in the figure. The lower density liquid immediately after it is released.
is open to the atmosphere having pressure P0 . 10. A bubble having surface tension T and radius R is formed
(1995 - 5 + 5 Marks) on a ring of radius b (b << R). Air is blown inside the tube
H/2 d with velocity v as shown. The air molecule collides
perpendicularly with the wall of the bubble and stops.
Calculate the radius at which the bubble separates from the
H/2 2d h ring. (2003 - 4 Marks)
b R
x
(a) A homogeneous solid cylinder of length L(L < H/2), v
cross-sectional area A/5 is immersed such that it floats
with its axis vertical at the liquid-liquid interface with
length L/4 in the denser liquid. Determine:
Mechanical Properties of Solids & Fluids P-55

11. Shown in the figure is a D 13. A uniform wire having mass per unit length l is placed over
P
container whose top and a liquid surface. The wire causes the liquid to depress by
bottom diameters are D and d y(y << a) as shown in figure. Find surface tension of liquid.
respectively. At the bottom of h r Neglect end effect. (2004 - 2 Marks)
the container, there is a
capillary tube of outer radius
b and inner radius a. y
The volume flow rate in the
capillary is Q. If the capillary d
is removed the liquid comes out with a velocity of v0. The
density of the liquid is given as r. Calculate the coefficient
of viscosity h. (2003 - 4 Marks) a a
12. A tube has two area of cross-sections as shown in figure.
w
The diameters of the tube are 8 mm and 2 mm. Find range of 14. A U tube is rotated about one of
water falling on horizontal surface, if piston is moving with it’s limbs with an angular
a constant velocity of 0.25 m/s, h = 1.25 m (g = 10 m/s2) velocity w. Find the difference H
(2004 - 2 Marks)
in height H of the liquid
(density r) level, where
diameter of the tube d << L.
(2005 - 2 Marks) L
h

F Match the Following

DIRECTIONS (Q. No. 1) : Following question has matching lists. The codes for the lists have choices (a), (b), (c) and (d) out of
which ONLY ONE is correct.
1. A person in lift is holding a water jar, which has a small hole at the lower end of its side. When the lift is at rest, the water jet coming
out of the hole hits the floor of the lift at a distance d of 1.2 m from the person. In the following, state of the lift’s motion is given
in List-I and the distance where the water jet hits the floor of the lift is given in List-II. Match the statements from List-I with those
in List-II and select the correct answer using the code given below the lists. (JEE Adv. 2014)
List - I List - II
P. Lift is accelerating vertically up 1. d = 1.2 m
Q. Lift is accelerating vertically down with an acceleration less 2. d > 1.2 m
than the gravitational acceleration
R. Lift is moving vertically up with constant speed 3. d < 1.2 m
S. Lift is falling freely 4. No water leaks out of the jar
Code: ension
(a) P-2, Q-3, R-2, S-4 (b) P-2, Q-3, R-1, S-4 (c) P-1, Q-1, R-1, S-4 (d) P-2, Q-3, R-1, S-1

G Comprehension Based Questions Situation I : Initially, the tank is filled with water of density r to a
height such that the height of water above the top of the block is
PASSAGE-I h1 (measured from the top of the block).
A cylindrical tank has a hole of diameter 2r in its bottom. The hole Situation II : The water is removed from the tank to a height h2
is covered wooden cylindrical block of diameter 4r, height h and (measured from the bottom of the block), as shown in the figure.
density r/3. The height h2 is smaller than h (height of the block) and thus the
block is exposed to the atmosphere.
h1 r/3
4r 1. Find the minimum value of height h1 (in situation 1), for
which the block just starts to move up? (2006 - 5M, –2)
h2 h
2h 5h 5h 5h
r (a) (b) (c) (d)
3 4 3 2

2r
EBD_7036
P-56 Topic-wise Solved Papers - PHYSICS
2. Find the height of the water level h2 (in situation 2), for 7. If the piston is pushed at a speed of 5 mms–1, the air comes
which the block remains in its original position without the out of the nozzle with a speed of (JEE Adv. 2014)
application of any external force (2006 - 5M, –2) (a) 0.1 ms–1 (b) 1 ms–1
h 4h 2h (c) 2 ms–1 (d) 8 ms–1
(a) (b) (c) (d) h 8. If the density of air is ra, and that of the liquid rl, then for a
3 9 3 given piston speed the rate (volume per unit time) at which
3. In situation 2, if h2 is further decreased, then the liquid is sprayed will be proportional to (JEE Adv. 2014)
(2006 - 5M, –2)
(a) cylinder will not move up and remains at its original ra rl
position (a) rl (b) ra rl (c) ra (d) r l
h
(b) for h2 =
3
, cylinder again starts moving up H Assertion & Reason Type Questions
h 1. STATEMENT-1 : The stream of water flowing at high speed
(c) for h2 = , cylinder again starts moving up from a garden hose pipe tends to spread like a fountain
4
when held vertically up, but tends to narrow down when
h held vertically down.
(d) for h2 = , cylinder again starts moving up
5 STATEMENT-2 : In any steady flow of an incompressible
PASSAGE-II fluid, the volume flow rate of the fluid remains constant.
When liquid medicine of density r is to put in the eye, it is done (2008)
with the help of a dropper. As the bulb on the top of the dropper (a) Statement-1 is True, Statement-2 is True; Statement-2
is pressed, a drop forms at the opening of the dropper. We wish to is a correct explanation for Statement-1
estimate the size of the drop. We first assume that the drop formed (b) Statement-1 is True, Statement-2 is True; Statement-2
at the opening is spherical because that requires a minimum is NOT a correct explanation for Statement-1
increase in its surface energy. To determine the size, we calculate (c) Statement-1 is True, Statement-2 is False
the net vertical force due to the surface tension T when the radius (d) Statement-1 is False, Statement-2 is True
of the drop is R. When this force becomes smaller than the weight
of the drop, the drop gets detached from the dropper. I Integer Value Correct Type
4. If the radius of the opening of the dropper is r, the vertical
force due to the surface tension on the drop of radius R 1. Two soap bubbles A and B are kept in a closed chamber
(assuming r << R) is (2010) where the air is maintained at pressure 8 N/m2. The radii of
(a) 2p rT (b) 2p RT bubbles A and B are 2 cm and 4 cm, respectively. Surface
tension of the soap-water used to make bubbles is 0.04 N/m.
2pr 2T 2pR 2T Find the ratio nB/nA, where n A and nB are the number of
(c) (d) moles of air in bubbles A and B, respectively. [Neglect the
R r
effect of gravity.] (2009)
5. If r = 5 ´ 10–4 m, r = 103 kgm –3 , g = 10 ms –2 , T = 0.11Nm –1 , 2. A cylindrical vessel of height 500 mm has an orifice (small
the radius of the drop when it detaches from the dropper is hole) at its bottom. The orifice is initially closed and water is
approximately (2010) filled in it up to height H. Now the top is completely sealed
(a) 1.4 ´ 10 m –3 (b) 3.3 ´ 10 m –3 with a cap and the orifice at the bottom is opened. Some
water comes out from the orifice and the water level in the
(c) 2.0 ´ 10 –3 m (d) 4.1 ´ 10–3 m vessel becomes steady with height of water column being
6. After the drop detaches, its surface energy is (2010) 200 mm. Find the fall in height (in mm) of water level due to
(a) 1.4 ´ 10–6 J (b) 2.7 ´ 10 –6 J opening of the orifice.
[Take atmospheric pressure = 1.0 × 105 N/m2, density of
(c) 5.4 ´ 10 –6 J (d) 8.1 ´ 10 –6 J water = 1000 kg/m3\ and g = 10 m/s2. Neglect any effect of
PASSAGE-III surface tension.] (2009)
A spray gun is shown in the figure where a piston pushes air out 3. A 0.1 kg mass is suspended from a wire of negligible mass.
of a nozzle. A thin tube of uniform cross section is connected to The length of the wire is 1m and its crosssectional area is
the nozzle. The other end of the tube is in a small liquid container. 4.9 × 10–7 m2. If the mass is pulled a little in the vertically
As the piston pushes air through the nozzle, the liquid from the downward direction and released, it performs simple
container rises into the nozzle and is sprayed out. For the spray harmonic motion of angular frequency 140 rad s–1. If the
gun shown, the radii of the piston and the nozzle are 20 mm and Young’s modulus of the material of the wire is n × 109 Nm–2,
1 mm respectively. The upper end of the container is open to the the value of n is (2010)
atmosphere. 4. Consider two solid spheres P and Q each of density 8 gm
cm–3 and diameters 1 cm and 0.5 cm, respectively. Sphere P
is dropped into a liquid of density 0.8 gm cm–3 and viscosity
h = 3 poiseulles. Sphere Q is dropped into a liquid of density
1.6 gm cm–3 and viscosity h = 2 poiseulles. The ratio of the
terminal velocities of P and Q is (JEE Adv. 2016)
Mechanical Properties of Solids & Fluids P-57

Section-B JEE Main / AIEEE


1. A spring of force constant 800 N/m has an extension of
5 cm. The work done in extending it from 5 cm to 15 cm is Vg (r1 – r2 ) Vg r1
(a) (b) [2008]
(a) 16 J (b) 8 J [2002] k k
(c) 32 J (d) 24 J
2. A wire fixed at the upper end stretches by length l by Vg r1 Vg (r1 – r2 )
(c) (d)
applying a force F. The work done in stretching is [2004] k k
F Fl 10. A jar is filled with two non-mixing liquids 1 and 2 having
(a) 2Fl (b) Fl (c) (d)
2l 2 densities r1 and, r2 respectively. A solid ball, made of a
3. Spherical balls of radius ‘R’ are falling in a viscous fluid of material of density r3 , is dropped in the jar. It comes to
viscosity ‘h’ with a velocity ‘v’. The retarding viscous force equilibrium in the position shown in the figure.Which of
acting on the spherical ball is [2004] the following is true for r1, r1and r3? [2008]
(a) inversely proportional to both radius ‘R’ and velocity ‘v’
(b) directly proportional to both radius ‘R’ and velocity ‘v’ (a) r3 < r1 < r2 r1

(c) directly proportional to ‘R’ but inversely proportional


to ‘v’ (b) r1 > r3 > r2 r3
(d) inversely proportional to ‘R’ but directly proportional
to velocity ‘v’ (c) r1 < r2 < r3
4. If two soap bubbles of different radii are connected by a
tube. [2004] (d) r1 < r3 < r2
(a) air flows from the smaller bubble to the bigger 11. A capillary tube (A) is dipped in water. Another identical
(b) air flows from bigger bubble to the smaller bubble till tube (B) is dipped in a soap-water solution. Which of the
the sizes are interchanged following shows the relative nature of the liquid columns in
(c) air flows from the bigger bubble to the smaller bubble the two tubes? [2008]
till the sizes become equal
(d) there is no flow of air. A B
5. If ‘S’ is stress and ‘Y’ is young’s modulus of material of a
wire, the energy stored in the wire per unit volume is
2
S
(a) (b) 2S 2Y [2005]
(a)
2Y
S 2Y
(c) (d)
2Y S2
6. A 20 cm long capillary tube is dipped in water. The water A B
rises up to 8 cm. If the entire arrangement is put in a freely
falling elevator the length of water column in the capillary
tube will be [2005] (b)
(a) 10 cm (b) 8 cm (c) 20 cm (d) 4 cm
7. A wire elongates by l mm when a load W is hanged from it.
If the wire goes over a pulley and two weights W each are A B
hung at the two ends, the elongation of the wire will be
(in mm) [2006]
(a) l (b) 2l (c) zero (d)l/2 (c)
8. If the terminal speed of a sphere of gold (density = 19.5 kg/m )
3

is 0.2 m/s in a viscous liquid (density = 1.5 kg/m3), find the


terminal speed of a sphere of silver (density = 10.5 kg/m3) of
the same size in the same liquid [2006] A B
(a) 0.4 m/s (b) 0.133 m/s (c) 0.1 m/s (d)0.2 m/s
9. A spherical solid ball of volume V is made of a material of (d)
density r1. It is falling through a liquid of density r2 (r2<
r1). Assume that the liquid applies a viscous force on the
ball that is proportional to the square of its speed v, i.e.,
Fviscous = –kv2 (k > 0). The terminal speed of the ball is
EBD_7036
P-58 Topic-wise Solved Papers - PHYSICS
12. Two wires are made of the same material and have the same 18. A uniform cylinder of length L and mass M having cross-
volume. However wire 1 has cross-sectional area A and wire sectional area A is suspended, with its length vertical,
2 has cross-sectional area 3A. If the length of wire 1 increases from a fixed point by a massless spring such that it is
by Dx on applying force F, how much force is needed to half submerged in a liquid of density s at equilibrium
stretch wire 2 by the same amount? [2009] position. The extension x0 of the spring when it is in
(a) 4 F (b) 6 F (c) 9 F (d)F equilibrium is:
13. A ball is made of a material of density r where [ JEE Main 2013 ]
roil < r < rwater with roil and rwater representing the Mg Mg æ LAs ö
densities of oil and water, respectively. The oil and water are (a) (b) ç1 – ÷
k k è M ø
immiscible. If the above ball is in equilibrium in a mixture of
this oil and water, which of the following pictures represents Mg æ LAs ö Mg æ LAs ö
its equilibrium position ? [2010] (c) ç1 – ÷ (d) ç1 + ÷
k è 2M ø k è M ø
19. Assume that a drop of liquid evaporates by decrease in its
surface energy, so that its temperature remains
unchanged.What should be the minimum radius of the drop
water oil
for this to be possible? The surface tension is T, density of
liquid is r and L is its latent heat of vaporization.
(a) (b) [JEE Main 2013 ]
oil
water (a) rL/T (b) T / rL
(c) T/rL (d) 2T/rL
20. On heating water, bubbles being formed at the bottom of the
vessel detach and rise. Take the bubbles to be spheres of
radius R and making a circular contact of radius r with the
water oil bottom of the vessel. If r << R and the surface tension of
water is T, value of r just before bubbles detach is:
(density of water is r w ) [JEE Main 2014]
(c) (d)
oil
water

14. Two identical charged spheres are suspended by strings of


equal lengths. The strings make an angle of 30° with each
other. When suspended in a liquid of density 0.8g cm–3, the
angle remains the same. If density of the material of the
sphere is 1.6 g cm–3 , the dielectric constant of the liquid is
(a) 4 (b) 3 [2010]
(c) 2 (d) 1
R
15. Work done in increasing the size of a soap bubble from a
radius of 3 cm to 5 cm is nearly (Surface tension of soap
solution = 0.03 Nm–1) [2011]
(a) 0.2 p mJ (b) 2p mJ 2r
(c) 0.4p mJ (d) 4p mJ
16. Water is flowing continuously from a tap having an internal rw g rw g
2 2
diameter 8 × 10–3 m. The water velocity as it leaves the tap is (a) R (b) R
0.4 ms–1. The diameter of the water stream at a distance 3T 6T
2 × 10–1 m below the tap is close to: [2011]
rw g 2 3r w g
(a) 7.5 × 10–3 m (b) 9.6 × 10–3 m (c) R
2
(d) R
(c) 3.6 × 10–3 m (d) 5.0 × 10–3 m T T
17. A thin liquid film formed between a U-shaped wire and a light 21. An open glass tube is immersed in mercury in such a way
slider supports a weight of 1.5 × 10–2 N (see figure). The that a length of 8 cm extends above the mercury level. The
length of the slider is 30 cm and its weight negligible. The open end of the tube is then closed and sealed and the tube
surface tension of the liquid film is [2012] is raised vertically up by additional 46 cm. What will be length
(a) 0.0125 Nm–1 of the air column above mercury in the tube now?
(b) 0.1 Nm–1 (Atmospheric pressure = 76 cm of Hg) [JEE Main 2014]
(c) 0.05 Nm–1 FILM (a) 16 cm (b) 22 cm
(d) 0.025 Nm –1 (c) 38 cm (d) 6 cm

W
Heat & Thermodynamics
CHAPTER

9 and Gases
Section-A JEE Advanced/ IIT-JEE
A Fill in the Blanks 9. A container of volume 1m3 is divided into two equal parts
by a partition. One part has an ideal gas at 300K and the
1. One mole of a mono-atomic ideal gas is mixed with one mole other part is vacuum. The whole system is thermally isolated
of a diatomic ideal gas. The molar specific heat of the mixture from the surroundings. When the partition is removed, the
at constant volume is ....... (1984- 2 Marks)
gas expands to occupy the whole volume. Its temperature
2. The variation of temperature of a material as heat is given to
will now be....... (1993-1 Mark)
it at a constant rate is shown in the figure. The material is in
solid state at the point O. The state of the material at the 10. An ideal gas with pressure P, volume V and temperature T is
point P is ........ (1985 - 2 Marks) expanded isothermally to a volume 2V and a final pressure
Pi. If the same gas is expanded adiabatically to a volume 2V,
the final pressure is Pa. The ratio of the specific heats of the
C
D Pa
T gas is 1.67 . The ratio P is ..... (1994 - 2 Marks)
A P 1
B
11. Two metal cubes A and B of same size are arranged as shown
O in Figure. The extreme ends of the combination are maintained
HEAT ADDED
at the indicated temperatures. The arrangement is thermally
3. During an experiment, an ideal gas is found to obey an
additional law VP2 =constant, The gas is initially at a insulated. The coefficients of thermal conductivity of A and
temperature T, and volume V. When it expands to a volume B are 300 W/m °C and 200 W/m°C, respectively. After steady
2V, the temperature becomes.......... (1987 - 2 Marks) state is reached the temperature t of the interface will be .....
4. 300 grams of water at 25° C is added to 100 grams of ice at (1996 - 2 Marks)
0°C. The final temperature of the mixture is .....°C.
(1989 - 2 Marks)
5. The earth receives at its surface radiation from the sun at
the rate of 1400 W m–2. The distance of the centre of the sun
from the surface of the earth is 1.5 × 1011 m and the radius of
the sun is 7 × 108 m. Treating the sun as a black body, it
follows from the above data that its surface temperature
is...........K. (1989 - 2 Marks)
6. A solid copper sphere (density r and specific heat c) of 12. A ring shaped tube contains two ideal gases with equal
radius r at an initial temperature 200 K is suspended inside masses and relative molar masses M1= 32 and M2 = 28. The
a chamber whose walls are at almost 0K. The time required gases are separated by one fixed partition and another
for the temperature of the sphere to drop to 100K is movable stopper S which can move freely without friction
.................... (1991 - 2 Marks) inside the ring. The angle a as shown in the figure is ...........
7. A point source of heat of power P is placed at the centre of degrees. (1997 - 2 Marks)
a spherical shell of mean radius R. The material of the shell
has thermal conductivity K. If the temperature difference M1 M2
between the outer and inner surface of the shell in not to
exceed T, the thickness of the shell should not be less than
............ (1991 - 1 Mark)
a
8. A substance of mass M kg requires a power input of P watts
to remain in the molten state at its melting point. When the
power source is turned off, the sample completely solidifies
S
in time t seconds. The latent heat of fusion of the substance
is .................... (1992 - 1 Mark)
EBD_7036
P-60 Topic-wise Solved Papers - PHYSICS
13. Earth receives 1400 W/m 2 of solar power. If all the solar 2. A metal ball immersed in alcohol weighs W1 at 0°C and W2
energy falling on a lens of area 0.2 m2 is focused on to a at 50°C. The coefficient of expansion of cubical the metal is
block of ice of mass 280 grams, the time taken to melt the ice less than that of the alcohal. Assuming that the density of
will be... minutes. (Latent heat of fusion of ice = 3.3 × 10 5 J/ the metal is large compared to that of alcohol, it can be
kg.) (1997 - 2 Marks) shown that (1980)
(a) W1 > W2 (b) W1 = W2
B True/False (c) W1 < W2 (d) None of these
3. A wall has two layers A and B, each made of different
1. The root-mean square speeds of the molecules of different material. Both the layers have the same thickness. The
ideal gases, maintained at the same temperature are the same. thermal conductivity of the meterial of A is twice that of B.
(1981- 2 Marks) Under thermal equilibrium, the temperature difference
2. The volume V versus temperature T graphs for a certain across the wall is 36°C. The temperature difference across
amount of a perfect gas at two pressure p1 and p2 are as the layer A is ( 1980)
shown in Fig. It follows from the graphs that p1 is greater (a) 6°C (b) 12°C (c) 18°C (d) 24°C
than p2. (1982 - 2 Marks) 4. An ideal monatomic gas is taken round the cycle ABCDA as
V p1
shown in the P – V diagram (see Fig.). The work done
during the cycle is P (1983 - 1 Mark)
p2 (a) PV 2P, V 2P, 2V
(b) 2 PV B C
1
(c) PV
T 2 A D
P, V P,2V
(d) zero V
3. Two different gases at the same temperature have equal
root mean square velocities. (1982 - 2 Marks) æ 5ö
5. If one mole of a monatomic gas ç g = ÷ is mixed with one
7 è 3ø
4. The ratio of the velocity of sound in Hydrogen gas ( g = )
5 æ 7ö
mole of a diatomic gas ç g = ÷ , the value of g for mixture
5 21 è 5ø
to that in Helium gas ( g = ) at the same temperature is . is (1988 - 1 Mark)
3 5
(1983 - 2 Marks) (a) 1.40 (b) 1.50 (c) 1.53 (d) 3.07
5. The curves A and B in the figure shown P-V graphs for an 6. From the following statements concerning ideal gas at any
isothermal and an adiabatic process for an ideal gas. The given temperature T, select the correct one(s) (1995S)
isothermal process is represented by the curve A. (a) The coefficient of volume expansion at constant
(1985 - 3 Marks) pressure is the same for all ideal gases
(b) The average translational kinetic energy per molecule
of oxygen gas is 3kT, k being Boltzmann constant
P A (c) The mean-free path of molecules increases with
increases in the pressure
B (d) In a gaseous mixture, the average translational kinetic
energy of the molecules of each component is different
V
7. Three rods of identical cross-sectional area and made from
6. At a given temperature, the specific heat of a gas at constant the same metal from the sides of an isosceles traingle ABC,
pressure is always greater than its specific heat at constant right-angled at B. The points A and B are maintained at
volume. (1987 - 2 Marks)
7. The root mean square (rms) speed of oxygen molecules (O2) temperatures T and ( 2 ) T respectively. In the steady state,
at a certain temperature T (degree absolute) is V. If the the temperature of the point C is Tc. Assuming that only
temperature is doubled and oxygen gas dissociates into heat conduction takes place, Tc / T is (1995S)
atomic oxygen, the rms speed remains unchanged. 1 3
(1987 - 2 Marks) (a) (b)
2( 2 - 1) 2 +1
8. Two spheres of the same meterial have radii 1 m and 4 m and
temperatures 4000K and 2000K respectively. The energy 1 1
(c) (d)
radiated per second by the first sphere is greater than that 3 ( 2 - 1) 2 +1
by the second. (1988 - 2 Marks) 8. Two metallic spheres S1 and S2 are made of the same material
and have got identical surface finish. The mass of S1 is
C MCQs with One Correct Answer thrice that of S2. Both the spheres are heated to the same
high temperature and placed in the same room having lower
1. A constant volume gas thermometer works on (1980) temperature but are thermally insulated from each other. The
(a) The Principle of Archimedes ratio of the initial rate of cooling of S1 to that of S2 is (1995S)
(b) Boyle’s Law 1
1 1 3 æ 1 ö3
(c) Pascal’s Law (a) (b) (c) (d) ç ÷
(d) Charle’s Law 3 3 1 è 3ø
Heat & Thermodynamics and Gases P-61

9. The average translational kinetic energy of O2 (relative molar Y Y


mass 32) molecules at a particular temperature is 0.048 eV.
The translational kinetic energy of N2 (relative molar mass
28) molecules in eV at the same temperature is (1997 - 1 Mark) (a) d (b) d
o X o X
(a) 0.0015 (b) 0.003 (c) 0.048 (d) 0.768 T T + DT T T + DT
10. A vessel contains 1 mole of O2 gas (relative molar mass 32) Temperature K Temperature K
at a temperature T. The pressure of the gas is P. An identical Y
vessel containing one mole of He gas (relative molar mass Y
4) at a temperature 2T has a pressure of (1997 - 1 Mark)
(a) P/8 (b) P (c) 2P (d) 8P d
(c) o X (d) d
11. A spherical black body with a radius of 12 cm radiates 450 T T + DT o X
W power at 500 K. if the radius were halved and the Temperature K T T + DT
temperature doubled, the power radiated in watt would be Temperature K
(1997 - 1 Mark) 18. Starting with the same initial conditions, an ideal gas expands
(a) 225 (b) 450 (c) 900 (d) 1800 from volume V1 to V2 in three different ways. The work done
12. A closed compartment containing gas is moving with some by the gas is W1 if the process is purely isothermal, W2 if
acceleration in horizontal direction. Neglect effect of gravity. purely isobaric and W3 if purely adiabatic. Then (2000S)
Then the pressure in the compartment is (1999S - 2 Marks) y
(a) W2 > W1 > W3
(a) same everywhere (b) lower in the front side Isobaric
(c) lower in the rear side (d) lower in the upper side (b) W2 > W3 > W1 W2
Isothermal
13. A gas mixture consists of 2 moles of oxygen and 4 moles of (c) W1 > W2 > W3 P W1
Adiabatic
argon at temperature T. Neglecting all vibrational modes, W3

the total internal energy of the system is(1999S - 2 Marks) (d) W1 > W3 > W2 x
o V1 V2
(a) 4 RT (b) 15 RT (c) 9 RT (d) 11 RT 19. The plots of intensity versus wavelength for three black
14. The ratio of the speed of sound in nitrogen gas to that in bodies at temperature T1, T2 and T3 respectively are as
helium gas, at 300 K is (1999S - 2 Marks) shown. Their temperatures are such that (2000S )
(a) (2 / 7 (b) ( 3 ) / 5 (d) ( 6 ) / 5
(1 / 7) (c) Y
T3
15. A monatomic ideal gas, initially at temperature T1, is enclosed T2
in a cylinder fitted with a frictionless piston. The gas is T1
allowed to expand adiabatically to a temperature T2 by I
releasing the piston suddenly. If L1 and L2 are the length of
the gas column before and after expansion respectively, then o x
T1 l
(a) T1 > T2 > T3 (b) T1 > T3 > T2
T2 is given by (2000S )
(c) T2 > T3 > T1 (d) T3 > T2 > T1
2/3 2/3 20. Three rods made of same material and having the same
æ L1 ö L1 L2 æ L2 ö cross-section have been joined as shown in the figure. Each
(a) çè L ÷ø (b) L2 (c) L1 (d) ç ÷ rod is of the same length. The left and right ends are kept at
2 èL ø
1
0oC and 90oC respectively. The temperature of the junction
16. A block of ice at –10°C is slowly heated and converted to of the three rods will be (2001S)
90 oC
steam at 100°C. Which of the following curves represents (a) 45oC
the phenomenon qualitatively ? (2000S) (b) 60oC 0 oC
(c) 30oC o
(d) 20oC 90 C
T T 21. In a given process on an ideal gas, dW = 0 and dQ < 0. Then
(a) (b) for the gas (2001S)
Heat supplied Heat supplied (a) the temperature will decrease
(b) the volume will increase
(c) the pressure will remain constant
T T (d) the temperature will increase
(c) (d) 22. P-V plots for two gases during adiabatic processes are
shown in the figure. Plots 1 and 2 should correspond
Heat supplied Heat supplied respectively to (2001S)
P
17. An ideal gas is initially at temperature T and volume V. Its (a) He and O2
volume is increased by DV due to an increase in temperature
(b) O2 and He
DV 1
DT, pressure remaining constant. The quantity d = (c) He and Ar
V DT 2
varies with temperature as (2000S) (d) O2 and N2
V
EBD_7036
P-62 Topic-wise Solved Papers - PHYSICS
23. When a block of iron floats in mercury at 0oC, fraction k1 of 28. Two rods, one of aluminum and the other made of steel,
its volume is submerged, while at the temperature 60 oC, a having initial length l1 and l2 are connected together to
fraction k2 is seen to be submerged. If the coefficient of form a single rod of length l1 + l2. The coefficients of linear
volume expansion of iron is gFe and that of mercury is gHg, expansion for aluminum and steel are aa and as and
then the ratio k1/k2 can be expressed as (2001S) respectively. If the length of each rod increases by the same
amount when their temperature are raised by t0C, then find
1 + 60g Fe 1 - 60 g Fe the ratiol1/(l1 + l2) (2003S)
(a) (b)
1 + 60g hg 1 + 60g Hg (a) a s/ a a (b) a a/ a s
(c) a s/( a a + a s ) (d) a a/( a a + a s )
1 + 60 g Fe 1 + 60 g Hg 29. The PT diagram for an ideal gas is shown in the figure,
(c) (d) where AC is an adiabatic process, find the corresponding
1 - 60 g Hg 1 + 60 g Fe PV diagram. (2003S)
24. An ideal gas is taken through the cycle A ® B ® C ® A ,
as shown in the figure. If the net heat supplied to the gas in
A
the cycle is 5J, the work done by the gas in the process P
C ® A is (2002S )
(a) –5 J C B
2
3 B C
V(m )
(b) –10 J
1 T
(c) –15 J A
A
A
(d) –20 J P(N/m2) 10 P
25. Which of the following graphs correctly represents the P

dV / dP (a) C (b)
B
variation of b = - with P for an ideal gas at constant C B
V
V V
temperature ? (2002S )
A A
P P

(a) (b) (c) (d)


C B
B C
V V

P 30. 2 kg of ice at –200C is mixed with 5kg of water at 200C in an


P insulating vessel having a negligible heat capacity. Calculate
the final mass of water remaining in the container. It is given
that the specific heats of water & ice are 1kcal/kg/0C & 0.5
(c) (d) kcal/kg/0C while the latent heat of fusion of ice is 80 kcal/kg
(a) 7 kg (b) 6 kg (2003S)
(c) 4 kg (d) 2 kg
31. Three discs A, B and C having radii 2, 4, and 6 cm respectively
P P are coated with carbon black. Wavelength for maximum
26. An ideal Black-body at room temperature is thrown into a intensity for the three discs are 300, 400 and 500 nm
furnace. It is observed that (2002S ) respectively. If QA, QB and QC are power emitted by A, B and
(a) initially it is the darkest body and at later times the D respectively, then (2004S)
brightest (a) QA will be maximum (b) QB will be maximum
(b) it is the darkest body at all times (c) QC will be maximum (d) QA = QB = QC
(c) it cannot be distinguished at all times 32. If liquefied oxygen at 1 atmospheric pressure is heated from
(d) initially it is the darkest body and at later times it cannot 50 k to 300 k by supplying heat at constant rate. The graph
be distinguished of temperature vs time will be (2004S)
27. The graph, shown in the adjacent diagram, represents the
variation of temperature (T) of two bodies, x and y having
same surface area, with time (t) due to the emission of T
radiation. Find the correct relation between the emissivity (a) (b) T
and absorptivity power of the two bodies (2003S)
t t
(a) Ex > Ey & ax < ay
(b) Ex < Ey & ax > ay T y
x
(c) Ex > Ey & ax > ay
(c) T (d) T
(d) Ex < Ey & ax < ay
t t t
Heat & Thermodynamics and Gases P-63

33. Two identical rods are connected between two containers through a diameter of the beaker. The force on water on one
one of them is at 100ºC and another is at 0ºC. If rods are side of this section by water on the other side of this section
connected in parallel then the rate of melting of ice is q1 gm/ has magnitude (2007)
sec. If they are connected in series then the rate is q2. The (a) | 2P0Rh + pR2rgh – 2RT |
2R
ratio q2/ q1 is (2004S)
B
(a) 2 (b) 4 (c) 1/2 (d) 1/4 (b) | 2P0Rh + R rgh2 – 2RT |
34. An ideal gas is initially at P1, V1 is expanded to P2, V2 and
A
then compressed adiabatically to the same volume V1 and (c) | P0pR2 + Rrgh2 – 2RT | h
pressure P3. If W is the net work done by the gas in complete C

process which of the following is true (2004S) (d) | P0 pR2 + Rrgh2 + 2RT |
(a) W > 0 ; P3 > P1 (b) W < 0 ; P3 > P1 D
(c) W > 0 ; P3 < P1 (d) W < 0 ; P3 < P1 41. An ideal gas is expanding such that PT2 = constant. The
35. Variation of radiant energy emitted by sun, filament of coefficient of volume expansion of the gas is – (2008)
tungsten lamp and welding arc as a function of its (a) 1/T (b) 2/T (c) 3/T (d) 4/T
wavelength is shown in figure. Which of the following option 42. A real gas behaves like an ideal gas if its (2010)
is the correct match ? (2005S) (a) pressure and temperature are both high
El (b) pressure and temperature are both low
(c) pressure is high and temperature is low
(d) pressure is low and temperature is high
43. 5.6 liter of helium gas at STP is adiabatically compressed to
T3
0.7 liter. Taking the initial temperature to be T1, the work
done in the process is (2011)
9 3 15 9
T2 (a) RT1 (b) RT (c) RT1 (d) RT
8 2 1 8 2 1
T1 44. A mixture of 2 moles of helium gas (atomic mass = 4 amu)
and 1 mole of argon gas (atomic mass = 40 amu) is kept at
l 300 K in a container. The ratio of the rms speeds
(a) Sun-T3, tungsten filament - T1, welding arc - T2 æ vrms (helium) ö
(b) Sun-T2, tungsten filament - T1, welding arc - T3 çè v (argon) ÷ø is (2012)
rms
(c) Sun-T3, tungsten filament - T2, welding arc - T1 (a) 0.32 (b) 0.45 (c) 2.24 (d) 3.16
(d) Sun-T1, tungsten filament - T2, welding arc - T3 45. Two moles of ideal helium gas are in a rubber balloon at
36. In which of the following process, convection does not 30°C. The balloon is fully expandable and can be assumed
take place primarily (2005S) to require no energy in its expansion. The temperature of
(a) sea and land breeze the gas in the balloon is slowly changed to 35°C. The amount
(b) boiling of water of heat required in raising the temperature is nearly (take R
(c) heating air around a furnace = 8.31 J/mol.K) (2012)
(d) warming of glass of bulb due to filament (a) 62 J (b) 104 J (c) 124 J (d) 208 J
37. A spherical body of area A and emissivity e = 0.6 is kept 46. Two rectangular blocks, having identical dimensions, can
inside a perfectly black body. Total heat radiated by the be arranged either in configuration-I or in configuration-II
body at temperature T (2005S) as shown in the figure. One of the blocks has thermal
(a) 0.4 AT4 (b) 0.8 AT4 conductivity k and the other 2k. The temperature difference
(c) 0.6 AT4 (d) 1.0 AT4 between the ends along the x-axis is the same in both the
38. Calorie is defined as the amount of heat required to raise configurations. It takes 9 s to transport a certain amount of
temperature of 1 g of water by 1oC and it is defined under heat from the hot end to the cold end in the configuration-I.
which of the following conditions? (2005S) The time to transport the same amount of heat in the
(a) From 14.5 oC to 15.5 oC at 760 mm of Hg configuration-II is (JEE Adv. 2013)
(b) From 98.5 oC to 99.5 oC at 760 mm of Hg
Configuration-I Configuration-II
(c) From 13.5 oC to 14.5 oC at 76 mm of Hg
(d) From 3.5 oC to 4.5oC at 76 mm of Hg
39. Water of volume 2 litre in a container is heated with a coil of
2k
1 kW at 27 oC. The lid of the container is open and energy
dissipates at rate of 160 J/s. In how much time temperature k 2k k
will rise from 27oC to 77oC [Given specific heat of water is x
4.2 kJ/kg] (2005S) (a) 2.0 s (b) 4.5 s (c) 3.0 s (d) 6.0 s
(a) 7 min (b) 6 min 2 s (c) 8 min 20 s (d) 14 min 47. Two non-reactive monoatomic ideal gases have their atomic
40. Water is filled up to a height h in a beaker of radius R as masses in the ratio 2 : 3. The ratio of their partial pressures,
shown in the figure. The density of water is r, the surface when enclosed in a vessel kept at a constant temperature, is
tension of water is T and the atmospheric pressure is P0. 4 : 3. The ratio of their densities is (JEE Adv. 2013)
Consider a vertical section ABCD of the water column (a) 1 : 4 (b) 1 : 2 (c) 6 : 9 (d) 8 : 9
EBD_7036
P-64 Topic-wise Solved Papers - PHYSICS
48. Parallel rays of light of intensity I = 912 Wm–2 are incident temperature of the same gas through the same range (30°C
on a spherical black body kept in surroundings of temperature to 35°C) at constant volume is : (1985 - 2 Marks)
300 K. Take Stefan-Boltzmann constant s = 5.7 × 10–8 (a) 30 (b) 50 (c) 70 (d) 90
Wm–2K–4 and assume that the energy exchange with the 3. Steam at 100°C is passed into 1.1 kg of water contained in a
surroundings is only through radiation. The final steady calorimeter of water equivalent 0.02 kg at 15°C till the
state temperature of the black body is close to temperature of the calorimeter and its contents rises to 80°C.
(JEE Adv. 2014) The mass of the steam condensed in kilogram is
(a) 330 K (b) 660 K (c) 990 K (d) 1550 K (1986 - 2 Marks)
49. A water cooler of storage capacity 120 litres can cool water (a) 0.130 (b) 0.065 (c) 0.260 (d) 0.135
at a constant rate of P watts. In a closed circulation system 4. A cylinder of radius R made of a material of thermal
(as shown schematically in the figure), the water from the conductivity K1 is surrounded by a cylindrical shell of inner
cooler is used to cool an external device that generates radius R and outer radius 2R made of a material of thermal
constantly 3 kW of heat (thermal load). The temperature of conductivity K2. The two ends of the combined system are
water fed into the device cannot exceed 30°C and the entire maintained at two different temperatures. There is no loss
stored 120 litres of water is initially cooled to 10°C. The of heat across the cylindrical surface and the system is in
entire system is thermally insulated. The minimum value of steady state. The effective thermal conductivity of the
P (in watts) for which the device can be operated for 3 hours system is (1988 - 2 Marks)
is (JEE Adv. 2016) (a) K1 + K2 (b) K1K2 / (K1 + K2)
(c) (K1 + 3K2) / 4 (d) (3K1 + 3K2) / 4
5. For an ideal gas : (1989 - 2 Marks)
(a) the change in internal energy in a constant pressure
process from temperature T1 to T2 is equal to nCv
(T2 – T1), where Cv is the molar specific heat at constant
volume and n the number of moles of the gas.
(Specific heat of water is 4.2 kJ kg–1K–1 and the density of (b) the change in internal energy of the gas and the work
water is 1000 kg m–3) done by the gas are equal in magnitude in an adiabatic
(a) 1600 (b) 2067 (c) 2533 (d) 3933 process.
50. A gas is enclosed in a cylinder with a movable frictionless (c) the internal energy does not change in an isothermal
piston. Its initial thermodynamic state at pressure Pi = 105 process.
Pa and volume Vi = 10–3 m3 changes to a final state at Pf = (1/ (d) no heat is added or removed in an adiabatic process.
32) × 105 Pa and Vf = 8 × 10–3 m3 in an adiabatic quasi-static 6. When an ideal diatomic gas is heated at constant pressure,
process, such that P3V5 = constant. Consider another the fraction of the heat energy supplied which increases the
thermodynamic process that brings the system from the same internal energy of the gas is (1990 - 2 Marks)
initial state to the same final state in two steps: an isobaric 2 3 3 5
expansion at Pi followed by an isochoric (isovolumetric) (a) (b) (c) (d)
5 5 7 7
process at volume Vf . The amount of heat supplied to the 7. Three closed vessels A, B and C are at the same temperature
system in the two-step process is approximately T and contain gases which obey the Maxwellian distribution
(JEE Adv. 2016) of velocities. Vessel A contain only O2, B only N2 and C a
(a) 112 J (b) 294 J (c) 588 J (d) 813 J mixture of equal quantities of O2 and N2. If the average
51. The ends Q and R of two thin wires, PQ and RS, are soldered speed of the O2 molecules in vessel A is v1 that of the N2
(joined) together. Initially each of the wires has a length of 1 molecules in vessel B is v2, the average speed of the O2
m at 10°C. Now the end P is maintained at 10°C, while the molecules in vessel C is (1992 - 2 Marks)
end S is heated and maintained at 400 °C. The system is 1
v1 + v2 3kT
thermally insulated from its surroundings. If the thermal (a) (b) v1 (c) (v1.v2 ) 2 (d)
2 M
conductivity of wire PQ is twice that of the wire RS and the where M is the mass of an oxygen molecule.
coefficient of linear thermal expansion of PQ is 1.2 × 10–5 8. An ideal gas is taken from the state A (pressure P, volume V)
K–1, the change in length of the wire PQ is (JEE Adv. 2016) to the state B (pressure P/2, volume 2V) along a straight line
(a) 0.78 mm (b) 0.90 mm (c) 1.56 mm (d) 2.34 mm path in the P-V diagram. Select the correct statement (s)
from the following: (1993-2 Marks)
D MCQs with One or More than One Correct (a) The work done by the gas in the process A to B exceeds
the work that would be done by it if the system were
1. At room temperature, the rms speed of the molecules of a taken from A to B along the isotherm.
certain diatomic gas is found to be 1930 m/s. The gas is (b) In the T-V diagram, the path AB becomes a part of a
(1984- 2 Marks) parabola
(a) H2 (b) F2 (c) O2 (d) Cl2 (c) In the P-T diagram, the path AB becomes a part of a
2. 70 calories of heat required to raise the temperature of hyperbola
2 moles of an ideal gas at constant pressure from 30°C to (d) In going from A to B, the temperature T of the gas first
35°C. The amount of heat required (in calories) to raise the increases to a maximum value and then decreases.
Heat & Thermodynamics and Gases P-65

9. Two bodies A and B ahave thermal emissivities of 0.01 and 17. Let v, v rms and vp respectively denote the mean speed. root
0.81 respectively. The outer surface areas of the two bodies
are the same. The two bodies emit total radiant power of the mean square speed, and most probable speed of the
same rate. The wavelength lB corresponding to maximum molecules in an ideal monatomic gas at absolute temperature
spectral radiancy in the radiation from B shifted from the T. The mass of a molecule is m. Then (1998S - 2 Marks)
wavelenth corresponding to maximum spectral radiancy in (a) no molecule can have a speed greater than 2 v rms
the radiation from A, by 1.00 mm. If the temperature of A is
5802 K : (1994 - 2 Marks) (b) no molecule can have a speed less than v p / 2
(a) the temperature of B is 1934 K
(c) v p < v < v rms
(b) l B = 1.5 mm
3
(c) the temperature of B is 11604 K (d) the average kinetic energy of a molecule is mv 2p .
(d) the temperature of B is 2901 K 4
10. The temperature of an ideal gas is increased from 120 K to 18. A vessel contains a mixture of one mole of oxygen and two
480 K. If at 120 K the root-mean-square velocity of the gas moles of nitrogen at 300 K. The ratio of the average rotational
molecules is v, at 480 K it becomes (1996 - 2 Marks) kinetic energy per O2 molecule to that per N2 molecule is
(a) 4v (b) 2v (c) v/2 (d) v/4 (a) 1 : 1 (1998S - 2 Marks)
11. A given quantity of a ideal gas is at pressure P and absolute (b) 1 : 2
temperature T. The isothermal bulk modulus of the gas is (c) 2 : 1
(1998S - 2 Marks) (d) depends on the moments of inertia of the two molecules
2 3 19. A black body of temperature T is inside chamber of T0
(a) P (b) P (c) P (d) 2P temperature initially. Sun rays are allowed to fall from a hole
3 2
12. Two cylinders A and B fitted with pistons contain equal in the top of chamber. If the temperature of black body (T)
amounts of an ideal diatomic gas at 300 K. The piston of A is and chamber (T0) remains constant, then (2006 - 5M, –1)
free to move, while that B is held fixed. The same amount of
heat is given to the gas in each cylinder. If the rise in T0
temperature of the gas in A is 30 K, then the rise in T
temperature of the gas in B is (1998S - 2 Marks)
(a) 30 K (b) 18 K (c) 50 K (d) 42 K
13. During the melting of a slab of ice at 273 K at atmospheric (a) Black body will absorb more radiation
pressure, (1998S - 2 Marks) (b) Black body will absorb less radiation
(a) positive work is done by the ice-water system on the (c) Black body emit more energy
atmosphere. (d) Black body emit energy equal to energy absorbed by it
(b) positive work is done on the ice- water system by the 20. Cv and Cp denote the molar specific heat capacities of a gas
atmosphere. at constant volume and constant pressure, respectively.
(c) the internal energy of the ice-water system increases. Then (2009)
(d) the internal energy of the ice-water system decreases. (a) Cp – Cv is larger for a diatomic ideal gas than for a
14. A blackbody is at a temperature of 2880 K. The energy of monoatomic ideal gas
radiation emitted by this object with wavelength between (b) Cp + Cv is larger for a diatomic ideal gas than for a
499 nm and 500 nm is U1, between 999 nm and 1000 nm is U2 monatomic ideal gas
and between 1499 nm and 1500nm is U3. The Wien constant (c) Cp / Cv is larger for a diatomic ideal gas than for a
b = 2.88×106nm K. Then (1998S - 2 Marks) monoatomic ideal gas
(a) U1 = 0 (b) U3 = 0 (c) U1 > U2 (d) U2 > U1 (d) Cp . Cv is larger for a diatomic ideal gas than for a
15. A bimetallic strip is formed out of two identical strips one of monoatomic ideal gas
copper and the other of brass. The coefficients of linear 21. The figure shows the P-V plot of an ideal gas taken through
expansion of the two metals are ac and aB. On heating, the a cycle ABCDA. The part ABC is a semi-circle and CDA is
temperature of the strip goes up by DT and the strip bends half of an ellipse. Then, (2009)
to form an arc of radius of curvature R. Then R is.
(a) proportional to DT (1999S - 3 Marks) P
A
(b) inversely proportional to DT 3
(c) proportional to |aB – aC|
(d) inversely proportional to |aB – aC| 2 B
D
16. Two identical containers A and B with frictionless pistons
contain the same ideal gas at the same temperature and the 1
same volume V. The mass of the gas in A is mA, and that in B C
is mB. The gas in each cylinder is now allowed to expand
0
isothermally to the same final volume 2V. The changes in 1 2 3 V
the pressure in A and B are found to be DP and 1.5 DP (a) the process during the path A ® B is isothermal
respectively. Then (1998S - 2 Marks) (b) heat flows out of the gas during the path B ® C ® D
(a) 4mA = 9mB (b) 2mA = 3mB (c) work done during the path A ® B ® C is zero
(c) 3mA = 2mB (d) 9mA = 4mB (d) positive work is done by the gas in the cycle ABCDA
EBD_7036
P-66 Topic-wise Solved Papers - PHYSICS
22. One mole of an ideal gas in initial state A undergoes a cyclic 25. A container of fixed volume has a mixture of one mole of
process ABCA, as shown in the figure. Its pressure at A is hydrogen and one mole of helium in equilibrium at
P0. Choose the correct option(s) from the following (2010) temperature T. Assuming the gases are ideal, the correct
V
B statement(s) is (are) (JEE Adv. 2015)
4V0
(a) The average energy per mole of the gas mixture is 2RT
(b) The ratio of speed of sound in the gas mixture to that in
V0 A
C helium gas is6/5
T0 T (c)
The ratio of the rms speed of helium atoms to that of
(a) Internal energies at A and B are the same hydrogen molecules is 1/2
(b) Work done by the gas in process AB is P0V0 ln 4 (d) The ratio of the rms speed of helium atoms to that of
1
P0 hydrogen molecules is
(c) Pressure at C is 2
4
T0 26. An ideal monoatomic gas is confined in a horizontal cylinder
(d) Temperature at C is by a spring loaded piston (as shown in the figure). Initially
4 the gas is at temperature T1, pressure P1 and volume V1 and
23. A composite block is made of slabs A, B, C, D and E of
the spring is in its relaxed state. The gas is then heated very
different thermal conductivities (given in terms of a constant
slowly to temperature T2, pressure P2 and volume V2. During
K and sizes (given in terms of length, L) as shown in the this process the piston moves out by a distance x. Ignoring
figure. All slabs are of same width. Heat ‘Q’ flows only from the friction between the piston and the cylinder, the correct
left to right through the blocks. Then in steady state statement(s) is (are) (JEE Adv. 2015)
(2011)

(a)
heat flow through A and E slabs are same.
(b)
heat flow through slab E is maximum. (a) If V2 = 2V1 and T2 = 3T1, then the energy stored in the
(c)
temperature difference across slab E is smallest. 1
(d)
heat flow through C = heat flow through B + heat flow spring is P1V1
4
through D. (b) If V2 = 2V1 and T2 = 3T1, then the change in internal
24. The figure below shows the variation of specific heat energy is 3P1V1
capacity (C) of a solid as a function of temperature (T). The (c) If V2 = 3V1 and T2 = 4T1, then the work done by the gas
temperature is increased continuously from 0 to 500 K at a 7
constant rate. Ignoring any volume change, the following is P1V1
3
statement(s) is (are) correct to a reasonable approximation. (d) If V2 = 3V1 and T2 = 4T1, then the heat supplied to the
(JEE Adv. 2013) 17
gas is PV
6 1 1
E Subjective Problems

C
1. A sinker of weight w0 has an apparent weight w1 when
weighed in a liquid at a temperature t1 and w2 when weight
in the same liquid at temperature t2. The coefficient of cubical
expansion of the material of sinker is b. What is the
coefficient of volume expansion of the liquid. (1978)
100 200 300 400 500 2. Three rods of material X and three rods of material Y are
(a) The rate at which heat is absorbed in the range 0 -100 K connected as shown in the figure. All the rods are of identical
varies linearly with temperature T. length and cross-sectional area. If the end A is maintained at
(b) Heat absorbed in increasing the temperature from 60°C and the junction E at 10°C. Calculate the temperature
0-100 K is less than the heat required for increasing the of the junctions B, C and D. The thermal conductivity of X
temperature from 400-500 K. is 0.92 cal/sec-cm-°C and that of Y is 0.46 cal/sec-cm-°C. (1978)
C
(c) There is no change in the rate of heat absorption in the X X
range 400-500 K. A B
X E
(d) The rate of heat absorption increases in the range Y
200-300 K. Y Y
D
Heat & Thermodynamics and Gases P-67

3. Given samples of 1 c.c. of hydrogen and 1c.c. of oxygen, (ii) The vessel is next thermally insulated and moved with
both at N.T.P. which sample has a larger number of molecules? a constant speed Vo. It is then suddenly stopped. The
(1979) process results in a rise of the temperature of the gas
4. A Solid material is supplied with heat at a constant rate. by 1°C. Calculate the speed Vo.
The temperature of the material is changing with the heat 11. The rectangular box shown in Fig has a partition which can
input as shown in the graph in figure. Study the graph slide without friction along the length of the box. Initially
carefully and answer the following questions : (1980) each of the two chambers of the box has one mole of a
mono-atomic ideal gas (g = 5/3) at a pressure P0, volume V0
and temperature T0. The chamber on the left is slowly heated
E by an electric heater. The walls of the box and the partition
TEMPERATURE

C D are thermally insulated. Heat loss through the lead wires of


A B the heater is negligible. The gas in the left chamber expands
pushing the partition until the final pressure in both
chambers becomes 243 P0 /32. Determine (i) the final
temperature of the gas in each chamber and (ii) the work
O HEAT INPUT
done by the gas in the right chamber. (1984- 8 Marks)
(i) What do the horizontal regions AB and CD represent?
(ii) If CD is equal to 2AB, what do you infer?
(iii) What does the slope of DE represent?
(iv) The slope of OA > the slope of BC. What does this
indicate?
5. A jar contains a gas and a few drops of water at T°K. The 12. Two glass bulbs of equal volume are connected by a narrow
pressure in the jar is 830 mm of Hg. The temperature of the tube and are filled with a gas at 0°C and a pressure of 76 cm
jar is reduced by 1%. The saturated vapour pressures of of mercury. One of the bulbs is then placed in melting ice
water at the two temperatures are 30 and 25 mm of Hg.(1980) and the other is placed in a water bath maintained at 62°C.
Calculate the new pressure in the jar. What is the new value of the pressure inside the bulbs? The
6. A cyclic process ABCA shown in the V-T diagram (fig) is volume of the connecting tube is negligible.
performed with a constant mass of an ideal gas. Show the (1985 - 6 Marks)
same process on a P-V diagram (1981- 4 Marks) 13. A thin tube of uniform cross-section is sealed at both ends.
It lies horizontally, the middle 5 cm containing mercury and
B the two equal end containing air at the same pressure P.
2
When the tube is held at an angle of 60° with the vertical
direction, the length of the air column above and below the
mercury column are 46cm and 44.5 cm respectively. Calculate
the pressure P in centimeters of mercury. (The temperature
1 of the system is kept at 30°C). (1986 - 6 Marks)
14. An ideal gas has a specific heat at constant pressure
1 2
(In the figure, CA is parallel to the V-axis and BC is parallel to 5R
CP = . The gas is kept in a closed vessel of volume 0.0083
the T-axis) 2
7. A lead bullet just melts when stopped by an obstacle. m3, at a temperature of 300 K and a pressure of 1.6 × 10 6 N/
Assuming that 25 per cent of the heat is absorbed by the m2. An amount of 2.49 × 104 Joules of heat energy is supplied
obstacle, find the velocity of the bullet if its initial temperature to the gas. Calculate the final temperature and pressure of
is 27°C. the gas. (1987 - 7 Marks)
(Melting point of lead = 327°C, specific heat of lead = 0.03 15. Two moles of helium gas (g = 5/3) are initially at temperature
calories /gm/°C, latent heat of fusion of lead = 6 calories / 27°C and occupy a volume of 20 litres. The gas is first
gm, J = 4.2 joules /calorie). (1981- 3 Marks) expanded at constant pressure until the volume is doubled.
8. Calculate the work done when one mole of a perfect gas is Then it undergoes an adiabatic change until the temperature
compressed adiabatically. The initial pressure and volume returns to its initial value. (1988 - 6 Marks)
of the gas are 105 N/m2 and 6 litres respectively. The final (i) Sketch the process on a p-V diagram.
volume of the gas is 2litre. Molar specific heat of the gas at (ii) What are the final volume and pressure of the gas ?
constant volume is 3R/2. (1982 - 8 Marks) (iii) What is the work done by the gas ?
9. A solid sphere of copper of radius R and a hollow sphere of 16. An ideal monatomic gas is confined in a cylinder by a spring-
the same material of inner radius r and outer radius A are loaded piston of cross-section 8.0 × 10–3 m2. Initially the
heated to the same temperature and allowed to cool in the gas is at 300 K and occupies a volume of 2.4 × 10–3 m3 and
same environment. Which of them starts cooling faster ? the spring is in its relaxed (unstretched, uncompressed)
(1982 - 2 Marks) state, fig. The gas is heated by a small electric heater until
10. One gram mole of oxygen at 27° and one atmospheric the piston moves out slowly by 0.1 m. Calculate the final
pressure is enclosed in a vessel. (1983 - 8 Marks) temperature of the gas and the heat supplied (in joules) by
(i) Assuming the molecules to be moving with Vrms, Find the heater. The force constant of the spring is 8000 N/m,
the number of collisions per second which the molecules atmospheric pressure is 1.0 × 105 Nm–2. The cylinder and
make with one square metre area of the vessel wall. the piston are thermally insulated. The piston is massless
EBD_7036
P-68 Topic-wise Solved Papers - PHYSICS
and there is no friction between the piston and the cylinder. 21. An ideal gas is taken through a cyclic thermodynamic
Neglect heat loss through lead wires of the heater. The heat process through four steps. The amounts of heat involved
capacity of the heater coil is negligible. Assume the spring in these steps are Q1 = 5960 J, Q2 = –5585 J, Q3 = –2980J and
to be massless. (1989 - 8 Mark)
Q4 = 3645J, respectively. The correspending quantities of
Open atmosphere
work involved are W1 = 2200 J, W2 = – 825 J,W3 = – 1100 J
and W4 respectively. (1994 - 6 Marks)
Rigid 1. Find the value of W4.
Heater support 2. What is the efficiency of the cycle
22. A closed container of volume 0.02 m 3 contains a mixture of
neon and argon gases, at a temperature of 27°C and pressure
of 1 × 105 Nm–2. The total mass of the mixture is 28 g. If the
17. An ideal gas having initial pressure P, volume V and molar masses of neon and argon are 20 and 40 g mol–1
temperature T is allowed to expand adiabatically until its
respectively, find the masses of the individual gases in the
T
volume becomes 5.66 V while its temperature falls to . container assuming them to be ideal (Universal gas constant
2 R = 8.314 J/mol – K). (1994 - 6 Marks)
(1990 - 7 Mark)
(i) How many degrees of freedom do the gas molecules 23. A gaseous mixture enclosed in a vessel of volume V consists
have? of one mole of a gas A with g (=Cp / Cv) = 5 / 3 and another
(ii) Obtain the work done by the gas during the expansion gas B with g = 7/5 at a certain temperature T. The relative
as a function of the initial pressure P and volume V. molar masses of the gases A and B are 4 and 32, respectively.
The gases A and B do not react with each other and are
18. Three moles of an ideal gas æç C p = 7 Rö÷ at pressure, PA
è 2 ø assumed to be ideal. The gaseous mixture follows the
and temperature TA is isothermally expanded to twice its equation PV19/13 = constant, in adiabatic processes.
(1995 - 10 Marks)
initial volume. It is then compressed at constant pressure to
its original volume. Finally gas is compressed at constant (a) Find the number of moles of the gas B in the gaseous
volume to its original pressure PA. (1991 - 4 + 4 Marks) mixture.
(a) Sketch P - V and P - T diagrams for the complete process. (b) Compute the speed of sound in the gaseous mixture at
(b) Calculate the net work done by the gas, and net heat T = 300 K.
supplied to the gas during the complete process. (c) If T is raised by 1K from 300 K, find the % change in
19. Two moles of helium gas undergo a cyclic process as shown the speed of sound in the gaseous mixture.
in Fig. Assuming the gas to be ideal, calculate the following (d) The mixture is compressed adiabatically to 1/5 of its
quantities in this process (1992 - 8 Marks)
initial volume V. Find the change in its adiabatic
A B compressibility in terms of the given quantities.
2 atm
24. At 27°C two moles of an ideal monoatomic gas occupy a
volume V. The gas expands adiabaticallly to a volume 2V.
1 atm C Calculate (i) the final temperature of the gas, (ii) change in
D
its internal energy, and (iii) the work done by the gas during
300 K 400 K this process. (1996 - 5 Marks)
(a) The net change in the heat energy
25. The temperature of 100g of water is to be raised from 24°C to
(b) The net work done
(c) The net change in internal energy 90°C by adding steam to it. Calculate the mass of the steam
20. One mole of a mono atomic ideal gas is taken through the required for this purpose. (1996 - 2 Marks)
cycle shown in Fig: (1993 - 4+4+2 Marks) 26. One mole of a diatomic ideal gas (g = 1.4) is taken through a
P cyclic process starting from point A. The process A ® B
A ® B : adiabatic expansion A
B
is an adiabatic compression, B ® C is isobaric expansion,
B ® C : cooling at constant volume
C ® D is an adiabatic expansion, and D ® A is isochoric.
C ® D : adiabatic compression The volume ratios are VA / VB = 16 and VC / VB = 2 and the
D C
D ® A : heating at constant volume.
temperature at A is TA = 300 K.Calculate the temperature of
V the gas at the points B and D and find the efficiency of the
The pressure and temperature at A, B, etc. are denoted by cycle. (1997 - 5 Marks)
PA , TA , PB , TB etc., respectively. Given that TA = 1000 K, 27. The apparatus shown in the figure consists of four glass
PB= (2/3)PA and PC = (1/ 3) PA , calculate the following columns connected by horizontal sections. The height of
quantities : two central columns B and C are 49 cm each. The two outer
(i) The work done by the gas in the process A ® B columns A and D are open to the atmosphere. A and C are
(ii) The heat lost by the gas in the process B ® C. maintained at a temperature of 95° C while the columns B
(iii) The temperature TD. [Given : (2 / 3)2/5 = 0.85] and D are maintained at 5°C. The height of the liquid in A
Heat & Thermodynamics and Gases P-69

and D measured from the base the are 52.8 cm and 51cm (a) the work done on the gas in the process AB and
respectively. Determine the coefficient of thermal expansion (b) the heat absorbed or released by the gas in each of the
of the liquid. (1997 - 5 Marks) processes.
Give answer in terms of the gas constant R.
32. An ice cube of mass 0.1 kg at 0oC is placed in an isolated
container which is at 227oC. The specific heat S of the
container varies with temperature T according to the
empirical relation S = A + BT, where A = 100 cal/kg-K and B =
A B C D
95°C 5°C 95°C 5°C 2 ´ 10-2 cal/kg-K2. If the final temperature of the container is
27oC, determine the mass of the container. (Latent heat of
28. One mole of an ideal monatomic gas is taken round the fusion of water = 8 ´ 104 cal/kg, Specific heat of water = 103
cyclic process ABCA as shown in Figure. Calculate cal/kg-K). (2001-5 Marks)
(1998 - 8 Marks) 33. A monoatomic ideal gas of two moles is taken through a
cyclic process starting from A as shown in figure. The volume
P
B VB V
3P0 ratios are = 2 and D = 4. If the temperature TA at A is
VA VA
27oC. (2001-10 Marks)

P0 V VD D C
A C

VB B
V0 2V0 V
(a) the work done by the gas. VA A
(b) the heat rejected by the gas in the path CA and the
O
heat absorbed by the gas in the path AB; TA TB
(c) the net heat absorbed by the gas in the path BC; T
Calculate,
(d) the maximum temperature attained by the gas during
(a) the temperature of the gas at point B,
the cycle.
29. A solid body X of heat capacity C is kept in an atmosphere (b) heat absorbed or released by the gas in each process,
whose temperature is TA = 300 K. At time t = 0 the temperature (c) the total work done by the gas during the complete cycle.
of X is T0 = 400 K. It cools according to Newton’s law of Express your answer in terms of the gas constant R.
cooling. At time t1, its temperature is found to be 350 K. 34. A cubical box of side 1 meter contains helium gas (atomic
(1998 - 8 Marks) weight 4) at a pressure of 100 N/m2. During an observation
At this time (t1), the body X is connected to a large box Y at time of 1 second, an atom travelling with the root-mean-
atmospheric temperature TA, through a conducting rod of square speed parallel to one of the edges of the cube, was
length L, cross-sectional area A and thermal conductivity K. found to make 500 hits with a particular wall, without any
The heat capacity of Y is so large that any variation in its 25
temperature may be neglected. The cross-sectional area A collision with other atoms. Take R = J/mol-K and
3
of the connecting rod is small compared to the surface area
of X. Find the temperature of X at time t = 3t1. k = 1.38 × 10-23 J/K (2002 - 5 Marks)
30. Two moles of an ideal monatomic gas, initially at pressure (a) Evaluate the temperature of the gas.
p1 and volume V1, undergo an adiabatic compression until (b) Evaluate the average kinetic energy per atom.
its volume is V2. Then the gas is given heat Q at constant (c) Evaluate the total mass of helium gas in the box.
volume V2. (1999 - 10 Marks) 35. An insulated container containing monoatomic gas of molar
(a) Sketch the complete process on a p – V diagram. mass m is moving with a velocity v0. If the container is
(b) Find the total work done by the gas, the total change in suddenly stopped, find the change in temperature.
its internal energy and the final temperature of the gas. (2003 - 2 Marks)
[Give your answer in terms of p1, V1, V2, Q and R] 36. Hot oil is circulated through an insulated container with a
31. Two moles of an ideal monatomic gas is taken through a wooden lid at the top whose conductivity
cycle ABCA as shown in the P-T diagram. During the K = 0.149 J/(m-ºC-sec), thickness t = 5 mm, emissivity = 0.6.
process AB, pressure and temperature of the gas vary such Temperature of the top of the lid is maintained at Tl = 127ºC.
that PT = Constant. If T1 = 300 K, calculate (2000 - 10 Marks) If the ambient temperature Ta = 27ºC. (2003 - 4 Marks)

Ta = 27ºC
Hot oil
EBD_7036
P-70 Topic-wise Solved Papers - PHYSICS
Calculate : surrounding has emissivity e. Also T2 = Ts + DT and Ts >> DT.
(a) rate of heat loss per unit area due to radiation from the If T1 – Ts µ DT, find the proportionality constant.
lid. (2004 - 4 Marks)
(b) temperature of the oil. Ts

(Given s = 17 ´ 10 -8 Wm -2 K -4 ) T2
3 T1 Insulation
37. A diatomic gas is enclosed in a vessel fitted with massless
movable piston. Area of cross section of vessel is 1 m 2.
Furnace
Initial height of the piston is 1 m (see the figure). The initial
40. A cubical block of co-efficient of linear expansion as is
temperature of the gas is 300 K. The temperature of the gas
submerged partially inside a liquid of co-efficient of volume
is increased to 400 K, keeping pressure constant, calculate
expansion gl. On increasing the temperature of the system
the new height of the piston. The piston is brought to its
by DT, the height of the cube inside the liquid remains
initial position with no heat exchange. Calculate the final
unchanged. Find the relation between as and gl.
temperature of the gas. You can leave answer in fraction.
(2004 - 4 Marks)
(2004 - 2 Marks)
41. A cylinder of mass 1 kg is given heat of 20,000J at
1m2 atmospheric pressure. If initially the temperature of cylinder
is 20°C, find (2005 - 6 Marks)
(a) final temperature of the cylinder.
1m (b) work done by the cylinder.
(c) change in internal energy of the cylinder
38. A small spherical body of radius r is falling under gravity in (Given that specific heat of cylinder = 400 J kg–1 °C–1,
a viscous medium. Due to friction the medium gets heated. coefficient of volume expansion = 9 × 10–5 °C–1, Atmospheric
How does the rate of heating depends on radius of body pressure = 105 N/m2 and Density of cylinder = 9000 kg/m3)
when it attains terminal velocity? (2004 - 2 Marks) 42. 0.05 kg steam at 373 K and 0.45 kg of ice at 253K are mixed in
39. A cylindrical rod of length l, thermal conductivity K and an insulated vessel. Find the equilibrium temperature of the
area of cross section A has one end in the furnace at mixture. Given, Lfusion = 80 cal/g = 336 J/g, Lvaporization = 540
temperature T1 and the other end in surrounding at cal/g = 2268 J/g, Sice = 2100 J/Kg K = 0.5 cal/gK and Swater =
temperature T 2 . Surface of the rod exposed to the 4200 J/Kg K = 1 cal/gK (2006 - 6M)

F Match the Following

DIRECTIONS (Q. No. 1-3) : Each question contains statements given in two columns, which have to p q r s t
be matched. The statements in Column-I are labelled A, B, C and D, while the statements in Column-
A p q r s t
II are labelled p, q, r and s. Any given statement in Column-I can have correct matching with ONE OR
B p q r s t
MORE statement(s) in Column-II. The appropriate bubbles corresponding to the answers to these
questions have to be darkened as illustrated in the following example : C p q r s t
If the correct matches are A-p, s and t; B-q and r; C-p and q; and D-s then the correct darkening of D p q r s t
bubbles will look like the given.
1. Heat given to process is positive, match the following option of Column I with the corresponding option of column II :
P(atm)
Column I Column II (2006 - 6M)
(A) JK (p) DW > 0 J
30
(B) KL (q) DQ < 0 20 M
(C) LM (r) DW < 0 10 L
K
(D) MJ (s) DQ > 0 3
V(m )
10 20

2. Column I contains a list of processes involving expansion of an ideal gas. Match this with Column II describing the thermodynamic
change during this process. Indicate your answer by darkening the appropriate bubbles of the 4 × 4 matrix given in the ORS.
Column I Column II (2008)
(A) An insulated container has two chambers (p) The temperature of the gas decreases
separated by a valve. Chamber I contains an ideal gas and the
Chamber II has vacuum. The valve is opened.

I II

Ideal gas vacuum


Heat & Thermodynamics and Gases P-71

(B) An ideal monoatomic gas expands to twice its remains original volume (q) The temperature of the gas increases or
such that its pressure P µ 1/V2 where V is the volume of the gas constant
(C) An ideal monoatomic gas expands to twice its original volume (r) The gas loses heat
such that its pressure P µ 1/V4/3 where V is its volume
(D) An ideal monoatomic gas expands such that its pressure P and (s) The gas gains heat
volume V follows the behaviour shown in the graph
P

V
V1 2V1
3. One mole of a monatomic gas is taken through a cycle ABCDA as shown in the P-V diagram. Column II give the characteristics
involved in the cycle. Match them with each of the processes given in Column I. (2011)

Column I Column II
(A) Process A ® B (p) Internal energy decreases
(B) Process B ® C (q) Internal energy increases
(C) Process C ® D (r) Heat is lost
(D) Process D ® A (s) Heat is gained
(t) Work is done on the gas

DIRECTIONS Q. No. 4 : Following question has matching lists. The codes for the list have choices (a), (b), (c) and (d) out of which
ONLY ONE is correct.
4. One mole of a monatomic ideal gas is taken along two cyclic processes E ® F ® G ® E and E ® F ® H ® E as shown in the PV
diagram. The processes involved are purely isochoric, isobaric, isothermal or adiabatic. (JEE Adv. 2013)
P
F
32 P0

P0 G
E H
V
V0
Match the paths in List I with the magnitudes of the work done in List II and select the correct answer using the codes given
below the lists.
List I List II
P. G ® E 1. 160 P0V0 ln2
Q. G ® H 2. 36 P0V0
R. F ® H 3. 24 P0V0
S. F ® G 4. 31 P0V0
Codes:
P Q R S
(a) 4 3 2 1
(b) 4 3 1 2
(c) 3 1 2 4
(d) 1 3 2 4
EBD_7036
P-72 Topic-wise Solved Papers - PHYSICS
PASSAGE – 2
G Comprehension Based Questions A small spherical monoatomic ideal gas bubble (g = 5/3) is trapped
PASSAGE – 1 inside a liquid of density r (see figure). Assume that the bubble
A fixed thermally conducting cylinder has a radius R and height does not exchange any heat with the liquid. The bubble contains
L0. The cylinder is open at its bottom and has a small hole at its n moles of gas. The temperature of the gas when the bubble is at
top. A piston of mass M is held at a distance L from the top the bottom is T0, the height of the liquid is H and the atmospheric
surface, as shown in the figure. The atmospheric pressure is P 0. pressure is P0 (Neglect surface tension). (2008)
2R P0
L
Liquid
L0 H
y

Piston 4. As the bubble moves upwards, besides the buoyancy force


the following forces are acting on it
1. The piston is now pulled out slowly and held at a distance
(a) Only the force of gravity
2L from the top. The pressure in the cylinder between its
top and the piston will then be (2007) (b) The force due to gravity and the force due to the
pressure of the liquid
P0
(a) P0 (b) (c) The force due to gravity, the force due to the pressure
2 of the liquid and the force due to viscosity of the liquid
P0 Mg P0 Mg (d) The force due to gravity and the force due to viscosity
(c) ∗ (d) , of the liquid
2 οR 2 2 οR 2
Therefore the pressure inside the cylinder is P0 throughout 5. When the gas bubble is at a height y from the bottom, its
the slow pulling process. temperature is –
2. While the piston is at a distance 2L from the top, the hole at 2/5
the top is sealed. The piston is then released, to a position æ P + rl gH ö
(a) T0 ç 0
where it can stay in equilibrium. In this condition, the è P0 + rl gy ÷ø
distance of the piston from the top is (2007)
æ 2P οR 2 ö÷ æ P οR 2 , Mg ÷ö
2/5
çç æ P + rl g (H - y) ö
(a) çç 2
0 ÷÷ (2L) (b) ççç 0 ÷÷ (2L) (b) T0 ç 0
è οR P ∗ Mg ÷÷
ø çè οR 2 P ÷÷
ø è P0 + rl gH ÷ø
0 0

æ P οR 2 ∗ Mg ö÷ æ P οR 2 ÷ö 3/5
çç 0 æ P + rl gH ö
(c) çç
÷÷ (2L)
÷ (d) ççç 0 ÷÷ (2L) (c) T0 ç 0
è οR P0 ÷ø
2 çè οR 2 P , Mg ÷÷ø è P + r gy ÷ø
0 l
0
3. The piston is taken completely out of the cylinder. The hole 3/5
at the top is sealed. A water tank is brought below the cylinder æ P + rl g (H - y) ö
(d) T0 ç 0
and put in a position so that the water surface in the tank is è P + r gH ÷ø
0 l
at the same level as the top of the cylinder as shown in the
figure. The density of the water is r. In equilibrium, the 6. The buoyancy force acting on the gas bubble is (Assume R
height H of the water column in the cylinder satisfies is the universal gas constant)
(2007)
(P0 + rl gH) 2 / 5
(a) rl nRgT0
(P0 + rl gy)7 / 5

rl nRgT0
L0 (b)
(P0 + rl gH) [P0 + rl g (H - y)]3/ 5
2/5

H
(P0 + rl gH)3 / 5
(c) rl nRgT0
(a) rg(L0 – H)2 + P0(L0 – H) + L0P0 = 0 (P0 + rl gy)8 / 5
(b) rg(L0 – H)2 – P0(L0 – H) – L0P0 = 0
rl nRgT0
(c) rg(L0 – H)2 + P0(L0 – H) – L0P0 = 0 (d)
(d) rg(L0 – H)2 – P0(L0 – H) + L0P0 = 0 (P0 + rl gH) [P0 + rlg (H - y)]2 / 5
3/ 5
Heat & Thermodynamics and Gases P-73

PASSAGE – 3 2. A piece of ice (heat capacity = 2100 J kg–1 °C–1 and latent heat
In the figure, a container is shown to have a movable (without = 3.36 × 105 J kg–1) of mass m grams is at –5°C at atmospheric
friction) piston on top. The container and the piston are all made pressure. It is given 420 J of heat so that the ice starts melting.
of perfectly insulated material allowing no heat transfer between Finally when the ice-water mixture is in equilibrium, it is found
outside and inside the container. The container is divided into that 1 gm of ice has melted. Assuming there is no other heat
two compartments by a rigid partition made of a thermally exchange in the process, the value of m is (2010)
conducting material that allows slow transfer of heat. The lower 3. Two spherical bodies A (radius 6 cm ) and B(radius 18 cm )
compartment of the container is filled with 2 moles of an ideal are at temperature T1 and T2, respectively. The maximum
monatomic gas at 700 K and the upper compartment is filled with intensity in the emission spectrum of A is at 500 nm and in
2 moles of an ideal diatomic gas at 400 K. The heat capacities per that of B is at 1500 nm. Considering them to be black bodies,
3 5 what will be the ratio of the rate of total energy radiated by
mole of an ideal monatomic gas are CV = R , CP = R , and A to that of B? (2010)
2 2
1
5 7 4. A diatomic ideal gas is compressed adiabatically to of
those for an ideal diatomic gas are CV = R , C P = R . 32
2 2
its initial volume. If the initial temperature of the gas is Ti (in
Kelvin) and the final temperature is a Ti , the value of a is
(2010)
5. Steel wire of lenght ‘L’ at 40°C is suspended from the ceiling
and then a mass ‘m’ is hung from its free end. The wire is
cooled down from 40°C to 30°C to regain its original length
‘L’. The coefficient of linear thermal expansion of the steel is
10–5 /° C, Young’s modulus of steel is 1011 N/m2 and radius
of the wire is 1 mm. Assume that L >>diameter of the wire.
7. Consider the partition to be rigidly fixed so that it does not Then the value of ‘m’ in kg is nearly (2011)
move. When equilibrium is achieved, the final temperature 6. A thermodynamic system is taken from an initial state i with
of the gases will be (JEE Adv. 2014) internal energy Ui = 100 J to the final state f along two different
(a) 550 K (b) 525 K paths iaf and ibf, as schematically shown in the figure. The
(c) 513 K (d) 490 K work done by the system along the paths af, ib and bf are
8. Now consider the partition to be free to move without friction Waf = 200 J, Wib = 50 J and Wbf =100 J respectively. The heat
so that the pressure of gases in both compartments is the supplied to the system along the path iaf, ib and bf are Qiaf,
same. The total work done by the gases till the time they Qib and Qbf respectively. If the internal energy of the system
achieve equilibrium will be (JEE Adv. 2014)
(a) 250 R (b) 200 R Qbf
(c) 100 R (d) – 100 R in the state b is Ub = 200 J and Qiaf = 500 J, The ratio is
Qib
H Assertion & Reason Type Questions (JEE Adv. 2014)

1. Statement-1 : The total translational kinetic energy of all


the molecules of a given mass of an ideal gas is 1.5 times a f
the product of its pressure and its volume. (2007) P
because
Statement-2 : The molecules of a gas collide with each other i
b
and the velocities of the molecules change due to the collision.
(a) Statement-1 is True, Statement-2 is True; Statement-2 V
is a correct explanation for Statement-1 7. Two spherical stars A and B emit blackbody radiation. The
(b) Statement-1 is True, Statement-2 is True; Statement-2 radius of A is 400 times that of B and A emits 104 times the
is NOT a correct explanation for Statement-1
(c) Statement-1 is True, Statement-2 is False æ lA ö
(d) Statement-2 is False, Statement-2 is True power emitted from B. The ratio ç l ÷ of their wavelengths
è Bø
I Integer Value Correct Type lA and lB at which the peaks occur in their respective
radiation curves is (JEE Adv. 2015)
1. A metal rod AB of length 10x has its one end A in ice at 0.°C, 8. A metal is heated in a furnace where a sensor is kept above
and the other end B in water at 100 °C. If a point P onthe rod the metal surface to read the power radiated (P) by the metal.
is maintained at 400 °C, then it is found that equal amounts The sensor has a scale that displays log 2, (P/P0 ), where P0
of water and ice evaporate and melt per unit time. The latent is a constant. When the metal surface is at a temperature of
heat of evaporation of water is 540 cal/g and latent heat of 487°C, the sensor shows a value 1. Assume that the
melting of ice is 80 cal/g. If the point P is at a distance of emissivity of the metallic surface remains constant. What is
l x from the ice end A, find the value l. the value displayed by the sensor when the temperature of
[Neglect any heat loss to the surrounding.] (2009) the metal surface is raised to 2767°C? (JEE Adv. 2016)
EBD_7036
P-74 Topic-wise Solved Papers - PHYSICS

Section-B JEE Main / AIEEE


1. Which statement is incorrect? [2002] 12. During an adiabatic process, the pressure of a gas is found
(a) all reversible cycles have same efficiency to be proportional to the cube of its absolute temperature.
(b) reversible cycle has more efficiency than an irreversible The ratio CP / CV for the gas is [2003]
one 4
(c) Carnot cycle is a reversible one (a) (b) 2
3
(d) Carnot cycle has the maximum efficiency in all cycles.
2. Heat given to a body which raises its temperature by 1°C is 5 3
(c) (d)
(a) water equivalent (b) thermal capacity [2002] 3 2
(c) specific heat (d) temperature gradient 13. Which of the following parameters does not characterize
3. Infrared radiation is detected by [2002] the thermodynamic state of matter? [2003]
(a) spectrometer (b) pyrometer (a) Temperature (b) Pressure
(c) nanometer (d) photometer (c) Work (d) Volume
4. Which of the following is more close to a black body? 14. A Carnot engine takes 3´ 10 6 cal. of heat from a reservoir
(a) black board paint (b) green leaves [2002]
at 627 o C , and gives it to a sink at 27 o C . The work done
(c) black holes (d) red roses
by the engine is [2003]
5. Cooking gas containers are kept in a lorry moving with
uniform speed. The temperature of the gas molecules inside (a) 4.2 ´ 10 6 J (b) 8.4 ´ 10 6 J
will [2002]
(c) 16.8 ´ 10 6 J (d) zero
(a) increase
15. The earth radiates in the infra-red region of the spectrum.
(b) decrease
The spectrum is correctly given by [2003]
(c) remain same (a) Rayleigh Jeans law
(d) decrease for some, while increase for others (b) Planck’s law of radiation
6. If mass-energy equivalence is taken into account, when (c) Stefan’s law of radiation
water is cooled to form ice, the mass of water should (d) Wien’s law
(a) increase [2002] 16. According to Newton’s law of cooling, the rate of cooling
(b) remain unchanged
(c) decrease of a body is proportional to ( Dq)n , where Dq is the
(d) first increase then decrease difference of the temperature of the body and the
7. At what temperature is the r.m.s velocity of a hydrogen surroundings, and n is equal to [2003]
molecule equal to that of an oxygen molecule at 47°C? (a) two (b) three (c) four (d) one
(a) 80 K (b) –73 K [2002] 17. One mole of ideal monatomic gas ( g = 5 / 3) is mixed with
(c) 3 K (d) 20 K. one mole of diatomic gas ( g = 7 / 5) . What is g for the
8. Even Carnot engine cannot give 100% efficiency because
we cannot [2002] mixture? g Denotes the ratio of specific heat at constant
(a) prevent radiation pressure, to that at constant volume [2004]
(b) find ideal sources (a) 35/23 (b) 23/15 (c) 3/2 (d) 4/3
(c) reach absolute zero temperature 18. If the temperature of the sun were to increase from T to 2T
and its radius from R to 2R, then the ratio of the radiant
(d) eliminate friction.
energy received on earth to what it was previously will be
9. 1 mole of a gas with g = 7/5 is mixed with 1 mole of a gas (a) 32 (b) 16 [2004]
with g = 5/3, then the value of g for the resulting mixture is (c) 4 (d) 64
(a) 7/5 (b) 2/5 [2002] 19. Which of the following statements is correct for any
(c) 24/16 (d) 12/7. thermodynamic system ? [2004]
10. Two spheres of the same material have radii 1 m and 4 m and (a) The change in entropy can never be zero
temperatures 4000 K and 2000 K respectively. The ratio of (b) Internal energy and entropy and state functions
(c) The internal energy changes in all processes
the energy radiated per second by the first sphere to that by
(d) The work done in an adiabatic process is always zero.
the second is [2002]
20. Two thermally insulated vessels 1 and 2 are filled with air at
(a) 1 : 1 (b) 16 : 1
(c) 4 : 1 (d) 1 : 9. temperatures (T1 , T2 ), volume (V1 ,V2 ) and pressure
11. “Heat cannot by itself flow from a body at lower temperature ( P1 , P2 ) respectively. If the valve joining the two vessels is
to a body at higher temperature” is a statement or opened, the temperature inside the vessel at equilibrium will
consequence of [2003] be [2004]
(a) second law of thermodynamics (a) T1T2 ( PV + P V ) /( PV T + P V T )
1 1 2 2 1 1 2 2 2 1
(b) conservation of momentum
(b) (T1 + T2 ) / 2
(c) conservation of mass
(c) T1 + T2
(d) first law of thermodynamics
(d) T1T2 ( PV 1 1 + P2V2 ) /( PV
1 1T1 + P2V2T2 )
Heat & Thermodynamics and Gases P-75

21. The temperature of the two outer surfaces of a composite 26. A gaseous mixture consists of 16 g of helium and 16 g of
slab, consisting of two materials having coefficients of Cp
thermal conductivity K and 2K and thickness x and 4x, oxygen. The ratio of the mixture is [2005]
Cv
respectively, are T2 and T1 (T2 > T1 ) . The rate of heat transfer (a) 1.62 (b) 1.59 (c) 1.54 (d) 1.4
27. Assuming the Sun to be a spherical body of radius R at a
æ A(T2 - T1 ) K ö temperature of TK, evaluate the total radiant powerd incident
through the slab, in a steady state is ç
è ÷ø f ,
x of Earth at a distance r from the Sun [2006]
with f equal to [2004] T4 T4
x 4x (a) 4pr02 R 2 s (b) pr02 R2 s
r2 r2
T4 T4
(c) r02 R 2 s (d) R2 s
T2 2K T1 4 pr 2 r2
K where r0 is the radius of the Earth and s is Stefan's constant.
28. Two rigid boxes containing different ideal gases are placed
on a table. Box A contains one mole of nitrogen at temperature
T0, while Box contains one mole of helium at temperature
2 1 1 æ 7ö
(a) (b) (c) 1 (d)
3 2 3 çè ÷ø T0 . The boxes are then put into thermal contact with
3
22. Which of the following is incorrect regarding the first law each other, and heat flows between them until the gases
of thermodynamics? [2005] reach a common final temperature (ignore the heat capacity
(a) It is a restatement of the principle of conservation of of boxes). Then, the final temperature of the gases, Tf in
energy terms of T0 is [2006]
(b) It is not applicable to any cyclic process 3 7
(c) It introduces the concept of the entropy (a) T f = T0 (b) T f = T0
(d) It introduces the concept of the internal energy 7 3
23. The figure shows a system of two concentric spheres of 3 5
(c) T f = T0 (d) T f = T0
radii r1 and r2 are kept at temperatures T1 and T2, respectively. 2 2
The radial rate of flow of heat in a substance between the 29. The work of 146 kJ is performed in order to compress one
two concentric spheres is proportional to [2005] kilo mole of gas adiabatically and in this process the
ær ö temperature of the gas increases by 7°C. The gas is
(a) In ç 2 ÷ (R = 8.3 J mol–1 K–1) [2006]
è r1 ø
(a) diatomic
( r2 - r1 ) r1 (b) triatomic
(b)
( r1 r2 ) T1 (c) a mixture of monoatomic and diatomic
(d) monoatomic
(c) ( r2 - r1 )
r2 T2 30. When a system is taken from state i to state f along the path
r1 r2 iaf, it is found that Q =50 cal and W = 20 cal. Along the path
(d)
( r2 - r1 ) ibf Q = 36 cal. W along the path ibf is [2007]
24. A system goes from A to B via two processes I and II as (a) 14 cal a f
shown in figure. If DU1 and DU2 are the changes in internal (b) 6 cal
energies in the processes I and II respectively, then [2005] (c) 16 cal
(d) 66 cal i b
(a) relation between DU1 and p
II 31. A Carnot engine, having an efficiency of h = 1/10 as heat
DU 2 can not be determined engine, is used as a refrigerator. If the work done on the
A B system is 10 J, the amount of energy absorbed from the
(b) DU1 = DU 2 I reservoir at lower temperature is [2007]
(c) DU 2 < DU1 (a) 100 J (b) 99 J (c) 90 J (d) 1 J
v 32. One end of a thermally insulated rod is kept at a temperature
(d) DU 2 > DU1
T1 and the other at T2. The rod is composed of two sections
25. The temperature-entropy diagram of a reversible engine
of length l1 and l2 and thermal conductivities K1 and K2
cycle is given in the figure. Its efficiency is [2005]
respectively. The temperature at the interface of the two
1
(a) T
section is [2007]
4 T1 l1 l2 T2
1
(b) 2T0
2
T0
2 K1 K2
(c)
3 ( K1l1T1 + K 2l2T2 ) ( K 2l2T1 + K1l1T2 )
S (a) (b)
1 S0 2S0 ( K1l1 + K 2l2 ) ( K1l1 + K 2l2 )
(d)
3 ( K 2l1T1 + K1l2T2 ) ( K1l2T1 + K 2l1T2 )
(c) (d)
( K 2l1 + K1l2 ) ( K1l2 + K 2l1 )
EBD_7036
P-76 Topic-wise Solved Papers - PHYSICS
33. If CP and CV denote the specific heats of nitrogen per unit 40. One kg of a diatomic gas is at a pressure of 8 × 104N/m2. The
mass at constant pressure and constant volume respectively, density of the gas is 4kg/m3. What is the energy of the gas
then [2007] due to its thermal motion? [2009]
(a) CP – CV = 28R (b) CP – CV = R/28 (a) 5 × 104 J (b) 6 × 104 J
(c) CP – CV = R/14 (d) CP – CV = R (c) 7 × 104 J (d) 3 × 104 J
34. The speed of sound in oxygen (O2) at a certain temperature
41. Statement 1 : The temperature dependence of resistance is
is 460 ms–1. The speed of sound in helium (He) at the same
usually given as R = Ro (1 + a Dt). The resistance of a wire
temperature will be (assume both gases to be ideal) [2008]
(a) 1421 ms–1 (b) 500 ms–1 changes from 100W to 150W when its temperature is
(c) 650 ms –1 (d) 330 ms–1 increased from 27°C to 227°C. This implies th at
35. An insulated container of gas has two chambers separated a = 2.5 × 10–3/°C.
by an insulating partition. One of the chambers has volume Statement 2 : R = Ro (1+ a Dt) is valid only when the change
V1 and contains ideal gas at pressure P1 and temperature in the temperature DT is small and
T1. The other chamber has volume V2 and contains ideal DR = (R – R0) << Ro. [2009]
gas at pressure P2 and temperature T2. If the partition is (a) Statement-1 is true, Statement-2 is true; Statement-2 is
removed without doing any work on the gas, the final the correct explanation of Statement-1.
equilibrium temperature of the gas in the container will be (b) Statement-1 is true, Statement-2 is true; Statement-2 is
1 1T1 + P2V2T2
PV not the correct explanation of Statement-1.
(a) 1 1 + P2V2 )
T1T2 ( PV
(b) [2008] (c) Statement-1 is false, Statement-2 is true.
1 1T2 + P2V2T1
PV 1 1 + P2V2
PV
(d) Statement-1 is true, Statement-2 is false.
PV1 1T2 + P2V2T1 1 1 + P2V2 )
T1T2 ( PV 42. A diatomic ideal gas is used in a Carnot engine as the working
(c) (d)
1 1 + P2V2
PV 1 1T1 + P2V2T2
PV substance. If during the adiabatic expansion part of the cycle
36. A long metallic bar is carrying heat from one of its ends to the volume of the gas increases from V to 32 V, the efficiency
the other end under steady–state. The variation of of the engine is [2010]
temperature q along the length x of the bar from its hot end (a) 0.5 (b) 0.75 (c) 0.99 (d) 0.25
is best described by which of the following figures? [2009] 43. A thermally insulated vessel contains an ideal gas of
q q molecular mass M and ratio of specific heats g. It is moving
with speed v and it's suddenly brought to rest. Assuming
no heat is lost to the surroundings, its temperature increases
(a) (b) by: [2011]
x x ( g - 1) Mv2 K gM 2v
(a) (b) K
2 gR 2R
q q
( g - 1) ( g - 1) 2
(c) Mv 2 K (d) 2( g + 1) R Mv K
2R
(c) (d)
44. Three perfect gases at absolute temperatures T1, T2 and T3
x x are mixed. The masses of molecules are m1, m2 and m3 and
DIRECTIONS for questions 37 to 39 : Questions are based on the number of molecules are n1, n2 and n3 respectively.
the following paragraph. Assuming no loss of energy, the final temperature of the
mixture is : [2011]
Two moles of helium gas are taken over the cycle ABCDA, as
shown in the P-T diagram. n1T1 + n2T2 + n3T3 n1T12 + n2T22 + n3T32
(a) n1 + n2 + n3 (b)
B
n1T1 + n2T2 + n3T3
5 A
2 × 10
n12T12 + n22T22 + n32T32 (T1 + T2 + T3 )
P (Pa) (c) n1T1 + n2T2 + n3T3
(d)
3
5
C
45. A Carnot engine operating between temperatures T1 and T2
1 × 10 D
T 1
300K 500K has efficiency . When T2 is lowered by 62 K its efficiency
6
37. Assuming the gas to be ideal the work done on the gas in
taking it from A to B is : 1
increases to . Then T1 and T2 are, respectively: [2011]
(a) 300 R (b) 400 R 3
(c) 500 R (d) 200 R (a) 372 K and 330 K (b) 330 K and 268 K
38. The work done on the gas in taking it from D to A is : (c) 310 K and 248 K (d) 372 K and 310 K
(a) + 414 R (b) – 690 R 46. 100g of water is heated from 30°C to 50°C. Ignoring the
(c) + 690 R (d) – 414 R slight expansion of the water, the change in its internal energy
39. The net work done on the gas in the cycle ABCDA is : is (specific heat of water is 4184 J/kg/K): [2011]
(a) 276 R (b) 1076 R (a) 8.4 kJ (b) 84 kJ
(c) 1904 R (d) zero (c) 2.1 kJ (d) 4.2 kJ
Heat & Thermodynamics and Gases P-77

47. A wooden wheel of radius R is made of two semicircular part The above p-v diagram represents the thermodynamic cycle
(see figure). The two parts are held together by a ring made of an engine, operatingwith an ideal monatomic gas. The
of a metal strip of cross sectional area S and length L. L is amount of heat, extracted from the source in a single cycle is
slightly less than 2pR. To fit the ring on the wheel, it is [JEE Main 2013]
heated so that its temperature rises by DT and it just steps
over the wheel. As it cools down to surrounding temperature, æ 13 ö æ 11 ö
it presses the semicircular parts together. If the coefficient
(a) p 0v 0 (b) ç ÷ p0 v0 (c) ç ÷ p0 v0 (d) 4p0v0
è2ø è2ø
of linear expansion of the metal is a, and its Young's modulus 52. If a piece of metal is heated to temperature q and then allowed
is Y, the force that one part of the wheel applies on the other to cool in a room which is at temperature q0, the graph between
part is : [2012] the temperature T of the metal and time t will be closest to
(a) 2pSY aDT [JEE Main 2013]
R
(b) SY aDT
(c) pSY aDT T T
q0
(d)
2SY aDT (a) (b)
48. Helium gas goes through a cycle ABCDA (consisting of O t O t

two isochoric and isobaric lines) as shown in figure


T T
Efficiency of this cycle is nearly : (Assume the gas to be q0 q0
(c) (d)
close to ideal gas) [2012]
B C t t
2P0 O O
(a) 15.4 %
53. The pressure that has to be applied to the ends of a steel wire
(b) 9.1 % P0 D of length 10 cm to keep its length constant when its
A
(c) 10.5% temperature is raised by 100ºC is:
(d) 12.5 % V0 2V0 (For steel Young’s modulus is 2 ´ 1011 Nm -2 and coefficient
49. A liquid in a beaker has temperature q(t) at time t and q0 is of thermal expansion is 1.1 ´ 10-5 K -1 ) [JEE Main 2014]
temperature of surroundings, then according to Newton's
(a) 2.2 ´ 108 Pa (b) 2.2 ´ 10 9 Pa
law of cooling the correct graph between log e (q - q0 ) and
t is : [2012] (c) 2.2 ´ 10 7 Pa (d) 2.2 ´ 10 6 Pa
54. There is a circular tube in a vertical plane. Two liquids which
loge (q – q 0 )

do not mix and of densities d1 and d2 are filled in the tube.


loge (q – q0)

Each liquid subtends 90º angle at centre. Radius joining their


d1
interface makes an angle a with vertical. Ratio is:
(a) (b) d2
0 [JEE Main 2014]
0
t t
1 + sin a
(a)
1 - sin a
loge (q – q0)

loge (q – q0)

1 + cos a
(b)
1 - cos a
(c) (d) a d2
1 + tan a
(c)
0 0 1 - tan a
t t
50. A Carnot engine, whose efficiency is 40%, takes in heat 1 + sin a
(d) d1
from a source maintained at a temperature of 500K. It is 1 - cos a
desired to have an engine of efficiency 60%. Then, the intake 55. Three rods of Copper, Brass and Steel are welded together to
temperature for the same exhaust (sink) temperature must form a Y shaped structure. Area of cross - section of each rod
be : [2012]
(a) efficiency of Carnot engine cannot be made larger than = 4cm 2 . End of copper rod is maintained at 100°C where as
50% ends of brass and steel are kept at 0°C. Lengths of the copper,,
(b) 1200 K (c) 750 K (d) 600 K brass and steel rods are 46, 13 and 12 cms respectively. The
rods are thermally insulated from surroundings excepts at
51. p
ends. Thermal conductivities of copper, brass and steel are
2p0 A B
0.92, 0.26 and 0.12 CGS units respectively. Rate of heat flow
p0
through copper rod is: [JEE Main 2014]
D C (a) 1.2 cal/s (b) 2.4 cal/s
v0
(c) 4.8 cal/s (d) 6.0 cal/s
2v0 v
EBD_7036
P-78 Topic-wise Solved Papers - PHYSICS
56. One mole of a diatomic ideal gas undergoes a cyclic process time of collision between molecules increases as Vq, where V
ABC as shown in figure. The process BC is adiabatic. The
æ Cp ö
temperatures at A, B and C are 400 K, 800 K and 600 K is the volume of the gas. The value of q is : ç g = ÷
respectively. Choose the correct statement: [JEE Main 2014] è Cv ø
[JEE Main 2015]
g +1 g -1
B (a) (b)
2 2
800 K
3g + 5 3g - 5
(c) (d)
6 6
P
60. 'n' moles of an ideal gas undergoes a process A ® B as
600 k shown in the figure. The maximum temperature of the gas
A C during the process will be : [JEE Main 2016]
400 K P
V A
(a) The change in internal energy in whole cyclic process is 2P0
250 R.
(b) The change in internal energy in the process CA is 700 R. P0 B
(c) The change in internal energy in the process AB is -350 R.
(d) The change in internal energy in the process BC is V0 2V0 V
-500 R.
57. A solid body of constant heat capacity 1 J/°C is being heated 9P0 V0 9P0 V0
(a) (b)
by keeping it in contact with reservoirs in two ways : 2nR nR
[JEE Main 2015] 9P0 V0 3P0 V0
(i) Sequentially keeping in contact with 2 reservoirs such (c) (d)
that each reservoir supplies same amount of heat. 4nR 2nR
61. A pendulum clock loses 12 s a day if the temperature is 40°C
(ii) Sequentially keeping in contact with 8 reservoirs such
that each reservoir supplies same amount of heat. and gains 4 s a day if the temperature is 20° C. The
In both the cases body is brought from initial temperature temperature at which the clock will show correct time, and
100°C to final temperature 200°C. Entropy change of the body the co-efficient of linear expansion (a) of the metal of the
in the two cases respectively is : pendulum shaft are respectively : [JEE Main 2016]
(a) ln2, 2ln2 (b) 2ln2, 8ln2 (a) 30°C; a = 1.85 × 10–3/°C
(c) ln2, 4ln2 (d) ln2, ln2 (b) 55°C; a = 1.85 × 10–2/°C
58. Consider a spherical shell of radius R at temperature T. The (c) 25°C; a = 1.85 × 10–5/°C
black body radiation inside it can be considered as an ideal gas (d) 60°C; a = 1.85 × 10–4/°C
U
of photons with internal energy per unit volume u = µ T4 62. An ideal gas undergoes a quasi static, reversible process in
V
1æUö which its molar heat capacity C remains constant. If during
and pressure p = ç ÷ . If the shell now undergoes an this process the relation of pressure P and volume V is given
3è V ø
adiabatic expansion the relation between T and R is : by PVn = constant, then n is given by (Here CP and CV are
molar specific heat at constant pressure and constant
[JEE Main 2015] volume, respectively) : [JEE Main 2016]
1 1
(a) T µ (b) T µ 3 CP - C C - CV
R R (a) n = - (b) n =
C CV C - CP
(c) T µ e–R (d) T µ e–3R
CP C – CP
59. Consider an ideal gas confined in an isolated closed chamber. (c) n= (d) n=
As the gas undergoes an adiabatic expansion, the average CV C – CV
CHAPTER
Simple Harmonic Motion
10 (Oscillations)
Section-A JEE Advanced/ IIT-JEE
5. For a particle executing SHM the displacement x is given by
A Fill in the Blanks x = A coswt. Identify the graph which represents the variation
of potential energy (PE) as a function of time t and
1. An object of mass 0.2 kg executes simple harmonic oscillation
displacement x (2003S)
along the x-axis with a frequency of (25/p) Hz. At the position
x = 0.04, the object has Kinetic energy of 0.5 J and potential I II PE
energy 0.4 J. The amplitude of oscillations is .......m. PE III
(1994 - 2marks) IV
t

C MCQs with One Correct Answer x

1. Two bodies M and N of equal masses are suspended from (a) 1, III (b) II, IV (c) II, III (d) I, IV
6. A simple pendulum has time period T1. The point of
two separate massless springs of spring constants k1 and
suspension is now moved upward according to the relation
k2 respectively. If the two bodies oscillate vertically such y = Kt2, (K = 1 m/s2) where y is the vertical displacement.
that their maximum velocities are equal, the ratio of the T12
The time period now becomes T2. The ratio of is
amplitude of vibration of M to that of N is (1988 - 1mark) T22
k1 k2 (g = 10 m/s2) (2005S)
(a) k (b) k1 / k 2 (c) k (d) k2 / k1 (a) 5/6 (b) 6/5 (c) 1 (d) 4/5
2 1
2. A particle free to move along the x-axis has potential energy 7. The x-t graph of a particle undergoing simple harmonic
given by U(x) = k [1–exp(–x2)] for -¥ £ x £ +¥, where k is motion is shown below. The acceleration of the particle at
t = 4 / 3 s is (2009)
a positive constant of appropriate dimensions. Then
(1999S - 2marks) 1
x (cm)

(a) at points away from the origin, the particle is in unstable


0 t(s)
equilibrium 4 8 12
(b) for any finite nonzero value of x, there is a force directed –1
away from the origin
(c) if its total mechanical energy is k/2, it has its minimum 3 2 –p2
(a) p cm/s 2 (b) cm/s 2
kinetic energy at the origin. 32 32
(d) for small displacements from x = 0, the motion is simple p2 3 2
harmonic (c) cm/s 2 (d) – p cm/s2
32 32
3. The period of oscillation of a simple pendulum of length L 8. A uniform rod of length L and mass M
suspended from the roof of a vehicle which moves without is pivoted at the centre. Its two ends
friction down an inclined plane of inclination a , is given by are attached to two springs of equal
L L spring constants k. The springs are
(a) 2p (b) 2p (2000S) fixed to rigid supports as shown in the
g cos a g sin a
figure, and the rod is free to oscillate in the horizontal plane.
L L The rod is gently pushed through a small angle q in one
(c) 2p (d) 2p
g g tan a direction and released. The frequency of oscillation is
4. A particle executes simple harmonic motion between x = -A (2009)
and x = +A. The time taken for it to go from 0 to A/2 is T1 and 1 2k 1 k
to go from A/2 to A is T2. Then (2001S) (a) (b)
2p M 2p M
(a) T1 < T2 (b) T1 > T2
(c) T1 = T2 (d) T1 = 2T2 1 6k 1 24k
(c) (d)
2p M 2p M
EBD_7036
P-80 Topic-wise Solved Papers - PHYSICS
9. The mass M shown in the figure oscillates in simple harmonic position. When the cylinder is given a small downward push
motion with amplitude A. The amplitude of the point P is and released it starts oscillating vertically with small
amplitude. If the force constant of the spring is k, the
k1 k2 (2009) frequency of oscillation of the cylinder is (1990 - 2mark)
M
1/ 2 1/ 2
P 1 æ k - Arg ö 1 æ k + Arg ö
(a) ç ÷ (b) ç ÷
2p è M ø 2p è M ø
k1A k2 A k1 A k2 A
(a) (b) (c) k + k (d) k + k 1/ 2 1/ 2
k2 k1 1 2 1 2
1 æ k + rgL ö 1 æ k + Arg ö
(c) çè ÷ø (d)
10. A point mass is subjected to two simultaneous sinusoidal 2p M 2 p èç Arg ø÷
displacements in x-direction, x1(t) = A sin wt and x2(t) = 4. A highly rigid cubical block A of small mass M and side L is
æ 2p ö fixed rigidly on to another cubical block B of the same
A sin ç wt + ÷ . Adding a third sinusoidal displacement dimensions and of low modulus of rigidity h such that the
è 3 ø
x3(t) = B sin (wt + f) brings the mass to a complete rest. The lower face of A completely covers the upper face of B. The
values of B and f are (2011) lower face of B is rigidly held on a horizontal surface. A
small force F is applied perpendicular to one of the sides
3p 4p 5p p faces of A. After the force is withdrawn, block A executes
(a) 2 A, (b) A, (c) 3 A, (d) A,
4 3 6 3 small oscillations the time period of which is given by
11. A small block is connected to one end of a massless spring of (1992 - 2mark)
un-stretched length 4.9 m. The other end of the spring (see
Mh ML M
the figure) is fixed. The system lies on a horizontal frictionless (a) 2p M hL (b ) 2p (c) 2p (d) 2p
surface. The block is stretched by 0.2 m and released from L h hL
rest at t = 0. It then executes simple harmonic motion with 5. One end of a long metallic wire of length L is tied to the
angular frequency w = p/3 rad/s. Simultaneously at t = 0, a ceiling. The other end is tied to a massless spring of spring
small pebble is projected with speed v form point P at an constant K.A mass m hangs freely from the free end of the
angle of 45° as shown in the figure. Point P is at a horizontal spring. The area of cross-section and the Young’s modulus
distance of 10 m from O. If the pebble hits the block at t = 1 s, of the wire are A and Y respectively. If the mass is slightly
the value of v is (take g = 10 m/s2) (2012) pulled down and released, it will oscillate with a time period
z T equal to: (1993-2marks)
m(YA + KL)
(a) 2p(m / K )1/ 2 (b) 2p
v YAK
45
° (c) 2p[(mYA / KL)1/ 2 (d) 2p[(mL / YA)1/ 2
x 6. A particle of mass m is executing oscillations about the origin
O P
10 m on the x axis. Its potential energy is V(x) = k | x |3 where k is
(a) 50 m/s (b) 51 m/s a positive constant. If the amplitude of oscillation is a, then
its time period T is (1998S - 2marks)
(c) 52 m/s (d) 53 m/s (a) proportional to 1/ a (b) independent of a
(c) proportional to a (d) proportional to a 3 2
D MCQs with One or More than One Correct 7. Three simple harmonic motions in the same direction having
the same amplitude a and same period are superposed. If
1. A particle executes simple harmonic motion with a frequency.
each differs in phase from the next by 45°, then.
f. The frequency with which its kinetic energy oscillates is (1999S - 3marks)
(1987 - 2marks)
(a) f/2 (b) f (a) the resultant amplitude is (1 + 2)a
(c) 2f (d) 4f (b) the phase of the resultant motion relative to the first is
2. A linear harmonic oscillator of force constant 2 × 106 N/m 90°
(c) the energy associated with the resulting motion is
and amplitude 0.01 m has a total mechanical energy of 160 J.
Its (1989 - 2 Mark) (3 + 2 2 ) times the energy associated with any single
(a) maximum potential energy is 100 J motion
(b) maximum kinetic energy is 100 J (d) the resulting motion is not simple harmonic.
(c) maximum potential energy is 160 J 8. The function x = A sin 2 wt + B cos 2 wt + C sin wt cos wt
(d) maximum potential energy is zero represent SHM for which of the option(s)
3. A uniform cylinder of length L and mass M having cross (a) for all value of A, B and C (C ¹ 0) (2006 - 5M, –1)
sectional area A is suspended, with its length vertical, from
(b) A = B, C = 2B
a fixed point by a massless spring, such that it is
(c) A = –B, C = 2B
half- submerged in a liquid of density r at equilibrium (d) A = B, C = 0
Simple Harmonic Motion (Oscillations) P-81

9. A metal rod of length ‘L’ and mass ‘m’ is pivoted at one end.
A thin disc of mass ‘M’ and radius ‘R’ (<L) is attached at its E Subjective Problems
center to the free end of the rod. Consider two ways the disc 1. A mass M attached to a spring, oscillates with a period of
is attached: (case A). The disc is not free to rotate about its 2sec. If the mass is increased by 2 kg the period increases
centre and (case B) the disc is free to rotate about its centre. by one sec. Find the initial mass M assuming that Hook’s
The rod disc system performs SHM in Law is obeyed. (1979)
vertical plane after being released from 2. Two masses m1 and m2 are suspended
the same displaced position. Which of together by a massless spring of
the following statement(s) is (are) true? spring constant k. When the masses
(2011) are in equilibrium, m1 is removed
(a) Restoring torque in case A = Restoring torque in without disturbing the system. Find
case B the angular frequency and amplitude
of oscillation of m2. (1981 - 3 marks) m1
(b) restoring torque in case A < Restoring torque in case B m2
(c) Angular frequency for case A > angular frequency for 3. Two light springs of force constants k1 and k2 and a block
case B. of mass m are in one line AB on a smooth horizontal table
(d) Angular frequency for case A < Angular frequency for such that one end of each spring is fixed on rigid supports
case B. and the other end is free as shown in the figure. The distance
10. Two independent harmonic oscillators of equal mass are CD between the free ends of the springs is 60 cms. If the
oscillating about the origin with angular frequencies w1 and block moves along AB with a velocity 120 cm/sec in between
w2 and have total energies E1 and E2, respectively. The the springs, calculate the period of oscillation of the block
variations of their momenta p with positions x are shown in (k1 = 1.8 N/m, k2 = 3.2 N/m, m = 200 gm) (1985 - 6 Marks)
a a k1 60 cm
k2
the figures. If = n2 and = n, then the correct equation(s) v
b R
is(are) (JEE Adv. 2015) A C m D B
p P 4. Two non-viscous, incompressible and
Energy = E2
immiscible liquids of densities r and
Energy = E1
1.5 r are poured into the two limbs of R
b x a circular tube of radius R and small
x O
a R cross section kept fixed in a vertical
q
plane as shown in fig. Each liquid
occupies one fourth the circumference
of the tube. (1991 - 4 + 4 marks)
w2
(a) E1w1 = E2w2 (b) 2 (a) Find the angle q that the radius to the interface makes
w1 = n
with the vertical in equilibrium position.
E1 E2 (b) If the whole is given a small displacement from its
(c) w1w2 = n2 (d) =
w1 w2 equilibrium position, show that the resulting
11. A block with mass M is connected by a massless spring oscillations are simple harmonic. Find the time period
with stiffness constant k to a rigid wall and moves without of these oscillations.
friction on a horizontal surface. The block oscillates with 5. Two identical balls A and B each of mass 0.1 kg, are attached
small amplitude A about an equilibrium position x0. Consider to two identical massless springs. The spring-mass system
two cases: (i) when the block is at x0; and (ii) when the block is constrained to move inside a rigid smooth pipe bent in
is at x = x0 + A. In both the cases, a particle with mass m(<M) the form of a circle as shown in Fig. The pipe is fixed in a
is softly placed on the block after which they stick to each horizontal plane.
other. Which of the following statement(s) is (are) true about The centres of the balls can move A B
the motion after the mass m is placed on the mass M? in a circle of radius 0.06 p meter. 6 m
(JEE Adv. 2016) p/6 0.0 p/6
Each spring has a natural length P Q
(a) The amplitude of oscillation in the first case changes zof 0.06p meter and spring constant
M 0.1 N/m. Initially, both the balls are
by a factor of , whereas in the second case it displaced by an angle q = p / 6
m+M
remains unchanged. radian with respect to the diameter PQ of the circle (as shown
(b) The final time period of oscillation in both the cases is in Fig.) and released from rest. (1993 - 6 marks)
same. (i) Calculate the frequency of oscillation of ball B.
(c) The total energy decreases in both the cases. (ii) Find the speed of ball A when A and B are at the two
(d) The instantaneous speed at x0 of the combined masses ends of the diameter PQ.
decreases in both the cases (iii) What is the total energy of the system
EBD_7036
P-82 Topic-wise Solved Papers - PHYSICS
6. A thin rod of length L and area of 7. A small body attached to one end
cross-section S is pivoted at its of a vertically hanging spring is
lowest point P inside a performing SHM about its mean
stationary, homogeneous and position with angular frequency w
non-viscous liquid . The rod is and amplitude a. If at a height y*
y
free to rotate in a vertical plane from the mean position, the body yo
about a horizontal axis passing gets detached from the spring,
calculate the value of y* so that the m
through P. The density d1 of the
height H attained by the mass is
material of the rod is smaller than the density d2 of the liquid.
maximum. The body does not interact with the spring during
The rod is displaced by a small angle q from its equilibrium its subsequent motion after detachment. (aw2 > g)
position and then released. Show that the motion of the rod (2005 - 4 Marks)
is simple harmonic and determine its angular frequency in
terms of the given parameters. (1996 - 5 Marks)

F Match the Following

DIRECTIONS (Q. No. 1-2) : Each question contains statements given in two columns, which have to p q r s t
be matched. The statements in Column-I are labelled A, B, C and D, while the statements in Column-
A p q r s t
II are labelled p, q, r and s. Any given statement in Column-I can have correct matching with ONE OR
MORE statement(s) in Column-II. The appropriate bubbles corresponding to the answers to these B p q r s t
questions have to be darkened as illustrated in the following example : C p q r s t
If the correct matches are A-p, s and t; B-q and r; C-p and q; and D-s then the correct darkening of D p q r s t
bubbles will look like the given.

1. Column I describes some situations in which a small object moves. Column II describes some characteristics of these motions.
Match the situations in Column I with the characteristics in Column II and indicate your answer by darkening appropriate
bubbles in 4 × 4 matrix given in the ORS. (2007)
Column I Column II
(A) The object moves on the x -axis under a (p) The object executes a simple harmonic motion.
conservative force in such a way that its
''speed " and position'' satisfy v = c c2 - x 2
1
where c1 and c2 are positive constants.
(B) The object moves on the x- axis in such a way that (q) The object does not change its direction.
its velocity and its displacement from the origin
satisfy v = – kx,where k is a positive constant.
(C) The object is attached to one end of a mass-less (r) The kinetic energy of the object keeps on
spring of a given spring constant. The other decreasing.
end of the spring is attached to the ceiling of an
elevator. Initially everything is at rest. The elevator
starts going upwards with a constant acceleration a.
The motion of the object is observed from the elevator
during the period it maintains this acceleration.
(D) The object is projected from the earth's surface (s) The object can change its direction only once.
vertically upwards with a speed 2 GM e / R e ,
where, Me is the mass of the earth and Re is the
radius of the earth, Neglect forces from objects other
than the earth.
2. Column I gives a list of possible set of parameters measured in some experiments. The variations of the parameters in the form of
graphs are shown in Column II. Match the set of parameters given in Column I with the graph given in Column II. Indicate your
answer by darkening the appropriate bubbles of the 4 × 4 matrix given in the ORS. (2008)
Simple Harmonic Motion (Oscillations) P-83

Column I Column II
(A) Potential energy of a simple pendulum (y axis) as (p) y
a function of displacement (x axis)

x
y

(B) Displacement (y axis) as a function of time (x axis) (q)


for a one dimensional motion at zero or constant
acceleration when the body is moving along the positive
x-direction.
x
O
(C) Range of a projectile (y axis) as a function of its (r) y
velocity (x axis) when projected at a fixed angle.

x
O
y

(D) The square of the time period (y axis) of a simple (s)


pendulum as a function of its length (x axis)
x
O
2. For periodic motion of small amplitude A, the time period T
G Comprehension Based Questions of this particle is proportional to

PASSAGE - 1 m 1 m a 1 a
(a) A (b) (c) A (d)
When a particle of mass m moves on the x-axis in a potential of the a A a m A m
form V(x) = kx 2 it performs simple harmonic motion. The 3. The acceleration of this particle for |x| > X0 is
m V0
corresponding time period is proportional to ,as can be seen (a) proportional to V0 (b) proportional to
k mX 0
easily using dimensional analysis. However, the motion of a particle V0
can be periodic even when its potential energy increases on both (c) proportional to (d) zero
mX 0
sides of x = 0 in a way different from kx2 and its total energy is
such that the particle does not escape to infinity. Consider a particle PASSAGE - 2
of mass m moving on the x-axis. Its potential energy is V(x) = ax4 Phase space diagrams are useful tools
(a > 0) for |x| near the origin and becomes a constant equal to V0 in analyzing all kinds of dynamical
Momentum

for |x| > X0 (see figure). (2010) problems. They are especially useful in
studying the changes in motion as initial
V(x)
position and momenum are changed.
Here we consider some simple dynamical
V0 systems in one dimension. For such systems, phase space is a
plane in which position is plotted along horizontal axis and
momentum is plotted along vertical axis. The phase space diagram
x
X0 is x(t) vs. p(t) curve in this plane. The arrow on the curve indicates
1. If the total energy of the particle is E, it will perform periodic the time flow. For example, the phase space diagram for a particle
motion only if moving with constant velocity is a straight line as shown in the
(a) E < 0 (b) E > 0 figure. We use the sign convention in which positon or momentum
(c) V0 > E > 0 (d) E > V0 upwards (or to right) is positive and downwards (or to left) is
negative. (2011)
EBD_7036
P-84 Topic-wise Solved Papers - PHYSICS
4. The phase space diagram for a ball thrown vertically up 6. Consider the spring-mass system, with the mass submerged
from ground is in water, as shown in the figure. The phase space diagram
for one cycle of this system is

(a) (b)

(c) (d)

(a) (b)

5. The phase space diagram for simple harmonic motion is a


circle centered at the origin. In the figure, the two circles
represent the same oscillator but for different initial
conditions, and E1 and E2 are the total mechanical energies
respectively. Then

(a) E1 = 2 E2 (c) (d)

(b) E1 = 2E2

(c) E1 = 4E2

(d) E1 = 16E2

Section-B JEE Main / AIEEE


1. In a simple harmonic oscillator, at the mean position [2002] 5. Two particles A and B of equal masses are suspended from
(a) kinetic energy is minimum, potential energy is maximum two massless springs of spring of spring constant k1 and
(b) both kinetic and potential energies are maximum k2, respectively. If the maximum velocities, during oscillation,
(c) kinetic energy is maximum, potential energy is minimum are equal, the ratio of amplitude of A and B is [2003]
(d) both kinetic and potential energies are minimum.
2. If a spring has time period T, and is cut into n equal parts, k1 k2 k2 k1
then the time period of each part will be [2002] (a) k2
(b) k (c) k1
(d) k
1 2
(a) T n (b) T / n 6. The length of a simple pendulum executing simple harmonic
(c) nT (d) T motion is increased by 21%. The percentage increase in the
3. A child swinging on a swing in sitting position, stands up, time period of the pendulum of increased length is [2003]
then the time period of the swing will [2002] (a) 11% (b) 21% (c) 42% (d) 10%
(a) increase 7. The displacement of a particle varies according to the
(b) decrease
relation x = 4(cos pt + sin pt ). The amplitude of the particle
(c) remains same
(d) increases of the child is long and decreases if the child is [2003]
is short (a) – 4 (b) 4 (c) 4 2 (d) 8
4. A mass M is suspended from a spring of negligible mass. 8. A body executes simple harmonic motion. The potential
The spring is pulled a little and then released so that the energy (P.E), the kinetic energy (K.E) and total energy (T.E)
mass executes SHM of time period T. If the mass is increased are measured as a function of displacement x. Which of the
5T m following statements is true ? [2003]
by m, the time period becomes . Then the ratio of is (a) K.E. is maximum when x = 0
3 M
[2003] (b) T.E is zero when x = 0
3 25 16 5 (c) K.E is maximum when x is maximum
(a) (b) (c) (d) (d) P.E is maximum when x = 0
5 9 9 3
Simple Harmonic Motion (Oscillations) P-85

9. The bob of a simple pendulum executes simple harmonic 16. The bob of a simple pendulum is a spherical hollow ball
motion in water with a period t, while the period of oscillation filled with water. A plugged hole near the bottom of the
of the bob is t0 in air. Neglecting frictional force of water and oscillating bob gets suddenly unplugged. During observa-
3 tion, till water is coming out, the time period of oscillation
given that the density of the bob is (4 / 3) ´ 1000 kg/m . would [2005]
What relationship between t and t0 is true [2004] (a) first decrease and then increase to the original value
(a) t = 2t 0 (b) t = t0 / 2 (b) first increase and then decrease to the original value
(c) increase towards a saturation value
(c) t = t0 (d) t = 4t 0 (d) remain unchanged
10. A particle at the end of a spring executes S.H.M with a 2
period t1. while the corresponding period for another spring d x
17. If a simple harmonic motion is represented by
2
+ ax = 0 ,
is t2. If the period of oscillation with the two springs in dt
series is T then [2004] its time period is [2005]
2p 2p
(a) T -1 = t1-1 + t 2-1 (b) T 2 = t12 + t 22 (a) (b) (c) 2p a (d) 2pa
a a
(c) T = t1 + t2 (d) T -2 = t1-2 + t 2-2 18. The maximum velocity of a particle, executing simple
11. The total energy of a particle, executing simple harmonic harmonic motion with an amplitude 7 mm, is 4.4 m/s. The
motion is [2004] period of oscillation is [2006]
(a) 0.01 s (b) 10 s (c) 0.1 s (d) 100 s
(a) independent of x (b) µ x2 19. Starting from the origin a body oscillates simple harmonically
1/ 2 with a period of 2 s. After what time will its kinetic energy be
(c) µ x (d) µ x
where x is the displacement from the mean position, hence 75% of the total energy? [2006]
total energy is independent of x. 1 1 1 1
(a) s (b) s (c) s (d) s
12. A particle of mass m is attached to a spring (of spring constant k) 6 4 3 12
and has a natural angular frequency w0. An external force 20. Two springs, of force constants k1 and k2 are connected to
F(t) proportional to cos wt (w ¹ w 0 ) is applied to the a mass m as shown. The frequency of oscillation of the
mass is f. If both k1 and k2 are made four times their original
oscillator. The time displacement of the oscillator will be
values, the frequency of oscillation becomes [2007]
proportional to [2004]
m
1 1 k1 k2
(a) m(w 2 + w 2 ) (b) m(w 2 - w 2 )
0 0
m m (a) 2 f (b) f /2 (c) f /4 (d) 4 f
(c) 2 2 (d) 2 2
21. A particle of mass m executes simple harmonic motion with
w0 - w (w 0 + w ) amplitude a and frequency n. The average kinetic energy
13. In forced oscillation of a particle the amplitude is maximum during its motion from the position of equilibrium to the end
for a frequency w1 of the force while the energy is maximum is [2007]
for a frequencyw2 of the force; then [2004]
(a) 2p 2 ma 2 n2 (b) p 2 ma 2 n2
(a) w1 < w2 when damping is small and w1 > w2 when
damping is large 1 2 2
(c) ma n (d) 4p 2 ma 2 n2
(b) w1 > w2 4
(c) w1 = w2 22. The displacement of an object attached to a spring and
(d) w1 < w2 executing simple harmonic motion is given by x = 2 × 10–2
14. Two simple harmonic motions are represented by the cos pt metre.The time at which the maximum speed first
occurs is [2007]
æ pö
equations y1 = 0.1 sin ç100pt + ÷ and y 2 = 0.1 cos pt . (a) 0.25 s (b) 0.5 s
è 3ø (c) 0.75 s (d) 0.125 s
The phase difference of the velocity of particle 1 with respect
to the velocity of particle 2 is [2005] 23. A point mass oscillates along the x-axis according to the
p -p p -p law x = x0 cos(wt - p / 4) . If the acceleration of the particle
(a) (b) (c) (d) is written as a = A cos(wt + d ) ,then [2007]
3 6 6 3
A = x0 w , d = 3p / 4 (b) A = x0, d = -p / 4
2
2
15. The function sin (wt ) represents [2005] (a)
p 2
(c) A = x0 w 2 , d = p / 4 (d) A = x0 w , d = -p / 4
(a) a periodic, but not SHM with a period
w 24. If x, v and a denote the displacement, the velocity and the
2p
(b) a periodic, but not SHM with a period acceleration of a particle executing simple harmonic motion
w of time period T, then, which of the following does not
p
(c) a SHM with a period change with time? [2009]
w (a) aT/x (b) aT + 2pv
2p
(d) a SHM with a period (c) aT/v (d) a2T2 + 4p2v2
w
EBD_7036
P-86 Topic-wise Solved Papers - PHYSICS
25. Two particles are executing simple harmonic motion of the 30. A particle moves with simple harmonic motion in a straight
same amplitude A and frequency w along the x-axis. Their line. In first t s, after starting from rest it travels a distance a,
mean position is separated by distance X0(X0 > A). If the and in next t s it travels 2a, in same direction, then:
maximum separation between them is (X0 + A), the phase [JEE Main 2014]
difference between their motion is: [2011] (a) amplitude of motion is 3a
p p p p (b) time period of oscillations is 8t
(a) (b) (c) (d)
3 4 6 2 (c) amplitude of motion is 4a
26. A mass M, attached to a horizontal spring, executes S.H.M. (d) time period of oscillations is 6t
with amplitude A1. When the mass M passes through its 31. A pendulum made of a uniform wire of cross sectional area
mean position then a smaller mass m is placed over it and A has time period T. When an additional mass M is added to
both of them move together with amplitude A2. The ratio of its bob, the time period changes to TM. If the Young's modulus
æ A1 ö 1
çè A ÷ø is: [2011] of the material of the wire is Y then is equal to :
2 Y
(g = gravitational acceleration) [JEE Main 2015]
1
M +m é æ T ö2 ù A
(b) æç
M ö2 é æ T ö2 ù A
(a)
M è M + m ÷ø (a) ê1 - ç
M
÷ ú (b) ê1 - ç T ÷ ú Mg
ë è T ø û Mg ëê è M ø úû
1
æ M + mö 2 M
(c) çè ÷ (d) éæ T ö2 ù A éæ T ö2 ù Mg
M ø M +m M
(c) êç ÷ - 1ú (d) êç M ÷ - 1ú
27. If a simple pendulum has significant amplitude (up to a factor ëè T ø û Mg ëè T ø û A
of 1/e of original) only in the period between t = 0s to t = t s, 32. For a simple pendulum, a graph is plotted between its kinetic
then t may be called the average life of the pendulum. energy (KE) and potential energy (PE) against its displacement d.
When the spherical bob of the pendulum suffers a Which one of the following represents these correctly?
retardation (due to viscous drag) proportional to its velocity (graphs are schematic and not drawn to scale)
with b as the constant of proportionality, the average life [JEE Main 2015]
time of the pendulum is (assuming damping is small) in
seconds : [2012] E
E KE PE
0.693 1 2
(a) (b) b (c) (d)
b b b
(a) d (b) KE
28. The amplitude of a damped oscillator decreases to 0.9 times
its original magnitude in 5s. In another 10s it will decrease to
a times its original magnitude, where a equals PE
(a) 0.7 (b) 0.81 [JEE Main 2013]
E PE
(c) 0.729 (d) 0.6 E
KE
29. An ideal gas enclosed in a vertical cylindrical container
supports a freely moving piston of mass M. The piston and PE KE
the cylinder have equal cross sectional area A. When the (c) d
(d) d
piston is in equilibrium, the volume of the gas is V0 and its
pressure is P0. The piston is slightly displaced from the 33. A particle performs simple harmonic mition with amplitude
equilibrium position and released. Assuming that the system A. Its speed is trebled at the instant that it is at a distance
is completely isolated from its surrounding, the piston executes
2A
a simple harmonic motion with frequency [JEE Main 2013] from equilibrium position. The new amplitude of the
3
1 AgP0 1 V0 MP0 motion is : [JEE Main 2016]
(a) (b)
2p V0 M 2p A 2 g 7A
(a) A 3 (b)
3
1 A 2 gP0 1 MV0 A
(c) (d) (c) 41 (d) 3A
2p MV0 2p AgP0 3
CHAPTER

11 Waves
Section-A JEE Advanced/ IIT-JEE
A Fill in the Blanks B True/False
1. A travelling wave has the frequency n and the particle 1. A man stands on the ground at a fixed distance from a siren
which emits sound of fixed amplitude. The man hears the
displacement amplitude A. For the wave the particle velocity sound to be louder on a clear night than on a clear day.
amplitude is ---------- and the particle acceleration amplitude (1980)
is ---------- . (1983 - 2 Marks) 2. A plane wave of sound travelling in air is incident upon a
plane water surface. The angle of incidence is 60°. Assuming
2. Sound waves of frequency 660 Hz fall normally on a perfectly snell’s law to be valid for sound waves, it follows that the
reflecting wall. The shortest distance from the wall at which sound wave will be refracted into water away from the normal.
the air particles have maximum amplitude of vibration is ..... (1984- 2 Marks)
metres. (1984- 2 Marks) 3. A source of sound with frequency 256 Hz is moving with a
velocity V towards a wall and an observer is stationary
3. Two simple harmonic motions are represented by the between the source and the wall. When the observer is
equations between the source and the wall he will hear beats
(1985 - 3 Marks)
y1 = 10sin (3pt + p / 4) and y2 = 5(sin 3pt + 3 cos3pt)
C MCQs with One Correct Answer
Their amplitudes are in the ratio of .......... (1986 - 2 Marks)
4. In a sonometer wire, the tension is maintained by suspending 1. A cylindrical tube open at both ends, has a fundamental
frequency ‘f’ in air. The tube is dipped vertically in air. The
a 50.7 kg mass from the free end of the wire. The suspended tube is dipped vertically in water so that half of it is in water.
3 The fundamental frequency of the air column in now
mass has a volume of 0.0075 m . The fundamental frequency (1981- 2 Marks)
of vibration of the wire is 260 Hz. If the suspended mass is f 3f
completely submerged in water, the fundamental frequency (a) (b) (c) f (d) 2f
2 4
will become .............Hz. (1987 - 2 Marks)
2. A wave represented by the equation y = a cos (kx - wt ) is
5. The amplitude of a wave disturbance propagating in the superposed with another wave to form a stationary wave
such that point x = 0 is a node. The equation for the other
1 wave is (1988 - 1 Mark)
positive x-direction is given by y = at time t = 0 and
(1 + x ) 2 (a) a sin (kx + wt ) (b) – a cos (kx - wt )
(c) – a cos (kx + wt ) (d) – a sin (kx - wt )
1
by y = at t = 2 seconds, where x and y are 3. An object of specific gravity r is hung from a thin steel
[1 + ( x - 1) 2 ] wire. The fundamental frequency for transverse standing
waves in the wire is 300 Hz. The object is immersed in water
in metres. The shape of the wave disturbance does not so that one half of its volume is submerged. The new
change during the propagation. The velocity of the wave is fundamental frequency in Hz is (1995S)
.............. m/s. (1990 - 2 Marks) 1/ 2 1/ 2
æ 2r - 1 ö æ 2r ö
6. A cylinder resonance tube open at both ends has (a) 300çç ÷÷ (b) 300çç ÷÷
fundamental frequency F in air. Half of the length of the è 2r ø è 2r - 1 ø
tube is dipped vertically in water. The fundamental frequency æ 2r ö æ 2r - 1 ö
(c)300çç ÷÷ (d) 300çç ÷÷
to the air column now is .................... (1992 - 1 Mark) è 2r - 1 ø è 2r ø
7. A bus is moving towards a huge wall with a velocity of 4. A wave disturbance in a medium is described by
5 ms–1. The driver sounds a horn of frequency 200 Hz. The æ pö
frequency of the beats heard by a passenger of the bus will y ( x, t ) = 0.02 cos ç 50pt + ÷ cos(10px) where x and y are
è 2ø
be..... Hz (Speed of sound in air = 342 ms–1)
in metre and t is in second (1995S)
(1994 - 2 Marks)
EBD_7036
P-88 Topic-wise Solved Papers - PHYSICS
(a) A node occurs at x = 0.15 m records a frequency of 5.5 kHz while the train approaches
(b) An antinode occurs at x = 0.3 m the siren. During his return journey in a different train B he
(c) The speed wave is 5 ms–1 records a frequency of 6.0 kHz while approaching the same
(d) The wave length is 0.3 m siren. The ratio of the velocity of train B to that train A is
5. The extension in a string, obeying Hooke’s law, is x. The (a) 242/252 (b) 2 (2002S )
speed of sound in the stretched string is v. If the extension (c) 5/6 (d) 11/6
in the string is increased to 1.5x, the speed of sound will be 14. A sonometer wire resonates with a given tuning fork forming
(1996 - 2 Marks) standing waves with five antinodes between the two bridges
when a mass of 9 kg is suspended from the wire. When this
(a) 1.22v (b) 0.61v (c) 1.50v (d) 0.75v mass is replaced by a mass M, the wire resonates with the
same tuning fork forming three antinodes for the same
6. An open pipe is suddenly closed at one end with the result
positions of the bridges. The value of M is (2002S )
that the frequency of third harmonic of the closed pipe is
(a) 25 kg (b) 5 kg (c) 12.5 kg (d) 1/25 kg
found to be higher by 100Hz than the fundamental frequency
15. A police car moving at 22 m/s, chases a motorcyclist. The
of the open pipe. The fundamental frequency of the open police man sounds his horn at 176 Hz, while both of them
pipe is (1996 - 2 Marks) move towards a stationary siren of frequency 165 Hz.
(a) 200 Hz (b) 300 Hz (c) 240 Hz (d) 480 Hz Calculate the speed of the motorcycle, if it is given that he
7. A travelling wave in a stretched string is described by the does not observes any beats. (2003S)
equation y = A sin (kx –wt) The maximum particle velocity is
Police Car Motorcycle
(1997 - 1 Mark)
(a) Aw (b) w / k (c) dw / dk (d) x / t
8. A train moves towards a stationary observer with speed 34 Stationary
22 m/s
V Siren
m/s. The train sounds a whistle and its frequency registered 176 Hz (165 Hz)
by the observer is f1. If the train's speed is reduced to 17
m/s, the frequency registered is f2. If the speed of sound is (a) 33m/s (b) 22m/s (c) zero (d) 11m/s
340 m/s, then the ratio f1/f2 is (2000S) 16. In the experiment for the determination of the speed of sound
in air using the resonance column method, the length of the
(a) 18/19 (b) 1/2 (c) 2 (d) 19/18
air column that resonates in the fundamental mode, with a
9. Two vibrating strings of the same material but lengths L and
tuning fork is 0.1 m. When this length is changed to 0.35 m,
2L have radii 2r and r respectively. They are stretched under
the same tuning fork resonates with the first overtone.
the same tension. Both the strings vibrate in their Calculate the end correction. (2003S)
fundamental nodes, the one of length L with frequency v1 (a) 0.012 m (b) 0.025 m (c) 0.05 m (d) 0.024 m
and the other with frequency v2. The raio v1/v2 is given 17. A pipe of length l1, closed at one end is kept in a chamber
by (2000S) of gas of density r1. A second pipe open at both ends is
(a) 2 (b) 4 (c) 8 (d) 1 placed in a second chamber of gas of density r2. The
10. Two monatomic ideal gases 1 and 2 of molecular masses m1 compressibility of both the gases is equal. Calculate the
and m2 respectively are enclosed in separate containers length of the second pipe if frequency of first overtone in
kept at the same temperature. The ratio of the speed of sound both the cases is equal (2004S)
in gas 1 to that in gas 2 is given by (2000S)
4 r 4 r
(a) l1 2 (b) l1 1
m1 m2 m1 m2 3 r1 3 r2
(a) m2 (b) m1 (c) m2 (d) m
1
r2 r1
(c) l1 (d) l 1
11. Two pulses in a stretched string whose centers are initially r1 r2
8 cm apart are moving towards each other as shown in the
18. In a resonance tube with tuning fork of frequency 512 Hz,
figure. The speed of each pulse is 2 cm/s. After 2 seconds,
first resonance occurs at water level equal to 30.3 cm and
the total energy of the pulses will be (2001S) second resonance occurs at 63.7 cm. The maximum possible
(a) zero error in the speed of sound is (2005S)
(b) purely kinetic (a) 51.2 cm/s (b) 102.4 cm/s
(c) purely potential (c) 204.8 cm/s (d) 153.6 cm/s
(d) partly kinetic and partly potential 8 cm 19. An open pipe is in resonance in 2nd harmonic with
12. The ends of a stretched wire of length L are fixed at x = 0 and frequency f 1 . Now one end of the tube is closed and
x = L. In one experiment, the displacement of the wire is frequency is increased to f2 such that the resonance again
y1 = A sin (px/L) sin wt and energy is E1 and in another occurs in nth harmonic. Choose the correct option
experiment its displacement is y2 = A sin (2px/L) sin 2wt and (2005S)
energy is E2. Then (2001S)
3 5
(a) E2 = E1 (b) E2 = 2E1 (a) n = 3, f 2 = f1 (b) n = 3, f 2 = f1
4 4
(c) E2 = 4E1 (d) E2 = 16 E1
13. A siren placed at a railway platform is emitting sound of 3 5
(c) n = 5, f 2 = f1 (d) n = 5, f 2 = f1
frequency 5 kHz. A passenger sitting in a moving train A 4 4
Waves P-89

20. A massless rod of length L is suspended by two identical (a) 5 grams (b) 10 grams (2010)
strings AB and CD of equal length. A block of mass m is (c) 20 grams (d) 40 grams
suspended from point O such that BO is equal to ‘x’. Further 25. A police car with a siren of frequency 8 kHz is moving with
it is observed that the frequency of 1st harmonic in AB is uniform velocity 36 km/hr towards a tall building which reflects
equal to 2nd harmonic frequency in CD. ‘x’ is the sound waves. The speed of sound in air is 320 m/s. The
(2006 - 3M, –1) frequency of the siren heard by the car driver is (2011)
L C (a) 8.50 kHz (b) 8.25 kHz
(a) A
5 (c) 7.75 kHz (d) 7.50 kHZ
26. A student is performing the experiment of resonance column.
4L The diameter of the column tube is 4 cm. The frequency of
(b)
5 the tuning fork is 512 Hz. The air temperature is 38°C in
O which the speed of sound is 336 m/s. The zero of the meter
3L B L
D
scale coincides with the top end of the resonance column
(c) x
4 tube. When the first resonance occurs, the reading of the
m water level in the column is (2012)
L (a) 14.0 cm (b) 15.2 cm (c) 16.4 cm (d) 17.6 cm
(d)
4
21. In the experiment to determine the speed of sound using a D MCQs with One or More than One Correct
resonance column, (2007)
(a) prongs of the tuning fork are kept in a vertical plane 1. A wave equation which gives the displacement along the
(b) prongs of the tuning fork are kept in a horizontal y-direction is given by y = 10–4 sin (60t + 2x) where x and y are
plane in metres and t is time in seconds. This represents a wave
(1982 - 3 Marks)
(c) in one of the two resonances observed, the length of
(a) travelling with a velocity of 30 m/s in the negative x
the resonating air column is close to the wavelength direction
of sound in air (b) of wavelength pm
(d) in one of the two resonances observed, the length of (c) of frequency 30/p hertz
the resonating air column is close to half of the (d) of amplitude 10–4 m traveling along the negative
wavelength of sound in air x-direction
22. A transverse sinusoidal wave moves along a string in the 2. A tr ansverse wave is descr ibed by the equation
positive x-direction at a speed of 10 cm/s. The wavelength of æ xö
the wave is 0.5 m and its amplitude is 10 cm. At a particular time y = y0 sin 2p ç ft - ÷ . The maximum particle velocity is
t, the snap–shot of the wave is shown in figure. The velocity of è lø
point P when its displacement is 5 cm is – (2008) equal to four times the wave velocity if (1984- 2 Marks)
3p ˆ y0 y0
(a) jm/s y (a) l = p (b) l = p
50 4 2
(c) l = py0 (d) l = 2py0
3p 3. An air column in a pipe, which is closed at one end, will be
(b) - ĵ m / s P
50 in resonance with a vibrating tuning fork of frequency 264
x Hz if the length of the column in cm is : (1985 - 2 Marks)
3p
(c) î m / s (a) 31.25 (b) 62.50 (c) 93.75 (d) 125
50 4. A tube, closed at one end and containing air, produces,
when excited, the fundamental note of frequency 512 Hz. If
3p
(d) - î m / s the tube is open at both ends the fundamental frequency
50 that can be excited is (in Hz) (1986 - 2 Marks)
23. A vibrating string of certain length l under a tension T (a) 1024 (b) 512 (c) 256 (d) 128
resonates with a mode corresponding to the first overtone 5. The displacement of particles in a string stretched in the
(third harmonic) of an air column of length 75 cm inside a x-direction is represented by y. Among the following
tube closed at one end. The string also generates 4 beats expressions for y, those describing wave motion are :
per second when excited along with a tuning fork of (1987 - 2 Marks)
frequency n. Now when the tension of the string is slightly (a) cos kx sin wt (b) k 2 x 2 - w 2 t 2
increased the number of beats reduces 2 per second.
Assuming the velocity of sound in air to be 340 m/s, the (c) cos 2 (kx + wt ) (d) cos (k2x2 – w2t2)
frequency n of the tuning fork in Hz is (2008) 6. An organ pipe P1 closed at one end vibrating in its first
(a) 344 (b) 336 (c) 117.3 (d) 109.3
harmonic and another pipe P2 open at ends vibrating in its
24. A hollow pipe of length 0.8 m is closed at one end. At its
third harmonic are in resonance with a given tuning fork.
open end a 0.5 m long uniform string is vibrating in its
second harmonic and it resonates with the fundamental The ratio of the length of P1 to that of P2 is
frequency of the pipe. If the tension in the wire is 50 N and (1988 - 2 Marks)
the speed of sound is 320 ms–1, the mass of the string is (a) 8/3 (b) 3/8 (c) 1/6 (d) 1/3
EBD_7036
P-90 Topic-wise Solved Papers - PHYSICS
7. Velocity of sound in air is 320 m/s. A pipe closed at one end 13. The (x, y) co-ordinates of the corners of a square plate are
has a length of 1 m. Neglecting end corrections, the air (0, 0), (L, 0), (L, L) and (0, L). The edges of the plate are
column in the pipe can resonate for sound of frequency : clamped and transverse standing waves are set up in it. If
(1989 - 2 Marks) u(x, y) denotes the displacement of the plate at the point
(a) 80 Hz (b) 240 Hz (c) 320 Hz (d) 400 Hz (x, y) at some instant of time, the possible expression(s) for
8. A wave is represented by the equation u is (are) (a = positive constant) (1998 - 2 Marks)
p (a) a cos (px/2L) cos (py/2L) (b) a sin (px/L) sin (py/L)
y = A sin (10 p x + 15 p t +) (c) a sin (px/L) sin (2py/L) (d) a cos (2px/L) sin (py/L)
3
where x is in meters and t is in seconds. The expression 14. A transverse sinusoidal wave of amplitude a, wavelength l
represents : (1990 - 2 Marks) and frequency f is travelling on a stretched string. The
(a) a wave travelling in the positive x-direction with a maximum speed of any point on the string is v/10, where v is
velocity 1.5 m/s. the speed of propagation of the wave. If a = 10–3m and
(b) a wave traveling in the negative x-direction with a v = 10 m s–1, then l and f are given by (1998 - 2 Marks)
velocity 1.5 m/s. l = 2 p ´ 10 - 2 m
(a) (b) l = 10 - 3 m
(c) a wave travelling in the negative x-direction having a
wavelength 0.2 m. (c) f = 103 Hz /(2p) (d) f = 104 Hz
(d) a wave travelling in the positive x-direction having a 15. y(x, t) = 0.8/[4x+5t)2+5] represents a moving pulse, where x
wavelength 0.2 m. and y are in meter and t in second. Then
9. Two identical straight wires are stretched so as to produce (1999 - 3 Marks)
6 beats per second when vibrating simultaneously. On (a) pulse is moving in + x direction
changing the tension slightly in one of them, the beat (b) in 2 s it will travel a distance of 2.5 m
frequency remains unchanged. Denoting by T1 , T2 the (c) its maximum displacement is 0.16 m
higher and the lower initial tension in the strings, then it (d) it is a sysmmetric pulse
could be said that while making the above changes in 16. In a wave motion y = a sin (kx - wt), y can represent
tension, (1991 - 2 Marks) (1999 - 3 Marks)
(a) T2 was decreased (b) T2 was increased (a) electric field (b) magnetic field
(c) displacement (d) pressure
(c) T1 was decreased (d) T1 was increased 17. Standing waves can be produced (1999 - 3 Marks)
10. The displacement y of a particle executing periodic motion (a) on a string clamped at both the ends.
æ1 ö (b) on a string clamped at one end free at the other
is given by y = 4cos 2 ç t ÷ sin(1000 t ) (c) when incident wave gets reflected from a wall
è2 ø
(d) when two identical waves with a phase difference of p
This expression may be considereed to be a result of the
are moving in the same direction
superposition of (1992 - 2 Marks)
18. As a wave propagates, (1999 - 3 Marks)
(a) two (b) three (c) four (d) five
11. A sound wave of frequency f travels horizontally to the (a) the wave intensity remains constant for a plane wave
right. It is reflected from a large vertical plane surface moving (b) the wave intensity decreases as the inverse of the
to left with a speed v. The speed of sound in medium is C distance from the source for a spherical wave
(1995S) (c) the wave intensity decreases as the inverse square of
(a) The number of wave striking the surface per second is the distance from the source for a spherical wave
(d) total intensity of the spherical wave over the spherical
(c + v ) surface centred at the source remains constant at all
f
c times.
c( c - v ) 19. A student performed the experiment to measure the speed
(b) The wavelength of reflected wave is of sound in air using resonance air-column method. Two
f (c + v )
resonances in the air-column were obtained by lowering the
(c + v) water level. The resonance with the shorter air-column is
(c) The frequency of the reflected wave is f
(c - v) the first resonance and that with the longer air-column is
(d) The number of beats heard by a stationary listener to the second resonance. Then, (2009)
(a) the intensity of the sound heard at the first resonance
vf was more than that at the second resonance
the left of the reflecting surface is
c-v (b) the prongs of the tuning fork were kept in a horizontal
12. A string of length 0.4 m and mass 10–2 kg is tightly clamped plane above the resonance tube
at its ends. The tension in the string is 1.6 N. Identical wave (c) the amplitude of vibration of the ends of the prongs is
pulses are produced at one end at equal intervals of time, typically around 1 cm
Dt. The minimum value of Dt which allows constructive (d) the length of the air-column at the first resonance was
interference between successive pulses is(1998 - 2 Marks) somewhat shorter than 1/4th of the wavelength of the
(a) 0.05 s (b) 0.10 s (c) 0.20 s (d) 0.40 s sound in air
Waves P-91

20. A person blows into open-end of a long pipe. As a result, a 24. One end of a taut string of length 3 m along the x-axis is fixed
high pressure pulse of air travels down the pipe. at x = 0. The speed of the waves in the string is 100 ms–1. The
When this pulse reaches the other end of the pipe, other end of the string is vibrating in the y-direction so that
(2012) stationary waves are set up in the string. The possible
(a) a high-pressure pulse starts travelling up the pipe, if waveform (s) of these stationary waves is(are)
the other end of the pipe is open. (JEE Adv. 2014)
(b) a low-pressure pulse starts travelling up the pipe, if the
px 50pt
other end of the pipe is open. (a) y ( t ) = A sin cos
(c) a low-pressure pulse starts travelling up the pipe, if the 6 3
other end of the pipe is closed.
px 100pt
(d) a high-pressure pulse starts travelling up the pipe, if (b) y ( t ) = A sin cos
the other end of the pipe is closed. 3 3
21. A horizontal stretched string, fixed at two ends, is vibrating
in its fifth harmonic according to the equation, y(x, t) = (0.01
5px 250pt
m) sin [(62.8 m–1)x] cos[(628 s–1)t]. Assuming p = 3.14, the (c) y ( t ) = A sin cos
correct statement(s) is (are) (JEE Adv. 2013) 6 3
(a) The number of nodes is 5 5 px
(b) The length of the string is 0.25 m (d) y ( t ) = A sin cos 250pt
(c) The maximum displacement of the midpoint of the string, 2
from its equilibrium position is 0.01 m 25. Two loudspeakers M and N are located 20 m apart and emit
(d) The fundamental frequency is 100 Hz sound at frequencies 118 Hz and 121 Hz, respectively. A car
22. Two vehicles, each moving with speed u on the same is initially at a point P , 1800 m away from the midpoint Q of
horizontal straight road, are approaching each other. Wind the line MN and moves towards Q constantly at 60 km/hr
blows along the road with velocity w. One of these vehicles along the perpendicular bisector of MN. It crosses Q and
blows a whistle of frequency f1. An observer in the other eventually reaches a point R, 1800 m away from Q. Let v(t)
vehicle hears the frequency of the whistle to be f2. The represent the beat frequency measured by a person sitting
speed of sound in still air is V. The correct statement(s) is in the car at time t. Let vP , vQ and vR be the beat frequencies
(are) (JEE Adv. 2013) measured at locations P, Q and R, respectively. The speed
(a) If the wind blows from the observer to the source, of sound in air is 330 ms–1 . Which of the following
f2 > f1 statement(s) is(are) true regarding the sound heard by the
(b) If the wind blows from the source to the observer, person? (JEE Adv. 2016)
f2 > f1 (a) vP + vR = 2 vQ
(c) If the wind blows from observer to the source, (b) The rate of change in beat frequency is maximum when
f2 < f1
the car passes through Q
(d) If the wind blows from the source to the observer,
(c) The plot below represents schematically the variation
f2 < f1
23. A student is performing an experiment using a resonance of beat frequency with time
column and a tuning fork of frequency 244 s–1. He is told v(t)
that the air in the tube has been replaced by another gas
(assume that the column remains filled with the gas). If the
minimum height at which r esonance occurs is P
( 0.350 ± 0.005) m, the gas in the tube is Q
vQ
(Useful information: 167 RT = 640J1/ 2 mole -1/ 2 ;
140 RT = 590J1/ 2 mole -1/ 2 . The molar masses M in grams R
t
10
are given in the options. Take the values of for each (d) The plot below represents schematically the variation
M
of beat frequency with time
gas as given there.) (JEE Adv. 2014)
æ 10 7ö v(t)
(a) Neon ç M = 20, = ÷
è 20 10 ø P
æ 10 3 ö
(b) Nitrogen ç M = 28, =
è 28 5 ÷ø vQ
Q
æ 10 9ö R
(c) Oxygen ç M = 32, = ÷
è 32 16 ø t
æ 10 17 ö
(d) Argon ç M = 36, =
è 36 32 ÷ø
EBD_7036
P-92 Topic-wise Solved Papers - PHYSICS
8. The vibrations of a string of length 60 cm fixed at both ends
E Subjective Problems are represented by the equation—
1. AB is a cylinder of length 1m fitted with a thin flexible æ px ö
diaphragm C at the middle and other thin flexible diaphragms y = 4 sin ç ÷ cos (96 pt) (1985 - 6 Marks)
è 15 ø
A and B at the ends. The portions AC and BC contain
hydrogen and oxygen gases respectively. The diaphragms Where x and y are in cm and t in seconds.
A and B are set into vibrations of same frequency. What is (i) What is the maximum displacement of a point at x = 5
the minimum frequency of these vibrations for which cm?
diaphragm C is a node? (Under the conditions of experiment (ii) Where are the nodes located along the string?
(iii) What is the velocity of the particle at x = 7.5 cm at
vH 2 = 1100 m/s, vO2 = 300 m/s). (1978) t = 0.25 sec.?
A C B (iv) Write down the equations of the component waves
whose superposition gives the above wave
H2 O2 9. Two tuning forks with natural frequencies of 340 Hz each
move relative to a stationary observer. One fork moves away
from the observer, while the other moves towards him at the
2. A copper wire is held at the two ends by rigid supports. At same speed. The observer hears beats of frequency 3 Hz.
30°C, the wire is just taut, with negligible tension. Find the Find the speed of the tuning fork. (1986 - 8 Marks)
speed of transverse waves in this wire at 10°C. 10. The following equations represent transverse waves :
Given : Young modulus of copper = 1.3 ´ 1011 N/m2 . z1 = A cos (kx - wt ); (1987 - 7 Marks)

Coefficient of linear expansion of copper = 1.7 ´ 10-5 oC -1. z2 = A cos (kx + wt ); z3 = A cos (ky - wt )
Identify the combination (s) of the waves which will produce
Density of copper = 9 ´ 103 kg/m3 . (1979)
(i) standing wave (s), (ii) a wave travelling in the directon
3. A tube of a certain diameter and of length 48 cm is open at making an angle of 45° degrees with the positive x and
both ends. Its fundamental frequency of resonance is found positive y axes. In each case, find the positions at which the
to be 320 Hz. The velocity of sound in air is 320 m/sec. resultant intensity is always zero.
Estimate the diameter of the tube. (1980) 11. A train approaching a hill at a speed of 40 km/hr sounds a
One end of the tube is now closed. Calculate the lowest whistle of frequency 580 Hz when it is at a distance of 1 km
frequency of resonance for the tube. from a hill. A wind with a speed of 40 km/hr is blowing in the
4. A source of sound of frequency 256 Hz is moving rapidly direction of motion of the train Find (1988 - 5 Marks)
towards wall with a velocity of 5 m/sec. How many beats (i) the Frequency of the whistle as heard by an observer
per second will be heard if sound travels at a speed of on the hill,
330 m/sec? (1981 - 4 Marks) (ii) the distance from the hill at which the echo from the hill
5. A string 25 cm long and having a mass of 2.5 gm is under is heard by the driver and its frequency.
tension. A pipe closed at one end is 40 cm long. When the (Velocity of sound in air =1,200 km/hr)
string is set vibrating in its first overtone and the air in the 12. A source of sound is moving along a circular orbit of radius
pipe in its fundamental frequency, 8 beats per second are 3 metres with an angular velocity of 10 rad/s. A sound
heard. It is observed that decreasing the tension in the string
detector located far away from the source is executing linear
decreases beat frequency. If the speed of sound in air is
simple harmonic motion along the line BD with an amplitude
320 m/s, find the tension in the string. (1982 - 7 Marks)
BC = CD = 6 metres. The frequency of oscillation of the
6. A uniform rope of length 12 m and mass 6 kg hangs vertically
from a rigid support. A block of mass 2 kg is attached to the 5
free end of the rope. A transverse pulse of wavelength 0.06 detector is per second. The source is at the point A when
p
m is produced at the lower end of the rope. What is the
the detector is at the point B. If the source emits a continous
wavelength of the pulse when it reaches the top of the rope?
sound wave of frequency 340 Hz, find the maximum and the
(1984 - 6 Marks) minimum frequencies recorded by the detector.
7. A steel wire of length 1 m, mass 0.1 kg and uniform (1990 - 7 Mark)
cross-sectional area 10–6 m2 is rigidly fixed at both ends.
The temperature of the wire is lowered by 20° C. If transverse 6m 6m
waves are set up by plucking the string in the middle, 3m · · ·
calculate the frequency of the fundamental mode of A B C D
vibration. A1
11 2
Given for steel Y = 2 ´ 10 N/m A2 · · ·
A B C D
-5 o
a = 1.21 ´ 10 per C (1984 - 6 Marks) A3
Waves P-93

13. The displacement of the medium in a sound wave is given (a) the time taken by the wave-pulse to reach the other
end R of the wire, and
by the equation y1 = A cos (ax + bt) where A, a and b are
(b) the amplitude of the reflected and transmitted
positive constants. The wave is reflected by an obstacle wave-pulses after the incident wave-pulse crosses the
situated at x = 0. The intensity of the reflected wave is 0.64 joint Q.
times that of the incident wave. (1991 - 4 × 2 Marks) 19. A 3.6 m long vertical pipe resonates with a source of
(a) What are the wavelength and frequency of incident frequency 212.5 Hz when water level is at certain height in
wave? the pipe. Find the height of water level (from the bottom of
(b) Write the equation for the reflected wave. the pipe) at which resonance occurs. Neglect end correction.
(c) In the resultant wave formed after reflection, find the Now, the pipe is filled to a height H ( » 3.6 m). A small hole is
maximum and minimum values of the particle speeds in drilled very close to its bottom and water is allowed to leak.
Obtain an expression for the rate of fall of water level in the
the medium.
pipe as a function of H. If the radii of the pipe and the hole
(d) Express the resultant wave as a superposition of a
are 2 × 10-2 m and 1 × 10-3 m respectively, calculate the time
standing wave and a travelling wave. What are the interval between the occurance of first two resonances.
positions of the antinodes of the standing wave ? Speed of sound in air is 340 m/s and g = 10 m/s2.
What is the direction of propagation of travelling wave? (2000 - 10 Marks)
14. Two radio stations broadcast their programmes at the same 20. A boat is traveling in a river with a speed 10 m/s along the
amplitude A and at slightly different frequencies w1 and stream flowing with a speed 2 m/s. From this boat, a sound
transmitter is lowered into the river through a rigid support.
w2 respectively, where w1 - w 2 = 103 Hz A detector The wavelength of the sound emitted from the transmitter
receives the signals from the two stations simultaneously. inside the water is 14.45 mm. Assume that attenuation of
sound in water and air is negligible.
It can only detect signals of intensity ³ 2A2 .
(a) What will be the frequency detected by a receiver kept
(1993 - 4 Marks)
inside the river downstream?
(i) Find the time interval between successive maxima of
(b) The transmitter and the receiver are now pulled up into
the intensity of the signal received by the detector.
(ii) Find the time for which the detector remains idle in air. The air is blowing with a speed 5 m/s in the direction
each cycle of the intensity of the signal. opposite the river stream. Determine the frequency of
15. A whistle emitting a sound of frequency 440 Hz is tied to a the sound detected by the receiver.
string of 1.5m length and rotated with an angular velocity of (Temperature of the air and water = 20oC; Density of river
20 rad s–1 in the horizontal plane. Calculate the range of water = 103 kg/m3;
frequencies heard by an observer stationed at a large Bulk modulus of the water = 2.088 ´ 109 Pa; Gas constant
distance from the whistle. (1996 - 3 Marks) R = 8.31 J/mol-K;
16. A band playing music at a frequency f is moving towards a Mean molecular mass of air = 28.8 ´ 10-3 kg/mol; CP/CV for
wall at a speed nb. A motorist is following the band with a air = 1.4) (2001 - 10 Marks)
speed vm. If v is the speed of sound, obtain an expression 21. Two narrow cylindrical pipes A and B have the same length.
for the beat frequency heard by the motorist. Pipe A is open at both ends and is filled with a monoatomic
(1997 - 5 Marks) gas of molar mass MA. Pipe B is open at one end and closed
17. The air column in a pipe closed at one end is made to vibrate in at the other end, and is filled with a diatomic gas of molar
its second overtone by a tuning fork of frequency 440 Hz. The mass MB. Both gases are at the same temperature.
speed of sound in air is 330 m s–1. End corrections may be (2002 - 5 Marks )
neglected. Let P0 denote the mean pressure at any point in the (a) If the frequency of the second harmonic of the
pipe, and DP0 the maximum amplitude of pressure variation. fundamental mode in pipe A is equal to the frequency
(a) Find the length L of the air column. (1998 - 8 Marks) of the third harmonic of the fundamental mode in pipe
(b) What is the amplitude of pressure variation at the middle B, determine the value of MA/MB.
of the column? (b) Now the open end of pipe B is also closed (so that the
(c) What are the maximum and minimum pressures at the pipe is closed at both ends). Find the ratio of the
open end of the pipe? fundamental frequency in pipe A to that in pipe B.
(d) What are the maximum and minimum pressures at the 22. A tuning fork of frequency 480 Hz resonates with a tube
closed end of the pipe? closed at one end of length 16 cm and diameter 5 cm in
18. A long wire PQR is made by joining two wires PQ and QR of fundamental mode. Calculate velocity of sound in air.
equal radii PQ has length 4.8 m and mass 0.06 kg. QR has (2003 - 2 Marks)
length 2.56 m and mass 0.2 kg. The wire PQR is under a 23. A string tied between x = 0 and x = l vibrates in fundamental
tension of 80 N. A sinusoidal wave-pulse of amplitude 3.5 mode. The amplitude A, tension T and mass per unit length
cm is sent along the wire PQ from the end P. No power is m is given. Find the total energy of the string.
dissipated during the propagation of the wave-pulse. (2003 - 4 Marks)
Calculate. (1999 - 10 Marks)
x=0 x=l
EBD_7036
P-94 Topic-wise Solved Papers - PHYSICS
24. A whistling train approaches a junction. An observer 25. A transverse harmonic disturbance is produced in a string.
standing at junction observes the frequency to be 2.2 KHz The maximum transverse velocity is 3 m/s and maximum
and 1.8 KHz of the approaching and the receding train transverse acceleration is 90 m/s2. If the wave velocity is 20
respectively. Find the speed of the train (speed of sound = m/s then find the waveform. (2005 - 4 Marks)
300 m/s) (2005 - 2 Marks)

F Match the Following

DIRECTIONS (Q. No. 1-2) : Each question contains statements given in two columns, which have to p q r s t
be matched. The statements in Column-I are labelled A, B, C and D, while the statements in Column-
A p q r s t
II are labelled p, q, r and s. Any given statement in Column-I can have correct matching with ONE OR
B p q r s t
MORE statement(s) in Column-II. The appropriate bubbles corresponding to the answers to these
questions have to be darkened as illustrated in the following example : C p q r s t
If the correct matches are A-p, s and t; B-q and r; C-p and q; and D-s then the correct darkening of D p q r s t
bubbles will look like the given.
1. Each of the properties of sound listed in the column A primarily depends on one of the quantities in column B. Write down the
matching pairs from the two columns. (1980)
Column A Column B
A. pitch p. Waveform
B. quality q. frequency
C. loudness r. intensity
2. Column I shows four systems, each of the same length L, for producing standing waves. The lowest possible natural frequency
of a system is called its fundamental frequency, whose wavelength is denoted as lf. Match each system with statements given
in Column II describing the nature and wavelength of the standing waves. (2011)
Column I Column II

(A) Pipe closed at one end (p) Longitudinal waves

(B) Pipe open at both ends (q) Transverse waves

(C) Stretched wire clamped at both ends (r) lf = L

(D) Stretched wire clamped at both ends (s) lf = 2L

and at mid-point (t) lf = 4L

PASSAGE - 2
G Comprehension Based Questions Two trains A and B moving with speeds 20 m/s and 30 m/s
respectively in the same direction on the same straight track, with
PASSAGE - 1 B ahead of A. The engines are at the front ends. The engine of
Waves y1 = A cos(0.5px - 100pt ) and y2 = A cos(0.46px - 92pt ) train A blows a long whistle.
Assume that the sound of the whistle is composed of components
are travelling along x-axis. (Here x is in m and t is in second) varying in frequency from f1 = 800 Hz to f2 = 1120 Hz, as shown in
1. Find the number of times intensity is maximum in time interval the figure. The spread in the frequency (highest frequency –
of 1 sec. (2006 – 5M, –2) lowest frequency) is thus 320 Hz. The speed of sound in still air is
(a) 4 (b) 6 (c) 8 (d) 10 340 m/s.
2. The wave velocity of louder sound is (2006 – 5M, –2) 4. The speed of sound of the whistle is (2007)
(a) 340 m/s for passengers in A and 310 m/s for passengers
(a) 100 m/s (b) 192 m/s in B
(c) 200 m/s (d) 96 m/s (b) 360 m/s for passengers in A and 310 m/s for passengers
3. The number of times y1 + y2 = 0 at x = 0 in 1 sec is in B
(a) 100 (b) 46 (2006 – 5M, –2) (c) 310 m/s for passengers in A and 360 m/s for passengers
(c) 192 (d) 96 in B
(d) 340 m/s for passengers in both the trains
Waves P-95

5. The distribution of the sound intensity of the whistle as


observed by the passengers in train A is best represented I Integer Value Correct Type
by (2007)
1. A 20 cm long string, having a mass of 1.0 g, is fixed at both
the ends. The tension in the string is 0.5 N. The string is set
Intensity

(a) into vibrations using an external vibrator of frequency 100


Hz. Find the separation (in cm) between the successive
f1 f 2 Frequency nodes on the string. (2009)
2. A stationary source is emitting sound at a fixed frequency
f0, which is reflected by two cars approaching the source.
Intensity

(b) The difference between the frequencies of sound reflected


from the cars is 1.2% of f0. What is the difference in the
f1 f 2 Frequency speeds of the cars (in km per hour) to the nearest integer ?
The cars are moving at constant speeds much smaller than
the speed of sound which is 330 ms–1. (2010)
Intensity

(c) 3. When two progressive waves y 1 = 4 sin (2x – 6t) and

f1 f 2 Frequency y2 = 3sin æç 2 x - 6t - p ö÷ are superimposed, the amplitude of


è 2ø
the resultant wave is (2010)
Intensity

4. Four harmonic waves of equal frequencies and equal


(d)
p 2p
f1 f 2 Frequency intensities I0 have phase angles 0, , and p. When they
3 3
6. The spread of frequency as observed by the passengers in are superposed, the intensity of the resulting wave is nI0.
train B is (2007)
The value of n is (JEE Adv. 2015)
(a) 310 Hz (b) 330 Hz
(c) 350 Hz (d) 290 Hz

Section-B JEE Main / AIEEE


1. Length of a string tied to two rigid supports is 40 cm. where x is expressed in metres and t in seconds. The speed
Maximum length (wavelength in cm) of a stationary wave of the wave - motion, in ms-1 , is [2003]
produced on it is [2002] (a) 300 (b) 600 (c) 1200 (d) 200
(a) 20 (b) 80 (c) 40 (d) 120. 7. A metal wire of linear mass density of 9.8 g/m is stretched
2. Tube A has both ends open while tube B has one end closed, with a tension of 10 kg-wt between two rigid supports 1
otherwise they are identical. The ratio of fundamental metre apart. The wire passes at its middle point between the
frequency of tube A and B is [2002] poles of a permanent magnet, and it vibrates in resonance
(a) 1 : 2 (b) 1 : 4 (c) 2 : 1 (d) 4 : 1. when carrying an alternating current of frequency n. The
3. A tuning fork arrangement (pair) produces 4 beats/sec with frequency n of the alternating source is [2003]
one fork of frequency 288 cps. A little wax is placed on the (a) 50 Hz (b) 100 Hz (c) 200Hz (d) 25Hz
unknown fork and it then produces 2 beats/sec. The 8. A tuning fork of known frequency 256 Hz makes 5 beats per
frequency of the unknown fork is [2002] second with the vibrating string of a piano. The beat
(a) 286 cps (b) 292 cps (c) 294 cps (d) 288 cps. frequency decreases to 2 beats per second when the tension
4. A wave y = a sin(wt–kx) on a string meets with another in the piano string is slightly increased. The frequency of
wave producing a node at x = 0. Then the equation of the the piano string before increasing the tension was
unknown wave is [2002] (a) 256 + 2 Hz (b) 256 – 2 Hz [2003]
(a) y = a sin( w t + kx) (b) y = –a sin( w t + kx) (c) 256 – 5 Hz (d) 256 + 5 Hz
(c) y = a sin( w t – kx) (d) y = –a sin( w t – kx) 9. The displacement y of a particle in a medium can be
5. When temperature increases, the frequency of a tuning fork expressed as,
(a) increases (b) decreases [2002] pö
æ
(c) remains same y = 10-6 sin ç100t + 20 x + ÷ m where t is in second and x
è 4ø
(d) increases or decreases depending on the material
6. The displacement y of a wave travelling in the x -direction is in meter. The speed of the wave is [2004]
given by (a) 20 m/s (b) 5 m/s
æ pö (c) 2000 m/s (d) 5p m/s
y = 10 - 4 sin ç 600 t - 2 x + ÷ metres
è 3 ø
EBD_7036
P-96 Topic-wise Solved Papers - PHYSICS
10. When two tuning forks (fork 1 and fork 2) are sounded 19. The equation of a wave on a string of linear mass density
simultaneously, 4 beats per second are heard. Now, some 0.04 kg m–1 is given by
tape is attached on the prong of the fork 2. When the
tuning forks are sounded again, 6 beats per second are é æ t x öù
y =0.02(m) sin ê2p ç - ÷ú .
heard. If the frequency of fork 1 is 200 Hz, then what was ë è 0.04(s ) 0.50(m) ø û
the original frequency of fork 2? [2005] The tension in the string is [2010]
(a) 202 Hz (b) 200 Hz (c) 204 Hz (d) 196 Hz (a) 4.0 N (b) 12.5 N (c) 0.5 N (d) 6.25 N
11. An observer moves towards a stationary source of 20. The transverse displacement y (x, t) of a wave on a string is
sound, with a velocity one-fifth of the velocity of sound.
What is the percentage increase in the apparent
given by y ( x, t ) = e
(
- ax 2 + bt 2 + 2 ab ) xt ) . This represents a:
frequency ? [2005]
(a) 0.5% (b) zero (c) 20 % (d) 5 % [2011]
12. A whistle producing sound waves of frequencies 9500 HZ b
and above is approaching a stationary person with speed v (a) wave moving in – x direction with speed
a
ms–1. The velocity of sound in air is 300 ms–1. If the person
can hear frequencies upto a maximum of 10,000 HZ, the (b) standing wave of frequency b
maximum value of v upto which he can hear whistle is 1
(c) standing wave of frequency
15 b
(a) 15 2 ms -1 (b) ms -1 [2006]
2 a
(d) wave moving in + x direction speed
(c) 15 ms -1 (d) 30 ms-1 b
21. A cylindrical tube, open at both ends, has a fundamental
13. A string is stretched between fixed points separated by 75.0
frequency, f, in air. The tube is dipped vertically in water so
cm. It is observed to have resonant frequencies of 420 Hz
that half of it is in water. The fundamental frequency of the
and 315 Hz. There are no other resonant frequencies between
air-column is now : [2012]
these two. Then, the lowest resonant frequency for this
(a) f (b) f/2 (c) 3f/4 (d) 2f
string is [2006]
22. A sonometer wire of length 1.5 m is made of steel. The tension
(a) 105 Hz (b) 1.05 Hz (c) 1050 Hz (d) 10.5 Hz
in it produces an elastic strain of 1%. What is the fundamental
14. A sound absorber attenuates the sound level by 20 dB. The
intensity decreases by a factor of [2007] frequency of steel if density and elasticity of steel are
(a) 100 (b) 1000 (c) 10000 (d) 10 7.7 × 103 kg/m3 and 2.2 × 1011 N/m2 respectively ?
15. While measuring the speed of sound by performing a [JEE-Main 2013]
resonance column experiment, a student gets the first (a) 188.5 Hz (b) 178.2 Hz
resonance condition at a column length of 18 cm during (c) 200.5 Hz (d) 770 Hz
winter. Repeating the same experiment during summer, she 23. A pipe of length 85 cm is closed from one end. Find the number
measures the column length to be x cm for the second of possible natural oscillations of air column in the pipe whose
resonance. Then [2008] frequencies lie below 1250 Hz. The velocity of sound in air is
(a) 18 > x (b) x > 54 340 m/s. [JEE Main 2014]
(c) 54 > x > 36 (d) 36 > x > 18 (a) 12 (b) 8 (c) 6 (d) 4
16. A wave travelling along the x-axis is described by the 24. A train is moving on a straight track with speed 20 ms–1. It is
equation y(x, t) = 0.005 cos (a x – bt). If the wavelength and blowing its whistle at the frequency of 1000 Hz. The
the time period of the wave are 0.08 m and 2.0s, respectively, percentage change in the frequency heard by a person
then a and b in appropriate units are [2008] standing near the track as the train passes him is (speed of
sound = 320 ms–1) close to : [JEE Main 2015]
0.08 2.0 (a) 18% (b) 24% (c) 6% (d) 12%
(a) a = 25.00 p , b = p (b) a= ,b =
p p 25. A uniform string of length 20 m is suspended from a rigid
0.04 1.0 p support. A short wave pulse is introduced at its lowest end.
(c) a= ,b = (d) a = 12.50p, b = It starts moving up the string. The time taken to reach the
p p 2.0 supports is : [JEE Main 2016]
17. Three sound waves of equal amplitudes have frequencies (take g = 10 ms–2)
(n –1), n, (n + 1). They superpose to give beats. The number
of beats produced per second will be : [2009] (a) 2 2s (b) 2 s (c) 2p 2 s (d) 2 s
(a) 3 (b) 2 (c) 1 (d) 4 26. A pipe open at both ends has a fundamental frequency f in
18. A motor cycle starts from rest and accelerates along a straight air. The pipe is dipped vertically in water so that half of it is
path at 2m/s2. At the starting point of the motor cycle there in water. The fundamental frequency of the air column is
is a stationary electric siren. How far has the motor cycle now : [JEE Main 2016]
gone when the driver hears the frequency of the siren at (a) 2f (b) f
94% of its value when the motor cycle was at rest? (Speed
f 3f
of sound = 330 ms–1) [2009] (c) (d)
(a) 98 m (b) 147 m (c) 196 m (d) 49 m 2 4
CHAPTER

12 Electrostatics
Section-A JEE Advanced/ IIT-JEE
6. The electric potential V at any point x, y, z (all in metres) in
A Fill in the Blanks space is given by V = 4x2 volts. The electric field at the point
1. Five identical capacitor plates, (1m, 0, 2 m) is .................... V/m. (1992 - 1 Mark)
each of area A, are arranged – 7. Five point charges, each of value + q
+ coul, are placed on five vertices of a
such that adjacent plates are
q q
at a distance d apart, the plates 1 2 34 5 regular hexagon of side L metres. The
are connected to a source of magnitude of the force on the point
emf V as shown in the figure charge of value – q coul. placed at the q
(1984- 2 Marks) centre of the hexagen is ....................
The charge on plate 1 is ..... and on plate 4 is ..... newton. (1992 - 1 Mark)
2. Figure shows line of constant q q

potential in a region in which an


electric field is present. The values
A B True/False
B
of the potential are written in 1. The work done in carrying a point charge from one point to
brackets. Of the points A, B and C, another in an electrostatic field depends on the path along
V

C
50

the magnitude of the electric field is which the point charge is carried. (1981- 2 Marks)
30 V
40

greatest at the point ...


1020V V

2. Two identical metallic spheres of exactly equal masses are


V

(1984- 2 Marks)
taken. One is given a positive charge Q coulombs and the
3. Two small balls having equal positive charges Q (coulomb)
other an equal negative charge. Their masses after charging
on each are suspended by two insulating strings of equal
are different. (1983 - 2 Marks)
length L (metre) from a hook fixed to a stand. The whole set
up is taken in a satellite into space where there is no gravity 3. A small metal ball is suspended in a uniform electric field
with the help of an insulated thread. If high energy X-ray
(state of weightlessness). The angle between the two strings
beam falls on the ball, the ball will be deflected in the direction
is ........... and the tension in each string is ........ newtons.
of the field. (1983 - 2 Marks)
(1986 - 2 Marks)
4. Two parallel plate capacitors of capacitances C and 2C are 4. Two protons A and B are placed in
between the two plates of a parallel
connected in parallel and charged to a potential difference
plate capacitor charged to a
V. The battery is then disconnected and the region between °
potential difference V as shown in B
the plates of the capacitor C is completely filled with a °
A
the figure. The forces on the two
material of dielectric constant K. The potential differences
protons are identical.
across the capacitors now becomes............. (1988 - 2 Marks)
(1986 - 3 Marks)
5. A point charge q moves from point V

P to point S along the path PQRS Y


5. A ring of radius R carries a uniformly distributed charge + Q.
(fig.) in a uniform electric field E
A point charge – q is placed on the axis of the ring at a
pointing parallel to the positive
distance 2R from the centre of the ring and released from
direction of the X-axis. The P rest. The particle executes a simple harmonic motion along
cooridnates of the points P, Q, R and S Q X the axis of the ring. (1988 - 2 Marks)
S are (a, b, O), (2a, O, O) (a, – b, O) R E
6. An electric line of forces in the x – y plane is given by the
and (O, O,O) respectively. The work
equation x2 + y2 = 1. A particle with unit positive charge,
done by the field in the above
initially at rest at the point x = 1, y = 0 in the x – y plane, will
process is given by the expression move along the circular line of force. (1988 - 2 Marks)
................ (1989 - 2 Marks)
EBD_7036
P-98 Topic-wise Solved Papers - PHYSICS
7. A parallel plate capacitor of area A, plate separation d and
C MCQs with One Correct Answer capacitance C is filled with three different dielectric materials
having dielectric constants k1, k2 and k3 as shown. If a
1. A hollow metal sphere of radius 5 cms is charged such that
single dielectric material is to be used to have the same
the potential on its surface is 10 volts. The potential at the
centre of the sphere is (1983 - 1 Mark) capacitance C in this capacitor, then its dielectric constant
(a) zero k is given by (2000S )
(b) 10 volts A/2 A/2
(c) same as at a point 5 cms away from the surface
(d) same as at a point 25 cms away from the surface K1 K2 d/2
2. Two point charges +q and –q are held fixed at (–d, o) and d
(d, o) respectively of a x-y coordinate system. Then (1995S) K3
(a) The electric field E at all points on the x-axis has the
same direction A
(b) Electric field at all points on y-axis is along x-axis 1 1 1
1 1 1 1
(c) Work has to be done in bringing a test charge from ¥ (a) = + + (b) = +
to the origin K K1 K 2 2 K3 K K1 + K 2 2 K3
(d) The dipole moment is 2qd along the x-axis
3. A parallel plate capacitor of capacitance C is connected to a (c) K = K1 K 2 + 2K3 (d) K = K1 + K2 + 2K3
K1 + K 2
battery and is charged to a potential difference V. Another
capacitor of capacitance 2C is similarly charged to a potential 8. Three positive charges of equal value q are placed at the
difference 2V. The charging battery is now disconnected vertices of an equilateral triangle. The resulting lines of force
and the capacitors are connected in parallel to each other in should be sketched as in (2001S)
such a way that the positive terminal of one is connected to
the negative terminal of the other. The final energy of the
configuration is (1995S)
3 25 9 (a) (b)
(a) zero (b) CV 2 (c) CV 2 (d) CV 2
2 6 2
4. Two identical metal plates are given positive charges Q1
and Q2 (<Q1) respectively. If they are now brought close
together to form a parallel plate capacitor with capacitance
C, the potential difference between them is (c) (d)
(1999 - 2 Marks)
(a) (Q1+Q2)/(2C) (b) (Q1+Q2) / C
(c) (Q1– Q2) /C (d) (Q1– Q2) /(2C) 9. Consider the situation shown in the figure. The capacitor A
5. For the circuit shown in Figure, which of the following has a charge q on it whereas B is uncharged. The charge
statements is true? (1999 - 2 Marks) appearing on the capacitor B a long time after the switch is
closed is (2001S)
q
+
+
+
+ S
+
+
A B
(a) With S1 closed V1=15 V, V2 = 20 V (a) zero (b) q/2
(b) With S3 closed, V1= V2 = 25 V (c) q (d) 2q
(c) With S1 and S2 closed, V1 = V2 = 0 10. A uniform electric field pointing in positive x-direction exists
in a region. Let A be the origin, B be the point on the x-axis
(d) With S1 and S3 closed, V1 = 30 V , V2 = 20 V at x = +1 cm and C be the point on the y-axis at y = +1 cm.
6. Three charges Q, +q and +q are placed at the vertices of a Then the potentials at the points A, B and C satisfy:
right-angled isosceles triangle as shown. The net (2001S)
electrostatic energy of the configuration is zero if Q is equal (a) VA < VB (b) VA > VB
to (2000S) (c) VA < VC (d) VA > VC
11. Two equal point charges are fixed at x = – a and x = + a on
-q
(a) the x-axis. Another point charge Q is placed at the origin.
1+ 2 Q
The change in the electrical potential energy of Q, when it is
-2q displaced by a small distance x along the x-axis, is
(b) approximately proportional to (2002S)
2+ 2 +q
(c) –2q
+q (a) x (b) x 2
a (c) x 3 (d) 1/x
(d) +q
Electrostatics P-99

12. Two identical capacitors, have the same capacitance C. One 17. A long, hollow conducting cylinder is kept coaxially inside
of them is charged to potential V1 and the other V2. The another long, hollow conducting cylinder of larger radius.
negative ends of the capacitors are connected together. Both the cylinders are initially electrically neutral. (2007)
When the positive ends are also connected, the decrease in (a) A potential difference appears between the two
energy of the combined system is (2002S)
cylinders when a charge density is given to the inner
(a)
1
4
(
C V12 - V22 ) (b)
1
4
C V12 + V22 ( ) cylinder.
(b) A potential difference appears between the two
1
( ) 1
( )
2 2 cylinders when a charge density is given to the outer
(c) C V1 - V2 (d) C V1 + V2
4 4 cylinder.
13. A metallic shell has a point charge ‘q’ kept inside its cavity. (c) No potential difference appears between the two
Which one of the following diagrams correctly represents cylinders when a uniform line charge is kept along the
the electric lines of forces? (2003S) axis of the cylinders
(d) No potential difference appears between the two
cylinders when same charge density is given to both
the cylinders.
(a) (b)
18. Consider a neutral conducting sphere. A positive point
charge is placed outside the sphere. The net charge on the
sphere is then (2007)
(a) negative and distributed uniformly over the surface of
the sphere
(c) (d) (b) negative and appears only at the point on the sphere
closest to the point charge
(c) negative and distributed non-uniformly over the entire
14. Six charges of equal magnitude, 3 positive and 3 negative surface of the sphere
are to be placed on PQRSTU corners of a regular hexagon, (d) zero
such that field at the centre is double that of what it would 19. A spherical portion has been removed from a solid sphere
have been if only one +ve charge is placed at R. Which of having a charge distributed uniformly in its volume as shown
the following arrangement of charge is possible for P, Q, R, in the figure. The electric field inside the emptied space is
S, T and U respectively. (2004S) (2007)
P Q
(a) zero everywhere
U R (b) non-zero and uniform
O
(c) non-uniform
T S
(a) + , +, +, –, –, – (b) –, + , +, +, –, – (d) zero only at its center
(c) –, + , +, –, +, – (d) + , –, +, –, +, – 20. Positive and negative point charges of equal magnitude are
15. A Gaussian surface in the figure is shown by dotted line. æ aö æ -a ö
The electric field on the surface will be (2004S) kept at çè 0, 0, ÷ø and ç 0, 0, ÷ respectively. The work
2 è 2ø
(a) due to q1 and q2 only
done by the electric field when another positive point charge
(b) due to q2 only q1 q2 is moved from (–a, 0, 0) to (0, a, 0) is (2007)
(c) zero –q1 (a) positive
(d) due to all (b) negative
16. Three infinitely long charge sheets are placed as shown in (c) zero
figure. The electric field at point P is (2005S) (d) depends on the path connecting the initial and final
positions
Z 21. Consider a system of three charges q/3, q/3 and –2q/3 placed
s at points A, B and C, respectively, as shown in the figure.
Z = 3a Take O to be the centre of the circle of radius R and angle
CAB = 60° (2008)
P y
-2s
Z=a
B
X
C
-s x
Z = -a O
60°
2s 4s 2s ˆ 4s ˆ A
(a) k̂ (b) k̂ (c) - k (d) - e k
e0 e0 e0 0
EBD_7036
P-100 Topic-wise Solved Papers - PHYSICS
q 1 2 2 1 2 1 s2 1 s2
(a) The electric field at point O is 2
directed along s R (b) s R (c)
8pe 0 R (a) e0 e0 (d)
e0 R e0 R 2
the negative x-axis
(b) The potential energy of the system is zero 26. A tiny spherical oil drop carrying a net charge q is balanced
(c) The magnitude of the force between the charges at C in still air with a vertical uniform electric field of strength
q2 81p
and B is ´ 105 Vm –1 . When the field is switched off, the drop
54pe 0 R 2 7
is observed to fall with terminal velocity 2 ´ 10 –3 ms –1. Given
q
(d) The potential at point O is g = 9.8 m s –2 , viscosity of the air = 1.8 ´ 10–5 Ns m–2 and
12pe0 R
22. A parallel plate capacitor C with plates of unit area and the density of oil = 900 kg m –3 , the magnitude of q is (2010)
separation d is filled with a liquid of dielectric constant (a) 1.6 ´ 10–19 C (b) 3.2 ´ 10 –19 C
K = 2. The level of liquid is d/3 initially. Suppose the liquid
level decreases at a constant speed v, the time constant as a (c) 4.8 ´ 10–19 C (d) 8.0 ´ 10 –19 C
function of time t is – (2008) r
27. Consider an electric field E = E0 xˆ where E0 is a constant.
6e 0 R
(a) The flux through the shaded area (as shown in the figure)
5d + 3vt z
due to this field is (2011)
(15d + 9vt ) e 0 R
(b) C (a) 2E0a2 (a,0,a) (a,a,a)
2d 2 - 3dvt - 9v 2t 2 d R 2
d/3 (b) 2 E0 a
6 e0 R
(c) (c) E0 a2
5d - 3vt
(15d - 9vt ) e 0 R E0 a 2 (0,0,0) (0,a,0)
y
(d) (d) x
2d 2 - 3dvt - 9v 2t 2 2
23. Three concentric metallic spherical shells of radii R, 2R, 3R, 28. A 2 mF capacitor is charged as shown in the figure. The
are given charges Q1, Q2, Q3, respectively. It is found that percentage of its stored energy dissipated after the switch
the surface charge densities on the outer surfaces of the S is turned to position 2 is (2011)
shells are equal. Then, the ratio of the charges given to the
(a) 0%
shells, Q1 : Q2 : Q3, is (2009)
(b) 20%
(a) 1 : 2 : 3 (b) 1 : 3 : 5
(c) 1 : 4 : 9 (d) 1 : 8 : 18 (c) 75%
24. A disc of radius a / 4 having a uniformly distributed charge (d) 80%
6C is placed in the x - y plane with its centre at (–a / 2, 0, 0). 29. Which of the field patterns given below is valid for electric
A rod of length a carrying a uniformly distributed charge 8C field as well as for magnetic field? (2011)
is placed on the x - axis from x = a /4 to x = 5a / 4. Two point
charges – 7 C and 3 C are placed at (a / 4, – a /4, 0) and
(– 3a /4, 3a / 4, 0), respectively. Consider a cubical surface (a) (b)
formed by six surfaces x = ± a / 2, y = ± a / 2, z = ± a / 2. The
electric flux through this cubical surface is (2009)
y

(c) (d)

30. A wooden block performs SHM on a frictionless surface


with frequency, n0. The block carries a charge +Q on its
r
–2C 2C 10C 12C surface. If now a uniform electric field E is switched-on as
(a) (b) e (c) (d) shown, then the SHM of the block will be (2011)
e0 0 e0 e0
25. A uniformly charged thin spherical shell of radius R carries
uniform surface charge density of s per unit area. It is made
of two hemispherical shells, held together by pressing them
with force F (see figure). F is proportional to (2010)

(a) of the same frequency and with shifted mean position.


F F (b) of the same frequency and with the same mean position
(c) of changed frequency and with shifted mean position.
(d) of changed frequency and with the same mean position.
Electrostatics P-101

31. Two large vertical and parallel metal plates having a


separation of 1 cm are connected to a DC voltage source of P
potential difference X. A proton is released at rest midway
between the two plates. It is found to move at 45° to the R
vertical JUST after release. Then X is nearly
(a) 1 ´ 10–5 V (b) 1 ´ 10–7 V (2012) 4Q
(c) 1 ´ 10 V –9 (d) 1 ´ 10–10 V
32. Consider a thin spherical shell of radius R with centre at the
origin, carrying uniform positive surface charge
density. The variation of the magnitude of the electric field 2R
r
E(r ) and the electric potential V(r) with the distance r from Sphere 3
the centre, is best represented by which graph? (2012) (a) E1 > E2 > E3 (b) E3 > E1 > E2
®
| E(r)| V(r) (c) E2 > E1 > E3 (d) E3 > E2 > E1

(a)
D MCQs with One or More than One Correct
O R r 1. Two equal negative charges –q are fixed at points (0, – a)
® and (0, a) on y – axis. A positive charge Q is released from
|E(r)| V(r) rest at the point (2a, 0) on the x - axis. The charge Q will
(1984- 2 Marks)
(a) execute simple harmonic motion about the origin
(b)
(b) move to the origin remain at rest
O R r (c) move to infinity
®
| E(r)| V(r) (d) execute oscillatory but not simple harmonic motion
2. A parallel plate air capacitor is connected to a battery. The
quantities charge, voltage, electric field and energy
(c) associated with this capacitor are given by Q0, V0, E0 and
U0 respectively. A dielectric slab is now introduced to fill
O R r
® the space between the plates with battery still in connection.
| E(r)| V(r) The corresponding quantities now given by Q, V, E and U
are related to the previous one as (1985 - 2 Marks)
(d) (a) Q > Q0 (b) V > V0

O R r (c) E > E0 (d) U > U 0


33. In the given circuit, a charge of +80 mC is given to the upper 3. A charge q is placed at the centre of the line joining two
plate of the 4 mF capacitor. Then in the steady state, the charge equal charges Q. The system of the three charges will be in
on the upper plate of the 3 mF capacitor is (2012) equilibrium if q is equal to : (1987 - 2 Marks)
(a) + 32 mC Q Q Q
(a) - (b) - (c) + Q (d) +
2 4 4 2
(b) + 40 mC 4. A parallel plate capacitor is charged and the charging battery
is then disconnected. If the plates of the capacitor are moved
(c) + 48 mC farther apart by means of insulating handles :
(1987 - 2 Marks)
(d) + 80 mC (a) the charge on the capacitor increases.
34. Charges Q, 2Q and 4Q are uniformly distributed in three (b) the voltage across the plates increases.
dielectric solid spheres 1, 2 and 3 of radii R/2, R and 2R (c) the capacitance increases.
respectively, as shown in figure. If magnitude of the electric (d) the electrostatic energy stored in the capacitor increases
fields at point P at a distance R from the centre of sphere 1, 2 5. A solid conducting sphere having a charge Q is surrounded
and 3 are E1, E2 and E3 respectively, then (JEE Adv. 2014) by an uncharged concentric conducting hollow spherical
P shell. Let the potential difference between the surface of the
P solid sphere and that of the outer surface of the hollow shell
R R
be V. If the shell is now given a charge of – 3Q, the new
Q
2Q potential difference between the same two surfaces is :
(1989 - 2 Marks)
R/2 (a) V (b) 2V
Sphere 1 Sphere 2 (c) 4V (d) – 2V
EBD_7036
P-102 Topic-wise Solved Papers - PHYSICS
6. Seven capacitors each of capacitance 2mF are to be 11. A dielectric slab of thickness d is inserted in a parallel plate
connected in a configuration to obtain an effective capacitor whose negative plate is at x = 0 and positive plate
is at x = 3d. The slab is equidistant from the plates. The
æ 10 ö capacitor is given some charge. As one goes from 0 to 3d,
capacitance of ç ÷ mF . Which of the combination (s) shown
è 11ø (a) the magnitude of the electric field remains the same.
in figure will achieve the desired result ? (1990 - 2 Marks) (b) the direction of the electric field remains the same.
(c) the electric potential increases continuously.
(d) the electric potential increases at first, then decreases
(a) (b) and again increases. (1998S - 2 Marks)
12. A charge +q is fixed at each of the points x = x0, x = 3x0,
x = 5x0,..... x = ¥ on the x axis, and a charge –q is fixed at
each of the points x = 2x0, x = 4x0, x = 6x0,.... x = ¥. Here x0
(c) (d) is a positive constant. Take the electric potential at a point
due to a charge Q at a distance r from it to be Q/(4pe0r).
7. A parallel plate capacitor of plate area A and plate separation Then, the potential at the origin due to the above system of
d is charged to potential difference V and then the battery is charges is (1998S - 2 Marks)
disconnected. A slab of dielectric constant K is then inserted
q
between the plates of the capacitor so as to fill the space (a) 0 (b)
between the plates. If Q, E and W denote respectively, the 8pe 0 x0 ln 2
magnitude of charge on each plate, the electric field between q ln2
the plates (after the slab is inserted), and work done on the (c) ¥ (d) 4pe0 x0
system, in question, in the process of inserting the slab, 13. A positively charged thin metal ring of radius R is fixed in
then (1991 - 2 Marks) the xy plane with its centre at the origin O. A negatively
e 0 AV e KAV charged particle P is released from rest at the point (0, 0, z0)
(a) Q = (b) Q = 0
d d where z0 > 0. Then the motion of P is (1998S - 2 Marks)
V e AV 2 é 1ù (a) periodic, for all values of z0 satisfying 0 < z0 < ¥
(c) E = (d) W = 0 1- (b) simple harmonic, for all values of z0 satisfying 0 < z0 £ R
Kd 2d êë K úû
(c) approximately simple harmonic, provided z0 < < R
8. Two identical thin rings, each of radius R metres, are coaxially
(d) such that P crosses O and continues to move along
placed a distance R metres apart. If Q1 coulomb, and Q2 the negative z axis towards z = - ¥
coulomb, are respectively the charges uniformly spread on 14. A non-conducting solid sphere of radius R is uniformly
the two rings, the work done in moving a charge q from the charged. The magnitude of the electric field due to the sphere
centre of one ring to that of the other is (1992 - 2 Marks) at a distance r from its centre (1998S - 2 Marks)
q(Q1 - Q2 ) ( 2 -1) (a) increases as r increases, for r < R.
(a) zero (b) (b) decreases as r increases, for 0 < r < ¥.
(4 2pe 0 R )
(c) decreases as r increases, for R < r < ¥.
q 2(Q1 + Q2 ) q (Q1 + Q2 ) ( 2 + 1) (d) is discontinuous at r = R.
(c) (d)
(4pe0 R) (4 2pe 0 R ) 15. An ellipsoidal cavity is carved within a perfect conductor. A
ur positive charge q is placed at the centre of the cavity. The
9. The magnitude of electric field E in the annular region of
points A and B are on the cavity surface as shown in the
a charged cylindrical capacitor. (1996 - 2 Marks) figure. Then (1999S - 3 Marks)
(a) is same throughout
(b) is higher near the outer cylinder than near the inner
cylinder
A
(c) varies as 1/r, where r is the distance from axis
(d) varies as 1/r2 where r is the distance from the axis.
10. A metallic solid sphere is placed in a uniform electric fied. B
q
The lines of force follow the path(s) shown in Figure as
(1996 - 2 Marks)

(a) electric field near A in the cavity = electric field near B


in the cavity
(b) charge density at A = charge density at B
(c) potential at A = potential at B
(a) 1 (b) 2 (d) total electric field flux through the surface of the cavity
(c) 3 (d) 4 is q/e0
Electrostatics P-103

16. A spherical symmetric charge system is centered at origin. 21. A cubical region of side a has its centre at the origin.
Given, Electric potential (2006S - 5 Marks) It encloses three fixed point charges, -q at (0, -a/4, 0), +3q
Q Q at (0, 0, 0) and -q at (0, +a/4, 0). Choose the correct options(s)
V= (r £ R0 ), V = (r > R0 ) (2012)
4pe0 R0 4pe0r z
V
a

–q
–q 3q

x
r (a) The net electric flux crossing the plane x = +a/2
R0
is equal to the net electric flux crossing the plane
(a) Within r = 2R0 total enclosed net charge is Q
x = -a/2
(b) Electric field is discontinued at r = R0
(b) The net electric flux crossing the plane y = +a/2 is more
(c) Charge is only present at r = R0
than the net electric flux crossing the plane
(d) Electrostatic energy is zero for r < R0
y = -a/2.
17. Under the influence of the Coulomb field of charge + Q, a
charge –q is moving around it in an elliptical orbit. Find out q
(c) The net electric flux crossing the entire region is e
the correct statement(s). (2009) 0
(a) The angular momentum of the charge – q is constant (d) The net electric flux crossing the plane z = +a/2 is equal
(b) The linear momentum of the charge – q is constant to the net electric flux crossing the plane x = +a/2.
(c) The angular velocity of the charge –q is constant 22. Six point charges are kept at the vertices of a regular hexagon
(d) The linear speed of the charge –q is constant of side L and centre O, as shown in the figure. Given that
18. A few electric field lines for a system of two charges Q1 and
Q2 fixed at two different points on the x-axis are shown in 1 q
K= , which of the following statement(s) is (are)
the figure. These lines suggest that (2010) 4pe 0 L2
correct? (2012)
L E–q
+q F
Q Q
P

(a) |Q1| > |Q2| A S T D


+2q O
(b) |Q1| < |Q2| –2q
(c) at a finite distance to the left of Q1 the electric field is
zero
(d) at a finite distance to the right of Q2 the electric field is R
zero B+q – qC
19. A spherical metal shell A of radius RA and a solid metal
(a) The electric field at O is 6K along OD
sphere B of radius RB(<RA) are kept far apart and each is
(b) The potential at O is zero
given charge ‘+Q’. Now they are connected by a thin metal
wire. Then (2011) (c) The potential at all points on the line PR is same
(d) The potential at all points on the line ST is same
(a) E inside
A =0 (b) QA > QB
23. Two non-conducting solid spheres of radii R and 2R, having
s A RB uniform volume charge densities r1 and r2 respectively,
(c) = (d) E onA
surface
< EB
on surface
s B RA touch each other. The net electric field at a distance 2R from
20. Which of the following statement(s) is/are correct? (2011) the centre of the smaller sphere, along the line joining the
(a) If the electric field due to a point charge varies as r –2.5 r1
instead of r –2, then the Gauss law will still be valid. centres of the spheres, is zero. The ratio can be
r2
(b) The Gauss law can be used to calculate the field
distribution around an electric dipole. (JEE Adv. 2013)
(c) If the electric field between two point charges is zero 32
somewhere, then the sign of the two charges is the same. (a) – 4 (b) -
25
(d) The work done by the external force in moving a
unit positive charge from point A at potential VA 32
(c) (d) 4
to point B at potential VB is (VB - VA). 25
EBD_7036
P-104 Topic-wise Solved Papers - PHYSICS
24. In the circuit shown in the figure, there are two parallel plate E1
capacitors each of capacitance C. The switch S1 is pressed (a) =1
E2 Q1 E1
first to fully charge the capacitor C1 and then released. The
switch S2 is then pressed to charge the capacitor C2. After E1 1
(b) =
some time, S2 is released and then S3 is pressed. After some E2 K
time (JEE Adv. 2013) Q1 3
(c) =
S1 S2 S3 Q2 K E2
Q2
C 2+ K
(d) =
C1 C2 C3 C1 K
2V0
28. The figures below depict two situations in which two infinitely
long static line charges of constant positive line charge
density l are kept parallel to each other. In their resulting
electric field, point charges q and –q are kept in equilibrium
(a) The charge on the upper plate of C1 is 2CV0 between them. The point charges are confined to move in
(b) The charge on the upper plate of C1 is CV0 the x direction only. If they are given a small displacement
(c) The charge on the upper plate of C2 is 0 about their equilibrium positions, th en the correct
(d) The charge on the upper plate of C2 is –CV0 statement(s) is(are) (JEE Adv. 2015)
25. Two non-conducting spheres of radii R1 and R2 and carrying
l l l l
uniform volume charge densities +r and –r, respectively,
are placed such that they partially overlap, as shown in the
x x
figure. At all points in the overlapping region +q –q

r –r (a) Both charges execute simple harmonic motion


(b) Both charges will continue moving in the direction of
their displacement
R1 R2 (c) Charge +q executes simple harmonic motion while
charge –q continues moving in the direction of its
displacement
(d) Charge –q executes simple harmonic motion while
(JEE Adv. 2013) charge +q continues moving in the direction of its
(a) The electrostatic field is zero displacement
(b) The electrostatic potential is constant 29. Consider a uniform spherical charge distribution of radius
(c) The electrostatic field is constant in magnitude R1 centred at the origin O. In this distribution, a spherical
(d) The electrostatic field has same direction cavity of radius R2, centred at P with distance OP = a
26. Let E1 (r), E2(r) and E3(r) be the respective electric field at a = R1 – R2 (see figure) is made. If the electric field inside the
distance r from a point charge Q, an infinitely long wire with r uruur
cavity at position r is E (r), then the correct statement(s) is
constant linear charge density l, and an infinite plane with
uniform surface charge density s. If E1(r0) = E2(r0) = E3(r0) (are) (JEE Adv. 2015)
at a given distance r0, then (JEE Adv. 2014)
(a) Q = 4spr02
l
(b) r0 =
2 ps
(c) E1 ( r0 / 2) = 2 E2 ( r0 / 2)
(d) E2 ( r0 / 2) = 4 E3 ( r0 / 2 ) ur
27. A parallel plate capacitor has a dielectric slab of dielectric (a) E is uniform, its magnitude is independent of R2 but
r
constant K between its plates that covers 1/3 of the area of its direction depends on r
its plates, as shown in the figure. The total capacitance of ur
(b) E is uniform, its magnitude depends on R2 and its
the capacitor is C while that of the portion with dielectric in r
direction depends on r
between is C1. When the capacitor is charged, the plate area ur
covered by the dielectric gets charge Q1 and the rest of the (c) E is uniform, its magnitude is independent of a but its
area gets charge Q2. The electric field in the dielectric is E1 r
direction depends on a
and that in the other portion is E2. Choose the correct option/ ur
options, ignoring edge effects. (JEE Adv. 2014) (d) E is uniform and both its magnitude and direction
r
depend on a
Electrostatics P-105

30. A parallel plate capacitor having plates of area S and plate is approaximately simple harmonic. Calculate the time period
separation d, has capacitance C1 in air. When two dielectrics of oscillations. (1982 - 5marks)
of different relative primitivities (e1 = 2 and e2 = 4) are 5. The figure shows two identical S
introduced between the two plates as shown in the figure, parallel plate capacitors connected
C2 to a battery with the switch S closed.
the capacitance becomes C2. The ratio C is The switch is now opened and the V A C B C
1
free space between the plates of the
(JEE Adv. 2015)
capacitors is filled with a dielectric
of dielectric constant (or relative
permittivity) 3. Find the ratio of the total electrostatic energy
stored in both capacitors before and after the introduction
of the dielectric. (1983 - 6 Marks)
6. Two fixed, equal, positive charges, each A +q
of magnitude 5 × 10–5 coul are located
at points A and B separated by a –q
O
distance of 6 m. An equal and opposite
charge moves towards them along the D C
line COD, the perpendicular bisector of
the line AB. (1985 - 6 Marks) B +q
The moving charge, when it reaches the point C at a
distance of 4 m from O, has a kinetic energy of 4 joules.
Calculate the distance of the farthest point D which the
negative charge will reach before returning towards C.
(a) 6/5 (b) 5/3 7. Three particles, each of mass 1 gm and carrying a charge q,
are suspended from a common point by insulated massless
(c) 7/5 (d) 7/3
strings, each 100 cm long. If the particles are in equilibrium
E Subjective Problems and are located at the corners of an equilateral triangle of
side length 3 cm, calculate the charge q on each particle.
1. Three charges each of value q, are placed at the corners of (Take g = 10 m/s2). (1988 - 5 Marks)
an equilateral triangle. A fourth charge Q is placed at the 8. A point particle of mass M is
centre of the triangle. (1978) attached to one end of a
massless rigid non- Q+q
(i) If Q = – q, will the charges at the corners move towards q
E
centre or fly away from it. conducting rod of length L. Q-q
(ii) For what value of Q will the charges remain stationary? Another point particle of the
In this situation how much work is done in removing same mass is attached to the
the charges to infinity? other end of the rod. The two particles carry charges +q and
2. A rigid insulated wire frame, in A
– q respectively. This arrangement is held in a region of a
the form of right triangle ABC is uniform electric field E such that the rod makes a small angle
a
set in a vertical plane. Two beads q (say of about 5 degree) with the field direction, fig. Find
of equal masses m each carrying an expression for the minimum time needed for the rod to
charges q1 and q2 are connected 30° 60° become parallel to the field after it is set free. (1989 - 8mark)
B C 9. Three concentric spherical metallic shells A, B and C of radii
by a chord of length l and can
a, b and c (a < b <c) have surface charge densities s , – s
slide without friction on the wires. Considering the case
and s respectively.. (1990 -7 Marks)
when the beads are stationary, determine : (1978)
(i) Find the potential of the three shells A, B and C.
(i) the angle a,
(ii) If the shells A and C are at the same potential, obtain
(ii) the tension in the chord, and the relation between the radii a, b and c.
(iii) the normal reactions on the beads. 10. Two fixed charges – 2Q and Q are located at the points with
If the chord is now cut, what are the values of the charges coordinates (–3a, 0) and (+ 3a, 0) respectively in the x-y
for which the beads continue to remain stationary? plane. (1991 - 4 + 2 + 2 Marks)
3. A charge ‘Q’ is distributed over two concentric hollow (a) Show that all points in the x-y plane where the electric
spheres of radii ‘r’ and ‘R’ (>r) such that the surface densities potential due to the two charges is zero, lie on a circle.
are equal. Find the potential at the common centre. Find its radius and the location of its centre.
(1981- 3 Marks) (b) Give the expression V (x) at a general point on the
4. A thin fixed ring of radius 1 metre has a positive charge 1 × 10–5 x - axis and sketch the function V (x) on the whole x-axis.
coulomb uniformly distributed over it. A particle of mass (c) If a particle of charge + q starts form rest at the centre
0. 9 gm and having a negative charge of 1 × 10–6 coulomb is of the circle, show by a short quantative argument that
placed on the axis at a distance of 1 cm from the centre of the the particle eventually crosses the circle. Find its speed
ring. show that the motion of the negatively charged particle when it does so.
EBD_7036
P-106 Topic-wise Solved Papers - PHYSICS
11. (a) A charge of Q coulomb is uniformly distributed over a 16. Two capacitors A and B with capacities 3 mF and 2 mF are
spherical volume of radius R metres. Obtain an charged to a potential difference of 100 V and 180 V
expression for the energy of the system. respectively. The plates of the capacitors are connected as
(b) What will be the corresponding expression for the shown in the figure with one wire from each capacitor free.
energy needed to completely disassemble the planet The upper plate of A is positive and that of B is negative. An
earth against the gravitational pull amongst its uncharged 2 mF capacitor C with lead wires falls on the free
constituent particles ? ends to complete the circuit. Calculate (1997 - 5 Marks)
Assume the earth to be a sphere of uniform
mass density. Calculate this energy, given the product
of the mass and the radius of the earth to be
2.5 × 1031 kg. m.
(c) If the same charge of Q coulomb as in part (a) above is
given to a spherical conductor of the same radius R,
what will be energy of the system ?(1992 - 10 Marks)
12. Two parallel plate capacitors A and B have the same (i) the final charge on the three capacitors. and
separation d = 8.85 × 10–4 m between the plates. The plate (ii) the amount of electrostatic energy stored in the system
area of A and B are 0.04 m2 and 0.02m2 respectively. A slab of before and after the completion of the circuit.
dielectric constant (relative permittivity) K = 9 has 17. A conducting sphere S1 of radius r is attached to an insulating
dimensions such that it can exactly fill the space between handle. Another conducting sphere S2 of radius R is
the plates of capacitor B. (1993 - 2 + 3 + 2 Marks) mounted on an insulating stand. S2 is initially uncharged.
S1 is given a charge Q, brought into contact with S2, and
A B A B removed. S1 is recharged such that the charge on it is again
Q; and it is again brought into contact with S2 and removed.
This procedure is repeated n times. (1998 - 8 Marks)
(a) Find the electrostatic energy of S2 after n such contacts
110V with S1.
(a) (b) (c)
(b) What is the limiting value of this energy as n ® ¥ ?
(i) The dielectric slab is placed inside A as shown in figure
(a). A is then charged to a potential difference of 110V. 18. A non-conducting disc of radius a and uniform positive
surface charge density s is placed on the ground, with its
Calculate the capacitance of A and the energy stored in it.
axis vertical. A particle of mass m and positive charge q is
(ii) The battery is disconnected and then the dielectric
dropped, along the axis of the disc, from a height H with
slab is moved from A. Find the work done by the external
agency in removing the slab from A. zero initial velocity. The particle has q m = 4 Î0 g / s
(iii) The same dielectric slab is now placed inside B, filling (1999 - 10 Marks)
it completely. The two capacitors A and B are then (a) Find the value of H if the particle just reaches the disc.
connected as shown in figure (c). Calculate the energy (b) Sketch the potential energy of the particle as a function
stored in the system. of its height and find its equilibrium position.
13. A circular ring of radius R with uniform positive charge 19. Four point charges +8mC, –1mC, –1mC, and +8mC are fixed
density l per unit length is located in the y-z plane with its
27 3 3 27
centre at the origin O. A particle of mass m and positive at the points - m,- m, + m and + m
2 2 2 2
charge q is projected from the point P ( R 3, 0, 0) on the respectively on the y-axis. A particle of mass 6 × 10–4 kg and
positive x-axis directly towards O, with an initial speed v. charge +0.1 mC moves along the -x direction. Its speed at
Find the smallest (non-zero) value of the speed v such that x = + ¥ is V0. Find the least value of V0 for which the particle
the particle does not return to P. (1993-4 Marks) will cross the origin. Find also the kinetic energy of the
14. Two square metal plates of side 1 m are kept 0.01 m apart like particle at the origin. Assume that space is gravity free.
a parallel plate capacitor in air in such a way that one of their 1
edges is perpendicular to an oil surface in a tank filled with Given = 9 ´ 109 Nm 2 / C 2 . (2000 - 10 Marks)
4pe 0
an insulating oil. The plates are connected to a battery of
emf 500 V. The plates are then lowered vertically into the oil 20. Charges +q and –q are located at the corners of a cube of
side as show in the figure. Find the work done to separate
at a speed of 0.001 ms–1. Calculate the current drawn from
the charges to infinite distance. (2003 - 2 Marks)
the battery during the process. (Dielectric constant of
–q
oil = 11, e0 = 8.85 × 10–12C2N–1m–1) (1994 - 6 Marks) +q
15. The capacitance of a parallel plate –q
+q
capacitor with plate area A and
separation d is C. The space
between the plates is filled with
two wedges of dielectric constants +q –q
K1 and K2, respectively. Find the capacitance of the resulting
–q +q
capacitor. (1996 - 2 Marks)
Electrostatics P-107

21. A charge +Q is fixed at the origin of the co-ordinate system 22. Two uniformly charged large plane sheets S1 and S2 having
uur charge densities s1 and s2 (s1 > s2) are placed at a distance
while a small electric dipole of dipole moment p pointing
d parallel to each other. A charge q0 is moved along a line of
away from the charge along the x-axis is set free from a point length a(a < d) at an angle 45º with the normal to S1. Calculate
far away from the origin. (2003 - 4 Marks) the work done by the electric field (2004)
(a) Calculate the K.E. of the dipole when it reaches to a 23. A conducting liquid bubble of radius a and thickness
point (d, 0). t (t << a) is charged to potential V. If the bubble collapses to
(b) Calculate the force on the charge +Q at this moment. a droplet, find the potential on the droplet. (2005 - 2 Marks)

F Match the Following

DIRECTIONS (Q. No. 1) : Each question contains statements given in two columns, which have to be p q r s t
matched. The statements in Column-I are labelled A, B, C and D, while the statements in Column-II are
A p q r s t
labelled p, q, r and s. Any given statement in Column-I can have correct matching with ONE OR MORE
B p q r s t
statement(s) in Column-II. The appropriate bubbles corresponding to the answers to these questions
have to be darkened as illustrated in the following example : C p q r s t
If the correct matches are A-p, s and t; B-q and r; C-p and q; and D-s then the correct darkening of D p q r s t
bubbles will look like the given.
1. Six point charges, each of the same magnitude q, are arranged in different manners as shown in Column-II. In each case, a point
M and line PQ passing through M are shown. Let E be the electric field and V be the electric potential at M (potential at infinity
is zero) due to the given charge distribution when it is at rest. Now, the whole system is set into rotation with a constant angular
velocity about the line PQ. Let B be the magnetic field at M and m be the magnetic moment of the system in this condition.
Assume each rotating charge to the equivalent to a steady current. (2009)

Column-I Column-II
+ – Q
A) E=0 (p) Charges are at the corners of a regular hexagon. M is at the centre of
– + the hexagon. PQ is perpendicular to the plane of the hexagon .
M

P+ –

– + – + – +
(B) V¹0 (q) Charges are on a line perpendicular to PQ at equal intervals. M is the
M mid-point between the two innermost charges.
Q

Q
+ – +

(C) B=0 (r) – M – Charges are placed on two coplanar insulating rings at equal intervals.
M is the common centre of the rings. PQ is perpendicular to the plane
of the rings.
P +
– + –
(D) m ¹ 0 (s) M Charges are placed at the corners of a rectangle of sides a and 2a and
P Q
at the mid points of the longer sides. M is at the centre of the rectangle.
– + – PQ is parallel to the longer sides.

P
+ –

(t) M–
Charges are placed on two coplanar, identical insulating rings at equal
+ + –
intervals. M is the mid-point between the centres of the rings. PQ is
Q perpendicular to the line joining the centres and coplanar to the rings.
EBD_7036
P-108 Topic-wise Solved Papers - PHYSICS

DIRECTIONS (Q. No. 2) : Following question has matching lists. The codes for the lists have choices (a), (b), (c) and (d) out of
which ONLY ONE is correct.
2. Four charges Q1, Q2, Q3 and Q4 of same magnitude are fixed along the x axis at x = – 2a, – a, + a and + 2a, respectively. A positive
charge q is placed on the positive y axis at a distance b > 0. Four options of the signs of these charges are given in List-I. The
direction of the forces on the charge q is given in List-II. Match List-I with List-II and select the correct answer using the code
given below the lists. (JEE Adv. 2014)
List - I List - II
P. Q1, Q2, Q3, Q4 all 1. +x q(0, b)
positive
Q. Q1, Q2 positive; 2. –x
Q3, Q4 negative
R. Q1, Q4 positive; 3. +y Q1 Q2 Q3 Q4
Q2, Q3 negative (– 2 a, 0) (– a, 0) (+ a , 0) (+ 2a, 0)
S. Q1, Q3 positive; 4. –y
Q2, Q4 negative
Codes:
(a) P-3, Q-1, R-4, S-2 (b) P-4, Q-2, R-3, S-1
(c) P-3, Q-1, R-2, S-4 (d) P-4, Q-2, R-1, S-3

of a light weight and soft material and coated with a conducting


G Comprehension Based Questions material are placed on the bottom plate. The balls have a radius r
<<h. Now a high voltage source (HV) is connected across the
PASSAGE-I conducting plates such that the bottom plate is at +V0 and the
The nuclear charge (Ze) is non-uniformly distributed within a top plate at –V0 . Due to their conducting surface, the balls will
nucleus of radius R. The charge density r (r) [charge per unit get charged, will become equipotential with the plate and are
volume] is dependent only on the radial distance r from the centre repelled by it. The balls will eventually collide with the top plate,
of the nucleus as shown in figure The electric field is only along where the coefficient of restitution can be taken to be zero due to
the radial direction. (2008) the soft nature of the material of the balls. The electric field in the
chamber can be considered to be that of a parallel plate capacitor.
Assume that there are no collisions between the balls and the
interaction between them is negligible. (Ignore gravity)
d
A


r HV
O a R
1. The electric field at r = R is +
(a) independent of a
(b) directly proportional to a
(c) directly proportional to a2 4. Which one of the following statements is correct?
(d) inversely proportional to a (JEE Adv. 2016)
2. For a = 0, the value of d (maximum value of r as shown in the (a) The balls will stick to the top plate and remain there
figure) is – (b) The balls will bounce back to the bottom plate carrying
3Ze 3Ze the same charge they went up with
(a) 3 (b) (c) The balls will bounce back to the bottom plate carrying
4pR pR 3 the opposite charge they went up with
4Ze Ze (d) The balls will execute simple harmonic motion between
(c) 3 (d) the two plates
3pR 3pR 3
3. The electric field within the nucleus is generally observed 5. The average current in the steady state registered by the
to be linearly dependent on r. This implies. ammeter in the circuit will be (JEE Adv. 2016)
(a) a = 0 (b) a = R/2 (a) zero
(c) a = R (d) a = 2R/3 (b) proportional to the potential V0
PASSAGE-II (c) proportional to V01/2
Consider an evacuated cylindrical chamber of height h having
rigid conducting plates at the ends and an insulating curved (d) proportional to V02
surface as shown in the figure. A number of spherical balls made
Electrostatics P-109

the figure. The magnitude of the electric field at the point P,


H Assertion & Reason Type Questions which is at a distance 2R from the axis of the cylinder, is
1. STATEMENT-1 : For practical purposes, the earth is used 23rR
as a reference at zero potential in electrical circuits. given by the expression . The value of k is
16K e 0
and
STATEMENT-2 : The electrical potential of a sphere of (2012)
radius R with charge Q uniformly distributed on the surface
Q
is given by 4pe R . (2008)
0
(a) Statement-1 is True, Statement-2 is True; Statement-2 is
a correct explanation for Statement-1
(b) Statement-1 is True, Statement-2 is True; Statement-2
is NOT a correct explanation for Statement-1
(c) Statement -1 is True, Statement-2 is False
(d) Statement -1 is False, Statement-2 is True

I Integer Value Correct Type


4. An infinitely long uniform line charge distribution of charge
1. A solid sphere of radius R has a charge Q distributed in its per unit length l lies parallel to the y-axis in the y – z plane at
volume with a charge density r = kra, where k and a are
constants and r is the distance from its centre. 3
z= a (see figure). If the magnitude of the flux of the
R 1 2
If the electric field at r =is times that at r = R, find the
electric field through the rectangular surface ABCD lying in
2 8
value of a. (2009) lL
2. Four point charges, each of +q, are rigidly fixed at the four the x – y plane with its centre at the origin is ne
0
corners of a square planar soap film of side ‘a’. The surface
tension of the soap film is g. The system of charges and (e0 = permittivity of free space), then the value of n is
z
éq 2 ù1/ N
planar film are in equilibrium, and a = k ê ú , where ‘k’
êë g úû D
L 3
a
x C 2
is a constant. Then N is (2011) a y
3. An infinitely long solid cylinder of radius R has a uniform O (JEE Adv. 2015)
volume charge density r. It has a spherical cavity of radius A
B
R/2 with its centre on the axis of the cylinder, as shown in

Section-B JEE Main / AIEEE


1. On moving a charge of 20 coulomb by 2 cm, 2 J of work is (a) q /4 p Î0 L H
D
E
done, then the potential difference between the points is A
(b) zero O
(a) 0.1 V (b) 8 V [2002] q q
(c) 2 V (d) 0.5 V. (c) q/2 p Î0 L G C
2. If there are n capacitors in parallel connected to V volt source, (d) q/3 p Î0 L
F B L
then the energy stored is equal to [2002]
4. If a charge q is placed at the centre of the line joining two
1 equal charges Q such that the system is in equilibrium then
(a) CV (b) nCV2
2 the value of q is [2002]
(a) Q/2 (b) -Q/2
1 (c) Q/4 (d) -Q/4
(c) CV2 (d) CV2
2n 5. Capacitance (in F) of a spherical conductor with radius 1 m
3. A charged particle q is placed at the centre O of cube of is [2002]
length L (A B C D E F G H). Another same charge q is placed
(a) 1.1´ 10 -10 (b) 10 -6
at a distance L from O. Then the electric flux through ABCD
is [2002] (c) 9 ´ 10 -9 (d) 10 -3
EBD_7036
P-110 Topic-wise Solved Papers - PHYSICS
6. If the electric flux entering and leaving an enclosed surface brought in contact with B, then brought in contact with C
respectively is f1 and f 2 , the electric charge inside the surface and finally removed away from both. The new force of
repulsion between B and C is [2004]
will be [2003]
(a) F/8 (b) 3 F/4
(a) (f2 - f1 )e o (b) (f1 + f2 ) / e o (c) F/4 (d) 3 F/8
(c) (f2 - f1) / e o (d) (f1 + f2 )e o 13. A charge particle ‘q’ is shot towards another charged particle
‘Q’ which is fixed, with a speed ‘v’. It approaches ‘Q’ upto
7. A sheet of aluminium foil of negligible thickness is
a closest distance r and then returns. If q were given a
introduced between the plates of a capacitor. The
speed of ‘2v’ the closest distances of approach would be
capacitance of the capacitor [2003]
[2004]
(a) decreases (b) remains unchanged
(a) r/2 (b) 2 r
(c) becomes infinite (d) increases
(c) r (d) r/4
8. A thin spherical conducting shell of radius R has a charge q.
Another charge Q is placed at the centre of the shell. The 14. Four charges equal to -Q are placed at the four corners of a
square and a charge q is at its centre. If the system is in
R equilibrium the value of q is [2004]
electrostatic potential at a point P a distance from the
2 Q Q
centre of the shell is [2003] (a) - (1 + 2 2) (b) (1 + 2 2)
2 4
2Q 2Q 2q
(a) 4pe o R (b) 4pe R - 4pe R (c) -
Q
(1 + 2 2) (d)
Q
(1 + 2 2)
o o
4 2
2Q q (q + Q)2 15. A charged oil drop is suspended in a uniform field of 3×104
(c) + (d) 4pe R v/m so that it neither falls nor rises. The charge on the drop
4pe o R 4pe o R o
will be (Take the mass of the charge = 9.9×10–15 kg and
9.
-18
The work done in placing a charge of 8 ´ 10 coulomb on g = 10 m/s2) [2004]
a condenser of capacity 100 micro-farad is [2003] (a) 1.6×10–18 C (b) 3.2×10–18 C
(c) 3.3×10–18 C (d) 4.8×10–18 C
(a) 16 ´ 10 -32 joule (b) 3.1´10 -26 joule 16. Two point charges + 8q and – 2q are located at x = 0 and
-10 -32 x = L respectively. The location of a point on the x axis at
(c) 4 ´10 joule (d) 32 ´ 10 joule which the net electric field due to these two point charges is
10. Three charges –q1 , +q2 and –q3 are place as shown in the zero is [2005]
figure. The x - component of the force on –q1 is proportional
L
to [2003] (a) (b) 2 L
4
Y (c) 4 L (d) 8 L
-q3 17. Two thin wire rings each having a radius R are placed at a
distance d apart with their axes coinciding. The charges on
the two rings are +q and -q. The potential difference
a between the centres of the two rings is [2005]
b
+q2 q é1 1 ù
-q1 X (a) ê - ú
2p Î0 êë R R + d2
2 úû
q2 q3 q2 q3
(a) - cos q (b) + sin q
b 2
a 2 b 2
a2 qR
(b)
4p Î0 d 2
q2 q3 q2 q3
(c) + cos q (d) - sin q
b2 a2 b2 a2 q é1 1 ù
(c) ê - ú
11. The length of a given cylindrical wire is increased by 100%. 4p Î0 ëê R R 2 + d 2 úû
Due to the consequent decrease in diameter the change in
the resistance of the wire will be [2003] (d) zero
(a) 200% (b) 100% 18. A parallel plate capacitor is made by stacking n equally
(c) 50% (d) 300% spaced plates connected alternatively. If the capacitance
12. Two spherical conductors B and C having equal radii and between any two adjacent plates is ‘C’ then the resultant
carrying equal charges on them repel each other with a force capacitance is [2005]
F when kept apart at some distance. A third spherical (a) (n + 1) C (b) (n – 1) C
conductor having same radius as that B but uncharged is (c) nC (d) C
Electrostatics P-111

19. A charged ball B hangs from a silk thread S, which makes an 25. Charges are placed on the vertices of a square as shown.
angle q with a large charged conducting sheet P, as shown ur
Let E be the electric field and V the potential at the centre.
in the figure. The surface charge density s of the sheet is If the charges on A and B are interchanged with those on D
proportional to [2005] and C respectively, then [2007]
(a) cot q P
ur q q
q (a) E changes, V remains unchanged A B
(b) cos q S ur
(b) E remains unchanged, V changes
(c) tan q ur
(c) both E and V change
(d) sin q B ur D C
(d) E and V remain unchanged -q -q
20. A fully charged capacitor has a capacitance ‘C’. It is
discharged through a small coil of resistance wire embed- 26. The potential at a point x (measured in m m) due to some
ded in a thermally insulated block of specific heat capacity charges situated on the x-axis is given by
‘s’ and mass ‘m’. If the temperature of the block is raised by V(x) = 20/(x2 – 4) volt
‘ DT ’, the potential difference ‘V’ across the capacitance is The electric field E at x = 4 m m is given by [2007]
mC DT 2mC DT (a) (10/9) volt/ m m and in the +ve x direction
(a) (b) [2005]
s s (b) (5/3) volt/ m m and in the –ve x direction
2ms DT ms DT (c) (5/3) volt/ m m and in the +ve x direction
(c) (d)
C C (d) (10/9) volt/ m m and in the –ve x direction
21. An electric dipole is placed at an angle of 30° to a non- 27. A parallel plate condenser with a dielectric of dielectric
uniform electric field. The dipole will experience [2006] constant K between the plates has a capacity C and is
(a) a translational force only in the direction of the field charged to a potential V volt. The dielectric slab is slowly
(b) a translational force only in a direction normal to the removed from between the plates and then reinserted. The
direction of the field net work done by the system in this process is [2007]
(c) a torque as well as a translational force 1
(a) zero (b) ( K - 1) CV 2
(d) a torque only 2
22. Two insulating plates are both uniformly charged in such a (c) CV 2 ( K - 1) (d) ( K - 1) CV 2
way that the potential difference between them is K
V2 – V1 = 20 V. (i.e., plate 2 is at a higher potential). The 28. If gE and gM are the accelerations due to gravity on the
plates are separated by d = 0.1 m and can be treated as surfaces of the earth and the moon respectively and if
infinitely large. An electron is released from rest on the inner Millikan’s oil drop experiment could be performed on the
surface of plate 1. What is its speed when it hits plate 2? two surfaces, one will find the ratio [2007]
(e = 1.6 × 10–19 C, me = 9.11 × 10–31 kg) [2006]
electronic charge on the moon
(a) 2.65 × 106 m/s to be
Y electronic charge on the earth
(b) 7.02 × 1012 m/s 0.1 m (a) gM / g E (b) 1
X
(c) 1.87 × 106 m/s (c) 0 (d) gE / gM
(d) 32 × 10–19 m/s 29. A parallel plate capacitor with air between the plates has
1 2
capacitance of 9 pF. The separation between its plates is
23. Two spherical conductors A and B of radii 1 mm and 2 mm
‘d’. The space between the plates is now filled with two
are separated by a distance of 5 cm and are uniformly
dielectrics. One of the dielectrics has dielectric constant k1
charged. If the spheres are connected by a conducting wire
then in equilibrium condition, the ratio of the magnitude of d
= 3 and thickness while the other one has dielectric
the electric fields at the surfaces of spheres A and B is 3
(a) 4 : 1 (b) 1 : 2 [2006] 2d
constant k2 = 6 and thickness . Capacitance of the
(c) 2 : 1 (d) 1 : 4 3
capacitor is now [2008]
24. An electric charge 10–3 m C is placed at the origin (0, 0) of
(a) 1.8 pF (b) 45 pF
X – Y co-ordinate system. Two points A and B are situated at (c) 40.5 pF (d) 20.25 pF
∋ (
2, 2 and (2, 0) respectively. The potential difference 30. A thin spherical shell of radus R has charge Q spread
uniformly over its surface. Which of the following graphs
between the points A and B will be [2007] most closely represents the electric field E(r) produced by
(a) 4.5 volts (b) 9 volts the shell in the range 0 £ r < ¥, where r is the distance from
(c) Zero (d) 2 volt the centre of the shell? [2008]
EBD_7036
P-112 Topic-wise Solved Papers - PHYSICS
E(r) E(r) 36. Let there be a spherically symmetric charge distribution with
æ5 rö
charge density varying as r(r ) = r0 çè - ÷ø upto r = R ,
4 R
(a) r (b) R
r
and r(r ) = 0 for r > R , where r is the distance from the
O R O

E(r) E(r)
origin. The electric field at a distance r(r < R) from the origin
is given by [2010]
r0 r æ 5 r ö 4pr0 r æ 5 r ö
(c) r (d) r (a) ç - ÷ (b) ç - ÷
O R O R 4e 0 è 3 R ø 3e 0 è 3 R ø
31. Two points P and Q are maintained at the potentials of 4r0 r æ 5 r ö r0 r æ 5 r ö
10 V and – 4 V, respectively. The work done in moving (c) ç - ÷ (d) 3ε çè 4 - R ÷ø
4ε 0 è 4 R ø 0
100 electrons from P to Q is : [2009]
(a) 9.60 × 10–17J (b) –2.24 × 10–16 J 37. Two identical charged spheres suspended from a common
(c) 2.24 × 10–16 J (d) –9.60× 10–17 J point by two massless strings of length l are initially a
32. A charge Q is placed at each of the opposite corners of a distance d(d << l) apart because of their mutual repulsion.
square. A charge q is placed at each of the other two corners. The charge begins to leak from both the spheres at a constant
If the net electrical force on Q is zero, then Q/q equals: [2009] rate. As a result charges approach each other with a velocity
1 v. Then as a function of distance x between them, [2011]
(a) –1 (b) 1 (c) - (d) -2 2 (a) v µ x–1 (b) v µ x½
2
33. This question contains Statement-1 and Statement-2. Of (c) v µ x (d) v µ x–½
the four choices given after the statements, choose the one 38. The electrostatic potential inside a charged spherical ball is
that best describes the two statements. [2009] given by f = ar2 + b where r is the distance from the centre
Statement-1 : For a charged particle moving from point P to and a, b are constants. Then the charge density inside the
point Q, the net work done by an electrostatic field on the ball is : [2011]
particle is independent of the path connecting point P to (a) –6ae 0 r (b) –24pae0
point Q.
(c) –6ae0 (d) –24pae0 r
Statement-2 : The net work done by a conservative force
on an object moving along a closed loop is zero. 39. In a uniformly charged sphere of total charge Q and radius
(a) Statement-1 is true, Statement-2 is true; Statement-2 is R, the electric field E is plotted as function of distance from
the correct explanation of Statement-1. the centre, The graph which would correspond to the above
(b) Statement-1 is true, Statement-2 is true; Statement-2 is will be: [2012]
not the correct explanation of Statement-1.
(c) Statement-1 is false, Statement-2 is true. E E
(d) Statement-1 is true, Statement-2 is false.
Q (a) (b)
34. Let P (r ) = r be the charge density distribution for a
pR 4 Rr Rr
solid sphere of radius R and total charge Q. For a point ‘p’
inside the sphere at distance r1 from the centre of the sphere,
the magnitude of electric field is : [2009] E E
Q Qr12
(a) (b) (c) (d)
4p Î0 r12 4p Î R 4 0
Rr Rr
Qr12 40. This questions has statement-1 and statement-2. Of the four
(c) (d) 0
3p Î0 R 4 choices given after the statements, choose the one that
35. A thin semi-circular ring of radius r has a positive charge q best describe the two statements. [2012]
ur
distributed uniformly over it. The net field E at the centre O An insulating solid sphere of radius R has a uniformly
is [2010] positive charge density r. As a result of this uniform charge
q ˆj j distribution there is a finite value of electric potential at the
(a)
4p 2 e 0 r 2 centre of the sphere, at the surface of the sphere and also at
q ˆ a point out side the sphere. The electric potential at infinite
(b) - 2 2 j is zero.
4p e 0 r
q i Statement -1 : When a charge q is take from the centre of
ˆ O
(c) - 2 2 j the surface of the sphere its potential energy changes by
2p e 0 r
q qr
ˆj
(d) 2
2p e 0 r 2 3e0 .
Electrostatics P-113
Statement -2 : The electric field at a distance r (r <R) from
(a) 6 ´ 10 -7 C m 2 (b) 3 ´ 10 -7 C m 2
rr
the centre of the sphere is .
3 e0 (c) 3 ´ 10 4 C m 2 (d) 6 ´ 10 4 C m 2
(a) Statement 1 is true, Statement 2 is true; Statement 2 is 46. In the given circuit, charge Q2 on the 2µF capacitor changes
not the correct explanation of statement 1. as C is varied from 1µF to 3µF. Q2 as a function of 'C' is given
(b) Statement 1 is true Statement 2 is false. properly by: (figures are drawn schematically and are not
(c) Statement 1 is false Statement 2 is true. to scale) [JEE Main 2015]

(d) Statement 1 is true, Statement 2 is true, Statement 2 is


the correct explanation of Statement 1 1µF
41. Two capacitors C1 and C2 are charged to 120 V and 200 V C
2µF
respectively. It is found that connecting them together the
potential on each one can be made zero. Then
[JEE Main 2013] E
(a) 5C1 = 3C2 (b) 3C1 = 5C2 Charge
(c) 3C1 + 5C2 = 0 (d) 9C1 = 4C2
Q2
42. Two charges, each equal to q, are kept at x = – a and x = a on
(a)
q
the x-axis. A particle of mass m and charge q0 = is placed C
2 1µF 3µF
at the origin. If charge q0 is given a small displacement Charge
(y <<a) along the y-axis, the net force acting on the particle
is proportional to [JEE Main 2013] Q2
(a) y (b) –y
(b)
1 1 C
(c) (d) – 1µF 3µF
y y
Charge
43. A charge Q is uniformly distributed over a long rod AB of
length L as shown in the figure. The electric potential at the
point O lying at distance L from the end A is Q2
[JEE Main 2013]
(c)
O A B
L L C
1µF 3µF
Q 3Q
(a) (b) 4pe L Charge
8pe 0 L 0

Q Q ln 2 Q2
(c) 4pe0 L ln 2 (d) 4pe L s
0
r (d)
44. Assume that an electric field E = 30x 2 ˆi exists in space. Then
C
the potential difference VA - VO , where VO is the potential 1µF 3µF
at the origin and VA the potential at x = 2 m is: 47. A uniformly charged solid sphere of radius R has potential
V0 (measured with respect to ¥) on its surface. For this sphere
[JEE Main 2014]
3V0 5V0 3V0
(a) 120 J/C (b) -120 J/C the equipotential surfaces with potentials , ,
2 4 4
(c) -80 J/C (d) 80 J/C V0
and have radius R1, R2, R3 and R4 respectively. Then
45. A parallel plate capacitor is made of two circular plates 4
separated by a distance 5 mm and with a dielectric of dielectric (a) R1 = 0 and R2 < (R4 – R3) [JEE Main 2015]
constant 2.2 between them. When the electric field in the
(b) 2R < R4
dielectric is 3 ´ 104 V m the charge density of the positive
(c) R1 = 0 and R2 > (R4 – R3)
plate will be close to: [JEE Main 2014]
(d) R1 ¹ 0 and (R2 – R1) > (R4 – R3)
EBD_7036
P-114 Topic-wise Solved Papers - PHYSICS
48. A long cylindrical shell carries positive surface charge s in (a) 420 N/C (b) 480 N/C
the upper half and negative surface charge - s in the lower (c) 240 N/C (d) 360 N/C
half. The electric field lines around the cylinder will look like 50. The region between two concentric spheres of radii 'a' and
figure given in : (figures are schematic and not drawn to 'b', respectively (see figure), have volume charge density
scale) [JEE Main 2015] A
r= , where A is a constant and r is the distance from the
r
(a) (b) centre. At the centre of the spheres is a point charge Q. The
value of A such that the electric field in the region between
the spheres will be constant, is : [JEE Main 2016]

(c) (d)
a
49. A combination of capacitors is set up as shown in the figure.
Q
The magnitude of the electric field, due to a point charge Q b
(having a charge equal to the sum of the charges on the 4
mF and 9 mF capacitors), at a point distance 30 m from it,
would equal : [JEE Main 2016] 2Q 2Q
(a) p a 2 - b 2 (b)
3mF ( ) pa 2
4mF
Q Q
9mF
(c) (d) 2p b2 - a 2
2pa 2 ( )
2mF
+ –

8V
CHAPTER

13 Current Electricity
Section-A JEE Advanced/ IIT-JEE
2. A constant voltage is applied between the two ends of a
A Fill in the Blanks uniform metallic wire. Some heat is developed in it. The heat
developed is doubled if (1980)
1. An electric bulb rated for 500 watts at 100 volts is used in a
(a) both the length and the radius of the wire are halved.
circuit having a 200 volts supply. The resistance R that
must be put in series with the bulb, so that the bulb delivers (b) both the length and the radius of the wire are doubled.
500 watt is ..........ohm. (1987 - 2 Marks) (c) the radius of the wire is doubled.
2. The equivalent resistance between points A and B of the (d) the length of the wire is doubled.
circuit given below is ......W . (1997 - 2 Marks) 3. The electrostatic field due to a point charge depends on the
1
2R 2R R distance r as 2 . Indicate which of the following quantities
A B r
shows same dependence on r. (1980)
(a) Intensity of light from a point source.
3. In the circuit shown below, each battery is 5V and has an (b) Electrostatic potential due to a point charge.
internal resistance of 0.2 ohm. (c) Electrostatic potential at a distance r from the centre of a
charged metallic sphere. Given r < radius of the sphere.
(d) None of these
V 4. In the circuit shown in fig the heat produced in the 5 ohm
resistor due to the current flowing through it is 10 calories
per second. (1981- 2 Marks)
The reading in the ideal voltmeter V is ..... V. (1997 - 2 Marks)
4W 6W
B True/False
1. In an electrolytic solution the electric current is mainly due 5W
to the movement of free electrons. (1980)
The heat generated in the 4 ohms resistor is
2. Electrons in a conductor have no motion in the absence of (a) 1 calorie / sec (b) 2 calories /sec
a potential difference across it. (1982 - 2 Marks) (c) 3 calories /sec (d) 4 calories /sec
3. The current –voltage graphs for a given metallic wire at 5. The current i in the circuit (see Fig) is (1983 - 1 Mark)
two different temperatures T1 and T2 are shown in the figure. i
(1985 - 3 Marks)
30 W 30 W
T1
2V
I T2
30 W
V 1 1
The temperature T2 is greater than T1. (a) A (b) A
45 15
1 1
C MCQs with One Correct Answer (c)
10
A (d)
5
A
6. A piece of copper and another of germanium are cooled
1. The temperature coefficient of resistance of a wire is 0.00125 from room temperature to 80° K. The resistance of
per °C. At 300 K, its resistance is 1 ohm. This resistance of (a) each of them increases (1988 - 1 Mark)
the wire will be 2 ohm at. (b) each of them decreases
(a) 1154 K (b) 1100 K (1980) (c) copper increases and germanium decreases
(c) 1400 K (d) 1127 K (d) copper decreases and germanium increases
EBD_7036
P-116 Topic-wise Solved Papers - PHYSICS
7. A battery of internal resistance 4W is connected to the 12. The effective resistance between points P and Q of the
network of resistances as shown. In order that the maximum electrical circuit shown in the figure is (2002S)
power can be delivered to the network, the value of R in W 2 Rr 2R 2R
should be (1995S) (a)
R+r
R 2R
R 8R( R + r )
(b) P Q
E R 6R R 3R + r r
2R
r
2R 2R
4W R 4R (c) 2r + 4R
5R
(d) + 2r
2
4 8 13. A 100 W bulb B1, and two 60 W bulb B2 and B 3, are
(a) (b) 2 (c) (d) 18
9 3 connected to a 250 V source, as shown in figure. Now W1,
W2 and W3 are the output powers of the bulbs B1, B2 and B3,
8. In the circuit P ¹ R , the
reading of the galvanometer is
P Q respectively. Then (2002S )
same with switch S open or B1 B2
S
closed. Then (1999 - 2 Marks)
(a) IR = IG R
G B3
(b) IP = IG
(c) IQ = IG
250V
(d) IQ = IR
V
9. In the given circuit, with steady current, the potential drop
(a) W1 > W2 = W3 (b) W1 > W2 > W3
across the capacitor must be (2001S)
(c) W1 < W2 = W3 (d) W1 < W2 < W3
(a) V (b) V/2 (c) V/3 (d) 2V/3
14. Express which of the following set ups can be used to verify
V R Ohm’s law? (2003S)

C
V \
(a)
2V 2R
10. A wire of length L and 3 identical cells of negligible internal
resistances are connected in series. Due to the current, the
temperature of the wire is raised by DT in a time t. A number
(b)
N of similar cells is now connected in series with a wire of
the same material and cross-section but of length 2L. The
temperature of the wire is raised by the same amount DT in
the same time t. the value of N is (2001S)
(a) 4 (b) 6
(c) 8 (d) 9
11. In the given circuit, it is observed that the current I is (c)
independent of the value of the resistance R6. Then the
resistance values must satisfy (2001S)

I
R5
R1 R6 R3

(d)
R2 R4

(a) R1R2R5 = R3R4R6


15. In the shown arrangement of the experiment of the meter
1 1 1 1
(b) + = + bridge if AC corresponding to null deflection of
R 5 R6 R1 + R2 R3 + R 4 galvanometer is x, what would be its value if the radius of
(c) R1R4 = R2R3 the wire AB is doubled? (2003S)
(d) R1R3 = R2R4 = R5R6
Current Electricity P-117

(a) P S
R1 R2 (b) Q R
1n I
(c) R
G Q
P
B (d) S
A x C t
20. Find out the value of current through 2W resistance for the
(a) x (b) x /4
given circuit. (2005S)
(c) 4 x (d) 2 x
16. The three resistance of equal value are arranged in the
different combinations shown below. Arrange them in
increasing order of power dissipation. (2003S) 5W 10 W
10 V 20 V
2W
i
i (a) zero (b) 2 A
(c) 5 A (d) 4 A
(I) (II) 21. A 4 mF capacitor, a resistance of 2.5 MW is in series with
12 V battery. Find the time after which the potential difference
across the capacitor is 3 times the potential difference across
the resistor. [Given ln(2) = 0.693] (2005S)
i i (a) 13.86s (b) 6.93s
(c) 7s (d) 14s
(III) (IV) 22. A moving coil galvanometer of resistance 100 W is used as
(a) III < II < IV < I (b) II < III < IV < I an ammeter using a resistance 0.1 W. The maximum
(c) I < IV < III < II (d) I < III < II < IV deflection current in the galvanometer is 100 mA. Find the
17. Shown in figure is a Post Office box. In order to calculate minimum current in the circuit so that the ammeter shows
the value of external resistance, it should be connected maximum deflection (2005S)
between (2004S) (a) 100.1 mA (b) 1000.1 mA
C B A (c) 10.01 mA (d) 1.01 mA
23. An ideal gas is filled in a closed rigid and thermally insulated
container. A coil of 100 W resistor carrying current 1 A for 5
D
minutes supplies heat to the gas. The change in internal
energy of the gas is (2005S)
B'
(a) 10 kJ (b) 30 kJ
C'
(c) 20 kJ (d) 0 kJ
(a) B' and C' (b) A and D 24. If a steady current I is flowing through a cylindrical element
(c) C and D (d) B and D ABC. Choose the correct relationship
18. Six identical resistors are connected as shown in the figure.
The equivalent resistance will be (2004S) A 2r B
r C
R I
P Q

l/2
R R l/2
R
R R (a) V AB = 2VBC
(b) Power across BC is 4 times the power across AB
(c) Current densities in AB and BC are equal
R
(d) Electric field due to current inside AB and BC are equal
(a) Maximum between P and R 25. A resistance of 2W is connected across one gap of a metre-
(b) Maximum between Q and R bridge (the length of the wire is 100 cm) and an unknown
(c) Maximum between P and Q
resistance, greater than 2W, is connected across the other
(d) All are equal
19. A capacitor is charged using an external battery with a gap. When these resistances are interchanged, the balance
resistance x in series. The dashed line shows the variation point shifts by 20 cm. Neglecting any corrections, the
of ln I with respect to time. If the resistance is changed to unknown resistance is
2x, the new graph will be (2004S) (a) 3W (b) 4W (2007)
(c) 5W (d) 6W
EBD_7036
P-118 Topic-wise Solved Papers - PHYSICS
26. A circuit is connected as shown in the figure with the switch 29. To verify Ohm’s law, a student is provided with a test resistor
S open. When the switch is closed, the total amount of RT, a high resistance R1, a small resistance R2, two identical
charge that flows from Y to X is (2007) galvanometers G1 and G2, and a variable voltage source V.
3mF 6m F The correct circuit to carry out the experiment is (2010)
X
G1

R2
S

3W (a) RT R1
6W
Y
V
9V
(a) 0 (b) 54 mC
(c) 27mC (d) 81 mC
27. Figure shows three resistor configurations R1, R2 and R3 (b)
connected to 3V battery. If the power dissipated by the
configuration R1, R2 and R3 is P1, P2 and P3, respectively,
then –

(c)

3V 3V

(d)

R1
R2
30. Consider a thin square sheet of side L and thickness t, made
of a material of resistivity r. The resistance between two
(a) P1 > P2 > P3 opposite faces, shown by the shaded areas in the figure is
(2010)
3V
(b) P1 > P3 > P2 (a) directly proportional to L
(c) P2 > P1 > P3 (b) directly proportional to t
(d) P3 > P2 > P1 (c) independent of L t
L
(2008) (d) independent of t
31. A meter bridge is set up as shown, to determine an unknown
R3 resistance ‘X’ using a standard 10 ohm resistor. The
28. Incandescent bulbs are designed by keeping in mind that galvanometer shows null point when tapping-key is at 52
the resistance of their filament increases with the increase cm mark. The end-corrections are 1 cm and 2 cm respectively
for the ends A and B. The determined value of ‘X’ is
in temperature. If at room temperature, 100 W, 60 W and
(2011)
40 W bulbs have filament resistances R100, R60 and R40,
respectively, the relation between these resistances is

1 1 1
(a) R = + (b) R100 = R40 + R60 (2010)
100 R40 R60

1 1 1 (a) 10.2 ohm (b) 10.6 ohm


(c) R100 > R60 > R40 (d) > > (c) 10.8 ohm (d) 11.1 ohm
R100 R60 R40
Current Electricity P-119

32. During an experiment with a metre bridge, the galvanometer Z: In a conducing solid, the rate of collisions between
shows a null point when the jockey is pressed at 40.0 cm free electrons and ions increases with increase of
using a standard resistance of 90 W, as shown in the figure. temperature
The least count of the scale used in the metre bridge is 1mm. Select the correct statement(s) from the following;
The unknown resistance is (JEE Adv. 2014) (a) Y is true but Z is false (b)Y is false but Z is true
(c) Both Y and Z are true (d) Y is true and Z is the
correct reason for Y
3. In the circuit shown in Figure the current through
R 90 W (1998S - 2 Marks)

3W 2W 2W

40.0 cm
(a) 60 ± 0.15 W (b) 135 ± 0.56 W 9V 8W 8W 4W

(c) 60 ± 0.25 W (d) 135 ± 0.23 W


33. An infinite line charge of uniform electric charge density l
lies along the axis of an electrically conducting infinite
2W 2W 2W
cylindrical shell of radius R. At time t = 0, the space inside
the cylinder is filled with a material of permittivity e and (a) the 3 W resistor is 0.50 A.
electrical conductivity s. The electrical conduction in the (b) the 3 W resistor is 0.25 A.
material follows Ohm's law. Which one of the following (c) the 4 W resistor is 0.50 A
graphs best describes the subsequent variation of the (d) the 4 W resistor is 0.25 A.
magnitude of current density j(t) at any point in the material? 4. When a potential difference is applied across, the current
(JEE Adv. 2016) passing through (1999S - 3 Marks)
(a) an insulator at 0 K is zero
j(t) j(t)
(b) a semiconductor at 0 K is zero
(c) a metal at 0 K is finite
(d) a p-n diode at 300K is finite, if it is reverse biased
(a) (b) 5. For the circuit shown in the figure (2009)
t t
(0, 0) (0, 0)
2kW R1
j(t) j(t)
I

(c) (d) 24 V 6kW R2 RL 1.5kW

t t
(0, 0) (0, 0)
(a) the current I through the battery is 7.5 mA
(b) the potential difference across RL is 18 V
D MCQs with One or More than One Correct (c) ratio of powers dissipated in R1 and R2 is 3
(d) if R1 and R2 are interchanged, magnitude of the power
1. Capacitor C1 of capacitance 1 micro-farad and capacitor C2 dissipated in RL will decrease by a factor of 9
of capacitance 2 microfarad are separately charged fully by 6. For the resistance network shown in the figure, choose the
a common battery. The two capacitors are then separately
correct option(s) (2012- I)
allowed to discharge through equal resistors at time t = 0.
(1989 - 2 Marks)
(a) The current in each of the two discharging circuits is
zero at t = 0.
(b) The currents in the two discharging circuits at t = 0 are
equal but not zero.
(c) The currents in the two discharging circuits at t = 0 are
unequal.
(d) Capacitor C1, losses 50% of its initial charge sooner
than C2 loses 50% of its initial charge.
2. Read the following statements carefully: (1993-2 Marks) (a) The current through PQ is zero.
Y : The resistivity of a semiconductor decreases with (b) I1 = 3A
increase of temperature. (c) The potential at S is less than that at Q.
(d) I2 = 2A
EBD_7036
P-120 Topic-wise Solved Papers - PHYSICS
7. Heater of an electric kettle is made of a wire of length L and (b)
The resistance over small sections of the filament
diameter d. It takes 4 minutes to raise the temperature of decreases with time
0.5 kg water by 40 K. This heater is replaced by a new heater (c) The filament emits more light at higher band of
having two wires of the same material, each of length L and frequencies before it breaks up
diameter 2d. The way these wires are connected is given in (d) The filament consumes less electrical power towards
the options. How much time in minutes will it take to raise the the end of the life of the bulb
temperature of the same amount of water by 40 K?
11. In the circuit shown below, the key is pressed at time t = 0.
(JEE Adv. 2014)
Which of the following statement(s) is(are) true?
(a) 4 if wires are in parallel (b) 2 if wires are in series
(c) 1 if wires are in series (d) 0.5 if wires are in parallel (JEE Adv. 2016)
8. Two ideal batteries of emf V1 and V2 and three resistances
R1, R2 and R3 are connected as shown in the figure. The 40 mF
current in resistance R2 would be zero if (JEE Adv. 2014) 25 kW


v
+
V1 R1
R2
A 50 W 20 mF

V2 –
+
R3
Key
5V
(a) V1 = V2 and R1 = R2 = R3
(a) The voltmeter displays –5V as soon as the key is
(b) V1 = V2 and R1 = 2R2 = R3
pressed, and displays +5V after a long time
(c) V1 = 2V2 and 2R1 = 2R2 = R3
(b) The voltmeter will display 0V at time t = ln 2 seconds
(d) 2V1 = V2 and 2R1 = R2 = R3
(c) The current in the ammeter becomes 1/e of the initial
9. In an aluminium (Al) bar of square cross section, a square value after 1 second
hole is drilled and is filled with iron (Fe) as shown in the (d) The current in the ammeter becomes zero after a long
figure. The electrical resistivities of Al and Fe are 2.7 × 10–8 time.
W m and 1.0 × 10–7 W m, respectively. The electrical resistance
between the two faces P and Q of the composite bar is E Subjective Problems
(JEE Adv. 2015) 1. A heater is designed to operate with a power of 1000 watts
in a 100 volt line. It is connected in a combinations with a
2475 resistance of 10 ohms and a resistance R to a 100 volts
(a) µW
64 mains as shown in the figure. What should be the value of
Q R so that the heater operates with a power of 62.5 watts.
1875 (1978)
(b) µW Heater
64 10W R
1875 Al 50mm
(c) µW
49 Fe 100V
2mm P 2. If a copper wire is stretched to make it 0.1% longer what is
2475 the percentage change in its resistance? (1978)
(d) µW 7mm
132 3. All resistances in the diagram below are in ohms. Find the
10. An incandescent bulb has a thin filament of tungsten that is effective resistance between the points A and B. (1979)
heated to high temperature by passing an electric current. 3
The hot filament emits black–body radiation. The filament
3 3
is observed to break up at random locations after a
sufficiently long time of operation due to non–uniform 6
6
evaporation of tungsten from the filament. If the bulb is 6
3 3
powered at constant voltage, which of the following
statement(s) is(are) true? (JEE Adv. 2016) A 3 B
(a) The temperature distribution over the filament is 4. In the diagram shown find the potential difference between
uniform the points A and B and between the points B and C in the
steady state. (1979)
Current Electricity P-121
B (ii) the potential difference across the terminals of each
3µf 1µf cells G and H E
3µf 1µf A B

1µf
F 2W H
10 W
100V
20 W
A C C
D
5. A battery of emf 2 volts and internal resistance 0.1 ohm is G
being charged with a current of 5 amps. (1980) 10. A part of ciucuit in a steady state along with the currents
In what direction will the current flow inside the battery? flowing in the branches, the values of resistances etc., is
What is the potential difference between the two terminal of shown in the figure. Calculate the energy stored in the
the battery? capacitor C (4µF) (1986 - 4 Marks)
6. State ohm’s law.
In the circuit shown in figure, a voltmeter reads 30 volts 1A
when it is connected across 400 ohm resistance. Calculate
what the same voltmeter will read when it is connected across 3W
the 300 ohm resistance. ( 1980) 4V 3W 5W

2A
300W 400W C 1W
4m F
3V 1W
2A 2W 4W
60V
3W
7. In the circuit shown in fig E1 =3 volts, E2 = 2 volts, E3 = 1
volt and R = r1 = r2 = r3 = 1 ohm. (1981 - 6 Marks)
1A
r1 + –
i1 E1 11. An infinite ladder network of resistances is constructed with
A R i2 r2 + – B a1 ohm and 2 ohm resistances, as shown in fig.
C D (1987 - 7 Marks)
E2 1W 1W 1W 1W
i3 r3 + – A
E3
6V
(i) Find the potential difference between the points A and 2W 2W 2W 2W
B and the currents through each branch.
(ii) If r2 is short circuited and the point A is connected to
point B, find the currents through E1, E2 E3 and the B
resistor R. Fig. 8. A and B has negligible internal
The 6 volt battery between
8. Calculate the steady state current in the 2-ohm resistor resistance :
shown in the circuit in the figure. The internal resistance of (i) Show that the effective resistance between A and B is
the battery is negligible and the capacitance of the 2 ohms.
condenser C is 0.2 microfarad. (1982 - 5 Marks) (ii) What is the current that passes through the 2 ohm
2W resistance nearest to the battery ?
12. In the given circuit (1988 - 5 Marks)
3W
E1 = 3E2 = 2 E3 = 6 volts R1 = 2 R4 = 6 ohms
R3 = 2 R2 = 4 ohms C = 5m f .
C 4W
Find the current in R3 and the energy stored in the capacitor.
R1 E1

C
2.8 W

V= 6 VOLTS
9. In the circuit shown in figure E, F, G, H are cells of emf 2, 1, 3
E2 R2 R3
and 1 volt respectively, and their internal resistances are 2,
1, 3 and 1 ohm respectively. (1984 - 6 Marks)
Calculate :
(i) the potential difference between B and D and E3 R4
EBD_7036
P-122 Topic-wise Solved Papers - PHYSICS
13. An electrical circuit is shown in Fig. Calculate the potential (a) Are there positive and negative terminals on the
difference across the resistor of 400 ohm, as will be measured galvanometer?
by the voltmeter V of resistance 400 ohm, either by applying (b) Copy the figure in your answer book and show the
Kirchhoff’s rules or otherwise. (1996 - 5 Marks) battery and the galvanometer (with jockey) connected
V at appropriate points.
16. How a battery is to be connected so that the shown rheostat
will behave like a potential divider? Also indicate the points
400W
about which output can be taken. (2003 - 2 Marks)
100W 100W C
200W
l2
100W
l1
A R B
l 10V 17. Draw the circuit diagram to verify Ohm’s Law with the help
14. In the circuit shown in Figure, the battery is an ideal one, of a main resistance of 100 W and two galvanometers of
with emf V. The capacitor is initially uncharged. The switch resistances 106 W and 10–3 W and a source of varying emf.
S is closed at time t = 0. (1998 - 8 Marks) Show the correct positions of voltmeter and ammeter.
(2004 - 4 Marks)
(a) Find the charge Q on the capacitor at time t.
18. An unknown resistance X is to be determined using
(b) Find the current in AB at time t. What is its limiting resistances R1, R2 or R3. Their corresponding null points
value as t ® ¥ : are A, B and C. Find which of the above will give the most
S A R accurate reading and why? (2005 - 2 Marks)

V R C

X R R = R1 or R2 or R3
R B
G
15. A thin uniform wire AB of length 1m, an unknown resistance
X and a resistance of 12 W are connected by thick A B C
conducting strips, as shown in the figure. A battery and a
galvanometer (with a sliding jockey connected to it) are 19. In the given circuit, the switch S is closed at time t = 0. The
also available. Connections are to be made to measure the charge Q on the capacitor at any instant t is given by
Q(t) = Q(1 – e–at). Find the value of Q0 and a in terms of
unknown resistance X using the principle of Wheatstone
given parameters as shown in the circuit.
bridge. Answer the following questions. (2002 - 5 Marks )
R1
(2005 - 4 Marks)
S
C R2
X 12W +
V

A B C D
F Match the Following

DIRECTIONS (Q. No. 1) : Each question contains statements given in two columns, which have to be p q r s t
matched. The statements in Column-I are labelled A, B, C and D, while the statements in Column-II are
A p q r s t
labelled p, q, r and s. Any given statement in Column-I can have correct matching with ONE OR MORE
B p q r s t
statement(s) in Column-II. The appropriate bubbles corresponding to the answers to these questions
have to be darkened as illustrated in the following example : C p q r s t
If the correct matches are A-p, s and t; B-q and r; C-p and q; and D-s then the correct darkening of D p q r s t
bubbles will look like the given.
1. Column I gives some devices and Column II gives some processes on which the functioning of these devices depend. Match
the devices in Column I with the processes in Column II and indicate your answer by darkening appropriate bubbles in the 4 ×
4 matrix given in the ORS. (2007)
Column I Column II
(A) Bimetallic strip (p) Radiation from a hot body
(B) Steam engine (q) Energy conversion
(C) Incandescent lamp (r) Melting
(D) Electric fuse (s) Thermal expansion of solids
Current Electricity P-123

(a) Statement-1 is True, Statement-2 is True; Statement-2


G Comprehension Based Questions is a correct explanation for Statement-1
PASSAGE (b) Statement-1 is True, Statement-2 is True; Statement-2
Electrical resistance of certain materials, known as is NOT a correct explanation for Statement-1
superconductors, changes abruptly from a nonzero value (c) Statement -1 is True, Statement-2 is False
to zero as their temperature is lowered below a critical (d) Statement-1 is False, Statement-2 is True
temperature T C (0). An in teresting pr operty of
superconductors is that their critical temperature becomes I Integer Value Correct Type
smaller than TC (0) if they are placed in a magnetic field, i.e., 1. When two identical batteries of internal resistance 1W each
the critical temperature TC (B) is a function of the magnetic are connected in series across a resistor R, the rate of heat
field strength B. The dependence of TC (B) on B is shown in produced in R is J1. When the same batteries are connected
the figure. (2010) in parallel across R, the rate is J2. If J1 = 2.25 J2 then the
TC (B) value of R in W is (2010)
2. At time t = 0, a battery of 10 V is connected across points
TC (0) A and B in the given circuit. If the capacitors have no
charge initially, at what time (in sceonds) does the voltage
across them become 4 V? [Take : ln5 =1.6, ln3 = 1.1] (2010)

O 2MW 2m F
B
1. In the graphs below, the resistance R of a superconductor is
shown as a function of its temperature T for two different
magnetic fields B1 (solid line) and B2 (dashed line). If B2 is A B
larger than B1 which of the following graphs shows the 2MW
2m F
correct variation of R with T in these fields?
3. Two batteries of different emfs and different internal
resistances are connected as shown. The voltage across
AB in volts is (2011)
(a) (b)

(c) (d)

2. A superconductor has TC (0) = 100 K. When a magnetic 4. A galvanometer gives full scale deflection with 0.006 A
field of 7.5 Tesla is applied, its TC decreases to 75 K. For this current. By connecting it to a 4990 W resistance, it can be
material one can definitely say that when converted into a voltmeter of range 0 – 30 V. If connected to
(a) B = 5 Tesla, TC (B) = 80 K 2n
(b) B = 5 Tesla, 75 K < TC (B) < 100 K a W resistance, it becomes an ammeter of range 0 – 1.5A.
249
(c) B = 10 Tesla, 75K < TC (B) < 100 K The value of n is (JEE Adv. 2014)
(d) B = 10 Tesla, TC (B) = 70K 5. In the following circuit, the current through the resistor
H Assertion & Reason Type Questions R (= 2 W) is I amperes. The value of I is (JEE Adv. 2015)

1. STATEMENT-1 : In a Meter Bridge experiment, null point (=2W)


for an unknown resistance is measured. Now, the unknown
resistance is put inside an enclosure maintained at a higher
temperature. The null point can be obtained at the same
point as before by decreasing the value of the standard
resistance.
STATEMENT-2 : Resistance of a metal increases with
increase in temperature. (2008)
EBD_7036
P-124 Topic-wise Solved Papers - PHYSICS

Section-B JEE Main / AIEEE


1. If an ammeter is to be used in place of a voltmeter, then we 8. The thermo e.m.f. of a thermo -couple is 25 mV / o C at room
must connect with the ammeter a [2002]
temperature. A galvanometer of 40 ohm resistance, capable
(a) low resistance in parallel
(b) high resistance in parallel of detecting current as low as 10 -5 A , is connected with
(c) high resistance in series the thermo couple. The smallest temperature difference that
(d) low resistance in series. can be detected by this system is [2003]
2. A wire when connected to 220 V mains supply has power (a) 16 o C (b) 12 o C
dissipation P1. Now the wire is cut into two equal pieces (c) 8 o C (d) 20 o C
which are connected in parallel to the same supply. Power
9. The negative Zn pole of a Daniell cell, sending a constant
dissipation in this case is P2. Then P2 : P1 is [2002]
current through a circuit, decreases in mass by 0.13g in 30
(a) 1 (b) 4
minutes. If the electeochemical equivalent of Zn and Cu are
(c) 2 (d) 3 32.5 and 31.5 respectively, the increase in the mass of the
3. If a current is passed through a spring then the spring will positive Cu pole in this time is [2003]
(a) expand (b) compress [2002] (a) 0.180 g (b) 0.141g
(c) remains same (d) none of these. (c) 0.126 g (d) 0.242 g
4. If in the circuit, power dissipation is 150 W, then R is 10. An ammeter reads upto 1 ampere. Its internal resistance is
R 0.81ohm. To increase the range to 10 A the value of the
required shunt is [2003]
2W (a) 0.03 W (b) 0.3 W
15 V (c) 0.9 W (d) 0.09 W
11. A 3 volt battery with negligible internal resistance is
(a) 2 W (b) 6 W [2002] connected in a circuit as shown in the figure. The current I,
(c) 5 W (d) 4 W in the circuit will be [2003]
5. The mass of product liberated on anode in an electrochemical
cell depends on [2002]
(a) (It)1/2 (b) It
(c) I/t (d) I2t 3W
(where t is the time period for which the current is passed). 3W
3V
6. If qi , is the inversion temperature, qn is the neutral
temperature, q c is the temperature of the cold junction,
then [2002] 3W

(a) qi + qc = qn (b) qi - qc = 2q n (a) 1 A (b) 1.5 A


qi + qC (c) 2 A (d) 1/3 A
(c) = qn (d) qc - qi = 2qn 12. A 220 volt, 1000 watt bulb is connected across a 110 volt
2
mains supply . The power consumed will be [2003]
7. The length of a wire of a potentiometer is 100 cm, and the e.
(a) 750 watt (b) 500 watt
m.f. of its standard cell is E volt. It is employed to measure (c) 250 watt (d) 1000 watt
the e.m.f. of a battery whose internal resistance is 0.5W. If 13. The total current supplied to the circuit by the battery is
the balance point is obtained at l = 30 cm from the positive [2004]
end, the e.m.f. of the battery is [2003]
2W
6V
30 E 6W
30 E 3W
(a) (b)
100.5 (100 - 0.5) 1.5W

30 ( E - 0.5i ) 30 E
(c) (d) (a) 4 A (b) 2 A
100 100
(c) 1 A (d) 6 A
where i is the current in the potentiometer wire.
Current Electricity P-125

14. The resistance of the series combination of two resistances


(c) repel each other with a force of m 0 i 2 /(2pd 2 )
is S. when they are joined in parallel the total resistance is P.
If S = nP then the Minimum possible value of n is (d) attract each other with a force of m 0 i 2 /(2pd 2 )
(a) 2 (b) 3 [2004]
22. A heater coil is cut into two equal parts and only one part is
(c) 4 (d) 1
now used in the heater. The heat generated will now be
15. An electric current is passed through a circuit containing
(a) four times (b) doubled [2005]
two wires of the same material, connected in parallel. If the
(c) halved (d) one fourth
4 2 23. In the circuit , the galvanometer G shows zero deflection. If
lengths and radii arein the ratio of and , then the ratio
3 3 the batteries A and B have negligible internal resistance, the
of the current passing through the wires will be value of the resistor R will be - [2005]
[2004]
(a) 8/9 (b) 1/3 500 W
G
(c) 3 (d) 2
16. In a meter bridge experiment null point is obtained at 20 cm. 2V
from one end of the wire when resistance X is balanced 12V R
B A
against another resistance Y. If X < Y, then where will be the
new position of the null point from the same end, if one
decides to balance a resistance of 4 X against Y
(a) 40 cm (b) 80 cm [2004] (a) 100 W (b) 200W
(c) 50 cm (d) 70 cm (c) 1000 W (d) 500 W
17. The termistors are usually made of [2004]
24. A moving coil galvanometer has 150 equal divisions. Its
(a) metal oxides with high temperature coefficient of
current sensitivity is 10-divisions per milliampere and voltage
resistivity
sensitivity is 2 divisions per millivolt. In order that each
(b) metals with high temperature coefficient of resistivity
division reads 1 volt, the resistance in ohms needed to be
(c) metals with low temperature coefficient of resistivity
connected in series with the coil will be - [2005]
(d) semiconducting materials having low temperature 5 3
(a) 10 (b) 10
coefficient of resistivity
(c) 9995 (d) 99995
18. Time taken by a 836 W heater to heat one litre of water from
25. Two sources of equal emf are connected to an external
10°C to 40°C is [2004]
resistance R. The internal resistance of the two sources are
(a) 150 s (b) 100 s
R1and R2 (R1 > R1). If the potential difference across the
(c) 50 s (d) 200 s
19. The thermo emf of a thermocouple varies with the temperature source having internal resistance R 2 is zero, then

q of the hot junction as E = aq + bq 2 in volts where the (a) R = R2 - R1 [2005]


ratio a/b is 700°C. If the cold junction is kept at 0°C, then the (b) R = R2 ´ ( R1 + R2 ) /( R2 - R1 )
neutral temperature is [2004]
(a) 1400°C (c) R = R1R2 /( R2 - R1 )
(b) 350°C (d) R = R1R2 /( R1 - R2 )
(c) 700°C
26. Two voltameters, one of copper and another of silver, are
(d) No neutral temperature is possible for this termocouple.
joined in parallel. When a total charge q flows through the
20. The electrochemical equivalent of a metal is 3.35109-7 kg voltameters, equal amount of metals are deposited. If the
per Coulomb. The mass of the metal liberated at the cathode electrochemical equivalents of copper and silver are Z1 and
when a 3A current is passed for 2 seconds will be Z 2 respectively the charge which flows through the silver
(a) 6.6×1057kg (b) 9.9×10–7 kg [2004]
voltameter is [2005]
(c) 19.8×10–7 kg (d) 1.1×10–7 kg
21. Two thin, long, parallel wires, separated by a distance ‘d’ q q
carry a current of ‘i’ A in the same direction. They will (a) (b)
Z Z
[2005] 1+ 2 1+ 1
Z1 Z2
(a) repel each other with a force of m 0 i 2 /(2pd )
Z Z
(b) attract each other with a force of m 0 i 2 /(2pd ) (c) q 2 (d) q 1
Z1 Z2
EBD_7036
P-126 Topic-wise Solved Papers - PHYSICS
27. In a potentiometer experiment the balancing with a cell is at (a) 0.33 A (b) 0.5 A
length 240 cm. On shunting the cell with a resistance of 2W, (c) 0.67 A (d) 0.17 A
the balancing length becomes 120 cm. The internal 34. The resistance of a bulb filmanet is 100W at a temperature of
resistance of the cell is [2005] 100°C. If its temperature coefficient of resistance be 0.005
(a) 0.5W (b) 1W per °C, its resistance will become 200 W at a temperature of
(c) 2W (d) 4W (a) 300°C (b) 400°C [2006]
28. The resistance of hot tungsten filament is about 10 times (c) 500°C (d) 200°C
the cold resistance. What will be the resistance of 100 W
35. In a Wheatstone's bridge, three resistances P, Q and R
and 200 V lamp when not in use ? [2005]
connected in the three arms and the fourth arm is formed by
(a) 20 W (b) 40 W two resistances S1 and S2 connected in parallel. The
(c) 200 W (d) 400 W condition for the bridge to be balanced will be
29. An energy source will supply a constant current into the [2006]
load if its internal resistance is [2005]
P 2R P R ( S1 + S2 )
(a) very large as compared to the load resistance (a) = (b) =
(b) equal to the resistance of the load Q S1 + S2 Q S1 S2
(c) non-zero but less than the resistance of the load
(d) zero P R ( S1 + S2 ) P R
(c) = (d) =
30. The Kirchhoff's first law (Si = 0) and second law (SiR = SE), Q 2S1S2 Q S1 + S2
where the symbols have their usual meanings, are
36. An electric bulb is rated 220 volt - 100 watt. The power
respectively based on [2006]
(a) conservation of charge, conservation of momentum consumed by it when operated on 110 volt will be
(b) conservation of energy, conservation of charge (a) 75 watt (b) 40 watt [2006]
(c) conservation of momentum, conservation of charge (c) 25 watt (d) 50 watt
(d) conservation of charge, conservatrion of energy 37. A battery is used to charge a parallel plate capacitor till the
31. A material 'B' has twice the specific resistance of 'A'. A circular potential difference between the plates becomes equal to
wire made of 'B' has twice the diameter of a wire made of 'A'. the electromotive force of the battery. The ratio of the energy
then for the two wires to have the same resistance, the ratio stored in the capacitor and the work done by the battery will
lB/lA of their respective lengths must be [2006] be [2007]
1 (a) 1/2 (b) 1
(a) 1 (b)
2 (c) 2 (d) 1/4
38. The resistance of a wire is 5 ohm at 50°C and 6 ohm at 100°C.
1
(c) (d) 2 The resistance of the wire at 0°C will be [2007]
4
(a) 3 ohm (b) 2 ohm
32. A thermocouple is made from two metals, Antimony and
(c) 1 ohm (d) 4 ohm
Bismuth. If one junction of the couple is kept hot and the
39. Shown in the figure below is a meter-bridge set up with null
other is kept cold, then, an electric current will
deflection in the galvanometer.
[2006]
(a) flow from Antimony to Bismuth at the hot junction 55W R
(b) flow from Bismuth to Antimony at the cold junction
(c) now flow through the thermocouple
(d) flow from Antimony to Bismuth at the cold junction G
33. The current I drawn from the 5 volt source will be
[2006] 20 cm
10W

5W 10W 20W

The value of the unknown resistor R is [2008]


I 10W (a) 13.75 W (b) 220 W
+– (c) 110 W (d) 55 W
5 volt
Current Electricity P-127

DIRECTIONS : Question No. 40 and 41 are based on the following The current in the 10W resistor is
paragraph. (a) 0.27 A P2 to P1 (b) 0.03 A P1 to P2
Consider a block of conducting material of resistivity ‘r’ shown (c) 0.03 A P2 to P1 (d) 0.27 A P1 to P2
in the figure. Current ‘I’ enters at ‘A’ and leaves from ‘D’. We 43. Let C be the capacitance of a capacitor discharging through
apply superposition principle to find voltage ‘DV’ developed a resistor R. Suppose t1 is the time taken for the energy
between ‘B’ and ‘C’. The calculation is done in the following stored in the capacitor to reduce to half its initial value and
steps: t2 is the time taken for the charge to reduce to one-fourth its
(i) Take current ‘I’ entering from ‘A’ and assume it to spread initial value. Then the ratio t1/ t2 will be [2010]
over a hemispherical surface in the block. 1
(ii) Calculate field E(r) at distance ‘r’ from A by using Ohm’s law (a) 1 (b)
2
E = r j, where j is the current per unit area at ‘r’.
(iii) From the ‘r’ dependence of E(r), obtain the potential V(r) at r. 1
(c) (d) 2
(iv) Repeat (i), (ii) and (iii) for current ‘I’ leaving ‘D’ and superpose 4
results for ‘A’ and ‘D’. 44. Two conductors have the same resistance at 0°C but their
temperature coefficients of resistance are a1 and a 2 . The
I DV I respective temperature coefficients of their series and parallel
combinations are nearly [2010]
a1 + a 2 a1 + a 2
a b a (a) , a1 + a 2 (b) a1 + a 2 ,
A B C D 2 2

a1a 2 a1 + a 2 a1 + a 2
(c) a1 + a 2 , (d) ,
a1 + a 2 2 2
45. If a wire is stretched to make it 0.1% longer, its resistance
will : [2011]
40. DV measured between B and C is [2008] (a) increase by 0.2% (b) decrease by 0.2%
rI rI rI rI (c) decrease by 0.05% (d) increase by 0.05%
(a) – (b) – 46. Two electric bulbs marked 25W – 220 V an d
pa p(a + b) a (a + b)
100W – 220V are connected in series to a 440 V supply.
rI rI rI Which of the bulbs will fuse? [2012]
(c) – (d) (a) Both (b) 100 W
2pa 2p(a + b) 2p(a - b)
(c) 25 W (d) Neither
41. For current entering at A, the electric field at a distance ‘r’ 47. The supply voltage to room is 120V. The resistance of the
from A is [2008] lead wires is 6W. A 60 W bulb is already switched on. What
rI rI is the decrease of voltage across the bulb, when a 240 W
(a) 2 (b) heater is switched on in parallel to the bulb?
8pr r2
(a) zero (b) 2.9 Volt [JEE Main 2013]
rI rI (c) 13.3 Volt (d) 10.04 Volt
(c) 2 (d) 48. This questions has Statement I and Statement II. Of the four
2pr 4pr 2
choices given after the Statements, choose the one that
42. A 5V battery with internal resistance 2W and a
best describes into two Statements. [JEE Main 2013]
2V battery with internal resistance 1W are connected to a
Statement-I : Higher the range, greater is the resistance of
10W resistor as shown in the figure. [2008]
ammeter.
Statement-II : To increase the range of ammeter, additional
P2
shunt needs to be used across it.
(a) Statement-I is true, Statement-II is true, Statement-II is
the correct explanation of Statement-I.
5V 2V
(b) Statement-I is true, Statement-II is true, Statement-II is
2W 10W 1W not the correct explanation of Statement-I.
(c) Statement-I is true, Statement-II is false.
(d) Statement-I is false, Statement-II is true.
EBD_7036
P-128 Topic-wise Solved Papers - PHYSICS
49. In a large building, there are 15 bulbs of 40 W, 5 bulbs of (a) 0.13 A, from Q to P (b) 0.13 A, from P to Q
100 W, 5 fans of 80 W and 1 heater of 1 kW. The voltage of (c) 1.3A from P to Q (d) 0A
electric mains is 220 V. The minimum capacity of the main fuse
52. The temperature dependence of resistances of Cu and
of the building will be: [JEE Main 2014]
undoped Si in the temperature range 300-400 K, is best
(a) 8 A (b) 10 A
(c) 12 A (d) 14 A described by : [JEE Main 2016]
50. When 5V potential difference is applied across a wire of length (a) Linear increase for Cu, exponential decrease of Si.
0.1 m, the drift speed of electrons is 2.5 × 10–4 ms–1. If the (b) Linear decrease for Cu, linear decrease for Si.
electron density in the wire is 8 × 1028 m–3, the resistivity of (c) Linear increase for Cu, linear increase for Si.
the material is close to : [JEE Main 2015] (d) Linear increase for Cu, exponential increase for Si.
–6
(a) 1.6 × 10 Wm –5
(b) 1.6 × 10 Wm
–8
(c) 1.6 × 10 Wm (d) 1.6 × 10–7 Wm
51. In the circuit shown, the current in the 1W resistor is :

6V P 2W [JEE Main 2015]

1W 9V

3W W 3W
CHAPTER
Moving Charges and
14 Magnetism
Section-A JEE Advanced/ IIT-JEE
6. A metallic block carrying
ur current I is subjected to a uniform
A Fill in the Blanks magnetic induction B as shown in Figure .
1. A neutron, a proton, and an electron and an alpha particle B
Y
enter a region of constant magnetic field with equal velocities.
The magnetic field is along the inward normal to the plane E
G
of the paper. The tracks of the particles are labelled in fig. F
H
X
The electron follows track ...... and the alpha particle follows
B I
track ..... (1984- 2 Marks) A

B C C
D
Z
A ur
The moving charges experience a force F given by .... which
D results in the lowering of the potential of the face .... Assume
the speed of the carriers to be v. (1996 - 2 Marks)
2. A wire of length L metre, carrying a current i ampere is bent
in the form of a circle. The magnitude of its magnetic moment B True/False
is ........in MKS units. (1987 - 2 Marks)
3. In a hydrogen atom, the electron moves in an orbit of radius 1. No net force acts on a rectangular coil carrying a steady
current when suspended freely in a uniform magnetic field.
0.5 Å making 1016 revolutions per second. The magnetic
(1981- 2 Marks)
moment associated with the orbital motion of the electron is
2. There is no change in the energy of a charged particle
............ (1988 - 2 Marks)
moving in a magnetic field although a magnetic force is
4. The wire loop PQRSP formed by joining two semicircular acting on it. (1983 - 2 Marks)
wires of radii R1 and R2 carries a current I as shown. The 3. A charged particle enters a region of uniform magnetic field
magnitude of the magnetic induction at the centre C is at an angle of 85° to the magnetic line of force . The path of
.................... (1988 - 2 Marks) the particle is a circle. (1983 - 2 Marks)
I 4. An electron and a proton are moving with the same kinetic
I energy along the same direction. When they pass through
a uniform magnetic field perpendicular to the direction of

S R
R2
.
C
R1
Q P
their motion, they describe circular paths of the same radius.
(1985 - 3 Marks)
5. A wire ABCDEF (with each side of length L) bent as shown
in figure and carrying a current I is placed in a uniform
C MCQs with One Correct Answer
magnetic induction B parallel to the positive y-direction. 1. A conducting circular loop of radius r carries a constant
The force experienced by the wire is ............ in the ............. r
direction. (1990 - 2 Marks) r i. It is placed in a uniform magnetic field B0 such
current
z
that B0 is perpendicular to the plane of the loop. The
D magnetic force acting on the loop is (1983 - 1 Mark)
(a) ir B0 (b) 2p ir B0
C
I
(c) zero (d) p ir B0
2. A battery is connected between two points A and B on the
circumference of a uniform conducting ring of radius r and
F resistance R. One of the arcs AB of the ring subtends an
E angle q at the centre.The value of the magnetic induction at
y
the centre due to the current in the ring is (1995S)
A (a) proportional to 2 (180° – q)
B
(b) inversely proportional to r
x
EBD_7036
P-130 Topic-wise Solved Papers - PHYSICS
(c) zero, only if q = 180° 9. An ionized gas contains both positive and negative ions. If
(d) zero for all values of q it is subjected simultaneously to an electric field along the
3. A proton, a deuteron and an a - particle having the same +x-direction and a magnetic field along the +z-direction, then
kinetic energy are moving in circular trajectories in a constant (a) positive ions deflect towards +y-direction and negative
magnetic field. If rp, rd, and ra denote respectively the radii ions towards -y direction (2000S)
of the trajectories of these particles, then (1997 - 1mark) (b) all ions deflect towards +y-direction
(a) ra = rp < rd (b) ra > rd > rp (c) all ions deflect towards –y-direction
(c) ra = rd > rp (d) rp = rd = ra (d) positive ions deflect towards –y-direction and negative
4. A circular loop of radius R, carrying current I, lies in x-y ions towards + y-direction.
plane with its centre at origin. The total magnetic flux through 10. A non-planar loop of conducting wire carrying a current I is
placed as shown in the figure. Each of the straight sections
x-y plane is (1999S - 2 Marks)
of the loop is of length 2a. The magnetic field due to this
(a) directly proportional to I
loop at the point P(a, 0, a) points in the direction (2001S)
(b) directly proportional to R
(c) inversely proportional to R
(d) zero
(a)
1
2
(
- ˆj + kˆ) z

5. A charged particle is released from rest in a region of steady


and uniform electric and magnetic fields which are parallel (b)
1
3
(
- ˆj + kˆ + iˆ ) I
y
to each other. The particle will move in a (1999S - 2 Marks)
(a) straight line
(c) helix
(b) circle
(d) cycloid (c)
3
(
1 ˆ ˆ ˆ
i + j+k )
6. A particle of charge q and mass m moves in a circular orbit x
of radius r with angular speed w . The ratio of the magnitude
of its magnetic moment to that of its angular momentum (d)
1 ˆ ˆ
2
( )
i +k 2a
depends on (2000S) 11. Two particles A and B of masses mA and mB respectively
(a) w and q (b) w , q and m
and having the same charge are moving in a plane. A uniform
(c) q and m (d) w and m
7. Two long parallel wires are at a distance 2d apart. They magnetic field exists perpendicular to this plane. The speeds
carry steady equal currents flowing out of the plane of the of the particles are vA and vB respectively and the trajectories
paper, as shown. The variation of the magnetic field B along are as shown in the figure. Then (2001S)
the line XX' is given by (2000S)
B
A
B

(a) X X’ (b) X X’ B

d d d d

B B
(a) mAvA < mBvB (b) mAvA > mBvB
(c) mA < mB and vA < vB (d) mA = mB and vA = vB
(c) X X’ (d) X X’ 12. A coil having N turns is wound tightly in the form of a spiral
with inner and outer radii a and b respectively. When a
d d d d
current I passes through the coil, the magnetic field at the
8. An infinitely long conductor PQR is bent to form a right center is (2001S)
angle as shown in Figure. A current I flows through PQR.
The magnetic field due to this current at the point M is H1. m o NI 2m o NI
(a) (b)
Now, another infinitely long straight conductor QS is b a
connected at Q so that current is I/2 in QR as well as in QS, m o NI b m0 IN a
the current in PQ remaining unchanged. The magnetic field (c) ln (d) ln
at M is now H2. The ratio H1/H2 is given by (2000S) 2(b - a ) a 2 (b - a ) b
M
13. A particle of mass m and charge q moves with a constant
velocity v along the positive x-direction. It enters a region
containing a uniform magnetic field B directed along the
90° Q negative z-direction, extending from x = a to x = b. The
–¥
P 90° S minimum value of v required so that the particle can just
enter the region x > b is (2002S)
R qbB q (b - a ) B
(a) (b)
+¥ m m
(a) 1/2 (b) 1 qaB q (b + a ) B
(c) (d)
(c) 2/3 (d) 2 m 2m
Moving Charges and Magnetism P-131

14. A long straight wire along the Z-axis carries a current I in the 18. A current carrying loop is placed in a uniform magnetic field
r in four different orientations, I, II, III & IV arrange them in
negative Z-direction. The magnetic vector field B at a point
having coordinates (x, y) in the Z = 0 plane is (2002S) the decreasing order of Potential Energy (2003S)

(a)
(
m 0 I yiˆ - xjˆ ) (b)
(
m 0 I xiˆ + yjˆ ) ^
n
(
2p x + y 2 2
) (
2p x + y 2 2
) I. B

(c)
(
m 0 I xjˆ - yiˆ ) (d)
(
m 0 I xiˆ - yjˆ ) II. B
(
2p x + y 2 2
) (
2p x + y 2 2
) n^
15. The magnetic field lines due to a bar magnet are correctly
shown in (2002S)
III B
N N
n^
^
n

(a) (b) IV B

S S (a) I > III > II > IV (b) I > II > III > IV
(c) I > IV > II > III (d) III > IV > I > II
N N 19. An electron travelling with a speed u along the positive x-
axis enters into a region of magnetic field where B = –B0 k̂
(x > 0). It comes out of the region with speed v then (2004S)
(c) (d) (a) v = u at y > 0 ×B y
(b) v = u at y < 0 e– u
S S
(c) v > u at y > 0 x®
16. For a positively charged particle moving in a x-y plane initially
(d) v > u at y < 0
along the x-axis, there is a sudden change in its path due to ur
the presence of electric and/or magnetic fields beyond P. 20. A magnetic field B = B0 Jˆ , exists in the region a < x < 2a,
The curved path is shown in the x-y plane and is found to be uv
and B = – B0 ˆj , in the region 2a < x < 3a, where B0 is a
non-circular. Which one of the following combinations is
positive constant. A positive point charge moving with a
possible? (2003S) r
uur uur y velocity v = v0iˆ , where v0 is a positive constant, enters the
(a) E = 0; B = biˆ + ck ˆ
uur uur magnetic field at x = a. The trajectory of the charge in this
(b) E = aiˆ; B = ckˆ + aiˆ region can be like (2007)
uur uur P x
(c) E = 0; B = cjˆ + bkˆ B0
uur uur
(d) E = aiˆ; B = ckˆ + bjˆ
17. A conducting loop carrying a current I is placed in a uniform 0 x
a 2a 3a
magnetic field pointing into the plane of the paper as shown.
The loop will have a tendency to (2003S) -B 0
B Y z z

x
(a) a 2a 3a
x (b) a 2a 3a
x

I
z z

a 2a 3a
(c) a 2a 3a
x (d) x
(a) contract (b) expand
(c) move towards +ve x-axis (d) move towards -ve x-axis.
EBD_7036
P-132 Topic-wise Solved Papers - PHYSICS
21. A thin flexible wire of length L is connected to two adjacent
fixed points and carries a current I in the clockwise direction, D MCQs with One or More than One Correct
as shown in the figure. When the system is put in a uniform 1. A magnetic needle is kept in a non uniform magnetic field. It
magnetic field of strength B going into the plane of the experiences (1982 - 3 Marks)
paper, the wire takes the shape of a circle. The tension in the (a) a force and a torque
wire is (2010) (b) a force but not a torque
(c) a torque but not a force
(d) neither a force nor a torque
2. A proton moving with a constant velocity passes through
a region of space without any change in its velocity. If E
an d B represent th e electric and magnetic fields
IBL IBL IBL respectively, this region of space may have :
(a) IBL (b) (c) (d) (1985 - 2 Marks)
p 2p 4p
22. A long insulated copper wire is closely wound as a spiral of (a) E = 0, B = 0 (b) E = 0, B ¹ 0
(c) E ¹ 0, B = 0 (d) E ¹ 0, B ¹ 0
‘N’ turns. The spiral has inner radius ‘a’ and outer radius
3. A rectangular loop carrying a current i is situated near a
‘b’. The spiral lies in the XY plane and a steady current ‘I’ long straight wire such that the wire is parallel to one of the
flows through the wire. The Z-component of the magnetic sides of the loop and is in the plane of the loop. If steady
field at the centre of the spiral is (2011) current I is established in the wire as shown in the figure,
m0 NI æbö the loop will : (1985 - 2 Marks)
(a) ln ç ÷
2(b - a) èaø i
m0 NI æb+ aö I
(b) ln çè ÷
2(b - a) b-aø
m NI æbö
(c) 0 ln ç ÷
2b èaø (a) rotate about an axis parallel to the wire
m0 NI æb+ aö (b) move away from the wire
(d) ln çè ÷ (c) move towards the wire
2b b-aø
23. A loop carrying current I lies in the x-y plane as shown in (d) remain stationary
4. Two thin long parallel wires seperated by a distance ‘b’ are
the figure. The unit vector k̂ is coming out of the plane of carrying a current ‘i’ amp each. The magnitude of the force
the paper. The magnetic moment of the current loop is per unit lenght exerted by one wire on the other is
(2012) (1986 - 2 Marks)
2 ˆ m 0i 2 m 0i 2 m 0i m 0i
(a) a I k (a) (b) (c) (d)
b 2 2 p b 2 pb 2pb 2
æ p ö a 2 I kˆ 5. Two particles X and Y having equal charges, after being
(b) çè + 1÷ø
2 accelerated through the same potential difference, enter a
p region of uniform magnetic field and describe circular paths
(c) - æç + 1ö÷ a 2 I kˆ of radii R1 and R2 respectively. The ratio of the mass of X to
è2 ø
that of Y is (1988 - 2 Marks)
(d) (2p + 1)a2 I kˆ (a) ( R1 / R2 )1/ 2 (b) R2 / R1
24. An infinitely long hollow conducting cylinder with inner (c) ( R1 / R2 )2 (d) R1 / R2 .
radius R/2 and outer radius R carries a uniform current
density along its length. The magnitude of the magnetic 6. A particle of charge + q and mass m moving under the
r influence of a uniform electric field Eiˆ and uniform magnetic
field, B as a function of the radial distance r from the axis
field B k̂ follows a trajectory from P to Q as shown in fig.
is best represented by (2012)
The velocities at P and Q are viˆ and – 2vjˆ . Which of the
r following statement (s) is/are correct ? (1991 - 2 Marks)
r B
(a) B (b) v E
P

aa B
r
r B
(c) B (d) Q
x
2a 2v
Moving Charges and Magnetism P-133

3 é mv 2 ù 13. A long current carrying wire, carrying current I1 such that I1


(a) E = ê ú is flowing out from the plane of paper is placed at O. A
4 êë qa úû
steady state current I2 is flowing in the loop ABCD
3 é mv3 ù
(b) Rate of work done by the electric field at P is ê ú C (2006 - 5M, –1)
4 êë a úû
B
(c) Rate of work done by the electric field at P is zero I2
(d) Rate of work done by both the fields at Q is zero O
7. A microameter has a resistance of 100 W and a full scale O'
I1
range of 50 mA . It can be used as a voltmeter or as a higher
range ammeter provided a resistance is added to it. Pick the A
correct range and resistance combination (s) D
(a) the net force is zero
(1991 - 2 Marks)
(a) 50 V range with 10 kW resistance in series (b) the net torque is zero
(b) 10 V range with 200 kW resistance in series (c) as seen from O, the loop will rotate in clockwise along
(c) 5 mA range with 1W resistance in parallel OO' axis
(d) 10 mA range with 1W resistance in parallel (d) as seen from O, the loop will rotate in anticlockwise
8. A current I flows along the length of an infinitely long, direction along OO' axis
straight, thin-walled pipe. Then (1993-2 Marks) 14. A particle of mass m and charge q, moving with velocity v
(a) the magnetic field at all points inside the pipe is the enters Region II normal to the boundary as shown in the
same, but not zero. figure. Region II has a uniform magnetic field B perpendicu-
(b) the magnetic field at any point inside the pipe is zero lar to the plane of the paper. The length of the Region II is l
(c) the magnetic field is zero only on the axis of the pipe Choose the correct choice(s). (2008)
(d) the magnetic field is different at different points inside Region I Region II Region III
the pipe. ××××××
9. H , He+ and O++ all having the same kinetic energy pass
+ ××××××
××××××
through a region in which there is a uniform magnetic field V ××××××
perpendicular to their velocity. The masses of H+, He+ and ××××××
O2+ are 1 amu, 4 amu and 16 amu respectively. Then
(1994 - 2 Marks) (a) The particle enters Region III only if its velocity
qlB
+
(a) H will be deflected most
(b) O2+ will be deflected most v>
m
(c) He+ and O2+ will be deflected equally (b) The particle enters Region III only if its velocity
(d) all will be deflected equally qlB
10. Two particles, each of mass m and charge q, are attached to v<
m
the two ends of a light rigid rod of length 2R. The rod is (c) Path length of the particle in Region II is maximum when
rotated at constant angular speed about a pependicular axis qlB
passing through its centre. The ratio of the magnitudes of velocity v =
m
the magnetic moment of the system and its angular (d) Time spent in Region II is same for any velocity v as
momentum about the centre of the rod is (1998S - 2 Marks) long as the particle returns to Region I
q q 2q q 15. Two metallic rings A and B, identical in shape and size but
(a) (b) (c) (d)
2m m m pm having different resistivities rA and rB, are kept on top of
11. Two very long, straight, parallel wires carry steady currents two identical solenoids as shown in the figure. When current
I & -I respectively. The distance between the wires is d. At I is switched on in both the solenoids in identical manner,
a certain instant of time, a point charge q is at a point the rings A and B jump to heights hA and hB, respectively,
equidistant from the two wires, in the plane of the wires. Its with hA > hB. The possible relation(s) between their
instantaneous velocity v is perpendicular to this plane. The resistivities and their masses mA and mB is(are) (2009)
magnitude of the force due to the magnetic field acting on A B
the charge at this instant is (1998S - 2 Marks)
m 0 Iqv m 0 Iqv 2m0 Iqv
(a) (b) (c) (d) 0
2 pd pd pd
12. The following field line can never represent (2006 - 5M, –1)
(a) induced electric field
(b) magnetostatic field
(c) gravitational field of a mass at rest (a) rA > rB and mA = mB (b) rA< rB and mA = mB
(c) rA > rB and mA > mB (d) rA < rB and mA < mB
(d) electrostatic field
EBD_7036
P-134 Topic-wise Solved Papers - PHYSICS
16. An electron and a proton are moving on straight parallel y
paths with same velocity. They enter a semi infinite region R R
of uniform magnetic field perpendicular to the velocity. l p/6 p/4 x
Which of the following statement(s) is / are true? (2011)
L R R L
(a) They will never come out of the magnetic field region. r
(b) They will come out travelling along parallel paths. If B is along $z , F µ (L + R)
(a)
(c) They will come out at the same time. r
(b) If B is along x̂ , F = 0
(d) They will come out at different times. r
17. Consider the motion of a positive point charge in a region (c) If B is along ŷ , F µ (L + R)
where there are simultaneous uniform electric and magnetic r
r r (d) If B is along $z , F = 0
fields E = E0 ˆj and B = B0 ˆj . At time t = 0, this charge has 21. Consider two identical galvanometers and two identical
r resistors with resistance R. If the internal resistance of the
velocity v in the in the x-y plane, making an angle q with the
x-axis. Which of the following option(s) is (are) correct for galvanometers RC < R/2, which of the following statement(s)
time t > 0? (2012) about any one of the galvanometers is (are) true?
(a) If q = 0°, the charge moves in a circular path in the x-z (JEE Adv. 2016)
plane. (a) The maximum voltage range is obtained when all the
(b) If q = 0°, the charge undergoes helical motion with components are connected in series
constant pitch along the y-axis. (b) The maximum voltage range is obtained when the two
(c) If q = 10°, the charge undergoes helical motion with its resistors and one galvanometer are connected in series,
pitch increasing with time, along the y-axis. and the second galvanometer is connected in parallel
(d) If q = 90°, the charge undergoes linear but accelerated to the first galvanometer
motion along the y-axis. (c) The maximum current range is obtained when all the
18. A particle of mass M and positive charge Q, moving with a components are connected in parallel
r (d) The maximum current range is obtained when the two
constant velocity u1 = 4iˆ ms-1 , enters a region of uniform galvanometers are connected in series and the
static magnetic field, normal to the x-y plane. The region of combination is connected in parallel with both the
the magnetic field extends from x = 0 to x = L for all values of resistors
y. After passing through this region, the particle emerges on
the other side after 10 milliseconds with a velocity E Subjective Problems
r
u2 = 2( 3iˆ + ˆj ) ms -1 . The correct statement(s) is (are) 1. A bar magnet with poles 25 cm apart and of strength 14.4
(JEE Adv. 2013) amp-m rests with centre on a frictionless pivot. It is held in
(a) The direction of the magnetic field is – z direction equilibrium at an angle of 60° with respect to a uniform
(b) The direction of the magnetic field is +z direction magnetic field of induction 0.25 Wb/m2, by applying a force
50pM F at right angles to its axis at a point 12 cm from pivot.
(c) The magnitude of the magnetic field units Calculate F. What will happen if the force F is removed?
3Q
(1978)
100pM
(d) The magnitude of the magnetic field is units 2. A bar magnet is placed with its north pole pointing north
3Q and its south pole pointing south. Draw a figure to show
19. A steady current I flows along an infinitely long hollow the location of neutral points. (1979)
cylindrical conductor of radius R. This cylinder is placed 3. A potential difference of 600 volts is applied across the
coaxially inside an infinite solenoid of radius 2R. The
plates of a parallel plate condenser. The separation between
solenoid has n turns per unit length and carries a steady
the plates is 3 mm. An electron projected vertically, parallel
current I. Consider a point P at a distance r from the common
axis. The correct statement(s) is (are) (JEE Adv. 2013) to the plates, with a velocity of 2 × 106 m/sec moves
undeflected between the plates. Find the magnitude and
(a) In the region 0 < r < R, the magnetic field is non-zero
direction of the magnetic field in the region between the
(b) In the region R < r < 2R, the magnetic field is along the
condenser plates. (Neglect the edge effects). (Charge of the
common axis
electron = – 1.6 × 10–19 coulomb) (1981- 3 Marks)
(c) In the region R < r < 2R, the magnetic field is tangential
to the circle of radius r, centered on the axis 600 VOLTS
+

(d) In the region r > 2R, the magnetic field is non-zero


20. A conductor (shown in the figure) carrying constantrcurrent
I is kept in the x-y plane in a uniform magnetic field B . If F is
the magnitude of the total magnetic force acting on the
conductor, then the correct statement(s) is(are) +

(JEE Adv. 2015)


Moving Charges and Magnetism P-135

4. A particle of mass m = 1.6 × 10–27 kg and located at a distance of 5 metres and 2 metres respectively
charge q = 1.6 × 10–19 C enters a region q from a collinear point R (see figure) (1990 - 8 Marks)
of uniform magnetic field of strength F 5m
1 tesla along the direction shown in 2m
fig. The speed of the particle is 107 m/ P Q R
E - -O
X - - - -OX - - - - O - - ............. ® x
s. (i) The magnetic field is directed 2.5A IA
along the inward normal to the plane 45° (i) An electron moving with a velocity of 4 × 105 m/s along
of the paper. The particle leaves the region of the field at the the positive x - direction experiences a force of
point F. Find the distance EF and the angle q. (ii) If the magnitude 3.2 × 10–20 N at the point R. Find the value
direction of the field is along the outward normal to the of I.
plane of the paper, find the time spent by the particle in the (ii) Find all the positions at which a third long parallel wire
region of the magnetic field after entering it at E. carrying a current of magnitude 2.5 amperes may be
(1984- 8 Marks) placed so that the magnetic induction at R is zero.
5. A beam of protons with a velocity 4 × 105 m/sec enters a uniform 9. A wire loop carrying a current I is placed in the x-y plane as
magnetic field of 0.3 tesla at an angle of 60° to the magnetic shown in fig. (1991 - 4 + 4 Marks)
field. Find the radius of the helical path taken by the proton
beam. Also find the pitch of the helix (which is the distance M
travelled by a proton in the beam parallel to the magnetic field y
a +Q
during one period of rotation). (1986 - 6 Marks)

120°
6. Two long straight parallel wires are A×
I P
O x
2 metres apart, perpendicular to the
plane of the paper (see figure). The
1.

N
6
m

wire A carries a current of 9.6 amps,


2m

(a) If a particle with charge +Q and mass m is placed at the


directed into the plane of the paper. S uur
The wire B carries a current such m centre P and given a velocity v along NP (see figure),
1.2
that the magnetic field of induction B find its instantaneous acceleration.
10 (b) If an external uniform magnetic induction field
at the point P, at a distance of 10/ m uur
11 11 B = Biˆ is applied, find the force and the torque acting
metre from the wire B, is zero. on the loop due to this field.
Find : (1987 - 7 Marks) P
10. A straight segment OC (of length L meter) of a circuit carrying
(i) The magnitude and direction of the current in B . a current I amp is placed along the x-axis (Fig.). Two infinetely
(ii) The magnitude of the magnetic field of induction at the
long straight wires A and B, each extending from z = - ¥ to
point S.
+¥ , are fixed at y = – a meter and y = + a meter respectively,,
(iii) The force per unit length on the wire B.
as shown in the figure.
7. A pair of stationary and infinitely long bent wires are placed
in the XY plane as shown in fig. The wires carry currents of Y
i = 10 amperes each as shown. The segments L and M are B
along the X-axis. The segments P and Q are parallel to the Y- I C X
O
axis such that OS = OR = 0.02 m. Find the magnitude and
direction of the magnetic induction at the origin O. A
(1989 - 6 Marks) Z
Y If the wires A and B each carry a current I amp into the plane
¥ of the paper, obtain the expression for the force acting on
the segment OC. What will be the force on OC if the current
Ii in the wire B is reversed ? (1992 - 10 Marks)
Q 11. An electron gun G emits
electrons of ener gy 2keV S
L R M ¥
¥ X travelling in the positive x-
Ii O S Ii B
P direction. The electrons are
required to hit the spot S where
Ii GS = 0.1m, and the line GS makes 60° x
an angle of 60° with the x-axis as v G
¥ shown in the fig. A uniform
8. Two long parallel wires carrying current 2.5 amperes and I ® r
magnetic field B parallel to GS exists. Find B parallel to
ampere in the same direction (directed into the plane of the GS exists in the region outside the electron gun. Find the
paper ) are held at P and Q respectively such that they are minimum value of B needed to make the electrons hit S.
perpendicular to the plane of paper. The points P and Q are (1993-7 Marks)
EBD_7036
P-136 Topic-wise Solved Papers - PHYSICS
12. A long horizontal wire AB, which is free to move in a vertical 16. The region between x = 0 and x = L is filled with uniform,
plane and carries a steady current of 20A, is in equilibrium at
steady magnetic field B kˆ . A particle of mass m, positive
0
a height of 0.01 m over another parallel long wire CD which
is fixed in a horizontal plane and carries a steady current of charge q and velocity n0iˆ travels along x-axis and enters
30A, as shown in figure . Show that when AB is slightly
the region of the magnetic field. Neglect gravity throughout
depressed, it executes simple harmonic motion. Find the
the question. (1999 - 10 Marks)
period of oscillations. (1994 - 6 Marks)
(a) Find the value of L if the particle emerges from the
A B region of magnetic field with its final velocity at angle
30° to its initial velocity.
C D (b) Find the final velocity of the particle and the time spent
13. An electron in the ground state of hydrogen atom is by it in the magnetic field, if the magnetic field now
revolving in anticlock-wise direction in a circular orbit of extends up to 2.1L.
radius R. (1996 - 5 Marks) 17. A circular loop of radius R is bent along a diameter and
(i) Obtain an expression for given a shape as shown in the figure. One of the semicircles
the orbital magnetic (KNM) lies in the x-z plane and the other one (KLM) in the y-
dipole moment of the z plane with their centres at the origin. Current I is flowing
electron. through each of the semi circles as shown in figure.
(ii) The atom is placed in a
uniform magnetic
ur L y
induction B such that
the plane-normal of the I M
electron-orbit makes an x
angle of 30° with the
magnetic induction. Find N z
the torque experienced K I
by the orbiting electron.
(a) A particle of charge q is released at the origin with a
ur uur
14. Three infinitely long thin wires, each carrying current i in velocity v = –v iˆ . Find the instantaneous force F
0
the same direction, are in the x-y plane of a gravity free
on the particle. Assume that space is gravity free.
space. The central wire is along the y-axis while the other
two are along x = ± d. (b) If an external uniform magnetic field Bo ĵ is applied,
(i) Find the locus of the points for which the magnetic uur uur
field B is zero. (1997 - 5 Marks) determine the force F1 and F2 on the semicircles KLM
(ii) If the central wire is displaced along the Z-direction by uur
and KNM due to the field and the net force F on the
a small amount and released, show that it will execute
simple harmonic motion. If the linear density of the loop. (2000 - 10 Marks)
wires is l, find the frequency of oscillation. 18. A current of 10 A flows around a closed path in a circuit
15. A u n i for m , c on st a n t which is in the horizontal plane as shown in the figure. The
magnetic field B is directed y l circuit consists of eight alternating arcs of radii r1 = 0.08 m
at an angle of 45° to the x S 0
R and r1 = 0.12 m. Each arc subtends the same angle at the
axis in the xy-plane. PQRS center.
is a rigid, square wire frame D
carrying a steady current I0, x
O r2 C
with its centre at the origin
O. At time t = 0, the frame is A
at rest in the position as P Q r1
shown in Figure, with its
sides parallel to the x and y axes. Each side of the frame is of
mass M and length L.
(a) What is the torque t about O acting on the frame due
to the magnetic field? (a) Find the magnetic field produced by this circuit at the
(b) Find the angle by which the frame rotates under the center. (2001-10 Marks)
action of this torque in a short interval of time Dt, and (b) An infinitely long straight wire carrying a current of 10
the axis about this rotation occurs. (Dt is so short that A is passing through the center of the above circuit
any variation in the torque during this interval may be vertically with the direction of the current being into
neglected.) Given : the moment of inertia of the frame the plane of the circuit. What is the force acting on the
about an axis through its centre perpendicular to its wire at the center due to the current in the circuit?
4 2 What is the force acting on the arc AC and the straight
plane is ML . (1998 - 8 Marks) segment CD due to the current at the center?
3
Moving Charges and Magnetism P-137

19. A wheel of radius R having charge Q, uniformly distributed 20. A proton and an a-particle are accelerated with same
on the rim of the wheel is free to rotate about a light horizontal potential difference and they enter in the region of constant
rod. The rod is suspended by light inextensible strings and magnetic field B perpendicular to the velocity of particles.
a magnetic field B is applied as shown in the figure. The Find the ratio of radius of curvature of proton to the radius
initial tensions in the strings are T0. If the breaking tension of curvature of a - particle. (2004 - 2 Marks)
3T0 21. In a moving coil galvanometer, torque on the coil can be
of the strings are , find the maximum angular velocity expressed as t = ki , where i is current through the wire and
2
k is constant. The rectangular coil of the galvanometer having
w0 with which the wheel can be rotated. (2003 - 4 Marks)
number of turns N, area A and moment of inertia I is placed
d
in magnetic field B. Find (2005 - 6 Marks)
(a) k in terms of given parameters N, I, A and B
(b) the torsion constant of the spring, if a current i 0
produces a deflection of p/2 in the coil.
(c) the maximum angle through which the coil is deflected,
w0 if charge Q is passed through the coil almost
instantaneously. (ignore the damping in mechanical
B
oscillations).

F Match the Following

DIRECTIONS (Qs. 1-3) : Each question contains statements given in two columns, which have to be p q r s t
matched. The statements in Column-I are labelled A, B, C and D, while the statements in Column-II are
A p q r s t
labelled p, q, r and s. Any given statement in Column-I can have correct matching with ONE OR MORE
statement(s) in Column-II. The appropriate bubbles corresponding to the answers to these questions B p q r s t
have to be darkened as illustrated in the following example : C p q r s t
If the correct matches are A-p, s and t; B-q and r; C-p and q; and D-s then the correct darkening of D p q r s t
bubbles will look like the given.
1. Match the following columns : (2006, 6M)
Column I Column II Column I
(A) Dielectric ring uniformly charged (p) Constant electrostatic field out of system
(B) Dielectric ring uniformly charged (q) Magnetic field strength
rotating with angular velocity w
(C) Constant current in ring i (r) Electric field (induced)
(D) i = i0coswt (s) Magnetic dipole moment
2. Column I gives certain situations in which a straight metallic wire of resistance R is used and Column II gives some resulting
effects. Match the statements in Column I with the statements in Column II and indicate your answer by darkening appropriate
bubbles in the 4 × 4 matrix given in the ORS. (2007)
Column I Column II
(A) A charged capacitor is connected to the ends of (p) A constant current flows through the wire
the wire
(B) The wire is moved perpendicular to its length with (q) Thermal energy is generated in the wire
a constant velocity in a uniform magnetic field
perpendicular to the plane of motion
(C) The wire is placed in a constant electric field that (r) A constant potential difference develops between the ends
has a direction along the length of the wire of the wire
(D) A battery of constant emf is connected to the (s) charges of constant magnitude appear at the ends of the
ends of the wire. wire
3. Two wires each carrying a steady current I are shown in four configurations in Column I. Some of the resulting effects are
described in Column II. Match the statements in Column I with the statements in column II and indicate your answer by
darkening appropriate bubbles in the 4 × 4 matrix given in the ORS. (2007)
Column I Column II
(A) Point P is situated midway between the wires. (p) The magnetic fields (B) at P due to the currents

P in the wires are in the same direction.


EBD_7036
P-138 Topic-wise Solved Papers - PHYSICS
(B) Point P is situated at the mid-point of the (q) The magnetic fields (B) at P due to the currents
line joining the centers of the circular in the wires are in opposite directions.
wires, which have same radii.

(C) Point P is situated at the mid-point of the (r) There is no magnetic field at P.
line joining the centers of the circular
wires, which have same radii.

(D) Point P is situated at the common center (s) The wires repel each other.
of the wires.

Q S
G Comprehension Based Questions d d

PASSAGE-1
Wire 1 a Wire 2
Advanced countries are making use of powerful electromagnets
to move trains at very high speed. These trains are called maglev
trains (abbreviated from magnetic levitation). These trains float P R
on a guideway and do not run on steel rail tracks. 4. When d » a but wires are not touching the loop, it is found
Instead of using a engine based on fossil fuels, they make use of that the net magnetic field on the axis of the loop is zero at a
magnetic field forces. The magnetized coils are arranged in the height h above the loop. In that case (JEE Adv. 2014)
guide way which repels the strong magnets placed in the train's (a) current in wire 1 and wire 2 in the direction PQ and RS,
under carriage. This helps train move over the guideway , a technic respectively and h » a
called electro-dynamic suspension. When current passes in the (b) current in wire 1 and wire 2 in the direction PQ and SR,
coils of guideway , a typical magnetic field is set up between the respectively and h » a
undercarriage of train and guideway which pushes and pull the (c) current in wire 1 and wire 2 in the direction PQ and SR,
train along the guideway depending on the requirement. respectively and h » 1.2a
The lack of friction and its aerodynamic style allows the train to (d) current in wire 1 and wire 2 in the direction PQ and RS,
more at very high speed. respectively and h » 1.2a
1. The levitation of the train is due to (2006 - 5M, –2) 5. Consider d >> a, and the loop is rotated about its diameter
(a) Mechanical force (b) Electrostatic attraction parallel to the wires by 30° from the position shown in the
(c) Electrostatic repulsion (d) Magnetic repulsion figure. If the currents in the wires are in the opposite
2. The disadvantage of maglev trains is that (2006 - 5M, –2) directions, the torque on the loop at its new position will be
(a) More friction (b) Less pollution (assume that the net field due to the wires is constant over
(c) Less wear & tear (d) High initial cost the loop). (JEE Adv. 2014)
3. The force which makes maglev move (2006 - 5M, –2) m0 I 2a2 m0 I 2a2
(a) Gravitational field (b) Magnetic field (a) (b)
d 2d
(c) Nuclear forces (d) Air drag
3m 0 I 2 a 2 3m 0 I 2 a 2
PASSAGE-2 (c) (d)
d 2d
The figure shows a circular loop of radius a with two long parallel
PASSAGE-3
wires (numbered 1 and 2) all in the plane of the paper. The distance In a thin rectangular metallic strip a constant current I flows along
of each wire from the centre of the loop is d. The loop and the wire the positive x-direction, as shown in the figure. The length, width
are carrying the same current I.The current in the loop is in the
counterclockwise direction if seen from above.
and thickness of the strip aresu l, w and d, respectively.
A uniform magnetic field B is applied on the strip along the
positive y-direction. Due to this, the charge carriers experience a
Moving Charges and Magnetism P-139

net deflection along the z-direction. This results in accumulation (b) Statement-1 is True, Statement-2 is True; Statement-2
of charge carriers on the surface PQRS and appearance of equal is NOT a correct explanation for Statement-1
and opposite charges on the face opposite to PQRS. A potential (c) Statement -1 is True, Statement-2 is False
difference along the z-direction is thus developed. Charge (d) Statement -1 is False, Statement-2 is True
accumulation continues until the magnetic force is balanced by
the electric force. The current is assumed to be uniformly I Integer Value Correct Type
distributed on the cross-section of the strip and carried by
electrons. 1. A steady current I goes through a wire loop PQR having
shape of a right angle triangle with PQ = 3x, PR = 4x and QR
= 5x. If the magnitude of the magnetic field at P due to this
m I
loop is k æç 0 ö÷ , find the value of k. (2009)
è 48 px ø
2. A long circular tube of length 10 m and radius
0.3 m carries a current I along its curved
6. Consider two different metallic strips (1 and 2) of the same surface as shown. A wire-loop of resistance
material. Their lengths are the same, widths are w1 and w2 0.005 ohm and of radius 0.1 m is placed inside
and thicknesses are d1 and d2 respectively. Two points K the tube with its axis coinciding with the axis
and M are symmetrically located on the opposite faces parallel
to the x-y plane (see figure). V1 and V2 are the potential of the tube. The current varies as I = I0cos(300 t) where I0 is
differences between K and M in strips 1 and 2, respectively. constant. If the magnetic moment of the loop is Nm0I0sin
Then, for a given current I flowing through them in a given (300 t), then ‘N’ is (2011)
magnetic field strength B, the correct statement(s) is(are) 3. A cylindrical cavity of diameter a exists inside a cylinder of
(JEE Adv. 2015) diameter 2a as shown in the figure. Both the cylinder and
(a) If w1 = w2 and d1 = 2d2, then V2 = 2V1 the cavity are infinity long. A uniform current density J flows
(b) If w1 = w2 and d1 = 2d2, then V2 = V1 along the length. If the magnitude of the magnetic field at
(c) If w1 = 2w2 and d1 = d2, then V2 = 2V1
N
(d) If w1 = 2w2 and d1 = d2, then V2 = V1 the point P is given by m 0 aJ , then the value of N is
7. Consider two different metallic strips (1 and 2) of same 12
dimensions (length l, width w and thickness d) with carrier (2012)
densities n1 and n2, respectively. Strip 1 is placed in magnetic
field B1 and strip 2 is placed in magnetic field B2, both along
positive y-directions. Then V1 and V2 are the potential
differences developed between K and M in strips 1 and 2,
respectively. Assuming that the current I is the same for
both the strips, the correct option(s) is(are) (JEE Adv. 2015)
(a) If B1 = B2 and n1 = 2n2, then V2 = 2V1
(b) If B1 = B2 and n1 = 2n2, then V2 = V1
(c) If B1 = 2B2 and n1 = n2, then V2 = 0.5V1 4. Two parallel wires in the plane of the paper are distance X0
(d) If B1 = 2B2 and n1 = n2, then V2 = V1 apart. A point charge is moving with speed u between the
wires in the same plane at a distance X1 from one of the
H Assertion & Reason Type Questions wires. When the wires carry current of magnitude I in the
same direction, the radius of curvature of the path of the
1. Statement-1 : The sensitivity of a moving coil galvanometer point charge is R1. In contrast, if the currents I in the two
is increased by placing a suitable magnetic material as a wires have directions opposite to each other, the radius of
core inside the coil. R
X0
and curvature of the path is R2. If = 3 , the value of 1 is
Statement-2 : Soft iron has a high magnetic permeability X1 R2
and cannot be easily magnetized or demagnetized. (2008) (JEE Adv. 2014)
(a) Statement-1 is True, Statement-2 is True; Statement-2
is a correct explanation for Statement-1
EBD_7036
P-140 Topic-wise Solved Papers - PHYSICS

Section-B JEE Main / AIEEE


1. If in a circular coil A of radius R, current I is flowing and in 7. A thin rectangular magnet suspended freely has a period of
another coil B of radius 2R a current 2I is flowing, then the oscillation equal to T. Now it is broken into two equal halves
ratio of the magnetic fields BA and BB, produced by them (each having half of the original length) and one piece is
will be [2002] made to oscillate freely in the same field. If its period of
(a) 1 (b) 2 (c) 1/2 (d) 4
T'
2. If an electron and a proton having same momenta enter oscillation is T ' , the ratio is [2003]
perpendicular to a magnetic field, then [2002] T
(a) curved path of electron and proton will be same 1 1
(ignoring the sense of revolution) (a) (b) 1 (c) 2 (d)
2 2 2 4
(b) they will move undeflected
(c) curved path of electron is more curved than that of the 8. A magnetic needle lying parallel to a magnetic field requiers
proton W units of work to turn it through 600 . The torque needed
(d) path of proton is more curved. to maintain the needle in this position will be [2003]
3. Wires 1 and 2 carrying currents i1 and i2 respectively are
inclined at an angle θ to each other. What is the force on a (a) 3 W (b) W (c) 3
W (d) 2 W
small element dl of wire 2 at a distance of r from wire 1 (as 2
shown in figure) due to the magnetic field of wire 1? 9. The magnetic lines of force inside a bar magnet [2003]
[2002] (a) are from north-pole to south-pole of the magnet
(b) do not exist
1 2
(c) depend upon the area of cross-section of the bar magnet
(d) are from south-pole to north-pole of the Magnet
i1 r i2 10. Curie temperature is the temperature above which
dl
[2003]
q
(a) a ferromagnetic material becomes paramagnetic
(b) a paramagnetic material becomes diamagnetic
m0 m0 (c) a ferromagnetic material becomes diamagnetic
(a) i1i2 dl tan q (b) i1i2 dl sin q (d) a paramagnetic material becomes ferromagnetic
2pr 2pr
11. A current i ampere flows along an infinitely long straight
m0 m0 thin walled tube, then the magnetic induction at any point
(c) i1i2 dl cos q (d) i1i2 dl sin q
2pr 4 pr inside the tube is [2004]
4. The time period of a charged particle undergoing a circular
motion in a uniform magnetic field is independent of its m 0 2i
(a) . tesla (b) zero
(a) speed (b) mass [2002] 4p r
(c) charge (d) magnetic induction
2i
5. A particle of mass M and charge Q moving with velocity (c) infinite (d) tesla
r r
v describe a circular path of radius R when subjected to a
uniform transverse magnetic field of induction B. The work 12. A long wire carries a steady current. It is bent into a circle of
done by the field when the particle completes one full circle one turn and the magnetic field at the centre of the coil is B.
is [2003] It is then bent into a circular loop of n turns. The magnetic
field at the centre of the coil will be [2004]
æ Mv 2 ö (a) 2n B (b) n B2 (c) nB (d) 2 n2 B
(a) ç ÷ 2pR (b) zero
13. The magnetic field due to a current carrying circular loop of
è R ø
radius 3 cm at a point on the axis at a distance of 4 cm from the
(c) BQ 2p R (d) B Qv2p R centre is 54 mT. What will be its value at the centre of loop?
[2004]
6. A particle of charge - 16 ´ 10 -18 coulomb moving with
(a) 125 mT (b) 150 mT (c) 250 mT (d) 75 mT
velocity 10ms -1 along the x-axis enters a region where a 14. Two long conductors, separated by a distance d carry current
magnetic field of induction B is along the y-axis, and an I1 and I2 in the same direction. They exert a force F on each
other. Now the current in one of them is increased to two
electric field of magnitude 10 4 V / m is along the negative
times and its direction is reversed. The distance is also
z-axis. If the charged particle continues moving along the
increased to 3d. The new value of the force between them is
x-axis, the magnitude of B is [2003]
[2004]
(a) 103Wb / m2 (b) 105 Wb / m 2
2F F F
(c) 1016 Wb / m 2 (d) 10 -3Wb / m 2 (a) - (b) (c) –2 F (d) -
3 3 3
Moving Charges and Magnetism P-141

15. The length of a magnet is large compared to its width and 23. A long solenoid has 200 turns per cm and carries a current i.
breadth. The time period of its oscillation in a vibration The magnetic field at its centre is 6.28 × 10–2 Weber/m2.
magnetometer is 2s. The magnet is cut along its length into Another long solenoid has 100 turns per cm and it carries a
three equal parts and these parts are then placed on each i
other with their like poles together. The time period of this current . The value of the magnetic field at its centre is
3
combination will be [2004] [2006]
–2
(a) 1.05 × 10 Weber/m 2 (b) 1.05 × 10 Weber/m2
–5
2 2
(a) 2 3s (b) s (c) 2 s (d) s –3 2
3 3 (c) 1.05 × 10 Weber/m (d) 1.05 × 10–4 Weber/m2
16. The materials suitable for making electromagnets should 24. A long straight wire of radius a carries a steady current i.
have [2004] The current is uniformly distributed across its cross section.
(a) high retentivity and low coercivity The ratio of the magnetic field at a/2 and 2a is [2007]
(b) low retentivity and low coercivity (a) 1/2 (b) 1/4 (c) 4 (d) 1
(c) high retentivity and high coercivity 25. A current I flows along the length of an infinitely long,
(d) low retentivity and high coercivity straight, thin walled pipe. Then [2007]
17. Two concentric coils each of radius equal to 2 p cm are placed (a) the magnetic field at all points inside the pipe is the
at right angles to each other. 3 ampere and 4 ampere are the same, but not zero
currents flowing in each coil respectively. The magnetic (b) the magnetic field is zero only on the axis of the pipe
induction in Weber / m 2 at the centre of the coils will be (c) the magnetic field is different at different points inside
the pipe
(m 0 = 4p ´ 10 -7 Wb / A.m ) [2005] (d) the magnetic field at any point inside the pipe is zero
26. A charged particle with charge q enters a region of constant,
(a) 10 -5 (b) 12 ´ 10 -5 (c) 7 ´ 10
-5
(d) 5 ´ 10 -5 ur ur
uniform and mutually orthogonal fields E and B with a
18. A charged particle of mass m and charge q travels on a ur ur
r
circular path of radius r that is perpendicular to a magnetic velocity v perpendicular to both E and B , and comes out
field B. The time taken by the particle to complete one r
without any change in magnitude or direction of v . Then
revolution is [2005] r ur ur r ur ur
(a) v = B ´ E / E 2 (b) v = E ´ B / B 2 [2007]
2pq 2 B 2 pmq 2pm 2pqB r ur ur 2 r ur ur 2
(a) (b) (c) (d) (c) v = B ´ E / B (d) v = E ´ B / E
m B qB m
19. A magnetic needle is kept in a non-uniform magnetic field. It 27. A charged particle moves through a magnetic field
experiences [2005] perpendicular to its direction. Then [2007]
(a) neither a force nor a torque (a) kinetic energy changes but the momentum is constant
(b) a torque but not a force (b) the momentum changes but the kinetic energy is
(c) a force but not a torque constant
(d) a force and a torque (c) both momentum and kinetic energy of the particle are
20. A uniform electric field and a uniform magnetic field are not constant
acting along the same direction in a certain region. If an (d) both momentum and kinetic energy of the particle are
electron is projected along the direction of the fields with a constant
certain velocity then [2005] 28. Two identical conducting wires AOB and COD are placed at
(a) its velocity will increase right angles to each other. The wire AOB carries an electric
(b) Its velocity will decrease current I1 and COD carries a current I2. The magnetic field
(c) it will turn towards left of direction of motion on a point lying at a distance d from O, in a direction
(d) it will turn towards right of direction of motion perpendicular to the plane of the wires AOB and COD, will
be given by [2007]
21. Needles N1, N2 and N3 are made of a ferromagnetic, a 1
paramagnetic and a diamagnetic substance respectively. A m0 m 0 æ I1 + I 2 ö 2
magnet when brought close to them will [2006] (a) ( I12 + I 2 2 ) (b) ç ÷
2pd 2p è d ø
(a) attract N1 and N2 strongly but repel N3 1
(b) attract N1 strongly, N2 weakly and repel N3 weakly (c)
m0
2pd
( )
I12 + I 22 2 (d)
m0
2pd
( I1 + I 2 )
(c) attract N1 strongly, but repel N2 and N3 weakly
29. A horizontal overhead powerline is at height of 4m from the
(d) attract all three of them ground and carries a current of 100A from east to west.
22. In a region, steady and uniform electric and magnetic fields The magnetic field directly below it on the ground is
are present. These two fields are parallel to each other. A (m0 = 4p×10 –7 Tm A–1) [2008]
charged particle is released from rest in this region. The (a) 2.5 × 10–7 T southward
path of the particle will be a [2006] (b) 5 × 10–6 T northward
(a) helix (b) straight line (c) 5 × 10–6 T southward
(c) ellipse (d) circle (d) 2.5 × 10–7 T northward
EBD_7036
P-142 Topic-wise Solved Papers - PHYSICS

30. Relative permittivity and permeability of a material e r and B


m r , respectively. Which of the following values of these
quantities are allowed for a diamagnetic material? [2008]
X X¢
(a) e r = 0.5, m r = 1.5 (b) e r = 1.5, mr = 0.5 (b)
(c) e r = 0.5, m r = 0.5 (d) e r = 1.5, mr = 1.5
d d
DIRECTIONS : Question numbers 31 and 32 are based on the
following paragraph.
B
PASSAGE
A current loop ABCD is held fixed on the plane of the paper as
shown in the figure. The arcs BC (radius = b) and DA (radius = a) X X¢
of the loop are joined by two straight wires AB and CD. A steady (c)
current I is flowing in the loop. Angle made by AB and CD at the
origin O is 30°. Another straight thin wire with steady current I1 d d
flowing out of the plane of the paper is kept at the origin.

B B
[2009]
a A
X X¢
I1 30° I (d)
O
D d d
b C 34. A current I flows in an infinitely long wire with cross section
in the form of a semi-circular ring of radius R. The magnitude
31. The magnitude of the magnetic field (B) due to the loop
of the magnetic induction along its axis is: [2011]
ABCD at the origin (O) is :
m0 I m0 I m0 I m0 I
m o I (b - a ) mo I é b - a ù (a) 2 (b) (c) (d)
(a) (b) 2p R 2pR 4p R p2 R
24ab 4p êë ab úû
35. A charge Q is uniformly distributed over the surface of non-
mo I conducting disc of radius R. The disc rotates about an axis
(c) [2(b - a) + p 3 ( a + b)] (d) zero
4p perpendicular to its plane and passing through its centre
32. Due to the presence of the current I1 at the origin: with an angular velocity w. As a result of this rotation a
(a) The forces on AD and BC are zero. magnetic field of induction B is obtained at the centre of the
(b) The magnitude of the net force on the loop is given by disc. If we keep both the amount of charge placed on the
disc and its angular velocity to be constant and vary the
I1I
m o [2(b - a ) + p (a + b] . radius of the disc then the variation of the magnetic induction
4p 3 at the centre of the disc will be represented by the figure :
(c) The magnitude of the net force on the loop is given by [2012]
m o II1
(b - a).
24ab
(d) The forces on AB and DC are zero.
B B
33. Two long parallel wires are at a distance 2d apart. They
(a) (b)
carry steady equal currents flowing out of the plane of the
paper as shown. The variation of the magnetic field B along
the line XX' is given by [2010] R R

(a) X X¢ B B
(c) (d)

d d R R
Moving Charges and Magnetism P-143

36. Proton, deuteron and alpha particle of same kinetic energy


are moving in circular trajectories in a constant magnetic
field. The radii of proton, deuteron and alpha particle are
respectively rp, rd and ra. Which one of the following relation
is correct? [2012]
(a) ra = rp = rd (b) ra = rp < rd
L
ra > rd > rp ra =rd > rp q
(c) (d)
37. Two short bar magnets of length 1 cm each have magnetic
moments 1.20 Am2 and 1.00 Am2 respectively. They are
placed on a horizontal table parallel to each other with their
N poles pointing towards the South. They have a common I I
magnetic equator and are separated by a distance of 20.0
cm. The value of the resultand horizontal magnetic induction
at the mid-point O of the line joining their centres is close to
pgL plgL
(Horizontal component of earth.s magnetic induction is 3.6× (a) 2 tan q (b) tan q
10.5Wb/m2) [JEE Main 2013] µ0 µ0
(a) 3.6 × 10.5 Wb/m2 (b) 2.56 × 10.4 Wb/m2
(c) 3.50 × 10.4 Wb/m2 (d) 5.80 × 10.4 Wb/m2 plgL plgL
(c) sin q (d) 2sin q
38. A conductor lies along the z-axis at -1.5 £ z < 1.5 m and µ0 cos q µ0 cos q
carries a fixed current of 10.0 A in -â z direction (see figure). 41. A rectangular loop of sides 10 cm and 5 cm carrying a current
r 1 of 12 A is placed in different orientations as shown in the
For a field B = 3.0 ´10-4 e -0.2x aˆ y T, find the power required figures below : [JEE Main 2015]
to move the conductor at constant speed to x = 2.0 m, y = 0 m z
in 5 ´10-3 s. Assume parallel motion along the x-axis. I
B
[JEE Main 2014] I I
(A) y
z I
x
1.5 z
I
B
(B) I
y I y
B I
2.0 x I
z
x
I B
–1.5
I
(C) I y
(a) 1.57 W (b) 2.97 W
I
(c) 14.85 W (d) 29.7 W x
z
39. The coercivity of a small magnet where the ferromagnet gets
demagnetized is 3 ´ 103 Am-1. The current required to be B
passed in a solenoid of length 10 cm and number of turns 100,
(D) I
so that the magnet gets demagnetized when inside the I y
solenoid, is: [JEE Main 2014] I
x I
(a) 30 mA (b) 60 mA If there is a uniform magnetic field of 0.3 T in the positive z
(c) 3 A (d) 6 A
direction, in which orientations the loop would be in (i) stable
40. Two long current carrying thin wires, both with current I, are
equilibrium and (ii) unstable equilibrium ? [JEE Main 2015]
held by insulating threads of length L and are in equilibrium
(a) (B) and (D), respectively
as shown in the figure, with threads making an angle 'q' with
the vertical. If wires have mass l per unit length then the (b) (B) and (C), respectively
value of I is : [JEE Main 2015] (c) (A) and (B), respectively
(g = gravitational acceleration) (d) (A) and (C), respectively
EBD_7036
P-144 Topic-wise Solved Papers - PHYSICS
42. Two identical wires A and B, each of length 'l', carry the 44. Hysteresis loops for two magnetic materials A and B are
same current I. Wire A is bent into a circle of radius R and given below : [JEE Main 2016]
wire B is bent to form a square of side 'a'. If BA and BB are
the values of magnetic field at the centres of the circle and D B

BA
square respectively, then the ratio B is: [JEE Main 2016]
B
H H
p2 p2
(a) (b)
16 8 2
(A) (B)
2 2
p p
(c) (d) These materials are used to make magnets for elecric
8 16 2
generators, transformer core and electromagnet core. Then
43. A galvanometer having a coil resistance of 100 W gives a
it is proper to use :
full scale deflection, when a currect of 1 mA is passed through
(a) A for transformers and B for electric generators.
it. The value of the resistance, which can convert this
(b) B for electromagnets and transformers.
galvanometer into ammeter giving a full scale deflection for
(c) A for electric generators and trasformers.
a current of 10 A, is : [JEE Main 2016]
(d) A for electromagnets and B for electric generators.
(a) 0.1 W (b) 3W
(c) 0.01W (d) 2W
CHAPTER
Electromagnetic Induction
15 and Alternating Current
Section-A JEE Advanced/ IIT-JEE
A Fill in the Blanks 2. A thin semi-circular conducting ring
of radius R is falling with its plane
1. A uniformly wound solenoidal coil of self inductance vertical in horizontal magnetic
1.8 × 10–4 henry and resistance 6 ohm is broken up into two ur
induction B . At the position MNQ
identical coils. These identical coils are then connected in
the speed of the ring is v, and the
parallel across a 15-volt battery of negligible resistance. The
potential difference developed across
time constant for the current in the circuit is ........... seconds
and the steady state current through the battery is ............... the ring is
amperes. (1989 - 2 Marks) (a) zero
2. In a straight conducting wire, a constant current is flowing (b) BvpR2/2 and M is at higher potential
from left to right due to a source of emf. When the source is (c) pRBv and Q is at higher potential (1996 - 2 Marks)
switched off, the direction of the induced current in the wire (d) 2RBv and Q is at higher potential.
will ................ (1993 - 1 Marks) 3. Two identical circular loops of metal wire are lying on a table
B True/False without touching each other. Loop-A carries a current which
increases with time. In response, the loop-B
1. An e.m.f. can be induced between the two ends of a straight (a) remains stationary (1999S - 2 Marks)
copper wire when it is moved through a uniform magnetic (b) is attracted by the loop-A
field. (1980) (c) is repelled by the loop-A
2. A coil of metal wire is kept stationary in a non-uniform
(d) rotates about its CM, with CM fixed
magnetic field. An e.m.f. is induced in the coil.
(1986 - 3 Marks) 4. A coil of inductance 8.4 mH and resistance 6 W is connected
3. A conducting rod AB moves parallel to the x-axis (see Fig.) to a 12 V battery. The current in the coil is 1.0 A at
in a uniform magnetic field pointing in the positive approximately the time (1999S - 2 Marks)
z-direction. The end A of the rod gets positively charged. (a) 500 s (b) 25 s (c) 35 ms (d) 1 ms
(1987 - 2 Marks) 5. A uniform but time-varying magnetic field B(t) exists in a
circular region of radius a and is directed into the plane of
the paper, as shown. The magnitude of the induced electric
field at point P at a distance r from the centre of the circular
region (2000S )
(a) is zero
(b) decreases as 1/r
(c) increases as r
(d) decreases as 1/r2
6. A coil of wire having inductance and resistance has a
C MCQs with One Correct Answer conducting ring placed coaxially within it. The coil is
1. A thin circular ring of area A is held perpendicular to a connected to a battery at time t = 0, so that a time-dependent
uniform magnetic field of induction B. A small cut is made in current l1(t) starts flowing through the coil. If I2(t) is the
the ring and a galvanometer is connected across the ends current induced in the ring, and B(t) is the magnetic field at
such that the total resistance of the circuit is R. When the the axis of the coil due to I1(t), then as a function of time
ring is suddenly squeezed to zero area, the charge flowing (t > 0), the product I2(t) B(t) (2000S )
through the galvanometer is (1995S) (a) increases with time
(b) decreases with time
BR AB B2 A
(a) (b) (c) ABR (d) (c) does not vary with time
A R R2 (d) passes through a maximum
EBD_7036
P-146 Topic-wise Solved Papers - PHYSICS
7. A metallic square loop ABCD is moving in its own plane 12. A small bar magnet is being
with velocity v in a uniform magnetic field perpendicular to slowly inserted with constant
its plane as shown in the figure. An electric field is induced velocity inside a solenoid as
(2001S) shown in figure. Which graph best represents the
A B
relationship between emf induced with time (2004S)
v

D C
(a) in AD, but not in BC (b) in BC, but not in AD (a) (b)
Time Time
(c) neither in AD nor in BC (d) in both AD and BC
8. Two circular coils can be arranged in any of the three
situations shown in the figure. Their mutual inductance will
be (2001S)
(c) Time (d)
Time
13. An infinitely long cylinder is kept parallel to an uniform
magnetic field B directed along positive z-axis. The
direction of induced current as seen from the z-axis will
(a) (b) (c) be (2005S)
(a) maximum in situation (a) (b) maximum in situation (b) (a) zero
(c) maximum in situation (c) (d) the same in all situations (b) anticlockwise of the +ve z axis
9. As shown in the figure, P and Q are two coaxial conducting (c) clockwise of the +ve z axis
loops separated by some distance. When the switch S is (d) along the magnetic field
closed, a clockwise current IP flows in P (as seen by E) and 14. Find the time constant (in ms) for the given RC circuits in the
an induced current IQ1 flows in Q. The switch remains closed given order respectively (2006 - 3M, –1)
for a long time. When S is opened, a current IQ2 flows in Q.
Then the direction IQ1 and IQ2 (as seen by E) are (2002S)

V V V
(a) respectively clockwise and anti-clockwise
R1 C1
(b) both clockwise R1 C1
(c) both anti-clockwise R2 C2
R1
(d) respectively anti-clockwise and clockwise
10. A short-circuited coil is placed in a time-varying magnetic R2
field. Electrical power is dissipated due to the current induced R2 C2
in the coil. If the number of turns were to be quadrupled and R1 = 1W, R2 = 2W, C1 = 4mF , C2 = 2mF
the wire radius halved, the electrical power dissipated would
be (2002S ) 8 8 8 8
(a) 18, 4, (b) 18, , 4 (c) 4, 18, (d) 4, , 18
(a) halved (b) the same 9 9 9 9
(c) doubled (d) quadrupled 15. The figure shows certain wire
c d
11. When an AC source of emf e = E0 sin(100t) is connected segments joined together to
a b
across a circuit, the phase difference between the emf e and form a coplanar loop. The
the current i in the circuit is observed to be p 4 , as shown loop is placed in a
perpendicular magnetic field
in the diagram. If the circuit consists possibly only of R-C
in the direction going into the
or R-L or L-C in series, find the relationship between the
plane of the figure. The
two elements (2003S)
magnitude of the field
(a) R = 1k W , C = 10 µF i increases with time. I1 and I2 are the currents in the segments
e
ab and cd. Then, (2009)
(b) R = 1k W , C = 1 µF
(a) I1 > I2
(c) R = 1k W , L = 10 H (b) I1 < I2
(c) I1 is in the direction ba and I2 is in the direction cd
(d) R = 1k W , L = 1 H (d) I1 is in the direction ab and I2 is in the direction dc
Electromagnetic Induction and Alternating Current P-147

16. An AC voltage source of variable angular frequency w and 7. A series R– C circuit is connected to AC voltage source.
fixed amplitude V0 is connected in series with a capacitance Consider two cases; (A) when C is without a dielectric
C and an electric bulb of resistance R (inductance zero). medium and (B) when C is filled with dielectric of constant 4.
When w is increased (2010) The current IR through the resistor and voltage VC across
(a) the bulb glows dimmer the capacitor are compared in the two cases. Which of the
(b) the bulb glows brighter following is/are true? (2011)
(c) total impedance of the circuit is unchanged
(d) total impedance of the circuit increases (a) I RA > I RB (b) I RA < I RB

D MCQs with One or More than One Correct (c) VCA > VCB (d) VCA < VCB
8. In the given circuit, the AC source has w = 100 rad/s.
1. L, C and R represent the physical quantities, inductance, Considering the inductor and capacitor to be ideal, the correct
capacitance and resistance respectively. The combination(s) choice(s) is (are) (2012)
which have the dimensions of frequency are (1984- 2 Marks) 100mF 100W
(a) 1/RC (b) R/L (c) 1/ LC (d) C/L
2. A conducting square loop of side L 0.5 H 50W
and resistance R moves in its plane
with a uniform velocity v
perpendicular to one of its sides. A
20 V
magnetic induction B, constant in
time and space, pointing
~
(a) The current through the circuit, I is 0.3 A.
perpendicular and into the plane of
the loop exists everywhere. (1989 - 2 Marks) (b) The current through the circuit, I is 0.3 2A
The current induced in the loop is: (c) The voltage across 100 W resistor = 10 2V
(a) BLv/R clockwise (b) BLv/R anticlockwise (d) The voltage across 50 W resistor = 10 V
(c) 2BLv/R anticlockwise (d) zero. 9. A current carrying infinitely long wire is kept along the
3. Two different coils have self-inductances L1 = 8 mH and L2 diameter of a circular wire loop, without touching it, the
= 2 mH. The current in one coil is increased at a constant correct statement(s) is(are) (2012)
rate. The current in the second coil is also increased at the (a) The emf induced in the loop is zero if the current is
constant.
same constant rate. At a certain instant of time, the power
(b) The emf induced in the loop is finite if the current is
given to the two coils is the same. At that time, the current,
constant.
the induced voltage and the energy stored in the first coil
(c) The emf induced in the loop is zero if the current
are i1, V1 and W1 respectively. Corresponding values for the
decreases at a steady rate.
second coil at the same instant are i2, V2 and W2 respectively. (d) The emf induced in the loop is infinite if the current
Then: (1994 - 2 Marks) decreases at a steady rate.
i1 1 i1 W1 1 V1 10. At time t = 0, terminal A in B D
(a) =
i2 4 (b) i = 4 (c) W = 4 (d) V = 4 the circuit shown in the A
2 2 2 figure is connected to B
4. A small square loop of wire of side l is placed inside a large by a key and an alternating 50 V
current I(t) = I0cos (wt), C = 20 mF
square loop of wire of side L(L>>l). The loops are co-planar
with I0 = 1 A and w = 500
and their centres coincide. The mutual inductance of the rad s–1 starts flowing in it
system is proportional to (1998S - 2 Marks) with the initial direction R = 10 W
(a) l/L (b) l2/L (c) L/l (d) L2/l
7p
5. The SI unit of inductance, the henry, can be written as shown in the figure. At t = , the key is switched from B
(1998S - 2 Marks) 6w
to D. Now onwards only A and D are connected. A total
(a) weber/ampere (b) volt-second/ampere
charge Q flows from the battery to charge the capacitor fully.
(c) joule/(ampere)2 (d) ohm-second
If C = 20 mF, R = 10 W and the battery is ideal with emf of 50
6. A metal rod moves at a constant velocity in a direction
perpendicular to its length. A constant, uniform magnetic V, identify the correct statement(s). (JEE Adv. 2014)
field exists in space in a direction perpendicular to the rod (a) Magnitude of the maximum charge on the capacitor
7p
as well as its velocity. Select the correct statement(s) from before t = is 1 × 10–3 C
the following (1998S - 2 Marks) 6w
(b) The current in the left part of the circuit just before
(a) The entire rod is at the same electric potential.
7p
(b) There is an electric field in the rod. t= is clockwise
(c) The electric potential is highest at the centre of the rod 6w
and decreases towards its ends. (c) Immediately after A is connected to D, the current in R
(d) The electric potential is lowest at the centre of the rod, is 10 A
and increases towards its ends (d) Q = 2 × 10–3 C
EBD_7036
P-148 Topic-wise Solved Papers - PHYSICS
11. A conducting loop in the shape of a right angled isosceles I(x)
triangle of height 10 cm is kept such that the 90° vertex is
very close to an infinitely long conducting wire (see the
figure). The wire is electrically insulated from the loop. The (c)
hypotenuse of the triangle is parallel to the wire. The current x
0 L 2L 3L 4L
in the triangular loop is in counterclockwise direction and F(x)
increased at a constant rate of 10 A s–1. Which of the
following statement(s) is(are) true? (JEE Adv. 2016)

10 cm 90° (d) x
0 L 2L 3L 4L

æ m0 ö
(a)The magnitude of induced emf in the wire is çè ÷ø
p
volt
(b) If the loop is rotated at a constant angular speed about E Subjective Problems
æ m0 ö
the wire, an additional emf of çè ÷ø volt is induced in 1. A current from A to B is increasing in magnitude. What is
p the direction of induced current, if any, in the loop as shown
the wire in the figure? (1979)
(c) The induced current in the wire is in opposite direction
to the current along the hypotenuse
(d) There is a repulsive force between the wire and the
loop
12. A rigid wire loop of square shape having side of length L
and resistance R is moving along the x-axis with a constant
velocity v0 in the plane of the paper. At t = 0, the right edge 2. The two rails of a railway track, insulated from each other and
of the loop enters a region of length 3L where there is a the ground, are connected to a milli voltmeter. What is the
uniform magnetic field B0 into the plane of the paper, as reading of the milli voltmeter when a train travels at a speed of
shown in the figure. For sufficiently large v0. the loop 180 km/hour along the track, given that the vertical component
eventually crosses the region. Let x be the location of the of earth’s magnetic field is 0.2 × 10–4 weber/m2 & the rails are
right edge of the loop. Let v(x), I(x) and F(x) represent the separated by 1 meter? (1981- 4 Marks)
velocity of the loop, current in the loop, and force on the 3. Three identical closed coils A, B and C are placed with their
loop, respectively, as a function of x. Counter-clockwise planes parallel to one another. Coils A and C carry equal
current is taken as positive. (JEE Adv. 2016) currents as shown in Fig. Coils B and C are fixed in position
and coil A is moved towards B with uniform motion. Is there
any induced current in B ? If no, give reasons. If yes mark
the direction of the induced current in the diagram.
R (1982 - 2 Marks)
L
v0
x
0 L 2L 3L 4L
Which of the following schematic plot(s) is (are) correct?
(Ignore gravity)
v(x)
v0 4. A square metal wire loop
of side 10 cms and
(a) r e s i s t a n c e
x
1 ohm is moved with a
0 L 2L 3L 4L constant velocity v0 in a
I(x)
uniform magnetic field of
induction B = 2 webers/
m2 as shown in the figure. The magnetic field lines are
3L 4L
perpendicular to the plane of the loop (directed into the
(b) x paper). The loop is connected to a network of resistors
0 L 2L each of value 3 ohms. The resistances of the lead wires OS
and PQ are negligible. What should be the speed of the loop
so as to have a steady current of 1 milliampere in the loop ?
Give the direction of current in the loop. (1983 - 6 Marks)
Electromagnetic Induction and Alternating Current P-149

5. Space is divided by the line AD into two regions. Region I 8. A rectangular frame ABCD, made of a
is field free and the Region II has a uniform magnetic field B uniform metal wire, has a straight
directed into the plane of the paper. ACD is a semicircular connection between E and F made of
conducting loop of radius r with centre at O, the plane of the same wire, as shown in Fig. AEFD
the loop being in the plane of the paper. The loop is now is a square of side 1m, and EB = FC =
made to rotate with a constant angular velocity w about an 0.5m. The entire circuit is placed in
axis passing through O and the perpendicular to the plane steadily increasing, uniform magnetic
of the paper. The effective resistance of the loop is R.
(1985 - 6 Marks) field directed into the plane of the paper and normal to it.
The rate of change of the magnetic field is 1T/s. The
resistance per unit length of the wire is 1W / m. Find the
magnitudes and directions of the currents in the segments
AE, BE and EF. (1993-5 Marks)
9. Two parallel vertical metallic rails AB R1
A C
and CD are separated by 1 m. They
(i) Obtain an expression for the magnitude of the induced are connected at two ends by L
current in the loop.
resistances R1 and R 2 as shown in
(ii) Show the direction of the current when the loop is
Figure. A horizontal metallic bar L of
entering into the Region II.
(iii) Plot a graph between the induced e.m.f and the time of mass 0.2 kg slides without friction
rotation for two periods of rotation. vertically down the rails under the R2
6. Two long parallel horizontal rails, a distance d apart and action of gravity. There is a uniform
horizontal magnetic field of 0.6 Tesla B D
each having a resistance l per unit length, are joined at
one end by a resistance R. A perfectly conducting rod MN perpendicular to the plane of the rails. It is observed that
of mass m is free to slide along the rails without friction when the terminal velocity is attained, the powers dissipated
(see figure). There is a uniform magnetic field of induction B in R1 and R2 are 0.76 Watt and 1.2 watt respectively. Find the
normal to the plane of the paper and directed into the paper. terminal velocity of the bar L and the values of R1 and R2.
A variable force F is applied to the rod MN such that, as the (1994 - 6 Marks)
rod moves, a constant current flows through R. 10. A metal rod OA of mass ‘m’ and length ‘r’ is kept rotating
(1988 - 6 Marks) with a constant angular speed w in a vertical plane about a
horizontal axis at the end O. The free end A is arranged to
slide without friction along a fixed conducting circular ring
in the same plane as that of rotation. A uniform and constant
r
magnetic induction B is applied perpendicular and into the
plane of rotation as shown in the figure below. An inductor
L and an external resistance R are connected through a switch
S between the point O and a point C on the ring to form an
electrical circuit. Neglect the resistance of the ring and the
(i) Find the velocity of the rod and the applied force F as
rod. Initially, the switch is open. (1995 - 10 Marks)
function of the distance x of the rod from R.
(ii) What fraction of the work done per second by F is
converted into heat ?
7. A circuit containing a two position switch S is shown in fig.
(1991 - 4 + 4 Marks)

(a) What is the induced emf across the terminals of the


switch?
(b) The switch S is closed at time t = 0.
(i) Obtain an expression for the current as a function
of time.
(ii) In the steady state, obtain the time dependence
(a) The switch S is in position ‘1’. Find the potential of the torque required to maintain the constant
difference VA – VB and the rate of production of joule angular speed, given that the rod OA was along
heat in R1. the positive X-axis at t = 0.
(b) If now the switch S is put in position 2 at t = 0 find 11. A solenoid has an inductance of 10 henry and a resistance
(i) steady current in R4 and of 2 ohm. It is connected to a 10 volt battery. How long will
(ii) the time when current in R4 is half the steady value. it take for the magnetic energy to reach 1/4 of its maximum
value? (1996 - 3 Marks)
Also calculate the energy stored in the inductor L at that time
EBD_7036
P-150 Topic-wise Solved Papers - PHYSICS
12. A pair of parallel now released and is found to stay in the horizontal position
horizontal conducting in equilibrium. (2002 - 5 Marks)
rails of negligible
resistance shorted at
one end is fixed on a
table. The distance
between the rails is L.
A conducting massless
rod of resistance R can
slide on the rails frictionlessly. The rod is tied to a massless
string which passes over a pulley fixed to the edge of the
table. A mass m, tied to the other end of the string hangs
vertically. A constant magnetic field B exists perpendicular
to the table. If the system is released from rest, calculate.
(1997 - 5 Marks)
(i) the terminal velocity achieved by the rod, a nd
(ii) the acceleration of the mass at the instant when the (a) What is the direction of the current I in PQ?
velocity of the rod is half the terminal velocity.
(b) Find the magnetic force on the arm RS.
13. A magnetic field B = B0 (y/a) k̂ is into the paper in the (c) Find the expression for I in terms of B0, a, b and m.
+z direction. B0 and a are positive constants. A square loop 16. A metal bar AB can slide on
EFGH of side a, mass m and resistance R, in x – y plane, two parallel thick metallic rails
starts falling under the influence of gravity see figure) Note
separated by a distance l . A
the directions of x and y axes in figure. (1999 - 10 Marks))
resistance R and an
inductance L are connected
to the rails as shown in the
figure. A long straight wire
carrying a constant current I0
is placed in the plane of the
rails and perpendicular to them as shown. The bar AB is
held at rest at a distance x0 from the long wire. At t = 0, it is
made to slide on the rails away from the wire. Answer the
Find
following questions. (2002 - 5 Marks)
(a) the induced current in the loop and indicate its direction. di df
(b) the total Lorentz force acting on the loop and indicate (a) Find a relation among i, and , where i is the
dt dt
its direction, and current in the circuit and f is the flux of the magnetic
(c) an expression for the speed of the loop, v(t) and its field due to the long wire through the circuit.
terminal value.
(b) It is observed that at time t = T, the metal bar AB is at a
14. An inductor of
distance of 2x0 from the long wire and the resistance R
inductance L = 400 L carries a current i1. Obtain an expression for the net
mH and resistors of E R1 charge that has flown through resistance R from t = 0
resistances R1 = 2W
to t = T.
and R2 = 2 W are S R2
(c) The bar is suddenly stopped at time T. The current
conn ected to a i1
battery of emf E = 12 V as shown in the figure. The internal through resistance R is found to be at time 2T. Find
4
resistance of the battery is negligible. The switch S is closed L
the value of in terms of the other given quantities.
at time t = 0. What is the potential drop across L as a function R
of time? After the steady state is reached, the switch is 17. A square loop of side ‘a’ with a capacitor of capacitance C
opened. What is the direction and the magnitude of current is located between two current carrying long parallel wires
through R1 as a function of time? (2001-5 Marks) as shown. The value of I in the wires is given as I = I0 sinwt.
(2003 - 4 Marks)
15. A rectangular loop PQRS made from a uniform wire has
length a, width b and mass m. It is free to rotate about the
arm PQ, which remains hinged along a horizontal line taken
as the y-axis (see figure). Take the vertically upward
direction as the z-axis. A uniform magnetic field
ur
B = (3iˆ + 4kˆ) B exists in the region. The loop is held in the
0
(a) Calculate maximum current in the square loop.
x-y plane and a current I is passed through it. The loop is (b) Draw a graph between charges on the upper plate of
the capacitor vs time.
Electromagnetic Induction and Alternating Current P-151

18. In a series L–R circuit (L = 35 mH and R = 11 W), a variable


emf source (V = V0 sin wt) of Vrms = 220 V and frequency 50
Hz is applied. Find the current amplitude in the circuit and
phase of current with respect to voltage. Draw current-time
graph on given graph (p = 22/7). (2004 - 4 Marks)

20. A long solenoid of radius a and


number of turns per unit length n
is enclosed by cylindrical shell of
radius R. thickness d (d << R ) and
length L. A variable current i = i0
sinwt flows through the coil. If the
19. In the figure both cells A and B are of equal emf. Find R for resistivity of the material of
which potential difference across battery A will be zero, cylindrical shell is r, find the
long time after the switch is closed. Internal resistance of induced current in the shell.
batteries A and B are r1 and r2 respectively (r1 > r2). (2005 - 4 Marks)
(2004 - 4 Marks)

F Match the Following

DIRECTIONS : Each question contains statements given in two columns, which have to be matched. p q r s t
The statements in Column-I are labelled A, B, C and D, while the statements in Column-II are labelled
A p q r s t
p, q, r and s. Any given statement in Column-I can have correct matching with ONE OR MORE
B p q r s t
statement(s) in Column-II. The appropriate bubbles corresponding to the answers to these questions
have to be darkened as illustrated in the following example : C p q r s t
If the correct matches are A-p, s and t; B-q and r; C-p and q; and D-s then the correct darkening of D p q r s t
bubbles will look like the given.

1. You are given many resistances, capacitors and inductors. These are connected to a variable DC voltage source (the first two
circuits) or an AC voltage source of 50 Hz frequency (the next three circuits) in different ways as shown in Column II. When a
current I (steady state for DC or rms for AC) flows through the circuit, the corresponding voltage V1and V2 , (indicated in circuits)
are related as shown in Column I. Match the two (2010)
Column I Column II
V1 V2 V1 V2

6mH 6mH 2W
3m F
(A) I ¹ 0, V1 is (p) (C) V1 = 0, V2 = V (r)

V V
V1 V2
proportional to I
V1 V2
6mH 3m F
(D) I ¹ 0, V2 is (s)
6mH 2W
(B) I ¹ 0, V2 > V1 (q) V
proportional to I
V1 V2
V

1kW 3m F
(t)

V
EBD_7036
P-152 Topic-wise Solved Papers - PHYSICS
plant side so that the current is reduced to a smaller value. At the
G Comprehension Based Questions consumers' end, a step-down transformer is used to supply power
to the consumers at the specified lower voltage. It is reasonable
PASSAGE 1 to assume that the power cable is purely resistive and the
In the given circuit the capacitor (C) may be charged through transformers are ideal with power factor unity. All the currents
resistance R by a battery V by closing switch S1. Also when S1 is and voltages mentioned are rms values. (JEE Adv. 2013)
opened and S2 is closed the capacitor is connected in series with 4. If the direct transmission method with a cable of resistance
inductor (L). 0.4 W km–1 is used, the power dissipation| (in %) during
transmission is
(a) 20 (b) 30 (c) 40 (d) 50
5. In the method using the transformers, assume that the ratio
of the number of turns in the primary to that in the secondary
in the step-up transformer is 1 : 10. If the power to the
consumers has to be supplied at 200 V, the ratio of the number
of turns in the primary to that in the secondary in the step-
down transformer is
(a) 200 : 1 (b) 150 : 1 (c) 100 : 1 (d) 50 : 1
1. At the start, the capacitor was uncharged. When switch S1 PASSAGE 3
is closed and S2 is kept open, the time constant of this
A point charge Q is moving in a circular orbit of radius R in the
circuit is t. Which of the following is correct x-y plane with an angular velocity w. This can be considered as
(2006 – 5M, –2) Qw
CV equivalent to a loop carrying a steady current . A uniform
(a) after time interval t, charge on the capacitor is 2p
2 magnetic field along the positive z-axis is now switched on, which
(b) after time interval 2t, charge on the capacitor of CV increases at a constant rate from 0 to B in one second. Assume
(1 – e–2) that the radius of the orbit remains constant. The application of
the magnetic field induces an emf in the orbit. The induced emf is
(c) the work done by the voltage source will be half of the defined as the work done by an induced electric field in moving a
heat dissipated when the capacitor is fully charged unit positive charge around a closed loop. It is known that, for an
(d) after time interval 2t, charge on the capacitor is orbiting charge, the magnetic dipole moment is proportional to
CV (1 – e–1) the angular momentum with a proportionality constant g.
2. When the capacitor gets charged completely, S1 is opened (JEE Adv. 2013)
and S2 is closed. Then, (2006 – 5M, –2) 6. The magnitude of the induced electric field in the orbit at
(a) at t = 0, energy stored in the circuit is purely in the form any instant of time during the time interval of the magnetic
of magnetic energy field change is
(b) at any time t > 0, current in the circuit is in the same BR BR
direction (a) (b) (c) BR (d) 2BR
4 2
(c) at t > 0, there is no exchange of energy between the 7. The change in the magnetic dipole moment associated with
inductor and capacitor the orbit, at the end of the time interval of the magnetic field
(d) at any time t > 0, instantaneous current in the circuit change, is

may be V
C BQR 2 BQR 2
(a) –gBQR2 (b) - g (c) g (d) g BQR 2
L 2 2
3. Given that the total charge stored in the LC circuit is Q0, for
t ³ 0, the charge on the capacitor is (2006 – 5M, –2) H Assertion & Reason Type Questions
æp t ö æp t ö 1. Statement-1 : A vertical iron rod has coil of
(a) Q = Q0 cos ç + ÷ (b) Q = Q0 cos ç - ÷
è2 LC ø è2 LC ø wire wound over it at the bottom end. An
alternating current flows in the coil. The rod
d 2Q 1 d 2Q goes through a conducting ring as shown
(c) Q = - LC (d) Q = -
dt 2 LC dt 2 in the figure. The ring can float at a certain
height above the coil.
PASSAGE 2 Statement-2 : In the above situation, a
current is induced in the ring which interacts with the
A thermal power plant produces electric power of 600 kW at 4000
horizontal component of the magnetic field to produce an
V, which is to be transported to a place 20 km away from the power
average force in the upward direction. (2007)
plant for consumers' usage. It can be transported either directly
(a) Statement-1 is True, Statement-2 is True; Statement-2
with a cable of large current carrying capacity or by using a
is a correct explanation for Statement-1
combination of step-up and step-down transformers at the two
(b) Statement-1 is True, Statement-2 is True; Statement-2
ends. The drawback of the direct transmission is the large energy
is NOT a correct explanation for Statement-1
dissipation. In the method using transformers, the dissipation is
(c) Statement-1 is True, Statement-2 is False
much smaller. In this method , a step-up transformer is used at the
(d) Statement-1 is False, Statement-2 is True.
Electromagnetic Induction and Alternating Current P-153
z
I Integer Value Correct Type 45°
1. A series R-C combination is connected to an AC voltage of
a
angular frequency w = 500 radian/s. If the impedance of the 3R
R-C circuit is R 1.25 , the time constant (in millisecond) of
R y
the circuit is (2011)
O
2. A circular wire loop of radius R is placed in the x-y plane
centered at the origin O. A square loop of side a(a<<R) x
18. Two inductors L1 (inductance 1 mH, internal resistance 3
having two turns is placed with its centre at z = 3R along W) and L2 (inductance 2 mH, internal resistance 4 W), and a
the axis of the circular wire loop, as shown in figure. The resistor R (resistance 12 W) are all connected in parallel
plane of the square loop makes an angle of 45° with respect to across a 5 V battery. The circuit is switched on at time t = 0.
the z-axis. If the mutual inductance between the loops is given
The ratio of the maximum to the minimum current
m0 a 2 (Imax / I min) drawn from the battery is (JEE Adv. 2016)
by p/2 , then the value of p is (2012)
2 R

Section-B JEE Main / AIEEE


1. The power factor of an AC circuit having resistance (R) and 7. In an oscillating LC circuit the maximum charge on the
inductance (L) connected in series and an angular velocity capacitor is Q. The charge on the capacitor when the energy
w is is stored equally between the electric and magnetic field is
(a) R/ w L (b) R/(R2 + w 2L2)1/2 [2003]
(c) w L/R (d) R/(R2 – w 2L2)1/2 Q Q Q
2. A conducting square loop of side L and resistance R moves (a) (b) (c) (d) Q
2 3 2
in its plane with a uniform velocity v perpendicular to one of
8. The core of any transformer is laminated so as to [2003]
its sides. A magnetic induction B constant in time and space,
(a) reduce the energy loss due to eddy currents
pointing perpendicular and into the plane at the loop exists
(b) make it light weight
everywhere with half the loop outside the field, as shown in
(c) make it robust and strong
figure. The induced emf is [2002]
(d) increase the secondary voltage
9. Alternating current can not be measured by D.C. ammeter
because [2004]
L v (a) Average value of current for complete cycle is zero
(b) A.C. Changes direction
(c) A.C. can not pass through D.C. Ammeter
(d) D.C. Ammeter will get damaged.
(a) zero (b) RvB 10. In an LCR series a.c. circuit, the voltage across each of the
(c) vBL/R (d) vBL components, L, C and R is 50V. The voltage across the LC
3. The inductance between A and D is [2002] combination will be [2004]
(a) 100 V (b) 50 2 V (c) 50 V (d) 0 V (zero)
11. A coil having n turns and resistance RW is connected with
A 3H 3H 3H D a galvanometer of resistance 4RW. This combination is
moved in time t seconds from a magnetic field W1 weber to
(a) 3.66 H (b) 9 H (c) 0.66 H (d) 1 H. W2 weber. The induced current in the circuit is [2004]
4. In a transformer, number of turns in the primary coil are 140 n (W - W
(W2 - W1 ) 2 1)
and that in the secondary coil are 280. If current in primary (a) - (b) - 5 Rt
coil is 4 A, then that in the secondary coil is [2002] Rnt
(a) 4 A (b) 2 A (c) 6 A (d) 10 A. (W2 - W1 ) n(W2 - W1 )
5. Two coils are placed close to each other. The mutual (c) - (d) -
5 Rnt Rt
inductance of the pair of coils depends upon [2003]
12. In a uniform magnetic field of induction B a wire in the form
(a) the rates at which currents are changing in the two
coils of a semicircle of radius r rotates about the diameter of the
(b) relative position and orientation of the two coils circle with an angular frequency w. The axis of rotation is
(c) the materials of the wires of the coils perpendicular to the field. If the total resistance of the circuit
(d) the currents in the two coils is R, the mean power generated per period of rotation is
6. When the current changes from +2 A to -2A in 0.05 second, ( B pr w )2 ( B pr 2 w ) 2
an e.m.f. of 8 V is induced in a coil. The coefficient of self - (a) (b)
2R 8R
induction of the coil is [2003]
(a) 0.2 H (b) 0.4 H (c) 0.8 H (d) 0.1 H B pr 2 w ( B pr w 2 ) 2
(c) (d) [2004]
2R 8R
EBD_7036
P-154 Topic-wise Solved Papers - PHYSICS
13. In a LCR circuit capacitance is changed from C to 2 C. For 23. The flux linked with a coil at any instant 't' is given by
the resonant frequency to remain unchanged, the
inductance should be changed from L to [2004] f = 10t 2 - 50t + 250
(a) L/2 (b) 2 L (c) 4 L (d) L/4 The induced emf at t = 3s is [2006]
14. A metal conductor of length 1 m rotates vertically about one (a) –190 V (b) –10 V (c) 10 V (d) 190 V
of its ends at angular velocity 5 radians per second. If the 24. An inductor (L = 100 mH), a resistor (R = 100 W) and a
horizontal component of earth’s magnetic field is 0.2×10–4T, battery (E = 100 V) are initially connected in series as shown
then the e.m.f. developed between the two ends of the in the figure. After a long time the battery is disconnected
conductor is [2004] after short circuiting the points A and B. The current in the
(a) 5 mV (b) 50 mV (c) 5 mV (d) 50mV circuit 1 ms after the short circuit is [2006]
15. One conducting U tube can slide inside another as shown
in figure, maintaining electrical contacts between the tubes. L
The magnetic field B is perpendicular to the plane of the
figure . If each tube moves towards the other at a constant
speed v, then the emf induced in the circuit in terms of B, l
R
and v where l is the width of each tube, will be [2005]
X A X X X
BX A B
X v X E
X v (a) 1/eA (b) eA (c) 0.1 A (d) 1 A
X X X
X 25. In an a.c. circuit the voltage applied is E = E0 sin wt. The
X C
X X æ pö
(a) – Blv (b) Blv resulting current in the circuit is I = I 0 sin ç wt - ÷ . The
è 2ø
(c) 2 Blv (d) zero power consumption in the circuit is given by [2007]
16. The self inductance of the motor of an electric fan is 10 H. In
order to impart maximum power at 50 Hz, it should be E0 I 0
connected to a capacitance of [2005]
(a) P = 2 E0 I0 (b) P=
2
(a) 8 m F (b) 4 mF (c) 2 mF (d) 1 mF
E I
17. The phase difference between the alternating current and (c) P = zero (d)P= 0 0
2
p
emf is . Which of the following cannot be the constituent 26. An ideal coil of 10H is connected in series with a resistance
2
of 5W and a battery of 5V. 2second after the connection is
of the circuit? [2005]
(a) R, L (b) C alone (c) L alone (d) L, C made, the current flowing in ampere in the circuit is [2007]
18. A circuit has a resistance of 12 ohm and an impedance of 15 (a) (1 – e–1) (b) (1 – e) (c) e (d) e–1
ohm. The power factor of the circuit will be [2005] 27. Two coaxial solenoids are made by winding thin insulated
(a) 0.4 (b) 0.8 (c) 0.125 (d) 1.25 wire over a pipe of cross-sectional area A = 10 cm2 and
19. A coil of inductance 300 mH and resistance 2 W is connected length = 20 cm. If one of the solenoid has 300 turns and the
to a source of voltage 2 V. The current reaches half of its other 400 turns, their mutual inductance is [2008]
steady state value in [2005]
(a) 0.1 s (b) 0.05 s (c) 0.3 s (d) 0.15 s (m0 = 4p × 10 –7 Tm A–1)
(a) 2.4p × 10–5 H (b) 4.8p × 10–4 H
ML2
20. Which of the following units denotes the dimension , (c) 4.8p × 10–5 H (d) 2.4p × 10–4 H
Q2 28. An inductor of inductance L = 400 E
where Q denotes the electric charge? [2006] mH and resistors of resistance R1 L
(a) Wb/m2 (b) Henry (H) = 2W and R2 = 2W are connected R1
(c) H/m2 (d) Weber (Wb)
to a battery of emf 12 V as shown
21. In a series resonant LCR circuit, the voltage across R is 100
volts and R = 1 kW with C = 2mF. The resonant frequency w in the figure. The internal R2
is 200 rad/s. At resonance the voltage across L is [2006] resistance of the battery is
negligible. The switch S is closed S
(a) 2.5 × 10–2 V (b) 40 V
(c) 250 V (d) 4 × 10–3 V at t = 0. The potential drop across
22. In an AC generator, a coil with N turns, all of the same area L as a function of time is : [2009]
A and total resistance R, rotates with frequency w in a
magnetic field B. The maximum value of emf generated in
the coil is [2006]
(a)
12 -3t
t
e V (b) ( )
6 1 - e -t / 0.2 V
(a) N.A.B.R.w (b) N.A.B
(c) 12e–5t V (d) 6e–5t V
(c) N.A.B.R. (d) N.A.B.w
Electromagnetic Induction and Alternating Current P-155

29. A rectangular loop has a sliding connector PQ of length l 33. A fully charged capacitor C with initial charge q0 is
and resistance R W and it is moving with a speed v as connected to a coil of self inductance L at t = 0. The time at
shown. The set-up is placed in a uniform magnetic field which the energy is stored equally between the electric and
the magnetic fields is: [2011]
going into the plane of the paper. The three currents I1, I2
p
and I are [2010] (a) LC (b) 2p LC
4
l
P (c) LC (d) p LC
34. A resistor ‘R’ and 2µF capacitor in series is connected
through a switch to 200 V direct supply. Across the capacitor
RW RW v RW is a neon bulb that lights up at 120 V. Calculate the value of
R to make the bulb light up 5 s after the switch has been
I closed. (log10 2.5 = 0.4) [2011]
(a) 1.7 × 105 W (b) 2.7 × 106 W
I2 (c) 3.3 × 107 W (d) 1.3 × 104 W
I1 Q
35. A coil is suspended in a uniform magnetic field, with the
Blv 2 Blv
(a) I1 = - I 2 = , I= plane of the coil parallel to the magnetic lines of force. When
6R 6R a current is passed through the coil it starts oscillating; It is
Blv 2 Blv very difficult to stop. But if an aluminium plate is placed
(b) I1 = I 2 = ,I =
near to the coil, it stops. This is due to : [2012]
3R 3R
Blv (a) developement of air current when the plate is placed
(c) I1 = I 2 = I = (b) induction of electrical charge on the plate
R (c) shielding of magnetic lines of force as aluminium is a
Bl n Bl n paramagnetic material.
(d) I1 = I 2 = , I=
6R 3R (d) electromagnetic induction in the aluminium plate giving
30. In the circuit shown below, the key K is closed at t = 0. The rise to electromagnetic damping.
current through the battery is [2010] 36. A metallic rod of length ‘l’ is tied to a string of length 2l and
V K made to rotate with angular speed w on a horizontal table
with one end of the string fixed. If there is a vertical magnetic
field ‘B’ in the region, the e.m.f. induced across the ends of
L R1 the rod is [JEE Main 2013]
2 Bwl2
(a)
R2 2
3Bwl 2
(b)
VR1R2 2
V 4 Bwl2
(a) at t = 0 and R at t = ¥ (c)
R12 + R22 2 2
V V ( R1 + R2 ) 5Bwl2
(d)
(b) R2 at t = 0 and R1 R2 at t = ¥ 2
37. A circular loop of radius 0.3 cm lies parallel to amuch bigger
circular loop of radius 20 cm. The centre of the small loop is
V VR1R2
on the axis of the bigger loop. The distance between their
(c) R2 at t = 0 and at t = ¥
R12 + R22 centres is 15 cm. If a current of 2.0 A flows through the
smaller loop, then the flux linked with bigger loop is
V ( R1 + R2 ) V [JEE Main 2013]
(d) R1 R2 at t = 0 and R2 at t = ¥ (a) 9.1 × 10–11 weber (b) 6 × 10–11 weber
31. In a series LCR circuit R = 200W and the voltage and the (c) 3.3 × 10–11 weber (d) 6.6 × 10–9 weber
frequency of the main supply is 220V and 50 Hz respectively. 38. In an LCR circuit as shown below both switches are open
On taking out the capacitance from the circuit the current initially. Now switch S1 is closed, S2 kept open. (q is charge
lags behind the voltage by 30°. On taking out the inductor on the capacitor and t = RC is Capacitive time constant).
from the circuit the current leads the voltage by 30°. The Which of the following statement is correct ?
power dissipated in the LCR circuit is [2010] [JEE Main 2013]
(a) 305 W (b) 210 W V
(c) Zero W (d) 242 W
32. A boat is moving due east in a region where the earth's R
magnetic field is 5.0 × 10–5 NA–1 m–1 due north and horizontal. S1
The boat carries a vertical aerial 2 m long. If the speed of the
boat is 1.50 ms–1, the magnitude of the induced emf in the
wire of aerial is: [2011] C S2
(a) 0.75 mV (b) 0.50 mV
(c) 0.15 mV (d) 1mV L
EBD_7036
P-156 Topic-wise Solved Papers - PHYSICS
(a) Work done by the battery is half of the energy 41. An LCR circuit is equivalent to a damped pendulum. In an
dissipated in the resistor LCR circuit the capacitor is charged to Q0 and then connected
(b) At t = t, q = CV/2 to the L and R as shown below :
(c) At t = 2t, q = CV (1 – e–2) L
(d) At t = 2 t, q = CV (1 – e–1) R
39. In the circuit shown here, the point ‘C’ is kept connected to
point ‘A’ till the current flowing through the circuit becomes
C
constant. Afterward, suddenly, point ‘C’ is disconnected from
point ‘A’ and connected to point ‘B’ at time t = 0. Ratio of the If a student plots graphs of the square of maximum charge
voltage across resistance and the inductor at t = L/R will be
equal to: [JEE Main 2014]
( Q2Max ) on the capacitor with time(t) for two different values
L1 and L2 (L1 > L2) of L then which of the following represents
A C R this graph correctly ? (plots are schematic and not drawn to
scale) [JEE Main 2015]

L
L1 2
2
QMax QMax Q0 (For both L1 and L2)
B (a) L2 (b)
t t

e
(a) (b) 1 L1 L2
1- e 2 2
QMax Q
(c) L2 (d) Max L1
1- e
(c) –1 (d) t t
e
40. An inductor (L = 0.03 H) and a resistor (R = 0.15 kW) are 42. Two coaxial solenoids of different radius carry current I in the
connected in series to a battery of 15V EMF in a circuit shown uur
same direction. F1 be the magnetic force on the inner solenoid
below. The key K1 has been kept closed for a long time. Then uur
at t = 0, K1 is opened and key K2 is closed simultaneously. At due to the outer one and F2 be the magnetic force on the
t = l ms, the current in the circuit will be : ( e5 @ 150 ) outer solenoid due to the inner one. Then : [JEE Main 2015]
uur uur
[JEE Main 2015] (a) F1 is radially inwards and F2 = 0
uur uur
0.03 H 0.15 kW (b) F1 is radially outwards and F2 = 0
uur uur
(c) F1 = F2 = 0
K2 uur uur
(d) F1 is radially inwards and F2 is radially outwards
43. An arc lamp requires a direct current of 10 A at 80 V to
15V K1 function. If it is connected to a 220 V (rms), 50 Hz AC supply,
the series inductor needed for it to work is close to :
(a) 6.7 mA (b) 0.67 mA (a) 0.044 H (b) 0.065 H
(c) 100 mA (d) 67 mA
(c) 80 H (d) 0.08 H
CHAPTER

16 Ray and Wave Optics


Section-A JEE Advanced/ IIT-JEE
f
A Fill in the Blanks 8. A thin rod of length is placed along the optic axis of a
3
1. A light wave of frequency 5 × 1014 Hz enters a medium of concave mirror of focal length f such that its image which is
refractive index 1.5. In the medium the velocity of the light real and elongated, just touches the rod. The magnification
wave is ......... and its wavelength is ..........(1983 - 2 Marks) is .................... (1991 - 1 Mark)
2. A convex lens A of focal length 20 cm and a concave lens B 9. A ray of light undergoes deviation of 30° when incident on
of focal length 5 cm are kept along the same axis with a
an equilateral prism of refractive index 2 . The angle made
distance d between them. If a parallel beam of light falling
on A leaves B as a parallel beam, then d is equal to ...... cm. by the ray inside the prism with the base of the prism is
(1985 - 2 Marks) .................... (1992 - 1 Mark)
3. A monochromatic beam of light of wavelength 6000 Å in 10. The resolving power of electron microscope is higher that
vacuum enters a medium of refractive index 1.5. In the that of an optical microscope because the wavelength of
medium its wavelength is ....., its frequency is ...... electrons is ..................... than the wavelength of visible light.
(1985 - 2 Marks) (1992 - 1 Mark)
4. In Young’s double-slit experiment, the two slits act as 11. If e 0 and m 0 are, respectively, the electric permittivity and
coherent sources of equal amplitude ‘A’ and of wavelength
magnetic permeability of free space, e and m the
‘ l ’. In another experiment with the same set-up the two
corresponding quantities in a medium, the index of refraction
slits are sources of equal amplitude ‘A’ and wavelength ‘ l ’, of the medium in terms of the above parameters is
but are incoherent. The ratio of the intensity of light at the ...................... (1992 - 1 Mark)
midpoint of the screen in the first case to that in the second 12. A light of wavelength 6000Å in air, enters a medium with
case is ............. (1986 - 2 Marks) refractive index 1.5 Inside the medium its frequency is .... Hz
5. A thin lens of refractive index 1.5 has a focal length of 15 cm
and its wavelength is .... Å. (1997 - 2 Marks)
in air. When the lens is placed in a medium of refractive
13. Two thin lenses, when in contact, produce a combination of
4 power +10 diopters. When they are 0.25 m apart, the power
index , its focal length will become ........cm.
3 reduces to +6 diopters. The focal length of the lenses are ....
(1987 - 2 Marks) m and ... m. (1997 - 2 Marks)
6. A point source emits sound equally in all directions in a 14. A ray of light is incident normally on one of the faces of a
non-absorbing medium. Two points P and Q are at a distance prism of apex angle 30° and refractive index 2 . The angle
of 9 meters and 25 meters respectively from the source. The
ratio of amplitudes of the waves at P and Q is ............ of deviation of the ray is... degrees. (1997 - 2 Marks)
(1989 - 2 Marks)
7. A slab of a material of refractive index 2 shown in fig. has a
B True/False
curved surface APB of radius of curvature 10 cm and a plane 1. The setting sun appears higher in the sky than it really is.
surface CD. On the left of APB is air and on the right of CD (1980)
is water with refractive indices as given in the figure. An 2. The intensity of light at a distance ‘r’ from the axis of a long
object O is placed at a distance of 15 cm from the pole P as
cylindrical source is inversely proportional to ‘r’.
shown. The distance of the final image of O from P, as viewed
(1981- 2 Marks)
from the left is ............... (1991 - 2 Marks)
3. A convex lens of focal length 1 meter and a concave lens of
A C focal length 0.25 meter are kept 0.75 meter apart. A parallel
n1 = 1.0 beam of light first passes through the convex lens, then
n 2 = 2.0 4
n3 = through the concave lens and comes to a focus 0.5 m away
3
P C O from the concave lens. (1983 - 2 Marks)
4. A beam of white light passing through a hollow prism give
15 cm no spectrum. (1983 - 2 Marks)
5. The two slits in a Young’s double slit experiment are
illuminated by two different sodium lamps emitting light of
B D the same wavelength. No interference pattern will be
20 cm
observed on the screen. (1984- 2 Marks)
EBD_7036
P-158 Topic-wise Solved Papers - PHYSICS
6. In a Young’s double slit experiment performed with a source (c) using a circular annular mark over the lens
of white light, only black and white fringes are observed. (d) increasing the size of the lens.
(1987 - 2 Marks)
7. A parallel beam of white light fall on a combination of a 7. A beam of light of wave length 600 nm from a distance source
concave and a convex lens, both of the same meterial. Their falls on a single slit 1 mm wide and a resulting diffraction
focal lengths are 15 cm and 30 cm respectively for the mean pattern is observed on a screen 2m away. The distance
wavelength in white light. On the other side of the lens between the first dark fringes on either side of central bright
system, one sees coloured patterns with violet colour at the fringe is (1994 - 1 Mark)
outer edge. (1988 - 2 Marks) (a) 1.2 cm (b) 1.2 mm
(c) 2.4 cm (d) 2.4 mm
C MCQs with One Correct Answer 8. An isosceles prism of angle 120° has a refractive index 1.44.
1. When a ray of light enters a glass slab from air, Two parallel monochromatic rays enter the prism parallel to
(a) its wavelength decreases. (1980) each other in air as shown. The rays emerge from the opposite
(b) its wavelength increases. faces (1995S)
(c) Its frequency decreases.
(d) neither its wavelength nor its frequency changes.
2. A glass prism of refractive index 1.5 is immersed in water
(refractive index 4/3). A light beam incident normally on the
face AB is totally reflected to reach on the face BC if 120°
(1981- 2 Marks)
8
(a) sin q ³ A B
9
q
2 8 (a) are parallel to each other
(b) < sin q <
3 9 (b) are diverging
2 (c) make an angle 2 [sin–1 (0.72) – 30°] with each other
(c) sin q £ C
3 (d) make an angle 2 sin –1 (0.72) with each other
(d) None of these 9. A diminished image of an object is to be obtained on a
screen 1.0 m from it. This can be achieved by appropriately
3. In Young’s double-slit experiment, the separation between
placing (1995S)
the slits is halved and the distance between the slits and the
screen is doubled. The fringe width is (1981- 2 Marks) (a) a concave mirror of suitable focal length
(a) unchanged. (b) halved. (b) a convex mirror of suitable focal length
(c) doubled (d) quadrupled (c) a convex lens of focal length less than 0.25 m
4. A ray of light from a denser medium strike a rarer medium at
an angle of incidence i (see Fig). The reflected and refracted (d) a concave lens of suitable focal length
rays make an angle of 90° with each other. The angles of 10. The focal lengths of the objective and the eye piece of a
reflection and refraction are r and r’ The critical angle is compound microscope are 2.0 cm and 3.0 cm, respectively.
(1983 - 1 Mark) The distance between the objective and the eye piece is
15.0 cm. The final image formed by the eye piece is at infinity.
The two lenses are thin. The distance in cm of the object
i r and the image produced by the objective, measured from
the objective lens, are respectively (1995S)
(a) 2.4 and 12.0 (b) 2.4 and 15.0
r' (c) 2.0 and 12.0 (d) 2.0 and 3.0
11. Consider Fraunhoffer diffraction pattern obtained with a
–1 –1 single slit illuminated at normal incidence. At the angular
(a) sin (tan r ) (b) sin (tan i ) position of the first diffraction minimum the phase difference
(in radians) between the wavelets from the opposite edges
(c) sin –1 (tan r' ) (d) tan –1 (sin i ) of the slit is (1995S)
5. Two coherent monochromatic light beams of intensities I (a) p / 4 (b) p / 2 (c) 2p (d) p
and 4 I are superposed. The maximum and minimum possible 12. In an interference arrangement similar to Young’s double-
intensities in the resulting beam are (1988 - 1 Mark) slit experiment, the slits S1 and S2 are illuminated with
(a) 5I and I (b) 5I and 3I coherent microwave sources, each of frequency 106 Hz. The
(c) 9I and I (d) 9I and 3I sources are synchronized to have zero phase difference.
6. Spherical aberration in a thin lens can be reduced by The slits are separated by a distance d = 150.0 m. The
intensity I (q) is measured as a function of q, where q is
(a) using a monochromatic light (1994 - 1 Mark)
defined as shown. If I0 is the maximum intensity, then I (q)
(b) using a doublet combination
Ray and Wave Optics P-159

for 0 £ q £ 90 ° is given by (1995S) S


(a) I (q) = I0 / 2 for q = 30° S1
i a i
d/2
(b) I (q) = I0 / 4 for q = 90°
m t (2000S)
(c) I (q) = I0 for q = 0° d/2
S2 (a) zero (b) a
(d) I (q) is constant for all
æ 1ö æ 1ö
values of q. (c) sin -1 ç ÷ (d) 2sin -1 ç ÷
è nø è nø
13. A concave lens of glass, refractive index 1.5 has both
surfaces of same radius of curvature R. On immersion in a 19. A rectangular glass slab ABCD of refractive index n1 is
medium of refractive index 1.75, it will behave as a immersed in water of refractive index n2(n1 > n2). A ray of
(1999S - 2 Marks) light is incident at the surface AB of the slab as shown. The
maximum value of the angle of incidence amax such that the
(a) convergent lens of focal length 3.5 R
ray comes out only from the other surface CD is given by
(b) convergent lens of focal length 3.0 R
(c) divergent lens of focal length 3.5 R A D
(d) divergent lens of focal length 3.0 R
n1 n2
14. Yellow light is used in a single slit diffraction experiment amax
(2000S)
with slit width of 0.6 mm. If yellow light is replaced by B C
X– rays, then the observed pattern will reveal,
(a) that the central maximum is narrower (1999S - 2 Marks) én æ æ n ööù
(b) more number of fringes (a) sin-1 ê 1 cos ç sin-1 ç 2 ÷ ÷ ú
(c) less number of fringes ëê n2 è è n1 ø ø ûú
(d) no diffraction pattern
15. A thin slice is cut out of a glass cylinder along a plane é æ æ 1 öö ù
(b) sin-1 ên1 cos ç sin-1 ç ÷ ÷ ú
parallel to its axis. The slice is placed on a flat glass plate as
ëê è è n2 ø ø ûú
shown in Figure.
The observed interference fringes from this combination æn ö æn ö
shall be (1999S - 2 Marks) (c) sin -1 ç 1 ÷ (d) sin -1 ç 2 ÷
è n2 ø è n1 ø
(a) straight
(b) circular 20. In a double slit experiment instead of taking slits of equal
(c) equally spaced widths, one slit is made twice as wide as the other. Then, in
(d) having fringe spacing which increases as we go the interference pattern (2000S)
(a) the intensities of both the maxima and the minima
outwards
increase
16. A hollow double concave lens is made of very thin
(b) the intensity of the maxima increases and the minima
transparent material. It can be filled with air or either of two has zero intensity
liquids L 1 or L 2 having refractive indices m1 and m 2 (c) the intensity of the maxima decreases and that of the
respectively (m2 > m1 > 1). The lens will diverge a parallel minima increases
beam of light if it is filled with (2000S) (d) the intensity of the maxima decreases and the minima
(a) air and placed in air (b) air and immersed in L1 has zero intensity
(c) L1 and immersed in L2 (d) L2 and immersed in L1 21. In a compound microscope, the intermediate image is
17. A point source of light B is placed at a (a) virtual, erect and magnified (2000S)
B
distance L in front of the centre of a (b) real, erect and magnified
d
mirror of width 'd' hung vertically on a (c) real, inverted and magnified
wall. A man walks in front of the mirror (d) virtual, erect and reduced
along a line parallel to the mirror at a L 22. Two beams of light having intensities I and 4I interfere to
distance 2L from it as shown in fig. The 2L produce a fringe pattern on a screen. The phase difference
greatest distance over which he can see between the beams is p/2 at point A and p at point B. Then
the image of the light source in the mirror the difference between the resultant intensities at A and B is
is (2000S) (a) 2I (b) 4I (2001S)
(a) d/2 (b) d (c) 2d (d) 3d (c) 5I (d) 7I
18. A diverging beam of light from a point source S having 23. In a Young’s double slit experiment, 12 fringes are observed
to be formed in a certain segment of the screen when light of
divergence angle a , falls symmetrically on a glass slab as
wavelength 600 nm is used. If the wavelength of light is
shown. The angles of incidence of the two extreme rays are
changed to 400 nm, number of fringes observed in the same
equal. If the thickness of the glass slab is t and the refractive
segment of the screen is given by (2001S)
index n, then the divergence angle of the emergent beam is
(a) 12 (b) 18 (c) 24 (d) 30
EBD_7036
P-160 Topic-wise Solved Papers - PHYSICS
24. A ray of light passes through four transparent media with intensity at the position where the central maximum occurred
refractive indices m1, m2, m3 and m4 as shown in the figure. previously remains unchanged. The minimum thickness of
The surfaces of all media are parallel. If the emergent ray CD the glass-plate is (2002S)
is parallel to the incident ray AB, we must have (2001S) (a) 2 l (b) 2 l /3 (c) l /3 (d) l
D 29. Two plane mirrors A and B are aligned parallel to each other,
m1 m2 m3 m4 as shown in the figure. A light ray is incident at an angle 30°
C at a point just inside one end of A. The plane of incidence
B coincides with the plane of the figure. The maximum number
of times the ray undergoes reflections (including the first
one) before it emerges out is (2002S)
B
A 2 3m
(a) m1 = m2 (b) m2 = m3
(c) m3 = m4 (d) m4 = m1
25. A given ray of light suffers minimum deviation in an 0.2m
equilateral prism P. Additional prism Q and R of identical 30°

shape and of the same material as P are now added as shown


in the figure. The ray will now suffer (2001S)
A
(a) 28 (b) 30 (c) 32 (d) 34
Q
30. In the adjacent diagram, CP represents a wavefront and AO
& BP, the corresponding two rays. Find the condition on
P R
q for constructive interference at P between the ray BP and
reflected ray OP. (2003S)
(a) greater deviation (b) no deviation
Q O R
(c) same deviation as before(d) total internal reflection
26. An observer can see through a pin-hole the top end of a
thin rod of height h, placed as shown in the figure. The
beaker height is 3h and its radius h. When the beaker is C
d
filled with a liquid up to a height 2h, he can see the lower
end of the rod. Then the refractive index of the liquid is
(2002S)
A
P

B
3h
(a) cos q =3 l /2d (b) cos q = l /4d
h (c) sec q - cos q = l /d (d) sec q - cos q =4 l /d
2h 31. The size of the image of an object, which is at infinity, as
formed by a convex lens of focal length 30 cm is 2 cm. If a
5 5 3 3 concave lens of focal length 20 cm is placed between the
(a) (b) (c) (d) convex lens and the image at a distance of 26 cm from the
2 2 2 2 convex lens, calculate the new size of the image. (2003S)
27. Which one of the following spherical lenses does not exhibit (a) d/2 (b) d (c) 2d (d) 3d
dispersion? The radii of curvature of the surfaces of the 32. A ray of light is incident at the glass-water interface at an
lenses are as given in the diagrams. (2002S) angle i, it emerges finally parallel to the surface of water,
then the value of m g would be (2003S)
Air
(a) R1 R2 (b) R

r
Water
r

(c) R R (d) R Glass i


(a) (4/3)sini (b) 1/sini
28. In the ideal double-slit experiment, when a glass-plate
(refractive index 1.5) of thickness t is introduced in the path (c) 4/3 (d) 1
of one of the interfering beams (wave-lenght l ), the
Ray and Wave Optics P-161

33. A beam of white light is incident on glass air interface from silvered. The image will form at (2006 - 3M, –1)
glass to air such that green light just suffers total internal
reflection. The colors of the light which will come out to air
are (2004S)
(a) Violet, Indigo, Blue (b) All colors except green
(c) Yellow, Orange, Red (d) White light
34. An equilateral prism is placed on a horizontal surface. A ray
PQ is incident onto it. For minimum deviation (2004S)
A 20 cm
R (a) 60 cm to the left of lens (b) 12 cm to the left of lens
Q S
P (c) 60 cm to the right of lens(d) 30 cm to the left of lens
42. The graph shows relationship between 31
(a) PQ is horizontal (b) QR is horizontal object distance and image distance for a v cm
(c) RS is horizontal equiconvex lens. Then, focal length of the
30
(d) Any one will be horizontal
35. Monochromatic light of wavelength 400 nm and 560 nm are lens is (2006 - 3M, –1)
incident simultaneously and normally on double slits (a) 0.50 ± 0.05 cm 10
apparatus whose slits separation is 0.1 mm and screen (b) 0.50 ± 0.10 cm
distance is 1m. Distance between areas of total darkness (c) 5.00 ± 0.05 cm
will be (2004S) 0 (–9, +9)
(a) 4 mm (b) 5.6 mm (c) 14mm (d) 28mm (d) 5.00 ± 0.10 cm u cm –31 –30 –20 –10

36. A source emits sound of frequency 600 Hz inside water. 43. Rays of light from Sun falls on a biconvex lens of focal
The frequency heard in air will be equal to (velocity of length f and the circular image of Sun of radius r is formed
sound in water = 1500 m/s, velocity of sound in air = 300 m/s) on the focal plane of the lens. Then (2007)
(a) 3000 Hz (b) 120 Hz (2004S) (a) Area of image is pr2 and area is directly proportional of f
(c) 600 Hz (d) 6000 Hz (b) Area of image is pr2 and area is directly proportional to f 2
37. A point object is placed at the centre of a glass sphere of
(c) Intensity of image increases if f is increased
radius 6 cm and refractive index 1.5. The distance of virtual
image from the surface is (2004S) (d) If lower half of the lens is covered with black paper
(a) 6 cm (b) 4 cm (c) 12 cm (d) 9 cm area will become half
38. In Young’s double slit experiment intensity at a point is (1/4) 44. In an experiment to determine the focal length
of the maximum intensity. Angular position of this point is (f ) of a concave mirror by the u - v method, a student places
(a) sin –1(l/d) (b) sin –1(l/2d) (2005S) the object pin A on the principal axis at a distance x from the
–1
(c) sin (l/3d) (d) sin –1(l/4d) pole P. The student looks at the pin and its inverted image
39. A convex lens is in contact with concave lens. The magnitude from a distance keeping his/her eye in line with PA. When
of the ratio of their focal length is 2/3. Their equivalent focal the student shifts his/her eye towards left, the image appears
length is 30 cm. What are their individual focal lengths?
(a) –15, 10 (b) –10, 15 (2005S) to the right of the object pin. Then, (2007)
(c) 75, 50 (d) –75, 50 (a) x < f (b) f < x < 2f
40. A container is filled with water (m = 1.33) upto a height of (c) x = 2f (d) x > 2f
33.25 cm. A concave mirror is placed 15 cm above the water 45. A ray of light traveling in water is incident on its surface
level and the image of an object placed at the bottom is open to air. The angle of incidence is q, which is less than
formed 25 cm below the water level. Focal length of the the critical angle. Then there will be (2007)
mirror is (2005S) (a) only a reflected ray and no refracted ray
(a) 15 cm (b) only a refracted ray and no reflected ray
(b) 20 cm
(c) a reflected ray and a refracted ray and the angle between
(c) –18.31 cm
15 cm them would be less than 180° – 2q
(d) 10 cm
(d) a reflected ray and a refracted ray and the angle between
them would be greater than 180° – 2q
46. Two beams of red and violet colours are made to pass
m=1.33 25 cm separately through a prism (angle of the prism is 60°). In the
33.25 cm position of minimum deviation, the angle of refraction will
be (2008)
I (a) 30° for both the colours
O
(b) greater for the violet colour
(c) greater for the red colour
(d) equal but not 30° for both the colours
41. Focal length of the plano-convex lens is 15 cm. A small object
is placed at A as shown in the figure. The plane surface is
EBD_7036
P-162 Topic-wise Solved Papers - PHYSICS
47. A light beam is travelling from Region I to IV (figure). The 51. A bi-convex lens is formed with two thin plano-convex lenses
as shown in the figure. Refractive index n of the first lens is
n 0 n0 1.5 and that of the second lens is 1.2. Both the curved surface
refractive index in regionals I, II, III and IV are n 0 , ,
2 6 are of the same radius of curvature R = 14 cm. For this bi-
convex lens, for an object distance of 40 cm, the image
n0 distance will be (2012)
and respectively. The angle of incidence q for which
8
the beam just misses entering region IV is – (2008)

Region I Region II Region III Region IV

n0 n0 n0 n0
2 6 8
(a) -280.0 cm (b) 40.0 cm
0 0.2m 0.6m (c) 21.5 cm (d) 13.3 cm
52. Young’s double slit experiment is carried out by using green,
(a) sin–1 (3/4) (b) sin–1 (1/8) red and blue light, one color at a time. The fringe widths
(c) sin–1 (1/4) (d) sin–1 (1/3) recorded are bG, bR and bB, respectively. Then,
48. A ball is dropped from a height of 20 m above the surface of (a) bG > bB > bR (b) bB > bG > bR (2012)
water in a lake. The refractive index of water is 4.3. A fish (c) bR > bB > bG (d) bR > bG > bB
inside the lake, in the line of fall of the ball, is looking at the 1 ˆ
53. A ray of light travelling in the direction (i + 3 ˆj ) is incident
ball. At an instant, when the ball is 12.8 m above the water 2
surface, the fish sees the speed of ball as [Take g = 10 m/s2.] on a plane mirror. After reflection, it travels along the direction
(a) 9 m/s (b) 12 m/s (2009)
1 ˆ
(c) 16 m/s (d) 21.33 m/s (i - 3 ˆj ) . The angle of incidence is (JEE Adv. 2013)
2
49. A biconvex lens of focal length 15 cm is in front of a plane
(a) 30° (c) 60°
mirror. The distance between the lens and the mirror is 10
(b) 45° (d) 75°
cm. A small object is kept at a distance of 30 cm from the
54. In the Young’s double slit experiment using a monochromatic
lens. The final image is (2010) light of wavelength l, the path difference (in terms of an
(a) virtual and at a distance of 16 cm from the mirror integer n) corresponding to any point having half the peak
(b) real and at a distance of 16 cm from the mirror intensity is (JEE Adv. 2013)
(c) virtual and at a distance of 20 cm from the mirror
l l
(d) real and at a distance of 20 cm from the mirror (a) (2n + 1) (b) (2n + 1)
50. A light ray travelling in glass medium is incident on glass- 2 4
air interface at an angle of incidence q. The reflected (R) and l
l
transmitted (T) intensities, both as function of q, are plotted. (c) (2 n + 1) (d) (2 n + 1)
8 16
The correct sketch is (2011)
55. A point source S is placed at the bottom of a transparent
block of height 10 mm and refractive index 2.72. It is immersed
(a) (b) in a lower refractive index liquid as shown in the figure. It is
found that the light emerging from the block to the liquid
Intensity

forms a circular bright spot of diameter 11.54 mm on the top


Intensity

of the block. The refractive index of the liquid is


(JEE Adv. 2014)

Liquid

(c) (d)
Block
Intesitya
Intensity

S
(a) 1. 21 (b) 1. 30
(c) 1. 36 (d) 1. 42
Ray and Wave Optics P-163

56. A parallel beam of light is incident from air at an angle a on (a) – 1.5 dioptres (b) – 6.5 dioptres
the side PQ of a right angled triangular prism of refractive (c) + 6.5 dioptres (d) + 6.67 dioptres
index n = 2 . Light undergoes total internal reflection in 3. White light is used to illuminate the two slits in a Young’s
double slit experiment. The separation between the slits is b
the prism at the face PR when a has a minimum value of 45°.
and the screen is at a distance d (> b) from the slits. At a
The angle q of the prism is (JEE Adv. 2016)
point on the screen directly in front of one of the slits, certain
P wavelengths are missing. Some of these missing
wavelengths are (1984- 2 Marks)
q b2 2b 2
(a) l= (b) l=
a d d
b2 2b 2
(c) l= (d) l =
3d 3d
n= 2 4. A converging lens is used to form an image on a screen.
Q R When the upper half of the lens is covered by an opaque
screen (1986 - 2 Marks)
(a) 15° (b) 22.5° (a) half the image will disappear.
(c) 30° (d) 45° (b) complete image will be formed.
57. A small object is placed 50 cm to the left of a thin convex (c) intensity of the image will increase.
lens of focal length 30 cm. A convex spherical mirror of radius (d) intensity of the image will decrease.
of curvature 100 cm is placed to the right of the lens at a 5. A short linear object of length b lies along the axis of a
distance of 50 cm.The mirror is tilted such that the axis of concave mirror of focal length f at a distanee u from the pole
the mirror is at an angle q =30° to the axis of the lens, as of the mirror. The size of the image is approximately equal to
shown in the figure. (1988 - 2 Mark)
1/ 2 1/ 2
f = 30 cm æu - f ö æ f ö
(a) bç (b) bç
è f ÷ø è u - f ø÷
q x
(0, 0) 2
(–50,0) R æu- f ö æ f ö
= bç bç
è f ÷ø
(c) (d)
10
0 è u - f ø÷
50 cm cm
6. A beam of light consisting of red, green and blue colours is
incident on a right angled prism, fig. The refractive indices
of the material of the prism for the above red, green and blue
(50 + 50 3, -50) wavelengths are 1.39, 1.44 and 1.47 respectively. The prism
will (1989 - 2 Mark)
If the origin of the coordinate system is taken to be at the
centre of the lens, the coordinates (in cm) of the point (x, y)
at which the image is formed are (JEE Adv. 2016)
(a) (0, 0) (b) (50 - 25 3, 25)
(c) (25, 25 3) (d) (125 / 3,25 3) 45°
(a) separate part of the red colour from the green and blue
colours
D MCQs with One or More than One Correct
(b) separate part of the blue colour from the red and green
1. In the Young’ s double slit experiment, the interference colours
pattern is found to have an intensity ratio between the bright (c) separate all the three colours from one another
and dark fringes as 9. This implies that (1982 - 3 Marks) (d) not separate even partially any colour from the other
(a) the intensities at the screen due to the two slits are 5 two colours.
units and 4 units respectively 7. An astronomical telescope has an angular magnification of
(b) the intensities at the screen due to the two slits are 4 magnitude 5 for distant objects. The separation between the
units and 1 units respectively objective and the eyepiece is 36 cm and the final image is
(c) the amplitude ratio is 3 formed at infinity. Thefocal length f0 of the objective and the
(d) the amplitude ratio is 2 focal length f0 of the eyepiece are (1989 - 2 Marks)
2. A convex lens of focal length 40 cm is in contact with a (a) f0 = 45 cm and fe = –9 cm (b) f0 = 50 cm and fe = 10 cm
concave lens of focal length 25 cm . The power of the (c) f0 = 7.2 cm and fe = 5 cm (d) f0 = 30 cm and fe = 6 cm.
combination is (1982 - 3 Marks)
EBD_7036
P-164 Topic-wise Solved Papers - PHYSICS
8. A thin prism P1 with angle 4° and made from glass of 14. A parallel monochromatic beam of light is incident normally
refractive index 1.54 is combined with another thin prism P2 on a narrow slit. A diffraction pattern is formed on a screen
made from glass of refractive index 1.72 to produce placed perpendicular to the direction of the incident beam.
dispersion without deviation. The angle of the prism P2 is At the first minimum of the diffraction pattern, the phase
difference between the rays coming from the two edges of
(1990 - 2 Marks)
the slit is (1998 - 2 Marks)
(a) 5.33° (b) 4° (c) 3° (d) 2.6° (a) 0 (b) p/2 (c) p (d) 2p
9. A planet is observed by an astronomical refracting telescope 15. A concave mirror is placed on a horizontal table, with its axis
having an objective of focal length 16 m and an eyepiece of directed vertically upwards. Let O be the pole of the mirror
focal length 2 cm. (1992 - 2 Marks) and C its centre of curvature. A point object is placed at C.
It has a real image, also located at C. If the mirror is now
(a) The distance between the objective and the eyepiece
filled with water, the image will be. (1998 - 2 Marks)
is 16.02 m
(a) real, and will remain at C.
(b) The angular magnification of the planet is – 800 (b) real, and located at a point between C and ¥ .
(c) The image of the planet is inverted (c) virtual, and located at a point between C and O.
(d) The objective is larger then the eyepiece (d) real, and located at a point between C and O
10. Two thin convex lenses of focal lengths f1 and f2 are separated 16. A spherical surface of radius of curvature R separates air
by a horizontal distance d (where d <f1, d< f2) and their centres (refractive index 1.0) from glass (refractive index 1.5). The
are displaced by a vertical separation D as shown in the fig. centre of curvature is in the glass. A point object P placed in
(1993-2 Marks) air is found to have a real image Q in the glass. The line PQ
y
cuts the surface at a point O, and PO = OQ. The distance PO
is equal to (1998 - 2 Marks)
D
o x (a) 5R (b) 3R (c) 2R (d) 1.5R
17. In a Young’s double slit experiment, the separation between
d the two slits is d and the wavelength of the light is l. The
intensity of light falling on slit 1 is four times the intensity of
Taking the origin of coordinates O, at the centre of the first
light falling on slit 2. Choose the correct choice(s). (2008)
lens the x and y coordinates of the focal point of this lens
system, for a parallel beam of rays coming from the left, are (a) If d = l, the screen will contain only one maximum
given by: (b) If l < d < 2l, at least one more maximum (besides the
central maximum) will be observed on the screen
f1 f 2
(a) x= ,y= D (c) If the intensity of light falling on slit 1 is reduced so
f1 + f 2 that it becomes equal to that of slit 2, the intensities of
f ( f + d) D the observed dark and bright fringes will increase
(b) x= 1 2 ,y= (d) If the intensity of light falling on slit 2 is increased so
f1 + f2 - d f1 + f2
that it becomes equal to that of slit 1, the intensities of
f f + d ( f1 - d ) D( f1 - d ) the observed dark and bright fringes will increase
(c) x= 1 2 ,y=
f1 + f 2 - d f1 + f2 - d 18. A student performed the experiment of determination of focal
length of a concave mirror by u-v method using an optical
f1 f2 + d ( f1 - d )
(d) x= ,y = 0 bench of length 1.5 meter. The focal length of the mirror
f1 + f 2 - d
used is 24 cm. The maximum error in the location of the
11. Which of the following form(s) a virtual and erect image for image can be 0.2 cm. The 5 sets of (u, v ) values recorded by
all positions of the object ? (1996 - 2 Marks) the student (in cm) are :
(a) Convex lens (b) Concave lens (42, 56), (48, 48), (60, 40), (66, 33), (78, 39). The data set(s)
(c) Convex mirror (d) Concave mirror. that cannot come from experiment and is (are) incorrectly
12. A real image of a distant object is formed by a plano-convex recorded, is (are) (2009)
lens on its principal axis. Spherical aberration (a) (42, 56) (b) (48, 48) (c) (66, 33) (d) (78, 39)
(1998 - 2 Marks) 19. A ray OP of monochromatic light is incident on the face AB
(a) is absent. of prism ABCD near vertex B at an incident angle of 60° (see
(b) is smaller if the curved surface of the lens faces the figure). If the refractive index of the material of the prism is
object.
3 , which of the following is (are) correct? (2010)
(c) is smaller if the plane surface of the lens faces the object.
(d) is the same whichever side of the lens faces the object
13. A ray of light travelling in a transparent medium falls on a
surface separating the medium from air at an angle of incidence
of 45°. The ray undergoes total internal reflection. If n is the
refractive index of the medium with respect to air, select the
possible value(s) of n from the following : (1998 - 2 Marks)
(a) 1.3 (b) 1.4 (c) 1.5 (d) 1.6
Ray and Wave Optics P-165

(a) The ray gets totally internally reflected at face CD (a) The refractive index of the lens is 2.5
(b) The ray comes out through face AD (b) The radius of curvature of the convex surface is 45 cm
(c) The angle between the incident ray and the emergent (c) The faint image is erect and real
ray is 90° (d) The focal length of the lens is 20 cm
(d) The angle between the incident ray and the emergent 24. A transparent slab of thickness d has a refractive index n(z)
ray is 120° that increases with z. Here z is the vertical distance inside
20. A transparent thin film of uniform thickness and refractive the slab, measured from the top. The slab is placed between
index n1 = 1.4 is coated on the convex spherical surface of two media with uniform refractive indices n1 and n 2 (> n1),
radius R at one end of a long solid glass cylinder of refractive as shown in the figure. A ray of light is incident with angle
index n2 = 1.5, as shown in the figure. Rays of light parallel to
qi, from medium 1 and emerges in medium 2 with refraction
the axis of the cylinder traversing through the film from air to
angle qt with a lateral displacement l. (JEE Adv. 2016)
glass get focused at distance f1 from the film, while rays of
light traversing from glass to air get focused at distance f2
from the film, Then (JEE Adv. 2014)
n1

Air n2

(a) f1 = 3R (b) f1 = 2.8R


(c) f2 = 2 R (d) f2 = 1.4R
21. A light source, which emits two wavelength l1 = 400 nm and Which of the following statement(s) is(are) true?
l2 = 600 nm, is used in a Young’s double slit experiment. If (a) nl sinqi = n2 sinqf
recorded fringe widths for l1 and l2 are b1 and b2 and the (b) n1 sin qi = (n 2 – n 1) sinqf
number of fringes for them within a distance y on one side of (c) l is independent of n2
the central maximum are m1 and m2 respectively, then (d) l is dependent on n(z)
(a) b2 > b1 (JEE Adv. 2014) 25. While conducting the Young’s double slit experiment, a
(b) m1 > m2 student replaced the two slits with a large opaque plate in
(c) Form the central maximum, 3rd maximum of l2 overlaps the x-y plane containing two small holes that act as two
with 5th minimum of l1 coherent point sources (S1, S2) emitting light of wavelength
(d) The angular separation of fringes for l1 is greater 600 nm. The student mistakenly placed the screen parallel
than l2. to the x-z plane (for z > 0) at a distance D = 3 m from the mid-
point of S1S2, as shown schematically in the figure. The
22. Two identical glass rods S1 and S2 (refractive index = 1.5)
distance between the sources d = 0.6003 mm. The origin O is
have one convex end of radius of curvature 10 cm. They are
at the intersection of the screen and the line joining S1S2.
placed with the curved surfaces at a distance d as shown in Which of the following is(are) true of the intensity pattern
the figure, with their axes (shown by the dashed line) aligned. on the screen? (JEE Adv. 2016)
When a point source of light P is placed inside rod S1 on its
axis at a distance of 50 cm from the curved face, the light rays
emanating from it are found to be parallel to the axis inside
S2. The distance d is (JEE Adv. 2015)
Screen

z
S1 S2
P
O
50 cm d y
S1 S2
d x
(a) 60 cm (b) 70 cm (c) 80 cm (d) 90 cm D
23. A plano–convex lens is made of a material of refractive index
n. When a small object is placed 30 cm away in front of the (a) Straight bright and dark bands parallel to the x-axis
curved surface of the lens, an image of double the size of (b) The region very close to the point O will be dark
the object is produced. Due to reflection from the convex (c) Hyperbolic bright and dark bands with foci
surface of the lens, another faint image is observed at a symmetrically placed about O in the x-direction
distance of 10 cm away from the lens. Which of the following (d) Semi circular bright and dark bands centered at point.
statement(s) is(are) true? (JEE Adv. 2016)
EBD_7036
P-166 Topic-wise Solved Papers - PHYSICS

E Subjective Problems C

1. A pin is placed 10 cm in front of a convex lens of focal length


20 cm, made a material of refractive index 1.5. The surface of
60°
the lens farther away from the pin is silvered and has a × P
A 60°
radius of curvature are of 22 cm. Determine the position of
the final image. Is the image real as virtual? (1978)
2. A ray of light is incident at an angle of 60° on one face of
prism which has an angle of 30°. The ray emerging out of B S
the prism makes an angle of 30° with the incident ray. Show
that the emergent ray is perpendicular to the face through 8. A plano convex lens has a thickness of 4 cm . When placed
which it emerges and calculate the refractive index of the
on a horizontal table, with the curved surface in contact with
material of the prism. (1978)
it, the apparent depth of the bottom most point of the lens is
3. A rectangular block of glass is placed on a printed page
lying on a horizontal surface. Find the minimum value of the found to be 3 cm. If the lens is inverted such that the plane
refractive index of glass for which the letters on the page are face is in contact with the table, the apparent depth of the
not visible from any of the vertical faces of the block. (1979) centre of the plane face is found to be 25/8 cm. Find the focal
4. What is the relation between the refractive indices m1 and length of the lens. (1984- 6 Marks)
m2, if the behaviour of light rays is as shown in the figure? 9. A beam of light consisting of two wavelengths, 6500Å and
(1979) 5200Å, is used obtain interference fringes in a Young’s
double slit experiment : (1985 - 6 Marks)
m1 m m1 m2 m m2 (i) Find the distance of the third bright fringe on the screen
from the central maximum for wavelength 6500Å.
(ii) What is the least distance from the central maximum
where the bright fringes due to both the wavelengths
5. An object is placed 21 cm in front of a concave mirror of coincide?
radius of curvature 10 cm. A glass slab of thickness 3 cm The distance between the slits is 2 mm and the distance
and refractive index 1.5 is then placed close to the mirror in between the plane of the slits and the screen is
the space between the object and the mirror. 120 cm.
Find the position of the final image formed. (You may take 10. Monochromatic light is incident on a plane interface AB
the distance of the near surface of the slab from the mirror to between two media of refractive indices n1 and n2 (n2 > n1)
be 1 cm. (1980) at an angle of incidence q as shown in fig. The angle q is
6. The convex surface of a thin concavo-convex lens of glass
of refractive index 1.5 has a radius of curvature 20 cm. The infinitesimally greater than the critical angle for the two media
concave surface has a radius of curvature 60 cm. The convex so that total internal reflection takes place. Now if a
side is silvered and placed on a horizontal surface. transparent slab DEFG of uniform thickness and of refractive
(1981- 6 Marks) index n3 is introduced on the interface (as shown in the
figure), show that for any value of n3 all light will ultimately
be reflected back again into medium II. Consider separately
the cases (1986 - 6 Marks)
(i) n3 < n1 and (ii) n3 > n1 .

(i) Where should a pin be placed on the optic axis such MEDIUM I
(n1 )
that its image is formed at the same place? D E
(ii) If the concave part is filled with water of refractive MEDIUM I
index 4/3, find the distance through which the pin (n 3 ) F
G
should be moved so that the image of the pin again A B
coincides with the pin. q
MEDIUM II
7. Screen S is illuminated by two point sources A and B. Another (n 2 )
source C sends a parallel beam of light towards point P on
the screen (see figure). Line AP is normal to the screen and
the lines AP, BP and CP are in one plane. The distance AP,
BP and CP are 3 m, 1.5 m and 1.5 m respectively. The radiant 11. A right prism is to be made by selecting a proper material
powers of sources A and B are 90 watts and 180 watts
respectively. The beam from C is of intensity 20 watts/m2. and the angles A and B ( B £ A) , as shown in Figure. It is
Calculate the intensity at P on the screen.(1982 - 5 Marks) desired that a ray of light incident on the face AB emerges
parallel to the incident direction after two internal reflections.
Ray and Wave Optics P-167

b
the relation. m(l) = 1.20 + where l is in Å and b is
l2
positive constant. The value of b is such that the condition
B
for total reflection of the face AC is just satisfied for one
A
wave length and is not satisfied for the other.
A (1991 - 2 + 2 + 4 Marks)
P q sin q = 0.8
C
(i)What should be the minimum refractive index n for this
to be possible ? d

(ii) For n = 5 is it possible to achieve this with the angle Q 90°


3
B equal to 30 degrees ? (1987 - 7 Marks) C
B
12. A parallel beam of light travelling in water (refractive index (a) Find the value of b.
= 4/3) is refracted by a spherical air bubble of radius 2 mm (b) Find the deviation of the beams transmitted through
situated in water. Assuming the light rays to be paraxial the face AC
(1988 - 6 Marks)
(c) A convergent lens is used to bring these transmitted
(i) Find the position of the image due to refraction at the first
surface and the position of the final image. beams into focus. If the intensities of transmission form
(ii) Draw a ray diagram showing the positions of both the the face AC, are 41 and I respectively, find the resultant
images. intensity at the focus.
13. In a modified Young’s double slit experiment, a 16. Light is incident at an angle a on one planar end of a
monochromatic uniform and parallel beam of light of transparent cylindrical rod of refractive index m. Determine
wavelength 6000 Å and intensity (10/ p ) W m–2 is incident the least value of m so that the light entering the rod does
normally on two circular apertures A and B of radii 0.001 m not emerge from the curved surface of rod irrespective of
and 0.002 m respectively. A perfectly transparent film of the value of a (1992 - 8 Marks)
thickness 2000 Å and refractive index 1.5 for the wavelength
of 6000 Å is placed in front of aperture A, see fig. Calculate
the power (in watts) received at the focal spot F of the lens. m
The lens is symmetrically placed with respect to the
apertures. Assume that 10% of the power received by each
aperture goes in the original direction and is brought to the a
focal spot. (1989 - 8 Mark)

17. In Fig., S is a monochromatic point source emitting light of


A wavelength l = 500nm. A thin lens of circular shape and
F focal length 0.10 m is cut into two identical halves L1 and L2
by a plane passing through a diameter. The two halves are
placed symmetrically about the central axis SO with a gap of
B
0.5 mm. The distance along the axis from S to L1 and L2 is
0.15 m while that from L1 and L2 to O is 1.30 m. The screen at
Glass O is normal to SO. (1993 - 5+1 Marks)
14. A narrow monochromatic beam of light plate
of intensity I is incident on a glass plate L1
as shown in figure. Another identical I 0.5mm A
S
glass plate is kept close to the first one O
and parallel to it. Each glass plate Screen
reflects 25 per cent of the light incident L2
0.15m 1.30m
on it and transmits the remaining. Find Glass
the ratio of the minimum and the plate
(i) If the third intensity maximum occurs at the point A on
maximum intensities in the interference pattern formed by
the screen, find the distance OA.
the two beams obtained after one reflection at each plate. (ii) If the gap between L1 and L2 is reduced from its original
(1990 - 7 Mark) value of 0.5mm, will the distance OA increase, decrease,
15. Two parallel beams of light P and Q (separation d) containing or remain the same?
radiations of wavelengths 4000 Å and 5000 Å (which are 18. An image Y is formed of point object X by a lens whose
mutually coherent in each wavelength separately) are optic axis is AB as shown in figure. Draw a ray diagram to
incident normally on a prism as shown in fig. The refractive locate the lens and its focus. If the image Y of the object X
index of the prism as a function of wavelength is given by is formed by a concave mirror (Having the same axis as AB)
EBD_7036
P-168 Topic-wise Solved Papers - PHYSICS
instead of lens, draw another ray diagram to locate the mirror 22. A thin plano-convex lens of focal length f is split into two
and its focus. Write down the steps of construction of the halves: one of the halves is shifted along the optical axis .
ray diagrams. (1994 - 6 Marks) The separation between object and image planes is 1.8 m.
X The magnification of the image formed by one of the half-
lenses is 2. Find the focal-length of the lens and separation
A B between the two halves. Draw the ray diagram for image
formation. (1996 - 5 Marks)
Y
19. A ray of light travelling in air is incident at grazing angle
(incident angle @ 90° ) on a long rectangular slab of a
transparent medium of thickness t = 1.0 m (see figure below). O
The point of incidence is the origin A(0, 0). The medium has
a variable index of refraction n(y) given by
-3 / 2 1.8 m
n( y ) = [ky 3 / 2 + 1]1/ 2 , where k = 1.0 (metre)
y 23. In Young’s experiment, the upper slit is covered by a thin
P( x1 , y1 )
glass plate of refractive index 1.4 while the lower slit is
AIR
covered by another glass plate, having the same thickness
as the first one but having refractive index 1.7. Interference
t =1.0m B(x, y) pattern is observed using light of wavelength 5400 Å. It is
MEDIUM found that the point P on the screen where the central
x
A(0, 0) AIR maximum (n = 0) fells before the glass plates were inserted
The refractive index of air is 1.0. (1995 - 10 Marks) now has 3/4 the original intensity. It is further observed that
(a) Obtain a relation between the slope of the trajectory of what used to be the fifth maximum earlier, lies below the
the ray at a pint B(x, y) in the medium and the incident point P while the sixth minimum lies above P. Calculate the
angle at that point. thickness of the glass plate. (Absorption of light by glass
(b) Obtain an equation for the trajectory y(x) of the ray in plate may be neglected.) (1997 - 5 Marks)
the medium. 24. A prism of refractive index n1 and another prism of refractive
(c) Determine the coordinates ( x1 , y1 ) of the point P, index n2 are stuck together without a gap as shown in Figure.
where the ray intersects the upper surface of the slab- The angles of the prisms are as shown. n1 and n2 depend on
air boundary.
l, the wavelength of light, according to
(d) Indicate the path of the ray subsequently.
20. A right angled prism (45° –90°–45°) of refractive index n has 1.80 ´ 10 4
10.8 ´ 10 4
a plate of refractive index n1(n1 < n) cemented to its diagonal n1 = 1.20 + an d n 2 = 1.45 +
face. The assembly is in air. A ray is incident on AB. l2 l2
where l is in nm. (1998 - 8 Marks)
D
A

C
70°
n1
n n2

n1
20
°
60° 40°
B B A
B
(i) Calculate the angle of incidence at AB for which the (a) Calculate the wavelength l0 for which rays incident at
ray strikes the diagonal face at the critical angle. any angle on the interface BC pass through without
(ii) Assuming n = 1.352 calculate the angle of incidence at bending at that interface.
AB for which the refracted ray passes through the
diagonal face undeviated. (1996 - 3 Marks) (b) For light of wavelength l0, find the angle of incidence
21. A double-slit apparatus is immersed in a liquid of refractive i on the face AC such that the deviation produced by
index 1.33. It has slit separation of 1mm, and distance the combination of prisms is minimum.
between the plane of slits and screen is 1.33 m. The slits are 25. A coherent parallel beam of microwaves of wavelength l =
illuminated by a parallel beam of light whose wavelength in 0.5 mm falls on a Young’s double slit apparatus. The
air is 6300 Å. (1996 - 3 Marks) separation between the slits is 1.0 mm. The intensity of
(i) Calculate the fringe-width.
(ii) One of the slits of the apparatus is covered by a thin microwaves is measured on a screen placed parallel to the
glass sheet of refractive index 1.53. Find the smallest plane of the slits at a distance of 1.0 m from it as shown in
thickness of the sheet to bring the adjacent minimum Fig.
on the axis.
Ray and Wave Optics P-169

30° d = 1.0 mm
x
P
mR R
D = 1.0m

Screen 29. (a) A convex lens of focal length 15 cm and a cancave


(a) If the incident beam falls normally on the double slit mirror of focal length 30 cm are kept with their optic
apparatus, find the y-coordinates of all the interference axes PQ and RS parallel but separated in vertical
minima on the screen. direction by 0.6 cm as shown. The distance between
(b) If the incident beam makes an angle of 30° with the x the lens and mirror is 30 cm. An upright object AB of
axis (as in the dotted arrow shown in Figure), find the height 1.2 cm is placed on the optic axis PQ of the lens
y-coordinate of the first minima on either side of the at a distance of 20 cm from the lens. If A' B ' is the
central maximum. (1998 - 8 Marks) image after refraction from the lens and reflection from
26. The Young’s double slit experiment is done in a medium of
refractive index 4/3. A light of 600 nm wavelength is falling the mirror, find the distance of A' B ' from the pole of
on the slits having 0.45 mm separation. The lower slit S2 is the mirror and obtain its magnification. Also locate
covered by a thin glass sheet of thickness 10.4 mm and position of A ' and B ' with respect to the optic axis
refractive index 1.5. The interference pattern is observed on RS. (2000 - 6 Marks)
a screen placed 1.5 m from the slits as shown in Figure.
(1999 - 10 Marks)
Y

S1
S* O
S2

(b) A glass plate of refractive index 1.5 is coated with a


thin layer of thickness t and refractive index 1.8. Light
of wavelength l travelling in air is incident normally on
(a) Find the location of the central maximum (bright fringe
the layer. It is partly reflected at the upper and the
with zero path difference) on the y – axis.
lower surface of the layer and the two reflected rays
(b) Find the light intensity at point O relative to the
interfere. Write the condition for their constructive
maximum fringe intensity.
interference. If l = 648 nm, obtain the least value of t for
(c) Now, if 600 nm light is replaced by white light of range
400 to 700 nm, find the wavelengths of the light that which the rays interfere constructively.
form maxima exactly at point O. (2000 - 4 Marks)
[All wavelengths in this problem are for the given 30. The refractive indices of the crown glass for blue and red
medium of refractive index 4/3. Ignore dispersion] lights are 1.51 and 1.49 respectively and those of flint glass
27. The x – y plane is the boundary between two transparent are 1.77 and 1.73 respectively. An isosceles prism of angle
media. Medium –1 with z ³ 0 has a refractive index 2 6o is made of crown glass. A beam of white light is incident
at a small angle on this prism. The other flint glass isosceles
and medium –2 with z £ 0 has a refractive index 3 . A ray prism is combined with the crown glass prism such that
there is no deviation of the incident light. Determine the
of light in medium –1 given by the vector A = 6 3i + 8 3 j
angle of the flint glass prism. Calculate the net dispersion of
–10 k is incident on the plane of separation. Find the unit the combined system. (2001 - 5 Marks)
vector in the direction of the refracted ray in medium –2.
31. A vessel ABCD of 10 cm width has two small slits S1 and S2
(1999 - 10 Marks)
sealed with identical glass plates of equal thickness. The
28. A quarter cylinder of radius R and refractive index 1.5 is
distance between the slits is 0.8 mm. POQ is the line
placed on a table. A point object P is kept at a distance of mR
perpendicular to the plane AB and passing through O, the
from it. Find the value of m for which a ray from P will emerge
middle point of S1 and S2. A monochromatic light source is
parallel to the table as shown in Figure. (1999 - 5 Marks)
kept at S, 40 cm below P and 2 m from the vessel, to illuminate
EBD_7036
P-170 Topic-wise Solved Papers - PHYSICS
the slits as shown in the figure below. Calculate the position 34. Find the focal length of the lens shown in the figure. The
of the central bright fringe on the other wall CD with respect radii of curvature of both the surfaces are equal to R.
to the line OQ. Now, a liquid is poured into the vessel and (2003 - 2 Marks)
filled upto OQ. The central bright fringe is found to be at Q. m1 < m 2 < m 3
Calculate the refractive index of the liquid.(2001-5 Marks)
m1 m2 m3
A
D
R R
S1
P
Q
O
40 cm S2

S 2m 10 cm 35. Shown in the figure is a prism of angle 30º and refractive


B C
index m p = 3 . Face AC of the prism is covered with a thin
32. A thin biconvex lens of refractive index 3/2 is placed on a film of refractive index mf = 2.2. A monochromatic light of
horizontal plane mirror as shown in the figure. The space wavelength l = 550 nm fall on the face AB at an angle of
between the lens and the mirror is then filled with water of incidence of 60º. (2003 - 4 Marks)
refractive index 4/3. It is found that when a point object is A
placed 15 cm above the lens on its principal axis, the object
coincides with its own image. On repeating with another
liquid, the object and the image again coincide at a distance
25 cm from the lens. Calculate the refractive index of the 30º
liquid. (2001-5 Marks) 60º

mp = 3
33. A point source S emitting light of wavelength 600 nm is B C m f = 2.2
placed at a very small height h above a flat reflecting surface
AB (see figure). The intensity of the reflected light is 36% Calculate
(a) angle of emergence.
of the incident intensity. Interference fringes are observed
(b) minimum value of thickness t so that intensity of
on a screen placed parallel to the reflecting surface at a
emergent ray is maximum.
very large distance D from it. (2002 - 5 Marks)
36. A ray is incident on a medium consisting of two boundaries,
one plane and other curved as shown in the figure. The
P plane surface makes an angle 60º with horizontal and curved
Screen
surface has radius of curvature 0.4 m. The refractive indices
of the medium and its environment are shown in the figure.
If after refraction at both the surfaces the ray meets principle
axis at P, find OP. (2004 - 2 Marks)
D
n=Ö2
45º O P
n=1 n=1.514

S 60º
h
A B 37. In YDSE a light containing two wavelengths 500 nm and
700 nm are used. Find the minimum distance where maxima
of two wavelengths coincide. Given D/d = 103, where D is
(a) What is the shape of the interference fringes on the the distance between the slits and the screen and d is the
screen? distance between the slits. (2004 - 4 Marks)
(b) Calculate the ratio of the minimum to the maximum 38. An object is moving with velocity 0.01 m/s towards a convex
intensities in the interference fringes formed near the lens of focal length 0.3 m. Find the magnitude of rate of
point P (shown in the figure). separation of image from the lens when the object is at a
(c) If the intensity at point P corresponds to a maximum, distance of 0.4 m from the lens. Also calculate the magnitude
calculate the minimum distance through which the of the rate of change of the lateral magnification.
reflecting surface AB should be shifted so that the (2004 - 4 Marks)
intensity at P again becomes maximum.
Ray and Wave Optics P-171

39. What will be the minimum angle of incidence such that the B D
total internal reflection occurs on both the surfaces?
60° 60°
m1 = 2
m2 = 2 60° 60°
(2005 - 2 Marks)
m3 = 3 A C E
(a) the angle of incidence, so that the emergent ray from
40. Two identical prisms of refractive index 3 are kept as the first prism has minimum deviation.
shown in the figure. A light ray strikes the first prism at face (b) through what angle the prism DCE should be rotated
AB. Find, (2005 - 4 Marks) about C so that the final emergent ray also has
minimum deviation.
F Match the Following

DIRECTIONS (Q. No. 1-4) : Each question contains statements given in two columns, which have to p q r s t
be matched. The statements in Column-I are labelled A, B, C and D, while the statements in Column-
A p q r s t
II are labelled p, q, r and s. Any given statement in Column-I can have correct matching with ONE OR
MORE statement(s) in Column-II. The appropriate bubbles corresponding to the answers to these B p q r s t
questions have to be darkened as illustrated in the following example : C p q r s t
If the correct matches are A-p, s and t; B-q and r; C-p and q; and D-s then the correct darkening of D p q r s t
bubbles will look like the given.

1. A simple telescope used to view distant objects has eyepiece and objective lens of focal lengths fe and fo, respectively. Then
(2006 - 6M)
Column I Column II
(A) Intensity of light received by lens (p) Radius of aperture
(B) Angular magnification (q) Dispersion of lens
(C) Length of telescope (r) Focal length of objective lens and eyepiece lens
(D) Sharpness of image (s) Spherical aberration
2. An optical component and an object S placed along its optic axis are given in Column I. The distance between the object and the
component can be varied. The properties of images are given in Column II. Match all the properties of images from Column II with
the appropriate components given in Column I. Indicate your answer by darkening the appropriate bubbles of the 4 × 4 matrix
given in the ORS. (2008)
Column I Column II

S
(A) (p) real image

S
(B) (q) virtual image

S
(C) (r) magnified image

S
(D) (s) image at infinity

3. Column-I shows four situations of standard Young’s double slit arrangement with the screen placed far away from the slits S1
and S2. In each of these cases S1P0 = S2P0, S1P1-S2P1= l/4 and S1P2– S2P2= l/3, where l is the wavelength of the light used. In
the cases B, C and D, a transparent sheet of refractive index m and thickness t is pasted on slit S2. The thicknesses of the sheets
are different in different cases. The phase difference between the light waves reaching a point P on the screen from the two slits
is denoted by d (P) and the intensity by I(P). Match each situation given in Column-I with the statetment(s) in Column-II valid
for that situation. (2009)
EBD_7036
P-172 Topic-wise Solved Papers - PHYSICS
Column-I Column-II
P2
S2 P1
(A) P0 (p) d(P0) = 0
S1

P2
S2 P1
P0
(B) (m – 1) t = l/4 (q) d (P1) = 0
S1

P2
S2
P1
(C) (m – 1) t = l/2 P0 (r) I (P1) = 0
S1

P2
S2 P1
P0
(D) (m – 1) t = 3l/4 S (s) I(P0) > I (P1)
1

(t) I(P2) > I (P1)


4. Two transparent media of refractive indices m1 and m3 have a solid lens shaped transparent material of refractive index m 2 between
them as shown in figures in Column II. A ray traversing these media is also shown in the figures. In Column I different relationships
between m1, m 2, and m3 are given. Match them to the ray diagrams shown in Column II. (2010)
Column I Column II

(A) m1 < m 2 (p) m m2 m1


3

(B) m1 > m 2 (q) m 3 m2 m1

(C) m 2 = m3 (r)
m3 m2 m1

(D) m 2 > m 3 (s)


m3 m2 m1

(t)
m3 m2 m1
Ray and Wave Optics P-173

DIRECTION (Q. No. 5 & 6) Following question has matching lists. The codes for the lists have choices (a), (b), (c) and (d) out of
which ONLY ONE is correct.
5. A right angled prism of refractive index m1 is placed in a rectangular block of refractive index m2, which is surrounded by a medium
of refractive index m3, as shown in the figure. A ray of light 'e' enters the rectangular block at normal incidence. Depending upon
the relationships between m1, m2 and m3, it takes one of the four possible paths 'ef', 'eg', 'eh' or 'ei'.
Match the paths in List I with conditions of refractive indices in List II and select the correct answer using the codes given below
the lists: (JEE Adv. 2013)
List I List II
P. e®f 1. m1 > 2m 2 f
Q. e®g 2. m2 > m1 and m2 > m3 45° g
e
R. e®h 3. m1 = m2
S. e®i 4. m2 < m1< 2m 2 and m2 > m3 m1 h
Codes: i
P Q R S
(a) 2 3 1 4
m3
(b) 1 2 4 3 m2
(c) 4 1 2 3
(d) 2 3 4 1
6. Four combinations of two thin lenses are given in List-I. The radius of curvature of all curved surfaces is r and the refractive index
of all the lenses is 1.5. Match lens combinations in List-I with their focal length in List-II and select the correct answer using the
code given below the lists. (JEE Adv. 2014)
List - I List - II

P. 1. 2r

r
Q. 2.
2

R. 3. –r

S. 4. r

Codes:
(a) P-1, Q-2, R-3, S-4 (b) P-2, Q-4, R-3, S-1 (c) P-4, Q-1, R-2, S-3 (d) P-2, Q-1, R-3, S-4

G Comprehension Based Questions b d

PASSAGE - 1
medium-1
The figure shows a surface XY separating two transparent media, a c
medium-1 and medium-2. The line ab and cd represent waveforms X Y
f h
of a light wave travelling in medium-1 and incident on XY. The
lines ef and gh represent wavefronts of the light wave in medium- medium-2
2 after refraction. (2007) e g
EBD_7036
P-174 Topic-wise Solved Papers - PHYSICS
1. Light travels as a 5. Choose the correct statement.
(a) parallel beam in each medium (a) The speed of light in the meta-material is v = c|n|
(b) convergent beam in each medium
c
(c) divergent beam in each medium (b) The speed of light in the meta-material is v =
(d) divergent beam in one medium and convergent beam n
in the other medium. (c) The speed of light in the meta-material is v = c.
2. The phases of the light wave at c, d, e and f are fc, fd, fe and (d) The wavelength of the light in the meta-material (lm)
ff respectively. It is given that fc ¹ ff. is given by lm = lair |n|, where lair is wavelength of the
(a) fc cannot be equal to fd light in air.
(b) fd can be equal to fe PASSAGE-3
(c) (fd – ff) is equal to (fc – fe)
(d) (fd – fc) is not equal to (ff – fe) Light guidance in an optical fibre can be understood by considering
3. Speed of light is a structure comprising of thin solid glass cylinder of refractive
(a) the same in medium-1 and medium-2 index n1 surrounded by a medium of lower refractive index n2.
(b) larger in medium-1 than in medium-2 The light guidance in the structure takes place due to successive
(c) larger in medium-2 than in medium-1 total internal reflections at the interface of the media n1 and n2 as
(d) different at b and d. shown in the figure. All rays with the angle of incidence i less
PASSAGE-2 than a particular value im are confined in the medium of refractive
index n1. The numerical aperture (NA) of the structure is defined
Most materials have the refractive index, n > 1. So, when a light
ray from air enters a naturally occurring material, then by Snell’s as sin im.
sin q1 n2
law, = , it is understood that the refracted ray bends
sin q2 n1
towards the normal. But it never emerges on the same side of the
normal as the incident ray. According to electromagnetism, the
refractive index of the medium is given by the relation, c n= /
v = ± er mr , where c is the speed of electromagnetic waves in
vacuum, v its speed in the medium, er and mr are the relative
permittivity and permeability of the medium respectively.
In normal materials, both e r and mr , are positive, implying positive
n for the medium. When both e r and mr are negative, one must 6. For two structure namely S1 with n1 = 45 / 4 and n2 = 3/2,
choose the negative root of n. Such negative refractive index and S2 with n1 = 8/5 and n2 = 7/5 and taking the refractive
materials can now be artificially prepared and are called meta- index of water to be 4/3 and that of air to be 1, the correct
materials. They exhibit significantly different optical behavior,
option(s) is(are) (JEE Adv. 2015)
without violating any physical laws. Since n is negative, it results
in a change in the direction of propagation of the refracted light. (a) NA of S1 immersed in water is the same as that of S2
However, similar to normal materials, the frequency of light remains 16
unchanged upon refraction even in meta-materials. (2012) immersed in a liquid of refractive index
3 15
4. For light incident from air on a meta-material, the appropriate
ray diagram is 6
(b) NA of S1 immersed in liquid of refractive index is
15
the same as that of S2 immersed in water
q1 (c) NA of S1 placed in air is the same as that of S2 immersed

Air
4
(a) in liquid of refractive index
Meta-material (b) 15
(d) NA of S1 placed in air is the same as that of S2 placed in
q2
water
7. If two structure of same cross-sectional area, but different
numerical apertures NA1 and NA2(NA2 < NA1) are joined
longitudinally, the numerical aperture of the combined
q1 q1
structure is (JEE Adv. 2015)
Air Air
(c) Meta-material (d) Meta-material NA1 NA2
q2 (a) (b) NA1 + NA2
q2 NA1 + NA2

(c) NA 1 (d) NA 2
Ray and Wave Optics P-175

5. A Young's double slit interference arrangement with slits S1


H Assertion & Reason Type Questions
4
and S2 is immersed in water (refractive index = ) as shown
1. STATEMENT-1 (2007) 3
The formula connecting u, v and f for a spherical mirror is in the figure. The positions of maximum on the surface of
valid for mirrors whose sizes are very small compared to water are given by x 2 = p2 m2 l2 – d2 , where l is the
their radii of curvature. wavelength of light in air (refractive index = 1), 2d is the
because separation between the slits and m is an integer. The value
STATEMENT-2 of p is (JEE Adv. 2015)
Laws of reflection are strictly valid for plane surfaces, but
not for large spherical surfaces.
S1
(a) Statement-1 is True, Statement-2 is True; Statement-2
is a correct explanation for Statement-1 d
x Air
(b) Statement-1 is True, Statement-2 is True; Statement-2
d
is NOT a correct explanation for Statement-1 Water
S2
(c) Statement-1 is True, Statement-2 is False
(d) Statement-1 is False, Statement-2 is True
6. Consider a concave mirror and a convex lens (refractive index
I Integer Value Correct Type = 1.5) of focal length 10 cm each, separated by a distance of
1. The focal length of a thin biconvex lens is 20 cm. When an 50 cm in air (refractive index = 1) as shown in the figure. An
object is moved from a distance of 25 cm in front of it to 50 object is placed at a distance of 15 cm from the mirror. Its
erect image formed by this combination has magnification
cm, the magnification of its image changes from m25 to m50.
M1. When the set-up is kept in a medium of refractive index
m25
The ratio is (2010) 7 M
m50 , the magnification becomes M2. The magnitude 2 is
6 M1
2. A large glass slab (m = 5 / 3) of thickness 8 cm is placed over
a point source of light on a plane surface. It is seen that light (JEE Adv. 2015)
emerges out of the top surface of the slab from a circular
area of radius R cm. What is the value of R? (2010)
3. Image of an object approaching a convex mirror of radius of 15cm
curvature 20 m along its optical axis is observed to move
25 50
from m to m in 30 seconds. What is the speed of the
3 7 50cm
object in km per hour? (2010)
7. The monochromatic beam of light is incident at 60° on one
4 face of an equilateral prism of refractive index n and emerges
4. Water (with refractive index = ) in a tank is 18 cm deep. Oil
3 from the opposite face making an angle q(n) with the normal
7 dq
of refractive index lies on water making a convex surface (see the figure). For n = 3 the value of q is 60° and = m.
4 dn
of radius of curvature ‘R = 6 cm’ as shown. Consider oil to The value of m is (JEE Adv. 2015)
act as a thin lens. An object ‘S’ is placed 24 cm above water
surface. The location of its image is at ‘x’ cm above the
bottom of the tank. Then ‘x’ is (2011)

60° q
EBD_7036
P-176 Topic-wise Solved Papers - PHYSICS

Section-B JEE Main / AIEEE


1. An astronomical telescope has a large aperture to 11. A plano convex lens of refractive index 1.5 and radius of
(a) reduce spherical aberration [2002] curvature 30 cm. Is silvered at the curved surface. Now this
(b) have high resolution lens has been used to form the image of an object. At what
(c) increase span of observation distance from this lens an object be placed in order to have a
(d) have low dispersion. real image of size of the object [2004]
(a) 60 cm (b) 30 cm
2. If two mirrors are kept at 60° to each other, then the number
(c) 20 cm (d) 80 cm
of images formed by them is [2002]
12. The angle of incidence at which reflected light is totally
(a) 5 (b) 6 (c) 7 (d) 8
polarized for reflection from air to glass (refractive index n),
3. Electromagnetic waves are transverse in nature is evident
is [2004]
by [2002]
(a) polarization (b) interference (a) tan -1 (1/ n) (b) sin -1 (1/ n)
(c) reflection (d) diffraction
4. Wavelength of light used in an optical instrument are (c) sin -1 (n) (d) tan -1 (n)
l1 = 4000 Å and l 2 = 5000 Å , then ratio of their 13. The maximum number of possible interference maxima for
slit-separation equal to twice the wavelength in Young’s
respective resolving powers (corresponding to l1 and l 2 ) double-slit experiment is [2004]
is [2002] (a) three (b) five
(a) 16 : 25 (b) 9 : 1 (c) 4 : 5 (d) 5 : 4. (c) infinite (d) zero
5. Which of the following is used in optical fibres? 14. An electromagnetic wave of frequency n = 3.0 MHz
(a) total internal reflection [2002] passes from vacuum into a dielectric medium with
(b) scattering
permittivity Î= 4.0 . Then [2004]
(c) diffraction
(d) refraction. (a) wave length is halved and frequency remains
6. Consider telecommunication through optical fibres. unchanged
Which of the following statements is not true? [2003] (b) wave length is doubled and frequency becomes half
(a) Optical fibres can be of graded refractive index (c) wave length is doubled and the frequency remains
(b) Optical fibres are subject to electromagnetic unchanged
interference from outside (d) wave length and frequency both remain unchanged.
15. A fish looking up through the water sees the outside world
(c) Optical fibres have extremely low transmission loss
contained in a circular horizon. If the refractive index of
(d) Optical fibres may have homogeneous core with a
suitable cladding. 4
water is and the fish is 12 cm below the surface, the
7. To demonstrate the phenomenon of interference, we require 3
two sources which emit radiation [2003] radius of this circle in cm is [2005]
(a) of nearly the same frequency
(b) of the same frequency 36
(a) (b) 36 7 (c) 4 5 (d) 36 5
(c) of different wavelengths 7
(d) of the same frequency and having a definite phase 16. Two point white dots are 1 mm apart on a black paper. They
relationship are viewed by eye of pupil diameter 3 mm. Approximately,
8. The image formed by an objective of a compound what is the maximum distance at which these dots can be
microscope is [2003] resolved by the eye? [Take wavelength of light = 500 nm]
(a) virtual and diminished (a) 1 m (b) 5 m [2005]
(b) real and diminished
(c) 3 m (d) 6 m
(c) real and enlarged
17. A thin glass (refractive index 1.5) lens has optical power of
(d) virtual and enlarged
– 5 D in air. Its optical power in a liquid medium with refractive
9. To get three images of a single object, one should have two
plane mirrors at an angle of [2003] index 1.6 will be [2005]
(a) – 1D (b) 1 D (c) – 25 D (d) 25 D
(a) 60º (b) 90º (c) 120º (d) 30º
18. A Young’s double slit experiment uses a monochromatic
10. A light ray is incident perpendicularly to one face of a 90°
prism and is totally internally reflected at the glass-air source. The shape of the interference fringes formed on a
interface. If the angle of reflection is 45°, we conclude that screen is [2005]
the refractive index n [2004] (a) circle (b) hyperbola
(c) parabola (d) straight line
1
(a) n > 19. If I 0 is the intensity of the principal maximum in the single
2
slit diffraction pattern, then what will be its intensity when
(b) n > 2 45°
45°
the slit width is doubled? [2005]
1 I0
(c) n < (a) 4 I 0 (b) 2 I 0 (c) (d) I 0
2 2
45°
(d) n< 2
Ray and Wave Optics P-177

20. When an unpolarized light of intensity I 0 is incident on a 25. A student measures the focal length of a convex lens by
polarizing sheet, the intensity of the light which does not putting an object pin at a distance ‘u’ from the lens and
get transmitted is [2005] measuring the distance ‘v’ of the image pin. The graph
between ‘u’ and ‘v’ plotted by the student should look like
1 1
(a) I0 (b) I0 (c) I 0 (d) zero [2008]
4 2
21. The refractive index of a glass is 1.520 for red light and 1.525 v(cm)
v(cm)
for blue light. Let D1 and D2 be angles of minimum deviation
for red and blue light respectively in a prism of this glass.
(a) (b)
Then, [2006]
O u(cm) O u(cm)
(a) D1 < D2
(b) D1 = D2
(c) D1 can be less than or greater than D2 depending upon v(cm)
the angle of prism v(cm)
(d) D1 > D2
22. In a Young’s double slit experiment the intensity at a point
(c) (d) O
l O u(cm) u(cm)
where the path difference is (l being the wavelength of
6
26. An experment is performed to find the refractive index of
I glass using a travelling microscope. In this experiment
light used) is I. If I0 denotes the maximum intensity, is
I0 distances are measured by [2008]
equal to [2007] (a) a vernier scale provided on the microscope
(b) a standard laboratory scale
3 1 3 1 (c) a meter scale provided on the microscope
(a) (b) (c) (d)
4 2 2 2 (d) a screw gauge provided on the microscope
23. Two lenses of power –15 D and +5 D are in contact with 27. A mixture of light, consisting of wavelength 590 nm and an
each other. The focal length of the combination is unknown wavelength, illuminates Young’s double slit and
(a) + 10 cm (b) – 20 cm [2007] gives rise to two overlapping interference patterns on the
(c) – 10 cm (d) + 20 cm screen. The central maximum of both lights coincide. Further,
24. In an experiment, electrons are made to pass through a narrow it is observed that the third bright fringe of known light
slit of width ‘d’ comparable to their de Broglie wavelength.
coincides with the 4th bright fringe of the unknown light.
They are detected on a screen at a distance ‘D’ from the slit
From this data, the wavelength of the unknown light is:
(see figure).
[2009]
(a) 885.0 nm (b) 442.5 nm
d
y=0 (c) 776.8 nm (d) 393.4 nm
28. A transparent solid cylindrical rod has a refractive index of
D
2
Which of the following graphs can be expected to represent . It is surrounded by air. A light ray is incident at the
the number of electrons ‘N’ detected as a function of the 3
detector position ‘y’(y = 0 corresponds to the middle of the mid-point of one end of the rod as shown in the figure.
slit) [2008]
y y
q

(a) N d (b) N d
The incident angle q for which the light ray grazes along the
wall of the rod is : [2009]

æ ö æ 2 ö
y (a) sin -1 ç 3 2 ÷ (b) sin -1 ç ÷
y è ø è 3ø

æ 1 ö
(c) N d (d) N d (c) sin -1 ç
è 3ø
÷ (d) ( 2)
sin -1 1
EBD_7036
P-178 Topic-wise Solved Papers - PHYSICS
29. In an optics experiment, with the position of the object fixed, (a) Statement – 1 is true, Statement – 2 is true, Statement –
a student varies the position of a convex lens and for each 2 is the correct explanation of Statement – 1.
position, the screen is adjusted to get a clear image of the (b) Statement – 1 is true, Statement – 2 is true, Statement –
object. A graph between the object distance u and the image
2 is not the correct explanation of Statement – 1.
distance v, from the lens, is plotted using the same scale for
the two axes. A straight line passing through the origin and (c) Statement – 1 is false, Statement – 2 is true.
making an angle of 45° with the x-axis meets the experimental (d) Statement – 1 is true, Statement – 2 is false.
curve at P. The coordinates of P will be : [2009] 35. A car is fitted with a convex side-view mirror of focal length
æ f fö 20 cm. A second car 2.8 m behind the first car is overtaking
(a) çè 2 , 2 ÷ø (b) ( f, f ) (c) ( 4 f, 4 f ) (d) ( 2 f, 2 f ) the first car at a relative speed of 15 m/s. The speed of the
image of the second car as seen in the mirror of the first one
DIRECTIONS : Questions number 30-32 are based on the is : [2011]
following paragraph.
An initially parallel cylindrical beam travels in a medium of 1 1
(a) m/s (b) 10 m/s (c) 15 m/s (d) m/s
refractive index m (I) =m0 + m2 I, where m0 and m2 are positive 15 10
constants and I is the intensity of the light beam. The intensity of 36. An electromagnetic wave in vacuum has the electric and
the beam is decreasing with increasing radius. r r
magnetic field E and B , which are always perpendicular to
30. As the beam enters the medium , it will [2010] r
(a) diverge each other. The direction of polarization is given by X and
(b) converge r
that of wave propagation by k . Then [2012]
(c) diverge near the axis and converge near the periphery r r r r r
(d) travel as a cylindrical beam (a) X || B and k || B ´ E
31. The initial shape of the wavefront of the beam is [2010] r r r r r
(b) X || E and k || E ´ B
(a) convex r r r r r
(b) concave (c) X || B and k || E ´ B
(c) convex near the axis and concave near the periphery r r r r r
(d) X || E and k || B ´ E
(d) planar
37. In Young's double slit experiment, one of the slit is wider
32. The speed of light in the medium is [2010]
than other, so that amplitude of the light from one slit is
(a) minimum on the axis of the beam
double of that from other slit. If Im be the maximum intensity,
(b) the same everywhere in the beam
the resultant intensity I when they interfere at phase
(c) directly proportional to the intensity I difference f is given by : [2012]
(d) maximum on the axis of the beam
33. Let the x-z plane be the boundary between two transparent Im Im æ 2 fö
(a) (4 + 5 cos f) (b) ç 1 + 2 cos ÷
9 3 è 2ø
media. Medium 1 in z ³ 0 has a refractive index of 2 and
medium 2 with z < 0 has a refractive index of 3 . A ray of Im æ 2 fö Im æ 2 fö
r (c) ç 1 + 4 cos ÷ (d) ç 1 + 8 cos ÷
light in medium 1 given by the vector A = 6 3iˆ + 8 3 ˆj - 10kˆ 5 è 2ø 9 è 2ø
is incident on the plane of separation. The angle of refraction
38. An object 2.4 m in front of a lens forms a sharp image on a
in medium 2 is: [2011]
film 12 cm behind the lens. A glass plate 1 cm thick, of
(a) 45° (b) 60° (c) 75° (d) 30°
refractive index 1.50 is interposed between lens and film
34. This question has a paragraph followed by two statements,
with its plane faces parallel to film. At what distance (from
Statement – 1 and Statement – 2. Of the given four
alternatives after the statements, choose the one that lens) should object shifted to be in sharp focus of film?
describes the statements. [2012]
A thin air film is formed by putting the convex surface of a (a) 7.2 m (b) 2.4 m
plane-convex lens over a plane glass plate. With (c) 3.2 m (d) 5.6 m
monochromatic light, this film gives an interference pattern 39. Diameter of a plano-convex lens is 6 cm and thickness at
due to light reflected from the top (convex) surface and the
the centre is 3 mm. If speed of light in material of lens is
bottom (glass plate) surface of the film. [2011]
2× 108 m/s, the focal length of the lens is
Statement – 1 : When light reflects from the air-glass plate
[JEE Main 2013]
interface, the reflected wave suffers a phase change of p.
Statement – 2 : The centre of the interference pattern is (a) 15 cm (b) 20 cm (c) 30 cm (d) 10 cm
dark.
Ray and Wave Optics P-179

40. Abeam of unpolarised light of intensity I0 is passed through 44. A green light is incident from the water to the air - water
a polaroidAand then through another polaroid B which is interface at the critical angle (q). Select the correct statement.
oriented so that its principal plane makes an angle of 45° [JEE Main 2014]
relative to that of A. The intensity of the emergent light is (a) The entire spectrum of visible light will come out of the
[JEE Main 2013] water at an angle of 90º to the normal.
(a) I0 (b) I0/2 (b) The spectrum of visible light whose frequency is less
than that of green light will come out to the air medium.
(c) I0/4 (d) I0/8
(c) The spectrum of visible light whose frequency is more
41. Two coherent point sources S1 and S2 are separated by a
than that of green light will come out to the air medium.
small distance 'd' as shown. The fringes obtained on the
(d) The entire spectrum of visible light will come out of the
screen will be [JEE Main 2013]
water at various angles to the normal.
45. Two beams, A and B, of plane polarized light with mutually
perpendicular planes of polarization are seen through a
d polaroid. From the position when the beam A has maximum
intensity (and beam B has zero intensity), a rotation of polaroid
S1 S2 Screen through 30° makes the two beams appear equally bright. If
D the initial intensities of the two beams are IA and IB

IA
(a) points (b) straight lines respectively, then equals: [JEE Main 2014]
IB
(c) semi-circles (d) concentric circles
42. The graph between angle of deviation (d) and angle of 3
(a) 3 (b)
incidence (i) for a triangular prism is represented by 2
[JEE Main 2013]
1
(c) 1 (d)
3
d d 46. Assuming human pupil to have a radius of 0.25 cm and a
(a) (b) comfortable viewing distance of 25 cm, the minimum
separation between two objects that human eye can resolve
o o at 500 nm wavelength is : [JEE Main 2015]
i i
(a) 100 µm (b) 300 µm
(c) 1 µm (d) 30 µm
47. On a hot summer night, the refractive index of air is smallest
d d near the ground and increases with height from the ground.
When a light beam is directed horizontally, the Huygens'
(c) (d)
principle leads us to conclude that as it travels, the light
o o beam : [JEE Main 2015]
i i (a) bends downwards
(b) bends upwards
æ 3ö (c) becomes narrower
43. A thin convex lens made from crown glass ç m = ÷ has focal
è 2ø (d) goes horizontally without any deflection
length f. When it is measured in two different liquids having 48. Monochromatic light is incident on a glass prism of angle A.
If the refractive index of the material of the prism is µ, a ray,
4 5
refractive indices and , it has the focal lengths f1 and f2 incident at an angle q, on the face AB would get transmitted
3 3 through the face AC of the prism provided : [JEE Main 2015]
respectively. The correct relation between the focal lengths
is: [JEE Main 2014]
(a) f1 = f2 < f A
(b) f1 > f and f2 becomes negative q
(c) f2 > f and f1 becomes negative
(d) f1 and f2 both become negative
B C
EBD_7036
P-180 Topic-wise Solved Papers - PHYSICS

é æ æ 1 öù (a) a = lL and bmin = 4lL


(a) q > cos -1 êµsin ç A + sin -1 ç ÷ ú
ëê è è µ ø ûú l2
(b) a= and bmin = 4lL
L
é æ æ 1 öù
(b) q < cos-1 êµsin ç A + sin -1 ç ÷ ú
ëê è è µ ø ûú l2 æ 2l2 ö
(c) a= and bmin = ç
ç L ÷
÷
L è ø
é æ æ 1 öù
(c) q > sin -1 êµsin ç A - sin -1 ç ÷ ú æ 2l2 ö
ëê è è µ ø ûú a = ll and bmin = ç ÷
(d) ç L ÷
è ø
-1 é æ -1 æ 1 ö ù 50. An observer looks at a distant tree of height 10 m with a
(d) q < sin êµsin ç A - sin ç ÷ ú
ëê è è µ ø ûú telescope of magnifying power of 20. To the observer the
49. The box of a pin hole camera, of length L, has a hole of tree appears : [JEE Main 2016]
radius a. It is assumed that when the hole is illuminated by (a) 20 times taller (b) 20 times nearer
a parallel beam of light of wavelength l the spread of the (c) 10 times taller (d) 10 times nearer
spot (obtained on the opposite wall of the camera) is the 51. In an experiment for determination of refractive index of glass
sum of its geometrical spread and the spread due to of a prism by i – d, plot it was found thata ray incident at
diffraction. The spot would then have its minimum size (say angle 35°, suffers a deviation of 40° and that it emerges at
angle 79°. In that case which of the following is closest to
bmin) when : [JEE Main 2016] the maximum possible value of the refractive index?
[JEE Main 2016]
(a) 1.7 (b) 1.8
(c) 1.5 (d) 1.6
CHAPTER

17 Modern Physics
Section-A JEE Advanced/ IIT-JEE
A Fill in the Blanks 11. The wavelength of the characteristic X-ray K a line emitted
by a hydrogen like element is 0.32 Å. The wavelength of the
1. To produce characteristic X-rays using a Tungsten target
in an X-ray generator, the accelerating voltage should be K b line emitted by the same element will be ............
greater than ___________ volts and the energy of the (1990 - 2 Marks)
characteristic radiation is _____eV. 12. The Bohr radius of the fifth electron of phosphorous atom
(The binding energy of the innermost electron in Tungsten (atomic number = 15) acting as a dopant in silicon (relative
is 40 keV). (1983 - 2 Marks) dielectric constant = 12) is .................... Å (1991 - 1 Mark)
2. The radioactive decay rate of a radioactive element is found 13. For the given circuit shown in fig. to act as full wave rectifier,
to be 103 disintegration/second at a certain time. If the half the a.c. input should be connected across .................... and
life of the element is one second , the decay rate after one .................... and the d.c. output would appear across
second is _______ and after three seconds is _______. .................... and ................ (1991 - 1 Mark)
(1983 - 2 Marks)
3. The maximum kinetic energy of electrons emitted in the B A
photoelectric effect is linearly dependent on the ...... of the
incident radiation. (1984- 2 Marks)
238
4. In the Uranium radioactive series the initial nucleus is 92 U
206 C D
and the final nucleus is 82 Pb . When the Uranium nucleus
decays to lead, the number of a-particles emitted is ......and 14. In an X-ray tube, electrons accelerated through a potential
the number of b-particles emitted is ..... (1985 - 2 Marks) difference of 15, 000 volts strike a copper target. The speed
5. When the number of electrons striking the anode of an of the emitted X-ray inside the tube is ................... m/s
X-ray tube is increased, the ........ of the emitted X-rays (1992 - 1 Mark)
increases, while when the speeds of the electrons striking 15. In the Bohr model of the hydrogen atom, the ratio of the
the anode are increased, the cut-off wavelength of the kinetic energy to the total energy of the electron in a quantum
emitted X-rays......... (1986 - 2 Marks) state n is .................... (1992 - 1 Mark)

6. When Boron nucleus ( B) is bombarded by neutrons,


10
5
16. In the nuclear process, 6C
11
¾¾ ® 5 B11 + b + + X , X
a -particles are emitted. The resulting nucleus is of the stands for ............. (1992 - 1 Mark)
element ........ and has the mass number ......(1986 - 2 Marks) 17. In a ..... biased p-n junction, the net flow of holes is from the
7. Atoms having the same ........... but different ........... are called n region to the p region. (1993 - 1 Mark)
isotopes. (1986 - 2 Marks) 18. A potential difference of 20 kV is applied across an X-ray
8. The binding energies per nucleon for deuteron (1H2) and tube. The minimum wavelength of X-rays generated
helium (2He4) are 1.1 MeV and 7.0 MeV respectively. The is...........Å. (1996 - 2 Marks)
energy released when two deuterons fuse to form a helium 19. The wavelength of Ka X-rays produced by an X-ray tube is
nucleus (2He4) is .......... (1988 - 2 Marks) 0.76Å. The atomic number of the anode material of the tube
9. In the forward bias arrangement of a p-n junction rectifier, is..... (1996 - 2 Marks)
the p end is connected to the ........terminal of the battery 20. Consider the following reaction :
and the direction of the current is from ............to ..................in 2 2 4
1 H + 1 H =2 He + Q
the rectifier. (1988 - 2 Marks)
Mass of the deuterium atom = 2.0141 u
10. ........... biasing of p-n junction offers high resistance to
Mass of helium atom = 4.0024 u
current flow across the junction. The biasing is obtained by
This is a nuclear ............. reaction in which the energy Q
connecting the p- side to the .......... terminal of the battery.
released is ............. MeV. (1996 - 2 Marks)
(1990 - 2 Marks)
EBD_7036
P-182 Topic-wise Solved Papers - PHYSICS
7. The equation
B True/False
4 11 H + ¾¾
® 42 He 2 + + 2e – + 26 MeV represents
1. The kinetic energy of photoelectrons emitted by a (1983 - 1 Mark)
photosensitive surface depends on the intensity of the (a) b -decay (b) g-decay
incident radiation. (1981- 2 Marks)
(c) fusion (d) fission
2. In a photoelectric emission process the maximum energy of 8. Fast neutrons can easily be slowed down by
the photo-electrons increases with increasing intensity of (1994 - 1 Mark)
the incident light. (1986 - 3 Marks) (a) the use of lead shielding
3. For a diode the variation of its anode current Ia with the (b) passing them through water
anode voltage Va at two different cathode temperatures T1 (c) elastic collisions with heavy nuclei
and T2 is shown in the figure. The temperature T2 is greater (d) applying a strong electric field.
than T1. (1986 - 3 Marks) 9. Consider a particles, b particles and g - rays, each having an
T2 energy of 0.5 MeV. In increasing order of penetrating powers,
T1 the radiations are: (1994 - 1 Mark)
Ia
(a) a, b, g (b) a, g, b (c) b, g , a (d) g, b, a
10. An energy of 24.6 eV is required to remove one of the
electrons from a neutral helium atom. The energy in (eV)
required to remove both the electrons from a neutral helium
Va atom is (1995S)
(a) 38.2 (b) 49.2 (c) 51.8 (d) 79.0
4. The order of magnitude of the density of nuclear matter is
11. A radioactive material decays by simultaneous emission of
104 kg m–2. (1989 - 2 Marks) two particles with respective half-lives 1620 and 810 years.
The time, in years, after which one-fourth of the material
C MCQs with One Correct Answer remains is (1995S)
(a) 1080 (b) 2430 (c) 3240 (d) 4860
1. The plate resistance of a triode is 3 × 103 ohms and its 12. The probability of electrons to be found in the conduction
mutual conductance is 1.5 × 10–3 amp/volt. The amplification band of an intrinsic semiconductor at a finite temperature
factor of the triode is (1981- 2 - 3 Marks) (1995S)
(a) 5 × 10–5 (b) 4.5 (c) 45 (d) 2 × 10 5 (a) increases exponentially with increasing band gap
2. The half life of radioactive Radon is 3.8 days. The time at the (b) decreases exponentially with increasing band gap
(c) decreases with increasing temperature
1 (d) is independent of the temperature and the band gap
end of which th of the radon sample will remain
20 13. A full-wave rectifier circuit along with the out-put is shown
in Figure. The contribution (s) from the diode 1 is (are)
undecayed is (given log10 e = 0.4343 ) (1981- 2 Marks)
(1996 - 2 Marks)
(a) 3.8 days (b) 16.5 days
(c) 33 days (d) 76 days.
V
3. An alpha particle of energy 5 MeV is scattered through 180°
by a fixed uranium nucleus. The distance of closest approach
is of the order of (1981- 2 Marks)
(a) 1 Å (b) 10–10 cm 0 t
1 out-put
(c) 10–12cm (d) 10–15 cm
+
4. Beta rays emitted by a radioactive material are
(1983 - 1 Mark)
(a) electromagnetic radiations V
(b) the electrons orbiting around the nucleus
(c) charged particles emitted by the nucleus A B C D
(d) neutral particles 0 t
out-put
5. If elements with principal quantum number n > 4 were not (a) C (b) A,C (c) B, D (d) A,B,C, D.
allowed in nature, the number of possible elements would 14. As per Bohr model, the minimum energy (in eV) required to
be (1983 - 1 Mark) remove an electron from the ground state of doubly ionized
(a) 60 (b) 32 (c) 4 (d) 64 Li atom (Z = 3) is (1997 - 1 Mark)
6. Consider the spectral line resulting from the transition (a) 1.51 (b) 13.6 (c) 40.8 (d) 122.4
n = 2 ® n = 1 in the atoms and ions given below. The 15. The circuit shown in the figure contains two diodes each
shortest wavelength is produced by (1983 - 1 Mark) with a forward resistance of 50 ohms and with infinite
(a) Hydrogen atom (b) Deuterium atom backward resistance. If the battery voltage is 6V, the current
(c) Singly ionized Helium (d) Doubly ionised Lithium through the 100 ohm resistance (in amperes) is
(1997 - 1 Mark)
Modern Physics P-183
150 W (a) Y ® 2Z (b) W ® X + Z
(c) W ® 2Y (d) X ® Y + Z
24. Imagine an atom made up of a proton and a hypothetical
50 W particle of double the mass of the electron but having the
same charge as the electron. Apply the Bohr atom model
and consider all possible transitions of this hypothetical
6V 100 W particle to the first excited level. The longest wavelength
photon that will be emitted has wavelength l (given in
(a) zero (b) 0.02 (c) 0.03 (d) 0.036. terms of the Rydberg constant R for the hydrogen atom)
16. Which of the following statements is not true? (1997 - 1 Mark) equal to (2000S)
(a) The resistance of intrinsic semiconductors decrease (a) 9/(5R) (b) 36/(5R) (c) 18/(5R) (d) 4/R
with increase of temperature 25. The electron in a hydrogen atom makes a transition from an
(b) Doping pure Si with trivalent impurities give p-type excited state to the ground state. Which of the following
semiconductors statements is true ? (2000S)
(c) The majority carriers in n-type semiconductors are holes (a) Its kinetic energy increases and its potential and total
(d) A p-n junction can act as a semiconductor diode energies decreases.
17. The maximum kinetic energy of photoelectrons emitted from (b) Its kinetic energy decreases, potential energy increases
a surface when photons of energy 6 eV fall on it is 4 eV. The and its total energy remains the same.
stopping potential, in volt, is (1997 - 1 Mark) (c) Its kinetic and total energies decrease and its potential
(a) 2 (b) 4 (c) 6 (d) 10 energy increases.
18. In hydrogen spectrum the wavelength of Ha line is 656 nm, (d) Its kinetic, potential and total energies decrease.
whereas in the spectrum of a distant galaxy, Ha line 26. Two radioactive materials X1 and X2 have decay constants
wavelength is 706 nm. Estimated speed of the galaxy with
respect to earth is, (1999S - 2 Marks) 10l and l respectively. If initially they have the same
(a) 2 × 108 m/s (b) 2 × 107 m/s number of nuclei, then the ratio of the number of nuclei of
(c) 2 × 106 m/s (d) 2 × 105 m/s X1 to that of X2 will be 1/e after a time (2000S)
19. A particle of mass M at rest decays into two particles of 1 1 11 1
masses m1 and m2, having non-zero velocities. The ratio of (a) (b) (c) (d)
the de Broglie wavelengths of the particles, l1/l2, is 10l 11l 10l 9l
(1999S - 2 Marks) 27. Electrons with energy 80 keV are incident on the tungsten
target of an X-ray tube. K-shell electrons of tungsten have
(a) m1/m2 (b) m2/m1
(d) m2 / m1 (c) 1.0 72.5 keV energy. X-rays emitted by the tube contain only
20. Which of the following is a correct statement? (a) a continuous X-ray spectrum (Bremsstrahlung) with a
(1999S - 2 Marks) minimum wavelength of 0.155Å (2000S)
(a) Beta rays are same as cathode rays (b) a continuous X-ray spectrum (Bremsstrahlung) with
(b) Gamma rays are high energy neutrons all wavelengths
(c) Alpha particles are singly ionised helium atoms
(c) the characteristic X-ray spectrum of tungsten
(d) Protons and neutrons have exactly the same mass
21. Order of magnitude of density of uranium nucleus is, [mp = (d) a continuous X-ray spectrum (Bremsstrahlung) with a
1.67 × 10–27kg] (1999S - 2 Marks) minimum wavelength of 0.155Å and the characteristic
(a) 1020kg/m3 (b) 1017kg/m3 X-ray spectrum of tungsten.
(c) 1014kg/m3 (d) 1011kg/m3 28. The electron emitted in beta radiation originates from
22. 22Ne nucleus, after absorbing energy, decays into two a- (a) inner orbits of atoms (2001S)
particles and an unknown nucleus. The unknown nucleus (b) free electrons existing in nuclei
is (1999S - 2 Marks) (c) decay of a neutron in a nucleus
(a) nitrogen (b) carbon (c) boron (d) oxygen (d) photon escaping from the nucleus
23. Binding energy per nucleon vs mass number curve for nuclei 29. The transition from the state n = 4 to n = 3 in a hydrogen-like
is shown in the Figure. W, X, Y and Z are four nuclei indicated atom results in ultraviolet radiation. Infrared radiation will
on the curve. The process that would release energy is be obtained in the transition (2001S)
(1999S - 2 Marks) (a) 2 ® 1 (b) 3 ® 2 (c) 4 ® 2 (d) 5 ® 4
Binding Energy/nucleon in Mev

30. The intensity of X-rays from a Coolidge tube is plotted


against wavelength l as shown in the figure. The minimum
wavelength found is lC and the wavelength of the Ka line is
Y lK. As the accelerating voltage is increased (2001S)
8.5 X
8.0 (a) lK - lC increases I
7.5 W

5.0 Z
(b) lK - lC decreases
(c) lK increases
0 30 60 90 120
Mass number of nuclei (d) lK decreases l
lC lK
EBD_7036
P-184 Topic-wise Solved Papers - PHYSICS
31. A radioactive sample consists of two distinct species having 38. For uranium nucleus how does its mass vary with volume?
equal number of atoms initially. The mean life time of one (2003S)
species is t and that of the other is 5t. The decay products (a) m µ V (b) m µ 1/V
in both cases are stable. A plot is made of the total number (c) m µ V (d) m µ V2
of radioactive nuclei as a function of time. Which of the
39. A nucleus with mass number 220 initially at rest emits an a -
following figures best represent the form of this plot?
particle. If the Q value of the reaction is 5.5 MeV, calculate
(2001S)
N
the kinetic energy of the a -particle (2003S)
N
(a) 4.4 MeV (b) 5.4 MeV
(c) 5.6 MeV (d) 6.5 MeV
40. In a photoelectric experiment anode potential is plotted
(a) (b) against plate current. (2004S)

t t t t
N N
I

(c) (d) C B
A
t t t t

32. The potential difference applied to an X-ray tube is 5kV and V


the current through it is 3.2mA. Then the number of electrons
striking the target per second is (2002S) (a) A and B will have different intensities while B and C
will have different frequencies
(a) 2 ´ 1016 (b) 5 ´ 10 6 (c) 1´ 1017 (d) 4 ´ 1015 (b) B and C will have different intensities while A and C
33. A Hydrogen atom and a Li++ ion are both in the second will have different frequencies
excited state. If l H and l Li are their respective electronic (c) A and B will have different intensities while A and C
angular momenta, and EH and ELi their respective energies, will have equal frequencies
then (2002S) (d) B and C will have equal intensities while A and B will
have same frequencies
(a) l H > l Li and |EH| > |ELi| (b) l H = l Li and |EH| < |ELi|
41. A 280 days old radioactive substance shows an activity of
(c) l H = l Li and |EH| > |ELi| (d) l H < l Li and |EH| < |ELi| 6000 dps, 140 days later its activity becomes 3000 dps.
34. The half-life of 215Atis 100 m s. The time taken for the What was its initial activity? (2004S)
(a) 20000 dps (b) 24000 dps
radioactivity of a sample of 215At to decay to 1/16th of its
(d) 12000 dps (d) 6000 dps
initial value is (2002S)
42. A proton has kinetic energy E = 100 keV which is equal to
m m
(a) 400 s (b) 6.3 s (c) 40 s m (d)300 m s
that of a photon. The wavelength of photon is l2 and that
35. Which of the following processes represents a g -decay ? of proton is l1. The ration of l2/l1 is proportional to
A A (a) E 2 (b) E 1/2 (2004S)
(a) X z + g ¾¾
® X Z -1 + a + b (2002S) –1 –1/2
(c) E (d) E
A A- 3 43. Ka wavelength emitted by an atom of atomic number Z = 11
(b) X z + 1no ¾¾
® X Z -2 + c
is l. Find the atomic number for an atom that emits Ka
A
(c) A
X z ¾¾
® XZ + f radiation with wavelength 4l. (2005S)
A (a) Z = 6 (b) Z = 4 (c) Z = 11 (d) Z = 44
(d) A
X z + e-1 ¾¾ ® X Z -1 + g 44. A photon collides with a stationary hydrogen atom in ground
36. The electric potential between a proton and an electron is state inelastically. Energy of the colliding photon is 10.2 eV.
After a time interval of the order of micro second another
r
given by V = V0 ln , where r0 is a constant. Assuming photon collides with same hydrogen atom inelastically with
r0
an energy of 15 eV. What will be observed by the detector?
Bohr’s model to be applicable, write variation of rn with n, n (2005S)
being the principal quantum number? (2003S) (a) One photon of energy 10.2 eV and an electron of
(a) rn µ n (b) rn µ 1/n energy 1.4 eV
(c) rn µ n2 (d) rn µ 1/n2 (b) 2 photon of energy of 1.4 eV
37. 257
If the atom 100Fm follows the Bohr model and the radius
(c) 2 photon of energy 10.2 eV
of 100Fm257 is n times the Bohr radius, then find n. (2003S)
(d) One photon of energy 10.2 eV and another photon of
(a) 100 (b) 200 (c) 4 (d) 1/4
1.4 eV
Modern Physics P-185

4 5 . A beam of electron is used in an YDSE experiment. The slit 52. A radioactive sample S1 having an activity 5µCi has twice
width is d. When the velocity of electron is increased, then the number of nuclei as another sample S2 which has an
(a) no interference is observed (2005S) activity of 10 µCi. The half lives of S1 and S2 can be
(a) 20 years and 5 years, respectively (2008)
(b) fringe width increases
(b) 20 years and 10 years, respectively
(c) fringe width decreases
(c) 10 years each
(d) fringe width remains same
(d) 5 years each
46. If a star can convert all the He nuclei completely into oxygen
53. Photoelectric effect experiments are performed using three
nuclei, the energy released per oxygen nuclei is [Mass of different metal plates p, q and r having work functions fp =
He nucleus is 4.0026 amu and mass of Oxygen nucleus is 2.0 eV, fq = 2.5 eV and fr = 3.0 eV, respectively. A light beam
15.9994 amu] (2005S) containing wavelengths of 550 nm, 450 nm and 350 nm with
(a) 7.6 MeV (b) 56.12 MeV equal intensities illuminates each of the plates. The correct
(c) 10.24 MeV (d) 23.9 MeV I-V graph for the experiment is [Take hc = 1240 eV nm]
(2009)
221
47. 87 Ra is a radioactive substance having half life of 4 days. I
I
Find the probability that a nucleus undergoes decay after p
two half lives (2006 - 3M, –1) q p
(a) (b)
r q
1 3 1
(a) 1 (b) (c) (d) r
2 4 4 V V
48. In the options given below, let E denote the rest mass energy I
I
of a nucleus and n a neutron.The correct option is r
(2007) (c) q
(d)
p
æ 236 ö æ137 ö æ 97 ö r q
(a) E ç U÷ > Eç I ÷ + E ç Y ÷ + 2 E (n ) p
è 92 ø è 53 ø è 39 ø V V
æ 236 ö æ137 ö æ 97 ö 54. The wavelength of the first spectral line in the Balmer series
(b) Eç U÷ < Eç I ÷ + E ç Y ÷ + 2 E (n ) of hydrogen atom is 6561 A°. The wavelength of the second
è 92 ø è 53 ø è 39 ø
spectral line in the Balmer series of singly-ionized helium
æ 236 ö æ140 ö æ 94 ö atom is (2011)
(c) Eç U÷ < Eç Ba ÷ + E ç Kr ÷ + 2 E (n)
è 92 ø è 56 ø è 36 ø (a) 1215 A° (b) 1640 A° (c) 2430 A° (d) 4687 A°
55. A pulse of light of duration 100 ns is absorbed completely
æ 236 ö æ 140 ö æ 94 ö by a small object initially at rest. Power of the pulse is 30 mW
(d) Eç U÷ = Eç Ba ÷ + E ç Kr ÷ + 2 E (n)
è 92 ø è 56 ø è 36 ø and the speed of light is 3×108 ms–1. The final momentum of
49. The largest wavelength in the ultraviolet region of the the object is (JEE Adv. 2013)
hydrogen spectrum is 122 nm. The smallest wavelength in (a) 0.3 × 10–17 kg ms–1 (c) 3.0 × 10–17 kg ms–1
the infrared region of the hydrogen spectrum (to the nearest (b) 1.0 × 10–17 kg ms–1 (d) 9.0 × 10–17 kg ms–1
56. If lCu is the wavelength of Ka X-ray line of copper (atomic
integer) is
number 29) and lMo is the wavelength of the Ka X-ray line
(a) 802 nm (b) 823 nm (2007) of molybdenum (atomic number 42), then the ratio lCu/lMo
(c) 1882 nm (d) 1648 nm is close to (JEE Adv. 2014)
50. Electrons with de-Broglie wavelength l fall on the target in (a) 1.99 (b) 2.14 (c) 0.50 (d) 0.48
an X-ray tube. The cut-off wavelength of the emitted X- 57. A metal surface is illuminated by light of two different
rays is (2007) wavelengths 248 nm and 310 nm. The maximum speeds of
the photoelectrons corresponding to these wavelengths are
2mcl 2 2h
(a) l0 = (b) l0 = u1 and u2, respectively. If the ratio u1 : u2 = 2 : 1 and hc =
h mc 1240 eV nm, the work function of the metal is nearly
(JEE Adv. 2014)
2m 2 c 2 l 3
(c) l0 = (d) l 0 = l (a) 3.7 eV (b) 3.2 eV (c) 2.8 eV (d) 2.5 eV
h2 58. In a historical experiment to determine Planck's constant, a
51. Which one of the following statements is WRONG in the metal surface was irradiated with light of different
context of X-rays generated from a X-ray tube? (2008) wavelengths. The emitted photoelectron energies were
(a) Wavelength of characteristic X-rays decreases when measured by applying a stopping potential. The relevant
the atomic number of the target increases. data for the wavelength (l) of incident light and the
(b) Cut-off wavelength of the continuous X-rays depends corresponding stopping potential (V0) are given below :
on the atomic number of the target
(c) Intensity of the characteristic X-rays depends on the l (mm) V0 (Volt)
electrical power given to the X-ray tube 0.3 2.0
(d) Cut-off wavelength of the continuous X-rays depends 0.4 1.0
on the energy of the electrons in the X-ray tube 0.5 0.4
EBD_7036
P-186 Topic-wise Solved Papers - PHYSICS
Given that c = 3 × 108m s–1 and e = 1.6 × 10–19 C , Planck's 4. In Bohr’s model of the hydrogen atom (1984- 2 Marks)
constant (in units of J s) found from such an experiment is (a) the radius of the nth orbit is proportional to n2
(JEE Adv. 2016) (b) the total energy of the electron in nth orbit is inversely
(a) 6.0 × 10–34 (b) 6.4 × 10–34 proportional to n
(c) 6.6 × 10–34 (d) 6.8 × 10–34 (c) the angular momentum of the electron in an orbit is an
59. The electrostatic energy of Z protons uniformly distributed h
integral multiple of
throughout a spherical nucleus of radius R is given by 2p
(d) the magnitude of potential energy of the electron in
2
3 Z (Z - 1) e any orbit is greater than its K.E.
E=
5 4πε 0 R 5. Select the correct statement from the following
(1984- 2 Marks)
The measured masses of the neutron 11 H, 15 15 (a) A diode can be used as a rectifier
7 N and 8 O are
(b) A triode cannot be used as a rectifier
1.008665 u, 1.007825 u, 15.000109 u and 15.003065 u, (c) The current in a diode is always proportional to the
respectively. Given that the radii of both the 15 15 applied voltage
7 N and 8 O
(d) The linear portion of the I–V characteristic of a triode
nuclei are same, 1 u = 931.5 Me V/c2 (c is the speed of light) is used for amplification without distortion
and e2/(4 pe0) = 1.44 MeV fm. Assuming that the difference 6. For a given plate voltage, the plate current in a triode valve
between the binding energies of 15 15 is purely due is maximum when the potential of (1985 - 2 Marks)
7 N and 8 O
(a) the grid is positive and plate is negative
to the electrostatic energy, the radius of either of the nuclei (b) the grid is zero and plate is positive
is (c) the grid is negative and plate is positive
(1 fm = 10–15 m) (JEE Adv. 2016) (d) the grid is positive and plate is positive
(a) 2.85 fm (b) 3.03 fm 7. The X-ray beam coming from an X-ray tube will be
(c) 3.42 fm (d) 3.80 fm (1985 - 2 Marks)
60. An accident in a nuclear laboratory resulted in deposition (a) monochromatic
of a certain amount of radioactive material of half-life 18 (b) having all wavelengths smaller than a certain maximum
days inside the laboratory. Tests revealed that the radiation wavelength
was 64 times more than the permissible level required for (c) having all wavelengths larger than a certain minimum
safe operation of the laboratory. What is the minimum number wavelength
of days after which the laboratory can be considered safe (d) having all wavelengths lying between a minimum and
a maximum wavelength
for use? (JEE Adv. 2016)
8. The mass number of a nucleus is (1986 - 2 Marks)
(a) 64 (b) 90
(a) always less than its atomic number
(c) 108 (d) 120 (b) always more than its atomic number
(c) sometimes equal to its atomic number
D MCQs with One or More than One Correct (d) sometimes more than and sometimes equal to its atomic
number
1. The shortest wavelength of X-rays emitted from an X-ray 9. Four physical quantities are listed in Column I. Their values
tube depends on (1982 - 3 Marks) are listed in Column II in a random order: (1987 - 2 Marks)
(a) the current in the tube Column I Column II
(b) the voltage applied to the tube (a) Thermal energy of air
(c) the nature of the gas in tube molecules at room temp (e) 0.02 eV
(d) the atomic number of the target material (b) Binding energy of heavy
2. The threshold wavelength for photoelectric emission from nuclei per nucleon (f) 2 eV
a material is 5200 Å. Photoelectrons will be emitted when (c) X-ray photon energy (g) 1k eV
this material is illuminated with monochromatic radiation (d) Photon energy of visible light (h) 7 MeV
from a (1982 - 3 Marks) The correct matching of Columns I and II is given by
(a) 50 watt infrared lamp (b) 1-watt infra-red lamp (a) a – e, b – h, c – g, d – f
(b) a – e, b – g, c – f, d – h
(c) 50 watt ultraviolet lamp (d) 1-watt ultraviolet lamp
(c) a – f, b –e, c – g, d – h
3. From the following equations pick out the possible nuclear (d) a – f, b – h, c – e, d – g.
fusion reactions (1984- 2 Marks) 10. Photoelectric effect supports quantum nature of light
(a) 6 C13 + 1H1 ® 6 C14 + 4.3MeV because (1987 - 2 Marks)
(b) 12 1 13 (a) there is a minimum frequency of light below which no
6 C + 1H ® 7 N + 2 MeV
photoelectrons are emitted
14 1 15
(c) 7 N + 1H ® 8 O + 7.3MeV (b) the maximum kinetic energy of photo electrons depends
235 only on the frequency of light and not on its intensity
(d) 92 U + 0 n1 ® 54 Xe140 + 38 Sr 94 + 0 n1
(c) even when the metal surface is faintly illuminated, the
+ 0 n1 + g + 200MeV photoelectrons leave the surface immediately
(d) electric charge of the photoelectrons is quantized
Modern Physics P-187

11. During a negative beta decay (1987 - 2 Marks) (a) the stopping potential will be 0.2 volt
(a) an atomic electron is ejected (b) the stopping potential will be 0.6 volt
(b) an electron which is already present within the nucleus (c) the saturation current will be 6.0 mA
is ejected (d) the saturation current will be 2.0 mA
(c) a neutron in the nucleus decays emitting an electron 19. In an n-p-n transistor circuit, the collector current is 10 mA.
(d) a part of the binding energy of the nucleus is converted If 90% of the electrons emitted reach the collector,
into an electron (1992 - 2 Marks)
12. During a nuclear fusion reaction (1987 - 2 Marks) (a) the emitter current will be 9 mA
(a) a heavy nucleus breaks into two fragments by itself (b) the base current will be 1 mA
(b) a light nucleus bombarded by thermal neutrons breaks (c) the emitter current will be 11 mA
(d) the base current will be – 1 mA
up
20. A star initially has 1040 deuterons. It produces energy via
(c) a heavy nucleus bombarded by thermal neutrons
2 2 3
breaks up the processes 1H +1 H ®1 H + p, and
(d) two light nuclei combine to give a heavier nucleus and 2 3 4
possibly other products 1H +1 H ® 2 He + n. If the average power radiated by
13. The potential difference applied to an X-ray tube is the star is 1016 W, the deuteron supply of the star is exhausted
increased. As a result, in the emitted radiation in a time of the order of (1993-2 Marks)
(a) the intensity increases (1988 - 2 Marks) (a) 106s. (b) 108s.
(b) the minimum wavelength increases (c) 1012 s. (d) 1016 s.
(c) the intensity remain unchanged The masses of the nuclei are as follows :
(d) the minimum wavelength decreases
14. A freshly prepared radioactive source of half life 2 hr emits M (H 2 ) = 2.014 amu;
radiation of intensity which is 64 times the permissibe safe M (p) = 1.007 amu; M(n) = 1.008 amu; M (He4 ) = 4.001amu.
level. The minimum time after which it would be possible to
21. When photons of energy 4.25 eV strike the surface of metal
work safely with this source is (1988 - 2 Marks)
A, the ejected photoelectrons have maximum kinetic energy,
(a) 6 hr (b) 12 hr
TA eV and de Broglie wavelength lA. The maximum kinetic
(c) 24 hr (d) 128 hr
energy of photoelectrons liberated from another metal B by
15. The impurity atoms with which pure silicon should be doped
to make a p-type semiconductor are those of (1988 - 2 Marks) photons of energy 4.70 eV is TB = (TA - 1.50) eV. If the de
(a) phosphorus (b) boron Broglie wavelength of these photoelectr ons is
(c) antimony (d) aluminium l B = 2l A , then (1994 - 2 Marks)
16. Two identical p-n junctions may be connected in series with
(a) The work function of A is 2.25 eV
a battery in three ways, fig. The potential drops across the
(b) The work function of B is 4.20 eV
two p – n junctions are equal in (1989 - 2 Marks)
(c) TA = 2.00 eV
+ – – (d) TB = 2.75 eV
p n n +
p
+ –
p n
+ –
p n
22. Which of the following statement(s) is (are) correct?
(1994 - 2 Marks)
(a) The rest mass of a stable nucleus is less than the sum
Circuit 1 Circuit 2 of the rest masses of its separated nucleons
– +
n p
– +
n p
(b) The rest mass of a stable nucleus is greater than the
sum of the rest masses of its separated nucleons
(c) In nuclear fission, energy is released by fusing two
nuclei of medium mass (approximately 100 amu)
Circuit 3 (d) In nuclear fission, energy is released by fragmentation
(a) circuit 1 and circuit 2 (b) circuit 2 and circuit 3 of a very heavy nucleus
(c) circuit 3 and circuit 1 (d) circuit 1 only 23. Holes are charge carriers in (1996 - 2 Marks)
17. The decay constant of a radioactive sample is l . The half- (a) intrinsic semiconductors(b) ionic solids
life and mean-life of the sample are respectively given by (c) p-type semiconductors (d) metals
(1989 - 2 Marks) 24. A transistor is used in the common emitter mode as an
(a) 1/ l and (ln 2)/ l (b) (ln 2)/ l and 1/ l amplifier. Then (1998S - 2 Marks)
(a) the base-emitter junction is forward-biased
(c) l (ln 2) and 1/ l (d) l /(ln 2) and 1/ l
(b) the base-emitter junction is reverse-biased
18. When a monochromatic point source of light is at a distance
(c) the input signal is connected in series with the voltage
of 0.2 m from a photoelectric cell, the cut off voltage and the
applied to bias the base-emitter junction
saturation current are respectively 0.6 V and 18.0 mA. If the
(d) the input signal is connected in series with the voltage
same source is placed 0.6 m away from the photoelectric
applied to bias the base-collector junction
cell, then (1992 - 2 Marks)
EBD_7036
P-188 Topic-wise Solved Papers - PHYSICS
25. Let mp be the mass of a proton, mn the mass of a neutron,
æ1ö
M1 the mass of a 20 nucleus and M2 the mass of a 32. The graph between the stopping potential (V0) and ç ÷ is
10 Ne
èlø
40 nucleus. Then (1998S - 2 Marks)
20 Ca
shown in the figure. f1 , f 2 and f3 are work functions,
(a) M 2 = 2 M1 (b) M 2 > 2 M1 which of the following is/are correct (2006 - 5M, –1)
(c) M 2 < 2 M1 (d) M1 < 10 ( mn + m p )
V0
26. The electron in a hydrogen atom makes a transition n1 ® n2
where n1 and n2 are the principal quantum numbers of the Metal 1 Metal 2 Metal 3
two states. Assume the Bohr model to be valid. The time
period of the electron in the initial state is eight times that in f1 f2 f3
the final state. The possible values of n1 and n2 are
(1998S - 2 Marks) 1 (nm–1)
(a) n1 = 4, n2 = 2 (b) n1 = 8, n2 = 2 0.001 0.002 0.004 l
(c) n1 = 8, n2 = 1 (d) n1 = 6, n2 = 3 (a) f1 : f2 : f3 = 1 : 2 : 4
27. The half-life of I is 8 days. Given a sample of 131I at time
131

t = 0, we can assert that (1998S - 2 Marks) (b) f1 : f2 : f3 = 4 : 2 : 1


(a) no nucleus will decay before t = 4 days
hc
(b) no nucleus will decay before t = 8 days (c) tan q µ
(c) all nuclei will decay before t = 16 days e
(d) a given nucleus may decay at any time after t = 0 (d) ultravioletlight can be used to emit photoelectrons from
28. In a p-n junction diode not connected to any circuit, metal 2 and metal 3 only
(1998S - 2 Marks) 33. Assume that the nuclear binding energy per nucleon (B/A)
(a) the potential is the same everywhere versus mass number (A) is as shown in the figure. Use this
(b) the p-type side is at a higher potential than the n-type plot to choose the correct choice(s) given below. (2008)
side B/A
(c) there is an electric field at the junction directed from
the n-type side to the p-type side
(d) there is an electric field at the junction directed from 8
the p-type side to the n-type side 6
29. X-rays are produced in an X-ray tube operating at a given 4
accelerating voltage. The wavelength of the continuous 2
X-rays has values from (1998S - 2 Marks) 0 A
(a) 0 to ¥ 100 200
(a) Fusion of two nuclei with mass numbers lying in the
(b) l min to ¥ where l min > 0 range of 1 < A < 50 will release energy.
(c) 0 to l max where l max < ¥ (b) Fusion of two nuclei with mass numbers lying in the
range of 51 < A < 100 will release energy
(d) l min to l max where 0 < l min < l max < ¥ (c) Fission of a nucleus lying in the mass range of 100 < A
< 200 will release energy when broken into two equal
30. The work function of a substance is 4.0 eV. The longest
fragments
wavelength of light that can cause photoelectron emission
(d) Fission of a nucleus lying in the mass range of 200 < A
from this substance is approximately (1998S - 2 Marks)
< 260 will release energy when broken into two equal
(a) 540 nm (b) 400 nm fragments
(c) 310 nm (d) 220 nm 34. The radius of the orbit of an electron in a Hydrogen-like
31. The half-life period of a radioactive element X is same as the atom is 4.5 a0, where a0 is the Bohr radius. Its orbital angular
mean-life time of another radioactive element Y. Initially both
3h
of them have the same number of atoms. Then momentum is . It is given that h is Planck constant and R
(1999S - 3 Marks) 2p
(a) X and Y have the same decay rate initially is Rydberg constant. The possible wavelength(s), when the
atom de-excites, is (are) (JEE Adv. 2013)
(b) X and Y decay at the same rate always
(c) Y will decay at a faster rate than X 9 9 9 4
(a) (b) (c) (d)
(d) X will decay at a faster rate than Y 32R 16R 5R 3R
Modern Physics P-189

35. For photo-electric effect with incident photon wavelength l, (a) le decreases with increase in f and lph
the stopping potential is V0. Identify the correct variation(s) (b) le is approximately halved, if d is doubled
of V0 with l and 1/l. (JEE Adv. 2015) (c) For large potential difference (V >> f/e), le is
approximately halved if V is made four times
V V0
0 (d) le increases at the same rate as lph for lph < hc/f

(a) (b)
E Subjective Problems
l 1. A single electron orbits around a stationary nucleus of
l
V0 charge + Ze. Where Z is a constant and e is the magnitude
(c) (d) V0 of the electronic charge. It requires 47.2 eV to excite the
electron from the second Bohr orbit to the third Bohr orbit.
Find (1981- 10 Marks)
1/l 1/l
(i) The value of Z.
236
36. A fission reaction is given by 92 U ® 140
54 Xe +
94
38 Sr + x + y, (ii) The energy required to excite the electron from the
236
third to the fourth Bohr orbit.
where x and y are two particles. Considering 92 U to be at (iii) The wavelength of the electromagnetic radiation
rest, the kinetic energies of the products are denoted by required to remove the electron from the first Bohr
KXe, KSr, Kx(2 MeV) and Ky(2 MeV), respectively. Let the orbit to infinity.
236 140 94 (iv) The kinetic energy, potential energy, potential energy
binding energies per nucleon of 92 U, 54 Xe and 38 Sr be
and the angular momentum of the electron in the first
7.5 MeV, 8.5 MeV and 8.5 MeV, respectively. Considering Bohr orbit.
different conservation laws, the correct option(s) is(are) (v) The radius of the first Bohr orbit.
(JEE Adv. 2015) (The ionization energy of hydrogen atom = 13.6 eV, Bohr
(a) x = n, y = n, KSr = 129 MeV, KXe = 86 MeV radius = 5.3 × 10–11 metre, velocity of light = 3 × 108 m/sec.
(b) x = p, y = e–, KSr = 129 MeV, KXe = 86 MeV Planck’s constant = 6.6 × 10–34 joules - sec).
(c) x = p, y = n, KSr = 129 MeV, KXe = 86 MeV 2. Hydrogen atom in its ground state is excited by means of
(d) x = n, y = n, KSr = 86 MeV, KXe = 129 MeV monochromatic radiation of wavelength 975Å. How many
37. Highly excited states for hydrogen–like atoms (also called different lines are possible in the resulting spectrum ?
Rydberg states) with nuclear charge Ze are defined by their Calculate the longest wavelength amongst them. You may
principal quantum number n, where n>>1. Which of the assume the ionization energy for hydrogen atom as 13.6 eV.
following statement(s) is(are) true? (JEE Adv. 2016) (1982 - 5 Marks)
(a) Relative change in the radii of two consecutive orbitals 3. How many electron, protons and neutrons are there in a
does not depend on Z nucleus of atomic number 11 and mass number 24 ?
(b) Relative change in the radii of two consecutive orbitals (1982 - 2 Marks)
varies as 1/n (i) number of electrons = (ii) number of protons =
(iii) number of neutrons =
(c) Relative change in the energy of two consecutive
4. The ionization energy of a hydrogen like Bohr atom is 4
orbitals varies as 1/ n 3
Rydbergs. (i) What is the wavelength of the radiation
(d) Relative change in the angular momenta of two
emitted when the electron jumps from the first excited state
consecutive orbitals varies as 1/n to the ground state ? (ii) What is the radius of the first orbit
38. Light of wavelength lph falls on a cathode plate inside a for this atom? (1984- 4 Marks)
vacuum tube as shown in the figure. The work function of 5. A double ionised Lithium atom is hydrogen-like with atomic
the cathode surface is f and the anode is a wire mesh of number 3. (1985 - 6 Marks)
conducting material kept at a distance d from the cathode. A (i) Find the wavelength of the radiation required to excite
potential difference V is maintained between the electrodes. the electron in Li++ from the first to the third Bohr
If the minimum de Broglie wavelength of the electrons orbit. (Ionisation energy of the hydrogen atom equals
passing through the anode is le, which of the following 13.6 eV.)
statement(s) is (are) true? (JEE Adv. 2016) (ii) How many spectral lines are observed in the emission
spectrum of the above excited system?
Light
6. A triode has plate characteristics in the form of parallel
lines in the region of our interest. At a grid voltage of – 1
volt the anode current I (in milli amperes) is given in terms
Electrons of plate voltage V (in volts) by the algebraic relation :
I = 0.125V – 7.5
For grid voltage of – 3 volts, the current at anode voltage
of 300 volts is 5 milliampere. Determine the plate resistance
V
– + (rp), transconductance (gm) and the amplification factor (µ)
for the triode. (1987 - 7 Marks)
EBD_7036
P-190 Topic-wise Solved Papers - PHYSICS
7. A particle of charge equal to that of an electron, – e, and (a) Calculate the photon flux at the centre of the screen
mass 208 times the mass of the electron (called a mu-meson) and the photocurrent in the detector.
moves in a circular orbit around a nucleus of charge + 3e. (b) If the concave lens L of focal length 0.6 m is inserted in
(Take the mass of the nucleus to be infinite). Assuming the aperture as shown, find the new values of photon
that the Bohr model of the atom is applicable to this system.
flux and photocurrent. Assume a uniform average
(1988 - 6 Marks)
(i) Derive an expression for the radius of the nth Bohr transmission of 80% from the lens.
orbit. (c) If the work function of the photoemissive surface is
(ii) Find the value of n for which the radius of the orbit is 1eV, calculate the values of the stopping potential in
approximately the same as that of the first Bohr orbit the two cases (without and with the lens in the
for the hydrogen atom. aperture).
(iii) Find the wavelength of the radiation emitted when the 12. A nucleus X, initially at rest, undergoes alpha decay
mu-meson jumps from the third orbit of the first orbit. according to the equation,
8. A gas of identical hydrogen-like atoms has some atoms in
A
the lowest (ground) energy level A and some atoms in a 92 X ® 228
Z Y +a (1991 - 4 + 4 Marks)
particular upper (excited) energy level B and there are no
atoms in any other energy level. The atoms of the gas make (a) Find the values of A and Z in the above process.
transition to a higher energy level by absorbing (b) The alpha particle produced in the above process is
monochromatic light of photon energy 2.7 eV. Subsequently, found to move in a circular track of radius 0.11 m in a
the atoms emit radiation of only six different photon uniform magnetic field of 3 Tesla. Find the energy (In
energies. Some of the emitted photons have energy 2.7 eV, MeV) released during the process and the binding
some have energy more and some have less than 2.7 eV. energy of the parent nucleus X.
(1989 - 8 Marks)
(i) Find the principal quantum number of the initially
excited level B.
( ) 1
Given that : m(Y) = 228.03 u; m 0 n = 1.009 u.

(iii) Find the ionization energy for the gas atoms. m ( He ) = 4.003 u; m ( H ) = 1.008 u
4
2
1
1
(iii) Find the maximum and the minimum energies of the
emitted photons. 13. Light from a discharge tube containing hydrogen atoms falls
9. Electrons in hydrogen like atom (Z = 3) make transitions on the surface of a piece of sodium. The kinetic energy of the
from the fifth to the fourth orbit and from the fourth to the fastest photoelectrons emitted from sodium is 0.73 eV. The
third orbit. The resulting radiations are incident normally work function for sodium is 1.82 eV. Find (1992 - 10 Marks)
on a metal plate and eject photoelectrons. The stopping (a) the energy of the photons causing the photoelectric
potential for the photoelectrons ejected by the shorter emission,
wavelength is 3.95 volts. Calculate the work function of the
(b) the quantum numbers of the two levels involved in
metal and the stopping potential for the photoelectrons
ejected by the longer wavelength. (1990 - 7 Marks) the emission of these photons,
(c) the change in the angular momentum of the electron in
(Rydberg constant = 1.094 × 10 7 m -1 ) the hydrogen atom in the above transition, and
10. It is proposed to use the nuclear fusion reaction (d) the recoil speed of the emitting atom assuming it to be
2 2
1 H +1 H ® 42 He (1990 - 8 Marks) at rest before the transition.
in a nuclear reactor of 200 MW rating. If the energy from (Ionization potential of hydrogen is 13.6 eV)
the above reaction is used with a 25 per cent efficiency in 14. A small quantity of solution containing Na24 radio nuclide
the reactor, how many grams of deuterium fuel will be needed (half life = 15 hour) of activity 1.0 microcurie is injected into
per day. (The masses of 12 H and 42 He are 2.0141 atomic the blood of a person. A sample of the blood of volume 1
cm 3 taken after 5 hour show an activity of 296
mass units and 4.0026 atomic mass units respectively)
disintegrations per minute. Determine the total volume of
11. A monochromatic point source radiating wavelength 6000
Å, with power 2 watt, an aperture A of diameter 0.1 m and a the blood in the body of the person. Assume that
large screen SC are placed as shown in fig. A photoemissive radioactive solution mixes uniformly in the blood of the
detector D of surface area 0.5 cm2 is placed at the centre of person. (1 curie = 3.7 ×1010 disintegrations per second)
the screen. The efficiency of the detector for the (1994 - 6 Marks)
photoelectron generation per incident photon is 0.9. 15. A hydrogen like atom (atomic number Z) is in a higher excited
(1991 - 2 + 4 + 2 Marks) state of quantum number n. The excited atom can make a
SC transition to the first excited state by successively emitting
A two photons of energy 10.2 and 17.0 eV respectively.
Alternately, the atom from the same excited state can make
S a transition to the second excited state by successively
D
L emitting two photons of energies 4.25 eV and 5.95 eV
0.6 m respectively. (1994 - 6 Marks)
Determine the values of n and Z. (Ionization energy of H-
6m atom = 13.6 eV)
Modern Physics P-191

16. An electron, in a hydrogen-like atom, is in an excited state. 24. In a nuclear reaction 235U undergoes fission liberating 200
It has a total energy of –3.4 eV. Calculate (i) the kinetic MeV of energy. The reactor has a 10% efficiency and
energy and (ii) the de Broglie wavelength of the electron. produces 1000 MW power. If the reactor is to function for
(1996 - 3 Marks) 10 years, find the total mass of uranium required.
17. At a given instant there are 25% undecayed radio-active (2001 - 5 Marks)
nuclei in a sample. After 10 seconds the number of 25. A nucleus at rest undergoes a decay emitting an a particle
undecayed nuclei reduces to 12.5%. Calculate (i) mean-life of de-Broglie wavelength l = 5.76 ´ 10-15 m. If the mass of
of the nuclei, and (ii) the time in which the number of the daughter nucleus is 223.610 amu and that of the a
undecayed nuclei will further reduce to 6.25% of the particles is 4.002 amu, determine the total kinetic energy in
reduced number. (1996 - 3 Marks) the final state. Hence, obtain the mass of the parent nucleus
18. Assume that the de Broglie wave associated with an electron in amu. (1 amu = 931.470 MeV/c2) (2001-5 Marks)
can form a standing wave between the atoms arranged in a 26. A radioactive nucleus X decays to a nucleus Y with a decay
one dimensional array with nodes at each of the atomic constant lx = 0.1 s-1. Y further decays to a stable nucleus Z
sites. It is found that one such standing wave is formed if with a decay constant lY = 1/30 s-1. Initially, there are only
the distance d between the atoms of the array is 2Å. A X nuclei and their number is No = 1020. Set up the rate
similar standing wave is again formed if d is increased to equations for the populations of X, Y and Z. The population
2.5 Å but not for any intermediate value of d. Find the of Y nucleus as a function of time is given by NY(t) = (NolX/
energy of the electrons in electron volts and the least value (lX - lY)){exp(-lYt)-exp(-lxt)}. Find the time at which NY is
of d for which the standing wave of the type described maximum and determine the populations X and Z at that
above can form. (1997 - 5 Marks) instant. (2001-5 Marks)
19. The element Curium 248 13 27. A hydrogen-like atom (described by the Bohr model) is
96 Cm has a mean life of 10 seconds.
Its pirmary decay modes are spontaneous fission and a- observed to emit six wavelengths, originating from all
decay, the former with a probability of 8% and the latter possible transitions between a group of levels. These levels
with a probability of 92%. Each fission releases 200 MeV of have energies between -0.85 eV and -0.544 eV (including
energy. The masses involved in a-decay are as follows: both these values). (2002 - 5 Marks)
(a) Find the atomic number of the atom.
248 244
96 Cm = 248.072220u, 94 Pu = 244.064100u and (b) Calculate the smallest wavelength emitted in these
4 transitions.
2 He=4.002603 u. Calculate the power output from a sample
(Take hc = 1240 eV-nm, ground state energy of hydrogen
of 1020 Cm atoms. (1 u = 931 MeV/c2.) (1997 - 5 Marks) atom = –13.6 eV)
20. Nuclei of a radioactive element A are being produced at a 28. Two metallic plates A and B, each of area 5 × 10–4 m2, are
constant rate a. The element has a decay constant l. At placed parallel to each other at a separation of 1 cm. Plate B
time t = 0, there are N0 nuclei of the element. carries a positive charge of 33.7 × 10–12 C. A monochromatic
(1998 - 8 Marks) beam of light, with photons of energy 5 eV each, starts
(a) Calculate the number N of nuclei of A at time t. falling on plate A at t = 0 so that 1016 photons fall on it per
(b) If a = 2N0l, calculate the number of nuclei of A after square meter per second. Assume that one photoelectron
one half-life of A, and also the limiting value of N as is emitted for every 106 incident photons. Also assume that
t ®¥. all the emitted photoelectrons are collected by plate B and
21. Photoelectrons are emitted when 40 nm radiation is incident the work function of plate A remains constant at the value
on a surface of work function 1.9 eV. These photoelectrons 2eV. Determine (2002 - 5 Marks)
pass through a region containing a-particles. A maximum (a) the number of photoelectrons emitted up to t = 10 s,
energy electron combines with an a–particle to form a He+ (b) the magnitude of the electric field between the plates
ion, emitting a single photon in this process. He+ ions thus A and B at t = 10 s, and
formed are in their fourth excited state. Find the energies in (c) the kinetic energy of the most energetic photoelectron
eV of the photons, lying in the 2 to 4 eV range, that are emitted at t = 10 s when it reaches plate B.
likely to be emitted during and after the combination. Neglect the time taken by the photoelectron to reach plate
[Take h = 4.14×10–15 eV.s.] (1999 - 5 Marks) B. Take e0 = 8.85 × 10–12 C2/N-m2
22. A hydrogen-like atom of atomic number Z is in an excited 29. Frequency of a photon emitted due to transition of electron
state of quantum number 2n. It can emit a maximum energy of a certain element from L to K shell is found to be
photon of 204 eV. If it makes a transition to quantum state 4.2 × 1018 Hz. Using Moseley’s law, find the atomic number
n, a photon of energy 40.8 eV is emitted. Find n, Z and the of the element, given that the Rydberg’s constant R = 1.1 ×
ground state energy (in eV) for this atom. Also calculate 107 m–1. (2003 - 2 Marks)
the minimum energy (in eV) that can be emitted by this 30. A radioactive sample emits n b-particles in 2 sec. In next 2
atom during de-excitation. Ground state energy of hydrogen sec it emits 0.75 n b-particle, what is the mean life of the
atom is –13.6 eV. (2000 - 6 Marks) sample? (2003 - 2 Marks)
23. When a beam of 10.6 eV photons of intensity 2.0 W/m 2 falls 31. In a photoelectric experiment set up, photons of energy 5
on a platinum surface of area 1.0 × 10-4 m2 and work function eV falls on the cathode having work function 3 eV. (a) If the
5.6 eV, 0.53% of the incident photons eject photoelectrons. saturation current is iA = 4mA for intensity 10–5 W/m2, then
Find the number of photoelectrons emitted per second and plot a graph between anode potential and current. (b) Also
their minimum and maximum energies (in eV). Take 1eV = draw a graph for in tensity of incident r adiation
1.6 × 10-19 J. (2000 - 4 Marks) 2 × 10–5 W/m2. (2003 - 2 Marks)
EBD_7036
P-192 Topic-wise Solved Papers - PHYSICS
32. A radioactive sample of 238U decays to Pb through a process l1 and l2 are the de-Broglie wavelengths of the particle,
for which the half-life is 4.5×109 years. Find the ratio of when 0 £ x £ 1 and x > 1 respectively. If the total energy of
number of nuclei of Pb to 238U after a time of 1.5×109 years. particle is 2E0, find l1 / l2. (2005 - 2 Marks)
Given (2)1/3 = 1.26. (2004 - 2 Marks) 35. Highly energetic electrons are bombarded on a target of an
element containing 30 neutrons. The ratio of radii of nucleus
33. The photons from the Balmer series in Hydrogen spectrum
to that of Helium nucleus is (14)1/3. Find (a) atomic number
having wavelength between 450 nm to 700 nm are incident
of the nucleus. (b) the frequency of Ka line of the X-ray
on a metal surface of work function 2 eV. Find the maximum
produced. (R = 1.1 × 107 m–1 and c = 3 × 108 m/s)
kinetic energy of ejected electron. (Given hc = 1242 eV nm)
(2005 - 4 Marks)
(2004 - 4 Marks)
36. In hydrogen-like atom (z = 11), nth line of Lyman series has
34. The potential energy of a particle of mass m is given by
wavelength l. The de-Broglie's wavelength of electron in
ì E ; 0 £ x £ 1ü the level from which it originated is also l. Find the value of
V ( x) = í 0 ý n? (2006 - 6M)
î 0; x >1 þ

F Match the Following

DIRECTIONS (Q. No. 1 to 4 & 6) : Each question contains statements given in two columns, which p q r s t
have to be matched. The statements in Column-I are labelled A, B, C and D, while the statements in
A p q r s t
Column-II are labelled p, q, r and s. Any given statement in Column-I can have correct matching with
B p q r s t
ONE OR MORE statement(s) in Column-II. The appropriate bubbles corresponding to the answers to
these questions have to be darkened as illustrated in the following example : C p q r s t
If the correct matches are A-p, s and t; B-q and r; C-p and q; and D-s then the correct darkening of D p q r s t
bubbles will look like the given.
1. In the following, column I lists some physical quantities and the column II gives approximate energy values associated with
some of them. Choose the appropriate value of energy from column II for each of the physical quantities in column I and write
the corresponding letter p, q, r, etc. against the number (A), (B), (C), (D) etc. of the physical quantity in the answer book. In your
answer, the sequence of column I should be maintained. (1997 - 4 Marks)
Column I Column II
(A) Energy of thermal neutrons (p) 0.025 eV
(B) Energy of X-rays (q) 0.5 eV
(C) Binding energy per nucleon (r) 3 eV
(D) Photoelectric threshold of a metal (s) 20 eV
(t) 10 k eV
(u) 8 M eV
2. Given below are certain matching type questions, where two columns (each having 4 items) are given. Immediately after the
columns the matching grid is given, where each item of Column I has to be matched with the items of Column II, by encircling
the correct match(es). Note that an item of column I can match with more than one item of column II. All the items of column II
must be matched. Match the following : (2006 - 6M)
Column I Column II
(A) Nuclear fusion (p) Converts some matter into energy
(B) Nuclear fission (q) Generally possible for nuclei with low atomic
number
(C) b-decay (r) Generally possible for nuclei with higher atomic
number
(D) Exothermic nuclear reaction (s) Essentially proceeds by weak nuclear forces
3. Some laws / processes are given in Column I. Match these with the physical phenomena given in Column II and indicate your
answer by darkening appropriate bubbles in the 4 × 4 matrix given in the ORS. (2007)
Column I Column II
(A) Transition between two atomic energy levels (p) Characteristic X-rays
(B) Electron emission from a material (q) Photoelectric effect
(C) Mosley's law (r) Hydrogen spectrum
(D) Change of photon energy into kinetic energy of electrons (s) b-decay
Modern Physics P-193

4. Column-II gives certain systems undergoing a process. Column-I suggests changes in some of the parameters related to the
system. Match the statements in Column-I to the approapriate process(es) from Column-II. (2009)
Column-I Column-II
(A) The energy of the system is increased (p) System : A capacitor, initially uncharged
Process : It is connected to a battery
(B) Mechanical energy is provided to the system, which is (q) System : A gas in an adiabatic container fitted with
converted into energy of random motion of its parts an adiabatic piston
Process: The gas is compressed by pushing the
piston
(C) Internal energy of the system is converted into its (r) System: A gas in a rigid container
mechanical energy Process: The gas gets cooled due to colder
atmosphere surrounding it
(D) Mass of the system is decreased (s) System: A heavy nucleus, initially at rest
Process: The nucleus fissions into two fragments
of nearly equal masses and some neutrons are
emitted
(t) System: A resistive wire loop
Process: The loop is placed in a time varying
magnetic field perpendicular to its plane

DIRECTION (Q.No. 5) : Following question has matching list I and II. The codes for the lists have choices (a), (b), (c) and (d) out
of which ONLY ONE is correct.

5. Match List I of the nuclear processes with List II containing parent nucleus and one of the end products of each process and
then select the correct answer using the codes given below the lists: (JEE Adv. 2013-II)
List I List II
15 15
P. Alpha decay 1. 8 O ® 7 O + ...

b+ decay 138 234


Q. 2. 92 U ® 90 Th + ...

185 184
R. Fission 3. 83 Bi ® 82 Pb + ...

239
S. Proton emission 4. 94 Pu ® 140
57 La + ...
Codes:
P Q R S
(a) 4 2 1 3
(b) 1 3 2 4
(c) 2 1 4 3
(d) 4 3 2 1
6. Match the nuclear processes given in column I with the appropriate option(s) in column II. (JEE Adv. 2015)
Column I Column II

(A) Nuclear fusion (p) Absorption of thermal neutrons by 235


92 U

(B) Fission in a nuclear reactor (q) 60 Co nucleus


27

(C) b-decay (r) Energy production in stars via hydrogen


conversion to helium
(D) g-ray emission (s) Heavy water
(t) Neutrino emission
EBD_7036
P-194 Topic-wise Solved Papers - PHYSICS
(a) 1.0 × 109 K < T < 2.0 × 109 K
G Comprehension Based Questions (b) 2.0 × 109 K < T < 3.0 × 109 K
(c) 3.0 × 109 K < T < 4.0 × 109 K
PASSAGE-I (d) 4.0 × 109 K < T < 5.0 × 109 K
In a mixture of H-He+ gas (He+ is singly ionized He atom), H 6. Results of calculations for four different designs of a fusion
atoms and He+ ions are excited to their respective first excited reactor using D-D reaction are given below. Which of these
states. Subsequently, H atoms transfer their total excitation energy is most promising based on Lawson criterion?
to He+ ions (by collisions). Assume that the Bohr model of atom (a) deuteron density = 2.0 × 1012 cm–3,
is exactly valid. (2008) confinement time = 5.0 × 10–3 s
1. The quantum number n of the state finally populated in He+ (b) deuteron density = 8.0 × 1014 cm–3,
ions is – confinement time = 9.0 × 10–1 s
(a) 2 (b) 3 (c) deuteron density = 4.0 × 1023 cm–3,
(c) 4 (d) 5 confinement time = 1.0 × 10–11 s
2. The wavelength of light emitted in the visible region by He+ (d) deuteron density = 1.0 × 1024 cm–3,
ions after collisions with H atoms is – confinement time = 4.0 × 10–12 s
(a) 6.5 × 10–7 m PASSAGE-3
(b) 5.6 × 10–7 m When a particle is restricted to move along x - axis between x = 0
(c) 4.8 × 10–7 m and x = a, where a is of nanometer dimension, its energy can take
(d) 4.0 × 10–7 m only certain specific values. The allowed energies of the particle
3. The ratio of the kinetic energy of the n = 2 electron for the H moving in such a restricted region, correspond to the formation of
standing waves with nodes at its ends x = 0 and x = a . The
atom to that of He+ ion is –
wavelength of this standing wave is related to the linear momentum
(a) 1/4 (b) 1/2 p of the particle according to the de Broglie relation. The energy
(c) 1 (d) 2 of the particle of mass m is related to its linear momentum as
PASSAGE-2
Scientists are working hard to develop nuclear fusion reactor. p2
E= . Thus, the energy of the particle can be denoted by a
2m
Nuclei of heavy hydrogen, 12 H, known as deuteron and denoted
quantum number ‘n’ taking values1, 2, 3, ... (n = 1, called the ground
by D, can be thought of as a candidate for fusion reactor. The state) corresponding to the number of loops in the standing wave.
D-D reaction is 12 H + 12 H ®32 He + n + energy. In the core of Use the model described above to answer the following three
questions for a particle moving in the line x = 0 to x = a. Take h =
fusion reactor, a gas of heavy hydrogen is fully ionized into 6.6 × 10–34 J s and e = 1.6 × 10–19 C.
deuteron nuclei and electrons. This collection of 12 H nuclei and 7. The allowed energy for the particle for a particular value of
n is proportional to (2009)
electrons is known as plasma. The nuclei move randomly in the (a) a–2 (b) a–3/2
reactor core and occasionally come close enough for nuclear (c) a–1 (d) a2
fusion to take place. Usually, the temperatures in the reactor core 8. If the mass of the particle is m = 1.0 × 10–30 kg and a = 6.6
are too high and no material wall can be used to confine the plasma. nm, the energy of the particle in its ground state is closest
Special techniques are used which confine the plasma for a time t0 to (2009)
before the particles fly away form the core. If n is the density (a) 0.8 meV (b) 8 meV
(number/volume) of deuterons, the product nt0 is called Lawson (c) 80 meV (d) 800 meV
number. In one of the criteria, a reactor is termed successful if 9. The speed of the particle, that can take discrete values, is
Lawson number is greater than 5 × 1014 s/cm3. proportional to (2009)
It may be helpful to use the following: Boltzmann constant (a) n–3/2 (b) n–1
(c) n1/2 (d) n
e2
k = 8.6 × 10 – 5 eV/K; = 1.44 × 10–9 eVm (2009) PASSAGE-4
4pe 0
The key feature of Bohr’s theory of spectrum of hydrogen atom is
4. In the core of nuclear fusion reactor, the gas becomes plasma the quantization of angular momentum when an
because of electron is revolving around a proton. We will extend this to a
(a) strong nuclear force acting between the deuterons general rotational motion to find quantized rotational
(b) coulomb force acting between the deuterons energy of a diatomic molecule assuming it to be rigid. The rule to
(c) coulomb force acting between deuteron-electron pairs be applied is Bohr’s quantization condition. (2010)
(d) the high temperature maintained inside the reactor core 10. A diatomic molecule has moment of inertia I. By Bohr’s
5. Assume that two deuteron nuclei in the core of fusion reactor quantization condition its rotational energy in the nth level
at temperature T are moving towards each other, each with (n = 0 is not allowed) is
kinetic energy 1.5 kT, when the separation between them is 1 æ h2 ö 1 æ h2 ö
large enough to neglect coulomb potential energy. Also (a) ç ÷ (b) n ç 2 ÷
n2 è 8p 2 I ø è 8p I ø
neglect any interaction from other particles in the core. The
minimum temperature T required for them to reach a æ h ö
2
2
æ h2 ö
(c) n ç 2 ÷ (d) n ç 2 ÷
separation of 4 × 10–15 m is in the range è 8p I ø è 8p I ø
Modern Physics P-195

11. It is found that the excitation frequency from ground to the known as the binding energy of the nucleus. A heavy nucleus of
first excited state of rotation for the CO molecule is close mass M can break into two light nuclei of masses m 1 and m2 only
if (m1 + m2) < M. Also two light nuclei of masses m3 and m4 can
4
to ´ 1011 Hz. Then the moment of inertia of CO molecule undergo complete fusion and form a heavy nucleus of mass M'
p only if (m3 + m4) > M'. The masses of some neutral atoms are
about its center of mass is close to given in the table below:
(Take h = 2p ´ 10 –34 J s) 1 1.007825 u 2 2.014102 u
1H 1H
–46 2 –46 2
(a) 2.76 ´ 10 kg m (b) 1.87 ´ 10 kg m 3 4
1H 3.016050 u 2 He 4.002603 u
–47 2 –47 2
(c) 4.67 ´ 10 kg m (d) 1.17 ´ 10 kg m 6 7
3 Li 6.015123 u 3 Li 7.016004 u
12. In a CO molecule, the distance between C (mass = 12 a.m.u.)
70 69.925325 u 82 81.916709 u
5 30 Zn 34 Se
and O (mass = 16 a.m.u.), where 1 a.m.u. = ´ 10 –27 kg , is
3 152 206
64 Gd 151.919803 u 82 Pb 205.974455 u
close to
209 208.980388 u 210 209.982876 u
(a) 2.4 ´ 10 –10 m (b) 1.9 ´ 10 –10 m 83 Bi 84 Po
(1u = 932 MeV/c2)
(c) 1.3 ´ 10 –10 m (d) 4.4 ´ 10 –11 m
(JEE Adv. 2013)
PASSAGE-5 15. The kinetic energy (in keV) of the alpha particle, when the
210
nucleus 84 Po at rest undergoes alpha decay, is
The b -decay process, discovered around 1900, is basically the
decay of a neutron (n). In the laboratory, a proton (p) and an (a) 5319 (b) 5422
electron (e–) are observed as the decay products of the neutron. (c) 5707 (d) 5818
Therefore, considering the decay of a neutron as a two-body 16. The correct statement is
decay process, it was predicted theoretically that the kinetic energy (a) The nucleus 63 Li can emit an alpha particle
of the electron should be a constant. But experimentally, it was
observed that the electron kinetic energy has continuous (b) The nucleus 210 can emit a proton
84 Po
spectrum. Considering a three-body decay process, i.e.
(c) Deuteron and alpha particle can undergo complete
n ® p + e - + n e , around 1930, Pauli explained the observed fusion
electron energy spectrum. Assuming the anti-neutrino (ne ) to be (d) The nuclei 70 and 82 can undergo complete
30 Zn 34 Se
massless and possessing negligible energy, and the neutron to fusion
be at rest, momentum and energy conservation principles are
applied. From this calculation, the maximum kinetic energy of the H Assertion & Reason Type Questions
electron is 0.8 × 106 eV. The kinetic energy carried by the proton is
1. STATEMENT-1 (2007)
only the recoil energy.
If the accelerating potential in an X-ray tube is increased,
13. If the anti-neutrino had a mass of 3 eV/c2 (where c is the
the wavelengths of the characteristic X-rays do not change.
speed of light) instead of zero mass, what should be the STATEMENT-2
range of the kinetic energy, K, of the electron? (2012) When an electron beam strikes the target in an X-ray tube,
(a) 0 £ K £ 0.8 ´ 106 eV part of the kinetic energy is converted into X-ray energy.
6
(b) 3.0 eV £ K £ 0.8 ´ 10 eV (a) Statement-1 is True, Statement-2 is True; Statement-2 is
(c) 3.0 eV £ K < 0.8 ´ 10 eV
6 a correct explanation for Statement-1
(b) Statement-1 is True, Statement-2 is True; Statement-2
(d) 0 £ K < 0.8 ´ 106 eV is NOT a correct explanation for Statement-1
14. What is the maximum energy of the anti-neutrino? (2012) (c) Statement-1 is True, Statement-2 is False
(a) Zero (d) Statement-1 is False, Statement-2 is True
(b) Much less than 0.8 × 106 eV .
(c) Nearly 0.8 × 106 eV I Integer Value Correct Type
(d) Much larger than 0.8 × 106 eV
1. An a-particle and a proton are accelerated from rest by a
PASSAGE-6 potential difference of 100 V. After this, their de Broglie
lp
The mass of a nucleus A is less than the sum of the masses of wavelengths are la and lp respectively. The ratio , to
ZX la
(A-Z) number of neutrons and Z number of protons in the nucleus.
the nearest integer, is (2010)
The energy equivalent to the corresponding mass difference is
EBD_7036
P-196 Topic-wise Solved Papers - PHYSICS
2. To determine the half life of a radioactive element, a student 8. A nuclear power plant supplying electrical power to a village
uses a radioactive material of half life T years as the fuel. The
dN (t ) dN (t )
plots a graph of ln versus t . Here is the rate amount of fuel at the beginning is such that the total power
dt dt
requirement of the village is 12.5% of the electrical power
of radioactive decay at time t. If the number of radioactive available from the plant at that time. If the plant is able to
nuclei of this element decreases by a factor of p after 4.16 meet the total power needs of the village for a maximum
years, the value of p is (2010) period of nT years, then the value of n is (JEE Adv. 2015)
9. Consider a hydrogen atom with its electron in the nth orbital.
An electromagnetic radiation of wavelength 90 nm is used
to ionize the atom. If the kinetic energy of the ejected electron
is 10.4 eV, then the value of n is (hc = 1242 eV nm)
(JEE Adv. 2015)
10. For a radioactive material, its activity A and rate of change of
dN dA
its activity R are defined as A = - and R = – , where
dt dt
3. The activity of a freshly prepared radioactive sample is 1010
disintegrations per second, whose mean life is 109 s. The N(t) is the number of nuclei at time t. Two radioactive sources
mass of an atom of this radioisotope is 10–25 kg. The mass P (mean life t) and Q (mean life 2t) have the same activity at
(in mg) of the radioactive sample is (2011) t = 0. Their rates of change of activities at t = 2t are RP and
4. A silver sphere of radius 1 cm and work function 4.7 eV is
suspended from an insulating thread in freespace. It is under RP n
RQ, respectively. If = , then the value of n is
continuous illumination of 200 nm wavelength light. As RQ e
photoelectrons are emitted, the sphere gets charged and acquires
a potential. The maximum number of photoelectrons emitted from (JEE Adv. 2015)
the sphere is A ´ 10z (where 1 < A < 10). The value of ‘z’ is 11. An electron is an excited state of Li2+ ion has angular
(2011) momentum 3h/2p. The de Broglie wavelength of the electron
5. A proton is fired from very far away towards a nucleus with
in this state is pp a0 (where a0 is the Bohr radius). The value
charge Q = 120 e, where e is the electronic charge.
It makes a closest approach of 10 fm to the nucleus. The de of p is (JEE Adv. 2015)
Broglie wavelength (in units of fm) of the proton at its start is: 12. The isotope 12 having a mass 12.014 u undergoes
(take the proton mass, mp= (5/3) ´ 10 – 27 kg; h/e = 4.2 ´ 10– 5 B
15 J.s / C;
b–decay to 12 12 has an excited state of the nucleus
6 C. 6 C
1
= 9 × 109 m/F; 1 fm = 10–15 m
4pe 0
(2012- I)
( 126 C*) at 4.041 MeV above its ground state. If 125 E decays
6. The work functions of Silver and Sodium are 4.6 and 2.3 eV,
respectively. The ratio of the slope of the stopping potential to 12
6 C * , the maximum kinetic energy of the b–particle in
versus frequency plot for Silver to that of Sodium is units of MeV is (1 u = 931.5 MeV/c2, where c is the speed of
(JEE Adv. 2013-I)
light in vacuum). (JEE Adv. 2016)
7. A freshly prepared sample of a radioisotope of half-life 1386
s has activity 103 disintegrations per second. Given that ln2 13. A hydrogen atom in its ground state is irradiated by light of
= 0.693, the fraction of the initial number of nuclei (expressed wavelength 970 Å. Taking hc/e = 1.237 × 10–6 eV m and the
in nearest integer percentage) that will decay in the first 80 s ground state energy of hydrogen atom as –13.6 eV, the
after preparation of the sample is (JEE Adv. 2013-I) number of lines present in the emission spectrum is
(JEE Adv. 2016)

Section-B JEE Main / AIEEE


1. If 13.6 eV energy is required to ionize the hydrogen atom, The emission at instant can be [2002]
then the energy required to remove an electron from n = 2 is (a) i, ii, iii (b) i, ii, iii, iv (c) iv (d) ii, iii
(a) 10.2 eV (b) 0 eV [2002] 4. Sodium and copper have work functions 2.3 eV and 4.5 eV
(c) 3.4 eV (d) 6.8 eV. respectively. Then the ratio of the wavelengths is nearest
2. At absolute zero, Si acts as [2002] to [2002]
(a) non-metal (b) metal (a) 1 : 2 (b) 4 : 1 (c) 2 : 1 (d) 1 : 4.
(c) insulator (d) none of these 5. Formation of covalent bonds in compounds exhibits
3. At a specific instant emission of radioactive compound is (a) wave nature of electron [2002]
deflected in a magnetic field. The compound can emit (b) particle nature of electron
(i) electrons (ii) protons (c) both wave and particle nature of electron
(iii) He2+ (iv) neutrons (d) none of these
Modern Physics P-197

6. If N0 is the original mass of the substance of half-life period 2h


(c) 2 2
t1/2 = 5 years, then the amount of substance left after 15 v1 + v2 = (f + f )
years is [2002] m 1 2
(a) N0/8 (b) N0/16 (c) N0/2 (d) N0/4 1/ 2
7. By increasing the temperature, the specific resistance of a é 2h ù
(d) v1 - v2 = ê ( f1 - f 2 )ú
conductor and a semiconductor [2002] ëm û
(a) increases for both (b) decreases for both 18. Which of the following cannot be emitted by radioactive
(c) increases, decreases(d) decreases, increases substances during their decay ? [2003]
8. The energy band gap is maximum in [2002] (a) Protons (b) Neutrinoes
(a) metals (b) superconductors (c) Helium nuclei (d) Electrons
(c) insulators (d) semiconductors. 19. In the nuclear fusion reaction
9. The part of a transistor which is most heavily doped to 2 3 4
produce large number of majority carriers is [2002] 1 H + 1 H ® 2 He + n
(a) emmiter given that the repulsive potential energy between the two
(b) base
(c) collector nuclei is ~ 7.7 ´ 10 -14 J , the temperature at which the gases
(d) can be any of the above three. must be heated to initiate the reaction is nearly
10. Which of the following are not electromagnetic waves?
[Boltzmann’s Constant k = 1.38 ´ 10 -23 J/K ] [2003]
(a) cosmic rays (b) gamma rays [2002]
(c) b -rays (d) X-rays. (a) 10 7 K (b) 10 5 K (c) 10 3 K (d) 10 9 K
11. A strip of copper and another of germanium are cooled from 20. Which of the following atoms has the lowest ionization
room temperature to 80K. The resistance of [2003] potential ? [2003]
(a) each of these decreases 14 133 40 16
(b) copper strip increases and that of germanium decreases (a) N (b) Cs (c) Ar (d) O
7 55 18 8
(c) copper strip decreases and that of germanium increases 21. The wavelengths involved in the spectrum of deuterium
(d) each of these increases
12. Which of the following radiations has the least wavelength ?
(a) g -rays (b) b -rays [2003]
( D) are slightly different from that of hydrogen spectrum,
2
1
because [2003]
(c) a -rays (d) X -rays (a) the size of the two nuclei are different
13. When a U 238 nucleus originally at rest, decays by emitting (b) the nuclear forces are different in the two cases
an alpha particle having a speed ‘u’, the recoil speed of the (c) the masses of the two nuclei are different
residual nucleus is [2003] (d) the atraction between the electron and the nucleus is
4u differernt in the two cases
(a) 4u (b) - 4 u (c) (d) - 4u
22. In the middle of the depletion layer of a reverse- biased
238 234 234 238 p-n junction, the [2003]
14. The difference in th e variation of resistance with (a) electric field is zero
temeperature in a metal and a semiconductor arises (b) potential is maximum
essentially due to the difference in the [2003] (c) electric field is maximum
(a) crystal sturcture (d) potential is zero
(b) variation of the number of charge carriers with temperature 23. If the binding energy of the electron in a hydrogen atom is
(c) type of bonding 13.6eV, the energy required to remove the electron from the
(d) variation of scattering mechanism with temperature
15. A radioactive sample at any instant has its disintegration first excited state of Li + + is [2003]
rate 5000 disintegrations per minute. After 5 minutes, the (a) 30.6 eV (b) 13.6 eV
rate is 1250 disintegrations per minute. Then, the decay (c) 3.4 eV (d) 122.4 eV
constant (per minute) is [2003] 24. A radiation of energy E falls normally on a perfectly reflecting
(a) 0.4 ln 2 (b) 0.2 ln 2 (c) 0.1 ln 2 (d) 0.8 ln 2 surface. The momentum transferred to the surface is [2004]
16. A nucleus with Z= 92 emits the following in a sequence: (a) Ec (b) 2 E / c (c) E / c (d) E / c 2
a, b - , b - a, a, a, a, a, b - , b - , a, b + , b + , a 25. According to Einstein’s photoelectric equation, the plot of
Then Z of the resulting nucleus is [2003] the kinetic energy of the emitted photo electrons from a
(a) 76 (b) 78 (c) 82 (d) 74 metal Vs the frequency, of the incident radiation gives as
17. Two identical photocathodes receive light of frequencies f1 straight the whose slope [2004]
and f2. If the velocites of the photo electrons (of mass m ) (a) depends both on the intensity of the radiation and the
coming out are respectively v1 and v2, then [2003] metal used
2 h (b) depends on the intensity of the radiation
(a) v12 - v2 2 = (f - f )
m 1 2 (c) depends on the nature of the metal used
1/ 2
é 2h ù (d) is the same for the all metals and independent of the
(b) v1 + v2 = ê ( f1 + f 2 )ú intensity of the radiation
ëm û
EBD_7036
P-198 Topic-wise Solved Papers - PHYSICS
26. The work function of a substance is 4.0 eV. The longest 37. A photocell is illuminated by a small bright source placed 1
wavelength of light that can cause photoelectron emission 1
from this substance is approximately. [2004] m away. When the same source of light is placed m away,,
2
(a) 310 nm (b) 400 nm (c) 540 nm (d) 220 nm the number of electrons emitted by photocathode would
27. A nucleus disintegrated into two nuclear parts which have (a) increase by a factor of 4 [2005]
their velocities in the ratio of 2 : 1. The ratio of their nuclear (b) decrease by a factor of 4
sizes will be [2004] (c) increase by a factor of 2
(a) 3½ : 1 (b) 1:21/3 (c) 21/3:1 (d) 1:3½ (d) decrease by a factor of 2
28. The binding energy per nucleon of deuteron 1 H and ( )
2 38. The electrical conductivity of a semiconductor increases
when electromagnetic radiation of wavelength shorter than
( )
4
helium nucleus 2 He is 1.1 MeV and 7 MeV respectively..
2480 nm is incident on it. The band gap in (eV) for the
semiconductor is [2005]
If two deuteron nuclei react to form a single helium nucleus, (a) 2.5 eV (b) 1.1 eV (c) 0.7 eV (d) 0.5 eV
then the energy released is [2004] 39. The intensity of gamma radiation from a given source is I.
(a) 23.6 MeV (b) 26.9 MeV (c) 13.9 MeV (d) 19.2 MeV
I
29. An a-particle of energy 5 MeV is scattered through 180º by a On passing through 36 mm of lead, it is reduced to . The
8
fixed uranium nucleus. The distance of closest approach is of
the order of [2004] I
thickness of lead which will reduce the intensity to will be
-12
(a) 10 cm (b) 10 cm (c) 1A-10 (d) 10 cm -15 2
30. When npn transistor is used as an amplifier [2004] (a) 9 mm (b) 6mm [2005]
(a) electrons move from collector to base (c) 12mm (d) 18mm
(b) holes move from emitter to base 40. In a common base amplifier, the phase difference between
the input signal voltage and output voltage is [2005]
(c) electrons move from base to collector
(d) holes move from base to emitter p p
(a) p (b) (c) (d) 0
31. For a transistor amplifier in common emitter configuration 4 2
for load impedance of 1k W (h fe = 50 and hoe = 25) the 41. The diagram shows the energy levels for an electron in a
current gain is [2004] certain atom. Which transition shown represents the
(a) – 24.8 (b) – 15.7 (c) – 5.2 (d) – 48.78 emission of a photon with the most energy? [2005]
32. A piece of copper and another of germanium are cooled n =4
from room temperature to 77K, the resistance of n =3
(a) copper increases and germanium decreases
(b) each of them decreases [2004] n =2
(c) each of them increases
(d) copper decreases and germanium increases
33. The manifestation of band structure in solids is due to n =1
I II III IV
(a) Bohr’s correspondence principle [2004]
(a) IV (b) III
(b) Pauli’s exclusion principle (c) II (d) I
(c) Heisenberg’s uncertainty principle 42. If the kinetic energy of a free electron doubles, it’s deBroglie
(d) Boltzmann’s law wavelength changes by the factor [2005]
34. When p-n junction diode is forward biased then [2004]
(a) both the depletion region and barrier height are reduced 1 1
(b) the depletion region is widened and barrier height is (a) 2 (b) (c) 2 (d)
2 2
reduced
(c) the depletion region is reduced and barrier height is 43. A nuclear transformation is denoted by X (n, a ) 73 Li . Which
increased
(d) Both the depletion region and barrier height are of the following is the nucleus of element X ?
increased (a) 10 (b) 12 [2005]
5 Be C6
27
35. If radius of the 13 Al nucleus is estimated to be 3.6 fermi 11 9
(c) 4 Be (d) 5B
then the radius of 125
52 Te nucleus be nearly [2005] 44. In a full wave rectifier circuit operating from 50 Hz mains
(a) 8 fermi (b) 6 fermi (c) 5 fermi (d) 4 fermi frequency, the fundamental frequency in the ripple would be
7 (a) 25 Hz (b) 50 Hz [2005]
36. Starting with a sample of pure 66 Cu , of it decays into (c) 70.7 Hz (d) 100 Hz
8
Zn in 15 minutes. The corresponding half life is [2005] 45. In a common base mode of a transistor, the collector current
(a) 15 minutes (b) 10 minutes is 5.488 mA for an emitter current of 5.60 mA. The value of
the base current amplification factor (b) will be [2006]
1
(c) 7 minutes (d) 5 minutes (a) 49 (b) 50 (c) 51 (d) 48
2
Modern Physics P-199

46. The threshold frequency for a metallic surface corresponds 54. In the following, which one of the diodes reverse biased?
to an energy of 6.2 eV and the stopping potential for a [2006]
radiation incident on this surface is 5 V. The incident radiation +10 V
lies in [2006]
(a) ultra-violet region (b) infra-red region
(c) visible region (d) X-ray region –10 V
(a) R (b)
1 2 R
47. An alpha nucleus of energy mv bombards a heavy +5 V –5 V
2
nuclear target of charge Ze. Then the distance of closest +5 V
approach for the alpha nucleus will be proportional to
1
(a) v 2 (b) [2006] R
m R
R
1 1 (c) (d)
(c) 2 (d)
v Ze –10 V
48. The time taken by a photoelectron to come out after the
photon strikes is approximately [2006] 55. The anode voltage of a photocell is kept fixed. The
(a) 10–4 s (b) 10–10 s wavelength l of the light falling on the cathode is gradually
(c) 10–16 s (d) 10–1 s changed. The plate current I of the photocell varies as
49. 7
When 3Li nuclei are bombarded by protons, and the
follows [2006]
resultant nuclei are 4Be8, the emitted particles will be
(a) alpha particles (b) beta particles [2006]
(c) gamma photons (d) neutrons
50. The energy spectrum of b-particles [number N(E) as a (a) I (b) I
function of b-energy E] emitted from a radioactive source is
[2006]
O l O l
N(E) N(E)

(a) E(b) E I I
E0 E0
(c) (d)

N(E) N(E) O l O l
56. If the binding energy per nucleon in 7
and 42 He nuclei
3 Li
E E
(c) E0 (d) E0 are 5.60 MeV and 7.06 MeV respectively, then in the reaction
51. A solid which is not transparent to visible light and whose p + 73 Li ¾¾ ® 2 42 He
conductivity increases with temperature is formed by [2006]
energy of proton must be [2006]
(a) Ionic bonding (b) Covalent bonding (a) 28.24 MeV (b) 17.28 MeV
(c) Vander Waals bonding (d) Metallic bonding (c) 1.46 MeV (d) 39.2 MeV
52. If the ratio of the concentration of electrons to that of holes 57. The 'rad' is the correct unit used to report the measurement of
7 7 (a) the ability of a beam of gamma ray photons to produce
in a semiconductor is and the ratio of currents is , ions in a target [2006]
5 4
then what is the ratio of their drift velocities? [2006] (b) the energy delivered by radiation to a target
5 4 5 4 (c) the biological effect of radiation
(a) (b) (c) (d) (d) the rate of decay of a radioactive source
8 5 4 7
58. If the lattice constant of this semiconductor is decreased,
53. The circuit has two oppositively connected ideal diodes in then which of the following is correct? [2006]
parallel. What is the current flowing in the circuit? [2006]
conduction
4W Ec
band width
D1 D2 band gap Eg
valence
12V band width Ev
3W 2W
(a) All Ec, Eg, Ev increase
(b) Ec and Ev increase, but Eg decreases
(a) 1.71 A (b) 2.00 A (c) Ec and Ev decrease, but Eg increases
(c) 2.31 A (d) 1.33 A (d) All Ec, Eg, Ev decrease
EBD_7036
P-200 Topic-wise Solved Papers - PHYSICS
59. The rms value of the electric field of the light coming from 66. Which of the following transitions in hydrogen atoms emit
the Sun is 720 N/C. The average total energy density of photons of highest frequency? [2007]
the electromagnetic wave is [2006] (a) n = 1 to n = 2 (b) n = 2 to n = 6
(a) 4.58 × 10–6 J/m3 (b) 6.37 × 10–9 J/m3 (c) n = 6 to n = 2 (d) n = 2 to n = 1
(c) 81.35 × 10–12 J/m3 (d) 3.3 × 10–3 J/m3
DIRECTIONS: Question No. 67 and 68 are based on the
60. If MO is the mass of an oxygen isotope 8 O17 ,MP and MN following paragraph.
are the masses of a proton and a neutron respectively, the
nuclear binding energy of the isotope is [2007] Wave property of electrons implies that they will show diffraction
(a) (MO –17MN)c2 effects. Davisson and Germer demonstrated this by diffracting
(b) (MO – 8MP)c2 electrons from crystals. The law governing the diffraction from a
(c) (MO– 8MP –9MN)c2 crystal is obtained by requiring that electron waves reflected from
(d) MOc 2 the planes of atoms in a crystal interfere constructively (see figure).
61. In gamma ray emission from a nucleus [2007]
(a) only the proton number changes
(b) both the neutron number and the proton number Inco m ng
Outgoi s
change Electr ing i n
ons Ele ctro
(c) there is no change in the proton number and the
d
neutron number
(d) only the neutron number changes
62. If in a p-n junction diode, a square input signal of 10 V is Crystal plane
applied as shown [2007] 67. Electrons accelerated by potential V are diffracted from a
crystal. If d = 1Å and i = 30°, V should be about
5V
(h = 6.6 × 10 – 34 Js, me = 9.1 × 10–31 kg, e = 1.6 × 10 – 19 C)
RL [2008]
(a) 2000 V (b) 50 V (c) 500 V (d) 1000 V
-5V 68. If a strong diffraction peak is observed when electrons are
incident at an angle ‘i’ from the normal to the crystal planes
Then the output signal across RL will be
with distance ‘d’ between them (see figure), de Broglie
10 V wavelength ldB of electrons can be calculated by the
+5V
relationship ( n is an integer) [2008]
(a) (b) (a) d sin i = nldB (b) 2d cos i = nldB
(c) 2d sin i = nldB (d) d cos i = nldB
69. This question contains Statement-1 and statement-2. Of the
four choices given after the statements, choose the one
(c) (d) that best describes the two statements. [2008]
-5V Statement-1:
-10 V
63. Photon of frequency n has a momentum associated with it. Energy is released when heavy nuclei undergo fission or
If c is the velocity of light, the momentum is [2007] light nuclei undergo fusion and
Statement-2 :
(a) hn / c (b) n /c (c) h n c (d) hn / c2
For heavy nuclei, binding energy per nucleon increases with
64. The half-life period of a radio-active element X is same as
increasing Z while for light nuclei it decreases with
the mean life time of another radio-active element Y. Initially
increasing Z.
they have the same number of atoms. Then [2007]
(a) Statement-1 is false, Statement-2 is true
(a) X and Y decay at same rate always
(b) Statement-1 is true, Statement-2 is true; Statement-2 is
(b) X will decay faster than Y a correct explanation for Statement-1
(c) Y will decay faster than X (c) Statement-1 is true, Statement-2 is true; Statement-2 is
(d) X and Y have same decay rate initially not a correct explanation for Statement-1
65. Carbon, silicon and germanium have four valence electrons (d) Statement-1 is true, Statement-2 is false
each. At room temperature which one of the following 70. A working transistor with its three legs marked P, Q and R is
statements is most appropriate ? [2007] tested using a multimeter. No conduction is found between
(a) The number of free electrons for conduction is P and Q. By connecting the common (negative) terminal of
significant only in Si and Ge but small in C. the multimeter to R and the other (positive) terminal to P or
(b) The number of free conduction electrons is significant Q, some resistance is seen on the multimeter. Which of the
in C but small in Si and Ge. following is true for the transistor? [2008]
(c) The number of free conduction electrons is negligibly (a) It is an npn transistor with R as base
small in all the three. (b) It is a pnp transistor with R as collector
(d) The number of free electrons for conduction is (c) It is a pnp transistor with R as emitter
significant in all the three. (d) It is an npn transistor with R as collector
Modern Physics P-201

71. Suppose an electron is attracted towards the origin by a 76. The logic circuit shown below has the input waveforms ‘A’
and ‘B’ as shown. Pick out the correct output waveform.
k
force where ‘k’ is a constant and ‘r’ is the distance of the [2009]
r
electron from the origin. By applying Bohr model to this A
Y
system, the radius of the nth orbital of the electron is found
B
to be ‘rn’ and the kinetic energy of the electron to be ‘Tn’.
Then which of the following is true? [2008] Input A
1 2
(a) Tn µ , rn µ n (b) Tn independent of n, rn µ n
2
n Input B

1 1 2
(c) Tn µ , rn µ n (d) Tn µ , rn µ n Output is
n n
72. In the circuit below, A and B represent two inputs and C (a)
represents the output. [2008]
A (b)
C
(c)
B
(d)

The circuit represents 77. A p-n junction (D) shown in the figure can act as a rectifier.
(a) NOR gate (b) AND gate An alternating current source (V) is connected in the circuit.
(c) NAND gate (d) OR gate
73. The transition from the state n = 4 to n = 3 in a hydrogen like D
atom results in ultraviolet radiation. Infrared radiation will R
be obtained in the transition from :
(a) 3 ® 2 (b) 4 ® 2
[2009] V ~
(c) 5 ® 4 (d) 2 ® 1
74. The surface of a metal is illuminted with the light of 400 nm.
The current (I) in the resistor (R) can be shown by :
The kinetic energy of the ejected photoelectrons was found
to be 1.68 eV. The work function of the metal is : [2009] I
(hc = 1240 eV.nm)
(a) 1.41 eV (b) 1.51 eV (a) (b)
(c) 1.68 eV (d) 3.09 eV
t
75.

(c) (d)

78. Statement -1 : When ultraviolet light is incident on a


photocell, its stopping potential is V0 and the maximum
kinetic energy of the photoelectrons is Kmax .When the
ultraviolet light is replaced by X-rays, both V0 and Kmax
increase.
The above is a plot of binding energy per nucleon Eb, against Statement -2 : Photoelectrons are emitted with speeds
the nuclear mass M; A, B, C, D, E, F correspond to different ranging from zero to a maximum value because of the range
nuclei. Consider four reactions : [2009] of frequencies present in the incident light. [2010]
(i) A + B ® C + e (ii) C ® A + B + e (a) Statement -1 is true, Statement -2 is true ; Statement -2
(iii) D + E ® F + e and (iv) F® D + E + e, is the correct explanation of Statement -1.
where e is the energy released? In which reactions is e (b) Statement -1 is true, Statement -2 is true; Statement -2
positive? is not the correct explanation of Statement -1
(a) (i) and (iii) (b) (ii) and (iv) (c) Statement -1 is false, Statement -2 is true.
(c) (ii) and (iii) (d) (i) and (iv) (d) Statement -1 is true, Statement -2 is false.
EBD_7036
P-202 Topic-wise Solved Papers - PHYSICS
DIRECTIONS: Questions number 79-80 are based on the (a) 14 min (b) 20 min [2011]
following paragraph. (c) 28 min (d) 7 min
87. This question has Statement – 1 and Statement – 2. Of the
A nucleus of mass M + Dm is at rest and decays into two daughter four choices given after the statements, choose the one
M that best describes the two statements. [2011]
nuclei of equal mass each. Speed of light is c.
2 Statement – 1: A metallic surface is irradiated by a
79. The binding energy per nucleon for the parent nucleus is E1 monochromatic light of frequency v > v0 (the threshold
and that for the daughter nuclei is E2. Then [2010] frequency). The maximum kinetic energy and the stopping
(a) E2 = 2E1 (b) E1 > E2 potential are Kmax and V0 respectively. If the frequency
(c) E2 > E1 (d) E1 = 2 E2 incident on the surface is doubled, both the Kmax and V0 are
80. The speed of daughter nuclei is [2010] also doubled.
Statement – 2 : The maximum kinetic energy and the
Dm 2Dm stopping potential of photoelectrons emitted from a surface
(a) c (b) c
M + Dm M are linearly dependent on the frequency of incident light.
Dm Dm (a) Statement–1 is true, Statement–2 is true, Statement – 2
(c) c (d) c is the correct explanation of Statement – 1.
M M + Dm
(b) Statement–1 is true, Statement–2 is true, Statement – 2
81. A radioactive nucleus (initial mass number A and atomic is not the correct explanation of Statement – 1.
number Z emits 3 a - particles and 2 positrons. The ratio of (c) Statement – 1 is false, Statement – 2 is true.
number of neutrons to that of protons in the final nucleus
(d) Statement – 1 is true, Statement – 2 is false.
will be [2010]
88. Hydrogen atom is excited from ground state to another state
A- Z -8 A- Z -4 with principal quantum number equal to 4. Then the number
(a) (b)
Z -4 Z -8 of spectral lines in the emission spectra will be : [2012]
A - Z - 12 A- Z -4 (a) 2 (b) 3 (c) 5 (d) 6
(c) (d) 89. Truth table for system of four NAND gates as shown in
Z -4 Z -2
82. The combination of gates shown below yields [2010] figure is : [2012]
(a) OR gate A
A
(b) NOT gate X Y
(c) XOR gate
B
(d) NAND gate B
83. If a source of power 4kW produces 1020 photons/second , A B Y A B Y
the radiation belongs to a part of the spectrum called [2010]
0 0 0 0 0 0
(a) X -rays (b) ultraviolet rays
(c) microwaves (d) g -rays 0 1 1 0 1 0
(a) (b)
84. This question has Statement – 1 and Statement – 2. Of the 1 0 1 1 0 1
four choices given after the statements, choose the one 1 1 0 1 1 1
that best describes the two statements. [2011]
A B Y A B Y
Statement – 1 : Sky wave signals are used for long distance
radio communication. These signals are in general, less 0 0 1 0 0 1
stable than ground wave signals. 0 1 1 0 1 0
Statement – 2 : The state of ionosphere varies from hour to (c) (d)
1 0 0 1 0 1
hour, day to day and season to season. 1 1 0 1 1 1
(a) Statement–1 is true, Statement–2 is true, Statement–2
90. A radar has a power of 1kW and is operating at a frequency
is the correct explanation of Statement–1.
of 10 GHz. It is located on a mountain top of height 500 m.
(b) Statement–1 is true, Statement–2 is true, Statement–2
The maximum distance upto which it can detect object
is not the correct explanation of Statement – 1.
located on the surface of the earth (Radius of earth
(c) Statement – 1 is false, Statement – 2 is true.
= 6.4 × 106m) is : [2012]
(d) Statement – 1 is true, Statement – 2 is false.
(a) 80 km (b) 16 km (c) 40 km (d) 64 km
85. Energy required for the electron excitation in Li++ from the
91. Assume that a neutron breaks into a proton and an electron.
first to the third Bohr orbit is : [2011]
The energy released during this process is : (mass of neutron
(a) 36.3 eV (b) 108.8 eV (c) 122.4 eV (d) 12.1 eV
= 1.6725 × 10–27 kg, mass of proton = 1.6725 × 10–27 kg,
86. The half life of a radioactive substance is 20 minutes. The
mass of electron = 9 × 10–31 kg). [2012]
approximate time interval (t2 – t1) between the time t2 when
(a) 0.73 MeV (b) 7.10 MeV (c) 6.30 MeV(d) 5.4 MeV
2 1 92. A diatomic molecule is made of two masses m1 and m2 which
of it had decayed and time t1 when of it had decayed is :
3 3 are separated by a distance r. If we calculate its rotational
Modern Physics P-203

energy by applying Bohr's rule of angular momentum 98. The current voltage relation of a diode is given by
quantization, its energy will be given by : (n is an integer)
I = ( e1000V T - 1) mA, where the applied voltage V is in volts
2 2 2 2 2
(m1 + m2 ) n h n h
(a) (b) and the temperature T is in degree kelvin. If a student makes
2m12 m22 r 2 2(m1 + m2 )r 2 an error measuring ±0.01 V while measuring the current of
2 2
2n h (m1 + m2 )n2 h 2 5 mA at 300 K, what will be the error in the value of current in
(c) (d) mA? [JEE Main 2014]
(m1 + m2 )r 2 2m1m2 r 2
(a) 0.2 mA (b) 0.02 mA (c) 0.5 mA (d) 0.05 mA
93. A diode detector is used to detect an amplitudemodulated 99. During the propagation of electromagnetic waves in a medium:
wave of 60% modulation by using a condenser of capacity
[JEE Main 2014]
250 picofarad in parallel with a load resistance 100 kilo ohm.
(a) Electric energy density is double of the magnetic energy
Find the maximum modulated frequency which could be
detected by it. [JEE Main 2013] density.
D
(a) 10.62 MHz (b) Electric energy density is half of the magnetic energy
density.
(b) 10.62 kHz
Signal C R (c) Electric energy density is equal to the magnetic energy
(c) 5.31 MHz
density.
(d) 5.31 kHz
(d) Both electric and magnetic energy densities are zero.
94. The magnetic field in a travelling electromagnetic wave has
100.The radiation corresponding to 3 ® 2 transition of hydrogen
a peak value of 20 nT. The peak value of electric field strength
is : [JEE Main 2013] atom falls on a metal surface to produce photoelectrons. These
(a) 3 V/m (b) 6 V/m electrons are made to enter a magnetic field of 3 × 10–4 T. If
(c) 9 V/m (d) 12 V/m the radius of the largest circular path followed by these
electrons is 10.0 mm, the work function of the metal is close
95. The anode voltage of a photocell is kept fixed. The
to: [JEE Main 2014]
wavelength l of the light falling on the cathode is gradually
(a) 1.8 eV (b) 1.1 eV (c) 0.8 eV (d) 1.6 eV
changed. The plate current I of the photocell varies as
follows : [JEE Main 2013] 101.Hydrogen ( 1 H1 ) , Deuterium ( 1 H 2 ) , singly ionised Helium
I I
( He4 ) and doubly ionised lithium ( Li 6 )
+ ++
2 3 all have one
electron around the nucleus. Consider an electron transition
(a) (b) from n = 2 to n = 1. If the wavelengths of emitted radiation are
O l O l l1 , l 2 , l3 and l4 respectively then approximately which
I I one of the following is correct? [JEE Main 2014]
(a) 4l1 = 2l 2 = 2l3 = l 4 (b) l1 = 2l 2 = 2l3 = l 4
(c) l1 = l 2 = 4l3 = 9l 4 (d) l1 = 2l 2 = 3l3 = 4l 4
(c) (d)
102.The forward biased diode connection is: [JEE Main 2014]
O l O l (a)
+2V –2V
96. The I-V characteristic of an LED is [JEE Main 2013] –3V –3V
(b)
B 2V 4V
G (c)
I Y –2V +2V
R (d)
(a) (b)
103.Match List - I (Electromagnetic wave type) with List - II (Its
O V O V association/application) and select the correct option from
V O the choices given below the lists: [JEE Main 2014]
I R List 1 List 2
Y I 1. Infrared waves (i) To treat muscular strain
(c) (d) G
B 2. Radio waves (ii) For broadcasting
O V 3. X-rays (iii) To detect fracture of bones
97. In a hydrogen like atom electron make transition from an 4. Ultraviolet rays (iv) Absorbed by the ozone
energy level with quantum number n to another with quantum layer of the atmosphere
number (n – 1). If n>>1, the frequency of radiation emitted is 1 2 3 4
proportional to : [JEE Main 2013] (a) (iv) (iii) (ii) (i)
1 1 1 1 (b) (i) (ii) (iv) (iii)
(a) (b) 2 (c) 3 (d) 3
n n n n (c) (iii) (ii) (i) (iv)
2
(d) (i) (ii) (iii) (iv)
EBD_7036
P-204 Topic-wise Solved Papers - PHYSICS
104. A red LED emits light at 0.1 watt uniformly around it. The (a) OR (b) NAND
amplitude of the electric field of the light at a distance of 1 m (c) NOT (d) AND
from the diode is : [JEE Main 2015] 110. Choose the correct statement : [JEE Main 2016]
(a) In frequency modulation the amplitude of the high
(a) 5.48 V/m (b) 7.75 V/m frequency carrier wave is made to vary in proportion to
(c) 1.73 V/m (d) 2.45 V/m the amplitude of the audio signal.
105.A signal of 5 kHz frequency is amplitude modulated on a (b) In frequency modulation the amplitude of the high
carrier wave of frequency 2 MHz. The frequencies of the frequency carrier wave is made to vary in proportion to
the frequency of the audio signal.
resultant signal is/are : [JEE Main 2015] (c) In amplitude modulation the amplitude of the high
(a) 2005 kHz, 2000 kHz and 1995 kHz frequency carrier wave is made to vary in proportion to
(b) 2000 kHz and 1995 kHz the amplitude of the audio signal.
(c) 2 MHz only (d) In amplitude modulation the frequency of the high
(d) 2005 kHz and 1995 kHz frequency carrier wave is made to vary in proportion to
the amplitude of the audio signal.
106.As an electron makes a transition from an excited state to the 111. Radiation of wavelength l, is incident on a photocell. The
ground state of a hydrogen - like atom/ion : [JEE Main 2015] fastest emitted electron has speed v. If the wavelength is
(a) kinetic energy decreases, potential energy increases but 3l
total energy remains same changed to , the speed of the fastest emitted electron
4
(b) kinetic energy and total energy decrease but potential will be: [JEE Main 2016]
energy increases 1 1
(c) its kinetic energy increases but potential energy and total æ 4 ö2 æ 3 ö2
energy decrease (a) = vç
ç ÷÷ (b) = vç
ç ÷ ÷
è3 ø è4 ø
(d) kinetic energy, potential energy and total energy 1 1
decrease æ 4 ö2 æ 4 ö2
107.Match List - I (Fundamental Experiment) with List - II (its (c) > v çç ÷÷ (d) < v çç ÷÷
è3ø è3ø
conclusion) and select the correct option from the choices 112. Half-lives of two radioactive elements A and B are 20 minutes
given below the list: [JEE Main 2015] and 40 minutes, respectively. Initially, the samples have
Lis t-I Lis t-II equal number of nuclei. After 80 minutes, the ratio of
decayed number of A and B nuclei will be :
A . Fran ck-Hertz (i) Particle n atu re of [JEE Main 2016]
Exp eriment ligh t (a) 1 : 4 (b) 5 : 4
B. Pho to -electric (ii) Dis crete en erg y (c) 1 : 16 (d) 4 : 1
experimen t levels of atom 113. Identify the semiconductor devices whose characteristics
are given below, in the order (a), (b), (c), (d) :
C. Davis on -Germer (iii) W av e natu re o f
[JEE Main 2016]
experimen t electro n
I I
(iv ) Structu re o f ato m
(a) (A)-(ii); (B)-(i); (C)-(iii)
(b) (A)-(iv); (B)-(iii); (C)-(ii)
V V
(c) (A)-(i); (B)-(iv); (C)-(iii)
(d) (A)-(ii); (B)-(iv); (C)-(iii)
108. For a common emitter configuration, if a and b have their
usual meanings, the incorrect relationship between a and
b is : [JEE Main 2016] (a) (b)
2
b b I dark Resistance
(a) a=
1 +b
(b) a= 2
1 +b
1 1 b
(c) a = b+ 1 (d) a= -b
1 V
Intensity
109. If a, b, c, d are inputs to a gate and x is its output, then, as per Illuminated of light
the following time graph, the gate is : [JEE Main 2016]
(c) (d)
d
(a) Solar cell, Light dependent resistance, Zener diode,
c simple diode
(b) Zener diode, Solar cell, simple diode, Light dependent
b resistance
(c) Simple diode, Zener diode, Solar cell, Light dependent
a resistance
(d) Zener diode, Simple diode, Light dependent resistance,
x
Solar cell
Solutions & Explanations

1 Units and Measurements


Section-A : JEE Advanced/ IIT-JEE
A 1. ML 2 T – 1 2. M –3 L –2 T 4 Q 4 3. ML 5 T – 2
C 1. (c) 2. (d) 3. (a) 4. (c) 5. (a) 6. (d)
7. (c) 8. (d) 9. (b) 10. (b) 11. (d) 12. (c)
13. (a) 14. (b) 15. (b)
D 1. (a, d) 2. (a, b, c) 3. (a, b, c, d) 4. (b, c) 5. (a, c) 6. (d)
7. (a,c,d) 8. (b, c) 9. (a, c) 10. (b, d) 11. (a, b, d)
E 1. (i) N/m2; (ii) Tesla; (iii) Dioptre; 2. a = – 5/6, b = 1/2, c = 1/3 3. (i) [M1L2T–1Q–1] (ii) [ML–1T–2]
4. Angular Momentum – [ML T ] ; Latent heat – [L T ] ; Torque – [ML2T–2]
2 –1 2 –2

Capacitance – [M–1L–2T2Q2] ; Inductance – [ML2Q–2] ; Resistivity – [ML3T–1Q–2]


5. Capacitance coulomb-volt–1, coulomb2-joule–1
Inductance ohm-sec, volt-second (ampere)–1
Magnetic Induction newton (ampere-metre)–1
1
Q (a) q = CV; U = CV 2
2
(b) Refer to solution of Q. 3, type D
(c) F = I l B
a
6. units 7. 2.6 cm2 8. 1.09 ´ 1010 Nm -2 9. 2.66 gm/cm3
n +1
F 1. (A) ® p, q ; (B) ® r, s ; (C) ® r, s ; (D) ® r,, s 2. (c)
G 1. (c) 2. (b)
I 1. (3) 2. (4) 3. (4)

Section-B : JEE Main/ AIEEE


1. (a) 2. (c) 3. (b) 4. (a) 5. (d) 6. (c)
7. (a) 8. (d) 9. (d) 10. (a) 11. (a) 12. (a)
13. (c) 14. (b) 15. (b) 16. (d) 17. (a) 18. (d)

Section-A JEE Advanced/ IIT-JEE


A. Fill in the Blanks C. MCQs with ONE Correct Answer
(c) Note : Here çæ ÷ö e 0 E 2 represents energy per unit
1
é 2 -2 ù 1.
E ë ML T û é 2 -1 ù è 2ø
1. E = hv h = v = é -1 ù = ë ML T û volume.
T
ë û
[e 0 ] [ E 2 ] =
[ Energy ] = ML2T -2 = ML-1T -2
2. [X] = [C] = [M –1 L–2 T 2 Q2] [ Volume] L3
[Z] = [B] = [MT –1 Q –1 ] 2. (d) Dimensionally e0 L = Capacitance (c)
-1 -2 2 2
[M L T Q ] DV C DV q
\ [Y] = = [ M -3 L-2T 4 Q 4 ] \ e0 L = = =I
-1 -1 2 Dt Dt Dt
[ MT Q ]

-2 3. (a) V = l3 = (1.2 × 10–2 m)3 = 1.728 × 10–6 m3


MLT Þ V = 1.7 × 10–6 m3.
3. [a] = [PV 2 ] = 2
L6 = ML5 T –2
L
EBD_7036
P-S-2 Topic-wise Solved Papers - PHYSICS
4. (c) Unit of k is joules per kelvin or dimensional formula of Note: We can also take value of y from options given
k is [ML2T–2 q -1 ] without calculating it as it is same in all options.
Note : The power of an exponent is a number. \ Y = ( 2 ± 0.2) ´ 1011 N/m 2
az 10. (b) The time period of a simple pendulum is given by
Therefore, dimensionally = M ° L°T °
kq
l l l
kq T = 2p \ T 2 = 4p 2 Þ g = 4p 2 2
\ a= g g T
z
[ML2T -2 q -1 ][q] é Dg Dl DT
\ a= = MLT -2 ù Þ ´ 100 = ´ 100 + 2 ´ 100
[ L] ë û g l T
a a Case (i)
and dimensionally P = Þ b= Dl = 0.1 cm, l = 64cm, DT = 0.1s, T = 128s
b P 11. (d) 20 divisions on the vernier scale
MLT -2 = 16 divisions of main scale
\ [b] = = M 0 L2T 0 \ 1 division on the vernier scale
-1 -2
ML T 16 16
= divisions of main scale = ´ 1mm = 0.8 mm
m 20 20
5. (a) r = We know that least count = 1MSD – 1VSD
l p r2
= 1 mm – 0.8 mm = 0.2 mm
Dr Dm 2Dr Dl 12. (c) Diameter D = M.S.R. + (C.S.R) × L.C.
= + + 0.5
r m r l D = 2.5 + 20 ×
Putting the values 50
D = 2.70 mm
Dl = 0.06 cm, l = 6cm; Dr = 0.005 cm; r = 0.5 cm, The uncertainty in the measurement of diameter
m = 0.3 gm; Dm = 0.003 gm DD = 0.01 mm.
Dr We know that
\ Dr = 4 \ ´ 100 = 4%. Mass M M
r 100 r r= = =
Volume 3
ur ur V 4 æ Dö
6. (d) Electric flux f E = E . S πç ÷
3 è 2ø
\ Dimensionally f E ¹ E Dr DM DD
\ ´ 100 = ´ 100 + 3 ´ 100
0.5 r M D
7. (c) Least count = = 0.01mm 0.01
50 =2+3× ´ 100 = 3.1%
Zero error = 5 × L.C = 5 × 0.01 mm = 0.05 mm 2.70
Diameter of ball = [Reading on main scale] + [Reading 13. (a) The maximum possible error in Y due to l and d are
on circular scale × L . C] – Zero error DY Dl 2 Dd
= +
= 0.5 × 2 + 25 × 0.01 – 0.05 = 1.20 mm Y l d
Dg Dl DT Pitch
8. (d) = +2 Least count =
g l T No. of divi on circular scale
Dl and DT are least and number of readings are 0.5
maximum in option (d), therefore the measurement of g = mm = 0.005 mm
100
is most accurate with data used in this option.
Dl 0.005 mm 1
4mgL 4 ´ 1 ´ 9.8 ´ 2 Error contribution of l = = =
9. (b) Y= = l 0.25 mm 50
pD 2 l
( ) ´ (0.8 ´ 10 )
2
p 0.4 ´ 10 -3 -3
2 Dd 2 × 0.005 mm 1
Error contribution of d = = =
d 0.5 mm 50
= 2.0 ´ 1011 N/m 2
14. (b) Reading = M.S.R + No of division of V.S matching the
main scale division (1MSD – 1VSD)
Now DY = 2DD + Dl
Y D l æ 2.45 ö
= 5.10 + 24 ç 0.05 - ÷
[Q the value of m, g and L are exact] è 50 ø
0.01 0.05 = 5.124 cm Option (b) is correct.
= 2´ + = 2 × 0.025 + 0.0625 15. (b) For C1
0.4 0.8
L.C. = 1MSD – 1VSD
= 0.05 + 0.0625 = 0.1125
= 1mm – 0.9 mm = 0.1 mm = 0.01 cm [10 VS D = 9 mm]
Þ DY = 2 × 1011 × 0.1125 = 0.225 × 1011
Reading = MSR + L.C × Verni scale division coinciding
= 0.2 ´ 1011 N/m 2 the Main scale division = 2.8 + (0.01) × 7 = 2.87 cm
Units and Measurements P-S-3

For C2 Dg Dl DT
L.C = 1 mm – 1.1 mm [10 VSD = 11 mm] \ ´ 100 = ´ 100 + 2 ´100
g l T
L.C = – 0.1 mm = – 0.01 cm
Reading = 2.8 + (10 – 7) × 0.01 = 2.83 cm Dg æ 1 ö
\ ´100 = 0 + 2 ç ÷ ´100 = 5
g è 40 ø
D. MCQs with ONE or MORE THAN ONE Correct
l
1. (a, d) t = F × r × sin q; W = F × d × cosq 6. (d) Given 2d sin q = l \ d= cosec q ... (i)
2
Dimensionally, light year = wavelength = [L]
d(d) l
2. (a, b, c) \ = [ - cosec q cot q]
Reynold’s number dq 2
= Coefficient of friction = [M0L0T0] l
Note : Curie is the unit of radioactivity (number of \ d (d) = – cosecq cotq dq ... (ii)
2
atoms decaying per second) and frequency also has
on dividing (i) and (ii) we get
the unit per second.
d(d)
Q W \ = cot q d q
Latent heat = and Gravitation potential = . d
m m As q increases from 0° to 90°, cot q decreases and
3. (a, b, c, d)
d(d)
f æ dI ö 2U therefore decreases option (d) is correct
L = ; L = -e ç ÷ ; L = d
; L=R×t
I è dt ø I2 l cos q
f weber From (ii) |d (d)| = 2
2 sin q
L= =
I ampere cos q
-e volt volt-sec This value of decreases as q increases from
= = sin 2 q
L=
dI / dt ampere/sec ampere 0° to 90°
2U joule 9. (a, c, d)
L=
2
= L µ hx cy Gz
I (ampere) 2
Dimensionally
L = R ´ t = ohm - sec
[M 0 L1 T 0 ] = [ML2T –1 ]x [LT –1 ]y [M –1L3T –2 ]z
Q1Q2
F µ0 I1I 2
4. (b, c) By definition F = and =
(4pe 0 ) r 2 l 2pL M 0 L1 T 0 = Mx – z L2x + y + 3z T–x – y – 2z
\ x–z=0 Þ x=z
[Q ]2 I 2T 2 \ 2x + y + 3z = 1 and –x – y – 2z = 0
Hence, [e0] = = -2 = M–1L–3 T 4 I 2
2
[ F ][ r ] MLT .L 2 On solving we get
1 3 1
[F ] MLT - 2 -2 -2 x= ,y= – ,z=
[µ0] = = = MLT I 2 2 2
[ I ]2 I2
\ Lµ h
5. (a,c) As the length of the string of simple pendulum is exactly
l m (given), therefore the error in length Dl = 0. Lµ G
Further the possibility of error in measuring time is 1s C, D are correct options
in 40s. M µ hxcyGz
2 –1 x –1 y –1 3 –2 Z
Dt DT 1 M ¢L °T ° µ [ML T ] [LT ] [M L T ]
\ = = x – z 2x + y + 3z
t T 40 \ M ¢L °T ° µ M L T – y –2z
– x

40 \ x–z=1
The time period T = = 2 seconds 2x + y + 3z = 0
20
–x – y – 2z = 0
DT 1 DT 1
\ = Þ = Þ DT = 0.05sec On solving we get
T 40 2 40
1 1 1
l l x= ,y= ,z=–
We know that T = 2p Þ T = 4p 2 2 2 2 2
g g
\ Mµ C
l A is the correct option.
\ g = 4p2
T2
EBD_7036
P-S-4 Topic-wise Solved Papers - PHYSICS
8. (b, c) 10. (b, d)
Vernier callipers q2
1cm We know that, dimensionally Î= ,
1 MSD = = 0.125cm l2 F
8 RT
5 VSD = 4MSD k BT = = PV = F ´ l
NA
1
\ 5VSD = 4 × cm = 0.5cm ek BT é q2 F ´l ù
1/2
8 = ê 2 ´ –3 2 ú =l
\ 1 VSD = 0.1cm Now
nq 2 ëê l F l q ûú
L.C = 1MSD – 1VSD 1/2
= 0.125cm – 0.1cm q2 é l2 F ´ q2 ù
Also = ê ú =l
= 0.025cm Î n1/3 k BT ëê q 2 l -1 ´ F ´ l ûú
Screw gauge
One complete revolution = 2M.S.D 1
11. (a, b, d) % error in measurement of ‘r’ = ´ 100 = 10%
If the pitch of screw gauge is twice the L.C of vernier callipers 10
then pitch = 2 × 0.025 = 0.05cm. 0.52 + 0.56 + 0.57 + 0.54 + 0.59
L.C of screw Gauge Tmean = = 0.556 » 0.56 S
6
pitch 0.04 + 0 + 0.01 + 0.02 + 0.03
= Total no. of divisions of circular scale DT = = 0.016 » 0.02S
6
\ % error in the measurement of ‘T’
0.05
= cm = 0.0005cm = 0.005 mm. 0.02
100 = ´ 100 = 3.57%
(b) is a correct option 0.56
Now if the least count of the linear scale of the screw gauge % error in the value of g
is twice the least count of venier callipers then. DT æ DR + Dr ö
=2 ´ 100 + ç ÷ ´ 100
L.C of linear scale of screw gauge = 2 × 0.025 = 0.05cm. T è R -r ø
Then pitch = 2 × 0.05 = 0.1cm.
0.1 = 2 ( 3.57 ) + æ 1 + 1 ö ´100 » 11%
ç 60 - 10 ÷
Then L.C of screw gauge = cm = 0.001cm = 0.01mm. è ø
100
(c) is a correct option. E. Subjective Problems
9. (a, c) We know that
1. The M.K.S. unit of Young’s modulus is Nm–2.
1 m0 The M.K.S. unit of magnetic induction is Tesla.
C= and R = The M.K.S. unit of power of lens is Dioptre.
m0e 0 e0
2. Given that T µ PadbEc
Now, m0 I2 = e0 V2 Þ [M0L0T1] = [ML–1T–2]a [ML–3]b [ML2T–2]c
m0 V 2 \ [M0L0T1] = [Ma + b + c L–a –3b + 2c T–2a – 2c]
\ = = R2 Þ Option A is correct \ a + b + c = 0, – a – 3b + 2c = 0
e0 I 2
– 2a – 2c = 1
Now, e0 I = m0 V
On solving, we get
m0 I 1
\ e =V = R Þ Option B is incorrect a = – 5/6, b = 1/2, c = 1/3
0 3. Magnetic Flux = [M1L2T–1Q–1]
Now, I = e0C V Modulus of Rigidity = [ML–1T–2]
1 V 4. Angular Momentum [ML2T–1]
\ = =R
e 0C I Latent heat [L2T–2]
Torque [ML2T–2]
\ 1
=R Capacitance [M–1 L–2T2Q2 ]
1
e0 Inductance [ML2Q–2]
m0e0
Resistivity [ML3T–1Q–2 ]
5. Capacitance coulomb-volt–1, coulomb2-
m0 joule–1
\ =R Þ Option C is correct
e0 Inductance ohm-sec, volt-second
Now, mo C I = e0 V (ampere)–1
Magnetic Induction newton (ampere-metre)–1
m0 V R m0 1 1 2
\ = = = ´ = µ0 Q (a) q = CV; U = CV
e0 I C C e0 1 2
(b) Refer to solution of Q. 3, type D
m 0 e0
(c) F = I l B
Þ Option (d) is incorrect
Units and Measurements P-S-5

6. (n + 1) divisions of vernier scale = n divisions of main scale. R PV ML-1T -2 ´ L3


n 2. (c) Boltzmannn constant = = =
\ One vernier division = main scale division N nTN K
n +1 = ML2 T -2 K -1
æ n ö
Least count = 1 M.S.D – 1VS.D = çè1 - ÷ MSD F MLT -2
n + 1ø Coefficient of viscosity = = = ML-1T -1
1 a 6prv L ´ LT -1
= M.S.D. = units [ Q 1 MSD = a units]
n +1 n +1 E ML2T -2
Planck constant = = -1
= ML-2T -1
1mm v T
7. Least Count = = 0.01mm
100 Hl ML2 T -2 ´ L
Diameter = MSR + CSR × (least count) Thermal conductivity = =
= 1 mm + 47 × (0.01) mm = 1.47 mm tADT T ´ L2 ´ K
= MLT K–3 –1
Surface Area = pDl
(c) is the correct option.
22
= × 1.47 × 56 mm2 = 2.58724 cm2 3. L.C. = 1 MSD – 1 VSD
7 9
= 2.6 cm2 (Rounding off to two significant figures) = 1MSD - MSD
W L 10
8. Y= ´ æ 9ö 1 æ 1 ö
2 X
pD = çè1 - ÷ø MSD = MSD = ç ´ 1÷ mm = 0.1 mm
10 10 è 10 ø
4
KEY CONCEPT : Maximum error in Y is given by The side of cube = 10 mm + 1 × 0.1 mm = 10.1 mm = 1.01 cm
æ DY ö æ DD ö DX DL mass 2.736 g
çè ÷ = 2ç + + Now, density = = = 2.66g / cm3
Y ø max è D ÷ø X L volume (1.01)3
(Rounding off to 3 significant figures)
= 2 æç
0.001ö æ 0.001ö æ 0.1 ö
+ + = 0.0489
è 0.05 ÷ø çè 0.125 ÷ø çè 110 ÷ø G. Comprehension Based Questions
It is given that W = 50 N; D = 0.05 cm = 0.05 × 10–2m;
X = 0.125 cm = 0.125 × 10–2m; 1. (c) e = [AT], w = [T–1]
L = 110 cm = 110 × 10–2m N = [L–3], Îo = [M–1 L–3 A2 T4]
50 ´ 4 ´ 110 ´ 10-2 We do not want Ampere [A] in the expression. This is
\ Y= = 2.24 × 1011N/m2 only possible when Î0 occurs as square. Therefore
3.14(0.05 ´ 10-2 ) ´ (0.125 ´ 10-2 )
options a and b are incorrect.
\ Maximum possible error in the value of
Y = DY = 0.0489 × 2.24 × 1011 Ne 2 L-3 A2T 2
= 1.09 × 1010 N/m2 = = T -2 = T -1
m Îo -1 -3 2 4
MM L A T
F. Match the Following Ne 2 c m Îo
1. A: p®q 2. (b) wp = = 2pn = 2p ; l = 2pc
m
m Îo l Ne 2
Reason : Unit of GMeMs = Fr2 = Nm 2 = kg × m2
s2 22 10 -30 ´10-11
= 2´ ´ 3 ´ 108 = 600 nm
= kg m3s–2 7 4 ´ 1027 ´ (1.6 ´ 10 -19 ) 2
Also (volt) (coulomb) (metre) = (joule) (metre)
= (N - m) (m) = Nm2 = kg m3s–2 I. Integer Value Correct Type
B:r®s
3RT 2 3 RT 1. (3) d µ rx S y f z
Reason : vrms = Þ vrms =
M M
M 0 L1T 0 = M x L-3x M yT - yT - z
3RT
Þ Unit of is m2 s–2 M 0 L1T 0 = M x + y L-3xT - y - z
M
Also (farad) (volt)2 (kg)–1 = (joule) kg–1 \ x + y = 0, – 3x = 1
= N-m kg–1 = kg ms–2 m kg–1 = m2s–2 1 1
C:r®s \ x=- and y =
3 3
F2 \n=3
Reason : F = qvB Þ v2 = 2 2
q B FL
\ Unit of v2 is m2s–2 which is further equal to FV2 kg–1. 2. (4) Y=
a ´l
D:r®s
Here F, a and L are accurately known.
2GM 2GM
Reason : Escape velocity ve = Þ ve2 = DY DL 1.0 ´ 10 -5
R R ´ 100 = ´ 100 = ´ 100 = 4%
\ The unit of
GM 2 –2
is m s .
Y l 25 ´ 10 -5
R
EBD_7036
P-S-6 Topic-wise Solved Papers - PHYSICS
3. (4) E = A2 e–0.2t
\ loge E = 2 loge A –0.2t
dE æ dA ö æ dt ö
´ 100 = 2 ç ´ 100 ÷ + 0.2t ç ´ 100 ÷
On differentiating we get E è A ø è t ø
dE
dE dA dt \ × 100 = 2 × 1.25% + 0.2 × 5 × 1.5%
=2 – 0.2 ´ t E
E A t
dE
As errors always add up therefore \ × 100 = 4%
E

Section-B JEE Main/ AIEEE


r r
1. (a) W = F × s = Fs cos q 1
= [ MLT -2 ][ L ] = [ ML2T -2 ] ;
Diameter of wire = MSR + CSR × L.C. = 0 + × 52
100
r r r = 0.52 mm = 0.052 cm
t = r ´ F Þ t = rF sin q
= [ L ] [ MLT -2 2 -2
] = [ ML T ] V
12. (a) R=
2. (c) We know that the velocity of light in vacuum is given I
by DR DV DI
1 1 ´ 100 = ´ 100 + ´100 = 3 + 3 = 6%
c= \ = c2 = L2T–2 R V I
mo eo mo eo
13. (c) Q Reading of Vernier = Main scale reading
3. (b) Momentum = mv = [MLT–1] + Vernier scale reading × least count.
E ML2T –2 Main scale reading = 58.5
Planck’s constant, h = = = ML2T -1 Vernier scale reading = 09 division
n T –1 least count of Vernier = 0.5°/30
F
4. (a) From stokes law F = 6phrv Þ h = 0.5°
6prv Thus R = 58.5° + 9 ×
MLT - 2 30
\h = Þ h = [ ML-1T -1 ] R = 58.65
[ L ][ LT -1 ]
1 q1q 2 q1q 2
5. (d) Moment of Inertia, I = Mr 2 14. (b) As we know, F = Þ e0 =
4 pe 0 R 2 4pFR 2
[I] = [ ML2 ]
r uur uur
Moment of force, t = r ´ F [AT]2
Hence, e0 = = [M -2 L-3 T 4 A 2 ]
r -2
MLT .L 2
é t ù = [ L ][ MLT -2 ] = [ ML2T -2 ]
ë û 15. (b) Measured length of rod = 3.50 cm
6. (c) We know that F = q v B For vernier scale with 1 Main Scale Division = 1 mm
F MLT -2 9 Main Scale Division = 10 Vernier Scale Division,
\ B= = = MT -1C -1 Least count = 1 MSD –1 VSD
qv C ´ LT -1
7. (a) Momentum, p = m × v = 0.1 mm
= (3.513) × (5.00) = 17.565 kg m/s 2 l
= 17.6 (Rounding off to get three significant figures) 16. (d) As, g = 4 p 2
T
0.5 Dg Dl DT
8. (d) Least count of screw gauge = mm = 0.01mm ´ 100 = ´ 100 + 2 ´ 100
50 So,
g l T
\ Reading = [Main scale reading + circular scale
reading × L.C] – (zero error) 0.1 1
= ´ 100 + 2 ´ ´ 100 = 2.72 ; 3%
= [3 + 35 × 0.01] – (–0.03) = 3.38 mm 20 90
9. (d) 30 Divisions of vernier scale coincide with 29 | DT1 | + | DT2 | + | DT3 | + | DT4 |
divisions of main scales 17. (a) DT =
29 4
Therefore 1 V.S.D = MSD 2 +1+ 3 + 0
30 = = 1.5
29 4
Least count = 1 MSD – 1VSD = 1 MSD - MSD As the resolution of measuring clock is 1.5 therefore
30
1 1 the mean time should be 92 ± 1.5
= MSD = ´ 0.5° = 1 minute.
30 30 0.5
10. (a) Number of significant figures in 23.023= 5 18. (d) L.C. = = 0.01 mm
50
Number of significant figures in 0.0003 = 1
Number of significant figures in 2.1 × 10–3 = 2 Zero error = 5 × 0.01 = 0.05 mm (Negative)
Reading = (0.5 + 25 × 0.01) + 0.05 = 0.80 mm
1
11. (a) L.C. = mm
100
2 Motion

Section-A : JEE Advanced/ IIT-JEE


A 1. 2R, pR 2. d/v 3. 0.6 m/s
B 1. T 2. T 3. F
C 1. (a) 2. (b) 3. (b) 4. (a) 5. (a)
6. (b) 7. (a) 8. (a) 9. (c)
D 1. (b) 2. (a, c, d) 3. (a, b, c)
ab 1 ab 2
E 1. t; t 2. (i) 0 ; (ii) 0 3. No 4. mid point of AB, 3.53 sec.
a+b 2 a+b
5. 17.32, 11.547 m from B 6. 1 sec, (5 3 , 5) in metres
2
u sin 2a u cos( a + q)
7. (a) (b) 8. u = (3.75î + 6.25 ĵ) m / s , t = 1 sec. 9. 45°, 2 m/sec.
g cos q cos q
H 1. (b)
I 1. 5 2. 5 3. 8
Section-B : JEE Main/ AIEEE
1. (c) 2. (b) 3. (c) 4. (d) 5. (b) 6. (a)
7. (c) 8. (a) 9. (b) 10. (d) 11. (c) 12. (d)
13. (c) 14. (d) 15. (a) 16. (d) 17. (c) 18. (b)
19. (b) 20. (a) 21. (d) 22. (d) 23. (c) 24. (c)
25. (a) 26. (a) 27. (d) 28. (c) 29. (b) 30. (b)
31. (c) 32. (b)

Section-A JEE Advanced/ IIT-JEE


A. Fill in the Blanks The velocity (v) of spot = dx / dt
1. Displacement = AOB = 2R df
distance C and the angular speed (w) of spot light =
dt
R From D SOP,
Distance = ACB = pR x
A O B tan f = \ x = h tan f
displacement h
2. The relative velocity of K w.r.t L along the line KL is dx df
r r r r r \ = h sec 2 f \ v = (h sec2 f) w
vK L = vK - vL = vK + (-vL ) dt dt
\ v = 3 sec2 45° × 0.1 [Q q + f = 90° ]
=v
(Q the component of velocity of L along KL is zero) \ v = 3 × 2 × 0.1 = 0.6 m/s
v
B. True/False
K v L 1. KEY CONCEPT
d When the two balls are thrown vertically upwards with the
The displacement of K till K and L meet is d. same speed u then their final speed v at the point of projection
d is v2 – u2 = 2 × g × s
\ Time taken for K and L to meet will be = v
Here, s = 0
3. \ v = u for both the cases
2. T.E. = P.E. + K.E.
T.E. = Constant
At P, K.E. is minimum and P.E. is maximum. Since K.E. is
minimum speed is also minimum.
3. The pressure exerted will be different because one train is
moving in the direction of earth’s rotation and other in the
opposite direction.
EBD_7036
P-S-8 Topic-wise Solved Papers - PHYSICS
C. MCQs with ONE Correct Answer 7. (a) The equation for the given v-x graph is
v
1. (a) N v = – 0 x + v0 ... (i)
vs vs cos q x0
d dv v
W E =- 0
q dx x0
S dv v v é v ù
vr \ v = – ´ v = – 0 ê – 0 x + v0 ú from (1)
dx x0 x0 ë x0 û
d 2 2
Time taken to cross the river t = v0 v0 é dv ù
vs cos q \ a= 2 x- .... (ii) Qa =v ú
x0 x0 ê
ë dx û
NOTE : For time to be minimum, cos q = maximum
Þ q = 0° On comparing the equation (ii) with equation of a
The swimmer should swim due north. straight line
r y = mx + c
2. (b) Shortest route corresponds to vb perpendicular to river
flow v02
®
vr we get m = = + ve,
d d x02
\t= =
vb vbr - vr2
2 ® i.e. tan q = + ve, i.e., q is acute.
®
vbr v
b d
1 1 v2
or = Also c = - 0 ,
4 x02
25 - vr2
i.e., the y-intercept is negative
Þ vr = 3 km/h The above conditions are satisfied in graph (a).
| displacement | 8. (a) At t = 0, the relative velocity will be zero.
3. (b) | Average velocity | = T
time At t = , the relative velocity will be maximum in
2r 1 4
= = 2 ´ = 2 m/s. magnitude.
t 1 T
4. (a) KEY CONCEPT At t = , the relative velocity will be zero.
Before hitting the ground, the velocity v is given by 2
v2 = 2 gd (quadratic equation and hence parabolic path) 3T
At t = , the relative velocity will be maximum in
Downwards direction means negative velocity. After 4
magnitude
collision, the direction become positive and velocity
At t = T, the relative velocity again becomes zero.
decreases. w
ædö 9. (c) R
Further, v '2 = 2 g ´ ç ÷ = gd ; R
è 2ø Q Q
v O O
æ vö
\ çè ÷ø = 2 or v = v ' 2 Þ v ' = 2 45°
v'
P P
As the direction is reversed and speed is decreased
p
graph (a) represents these conditions correctly. t =0 t=
4w
a p pR
5. (a) sn = (2n – 1); The x-coordinate of P = vx × t = wR × =
2 4w 4
a a This horizontal distance travelled will be greater than
sn + 1 = [2(n + 1) – 1] = (2n + 1)
2 2 any point on the disc between O and P. Therefore the
sn 2n - 1 landing will be in unshaded area. In the same way, the
=
sn +1 2n + 1 horizontal distance travelled by Q is always less than
6. (b) Change in velocity = area under the graph that of any point between O and R. Therefore the landing
will be in unshaded area.
1
= ´ 10 ´ 11 = 55 m / s
2 D. MCQs with ONE or MORE THAN ONE Correct
1. (b) Average acceleration
uur ur uur ur r
r v f - vi v f + (-vi ) D v
a= = =
t t t
uur ur
To find the resultant of v f and -vi , we draw a diagram
Since, initial velocity is zero, final velocity is 55 m/s.
Motion P-S-9

N
® x2 y2
vf +=1
® a 2 b2
Dv \ The path of the particle
W E is an ellipse.
From the given equations
we can find,
dx d2x
S -vi = v x = - ap sin pt ; 2
= ax = - ap 2 cos pt
r dt dt
| Dv |= v 2f + vi2 = 52 + 52 = 5 2 m/s dy
= v y = pb cos pt
r 5 2 dt
1
a = = d2y
10 2 and = a y = -bp 2 sin pt
uur uuur r 2
dt
Since, | v f | = | -vi | , \ v is directed towards N – W.
p p
2. (a, c, d) Note : a cannot remain positive for all t in the At time t = or pt =
interval 0 < t < 1. This is because since the body starts 2p 2
from rest, it will first accelerate. Finally it stops therefore p
ax and v y become zero (because cos = 0 ). Only
a will become negative. Therefore a will change its 2
direction. Options (a) and (d) are correct. vx and ay are left,
or we can say that velocity is along negative x-axis and
t=0 t=1 acceleration along negative y-axis.
A C p
x=0 B1 B B2 x=1 Hence, at t = , velocity and acceleration of the
2p
Let the particle accelerate uniformly till half the distance
particle are normal to each other.
(A to B) and then retard uniformly in the remaining half r
distance (B to C). At t = t, position of the particle r (t ) = xi$ + y $j
= a cos pt $i + b sin pt ˆj and acceleration of the particle
The total time is 1 sec. Therefore the time taken from
A to B is 0.5 sec.
is
For A to B r
1 2 a(t ) = ax $i + a y $j = - p 2 [a cos pt $i + b sin pt $j ]
1
S = ut + at 0.5 = 0 + ´ a ´ (0.5) 2 r
2 2 = – p2 [ xi$ + y $j ] = - p 2 r (t )
\ a = 4 m/s2 Therefore, acceleration of the particle is always directed
\ VB = 0 + 4 × 0.5 = 2 m/s2 towards origin.
Note : Now, if the particle accelerates till B2 then for p
covering the same total distance in same time, At t = 0, particle is at (a, 0) and at t = , particle is at
2p
acceleration should be less than 4 m/s2 but |deceleration|
(0, b). Therefore, the distance covered is one fourth of
should be greater than 4 m/s2. And if the particle
the elliptical path and not a.
accelerates till B1, then for covering the same total
distance in the same time, the acceleration should be E. Subjective Problems
greater than 4 m/s2 and | deceleration | < 4 m/s2.
The same is depicted by the graph. 1.

Distance travelled = area of D ABC


1 1
= ´ base ´ altitude = ´ t ´ vmax
2 2
So, the | acceleration | must be greater than or equal to
4 m/s2 at some point or points in the path. 1 ab 1 æ ab ö 2
= ´t ´ t = ç t
x 2 a+b 2 è a + bø÷
3. (a, b, c) x = a cos pt Þ cos (pt) = ... (1)
a dx
y 2. x = t - 3 Þ x = t2 + 9 – 6 t \ v = = 2t - 6
y = b sin pt Þ sin (pt) = ... (2) dt
b (i) For velocity to be zero, 2t – 6 = 0 Þ t = 3 sec.
Squaring and adding (1) and (2), we get, The displacement is x = 9 + 9 – 6 × 3 = 0
EBD_7036
P-S-10 Topic-wise Solved Papers - PHYSICS
Since, the horizontal momentum comes out to be zero, the
dv
(ii) a = = 2 \ At t = 0, v = – 6 ms–1 combination of masses will drop vertically downwards and
dt fall at E.
At t = 6 sec, v = 6 ms–1 2
\ Work done = Change in K.E. = [K.Ef – K.Ei] BE = PM = 10 t = 10 × = 11.547 m
3
1 1 6. For Bullet A. Let t be the time taken by bullet A to reach P.
= m(6)2 - m(6)2 = 0
2 2 Vertical motion
3. uy = 0; sy = 10 – y; ay = 10 m/s2; ty = t
1 2
sy = uy t + a t
2 y
10 – y = 5t 2 ... (i)

As shown, at a given instant of time, the body is at two


different positions A and B which is not possible.
4. If a body drops from a height H above the ground then
the time taken by it to reach the ground
2H 2 ´ 61.25
t= \ t= = 3.53 s
g 9.8
Horizontal motion
5. (i) Let t be the time taken for collision.
For mass m thrown horizontally from A. x = 5 3t ... (ii)
For horizontal motion For bullet B.
PM = 10 t ... (i) Let (t + t ') be the time taken by bullet B to reach P
For vertical motion Vertical Motion
uy = 0; sy = y; ay = g; ty = t Let us consider upward direction negative and downward
positive. Then
1 2
\ y= gt ... (ii) uy = - 5 3 sin 60° = –7.5 m/s, ay = + 10 m/s2
2
1 2
vy = uy + ay t = gt ... (iii) sy = + (10 – y); ty = t + t ', sy = uy t + a y t
2
10 – y = – 7.5 (t + t ') + 5 (t + t')2 ... (iii)
For mass 2m thrown from C
Horizontal motion
For horizontal motion QM = [10 cos 60°] t
QM = 5 t ... (iv) x = (5 3 cos 60°) (t + t ')

For vertical motion vy = 10 sin 60° = 5 3 ; ay = g Þ 5 3 t + 5 3 t ' = 2x ... (iv)


sy = y + 10; ty = t Substituting the value of x from (ii) in (iv), we get
Now, vy = 5 3 + gt ... (v) 5 3 t + 5 3 t ' = 10 3 t
1 Þ t = t'
and (sy ) = uy t + ay t2 Putting t = t ' in eq. (iii)
2
1 2 y – 10 = 15 t – 20 t2 ... (v)
Þ y + 10 = 5 3 t + gt ... (vi) Adding (i) and (v)
2
From (ii) and (vi) 0 = 15 t – 15t2 Þ t = 1 sec.
1 2 1 2 (ii) Putting t = 1 in eq. (ii), we get x = 5 3
gt + 10 = 5 3 t + gt 2 Þ t = sec Putting t = 1 in eq. (i), we get y = 5
2 2 3
2 Therefore, the coordinates of point P are ( 5 3 , 5) in metres.
\ BD = PM + MQ = 10 t + 5 t = 15 t = 15 × 7. (a) u is the relative velocity of the particle with respect to
3
the box. Resolve u.
= 10 3 = 17.32 m ux is the relative velocity of particle with respect to the box
(ii) Applying conservation of linear momentum (during in x-direction.
collision of the masses at M) in the horizontal uy is the relative velocity with respect to the box in
direction y-direction.
m × 10 – 2 m 10 cos 60° = 3 m × vx Since, there is no velocity of the box in the y-direction,
Þ 10 m – 10 m = 3 m × vx Þ vx = 0 therefore this is the vertical velocity of the particle with
respect to ground also.
Motion P-S-11

Y-direction motion (Taking relative terms w.r.t. box) (ii) Horizontal displacement of stone
uy = + u sin a = 3 + displacement of object A.
ay = – g cos q 1
sy = 0 (activity is taken till the time the particle comes Therefore, (u cos q )t= 3 + at2
2
back to the box.)
where a = 1.5 m/s2
ty = t
1 1 or (u cos q )t = 3 + 0.75 t2 ... (ii)
2
sy = u y t + a y t Þ 0 = (u sin a) t – g cos q ´ t 2 (iii) Horizontal component of velocity of stone = vertical
2 2
2u sin a component (because velocity vector is inclined at 45°
Þ t = 0 or t = with horizontal.)
g cos q
X - direction motion (Taking relative terms w.r.t. box) Therefore (u cos q ) = gt – (u sin q ) ... (iii)
ux = + u cos a ; ax = 0, tx = t, sx = sx (The right hand side is written gt – u sin q because the
stone is in its downward motion. Therefore, gt > u sin q .
1 2 2u sin a u 2 sin 2a
u
sx = x t + a x t Þ sx= u cos a × = In upward motion usin q > gt). Multiplying equation
2 g cos q g cos q
(iii) with t we can write,
(b) For the observer (on ground) to see the horizontal
displacement to be zero, the distance travelled by the box in (u cos q ) t + (u sin q ) t = 10 t2 ... (iv)
2
Now, (iv) – (ii) – (i) gives 4.25 t – 4.25 = 0 or t = 1s
æ 2u sin a ö Substituting t = 1s in (i) and (ii), we get,
time ç should be equal to the range of the particle.
è g cos q ø÷ u sin q = 6.25 m/s or uy = 6.25 m/s
Let the speed of the box at the time of projection of particle and u cos q = 3.75 m/s.
r
or ux = 3.75 m/s therefore u = u x $i + u y $j
be U. Then for the motion of box with respect to ground.
r
or u = (3.75 $i + 6.25 $j ) m/s
9. (a) Let the ball strike the trolley at B. Let
r
v BG = velocity of ball w.r.t.ground
r
vTG = velocity of trolley w.r.t.ground
\ Velocity of ball w.r.t. trolley
r r r B
2u sin a -u 2 sin 2a v BT = v BG - vTG ... (i)
ux = – U ; ax= – g sin q ; t y = ; sx = v BG vTG
g cos q g cos q From triangle OAB
uuur uuur uuur A
1 2 OA + AB = OB
sx = u xt + axt uuur r r
2 \ OA + vTG = v BG
-u 2 sin 2a æ 2u sin a ö 1 æ 2u sin a ö
2 uuur r r
= -U ç - g sin q ç \ OA = v BG - vTG ... (ii) 45°
g cos q è g cos q ø÷ 2 è g cos q ø÷ uuur r O M
From (i) and (ii) OA = v BT
u cos (a + q)
On solving we get U = Þ velocity of ball w.r.t. trolley makes an angle of 45°
cos q with the X-axis
8. Let 't' be the time after which the stone hits the object and q
r (b) Here q = 45° B
be the angle which the velocity vector u makes with 4q 4 ´ 45 v BG
horizontal. \ f= = = 60° vTG
3 3
In DOMA, 135° 45°
q = 45° Þ ÐOAM = 45° 45° A
\ ÐOAB = 135° f
15°
Also ÐBOA = 60° – 45° = 15° q = 45°
Using sine law in DOBA O M
v BG vTG
= Þ vBG = 2 m/s
sin135 ° sin15°
H. Assertion & Reason Type Questions
According to question, we have following three conditions. 1. (b) Statement-1 is true. For a moving observer, the near by
(i) Vertical displacement of stone is 1.25 m. objects appear to move in the opposite direction at a
1 large speed. This is because the angular speed of the
Therefore, 1.25 = ( u sin q ) t – gt2 near by object w.r.t observer is large. As the object moves
2 away the angular velocity decreases and therefore its
where g = 10 m/s2 speed seems to be less. The distant object almost
or (u sin q ) t = 1.25 + 5 t 2 ... (i) remains stationary.
EBD_7036
P-S-12 Topic-wise Solved Papers - PHYSICS
Statement-2 is the concept of relative velocity which 2. 5 vA
states that
r r r
v 21 = v 2 G – v1G
vB vB cos 30°
where G is the laboratory frame. vB sin 30°
Thus both the statement are true but statement-2 is not 30°
the correct explanation of statement-1. 30° 60°
I. Integer Value Correct Type Here
vA = vB cos 30°
1. 5 From the perspective of observer A, considering
vertical motion of the ball from the point of throw till it 3
reaches back at the initial height. \ 100 3 = vB ´
2
Uy = + 5 3 m/s, Sy = 0, ay = – 10m/s2, t = ? \ vB = 200 ms–1
5 3 m/s 10m/s displacement
Time =
A velocity
B
500 500
u \ t0 = = = 5 sec
vB sin 30° 200 ´ sin 30°
5m/s
3. 8
1 2 0.3 ms –1 0.2 ms –1 a = 2 ms –2
Applying S = ut + at Þ 0 = 5 3t - 5t 2 A x
B
2
\ t = 3 sec 4m
Considering horizontal motion from the perspective of For ball A
observer B. Let u be the speed of train at the time of throw. u1 = 0.3 ms -1 , a1 = -2ms -2 , s1 = x, t1 = t
The horizontal distance travelled by the ball = (u + 5) 3 .
1
The horizonal distance travelled by the boy \ s1 = u1t1 + a1t12
2
é 1 2ù x = 0.3t – t2 ...(1)
= êu 3 + a( 3) ú + 1.15
ë 2 û For ball B
As the boy catches the ball therefore u2 = 0.2 ms–1, a2 = 2ms–2, s2 = 4 – x, t2 = t
3 1
(u + 5) 3 = u 3 + a + 1.15 \ s2 = u2t2 + a 2 t 22
2 2
\ 5 3 = 1.5a + 1.15 \ 7.51 = 1.5a 4 – x = 0.2 t + t2 ...(2)
\ a » 5 m/s2 From (1) and (2) t = 8 sec

Section-B JEE Main/ AIEEE


1. (c) Let u be the speed with which the ball of mass m is 2. (b) Ball A is thrown upwards from the
projected. Then the kinetic energy (E) at the point of building. During its downward u
projection is journey when it comes back to the A u
u
point of throw, its speed is equal to
1 2 u 2
E = mu ...(i) the speed of throw. So, for the
2 45° journey of both the balls from point
h
When the ball is at the highest point of its flight, the A to B . B
u We can apply v2 – u2 = 2gh.
speed of the ball is (Remember that the horizontal As u, g, h are same for both the balls, vA = vB
2
component of velocity does not change during a 3. (c) Case-1 : u = 50 ´ 5 m / s, v = 0,s = 6m, a = a
projectile motion). 18
\ The kinetic energy at the highest point 2
æ 5ö
v 2 - u 2 = 2as Þ 02 - ç 50 ´ ÷ = 2 ´ a ´ 6
2 è 18 ø
1 æ u ö
m 1 mu 2 E
= = = [From (i)]
2 çè 2 ÷ø 2 2 2 æ 5ö
2
Þ - ç 50 ´ ÷ = 2 ´ a ´ 6 ....(i)
è 18 ø
Motion P-S-13

5 or u2 = 2ad … (1)
Case-2 : u = 100 ´ m/sec , v = 0, s = s, a = a 2
18 and (0)2 – u'2 = –2ad'
or u ' = 2 ad ' …(2)
\ v - u = 2as
2 2 d'
(2) divided by (1) gives, 4 = Þ d ' = 4 ´ 20 = 80m
2 d
2 æ 5ö 11. (c) Yes, the person can catch the ball when horizontal
Þ 0 - ç100 ´ ÷ = 2as
è 18 ø velocity is equal to the horizontal component of ball’s
2 velocity, the motion of ball will be only in vertical
æ 5ö direction with respect to person for that,
Þ - ç100 ´ ÷ = 2as … (ii)
è 18 ø vo
Dividing (i) and (ii) we get = vo cos q or q = 60°
2
100 ´ 100 2 ´ a ´ s 1 2 2
= Þ s = 24m 12. (d) Distance from A to B = S = f t1 Þ ft1 = 2 S
50 ´ 50 2´a´6 2
Distance from B to C = ( f t1) t
4. (d) From the figure it is clear that range is required
u u2 ( f t1) 2
u 2 sin 2q Distance from C to D = = = ft12 = 2S
R= 30° 2a 2( f / 2)
g Range R
A f B C f /2 D
(10)2 sin(2 ´ 30°)
10m

10m
= =5 3 t1 t 2t 1
10 Tower

5. (b) x = at 3 and y = bt 3 15 S
Þ S + f t1t + 2 S = 15 S Þ f t1t = 12 S
dx dy
vx = = 3at 2 and v y = = 3bt 2 1 2
dt dt But f t1 = S
2 2 2 2
\ v = v 2x + v2y = 9a 2t 4 + 9b2 t 4 = 3t a + b
t
On dividing the above two equations, we get t1 =
1 1 2 6
6. (a) We know that s = ut + gt 2 , or h = gT (Q u = 0) 2
2 2 1 ætö f t2
now for T/3 second, vertical distance moved is given Þ S= fç ÷ =
2 è 6ø 72
by 13. (c) Average acceleration v2
2
1 æTö 1 gT 2 h change in velocity N
h' = g ç ÷ Þ h' = ´ = = D v = v 2 + ( -v 1 )
2 è 3ø 2 9 9 time interval
h uur
\ position of ball from ground = h - = 8h Dv
9 9 q
r r r r r r r r = W E
7. (c) A ´ B - B ´ A = 0 Þ A ´ B + A ´ B = 0 t - v1 v1
r r uur uur
\ A´ B = 0 v1 = 5iˆ, v2 = 5 ˆj
Angle between them is 0, p, or 2 p r 5 ˆj - 5iˆ ˆj - iˆ
\ a= = S
from the given options, q = p 10 2
8. (a) The angle for which the ranges are same is
12 + (-1) 2 2 1
complementary. \ a= = = ms -2
Let one angle be q, then other is 90° – q 2 2 2
2u sin q 2u cos q v 5
tan q = 2 = = 1 \ q = 45°
T1 = , T2 = v1 5
g g
Therefore the directon is North-west.
4u 2 sin q cos q u 2 sin 2 q
T1T2 = = 2 R (Q R = ) 14. (d) t = ax 2 + bx ; Diff. with respect to time (t)
g g d d dx dx dx
Hence it is proportional to R. (t ) = a ( x 2 ) + b = a.2 x + b.
dt dt dt dt dt
9. (b) Only option (b) is false since acceleration vector is
always radial (i.e . towards the center) for uniform 1
1 = 2axv + bv = v (2ax + b) Þ 2ax + b = .
circular motion. v
5 50 dx 1 dv
10. (d) Speed, u = 60 ´ m/s = m/s Again differentiating, 2a + 0 = - 2
18 3 dt v dt
5 100
d = 20m, u' = 120 ´ = m/s dv 3 æ dv ö
18 3 Þ = f = - 2av çQ = f = acc ÷
Let declaration be a then (0)2 – u2 = –2ad dt è dt ø
EBD_7036
P-S-14 Topic-wise Solved Papers - PHYSICS
dx dx For upward motion.
15. (a) v=a x , =a xÞ = a dt The ball suffer elastic collision with the horizontal
dt x
elastic plate therefore the direction of velocity is
x t
dx x 2 reversed and the magnitude remains the same.
ò x = a ò dt Þ éê 2 x ùú = a[t ]0t Þ 2 x = at Þ x = a t 2 Here v = u – gt where u is the velocity just after collision.
4
ë 1 û0
0 0
As t increases, v decreases. We get a straight line
16. (d) Let u be the velocity with which the particle is thrown between v and t with negative slope.
and m be the mass of the particle. Then 1 2
Also y = ut - gt
1 2
K = mu 2 . ... (1) All these characteristics are represented by graph (b).
2
r r
At the highest point the velocity is u cos 60° (only the 20. (a) Given u = 3iˆ + 4 ˆj , a = 0.4iˆ + 0.3 ˆj , t = 10 s
horizontal component remains, the vertical component
being zero at the top-most point). Therefore kinetic r r r
v = u + at = 3iˆ + 4 ˆj + (0.4iˆ + 0.3 ˆj) ´ 10 = 7iˆ + 7 ˆj
energy at the highest point.
r
1 1 K \ | v |= 72 + 72 = 7 2 units
K ' = m(u cos 60°)2 = mu 2 cos2 60° = [From 1]
2 2 4 r dx ˆ dy ˆ
21. v = k ( y iˆ + x ˆj ) = v x iˆ + v y ˆj =
(d) i+ j
dx dt dt
17. (c) We know that, v = Þ dx = v dt
dt dx dy
\ = ky and \ = kx
x t dt dt
Integrating, ò dx = ò v dt dy x
\ = y dy = x dx Þ y 2 = x 2 + constant
0 0 dx y Þ
t t
é gt 2 ft 3 ù ds
2
or x = ò (v0 + gt + ft ) dt = êv0 t + + ú 22. (d) s = t 3 + 5 Þ velocity,, v = = 3t 2
2 3 úû dt
0 ëê 0
dv
gt 2 ft 3 Tangential acceleration at = = 6t
or, x = v0 t + + dt
2 3
v 2 9t 4
g f Radial acceleration ac = =
At t = 1, x = v0 + + . R R
2 3
18. (b) For the body starting from rest At t = 2s, at = 6 ´ 2 = 12 m/s2

1 2 1 2 9 ´ 16
x1 = 0 + at Þ x1 = at ac = = 7.2 m/s2
2 2 20
For the body moving with constant speed \ Resultant acceleration
x2 = vt = at2 + ac2 =
(12) 2 + (7.2) 2 = 144 + 51.84
1 2 d ( x1 - x2 ) = 195.84 = 14 m/s2
\ x1 - x2 = at - vt Þ = at - v
2 dt r
23. (c) Clearly a = ac cos q(-iˆ) + ac sin q(- ˆj )
at t = 0, x1–x2 = 0 x1 – x2
v -v 2 v2
For t < ; the slope is negative = cos q iˆ - sin q ˆj
a R R
Y
v
For t = ; the slope is zero
a v/a ac cos q P( R, q )
t q
v R
For t > ; the slope is positive q ac sin q
a X
O
These characteristics are represented by graph (b).
19. (b) For downward motion v = –gt
The velocity of the rubber ball increases in downward r r r
direction and we get a straight line between v and t 24. (c) L = m( r ´ v )
with a negative slope. r
L = m éêv0 cos qt iˆ + (v0 sin qt - gt 2 ) ˆj ùú
1
1 2 ë 2 û
Also applying y - y0 = ut + at
2 ´ éë v0 cos q iˆ + (v0 sin q - gt ) ˆj ùû
1 2 1 2
We get y - h = - gt Þ y = h - gt
2 2 é 1 ùˆ 1
= mv0 cos qt ê - gt ú k = - mgv0 t cos qkˆ
2
The graph between y and t is a parabola with y = h at ë 2 û 2
t = 0. As time increases y decreases.
Motion P-S-15

dv dv v
t
= -2.5 v Þ dv F0 -bt F0 - bt
25. (a) = – 2.5 dt Þò

dt v = e dv = e dt
dt m
0 -½ t 0 0
Integrating, ò6.25 v dv = -2.5ò dt
0
-bt ù t
ée
é v +½ ù
ê
0
ú = -2.5 [ t ]0
t
v=
F0
m
ê ú =
êë -b úû 0
F0 é
mb ë (
- e - bt - e -0 ù
û )
Þ
êë (½) úû 6.25
F0 é
Þ – 2(6.25)½ = – 2.5t Þ t = 2 sec Þ v= 1 - e -bt ù
mb ë û
26. (a) Total area around fountain r ˆ
4 30. (b) u = i + 2 ˆj = u x iˆ + u y ˆj Þ u cos q = 1, u sin q = 2
2
A = pRmax v
= p 2
g 1 gx 2
y = x tan q -
v 2 sin 2q v 2 sin 90° v 2 2 u x2
[Q Rmax = = = ]
g g g 1 2
\ y = 2x - gx = 2 x - 5x 2
u2 sin 2 q u sin q 2 2 2
27. (d) R= , H= ; Hmax at 2q =90 u
g 2g 31. (c) Speed on reaching ground v = u 2 + 2 gh
Now, v = u + at H
u2
Hmax = ; = 10 Þ u 2 = 10 g ´ 2
2g Þ u 2 + 2 gh = -u + gt
u 2 sin 2q u2 u
R= Þ Rmax = Time taken to reach highest point is t = ,
(g ) g g
10 ´ g ´ 2 u + u 2 + 2 gH nu
Rmax = = 20 meter Þ t= = (from question)
g g g
2
æ 2p ö Þ 2gH = n(n –2)u2
28. (c) a = r w2 = r ´ ç ÷ 32. (b) y1 = 10t – 5t2 ; y2 = 40t – 5t2
èT ø
a1 r1 for y1 = – 240m, t = 8s
\ a =r [Q T is same] \ y2 – y1 = 30t for t < 8s.
2 2 for t > 8s,
dv 1
29. (b) Given that F(t) = F0 e -bt Þ m = F0 e -bt y2 – y1 = 240 – 40t – gt2
dt 2
EBD_7036
3
P-S-16 Topic-wise Solved Papers - PHYSICS

Laws of Motion
Section-A : JEE Advanced/ IIT-JEE
A 1. 5 2. rLa / 2
B 1. F 2. F 3. T 4. F
C 1. (c) 2. (a) 3. (b) 4. (a) 5. (a)
6. (c) 7. (d) 8. (a) 9. (a) 10. (c)
11. (b) 12. (d) 13. (b) 14. (a) 15. (d)
16. (c)
D 1. (b) 2. (b, d) 3. (b, c) 4. (a) 5. (a, c) 6. (d)
(m sin a + m2 sin b) g m m g sin(a - b)
E 1. 71.05 N 2. f = 1 ; T= 1 2
m1 cos a + m2 cos b m1 cos a + m2 cos b
æ lö
3. T = F ç1 - ÷ 4. 4.2 Kg, 9.8 N
è Lø
5. mgsinq, tan–1m 6. 20N, 50N

5 3 3mg æp 3 ö÷
ç
7. g, 8. (a) -1m / s (b) ç 3 + 4 ÷ sec
8 8 è ø
9. (b) F = 60 N; T = 18 N
3
a = m/s2 , f1 = 15 N, f2 = 30 N
5
10. 8 2m, 7 2 m, 2 sec . 11. 10 m/s2
F 1. (d)
G 1. (a) 2. (b)
H 1. (b) 2. (b)
I 1. 5
Section-B : JEE Main/ AIEEE
1. (a) 2. (c) 3. (a) 4. (b) 5. (d) 6. (b)
7. (b) 8. (c) 9. (a) 10. (d) 11. (d) 12. (d)
13. (d) 14. (a) 15. (b) 16. (c) 17. (c) 18. (b)
19. (d) 20. (c) 21. (c) 22. (d) 23. (c) 24. (c)
25. (a) 26. (d) 27. (d) 28. (c) 29. (b) 30. (d)
31. (a) 32. (b) 33. (a) 34. (a) 35. (a)

Section-A JEE Advanced/ IIT-JEE


A. Fill in the Blanks B. True/ False
1. As seen by the observer on the ground, the frictional force 1. KEY CONCEPT : The rocket moves Reaction
is responsible to move the mass with an acceleration of forward when the exhaust gases are
5 m/s2. thrown backward.
Therefore, frictional force = m × a = 1 × 5 = 5 N. Here exhaust gases thrown backwards
2. Let A be the area of cross-section of the rod. is action and rocket moving forward
Consider the back half portion of the rod. is reaction.

rAL Note : This phenomenon takes place Exhaust gases


Mass of half portion of the rod =
2 in the absence of air as well. Action
The force responsible for its acceleration is 2. KEY CONCEPT : Friction force opposes the relative motion
of the surface of contact.
rAL f rLa When a person walks on a rough surface, the foot is the
f= ´a \ Stress = =
2 A 2 surface of contact. When he pushes the foot backward, the
Laws of Motion P-S-17

motion of surface of contact tends to be backwards. The limiting frictional force


Therefore the frictional force will act forward (in the direction
of motion of the person) 3
fl = µs mg cos q = 0.7 × 2 × 9.8 × = 11.8 N
2
Note : The frictional force is never greater than the force
tending to produce relative motion.
f Therefore the frictional force is 9.8 N
3. (b) Limiting frictional force, fl = µsN = 0.5 × 5 = 2.5 N. But
f force
3. As the angular amplitude of the pendulum is 40°, the bob tending to produce relative
m=0.5
will be in the mid of the equilibrium position and the extreme motion is the weight (W) of
position as shown in the figure the block which is less than
f
mv 2 fl. Therefore, the frictional
Note : For equilibrium of the bob, T – mg cos 20° = , force is equal to the weight, 5N
l the magnitude of the 5N
where l is the length of the pendulum and is the velocity of
frictional force f has to 5N
the bob.
balance the weight 0.98 N
40°
mv 2 acting downwards. 0.1 × 9.8
\ T = mg cos 20° + 20°
= 0.98 N = W
l T
l Therefore the frictional force = 0.98 N.
mv 2
is always a positive quantity.. 4. (a) Since the body presses the surface with a force N hence
l according to Newton's third law the surface presses
Hence, T > mg cos 20°. 20° the body with a force N. The other force acting on the
4. Case (i) For mass m mg cos 20°
mg body is its weight mg.
T – mg = ma ... (i)
For mass 2m N
2mg – T = 2ma ... (ii)
N

mg

a
T
T T=2mg
For circular motion to take place, a centripetal force is
T
a a' required which is provided by (mg + N).
mg
Case (i) 2 mg mg F=2mg mv 2
\ mg + N =
Case (ii) r
From (i) and (ii) where r is the radius of curvature at the top.
a = g/3
If the surface is smooth then on applying conservation
Case (ii) T – mg = ma'
of mechanical energy, the velocity of the body is always
2mg – mg = ma' [Q T = 2mg]
\ a' = g same at the top most point. Hence, N and r have inverse
Hence, a < a' relationship. From the figure it is clear that r is minimum
for first figure, therefore N will be maximum.
C. MCQs with ONE Correct Answer Note : If we do not assume the surface to be smooth,
1. (c) F = ma we cannot reach to a conclusion.
5. (a) KEY CONCEPT :
F 5 ´ 104 5
Þ a= = = ´ 10-3 ms -2 For the maximum possible value of a,
m 3 ´ 10 7 3 mg sin a will also be maximum and equal to the frictional
Also, v2 – u2 = 2as force.
5 -3 -2 In this case f is the limiting friction. The two forces
Þ v2 – 02 = 2 ´ ´ 10 ´ 3 = 10
3 acting on the insect are mg and N. Let us resolve mg
Þ v = 0.1 ms–1 into two components.
m=1/3
2. (a) The force acting on the block along the incline to shift mg cos a balances N. a
the block downwards mg sin a is balanced by f N
N f
the frictional force.
\ N = mg cos a mgcosa
a
f = mg sin a mg mgsina
mg cos 30°
mg sin 30° mg But f = µN = µ mg cos a
1
30° \ µ mg cos a = mg sin a Þ cot a = Þ cot a = 3
µ
= mg sin q = 2 × 9.8 sin 30° = 9.8 N
EBD_7036
P-S-18 Topic-wise Solved Papers - PHYSICS
6. (c) The tension in both strings will be same due to symmetry. The force of friction provides this acceleration.
æ kA ö kA
\ f = ma = m ç =
è 2m ÷ø 2
Tcosq Tcosq
10. (c) In situation 1, the tension T has to hold both the masses
q q
T T 2m and m therefore,
T Tsinq B Tsinq T T = 3mg
A C In situation 2, when the string is cut, the mass m is a
Ö2 mg
mg mg
freely falling body and its acceleration due to gravity
For equilibrium in vertical direction for body B we have is g.
For mass 2m, just after the string is cut, T remains 3mg
2 mg = 2T cos q because of the extension of string.
\ 2 mg = 2(mg ) cos q [Q T = mg, (at equilibrium] g
\ 3mg – 2mg = 2m × a \ =a
1 2
\ cos q = Þ q = 45° 11. (b) The acceleration of mass m is due to the force T cos q
2
7. (d) At equilibrium T = Mg T cos q
\ T cos q = ma Þ a= ... (i)
T m
F
also, F = 2T sin q Þ T= ... (ii)
T=Mg 2 sin q
T From (i) and (ii)
mg T F
æ F ö cos q
a= ç ÷
F è 2sin q ø m T T
Mg F1=(m+M)g
F.B.D. of pulley
F F x a
= = Tsinq Tsinq a
F1 = (m + M) g 2m tan q 2m a 2 - x 2 T
The resultant force on pulley is é q Tcosq
a 2 - x2 ù
2 2 êQ tan q ú x x
F = F12 + T 2 = [ (m + M ) + M ] g ê x ú
ë û
8. (a) The forces acting on the block are shown. Since the ur
block is not moving forward for the maximum force F 12. (d) p (t ) = A [i$ cos (kt ) - $j sin (kt )]
applied, therefore ur
ur d p
F cos 60° = f = µN ... (i) (Horizontal Direction) F= = Ak [-$i sin (kt ) - $j cos (kt )]
Note : For maximum force F, the frictional force is the dt
r r ur ur
limiting friction = µN] Here, F . P = 0 But F . p = Fp cos q
and F sin 60° + mg = N... (ii)
\ cos q = 0 Þ q = 90°.
From (i) and (ii)
13. (b) For the block to slide, the angle of inclination should
N be equal to the angle of repose, i.e.,
tan -1 m = tan -1 3 = 60°.
o
F cos 60
Therefore, option (a) is wrong.
o
mg 60 For the block to topple, the condition of the block will
f
be as shown in the figure.
F N
Fsin60o
f
F cos 60° = µ [F sin 60° + mg]
O 15 c
m
m mg q
Þ F= 10
cm
cos 60° - m sin 60° P
M
1 w
´ 3 ´ 10
2 3 5
= = = 20 N
1 1 3 1 q
- ´
2 2 3 2 4
PM 5 cm 2
9. (a) Let w be the angular frequency of the system. In D POM , tan q = = =
OM 7.5 cm 3
The maximum acceleration of the system,
For this, q < 60°. From this we can conclude that the
é k k ù block will topple at lesser angle of inclination. Thus
æ k ö êw = = ú
a = w2 A = ç A the block will remain at rest on the plane up to a certain
è 2m ÷ø ë m+m 2m û
anlgle q and then it will topple.
Laws of Motion P-S-19

14. (a) As tan q > m, the block has a tendency to move down D. MCQs with ONE or MORE THAN ONE Correct
the incline. Therfore a force P is applied upwards along
the incline. Here, at equilibrium P + f = mg sin q Þ f = 1. (b) This is a problem based on constraint motion. The
mg sin q - P motion of mass M is constraint with the motion of P
N and Q. Let AN = x, NO = z. Then velocity of mass is
f
N dz dl
. Also, let OA = l . then =U
P dt dt
q
mg sinq mg cosq From DANO, using pythagorous theorem
P q
mg cosq A x N B
mg sinq f \ x2 + z2 = l2
mg
mg Here x is a constant. Z
Differentiating the above l
q q
equation w.r.t to t
P U qq U
Q
Now as P increases, f decreases linearly with respect O
to P. dz dl
0 + 2z = 2l Þ zvM = lU M
When P = mg sin q, f = 0. dt dt
When P is increased further, the block has a tendency
l U U æ zö
to move upwards along the incline. Þ vM = U= = çèQ cos q = ÷ø
Therefore the frictional force acts downwards along the z z / l cos q l
incline. 2. (b, d) Since earth is an accelerated frame and hence, cannot
Here, at equilibrium P = f + mg sin q be an inertial frame.
Note : Strictly speaking Earth is accelerated reference
\ f = P – mg sin q
frame. Earth is treated as a reference frame for practical
Now as P increases, f increases linearly w.r.t P. examples and Newton's laws are applicable to it only
This is represented by graph (a) . as a limiting case.
15. (d) Here, the horizontal component of tension provides 3. (b, c) Since the body is moving
the necessary centripetal force. in a circular path therefore q
\ T sin q = mrw2 it needs centripetal force l
æ Mv ö2
T
From (i) and (ii) q
ç ÷.
L
è l ø
r r
T´ = mr w2 [Q sin q = ] T cos q T
L L Mv 2 v q
q \ T – Mg cos q =
T sinq l Mgsinq Mg Mgcos q
T = 324 18
\w= = = r
mL 0.5 ´ 0.5 0.5
mg
q
324 18
= = = 36 rad/s
0.5 0.5 0.5
16. (c) For a plano convex lens
1 (µ - 1) 1 1 q
= = - ...(i) gsinq g gcosq
f R v u
Also, the tangential acceleration acting on the mass is
l l 3
Here µ = a = a = = 1.5 g sin q .
lm 2 l 2 4. (a) At A the horizontal speeds of both the masses is the
3 a same. The velocity of Q remains the same in horizontal
Where l = wavelength of light in air as no force is acting in the horizontal direction. But in
a
l = wavelength of light in water case of P as shown at any intermediate position, the
m horizontal velocity first increases (due to N sin q ),
R reaches a max value at O and then decreases. Thus it
v = 8m always remains greater than v. Therefore tP < tQ.
v 1
Also m = =- Q q
u 3 A B
v
\ u = -24 cm. Ncosq
q
N
1.5 - 1 1 æ 1 ö 1 1 1 Nsinq
From (i) = -ç ÷= + = P
R 8 è -24 ø 8 24 6 mg O
\ R = 3m option (c) is correct 5. (a, c) The forces are resolved as shown in the figure.
When q = 45º, sinq = cosq
EBD_7036
P-S-20 Topic-wise Solved Papers - PHYSICS
Q The F.B.D. for mass C is
cos q f2 = m (m A + mB ) g
R F C T
q f1 = m (mA + mB + mC ) g
1N
sin q As C is moving with constant speed F = f1 + f2 + T ... (i)
q
sin q F.B.D. for mass B is
cos q
1N mm A g = f3
B T
FIXED q m (m A + mB ) g = f 2
P
The block will remain stationary and the frictional As B is moving with constant speed f2 + f3 = T (ii)
force is zero. Subtracting (ii) from (i)
When q > 45º, sinq > cosq F – (f2 + f3) = f1 + f2 + T – T = f1 + f2
Therefore a frictional force acts towards Q. Þ F = f1 + 2f2 + f3 = µ (mA + mB + mC) g +
6. (d) h N 2µ (mA + mB) g + µ mA g
v F = µ (4 mA + 3 mB + mC) g
mg sin q = 0.25 [4 × 3 + 3 × 4 + 5] × 9.8 = 71.05 N
q
2. Without Pseudo Force
mg cosq mg F.B.D for mass m2
N2 cos b = T sin b + m2g ... (i)
q
and (N2 sin b + T cos b) = m2 f ... (ii)
N2 cos b N1 cos a
N2 N1
As the bead is moving in the circular path T T sin a

mv 2 b N2 sin b a a N1 sin b
\ mg cos q - N = )

)
)

R ) b T cos a
T cos b
2
mv T sin b
\ N = mg cos q - ...(1) T
R m2g m1g
1 FBD for mass m1
By energy conservation, mv 2 = mg [ R - R cos q ]
2 N1 cos a + T sin a = m1g ... (iii)
and (N1 sin a – T cos a) = m1 f ... (iv)
v2
\ = 2 g (1 - cos q ) ..(2) On solving the four equations, we get the above results.
R
From (1) and (2) M
3. From equation (i) T = ( L - l) a
L
N = mg cos q - m [2 g - 2 g cos q]
T æ L - lö æ lö
N = mg cos q - 2mg + 2mg cos q Also, F = Ma \ =ç ÷ Þ T = F ç1 - ÷
F è L ø è Lø
N = 3mg cos q - 2mg 4. (a) If M1, M2 and M3 are considered as a system, then the
force responsible to more them is M1g and the retarding
Þ N = mg ( 3cos q - 2 ) force is (M2g sin q + µM2g cos q + µM3g). These two should
Clearly N is positive (acts radially outwards) when be equal as the system is moving with constant velocity.
2 5. Let F be the force applied to move the body at an angle q to
cos q >
3 the horizontal. Fsinq
2 The body will move when
Similarly, N acts radially inwards if cos q < F
N
3 F cos q = µ N ... (i)
µN q
E. Subjective Problems Applying equilibrium of forces Fcosq
1. When force F is applied on C, the block C will move towards in the vertical direction we get
left. F sin q + N = mg
mg
A Þ N = mg – F sin q ... (ii)
T Þ From (i) and (ii)
B
C m mg
F T F= ... (iii)
cos q + m sin q
Laws of Motion P-S-21

Differentiating the above equation w.r.t. q , we get In DOCP, OC = CP = R


\ ÐCOP = ÐCPO = 60° Þ ÐOCP = 60°
dF m mg
= [- sin q + m cos q] = 0 \ DOCP is an equilateral triangle Þ OP = R
d q (cos q + m sin q ) 2
3R R
Þ q = tan–1µ \ Extension of string = R - = =x
4 4
This is the angle for minimum force. The forces acting are shown in the figure (i)
To find the minimum force substituting these values in The free body diagram of the ring is shown in fig. (ii)
equation (iii) Force in the tangential direction
µ 1 = F cos 30° + mg cos 30
sin q = , cos q = m

+1
2 2 = [kx + mg] cos 30°
µ +1 µ +1

Öm
5 mg 5 3
µ mg Ft = 3 \ Ft = mat Þ at = g
F= q 8 8
1 m 1 Also, when the ring is just released
+ ´m
2 2 N + F sin 30° = mg sin 30°
µ +1 µ +1
æ mg ö 1 3mg
µ mg ( µ2 + 1) µ mg Þ N = (mg – F) sin 30° = çè mg - ÷´ =
Þ F= = 4 ø 2 8
µ2 + 1 µ2 + 1 8. –2
m = 10 kg, motion is along positive X-axis
v=0
Þ F = mg sin q
0 x
6. Let l be the mass per unit length of lower wire. F 1m
Let us consider the dotted portion as a system and the
t=0
tension T accelerates the system upwards
K
\ T - (m1 + l l) g = (m1 + l l) a F (x) = – 2 , K = 10–2 Nm2; At t = 0, x = 1.0 m
2x
and V = 0
\ T = (m1 + l l) (a + g )
-K æ dV ö K
= (1.9 + 0.2 × 0.5) (9.8 + 0.2) = 2 × 10 = 20 N (a) F (x) = or m ç ÷ V=- 2
2
To find tension T ' Let us consider the dotted portion given 2x è dx ø 2x
in figure (2) v x K
or
0
ò
m VdV = -
1 2 x2
ò
dx

x
T' mV 2 é K ù K æ1 ö
a=0.2m/s2 a or = ê ú = ç - 1÷
m2 m2 2 ë û1 2 è x ø
2 x
T l
2l K æ1 ö K æ1 ö
l 2l or V2 = çè - 1÷ø or | V | = ± ç - 1÷ ... (i)
m x mèx ø
m1 m1
Initially the particle was moving in + X direction at x = 1.
m1g+l lg (m1+m2+2ll)g
When the particle is at x = 0.5, obviously its velocity will be
Fig. (1) Fig. (2)
in –X direction. The force acting in –X direction first
T '- (m2 g + l ´ 2 lg + m1g ) = (m1 + l 2 l + m2 ) a decreases the speed of the particle, bring it momentarily at
rest and then changes the direction of motion of the particle.
\ T ' = (m1 + l 2 l + m2 ) (a + g )
= (1.9 + 0.2 × 1 + 2.9) (10) = 5 × 10 = 50 N. Kæ 1 ö
When x = 0.5 m : | V | = - ç - 1÷
Alternatively considering m1, m2 and lower wire as a system m è 0.5 ø
T' – 5 g = 5 a
7. K 10 -2
=- =- = -1m / s
m 10 -2
K
N (b) As = 1 m/s, hence from (i)
Fsin30 o m
C 60o dx 1- x
R
30o mgsin30o V= =-
F 30 o dt x
R N
M P O o Note : We have chosen –ve sign because force tends
F=kx Fcos30 mg
60o
to decrease the displacement with time
mg mgcos30o
x 0.25 x t
O
Fig (i) tangential Fig (ii)
1- x
dx = - dt ; ò1 1- x
dx = ò0 -dt
EBD_7036
P-S-22 Topic-wise Solved Papers - PHYSICS
Put x = sin2 q , dx = 2 sin q cos q d q 10. mN = fK
N = mg cos q
p/6
ò p / 2 2sin
2
So, q d q = -t
a
q
2 2
cos 2q = 1 - 2sin q ; 2sin q = 1 - cos 2q mgcosq mgsinq
p/6 p/6 mg
é 2q ù
ò p / 2 (1 - cos 2q) d q = -t; êëq - sin 2 úû p /2 = -t a=
mg sin q - m k mg cos q
m
p 1 p p 1
- sin - - sin p = -t \ aA = g sin q – µk,A g cos q ... (i)
6 2 3 2 2 and aB = g sin q – µk,B g cos q ... (ii)
æp 3ö Putting values we get
\ t= ç + ÷ sec.
è3 4 ø aA = 4 2 m / s 2 and aB = 3.5 2 m / s 2
9. Given m1 = 20 kg, m2 = 5 kg, M = 50 kg, µ = 0.3 and Let aAB is relative acceleration of A w.r.t. B. Then
g = 10 m/s2 aAB = aA – aB
(A) Free body diagram of mass M is
L= 2m
T N1 [where L is the relative distance between A and B]
T f1 1
T
Mg F Then L = a AB t 2
2
T
2L 2L
or t2 = =
N a AB a A - aB
(B) The maximum value of f1 is Putting values we get, t2 = 4 or t = 2s.
(f1)max = (0.3) (20) (10) = 60 N Distance moved by B during that time is given by
The maximum value of f2 is
(f2)max = (0.3) (5) (10) = 15 N 1 1
S= a B t 2 = ´ 3.5 2 ´ 4 = 7 2 m
Forces on m1 and m2 in horizontal direction are as 2 2
follows : Similarly for A = 8 2 m.
T m1 f1 f2 m2 T 11. Applying pseudo force ma and resolving it.
Applying Fnet = mar
Note : There are only two possibilities. ma cos q – (f1 + f2) = mar
(1) Either both m1 and m2 will remain stationary (w.r.t.
ma cos q – µN1 –µN2 = mar
ground) or (2) both m1 and m2 will move (w.r.t.
ground). First case is possible when. ma cos q – µma sin q – µ mg = mar
T < (f1)max or T < 60 N Þ ar = a cos q – µa sin q – µg
and T < (f2)max or T < 15 N 4 2 3 2
These conditions will be satisfied when T < 15 N say T = 25 ´ - ´ 25 ´ - ´ 10 = 10 m / s 2
= 14 then f1 = f2 = 14 N. 5 5 5 5
Therefore the condition f1 = 2f2 will not be satisfied.
Thus m1 and m2 both can't remain stationary.
F. Match the Following
In the second case, when m1 and m2 both move 1. (d) If (m1 + m2) sin q < mN2 the bodies will be at rest
f2 = (f2) max = 15 N i.e., (m1 + m2)g sin q < mm2 g cos q
Therefore, f1 = 2f2 = 30 N
Note : Since f1 < (f1)max, there is no relative motion f = mN2
between m1 and M, i.e., all the masses move with same N2
acceleration, say 'a'.
Free body diagrams and equations of motion are as
q
follows: (m1 + m2)g cos q ( m1 + m2) sin q
a f1=30N a (m1 + m2)g
T q
m1 m2 M
T F mm2 g
f1=30N f2=15N tan q <
( m1 + m2 ) g
For m1 : 30 – T = 20 a ... (i)
For m2 : T – 15 = 5 a ... (ii) mm2
Þ tan q <
For M : F – 30 = 50 a ... (iii) m1 + m2
Solving these three equations, we get,
0.3 ´ 2
3 2 Þ tan q <
F = 60 N, T = 18 N and a = m / s . 1+ 2
5
Laws of Motion P-S-23

Þ tan q < 0.2 From (i) and (ii)


i.e., If the angle q < 11.5° the frictional force is less than ® Rw wt - wt Ù
mN2 = mm2g = 0.3 × 2 × g = 0.6 g
Fin = 2m
4
(e - e ) w j
and is equal to (m1 + m2)g sin q ®
mRw2 wt Ù

At q = 11.5° the bodies are on the verge of moving,


\ Fin =
2
( e - e - wt ) j

f = 0.6 g ® mR w2 wt Ù Ù

At q > 11.5° the bodies start moving and f = 0.6 g \ Freaction =


2
( e - e - wt ) j+ mg K

The above relationship is true for (d).


H. Assertion & Reason Type Questions
G. Comprehension Based Questions 1. (b) Statement 1 : Cloth can be pulled out without
dislodging the dishes from the table because of inertia.
dv Therefore, statement – 1 is true.
1. (a) Force on the block along slat = m rw2 = m v
dr Statement 2 : This is Newton's third law and hence
true. But statement 2 is not a correct explanation of
v r
R 2 dr statement 1.
ò Vdv = ò w 2rdr Þ V = w r - =
2
\ 2. (b) It is easier to pull a heavy object than to push it on a
o R
4 dt
2 level ground. Statement-1 is true. This is because the
r
dr
t normal reaction in the case of pulling is less as compared
\ ò R2 o
= ò w dt by pushing. (f = m N). Therefore the frictional force is
R
4 r2 - small in case of pulling.
4 statement-2 is true but is not the correct explanation of
On solving we get statement-1.
R2 R wt I. Integer Value Correct Type
r + r2 - = e
4 2
1. 5
R 2 R 2 2wt 2 -2r R e wt
or r2 – = e +r f = mN
4 4 2
R wt
\ r=
4
( e + e-wt ) F1 q F2
mg sin q mg cos q mg sin q
® ®
æ ö æÙ Ù
ö Ù Ù Ù
2. (b) Frot = Fin + 2m ç Vrot i ÷ ´ w k + m ç w k ´ r i ÷ ´ w k f mg
è ø è ø
Ù ®
æ Ù
ö Ù 45°
\ m rw2 i = Fin + 2m Vrot wç - j ÷ + mw2 r i The pushing force F1 = mg sinq + f
è ø
®
\ F1 = mg sin q + m mg cos q = mg (sin q + m cos q)
Fin = mrV Ù The force required to just prevent it from sliding down
rot w j – (i)
F2 = mg sin q – mN = mg (sin q – m cos q)
R wt Given , F1 = 3F2
But r = ée + e - wt ùû
4ë \ sin q + mcos q = 3(sin q – m cos q )
dr R \ 1 + m = 3(1 – m) [Q sin q = cos q ]
= Vr = éëwe - we ùû – (ii)
wt - wt
\ \ 4m = 2 \ m = 0.5
dt 4
\ N = 10 m = 5

Section-B JEE Main/ AIEEE


1. (a) W = DK = FS Where 'a' is the acceleration of the mass (because
1 2 1 ævö
2 the acceleration of the lift is 'a' )
mv - m ç ÷ = F ´ 3 …(i) • For the man standing on the ground, the
2 2 è2ø acceleration of the ball
1 æ vö
2 r r r
m ç ÷ - 0 = F ´ S … (ii) abm = ab - am Þ abm = g – 0 = g
2 è 2ø 3. (a) When F1 , F2 and F3 are acting on a particle then the
On dividing particle remains stationary. This means that the
1/ 4 resultant of F1 F2 and F3 is zero. When F1 is removed,
= S /3 F2 and F3 will remain. But the resultant of F2 and F3
3/ 4 r r r
\ S =1 cm should be equal and opposite to F1. i.e. | F2 + F3 | = | F1 |
2. (c) • For the man standing in the left, the acceleration r r
of the ball | F2 + F3 | F1
r r r \ a= Þ a=
abm = ab - am Þ abm = g – a m m
EBD_7036
P-S-24 Topic-wise Solved Papers - PHYSICS
4. (b) Let the two forces be F1 and F2 and let F2 < F1 . R is the 9. (a) For the bag accelerating down
resultant force. mg – T = ma
Given F1 + F2 = 18 ...(i) \ T = m( g – a )
From the figure F2 + R = F12
2 2
49 T
= (10 – 5) = 24.5 N Bag
F12 - F22 = R 2 10 a
mg
\ F12 - F22 = 144 ...(ii)
Only option (b) follows equation (i) and (ii).
F1 10. (d) As shown in the figure, the three forces are represented
by the sides of a triangle taken in the same order.
r r
R
Therefore the resultant force is zero. Fnet = ma.
F2
Therefore acceleration is also zero ie velocity remains
F1 unchanged.
11. (d) For the block to remain stationary with the wall
5. (d) DK = FS
f = W \ mN = W
1 f = mN
mu 2 = F ×S1 … (i)
2
1
m(4u ) 2 = FS2 ...(ii)
2 10N 10N 10N
Dividing (i) and (ii),
W
u22as1 1 s 0.2 × 10 = W Þ W = 2 N
= Þ = 1
2 2as2 16 s 12. (d) u = 6 m/s, v = 0, t = 10s,
16u 2
6. (b) For mass m1 f - µmg
m1g – T = m1a a= - = = -mg = -10m
m m
For mass m2 v = u + at
T–m2g = m2a 0 = 6 – 10µ × 10
\ µ = 0.06
Adding the equations we get T T 13. (d) Taking the rope and the block as a system
a m2 a
(m1 - m2 ) g m1 a
M
a= m2 g m
m1 + m2 m1 g T P
m1 P
-1 we get P = (m + M) a \a =
1 m2 m m m 9 m+ M
\ = Þ 1 +1 = 8 1 - 8 Þ 1 =
8 m1 +1 m2 m2 m2 7
m2 Taking the block as a system, we get T = Ma
MP
10.2 \ T=
7. (b) F = ( m + m + m) ´ a \a = m / s2 m+M
6 14. (a) The Earth pulls the block by a force
10.2 Mg. The block in turn exerts a force
\ T2 = ma = 2 ´ = 3.4N
6 Mg on the spring of spring balance s2 Mkgf

F S1 which therefore shows a reading


T2 of M kgf.
C B A s1 Mkgf
T2 T1 T1
The spring S1 is massless. Therefore
8. (c) mg – T = ma it exerts a force of Mg on the spring
M
of spring balance S2 which shows
T 360 Mg
\ a = g - = 10 - = 4m / s 2 the reading of M kgf. Thrust (F)
m 60
T
15. (b) As shown in the figure F – mg = ma
a
\ F = m ( g + a)
®

T
= 3.5 × 104 ( 10+10) a
= 7 × 105 N
mg
mg
mg
Laws of Motion P-S-25

æ m - m2 ö x 1
16. (c) Acceleration a = ç 1 Dividing eqns. (1) and (2) we get =
g 3 3
è m1 + m2 ÷ø or x = 1 cm
21. (c) Force experienced by the particle, F = mw2R
(5 - 4.8) ´ 9.8
= m / s2 = 0.2 m/s2 F R
(5 + 4.8) \ 1 = 1
F2 R2
17. (c) fs 22. (d) According to work-energy theorem, W = DK = 0
N (Since initial and final speeds are zero)
\ Workdone by friction + Work done by gravity = 0
mg l
i.e., - ( µ mg cos f ) + mg l sin f = 0
2
30° µ
or cos f = sin f or µ = 2 tan f
mg sinq = f s ( for body to be at rest) 2
23. (c) Mass (m) = 0.3 kg Þ F = m.a = 15x
Þ m ´ 10 ´ sin 30° = 10
15 150
Þ m ´ 5 = 10 Þ m = 2.0 kg a=– x= x = 50 x a = 50 × 0.2 = 10 m / s 2
0.3 3
18. (b) g sin q - mg cos q 24. (c) From diagram,
ma
g cos
d q N
n
g si d a
45° 45° (pseudo ma a
force) mg cos a
smooth rough + ma sina mg mg sin a
When surface is When surface is For block to remain stationary,
smooth rough
mg sin a = ma cos a Þ a = g tan a
1 1 2
d = ( g sin q)t12 , d = ( g sin q - mg cos q) t2 25. (a) v 2 - u 2 = 2as or 02 - u 2 = 2( -m k g ) s
2 2
2d 1
t1 =
2d
, t2 = -1002 = 2 ´ - ´ 10 ´ s Þ s = 1000 m
g sin q g sin q - mg cos q 2
26. (d) Work done by tension + Work done by force (applied)
According to question, t 2 = nt1 + Work done by gravitational force = change in kinetic
2d 2d energy
n = Work done by tension is zero O
g sin q g sin q - mg cos q
Þ 0 + F ´ AB - Mg ´ AC = 0
1 æ 1 ö é 1 ù 45°
n= ççQ cos 45° = sin 45° = ÷÷ 1- l
1 - mk æ AC ö ê ú
è 2ø 2
Þ F = Mgç ÷ = Mg ê ú B
1 1 1 è AB ø ê 1 ú A F
n2 = or 1 - m k = 2 or m k = 1 - 2 êë 2 úû
1 - mk n n C F
19. (d) The velocity of parachutist when parachute opens is l
[Q AB = l sin 45° =
u = 2gh = 2 ´ 9.8 ´ 50 = 980 2
The velocity at ground, v = 3m/s æ 1 ö
50 m and AC = OC - OA = l - l cos 45° = l ç1 - ÷
è 2ø
v2 - u 2 32 - 980 u
\ S= = » 243 m a = - 2 m / s2 where l = length of the string.]
2 ´ ( -2) –4
Initially he has fallen 50 m. Þ F = Mg ( 2 - 1)
\ Total height from where 27. (d) Whand + Wgravity = DK
3m / s = v
he bailed out = 243 + 50 = 293 m Þ F (0.2) – (0.2)(10)(2.2) = 0 Þ F = 22 N
20. (c) Let K be the initial kinetic energy and F be the resistive
force. Then according to work-energy theorem, 28. (c) m(v - u ) 0.15(0 - 20)
F= = = 30 N
W = DK t 0.1
2 29. (b) For block A to move in SHM.
1 2 1 æ vö N
i.e., 3F = mv - m ç ÷ …(1)
2 2 è 2ø A
For B to C :
2
1 æ vö 1 x
Fx = m ç ÷ - m(0)2 … (2) mg mean
2 è 2ø 2 position
EBD_7036
P-S-26 Topic-wise Solved Papers - PHYSICS
mg – N = mw2x Q Coefficient of friction m = 0.5
where x is the distance from mean position
For block to leave contact N = 0 x2
\ 0.5 =
g 2
Þ mg = mw 2 x Þ x =
w2 Þ x=+1
30. (d) Drawing free body-diagrams for m & M,
x3 1
M Now, y = = m
m 6 6
K
F
f1 f2
N N
a
34. (a) F N
T T M A B
m F

mg Mg f1
20N 100N
we get T = ma and F – T = Ma
where T is force due to spring Assuming both the blocks are stationary
Þ F – ma = Ma or,, F = Ma + ma N= F
F f1 = 20 N
\ a= . f2 = 100 + 20 = 120N
M +m
Now, force acting on the block of mass m is f
æ F ö = mF
ma = m ç .
è M + m ÷ø m + M
31. (a) mg sin q = ma \ a = g sin q
where a is along the inclined plane
\ vertical component of acceleration is g sin2 q
\ relative vertical acceleration of A with respect to B is
120N
2 2 g
g (sin 60 - sin 30] = = 4.9 m/s 2 in vertical Considering the two blocks as one system and due to
2 equilibrium f = 120N
direction 35. (a) Loss in P.E. = Work done against friction from p ® Q
32. (b) From figure,
+ work done against friction from Q ® R
Acceleration a = Ra …(i)
and mg – T = ma …(ii) a mgh = m(mgcosq) PQ + mmg (QR)
From equation (i) and (ii) R h = m cos q × PQ + m(QR)

æ aö 3 2
T × R = mR2a = mR2 çè ÷ø T 2= m× × + mx
R 2 sin 30°
or T = ma T
Þ mg – ma = ma 2 = 2 3 m + mx --- (i)
g m a 2
Þ a= mg [sin 30° = ]
2 PQ
33. (a) At limiting equilibrium, m = tanq
dy x 2 Also work done P ® Q = work done Q ® R
tanq = m = = (from question)
dx 2 \ 2 3 m = mx
\ x » 3.5m
m From (i) 2 = 2 3 m + 2 3 m = 4 3 m
y
q
2 1
m= = = 0.29
4 3 2 ´ 1.732
4
Work, Energy and Power P-S-27

Work, Energy and Power


Section-A : JEE Advanced/ IIT-JEE

C 1. (b) 2. (c) 3. (b) 4. (b) 5. (c) 6. (d)


7. (b) 8. (b) 9. (b) 10. (c) 11. (c) 12. (b)
13. (c) 14. (d) 15. (b)
D 1. (c) 2. (d) 3. (c, d) 4. (c) 5. (d) 6. (a, b)
E 1. Less than 2. 3 times 3. 25%
4. No 5. vC = 2 g ( y - mx) ; vF = 2 g ( y - mx )
6. 10 kg, 0.098 J 7. 4.24 m 8. 3.29 m/s 9. 4
10. 0.84; 15.02 kg
11. (a) NA = mg (3cosq – 2)

(b) For q £ cos -1 æç ö÷ ; N B = 0, N A = mg (3cos q - 2)


2
è3ø

For q > cos-1 çæ ÷ö ; N A = 0, N B = mg (2 - 3cos q)


2
è3ø
F 1. (A) p, q, r, t; (B) q, s; (C) p, q, r, s; (D) p, r, t
G 1. (b) 2. (b) 3. (c) 4. (a) 5. (b)
H 1. (c)
I 1. 8J 2. 4 m/s 3. 4 4. 5 5. 5

Section-B : JEE Main/ AIEEE

1. (c) 2. (d) 3. (b) 4. (b) 5. (c) 6. (b) 7. (b) 8. (b)


9. (a) 10. (b) 11. (b) 12. (b) 13. (b) 14. (a) 15. (b) 16. (d)
17. (c) 18. (c) 19. (a) 20. (c) 21. (b)

Section-A JEE Advanced/ IIT-JEE


C. MCQs with ONE Correct Answer 1 2 m 2 22
Þ mv = k r t ... (i)
2 2
Output work m 2 22 d
1. (b) Mechanical efficiency = Þ K.E. = k r t Þ (K.E.) = mk 2 r 2 t
Input energy 2 dt
The output work will increase because the friction Þ Power = mk2r2t
becomes less. Thus the mechanical efficiency increases. 4. (b) KEY CONCEPT
2 The force constant of a spring is inversely proportional
p to the length of the spring.
2. (c) K.E. =
2m Let the original length of spring be L and spring
constant is K (given)
p2 p2 Therefore,
E1 = E2 \ 1 = 2
m1 m2
2L 3
K×L= ×K' Þ K'= K
p12 ml p m1 1 1 3 2
\ = Þ l = = =
p22 m2 p2 m2 4 2 æ dm ö d d
5. (c) F = v ç ÷ = v (r ´ Volume) = vr (Volume)
3. (b) The centripetal acceleration
è dt ø dt dt
= vr × (Av) = Arv2
v2 Power = Force × Velocity = Arv2 × v = Arv3
ac = k2 r t2 Þ = k 2 rt 2
r Þ P µ v3
EBD_7036
P-S-28 Topic-wise Solved Papers - PHYSICS
A
6. (d) dU(x) = – Fdx 2v ´ t + v ´ t = 2 pr v 2v
x kx 2
ax 4 where t is the time taken
\ Ux = - ò Fdx = - for first collision to occur.
0 2 4
2 pr
2k \ t=
U = 0 at x = 0 and at x = ; Þ we have potential 3v
a
energy zero twice (out of which one is at origin). \ Distance travelled by particle with velocity v is
Also, when we put x = 0 in the given function, 2 pr 2pr
equal to v ´ = .
dU 3v 3
we get F = 0. But F = -
dx Therefore the collision occurs at B.
dU A A
Þ At x = 0; = 0 i.e. the slope of the graph
dx
should be zero. These characteristics are represented
by (d). 2v v
120°
7. (b) The above situation can also be looked upon as the 120°
decrease in the gravitational potential energy of spring B C
B
mass system is equal to the gain in spring elastic
potential energy.
v 2v
1 2 Mg As the collision is elastic and the particles have equal
Mgx = kr 2 , x =
2 k masses, the velocities will interchange as shown in the
8. (b) Note : In a conservative field work done does not figure. According to the same reasoning as above, the
depend on the path. The gravitational field is a 2nd collision will take place at C and the velocities will
conservative field. again interchange.
\ W1 = W2 = W3 With the same reasoning the 3rd collision will occur at
9. (b) We know that DU = – W for conservative forces the point A. Thus there will be two elastic collisions
before the particles again reach at A.
x x
12. (b) The forces acting on the bead as seen by the observer
DU = - ò Fdx or DU = - ò k xdx
0 0 in the accelerated frame are : (a) N ; (b) mg ; (c) ma
(pseudo force).
kx 2 Let q is the angle which the tangent at P makes with the
Þ U(x) – U(0) = - X- axis. As the bead is in equilibrium with respect to the
2
wire, therefore
kx 2 N sin q = ma and N cos q = mg
Given U(0) = 0 U(x) = - a
2
\ tan q = g … (i) Y
10. (c) When the block B is displaced towards wall 1, only
2
But y = k x . Therefore, Ncosq
spring S1 is compressed and S2 is in its natural state.
N
This happens because the other end of S2 is not attached dy q
to the wall but is free. Therefore the energy stored in = 2kx = tan q … (ii) ma Nsinq
dx P
1 From (i) & (ii)
the system = k1 x2 . When the block is released, it will mg
2 a a
come back to the equilibrium position, gain momentum, 2kx = Þ x= q

overshoot to equilibrium position and move towards g 2kg X

wall 2. As this happens, the spring S1 comes to its natural 13. (c) Area under F – t graph gives the impulse or the change
length and S2 gets compressed. As there are no frictional in the linear momentum of the body. As the initial
velocity (and therefore the initial linear momentum) of
forces involved, the P.E. stored in the spring S1 gets
the body is zero, the area under F – t graph gives the
stored as the P.E. of spring S2 when the block B reaches final linear momentum of the body.
its extreme position after compressing S2 by y. Area of D AOB F(t)
1 1 1
\ k1 x2 = k2 y2 = ´ 3´ 4 = 6 N - s 4N
A
2 2 2
1 1 y 1 OA CD
× kx2 = 4 ky2, x2 = 4y2 \ = Also = O B 4.5 s
2 2 x 2 OB CB 3s C t
11. (c) Let the radius of the circle be r. Then the two distance 4 CD
Þ = f D
travelled by the two particles before first collision is
3 1.5
2pr. Therefore Þ CD = 2
Work, Energy and Power P-S-29

é1 ù
\ Area of DBCD = = - ê ´1.5 ´ 2 ú = -1.5 N-s
ë 2 û
\ The final linear momentum = 6 – 1.5 = 4.5 N-s

p 2 (4.5)2 T
L/6 = distance of the centre
\ Kinetic energy of the block = = = 5.06 J M_g
2m 2 ´ 2 3
from top of the table.
14. (d) Let us consider a point on the circle W = Change in potential energy
The equation of circle is x2 + y2 = a2
M L MgL
The force is = mgh = ´g´ =
3 6 18
r é xiˆ yjˆ ù 3. (c, d)When the force is perpendicular to the velocity and
F= Kê + ú constant in magnitude then the force acts as a centripetal
êë (x 2 + y2 )3/ 2 (x 2 + y2 )3/ 2 úû force, and the body moves in a circular path. The force
y
r é xiˆ is constant in magnitude, this show the speed is not
yjˆ ù (0, a) ds
F = K ê 2 3/ 2 + 2 3 / 2 ú changing and hence kinetic energy will remain constant.
P(x, y)
êë (a ) (a ) úû v
r K F
yjˆ
F = 3 éë xiˆ + yjˆùû x F
a xiˆ (a,0) v
The force acts radially outwards as shown in the figure
and the displacement is tangential to the circular path. F
Therefore the angle between the force and displacement
F
is 90° and W = 0
v
option (d) is correct. v
Note : The velocity changes continuously due to
1 2 1 2 1
mv = m [u + at ] = m [ 0 + gt ]
2
15. (b) K.E. = change in the direction. The acceleration also changes
2 2 2 continuously due to change in direction.
1 4. (c) The expression of work done by the variable force F on
\ K .E = mgt 2 \ K .E µ t 2 ...(1) the particle is given by
2
ur uur
First the kinetic energy will increase as per eq (1). As W = ò F .d l
the balls touches the ground it starts deforming and
In going from (0, 0) to (a, 0), the coordinate of x varies
loses its K.E. (K.E. converting into elastic potential
from 0 to 'a', while that of y remains zero. Hence, the
energy). When the deformation is maximum, K.E. = 0.
work done along this path is :
The ball then again regain its shape when its elastic
[Q $j . $i = 0 ]
a
potential energy changes into K.E. As the ball moves
up it loses K.E. and gain gravitational potential energy.
W1 = ò 0 ( - Kx $j ). dx $i = 0
These characteristics are according to graph (b). In going from (a, 0) to (a, a) the coordinate of x remains
constant (= a) while that of y changes from 0 to 'a'.
D. MCQs with ONE or MORE THAN ONE Correct Hence, the work done along this path is
a a
ò 0 [(- K ( yi$ + a $j). dy $j ] = kaò 0 dy = - Ka
2
1 2 W2 =
E mv
1. (c) P= = constt \ 2 = constt
t t Y
(a, a)
v2 ds
Þ = constt (k ) \ v = kt1/2 and = kt1/ 2
t dt dy

or, ds = kt1/ 2 dt
By integrating, we get O dx (a, 0) X

2kt 3/ 2 Hence, W = W1 + W2 = – ka2


Þ s= + C Þ s µ t3/2 5. (d) Applying the principle of conservation of energy
3
(K.E.)B + (P.E.)B = (K.E.)A + (P.E.)A,
2. (d) The hanging part of the chain which is to be pulled up
we get
can be considered as a point mass situated at the centre
of the hanging part. The equivalent diagram is drawn. 1 2 1
mv + mgL = mu 2
Note : The work done in bringing the mass up will be 2 2
equal to the change in potential energy of the mass.
EBD_7036
P-S-30 Topic-wise Solved Papers - PHYSICS
v 1
2
u - 2 gL ... (i) \ % loss in velocity = ´ 100 = 25%
Hence, v = 4
4. No. An external force, the gravitational pull of earth, is acting
r r 2 2 on the ball which is responsible for the change in momentum.
Change in velocity = | v - u | = v + u
5. (a) K.E. at C = Loss in P.E. – Work done by friction.
2
= 2(u - g l) [From (i)] u 1 2
mvc = mg y - mmg cos q ´ AC
6. (a, b) At point A, potential energy of the ball = mghA 2
At point B, potential energy of the ball = 0 A
At point C, potential energy of the ball = mghC 1 2 BC
\ vc = g y - mg ´ AC
Total energy at point A, EA = KA + mghA 2 AC
Total energy at point B, EB = KB = gy - mgx
Total energy at point C, EC = KC + mghC q(
According to the law of conservation of energy. \ vc = 2 g ( y - mx ) B C
EA = EB = EC ... (i)
K.E. at F = loss in P.E. – Work done by friction
EA = EB Þ EC > KC ...(ii)
EA = EC 1 2
KA + mghA = KB + mghC mvF = mgy - mmg cos a DG - mmg cos b GF
2
K - KA
or, h A - h C = C ...(iii) 1 2 GM FN
mg vF = gy - mg ´ DG - mg ´ GF
2 DG GF
Þ hA > hC ;KC > KA ...(iv)
Option (b) is correct 1 2
\ vF = gy - mg (GM + FN )
From (i),(ii) and (iv), we get hA> hC; KB > KC.Option (a) is 2
correct.
\ vF = 2 g ( y - mx )
E. Subjective Problems
D
p2 1
1. K.E. = For equal value of p, K.E. µ
2m mass
M a(
G
2. KEY CONCEPT
For a spring, (spring constant) × (length) = Constant
If length is made one third, the spring constant becomes b(
three times. E N F
3. Let v be the velocity of bullet before striking A. Applying
Note : The result does not depend on the angles a and b. It
conservation of linear momentum for the system of bullet
and plate A, we get only depends on the values of x and y.
0.02v = 0.02 v1 + 1 × v2 6. Since the two blocks A and B are moving with constant
velocity, therefore, the net force acting on A is zero
A B and the net force acting on v
N
B will be zero. Since the
T
spring is loaded, it will be B
v1 v2 in a deformed state. Let the C f T
v mBg
extension of the spring be
v2
x. T v
A
The forces are drawn.
mgA
Again applying conservation of linear momentum for
collision at B. Note : There will be no friction force between block A and C
0.02 v1 = (2.98 + 0.02) v2 = 3v2 Q f = µN. Here there is no normal reaction on A
0.02 v1 (because A is not pushing C)
Þ v2 = ... (ii) Applying Fnet = ma on A, we get
3
From (i) and (ii) mAg – T = mA × 0
0.02 v1 4 v 4 \ T = mAg ... (i)
0.02 v = 0.02 v1 + , v = v1 Þ = Applying Fnet = ma on B, we get
3 3 v1 3
T – f = mB × 0
v1 3 v 3 1 v - v1
= Þ 1 - 1 = 1 - = = 0.25 Þ = 0.25 \ T = f = µN
v 4 v 4 4 v
= µmBg ... (ii)
Work, Energy and Power P-S-31

From (i) and (ii) From the figure, by velocity constraint


mA 2 v = V cos q
O 2
µmBg = mAg Þ mB = = = 10 kg From DOAC, A
m 0.2 q
Here the force acting on the spring is the tension (equal to 2
restoring force) cos q =
5 1
T 5
\ T = kx \ x= 2V
k \ v=
5 C
19.6
\ x= [Q T = 2 × 9.8 = 19.6 N from (i)] (OC + CA) – OA = height attained by m
k
The P.E. stored in spring is given by 1 + 22 + 12 - 2 = height attained by m = 5 -1
1 1 19.6 19.6
U = kx 2 = ´ k ´ ´ \ Increase in P.E. of m = mgh' = 0.5 × 9.8 ( 5 - 1 )
2 2 k k NOTE THIS STEP
19.6 ´ 19.6 By the principle of energy conservation
= = 0.098 J
2 ´ 1960 1 2 1 2
7. K.E. of block = work against friction + P.E. of spring Mgh = MV + mv + mgh '
2 2
1 2 1 1 2 1 2
mv = µk mg (2.14 + x) + kx2 = MV + m(V cos q) + mgh '
2 2 2 2
1 1 1 1 4V 2
× 0.5 × 32 = 0.2 × 0.5 × 9.8 (2.14 + x) + 2 × x2 \ 19.6 = ´ 2 ´ V 2 + 0.5 ´ + 0.5 × 9.8 ( 5 - 1 )
2 2 2 2 5
2.14 + x + x2 = 2.25 On solving, we get V = 3.29 m/s
\ x2 + x – 0.11 = 0 9. The pendulum bob is left from position A. When it is at
11 1 position C, the angular amplitude is 60°.
On solving, we get x = – or x = = 0.1 (valid answer)
10 10 In DOCM
Here the body stops momentarily.
OM l
Restoring force at Y = kx = 2 × 0.1 = 0.2 N cos 60° = Þ OM =
Frictional force at Y = µs mg = 0.22 × 0.5 × 9.8 = 1.078 N l 2
Since frictional force > restoring force, the body will stop here. The velocity of bob at B,
\ The total distance travelled vB before first collision is
= AB + BD + DY = 2 + 2.14 + 0.1 = 4.24 m.
1
mg l = mvB2 Þ vB = 2gl
2
Let after n collisions, the angular amplitude is 60° when the
bob again moves towards the wall from C, the velocity v'B
A B D C before collision is
2m l 1
2.14m mg = mv '2B Þ v'B = gl
x 2 2
This means that the velocity of the bob should reduce from
2gl to gl due to collisions with walls.
Y
8. When mass m is released, since M > m, the mass M will The final velocity after n collisions is gl
move on a dotted path with O as the centre. There will be n
\ e ( 2gl ) = gl
decrease in the potential energy of M which will be converted where e is coefficient of restitution.
into kinetic energy of M, and increase in potential energy n n
of m. æ 2 ö æ 2 ö 1
Decrease in P.E. of M is Mgh M çè ÷ø ´ 2 g l = g l Þ ç ÷ =
O 1m 5 è 5ø 2
A
= 2 × 9.8 × 1 = 19.6 J 1m q Taking log on both sides we get
1 æ 2 ö 1
K.E. of M = MV 2 1m
n log ç = log Þ n = 3.1
2 5 m è 5ø ÷
B
2
(Let V be the velocity attained v Therefore, number of collisions will be 4.
q C
by M just before striking the 10. From A to B.
v
wall) u = 10 m/s (given)
1 2 é mg sin q + f ù é mg sin q + mmg cos q ù
K.E. of m = mv a= -ê ú = -ê úû
2 ë m û ë m
EBD_7036
P-S-32 Topic-wise Solved Papers - PHYSICS

B
mv 2 C
NA
M C mg sin q – NA = ... (i) A d/2
æ dö D
çè R + ÷ø mgcosq
q
V
uA B 2 q mgsinq
R
mg
N According to energy
m conservation
mg sinq q mg cosq
A
f mg 1 2 æ dö
q mv = mg ç R + ÷ (1 - cos q) ... (ii)
2 è 2ø
= – [g sin q + µg cos q ] = – g [sin q + µ cos q ]
= – 10 [0.05 + 0.25 × 0.99] = – 2.99 m/s2 From (i) and (ii)
v=? NA = mg (3 cos q – 2) ... (iii)
s=6m The above equation shows that as q increases NA decreases.
v2 – u2 = 2as Þ v2 = 100 + 2 (– 2.99) × 6 Þ v = 8 m/s
At a particular value of q , NA will become zero and the ball
Þ The velocity of mass m just before collision is 8 m/s.
The velocity of mass M just before collision is 0 m/s (given). will lose contact with sphere A. This condition can be found
AFTER COLLISION by putting NA = 0 in eq. (iii)
Let v1 be the velocity of mass m after collision and v2 be the æ 2ö
velocity of mass M after collision. Body of mass M moving 0 = mg (3 cos q – 2) \ q = cos–1 ç ÷
è 3ø
from B to C and coming to rest.
u = v2 The graph between NA and cos q
v=0 From equation (iii) when q = 0, NA = mg.
a = – 2.99 m/s2 NA NB
(same as of previous case because all other things are same 2mg
except mass. a is independent of mass) s = 0.5 5mg

v2 – u2 = 2as Þ (0)2 – v22 = 2 (– 2.99) × 0.5 mg


Þ v2 = 1.73 m/s
2mg
Body of mass m moving from B to A after collision
h 1 -1 +1
sin q = B cosq
6
S=
6 h æ 2ö
h = 6 sin q = 6 × 0.05 q When q = cos–1 ç ÷ ; NA = 0
è 3ø
u = v1 A
v = + 1m/s The graph is a straight line as shown.
(K.E. + P.E.)initial = (K.E. + P.E.)final + Wfriction æ 2ö mv 2
when θ > cos-1 ç ÷ ; NB – (mg cos q ) =
1 2 1 è 3ø d
mv1 + mgh = mv 2 + 0 + m mgs R+
2 2 2
mv 2
1 2 1 Þ NB + mg cos q = ... (iv)
v1 + 10 ´ (6 ´ 0.05) = (1) 2 + 0.25 ´ 10 ´ 6 æ dö
2 2 çè R + ÷ø
2
v1 = – 5 m/s
Using energy conservation
\ Coefficient of restitution
1 2 éæ dö æ dö ù
Re lative velocity of separation mv = mg ê ç R + ÷ø - çè R + ÷ø cos q ú
e= 2 ëè 2 2 û
Re lative velocity of approach
mv 2
= 2mg [1 – cos q ] ... (v)
-5 - 1.73 æ dö
= = 0.84 çè R + ÷ø
8-0 2
From (iv) and (v), we get
On applying conservation of linear momentum before and
after collision, we get NB + mg cos q = 2mg – 2mg cos q
2 × 8 + M × 0 = 2 × (– 5) + M (1.73) NB = mg (2 – 3 cos q )
26 2
\ M= = 15.02 kg When cos q = , NB = 0
1.73 3
11. The ball is moving in a circular motion. The necessary When cos q = – 1, NB = 5 mg
centripetal force is provided by (mg cos q – N). Therefore, Therefore the graph is as shown.
Work, Energy and Power P-S-33

F. Match the Following Also in right angled triangle BEC, ÐEBC= 60°.
Therefore v makes an angle of 30° with the second
1. A ® p, q, r, t; B ® q, s; C ® p, q, r, s; D ® p, r, t inclined plane BC. The component of v along BC is v
For A cos 30°.
2 It is given that the collision at B is perfectly inelastic
dU –d éU æ æ x ö2 ù -2U 0 therefore the impact forces act normal to the plane such
Fx = – dx = dx ê 0 ç1 - ç ÷ ú = ( x - a) x ( x + a) that the vertical component of velocity becomes zero.
êë 2 ç èaø ú a3
è û The component of velocity along the incline BC
U U remains unchanged and is equal to v cos 30°
U0 3 180
= 60 ´ = = 45 m / s
2 2 4
BE 1 BE
2. (b) In D BCE, tan30° = Þ = Þ BE = 3m
CE 3 3 3
Applying mechanical energy conservation .
–a +a x x Mechanical energy at B = Mechanical energy at C
U U 1
( ) 1
2
M 45 + M ´ 10 ´ 3 = Mvc2
2 2
45 + 60 = vc 2 \ vc = 105 m / s
3. (c) The velocity of the block along BC just before collision
is v cos 30°. The impact forces act perpendicular to the
+a x –a +a x surface so the component of velocity along the incline
remains unchanged.

– dU æ xö v sin 30°
For B Fx = = -U 0 ç ÷
dx èaø
v
e- x / a
2 2
– dU B 30°
For C Fx = = U0 x( x - a)( x + a) v co
s 30
dx a3 °
dU U 30° C
For D Fx = - = - 03 [( x - a)( x + a)] Just after collision
dx 2a Also since the collision is elastic, the vertical
G. Comprehension Based Questions component of velocity (v sin 30°) before collision
changes in direction, the magnitude remaining the same
1. (b) As the inclined plane is frictionless,
as shown in the figure. So the rectangular components
The K. E . at B = P.E. at A
of velocity after collision are as shown in the figure.
1 2 This means that the final velocity of the block should
mv = mgh Þ v = 2 gh
2 be horizontal making an angle 30º with BC. Therefore
A the vertical component of the final velocity of the block
is zero.
30°
h 30° 40 sin 30°
2
mv
4. (a) N – mg cos 60° = 40m
60° B r
D N
v cos 30° mv 2
30° 30° \ N = mg cos 60° + ...(1) mg sin 60°
v sin 30° v = 2gh r 60° v
mg cos 60° mg
30° Loss in P.E. = mg × 40 sin 30° = 200 J
E
C
3m 3 3m Work done in over coming friction = 150 J
h \ K.E. possessed by the particle = 50 J
In D ADB, tan 60° =
3 1
\h=3m \ mv 2 = 50J
2
\ v = 6g = 60 m / s
\ mv2 = 100 J ...(2)
This is the velocity of the block just before collision.
This velocity makes an angle of 30° with the vertical.
EBD_7036
P-S-34 Topic-wise Solved Papers - PHYSICS

1 100 0 + 2m ´ 9
From (1) and (2), N = 1 × 10 × + = 5 + 2.5 = 7.5 N = = 6m/s
2 40 m + 2m
(a) is the correct option. The collision between B and C is completely inelastic.
5. (b) From (2), mv2 = 100 \ v = 10 ms–1 \ mB vB = (mB + mc) v
(b) is the correct option. 6 ´ 2m
\ v= = 4m/s.
2m + m
H. Assertion & Reason Type Questions
3. 4
1. (c) Statement 1 : In the first case the mechanical energy is Let v be the speed of the block just after impulse. At B, the
completely converted into heat becuase of fiction. While block comes to rest. Therefore
is second case, a part of mechanical energy is
A
converted into heat due to fiction but another part of v
mechanical energy is retained in the form of potential
energy of the block.
Therefore statement 1 is correct. B
Statement 2 : This is a wrong statement because the
coefficient of friction between the block and the surface Loss in K.E. of the block = Gain in P.E. of the spring
does not depend on the angle of inclination.
+ Work done against friction
I. Integer Value Correct Type 1 2 1 2
mv = kx + mmg.x
1. 8 Given m = 0.36 kg, M = 0.72 kg. 2 2
The figure shows the forces on m and
M. When the system is released, let k 2
\v= x + mgx .
the acceleration be a. Then m
T – mg = ma
Mg – T = Ma T 2
v= ´ 0.06 ´ 0.06 + 0.1 ´ 10 ´ 0.06
( M - m) g 0.18
\ a= = g /3 a m T
M +m 4
and T = 4 mg/3 mg a \v= \N=4
M 10
For block m :
u = 0, a = g/3, t = 1, s = ? 4. 5 Here D K.E. = W = P × t
1 Mg
1 g 1
s = ut + at2 = 0 + ´ ´ 12 = g / 6 \ mv 2 = P ´ t
2 2 3 2
\ Work done by the string on m is
ur r mg g 4 ´ 0.36 ´ 10 ´ 10 2Pt 2 ´ 0.5 ´ 5
T .s = Ts = 4 ´ = = 8J \v = = = 5ms -1
3 6 3´ 6 m 0.2
2. 4 The velocity of B just after collision with A is 5. 5 Work done = Increase in potential energy
(mB - m A )uB 2m A u A + gain in kinetic energy
vB = +
mB + mA m A + mB F × d = mgh + gain in K.E.
18 × 5 = 1 × 10 × 4 + gain in K.E.
m 2m m \ Gain in K.E. = 50 J = 10n
\n=5
A B C

Section-B JEE Main/ AIEEE


1. (c) Kinetic energy of a system of particle is zero only when \ A does not imply B but B implies A.
the speed of each particles is zero. And if speed of each 2. (d) The elastic potential energy
particle is zero, the linear momentum of the system of 1 1
particle has to be zero. = ´ Force ´ extension = ´ 200 ´ 0.001 = 0.1 J
2 2
Also the linear momentum of the system may be zero
3. 3
(b) k = 5 ´ 10 N/m
even when the particles are moving. This is because
linear momentum is a vector quantity. In this case the
kinetic energy of the system of particles will not be zero. W=
1
2
(
k x22 - x12 ) =
1
2
´ 5 ´ 103 é(0.1)2 - (0.05)2 ù
ë û
Work, Energy and Power P-S-35

5000 9. (a) Work done by such force is always zero since force is
= ´ 0.15 ´ 0.05 = 18.75 Nm acting in a direction perpendicular to velocity.
2 \ from work-energy theorem = DK = 0
4. (b) We know that F × v = Power
K remains constant.
\ F ´ v = c where c = constant
æ mdv ö 1 1 k
\m
dv
´v =c 10. (b) Mv 2 = k L2 Þ v = .L
çè\ F = ma = ÷ 2 2 M
dt dt ø
v t
1 2 k
\ mò vdv = c ò dt \ mv = ct Momentum = M × v = M × .L = kM . L
2 M
0 0
11. (b)
2c 1
\ v= ´t 2
m
100
30 20
dx 2c 1 2 dx
\ = ´t where v =
dt m dt Loss in potential energy = gain in kinetic energy
x t
2c 1 1 1
\ ò dx = ´ ò t 2 dt m ´ g ´ 80 = mv 2 , 10 ´ 80 = v 2
m 2 2
0 0
3 v2 = 1600 or v = 40 m/s
2c 2t 2 3 12. (b) u = 0; v = u + aT; v = aT
x= ´ Þ xµt 2
m 3 Instantaneous power = F × v = m. a. at = m. a2 . t
5. (c) Given : retardation µ displacement
i.e., a = -kx v2
\ Instantaneous power = m t
v2 x T2
vdv
But a = v
dv \
dx
= - kx Þ ò v dv = -k ò xdx 13. (b) K.E =
1 2 1
mv = ´ 0.1 ´ 25 = 1.25 J
dx v1 0
2 2

(v 2
2 )
- v12 = - k
x2
2
æ1 ö
W = - mgh = - ç mv 2 ÷ = -1.25 J
è2 ø
æ -x2 ö
Þ
1
2
( 1
m v22 - v12 = mk ç
2
)
ç 2 ÷
è
÷
ø
é 1 2
êëQ mgh = 2 mv by energy conservation úû
ù

\ Loss in kinetic energy, \ DK µ x 2 14. (a) Velocity is maximum when K.E. is maximum
For minimum. P.E.,
4
6. (b) Mass of over hanging chain m¢ = ´ (0.6)kg dV
2 = 0 Þ x 3 - x = 0 Þ x = ±1
Let at the surface PE = 0 dx
C.M.of hanging part = 0.3 m below the table 1 1 1
Þ Min. P.E. = - =- J
4 4 2 4
U i = - m ¢gx = - ´ 0.6 ´ 10 ´ 0.30
2 K.E.(max.) + P.E.(min.) = 2 (Given)
DU = m ' gx = 3.6J = Workdone in putting the entire 1 9 1 2
\ K.E.(max.) = 2 + = = mvmax.
chain on the table. 4 4 2
7. (b) Workdone in displacing the particle, 1 2 9 3
rr Þ ´ 1 ´ vmax . = Þ vmax. =
W = F . x = (5iˆ + 3 ˆj + 2kˆ).(2iˆ - ˆj) 2 4 2
= 10 – 3 = 7 joules 15. (b) Let the block compress the spring by x before stopping.
8. (b) Let acceleration of body be a kinetic energy of the block = (P.E of compressed spring)
+ work done against friction.
v
\ v1 = 0 + at1 Þ a = 1 1 1
t1 ´ 2 ´ (4)2 = ´ 10,000 ´ x 2 + 15 ´ x
2 2
vt 10,000 x2 + 30x – 32 = 0
\ v = at Þ v = 1
t1
rr r r Þ 5000 x 2 + 15 x - 16 = 0
Pinst = F .v = (ma ).v
-15 ± (15)2 - 4 ´ (5000)(-16)
æ mv ö æ v t ö æv ö
2 \ x=
= ç 1÷ ç 1 ÷ = m 1 t 2 ´ 5000
è t1 ø è t1 ø çè t ÷ø = 0.055m = 5.5cm.
1
EBD_7036
P-S-36 Topic-wise Solved Papers - PHYSICS
16. (d) The average speed of the athelete 19. (a) When force is same
100 1 2 1 2
v= = 10 m/s \ K.E. = mv W= kx
10 2 2
1 1 F2
If mass is 40 kg then, K.E. = ´ 40 ´ (10) 2 = 2000 J W = k [Q F = kx]
2 2 k2
1
If mass is 100 kg then, K.E. = ´ 100 ´ (10) 2 = 5000 J F2
2 \ W=
17. (c) Initial kinetic energy of the system 2x
As W1 > W2
1 1 1 \ k1 < k 2
K.Ei = mu 2 + M (0)2 = ´ 0.5 ´ 2 ´ 2 + 0 = 1J
2 2 2 When extension is same
For collision, applying conservation of linear momentum W µ k (Q x is same)
m × u = (m + M) × v \ W1 < W2
2 Statement 1 is false and statement 2 is true.
\ 0.5 ´ 2 = (0.5 + 1) ´ v Þ v = m/s 20. (c) Work done in stretching the rubber-band by a distance
3
Final kinetic energy of the system is dx is
dW = F dx = (ax + bx2)dx
1 1 2 2 1 Integrating both sides,
K.E f = (m + M )v2 = (0.5 + 1) ´ ´ = J
2 2 3 3 3 L L
aL2 bL3
æ 1ö W = ò axdx + ò bx 2 dx = +
\ Energy loss during collision = ç1 - ÷ J = 0.67J 2 3
0 0
è 3ø
dU ( x ) W mgh ´ 1000 10 ´ 9.8 ´ 1 ´ 1000
18. (c) At equilibrium : =0 21. (b) n = = =
dx input input input
1
-12a -6b æ 2a ö 6 98000
Þ 11
= 5 Þ x=ç ÷ Input = = 49 × 104J
x x è b ø 0.2
a b b2 49 ´ 10 4
\ U at equilibrium = - = - and U ( x=¥ ) = 0
æ 2a ö
2 æ 2a ö 4a Fat used = = 12.89 × 10–3kg.
ç ÷ ç ÷ 3.8 ´ 107
è b ø è b ø
æ b2 ö b2
\ D = 0 -ç- ÷ =
è 4a ø 4a
5
Momentum and Impulse P-S-37

Momentum and Impulse

Section-A : JEE Advanced/ IIT-JEE


3
A 1. mv 2 2. 0.005 Ns
2
C 1. (d) 2. (c) 3. (d) 4. (a)
D 1. (c, d) 2. (a) 3. (b, d) 4. (a, d) 5. (a, c) 6. (a, d)
2 2
æ Mv ö mM (v + V )
E 1. (i) q = tan -1 ç , m 2V 2 + M 2 v 2 ; (ii)
è mV ÷ø ( m + M )( mV 2 + Mv 2 )

v0 2mv02
2. V, opposite direction to the retraced velocity of B 3. 3 , 4. 6.53 sec
3 x02
5
5. 37°, 122.6 m, 46 m 6. (a) 6m gd (b) 6d 3m
2
v2
7. 2.5 m/s, 0.318 m 8. 12 s, 15.75 m/s 9. -mv2 sin wtiˆ + m(v2 cos wt - v1 ) ˆj where w =
R
H 1. (d)
I. 1. 5

Section-B : JEE Main/ AIEEE


1. (d) 2. (d) 3. (b) 4. (a) 5. (b) 6. (d) 7. (a)

Section-A JEE Advanced/ IIT-JEE


A. Fill in the Blanks The magnitude of total impulse of force on the body from
1. 2mv' cos q = mv ... (i) t = 4 µs to t = 16 µs
v
2mv' sin q = mv = area (BCDFEB)
m = area of BCFEB + area CDFC
1
Þ sin q = cos q = 2mv'cos q
2 1 1
q m v = (200 + 800) ´ 2 ´ 10 -6 + ´ 10 ´ 800 ´ 10-6
Putting this value in equation 2 2
(i), we get 2m = 0.001 + 0.004 = 0.005 Ns
2mv ' v v' 2mv'sinq
= mv , v' = C. MCQs with ONE Correct Answer
2 2
1. (d) If we consider the two particles as a system then the
2
1 2 1 2 1 æ v ö external force acting on the system is the gravitational
Total K.E. = mv + mv + (2m) ç
2 2 2 è 2 ÷ø pull (m1 + m2) g.
Dp
1 2 1 2 mv 2 3 2 Fext =
mv + mv +
= = mv Dt
2 2 2 2 r r r r
2. KEY CONCEPT Area under the F – t graph gives the \ Dp = Fext Dt = ( m1v1 '+ m2v2 ') - ( m1v1 + m2 v2 )
impulse imparted to the body.
F
= (m1 + m2) g × 2t0
(in N) 2. (c) Just after collision
800 C
m1v1 + m2 v2 10 ´ 14 + 4 ´ 0
600 vc = = = 10 m / s;
m1 + m2 10 + 4
400
Note : Spring force is an internal force, it cannot change
200 A B
the linear momentum of the (two mass + spring) system.
E F D
Therefore vc remains the same.
0 2 4 6 8 10 12 14 16 T
EBD_7036
P-S-38 Topic-wise Solved Papers - PHYSICS
3. (d) For vertical motion of bullet or ball (c) is correct because for earth + ball system the impact
u = 0, s = 5m, t = ? , a = 10m/s2 force is an internal force.
M = 0.2 kg (d) is correct.
v 2. (a) As one piece retraces its path, the speed of this piece
m = 0.01 kg
just after explosion should be v cos q
– +
vcosq
5m m vcosq v'
vsinq v
m m
q 2 2
vcosq
20 m (At highest point just after explosion)
100 m
NOTE THIS STEP
Applying conservation of linear momentum at the
1 2 1
S = ut + at Þ 5 = ´ 10 ´ t 2 highest point;
2 2
m m
Þ t = 1 sec m (v cos q ) = ´ v '- ´ v cos q
For horizontal motion of ball 2 2
xball = Vball t Þ 20 = Vball × 1 = Vball 3 v cos q = v'
For horizontal motion of bullet 3. (b, d) In situation 1, mass C is moving towards right with
xbullet = Vbullet × t Þ 100 = Vbullet × 1 = Vbullet velocity v. A and B are at rest.
Applying conservation of linear momentum during In situation 2, which is just after the collision of C with
collision, w e get A, C stop and A acquires a velocity v. [head-on elastic
mV = mVbullet + MVball collision between identical masses]
0.01 V = 0.01 × 100 + 0.2 × 20 When A starts moving towards right, the spring suffer
5 a compression due to which B also starts moving
\V= = 500 m/s towards right. The compression of the spring continues
0.01
4. (a) Activity B to M for particle thrown upwards till there is relative velocity between A and B. When
this relative velocity becomes zero, both A and B move
é u 2 sin 2 a ù with the same velocity v' and the spring is in a state of
v12 - u 02 = 2(-g) ê 0 ú maximum compression.
ëê 2g ûú Applying momentum conservation in situation 1
and 3,
\ v12 = u 02 (1 - sin 2 a ) = u 02 cos 2 a
v
\ v1 = u0 cos a ...(i) v Vel=0
C A B C A B
2mv sinq 2mv
y Fig(i) Fig(ii)
q
v' v'
x 2mv cosq Vel=0
u 20 sin 2 a C A B
H= Max Compression
u0 2g Fig(iii)
a u0
m muo cosa m v
mv = mv' + mv' Þ v' =
2
Applying conservation of linear momentum in \ K.E. of the system in situation 3 is
Y-direction
2mv sinq = mv1 = mu0cosa ...(ii) [from (i)] 1 1 mv 2 æ vö
Applying conservation of linear momentum in
mv '2 + mv '2 = mv '2 = çèQ v ' = ÷ø
2 2 4 2
X-direction This is the kinetic energy possessed by A – B system (since,
2mv cosq = mu0 cosa ...(iii) C is at rest).
on dividing (ii) and (iii) we get Let x be the maximum compression of the spring.
p Applying energy conservation
tan q = 1 \q=
4 1 2 1 1 1
mv = mv '2 + mv '2 + Kx 2
D. MCQs with ONE or MORE THAN ONE Correct 2 2 2 2
1 2 1 2 1 2
1. (c,d) (a) is wrong because the momentum of ball changes Þ mv = mv + Kx
2 4 2
in magnitude as well as direction.
(b) is wrong because on collision, some mechanical 1 2 1 2 m
Þ Kx = mv \ x= v
energy is converted into heat, sound energy. 2 4 2K
Momentum and Impulse P-S-39

4. (a, d) KEY CONCEPT option (b) is incorrect


Use law of conservation of linear momentum. The time taken for the maximum compression
= tAB + tBA + tAC
The initial linear momentum of the system is piˆ - piˆ = 0
Therefore the final linear momentum should also be p m p m π m m é1 1 1 ù
= + + =π + +
zero. 3 k 3 k 3 k k êë 3 3 2 úû
Option a :
7π m
p1' + p2' = (a + a )iˆ + (b + b ) ˆj + c kˆ = Final = . Therefore option c is incorrect.
1 2 1 2 1 6 k
momentum. The time taken for particle to pass through the
It is given that a1, b1, c1, a2, b2 and c2 have non-zero
equilibrium position second time
values. If a1 = x and a2 = – x. Also if b1 = y and b2 =
éπ m ù m m æ2 ö 5 m
– y then the î and ĵ components become zero. But = 2ê ú+π =π çè + 1÷ø = π
the third term having k̂ component is non-zero. This ë3 k û k k 3 3 k
gives a definite final momentum to the system which option (d) is correct.
violates conservation of linear momentum, so this is
an incorrect option. E. Subjective Problems
Option d: 1. Let V¢ be the velocity of the final body after collision.
p1' + p2' Suppose, V¢ makes an angle q with x-direction.
= (a1 + a2 )iˆ + 2b1 ˆj ¹ 0 because b1 ¹ 0
Following the same reasoning as above this option is
also ruled out. (m+M) V¢sinq
5. (a, c) (m+M) V¢
According to law of conservation of linear momentum
1 × u1 + 5 × 0 = 1 (–2) + 5 (v2) mV q
Þ u1 = – 2 + 5 v2 ...(i)
The coefficient of restituition (m+M) V¢cosq
v2 - v1 v - ( -2)
e= Þ 1= 2 Mv
u1 - u 2 u1 - 0
Þ u1 = v2 + 2 ...(ii)
On solving (i) & (ii) we get desired results. (i) Applying conservation of linear momentum in X
6. (a, d) The particle collides elastically with rigid wall. Here direction
V (m + M) V' cos q = mV ... (i)
e= =1 \V = 0.5u 0
0.5u 0 Applying conservation of linear momentum in Y
direction
u0 (m + M) V' sin q = Mv ... (ii)
Dividing equation (i) and (ii)
B Mv æ Mv ö
tan q = Þ q = tan–1 çè ÷
A 0.5u0 mV mV ø
C
This gives the direction of the momentum of the final
Equilibrium position body.
i.e. the particle rebounds with the same speed. Therefore Squaring and adding (i) and (ii), we get
the particle will return to its equilibrium position with (m + M)2 V'2 cos2 q + (m + M)2 V'2 sin2 q
speed u0. option (a) is correct. = m2V2 + M2v2
The velocity of the particle becomes 0.5u0 after time t.
Then using the equation V = Vmax cos wt we get m 2V 2 + M 2v2
\ V' =
0.5u0 = u0 cos wt m+ M
p 2p T Thus the magnitude of the momentum of the final body
\ = ´T \t =
3 t 6 = ( m + M ) V ' = m2V 2 + M 2 v 2
m p m 1
The time period T = 2 p . Therefore t = (m + M )V '2
k 3 k (ii) K .E.i - K .E. f K .E. f 2
=1- = 1-
The time taken by the particle to pass through the K .E.i K .E.i é1 2 M 2ù
êë 2 mV + 2 v úû
2p m
equilibrium for the first time = 2t = . Therefore
3 k
EBD_7036
P-S-40 Topic-wise Solved Papers - PHYSICS

m 2V 2 + M 2 v 2 1
(m + M ) s1 = u1t1 + a1t12
2
DKE (m + M )2 +
=1- h = 49t – 4.9 t2 ... (ii) 98m
K .E.i 2 2 C
mV + Mv For the ball dropped from height

78.4m
K .E.i - K .E. f m 2V 2 + M 2 v 2 v22 - u22 = 2a2 s2
\ =1-
K .E.i (m + M ) (mV 2 + Mv 2 ) \ v22 = 19.6 (98 – h) ... (iii)

m2V 2 + mM 2 v 2 + MmV 2 + M 2 v 2 - m 2V 2 - M 2 v 2 1 2
= 2 2
s2 = u2 t2 + a2 t2
(m + M ) (mV + Mv ) 2
98 – h = 4.9 t2 ... (iv)
mM (v 2 + V 2 ) From (ii) and (iv)
= 98 – (49t – 4.9t2) = 4.9t2 \ 98 – 49t = 0
(m + M ) (mV 2 + Mv 2 )
\ t = 2 sec
2. mV mV mV \ h = 49 × 2 – 4.9 × 22 = 78.4 m (from (ii))
C Substituting this value of h in (i) and (ii), we get
C B
v12 – 2401 = – 19.6 × 78.4 v22 = 19.6 (98 – 78.4)

v12 = 864.36 Þ v22 = 384.16


A mV
B v1 = 29.4 m/s Þ v2 = 19.6 m/s
Initially Finally Note : At point C where the two bodies collide, thereafter
mV
both bodies stick and behave as a single body.
Initially By symmetry, the momentum of the system is zero. Thus, we apply conservation of linear momentum, which
Finally The momentum of the system should be zero. gives
\ mV = mV' Þ V' = V m1v1 – m2v2 = 2mv
The velocity of C is V and is in opposite direction to the v -v 29.4 - 19.6
retraced velocity of B as shown in the figure. \ v= 1 2 = = 4.9 m/s
2 2
3. Since the collision between C and A is elastic and their For the combined body
masses are equal and A was initially at rest, therefore the u = 4.9 m/s; s1 = – 78.4; a1 = – 9.8 m/s2; t = ?
result of collision will be that C will come to rest and A will
1 2
initially start moving with a velocity v0. But since A is s = ut + at Þ – 78.4 = 4.9t – 4.9t2
connected to B with a spring, the spring will get compressed. 2
m v A m B 2m
Vel=0
A m v' B 2m 1 ± 1 + 64
C o
C \ t2 – t – 16 = 0 Þ t = = 4.53
v' 2
At t = t0, the velocities of A and B become same. Total time = 4.53 + 2 = 6.53 sec.
Applying energy conservation; 5. (i) In D AQR

1 2 1 1 1 QR 5
mv0 = mv '2 + 2mv '2 + kx02 sin 30° = , QR =
2 2 2 2 10 / 3 3
u = 50 m/s (Given)
where x0 is the compression in the spring at t = t0
At the highest point P, the velocity of the bullet = u cos q
k 2
\ v02 = 3v '2 +
x0 ... (i)
m R
Applying momentum conservation, we get
30
O

v0 Y Q
mv0 = mv' + 2mv' \ v' = ... (ii) A
3
10/3
From (i) and (ii)

v2 k 2mv02 u cosq P
v02 - 3 ´ 0 = x02 Þ k= u
9 m 3x02 ( R ,H )
2
4. For the ball thrown up q
v12 - u12 = 2a1s1 O X
Applying conservation of linear momentum at the highest
\ v12 - 2401 = 19.6 h ... (i) point
M (u cos q ) + 3M × 0 = (M + 3M) v
Momentum and Impulse P-S-41

Mu cos q u cos q 6µgd


v= = = -3 ´ 5 = -3 6µgd
4M 4 5
Applying energy conservation principle for P and R
K.E. of the bullet-mass system at P = P.E. of the bullet-mass Note : The negative sign shows that the mass m rebounds.
system at R It then follows a projectile motion.
Considering the vertical direction motion of this projectile.
1
(4 M )v 2 = (4 M ) gh uy = 0, sy = d, ay = g, ty = ?
2
1 2 1 2 2d
1 u 2 cos2 q æ 10 5 ö S = ut + at Þ d = gt Þ t =
(4M ) = 4Mg ´ ç + ÷ 2 2 g
2 16 è 3 3ø
The horizontal distance travelled by mass m during this
9.8 ´ 5 ´ 2 ´ 16 time t
cos2 q = Þ q = 37°
50 ´ 50
2d
R u 2 sin 2q 50 ´ 50sin 2 ´ 37o x = 3 6µgd ´ = 6 3µd 2 = 6d 3µ
(ii) = = = 122.6 m g
2 2g 2 ´ 9.8
7. When the stone reaches the point Q, the component of
u 2 sin 2 q 50 ´ 50 ´ (sin 37°)2 velocity in the + Z direction (v cos q ) becomes zero due to
H= = = 46 m
2g 2 ´ 9.8 the gravitational force in the – Z direction.
6. KEY CONCEPT
Since the collision is elastic in nature applying conservation
+Z Z L v'
of linear momentum and conservation of kinetic energy +Y vx = 4m/s
mv = (4m) u + mv' Q Y
v
vcosq q
where u is the velocity of mass 4m after collision and v' is vsinq
P X
the velocity of mass 2m vx = 4m
t=t
t=0
Þ v' = v – 4u ... (i)
2m
1 2 1 2 1 2 A The stone has two velocities at Q
Also, mv = (4m) u + mv ' m v
2 2 2 B
4m
• vx in the + X direction (4 m/s)
Þ v2 = 4u2 + v'2 ... (ii) • v sin q in the + Y direction (6 sin 30° = 3 m/s)
From (i) to (ii) 4d
The resultant velocity of the stone
2v
v2 = 4u2 + (v – 4u)2 Þ u = v' = (v x )2 + (v sin q)2 = 42 + 32 = 5m / s
5
Block B starts moving but the block A remains at rest as (i) Applying conservation of linear momentum at Q for
there is no friction between A and B. collision with a mass of equal magnitude
For block A to topple, block B should move a distance 2d. m × 5 = 2m × v
Let the retardation produced in B due to friction force
Note : Since, the collision is completely inelastic the two
between B and the table be a
F = mN Þ (4 m)a = m (6 mg) Þ a = 1.5 µg masses will stick together. v is the velocity of the two masses
For the motion of B, just after collision.]
\ v = 2.5 m/s
2v
u= , v = 0, s = 2d, a = – 1.5 µg (ii) When the string is undergoing circular motion, at any
5
2mv 2
æ 2v ö
2 arbitrary position T – 2mg cos a =
Now, v2 – u2 = 2as Þ (0)2 – ç ÷ = 2( -1.5µg ) 2d l
è 5ø
2mv 2
5 Given that, T = 0 when a = 90° \0 – 0 = Þ v=0
Þ v= 6µgd l
2
Þ Velocity is zero when a = 90°, i.e., in the horizontal
For elastic collision between two bodies
position. M
(m1 - m2 ) u1 2m2 u2 Applying energy conservation
a
v1 = + from Q to M, we get
m1 + m2 m1 + m2
1 T
Here m1 = m, m2 = 4m, u1 = 5 6µgd , u2 = 0 2mv 2 = 2mg l
2
(m - 4m)5 6µgd 3 6m gd v 2 (2.5) 2
Þ v1 = +0 2mg sin a
m + 4m Þ l= = = 0.318 m a 2mg cosa
d 2 g 2 ´ 9.8 2mg
Q
EBD_7036
P-S-42 Topic-wise Solved Papers - PHYSICS
8. Consider the vertical motion of the cannon ball
\ a = 0 m/s2 \ v = u + at = 5 3 m/s.
uy = + 100 sin 30°
To find the final velocity of the carriage after the second
sy = – 120 m + P uy u
_ 30 o
impact we again apply conservation of linear momentum in
ay = – 10 m/s2 ux
C the horizontal direction
t = t0 120m m = 9kg
mux + (M + m) vx= (M + 2m) vf
A B u =0
1 2 2 T
s = ut + at \ – 120 = 50 t0 – 5 t0 \ 1× 50 3 + (9 + 1) 5 3 = (9 + 2) vf
2
Þ vf = 15.75 m/s
Þ t02 – 10 t0 – 24 = 0 r r v2
9. v2 = (- v2 sin wt $i + v2 cos wt $j ); v1 = v1 $j
t
r r r
(-10) ± 100 - 4(1) (-24) v PM = v2 - v1
\ t0 = – = 12 or – 2 [Not valid]
2 t
= -v2 sin wt $i + (v2 cos wt - v1 ) $j
\ t0 = 12 sec.
r r
p PM = mv PM
The horizontal velocity of the cannon ball remains the same
= -mv2 sin wt i$ + m(v2 cos wt - v1 ) $j
\ ux = 100 cos 30° = 50 3 ms -1
v2
\ Applying conservation of linear momentum to the where w =
R
cannon ball-trolley system in horizontal direction. If m is the
mass of cannon ball and M is the mass of the trolley then, r é ˆj ù
or, p PM = m êæç -v2 sin 2 tiˆö÷ + æç v2 cos 2 t - v1 ö÷
v v
mux + M × 0 = (m + M) vx è R ø è R ø ú
ë û
mu x
\ vx = where vx is the velocity of the
m+M H. Assertion & Reason Type Questions
cannon ball-trolley system.
1. (d) Statement 1 : For an elastic collision, the coefficient of
1 ´ 50 3 restitution = 1
\ vx= = 5 3m/s
1+ 9 | v2 - v1 |
e= Þ | v2 – v1| = | u1 – u2 |
The second ball was projected after 12 second. Horizontal | u1 - u2 |
distance covered by the car P
Þ Relative velocity after collision is equal to relative
= 12 × 5 3 = 60 3 m velocity before collision. But in the statement relative
Since the second ball also struck the trolley, speed is given.
Therefore, in time 12 seconds, the trolley covers a distance Statement 2 : Linear momentum remains conserved in
an elastic collision. This statement is true.
of 60 3 m.
For trolley after 12 seconds; I. Integer Value Correct Type
m ux
u = 5 3 m/s , v = ?, t = 12 s M 1. 5 Velocity at the highest point of bob tied to string l1 is
acquired by the bob tied to string l2 due to elastic
1 2 head-on collision of equal masses
s = 60 3 m, s = ut + at
2
l1
Therefore gl1 = 5gl 2 \ =5
1 l2
Þ 60 3 = 5 3 × 12 + × a × 144
2
Momentum and Impulse P-S-43

Section-B JEE Main/ AIEEE


1. (d) Let n be the number of bullets that the man can fire in 5. (b) During each collision
one second.
2
\ change in momentum per second = n ´ mv = F Initial velocity = = 1 ms -1
2
[ m= mass of bullet, v = velocity] (Q F is the force)
F 144 ´ 1000 2
\n = = =3 Final velocity = - = -1 ms -1
mv 40 ´ 1200 2
2. (d) In x-direction, mv = mv1 cosq ...(1) Impulse = Change in momentum
where v1 is the velocity of second mass = m v2 - v1 = 0.4 × 2 = 0.8 Ns

In y-direction,
mv
= mv1 sin q ...(2) P2 P2
6. (d) Maximum energy loss = -
3 2m 2(m + M)
é P2 1 ù
mv / 3 êQ K.E. = = mv2 ú
êë 2m 2 úû
v
m v
mv1 cos q P2 é M ù 1 ì M ü
q = ê ú = mv 2 í ý
2m ë (m + M) û 2 îm + M þ
mv1 Statement II is a case of perfectly inelastic collision.
mv1 sin q By comparing the equation given in statement I with
above equation, we get
Squaring and adding eqns. (1) and (2)
æ M ö æ m ö
v2 f=ç
v12 =v +2
Þ v1 = v
2 è m + M ÷ø instead of çè M + m ÷ø
3 3 Hence statement I is wrong and statement II is correct.
3. (b) Let the velocity and mass of 4 kg piece be v1 and m1
Y
and that of 12 kg piece be v2 and m2.
pf = 3 m V
16 kg
m pi
2v 45°
Initial momentum 7. (a) X
=0
Situation 1

m2 = 12 kg Final momentum v
4 kg = m1 2m
v1 v2 = m2v2 – m1v1
Situation 2
Initial momentum of the system
pi = [m(2V)2 ´ m(2V)2 ]
Applying conservation of linear momentum
= 2m ´ 2V
12 ´ 4
m2v2 = m1v1 Þ v1 = = 12 ms -1 Final momentum of the system = 3mV
4 By the law of conservation of momentum

1 1 2 2mv = 3mV
\ K .E.1 = m1v12 = ´ 4 ´ 144 = 288 J
2 2 2 2v
Þ = Vcombined
4. (a) In completely inelastic collision, all energy is not lost 3
(so, statement -1 is true) and the principle of Loss in energy
conservation of momentum holds good for all kinds of 1 1 1
DE = m1V12 + m2V22 - (m1 + m2 )Vcombined
2
collisions (so, statement -2 is true) . Statement -2 2 2 2
explains statement -1 correctly because applying the 4 5
principle of conservation of momentum, we can get the DE = 3mv2 - mv2 = mv2 = 55.55%
3 3
common velocity and hence the kinetic energy of the Percentage loss in energy during the collision = 56%
combined body.
EBD_7036
6
P-S-44 Topic-wise Solved Papers - PHYSICS

Rotational Motion

Section-A : JEE Advanced/ IIT-JEE


2 M w0 1
A 1. mg 2. 3. MRAw 2 4. –3
3 M + 6m 3
æ d - xö xW
5. çè ÷W , 6. 4.8 Ma2
d ø d
B 1. F 2. F 3. F 4. F
C 1. (c) 2. (c) 3. (c) 4. (c) 5. (a) 6. (a) 7. (c) 8. (d)
9. (b) 10. (a) 11. (b) 12. (b) 13. (a) 14. (a) 15. (b) 16. (b) 17. (d)
18. (a) 19. (b) 20. (b) 21. (d) 22. (d) 23. (a) 24. (c) 25. (b) 26. (d)
D 1. (d) 2. (b) 3. (a,c) 4. (b, d) 5. (a, c, d) 6. (a, b, c) 7. (a) 8. (c)
9. (a) 10. (a, b, c) 11. (c, d) 12. (a) 13. (b,c) 14. (a, b, c) 15. (c) 16. (a, b) 17. (d)
18. (c, d) 19. (d) 20. (a,c) 21. (a,b)
1 gM
E 1. T = mg[3cos q - 2cos q0 ] , q0 = 30° 2. n=
2p lm
m( R - r ) 2 g (R - r)
3. 9 cm from the origin towards left 4. ,m
M +m M (m + M )

mv03
5. , perpendicular to the plane of motion and is directed away from the reader.. 6. 2m, yes
2 2g
12v
7. 2.72 J 8. (a) (b) 3.5 ms–1
7L
2 2
é x ù æ yö
9. (i) Straight line, (ii) ê
L ú + çè r ÷ø = 1, Ellipse 10. (i) 1.63N (ii) 1.22 m 11. 6.3 m/s
ê -rú
ë2 û

4 4 gR
12. (a) q = cos -1 (b) (c) 6 13. (L + 2R, 0)
7 7
14. (i) 6 î (ii) 0.6(kˆ - ˆj ), 0.6(- ˆj - kˆ)

2 mv æ 2 mv ö 4mv
15. (a) ( 3iˆ - kˆ), ç + mg ÷ kˆ (b) ´h 16. 5gR
3Dt è 3Dt ø 3Dt
8F 4F 3FM1 FM1
17. (a) 3M + 8m , 3M + 8m (b) 3M + 8m , 3M + 8m
1 2 1 2 1 2 1 2
F 2
18. (a) 0.1 m (b) 1 radian/sec. (c) infinite 19. (a) 3mlw 2 (b) ( Fnet ) x = - , ( Fnet ) y = 3mlw
4
3mv 2
20. w = 21. a = g sin q
( M + 3m) L 3
é gù
22. ê( M + m ) ú cot q, along AB. 23. 10 m/s
ë 2û
F 1. (A) ® p, t; (B) ® q, s, t; (C) ® p, r, t; (D) ® q, p
G 1. (c) 2. (a) 3. (b) 4. (d ) 5. (d) 6. (c) 7. (a) 8. (b)
H 1. (d) 2. (d)
I 1. 6 2. 4 3. 9 4. 3 5. 8 6. 4 7. 2 8. 7 9. 6
Rotational Motion P-S-45

Section-B : JEE Main/ AIEEE

1. (c) 2. (b) 3. (b) 4. (c) 5. (d) 6. (a) 7. (d) 8. (d) 9. (a)


10. (d) 11. (b) 12. (a) 13. (a) 14. (c) 15. (d) 16. (c) 17. (c) 18. (c)
19. (d) 20. (b) 21. (b) 22. (d) 23. (d) 24. (a) 25. (d) 26. (c) 27. (b)
28. (c) 29. (a) 30. (c) 31. (c) 32. (d) 33. (a) 34. (c)

Section-A JEE Advanced/ IIT-JEE


A. Fill in the Blanks
3f MAw 2
1. KEY CONCEPT N or, Aw 2 = or,, f =
M 3
When the cube begins to tip
about the edge the normal B C Thus, maximum torque,
reaction will pass through the F
MAw 2 R 1 2
edge about which rotation 3a/4 tmax = f×R = = MARw
takes place. The torque due to 3 3
N and f will be zero. 4. Let at any instant of time t, the radius of the horizontal
A f D
Taking moment of force about D mg surface be r.
T = mrw2 ... (i)
3a a 2 Where m is the mass of stone
F× = mg ´ \ F = mg w
4 2 3 and w is the angular velocity r
2. Note : Since no external force and hence no torque is applied, at that instant of time t.
the angular momentum remains constant Also, L = Iw ... (ii) T m
\ I1w1 = I2w2 From (i) and (ii)
ML2 mrL2 mL2 æ L2 ö
I1w1 ´ w0 M w0 T= = ´ r , T = ç ÷ r -3
\ w2 = = 12 = I2 (mr 2 )2 è mø
I2 ML2 æ Lö
2 M + 6m æ ö
+ 2m ´ ç ÷ L2
12 è 2ø = Ar -3 çè where = A is constant ÷
ø
m
Thus, n = –3
3. Considering the motion of the platform
5. RA + RB = W
x = A cos wt
\ RA = W – RB
dx d2x 6. Assuming symmetric lamina to be in xy plane, we will have
Þ = - Aw sin wt Þ = - Aw 2 cos wt Ix = Iy (Since the mass distribution is same about x-axis and
dt 2
dt
y-axis)
The magnitude of the maximum acceleration of the platform Ix + Iy = Iz (perpendicular-axis theorem)
is It is given that Iz = 1.6 Ma2.
\ | Max acceleration | = Aw2 Hence
Y
A B
When platform moves a torque acts on the cylinder and the
a
cylinder rotates about its axis. Iz
Ix = Iy = = 0.8 Ma 2
f 2 a
Acceleration of cylinder, a1 = Now, according to parallel-axis X
m O
theorem, we get
Torque t = fR \ Ia = fR
IAB = Ix + M(2a)2
fR fR = 0.8 Ma2 + 4Ma2
a = =
I MR 2 / 2 = 4.8 Ma2
2f 2f B. True/False
or, a = or Ra =
MR M
t
\ Equivalent linear acceleration ( Ra = a2 ) 1. t = Ia \ a =
I
2f t = Force × perpendicular distance. Torque is same in both
a2 = the cases. But since, I will be different due to different mass
M
distribution about the axis,
\ Total linear acceleration,
\ a will be different.
f 2f 3f
amax = a1 + a2 = + =
M M M
EBD_7036
P-S-46 Topic-wise Solved Papers - PHYSICS
ur
r dL r w1 w2
2. t= Since, t = 0
dt
w
ur
\ L = constant M r
m m
\ I1w1 = I2w2
R
1 2 2 2 2
I1 = MR I = Mr I2=Mr +mr
2
w1 = w i.e., I1w1 = I2 w2
Mr2 × w1 = (Mr2 + 2mr2) w2 (Q w1 = w )
1 2 1 M 2 æ 4 + 1ö 2 5 2
I2 = MR + R =ç ÷ø MR = MR Mw
2 2 4 è 8 8 \ w2 =
M + 2m
w2 = ?
2. (c) The moment of inertia of the system about axis of
1 rotation O is 0.3 kg
I1w1 MR 2 ´ w I = I1 + I2 = 0.3x2 + 0.7 (1.4 – x)2
2 8 4
w2 = =+ = w= w = 0.3x2 + 0.7 (1.96 + x2 – 2.8x) x
I2 5 2´5 5
MR 2 = x2 + 1.372 – 1.96x
8 The work done in rotating the O
3. Total energy of the ring rod is converted into its
rotational kinetic energy.
= (K.E.)Rotation + (K.E.)Translational 1.4 – x
1 2
1 2 1 2 \ W= Iw
= I w + mvc 2
2 2 0.7 kg
1
= [ x 2 + 1.372 - 1.96 x ] w2
1 1 2
= ´ mr 2 w 2 + m(r w )2 (Q I = mr2, vc = rw)
2 2 For work done to be minimum
dW
= mr 2 w 2 =0 Þ 2x – 1.96 = 0
dx
Total kinetic energy of the cylinder
= (K.E.)Rotation + (K.E.)Translational 1.96
Þ x= = 0.98 m
2
1 1
= I ' w '2 + Mv 'c2 3. (c) As the spheres are smooth there will be no friction (no
2 2
torque) and therefore there will be no transfer of angular
1æ1 2ö 2 1 2
momentum. Thus A, after collision will remain with its
= çè Mr ÷ø w ' + M (r w ') initial angular momentum. i.e., w A = w
2 2 2
4. (c) KEY CONCEPT
3 2 2
= Mr w ' ... (i) The disc has two types of motion namely translational
4
and rotational. Therefore there are two types of angular
Equating (i) and (ii)
momentum and the total angular momentum is the
3 2 2 vector sum of these two.
mr2w2 = Mr w ' y
4 In this case both the
angular momentum have
w
w '2 4m 4 0.3 the same direction
Þ 2
= = ´ =1
w 3M 3 0.4 (perpendicular to the R
v
plane of paper and away
Þ w' = w C
from the reader).
Both will reach at the same time. x
O
4. Since no external force is acting on the two particle system ur ur ur
\ ac.m = 0 L = LT + L R
Þ Vc.m = Constant. LT = angular momentum due to translational motion.
C. MCQs with ONE Correct Answer LR = angular momentum due to rotational motion
about C.M.
1. (c) Since the objects are placed gently, therefore no L = MV × R + Icmw
external torque is acting on the system. Hence angular
momentum is constant. Icm = M.I. about centre of mass C.
Rotational Motion P-S-47

The moment of inertia of the loop about XX' axis is


1 2
= M (R w) R + MR w
2 MR 2 3
IXX' = + MR 2 = MR 2
(v = Rw in case of rolling motion and surface at rest) 2 2
Where M = mass of the loop and R = radius of the loop
3
= MR 2 w L
;
2 Here M = Lr and R =
2p
a 2
3 æ Lö 3L3r
\ IXX' = ( Lr) ç ÷ =
v M 2 è 2p ø 8p 2
a C
5. (a) M r w 9. (b) The M.I. about the axis of rotation is not constant as
the perpendicular distance of the bead with the axis of
O O rotation increases.
a 2 Also since no external torque is acting.
a
r= 2 or r2 = dL
2 2 \ text = Þ L = constant Þ Iw = constant
Net torque about O is zero. dt
Therefore, angular momentum (L) about O will be Since, I increases, w decreases.
conserved, or Li = Lf 10. (a) The mass distribution of this sector is same about the
axis of rotation as that of the complete disc about the
æ aö 2
MV ç ÷ = I 0 w = ( Icm + Mr )w axis. Therefore the formula remains the same as that of
è 2ø the disc.
11. (b) Imagine the cylinder to
ìï Ma 2 æ a 2 ö üï 2 3v
w= í + M ç ÷ ý w = Ma w =
2 be movin g on a F
3 4a frictionless surface. In
îï 6 è 2 ø þï N
g sin q Point of
both the cases the contact
6. (a) Note : When we are giving an angular acceleration to acceleration of the g sin q
the rod, the bead is also having an instantaneous centre of mass of the
acceleration a = La. This will happen when a force is
cylinder is g sin q. This mg sin q
exerted on the bead by the rod. The bead has a tendency mg
to move away from the centre. But due to the friction is also the acceleration q
of the point of contact
between the bead and the rod, this does not happen to
of the cylinder with the inclined surface. Also no torque
the extent to which frictional force is capable of holding
the bead. (about the centre of cylinder) is acting on the cylinder
The frictional force here provides the necessary since we assumed the surface to be frictionless and
centripetal force. If instantaneous angular velocity is the forces acting on the cylinder is mg and N which
w then pass through the centre of cylinder. Therefore the net
movement of the point of contact in both the cases is
mLw2 = µ(ma) Þ mLw2 = µ mLa Þ w2 = µa
in the downward direction as shown. Therefore the
By applying
a frictional force will act in the upward direction in both
Þ w = w0 + at,
the cases.
We get w = at
Note : In general we find the acceleration of the point
µ of contact due to translational and rotational motion
\ a2t2 = µa Þ t = L
a and then find the net acceleration of the point of contact.
The frictional force acts in the opposite direction to
7. (c) The applied force shifts the normal reaction to one
that of net acceleration of point of contact.
corner as shown. At this situation, the cubical block
12. (b) Since there is no external
starts topping about O. Taking torque about O
torque, angular momentum C
F remains conserved. As
L moment of inertia initially
N
decreases and then B
L/2 increases, so w will increase
initially and then decreases.
A
O Note : The M.I. of the system decreases when the
mg tortoise move from A to B and then increases from B
L mg to A.
F × L = mg × Þ F= So the variation of w is nonlinear.
2 2
8. (d) Moment of inertia about the diameter of the circular 13. (a) Change in angular momentum of the system = angular
impulse given to the system about the centre of mass
1 (Angular momentum)f – (Angular momentum)i
loop (ring) = MR 2
2
Using parallel axis theorem
EBD_7036
P-S-48 Topic-wise Solved Papers - PHYSICS

L 18. (a) Let s be the mass per unit area.


= Mv × ... (i)
2 R/ 3
Let the system starts rotating with the angular O'
velocity w.
Moment of Inertia of the system about its axis of rotation
[centre of mass of the system] 2R/3

2 2 O
æ Lö æ Lö 2ML2 ML2
= M ç ÷ +Mç ÷ = =
è 2ø è 2ø 4 2
L
From (i) Iw – 0 = Mv
2
Mv L Mv L v The total mass of the disc
Þ w= ´ = ´ =
I 2 ML2 / 2 2 L = s × p R2 = 9M
14. (a) The net force acting on a particle undergoing uniform The mass of the circular disc cut
circular motion is centripetal force which always passes 2
through the centre of the circle. The torque æ Rö p R2
ur due to this = s ´ p çè ÷ø = s ´ =M
force about the centre is zero, therefore, L is conserved 3 9
about O. Let us consider the above system as a complete disc of
mass 9M and a negative mass M super imposed on it.
L2
15. (b) KEY CONCPET : (K.E.)rotation = . Moment of inertia (I 1 ) of the complete disc =
2I
Here , L = constant 1
9 MR 2 about an axis passing th rough O and
\ (K.E.)rotational × I = constant. 2
When I is doubled, K.E.rotational becomes half. perpendicular to the plane of the disc.
16. (b) Note : In pure rolling, the point of contact is the M.I. of the cut out portion about an axis passing through
instantaneous centre of rotation of all the particles of O' and perpendicular to the plane of disc
the disc. On applying v = rw 2
1 æ Rö
We find w is same for all the particles then v µ r. = ´M ´ç ÷
2 è 3ø
Farther the particles from O, higher is its velocity.
17. (d) The cubical block is in equilibrium. \ M.I. (I2) of the cut out portion about an axis passing
For translational equilibrium through O and perpendicular to the plane of disc
(a) SFx = 0 Þ F = N é1 æ 2R ö ù
2 2
æ Rö
(b) SFy = 0 Þ f = mg = ê ´M ´ç ÷ + M ´ç ÷ ú
êë 2 è 3ø è 3 ø ú
û
y
f [Using perpendicular axis theorem]
2a \ The total M.I. of the system about an axis passing
x through O and perpendicular to the plane of the disc is
a I = I1 + I2

1 é1 æ Rö
2
æ 2R ö ù
2
b C F = 9 MR 2 - ê ´ M ´ ç ÷ + M ´ ç ÷ ú
2 êë 2 è 3ø è 3 ø ú
N û

9MR 2 9MR 2 (9 - 1)MR 2


mg = - = = 4 MR 2
2 18 2
For Rotational Equilibrium 19. (b) Since v is changing (decreasing), L is not conserved in
Stc = 0 magnitude. Since it is given that a particle is confined
Where tc = torque about c.m. to rotate in a circular path, it cannot have spiral path.
Torque created by frictional force (f) about C = f × a in Since the particle has two accelerations ac and at
clockwise direction. therefore the net acceleration is not towards the centre.
There should be another torque which should counter L
this torque. The normal reaction N on the block acts as v
shown. This will create a torque N × b in the
anticlockwise direction. ac
at
Such that f×a=N×b
Note : The normal force does not act through the centre ur
of the body always. The point of application of normal The direction of L remains same even when the speed
force depends on all the forces acting on the body. decreases.
Rotational Motion P-S-49

20. (b) point masses where the mass of each body is


A' considered at its geometrical centre.
Y
Y Y
M B
(–a , a) (a, a)
(–a, a) (a, a)
m m
r M
m m
R m
(0, 0)
X 6m
(0, 0)
X
A (0, –a)
m m
B' Y' (0, –a)
For solid sphere The y-coordinate of the centre of mass is
2 2 m y + m2 y2 + m3 y3 + m4 y4 + m5 y5
IAB = MR = I (given) ... (i) ycm = 1 1
5 m1 + m2 + m3 + m4 + m5
For solid disc
6m ´ 0 + m ´ 0 + m ´ a + m ´ a + m ( - a )
1 3 \ ycm =
IA'B' = IYY' + Mr2 = Mr 2 + Mr 2 = Mr 2 6m + m + m + m + m
2 2
IAB = IA'B' (given) ... (ii) ma a
==
From (i) and (ii), 10 m 10
24. (c) The angular LP
2 3 2 momentum of the mass
MR 2 = Mr 2 Þ r = R
5 2 15 m about O is mr2w and P

21. (d) By the concept of energy conservation is directed toward +z


direction for all
1 2 1 2 æ 3v 2 ö locations of m.
mv + I w = mg ç ÷ The angular l
2 2 è 4g ø
momentum of mass m
1 2 1 v2 3 2 about P is mvl and is LO
v
\ mv + I 2 = mv [Q v = Rw] directed for the given
2 2 R 4 location of m as shown m
O r
1 v2 3 2 1 2 1 2 in the figure.
\ I = mv - mv = mv r
2 R2 4 2 4 The direction of LP remains changing for different
locations of m.
1 2 r
Þ I = mR r
2 dL
25. (b) We know that |t|= where L = Iw
This is the formula of the moment of inertia of the disc. dt
22. (d) This is the case of vertical motion when the body just
completes the circle. Here d dI
\ t= (I w) = w ...(i)
v = 5gL ..... (i) dt dt
From the situation it is clear that the moment of inertia
Applying energy conservation, for (rod + insect) system is increasing.
Total energy at A = Total energy at P z
2
1 2 1 æ vö w
mv = m ç ÷ + mgh.
2 2 è 2ø O
v
L–h q L
3v 2 2
v
Þh= M P
8g h O
v x
Reference A
3 15L Level for PE. Let at any instant of time 't', the insect is at a distance x
= × 5gL = ...(ii)
8g 8 from O. At this instant, the moment of inertia of the
15L system is
L-
L-h 8 -7 1
In D OPM, cosq = = = I = ML2 + mx2 ...(ii)
L L 8 3
3p From (i) & (ii)
Therefore, the value of q lies in the range <q<p
4 d é1 ù d
t =w ê ML2 + mx 2 ú = w m ( x 2 )
23. (a) The system is made up of five bodies (three circles and dt ë 3 û dt
two straight lines) of uniform mass distribution.
Therefore we assume the system to be made up of five dx
= 2wmx = 2wmxv
dt
EBD_7036
P-S-50 Topic-wise Solved Papers - PHYSICS
= 2wmv2t [Q x = vt] The quantities on the right side of the equation are not
\ tµt (till t = T) changing.
ur The magnitude is constant. The direction is also constant.
When the insect stops moving, L does not change 3. (a, c) When the cycle is not pedalled but the cycle is in
and therefore t becomes constant. motion (due to previous effort) the wheels move in the
26. (d) Considering the normal reaction of the floor and wall direction such that the centre of mass of the wheel move
to be N and with reference to the figure. forward. Rolling friction will act in the opposite direction to
the relative motion of the centre of mass of the body with
Nsin 30° respect to ground. Therefore the rolling friction will act in
N 30°
backward direction in both the wheels. The sliding friction
will act in the forward direction of rear wheel during pedalling.
Ncos 30°
4. (b, d) Angular momentum = (momentum) × (perpendicular
distance of the line of action of momentum from the axis of
30°
rotation)
Angular momentum about O
x
h
mv
L= ´h ... (i)
2
l/2 1.6g
N
v 2 sin 2 q v 2
Now, h = = [Q q = 45°] ... (ii)
2g 4g
A f B
From (i) and (ii)
By vertical equilibrium. m
L= (2 gh ) h = m 2 gh3 v cos45°
2 v
3.2 g Ö2
N + N sin 30° = 1.6 g Þ N = … (i) Also, from (i) and (ii) v
3 h h
By horizontal equilibrium 2 3 45°
mv v mv
L= ´ = O
3 16 3 2 4g 4 2 g
f = Ncos 30° = N= From (i)
2 3 5. (a, c, d)
Taking torque about A we get Applying conservation of linear momentum
1.6g × AB = N × x 2m (– v) + m (2v) + 8m × 0 = (2m + m + 8m) vc
Þ vc = 0
l 3.2 g 3l
1.6 g × cos 60° = ´x \ = x … (ii)
2 3 8 +
2m
h h -
But cos 30° = \ x= … (iii) v
3a
x cos 30°

h 3l h 3 3 a 2a
From (ii) and (iii) = \ = 2v
cos 30° 8 l 16
m
D. MCQs with ONE or MORE THAN ONE Correct Applying conservation of angular momentum about centre
1. (d) We know that Fext = Mac.m. ... (i) of mass
We consider the two particles in a system. Mutual force 2mv × a + m (2v) × 2a = Iw ...(i)
of attraction is a internal force. There are no external 1
forces acting on the system. From (i) Where I = (8m) × (6a)2 + 2m × a2 + m × 4a2
12
ac.m. = 0 Þ vc.m. = constant. I = 30ma2 ...(ii)
Since, initially the vc.m. = 0
From (i) and (ii)
\ Finally vc.m. = 0
2. (b) Angular momentum v
ur r ur 2mv ( a ) + m ( 2v ) ´ 2 a = 30ma 2 ´ w Þ w =
Y 5a
L=r´ p
L = Momentum × 1 2
M v Energy after collision, E = I w
perpendicular distance of line 2
of action of momentum w.r.t
point of rotation y 1 v2 3mv 2
= ´ 30 ma 2 ´ =
L = Mv × y. X 2 25a 2 5
O
Rotational Motion P-S-51

6. (a, b, c) To find the moment of inertia of ABCD about an axis perpendicular axes, A'
passing through the centre O and perpendicular to the plane C' D
of the plate, we use perpendicular axis theorem. If we Izz = IAB + IA'B' = ICD + IC'D'
consider ABCD to be in the X-Y plane then we know that
Izz' = Ixx' + Iyy' Þ 2IAB = 2ICD A q
B
\ Izz' = I1 + I2 4 1
\ IAB = ICD
Also, Izz' = I3 + I4 A B
ur
Adding (i) and (ii), r dL
2Izz' = I1 + I2 + I3 + I4 10. (a, b, c) KEY CONCEPT t = C D'
dt B'
But I1 = I2 and I3 = I4 O 3 Given that
uur
(By symmetry) r ur ur dL ur ur
\ 2I zz ' = I1 + I1 + I3 + I3 D
t = A´ L Þ = A´ L
C dt
= 2I1 + 2I3 2 uur
dL
Þ Izz' = I1 + I3 From cross-product rule, is always perpendicular to the
dt
7. (a) The force acting on the mass of liquid dm of length dx ur ur
at a distance x from the axis of rotation O. plane containing A and L.
By the dot product definition
dF = (dm) x w2
F r ur
x L . L = L2
dx
M dF Differentiating with respect to time
\ dF = dx ´ xw 2 uur uur uur
L ur dL ur dL ur dL
dL dL
M
O L× + L× = 2L Þ 2L × = 2L
where is mass of liquid in unit length. dt dt dt dt dt
L uur
dL r ur
\ The force acting at the other end is for the whole liquid Since, i.e. t is perpendicular to L
in tube. dt
uur
LM M 2 L r dL dL
F= ò0 L
w 2 x dx =
L
w ò x dx
0
\ L×
dt
=0 Þ
dt
=0

L Þ L = constant
M 2 é x2 ù M 2 é L2 ù MLw 2 Thus, the magnitude of L always remains constant.
= w ê ú = w ê - 0ú = r
L êë 2 ûú 0 L êë 2 úû 2 As A is a constant vector and it is always perpendicular
r
to t,
8. (c) When the car is moving in a circular horizontal track of r r
radius 10 m with a constant speed, then the bob is also Also, L is perpendicular to A
undergoing a circular motion. The bob is under the influence r r rr
of two forces. Q L ^ A \ L. A = 0
r r
(i) T (tension in the rod) v Thus, it can be concluded that component of L along A is
zero i.e., always constant.
(ii) mg (weight of the bob) 11. (c, d) As shown in the figure, the component of weight
Resolving tension, we get Tcosq mg sin q tends to slide the point of contact (of the cylinder
q T
q with inclined plane) along its direction. The sliding friction
T cos q = mg ... (i) Tsinq acts in the opposite direction to oppose this relative motion.
mg Because of frictional force the cylinder rolls.
2
mv N
And T sin q = ... (ii)
r f
(Here T sin q is producing the necessary centripetal
force for circular motion)
q
inq
Dividing (ii) by (i), we get
s mg cosq
v 2 10 ´ 10 mg q mg
tan q = = = 1 Þ q = 45° Thus frictional force adds rotation but hinders translational
rg 10 ´ 10
motion.
9. (a) A'B' ^ AB and C' D' ^ CD Applying Fnet = ma along the direction of inclined plane,
we get mg sin q – f = mac,
From symmetry IAB = IA'B' and ICD = IC'D' where ac = acceleration of centre of mass of the cylinder
\ f = mg sin q – mac … (i)
From theorem of
EBD_7036
P-S-52 Topic-wise Solved Papers - PHYSICS
15. (c) The frictional force between òtdt
g sin q g sin q 2
But ac = = = g sin q ... (ii) the ring and the ball is Jx=2N-s
Ic 2
mR / 2 3 impulsive. The angular
1+ 2 1+ o
mR mR 2 impulse created by this force
Jy=1Ns
tends to decrease the angular
mg sin q speed of the ring about O. c
From (i) and (ii), f =
3 After the collision the angular speed decreases but the
If q is reduced, frictional force is reduced. ring remains rotating in the anticlock wise direction.
r Therefore the friction between the ring and the ground
r d psystem
12. (a)
åFext =
dt 16.
(at the point of contact) is towards left.
(a, b) For rolling motion, the velocity of the point of contact
r r
Given åFext = 0 Þ psystem = Constant with respect to the surface should be zero. For this
r r
Due to internal forces acting in the system, the kinetic 3Rw( -iˆ) + v = 0 o \ v = 3Rw iˆ o
and potential energy may change with time.
Also zero external force may create a torque if the line R R
of action of forces are along different direction. Thus w w cos30º kˆ
2 2
the torque will change the angular momentum of the
system.
R
13. (b, c) w sin 30º ( -iˆ ) 3R w iˆ
2
V C = 2V 0 P

30º
V B =V0
A shown in the figure, the point P will have two
velocities
VA =0 (i) 3Rw iˆ (due to translational motion)
ur
If V 0 is the velocity of centre of the sphere, then R
(ii) w making an angle of 30º with the vertical due
ur ur ur ur ur 2
V C = 2V 0 , V B = V 0 and V A = 0 to rotation
ur ur ur ur ur
\ V C - V B = 2V 0 - V 0 = V 0 uur é ˆ Rw ù 3 Rw ˆ
ur ur ur r ur \ vP = ê 3 Rw i - iˆ ú + k
ë 4 û 4
V B -V A = V 0 - 0 = V 0
ur ur ur ur 11 3
\ V C -V B = V B -V A = Rw iˆ + Rw kˆ
4 4
(b) is the correct option.
ur ur ur ur ur 17. (d) The acceleration of the center of mass of cylinder rolling
Now, | V C - V A | = | 2V 0 - 0 | = | 2V 0 |= 2 | V 0 | down an inclined plane is
C
ur ur ur ur g sin q
and | V C - V A | = 2 | V B - V C | ac =
I
(c) is the correct option. 1+ P 2 ac
14. (a,b,d) MR
Let V be the volume of spheres. Here IP > IQ because in
VdF g case of P the mass is
For equilibrium of A :
T + VdAg = Vdfg A VdA g concentrated away from
the axis.
\ T = Vg (df – dA) ...(1)
T \ aP < aQ Þ vP < vQ Þ wP < wQ
For T > 0, df > dA or dA < df
18. (c, d) When m1 ¹ 0 and m2 ¹ 0
(a) is the correct option
T
For equilibrium of B : VdF g
T + Vdfg = VdBg B N1 f1 = m1N1
\ T = Vg (dB – df) ...(2)
For T > 0, dB > df VdB g m1
(b) is the correct option
From (1) & (2) Vg (df – dA) = Vg (dB – df) N2
\ df – dA = dB – df m2 mg
q
\ 2df = dA + dB
(d) is the correct option. P f2 = m2N2
Rotational Motion P-S-53

N1 = m2N2 [... horizontal equilibrium]


81mwa 2 24
mg = N2 + m1N1 [... vertical equilibrium] LZ = cos q - Icwsin q
5
Solving the above equation we get
24 24 17ma 2 w 1134
mg = 81mwa
2
´ - = mwa 2
N2 = 5 5 10 50
1 + m1m2
21. (a, b)
\ (c) is the correct option. r
When m1 = 0 r = at 3iˆ + bt 2 ˆj
Taking torque about P we get r 10
r = t 3iˆ + 5t 2 ˆjm
l 3
mg ´ cos q = N1 ´ l sin q
2 r
r dr
v= = 10t 2iˆ + 10tjˆ ms –1
mg dt
N1 tan q =
2 r
r dv
\ (d) is correct a= = 20tiˆ + 10 ˆjms -2
dt
19. (d) Applying conservation of angular mumentum about
the axis At t = 1s
r 10
8w M 9R2 8w M 2 8w rt =1 = iˆ + 5 ˆjm ;
MR2 × w = MR2 × + × × + r × 3
9 8 25 9 8 9
r
4R vt =1 = 10iˆ + 10 ˆj ms –1
Þ r= r
5 pt =1 = iˆ + ˆj kgms –1
D is the correct option
r
at =1 = 20iˆ + 10 ˆj ms -2
20. (a, c) In DOAM, OM = l2 + a 2 = 2ha 2 + a 2 = 5a
The circumference of a circle of radius OM will be 2p(5a) iˆ ˆj kˆ
= 10pa. r r r
L = r ´ p = 10 5 0 = kˆ é 10 - 5ù = - 5 kˆ kgms -1
Sm vc r^ z 3 êë 3 úû 3
1 1 0
w
l r r
A F = ma = (2iˆ + ˆj ) N
q c
l a
q iˆ ˆj kˆ
O M
r r r 10 é10 ù -20 ˆ
For completing this circle once, the smaller disc will t = r ´F = 5 0 = kˆ ê - 10 ú = k N–m
3 ë3 û 3
10pa 2 1 0
have to take = 5 rounds.
2pa
Therefore the C.M. of the assembly rotates about z- E. Subjective Problems
axis with an angular speed of w/5.
The angular momentum about the C.M. of the system mv 2
1. T – mg cos q =
é1 2ù l O
q0
Lc = Icw = ê ma ú w 2
ë2 û mv q
\ T= + mg cos q ... (i)
l
é1 2ù 17ma 2w
+ ê ´ 4m ´ ( 2a ) ú ´ w = OM M
T
P
ë2 û 2 In DOPM, cos q0 = M' P'
l Reference mg sin q q
m ´ wa + 4m ´ 2wa 9wa Þ OM = l cos q0
level for P.E. mg mg cos q
Now vc = =
5m 5
OM '
In DOP'M', cos q =
ml + 4m ´ 2l 9l l
and r^ = =
5m 5 Þ OM' = l cosq
5m ´ 9wa 9l 24 OM' – OM = l (cos q - cos q0 )
L of C.M = ´ = 81mwa 2 ´
5 5 5 Loss in potential energy = Gain in kinetic energy
(ActivityP to P')
EBD_7036
P-S-54 Topic-wise Solved Papers - PHYSICS

1 2 3. Let s be the mass per unit area.


Þ mg l (cos q - cos q0 ) = mv Then the mass of the whole disc = s × p R2
2
Mass of the portion removed = s × p r2
Þ v2 = 2 g l (cos q - cos q0 ) ... (ii) R = 28 cm; r = 21 cm; OP = 7 cm
From (i) and (ii) Taking O as the origin R
m The position of c.m.
T = ´ 2 g l (cos q - cos q0 ) + mg cos q
l m1 x1 - m2 x2 r
\ T = 3mg cos q – 2 mg cos q0 x= O P
m1 - m2
Þ T = mg (3 cos q – 2 cos q0 )
From equation (i) it is clear that the tension is maximum s ´ p R 2 (0) - s ´ p r 2 ´ 7
=
when cos q = 1 i.e., q = 0° s p R2 - s p r 2
\ T = mg
- (21)2 ´ 7 21 ´ 21 ´ 7
2 = =- = -9 cm
mv (28) - (21) 2 2 7 ´ 49
Hence, Tmax = + mg ... (iii)
l This means that the c.m. lies at a distance of 9 cm from the
From eqn. (ii) origin towards left.
v2 = 2 g l (1 - cos q0 ) ... (iv) 4. C.M. of the system of two bodies in situation (i) in
From (iii) and (iv) x-coordinate
m M ´ 0 + mx1 mx1
Tmax = [2 g l (1 - cos q0 )] + mg xC = = ... (i)
l M +m M +m
\ Tmax = 3mg – 2mg cos q0 x
+Y
m r
1

=(R–r)
80 = 3 × 40 – 2 × 40 cos q0
R–r R –r
1 M v v'
Þ 80 cos q0 = 40 Þ cos q0 = Þ q0 = 30° r
2
2. Suppose mass m moves around a circular path of radius r. O
+Y Situation (1) +X x 2
Situation (2)
Let the string makes an angle q with the vertical. Resolving
tension T, we get C.M. of the system in situation (ii) in x-coordinate is
and, T sin q = mrw2 ... (i) M ´ x2 + m ´ x2
T cos q = mg ... (ii) l x'C = = x2 ... (ii)
M +m
q Since no external force is in x-direction
r w2 T
\ tan q = \ xC = x'C
g T cos q
q mx1 m( R - r )
r r \ x2 = =
From diagram, sin q = M +m M +m
l T sin q
Applying conservation of linear momentum,
Þ r = l sin q mg Initial Momentum = Final Momentum
0 = MV – mv
w2
\ tan q = l sin q \
MV
v= .... (iii)
g
m
Applying the concept of conservation of energy, we get
tan q.g g
w2 = w= Loss in P.E. of mass m = Gain in K.E. of mass M and Gain in
l sin q l cos q K.E. of mass m
1 2 1 2
1 g Þ mg (R – r) = MV + mv
Þ n= ... (iii) 2 2
2p l cos q
M 2V 2
From (ii), T cos q = mg. Þ 2mg (R – r) = MV2 + m [from (iii)]
For M to remain stationary, T = Mg m2
2 2
\ Mg cos q = mg Þ 2mg (R – r) = MV2 + M V
m m
Þ cos q = ... (iv) é M ù ém + M ù
M 2mg (R – r) = MV2 ê1 + ú = MV 2 ê
ë mû ë m úû
1 gM
From (iii) and (iv), n = 2m 2 g ( R - r ) 2g (R - r )
2p l m Þ =V2 Þ V=m
M (m + M ) M (m + M )
Rotational Motion P-S-55

5. The angular momentum is given by L = xpy – ypx


v
= m[xvy – yvx] \ w= where r is the radius of the sphere
r
v D
(x, y) are the coordinates of the particle after time t = 0 and
g
2 2
vx, vy are the components of velocities at that time. Also, I= mr 2.4 m Q 4.43 m/s
y
5
v0 v0 1m
For vx and vy
Ö2 Putting in equation (i) A B C
o v0
vx = v0 cos 45 =
2 45
O

x 1æ2 2
1 öv
v0 mg ( 2.4 - 1) = mv 2 + ç mr 2 ÷ 2
Ö2 2 2è5 ør
(The horizontal velocity does not change with time)
Applying v = u + at in the vertical direction to find vy
7v 2
or, g ´ 1.4 = Þ v = 4.43 m/s
( )æv ö
è gø
v
v y = v0 sin 45o - g ç 0 ÷ = v0 - g ´ v0 = 0 - v0 10
2 g 2 After point Q, the body takes a parabolic path.
For x and y The vertical motion parameters of parabolic motion will be
In horizontal direction x = vx ´ t uy = 0 Sy = 1m
ay = 9.8 m/s2 ty = ?
v0 v0 v2 1 2
\ x= ´ = 0 \ S = ut + at Þ 1 = 4.9 t 2y
2 g 2g 2
1 2 1
In vertical direction applying S = ut + at ty = = 0.45sec
2 4.9
v0 v0 1 v02 v02 v02 Applying this time in horizontal motion of parabolic path,
y= ´ - g = - BC = 4.43 × 0.45 = 2m
2 g 2 g2 2g 2 g
Note : During its flight as a projectile, the sphere continues
Putting the values in the above equation to rotate because of conservation of angular momentum.
év2 æv 7. Initial Kinetic Energy
ö æ v
2
v2 ö v ù
L = m ê 0 ´ ç 0 - v0 ÷ - ç 0 - 0 ÷ 0 ú 1 1 1
êë 2 g è 2 ø è 2 g 2g ø 2 ú
û = m1v12 + m2 v22 + MV 2
2 2 2
é v03 v3 v3 v3 ù 1 1
L=m ê - 0 - 0 + 0 ú = 0.08 ´ 102 + 0.08 ´ 62 + 0 = 5.44 J ... (i)
ëê 2 g 2 g 2 g 2 2 g ûú 2 2
M
mv03 é 1 1 ù - mv03 10 m/s
L= - L =
g êë 2 2 ú
2û 2 2g m1
x = 0.5m
ur r ur
Now, L = r ´ p x = 0.5m
Note : The direction of L is perpendicular to the plane of m2
6 m/s
motion and is directed away from the reader.
r
Applying law of conservation of linear momentum during
p collision
m1 × v1 + m2 × v2 = (M + m1 + m2) Vc
6. KEY CONCEPT : Applying law of conservation of energy where Vc is the velocity of centre of mass of the bar and
at point D and point A particles sticked on it after collision
P.E. at D = P.E. at Q + (K.E.)T + (K.E.)R where 0.08 × 10 + 0.08 × 6 = (0.16 + 0.08 + 0.08) Vc
(K.E.)T = Translational K.E. and (K.E.)R = Rotational K.E. Þ Vc = 4 m/s
\ Translational kinetic energy after collision
1 2 1 2
Þ mg (2.4) = mg (1) + mv + I w ...(i) 1
2 2 = (M + m1 + m2 )Vc2 = 2.56 J ... (ii)
Since the case is of rolling without slipping 2
Applying conservation of angular momentum of the bar
\ v = rw and two particle system about the centre of the bar.
EBD_7036
P-S-56 Topic-wise Solved Papers - PHYSICS
Since external torque is zero, the initial angular momentum Let at any instant of time t the insect be at a distance x from
is equal to final angular momentum. the centre of the rod and the rod has turned through an
Initial angular momentum angle q (= wt) w.r.t its original position.
= m1v1 × x – m2v2x
Instantaneous torque,
= 0.08 × 10 × 0.5 – 0.08 × 6 × 0.5 A
= 0.4 – 0.24 = 0.16 kg m2s–1 (In clockwise direction) dL d
Final angular momentum = Iw t= = ( I w) M
dt dt
é M l2 ù dI A O q=w t B
=ê + m1 x 2 + m2 x 2 ú w =w N
ëê 12 ûú dt x

é (0.16) ( 3)2 ù d é1 ù
+ 0.08 ´ (0.5)2 + (0.08) (0.5) 2 ú w =w ML2 + Mx 2 ú B

12 dt êë12 û Mg
ëê ûú
= 0.08 w dx
= 2 M wx ... (i)
\ 0.08 w = 0.16 Þ w = 2 rad/s ... (iii) dt
The rotational kinetic energy This torque is balanced by the torque due to weight of
1 2 1 insect.
= I w = ´ 0.08 ´ 2 2 = 0.16 J ... (iv)
2 2 t = Force × Perpendicular distance of force with axis of
The final kinetic energy rotation = Mg × (OM)
= Translational K.E. + Rotational K.E. = Mg (x cosq) ... (ii)
= 2.56 + 0.16 = 2.72 J From (i) and (ii)
The change in K.E. = Initial K.E. – Final K.E.
dx æ g ö
= 5.44 – 2.72 = 2.72 J 2M w x = Mg (x cosq) Þ dx = ç ÷ cos wt dt
8. (a) Let us consider the system of homogeneous rod and dt è 2w ø
insect and apply conservation of angular momentum during On integration, taking limits
collision about the point O. L/2 p / 2w
g
M ò L / 4 dx = 2w ò 0 cos wt dt

v
L
when x= , wt = 0
A B 4
O
L/4 L/2 g p / 2w
L/2 [ x] L / 4 = 2
[sin wt ] 0
Angular momentum of the system before collision = angular
2w
momentum of the system after collision. L p
when x= , wt =
L 2 2
Mv × = Iw
4
æ L Lö g é p ù
Where I is the moment of inertia of the system just after Þ çè - ÷ø = êësin 2 - sin 0úû
collision and w is the angular velocity just after collision. 2 4 2w 2

L é æ Lö ù
2 L g 2g
1 Þ = Þ w=
Þ Mv = ê M ç ÷ + ML2 ú w 4 2w 2 L
4 êë è 4 ø 12 úû
12 v 12 v 2g 7
L ML2 é1 1ù ML2 é 3 + 4 ù But w = Þ = Þ v= 2 gL
Þ Mv ´ = +
ê 4 3ú w = 7 L 7 L L 12
4 4 ë û 4 êë 12 úû
7
Þ v= 2 ´ 10 ´ 1.8 = 3.5 ms–1
ML2 7 12 v 12
= ´ ´w Þ w= .
4 12 7 L 9. (i) Initially, the rod stands vertical. A straight disturbance
(b) Note : Initially the torque due to mass OB of the rod makes the rod to rotate. While rotating, the force acting on
(acting in clockwise direction) was balanced by the torque the rod are its weight and normal reaction. These forces are
due to mass OA of the rod (acting in anticlockwise direction). vertical forces and cannot create a horizontal motion.
But after collision there is an extra mass M of the insect Therefore the centre of mass of the rod does not move
which creates a torque in the clockwise direction, which horizontally. The center of mass moves vertically
tends to create angular acceleration in the rod. But the same downwards. Thus the path of the center of mass is a straight
is compensated by the movement of insect towards B due line.
to which moment of inertia I of the system increases.
Rotational Motion P-S-57

(ii) Trajectory of an arbitrary point of the rod


2T æ 2m ö
Consider an arbitrary point P on the rod located at (x, y) and mg sin q – T = m × Þ mg sin q = ç1 + ÷T
M è Mø
at a distance r from the end B. Let q be the angle of
inclination of the rod with the horizontal at this position. (mg sin q) ´ M 0.5 ´ 9.8 ´ sin 30° ´ 2
\ T= = = 1.63 N
y M + 2m 2 + 2 ´ 0.5
In D BNP, sin q = ... (i) (ii) The total kinetic energy of the drum and the block at
r
A the instant when the drum is having angular velocity
Y
10 rads–1 gets converted into the potential energy of the
x + BN x + r cos q block.
cos q = = C
L/2 L/2 P (x,y)
y r
q
x O N B X
Þ cos q = ... (ii) x
S
L
-r h=S sinq
2 q

From (i) and (ii) sin 2 q + cos 2 q = 1 q mg

[(K.E.)Rotational]drum + {(K.E.)Translational]block = mgh


y2 x2
Þ + =1 1 2 1 2
r2 æL ö
2
I w + mv = mgS sin q
çè - r ÷ø 2 2
2
1 2 1
This is equation of an ellipse. I w + m( Rw )2 = mgS sin q [Q v = Rw ]
10. (i) The drum is given an initial velocity such that the block 2 2
X starts moving up the plane. 1 1
Þ MR 2 w 2 + mR 2 w 2 = mgS sin q
2 2
Y
T R
w 1 R 2 w 2 ( M + m)
M Þ =S
m 2 mg sin q
T
X
1 0.2 ´ 0.2 ´ 10 ´ 10(2 + 0.5)
q Þ S= ´ = 1.22 m
2 0.5 ´ 9.8 ´ sin 30°
mg sinq mg mg cosq
11. During collision, the torque of the system about P will be
q=30o
zero because the only force acting on the system is through
P (namely weight of rods/mass m/reaction at P)
As the time passes, the velocity of the block decreases. The P
linear retardation a, of the block X is given by Given : l = 0.6 m
mA = 0.01 kg
mg sin q - T = ma ... (i) mB = 0.02 kg mA A
The linear retardation of the block and the angular m = 0.05 kg
acceleration of the drum (a) are related as 2l
dL
a = Ra ... (ii) Since, t = and t = 0
where R is the radius of the drum. dt
mB B
The retarding torque of the drum is due to tension T in the Þ L is constant. v
string. m
t= T× R
But t = Ia. where I = M.I. of drum about its axis of rotation. Angular momentum before collision = mv × 2 l ... (i)
Angular momentum after collision = Iw ... (ii)
1 é 1 2ù
\ T×R=
2
MR 2 a ... (iii) êëQ I = 2 MR úû Where I is the moment of inertia of the system after collision
about P and w is the angular velocity of the system.
1 a 2T M.I. about P : I1 = M.I. of mass m
From (ii), TR = MR 2 Þ a = I2 = M.I. of rod mA
2 R M
I3 = M.I. of rod mB
Substituting this value in (i)
I = I1 + I2 + I3
EBD_7036
P-S-58 Topic-wise Solved Papers - PHYSICS
From (ii), (iii) and (iv), we get
é ì æ l 2 ö æ l ö 2 üï ìï æ l2 ö æ l ö 2 üï ù
2 ï
= ê m(2l) + í mA ç ÷ + ç ÷ ý + í mB ç ÷ + ç + l÷ ý ú 1 2 1 1 v2
êë 12 è 2 ø ï ï è 12 ø è 2 ø ï ú mvc + ´ mR 2 ´ c2
îï è ø þ î þû mgh =
2 2 2 R
é æ l 2 l2 ö æ l 2 9l 2 ö ù 1 2 1 2 3 2 4 gh
= ê 4 ml 2
+ m Aç + ÷ + m Bç + ÷ú Þ gh = vc + vc = vc Þ vc2 =
êë è 12 4 ø è 12 4 ø úû 2 4 4 3
R-h
é 1 7 ù In DO¢MP, cos q =
= ê 4ml2 + m A l2 + mB l 2 ú = 0.09 kg m2 R
ë 3 3 û Þ h = R (1 – cos q )
From (i) and (ii)
4g
Iw = mv × 2 l \ vc2 = R (1 - cos q ) ... (v)
3
mv ´ 2l 0.05 ´ v ´ 2 ´ 0.6 From (i) and (v), we get
Þ w= = = 0.67 v
I 0.09 4 gr
Applying conservation of mechanical energy after collision. cos q = (1 - cos q)
3Rg
(Using the concept of mass)
Loss of K.E. = Gain in P.E. 4
Þ 3 cos q = 4 – 4 cos q Þ cos q =
1 2 æ lö æ 3l ö 7
I w = mg (2l) + m A ç ÷ g + mB g ç ÷ (b) From (v) speed of C.M. of cylinder before leaving
2 è 2ø è 2ø
contact with edge.
1
Þ ´ 0.09 ´ (0.67 v) 2 4 gR æ 4 ö 4 gR 4 gR
2 vc2 = çè1 - ÷ø = Þ vc =
3 7 7 7
é 1 3ù (c) Before the cylinder's c.m. reaches the horizontal line of
= ê 0.05 ´ 2 + 0.01 ´ + 0.02 ´ ú ´ 9.8 ´ 0.6
ë 2 2û the edge, it leaves contact with the edge as
Þ v = 6.3 m/s 4
12. (a) Let the original position of centre of mass of the q = cos -1 = 55.15°
7
cylinder be O. While rolling down off the edge, let the cylinder Therefore the rotational K.E., which the cylinder gains at
be at such a position that its centre of mass is at a position the time of leaving contact with the edge remains the same
O'. Let ÐNPO be q. As the cylinder is rolling, the c.m. rotates in its further motion. Thereafter the cylinder gains
in a circular path. The centripetal force required for the translational K.E.
circular motion is given by the equation. Again applying energy conservation from O to the point
N where c.m. is in horizontal line with edge
mvc2 O
mg cos q – N = 1 2 1
R R h w mgR = I w + m(v 'c ) 2
M O' 2 2
Where N is the normal reaction R–h gc
os
q
q
vc
m
and m is mass of cylinder. 2
2 æ 4g ö
q R
mg mgsin q 1 1 1
The condition for the cylinder P mgR = ´ mR ´ ç ÷ + m(v 'c ) 2
leaving the edge is N = 0 2 2 è 7R ø 2

mvc2 vc2 vc 4 gR / 7
mg cos q = Þ cos q = ... (i) Q w= =
R Rg R R
Applying energy conservation from O to O'. mgR 6mgR
Loss of potential energy of cylinder Þ mgR – = Translational K.E. =
7 7
= Gain in translational K.E. + Gain in rotational K.E.
1 2 mgR
1 2 1 2 Also, Rotational K.E. = Iw =
mgh = mvc + I w ... (ii) 2 7
2 2
Where I is the moment of inertia of the cylinder about O', its Translational K .E.
axis of rotation, w is the angular speed, Vc is the velocity of \ =6
Rotational K .E.
center of mass.
Also for rolling, vc = wR 13. KEY CONCEPT : The concept of center of mass can be
applied in this problem.
vc When small sphere M changes its position to other extreme
Þ w= ... (iii)
R position, there is no external force in the horizontal direction.
Therefore the x-coordinate of c.m. will not change.
1
I= MR 2 ... (iv) [xc.m.]initial = [xc.m.]final
2
Rotational Motion P-S-59

Y ma 2 ´ 9
Þ 3f = ma Þ f = = = 6N (In + X direction)
6R 3 3
4m
ur
O m f = (6 $i) N
L'-5 R (L,O) (L',O) R X
L (ii) The position vector of point M, taking O as the origin
L'+5R uur
rm = - 0.1$j - 0.1k$ and position vector of point N
uur
Thin line of sphere represents initial state, dotted line of rN = 0.1$j - 0.1 k$
sphere represents final state.
The torque due to friction on disc 1 about O
From (i) uur uuur ur
(xc.m.)initial = (xc.m.)final t = r ´ f = ( -0.1$j - 0.1 k$ ) ´ (6 i$)
1 M
M1 x1 + M 2 x2 M1 x '1 + M 2 x '2
Þ = = 0.6 (k$ - $j ) N - m
M1 + M 2 M1 + M 2 The torque due to friction on disc 2 about O
uur uur ur
Þ
4m ´ L + m ´ (5R + L) 4m ´ L '+ m ´ ( L '- 5R )
= t = r ´ f = (+ 0.1 $j - 0.1 k$ ) ´ (6 $i )
2 N
4m + m 4m + m
Þ 5L + 5R = 5L'– 5R = 0.6 (- $j - k$ ) N - m
Þ 5L + 10R = 5L' Þ L + 2R = L' The magnitude of torque on each disc
Since, the individual center of mass of the two spheres
has a y co-ordinate zero in its initial state and its final state | t1 | = | t 2 | = 0.6 2 N - m
therefore the y-coordinate of c.m. of the two sphere system 15. (a)
will remain zero. Z
Therefore the coordinate of c.m. of bigger sphere is (L Y
+ 2R, 0). X
14. (i) The observer, let us suppose, is on the accelerated
frame. Therefore a pseudo force ma is applied individually
F
on each disc on the centre of mass. The frictional force is 30
°
30°
acting in the + X direction which is producing an angular 60°
acceleration a.
The torque acting on the Truck Resolving the force F acting on the wedge
disc is 1 ma 2 ma Fx = F cos 30°; Fy = F sin 30°
a'
t = Ia = f × R a a Note : The collision is elastic and since the sphere is fixed,
the wedge will return back with the same velocity
Ia P O
(in magnitude).
Þ f= ... (i) R R
N
R M The force responsible to change the velocity of the wedge
Let a' is the acceleration of f f in X-direction is Fx.
c.m. of the disc as seen by Fx × Dt = mv – ( – mv)
the observer. Since the +Z r (Impulse) = (Change in momentum)
observe
case is of pure rolling and
from the perspective of the +Y 2 mv 2 mv 4 mv
a = 9 m/s2 +X \ Fx = Þ F cos 30° = Þ F=
observer Dt Dt 3 Dt
a' = aR ... (ii)
In vector terms
Þ From (i) and (ii) ur
Ia ' F = F $i + F (- k$ ) = F cos 30°$i + F sin 30°(- k$ )
x y
f= 2 ... (iii)
R 3$ 1
Applying Newton's law for motion in X-direction =F× i + F ´ ( - k$ )
ma – f = ma' 2 2
ur F 2mv
æ fö
Þ a' = ç a - ÷ ... (iv) Þ F = ( 3 $i - k$ ) = ( 3 $i - k$ )
è mø 2 3Dt
Also moment of inertia Taking equilibrium of force in Z-direction (acting on wedge)
we get
1
I= mR 2 ... (v) Fy + mg = N
2
From (iii), (iv) and (v) F 2mv _
Þ N= + mg = + mg
æ fö
2 3Dt
mR 2 ç a - ÷
1 è mø æ 2mv ö
f= 2
Þ 2f = ma – f N= ç + mg ÷ k$
2 R è 3D t ø +
EBD_7036
P-S-60 Topic-wise Solved Papers - PHYSICS
(b) Taking torques on wedge about the c.m. of the wedge.
f
F × h – Torque due to N + mg × 0 = 0
4mv a1 a
Þ Torque due to N = F × h = ´h a
3D t M1g F f mg N
16. KEY CONCEPT : During the fall, the disc-mass system gains
rotational kinetic energy. This is at the expense of potential P f'
energy.
A m
Direction of f ' : A force f is acting on the cylinder. This force
is trying to move the point of contact P towards right by an
R acceleration

C
f
R/4 acm = acting towards right.
P Q
M1
R/4
C' At the same time, the force f is trying to rotate the cylinder
about its centre of mass.
R
f× R= I× a
v
Reference f ´R f ´R 2f
level for P.E Þ a= = = in clockwise direction.
I 1 M1 R
Applying energy conservation M1R2
2
Total energy initially = total energy finally
æ 2R ö æ 2R ö 1 2 f 2f f
mg ç 2 R + ÷ø + mg çè R + ÷ø = mgR + I w \ acm + aR = - ´R=- , i.e., towards left.
è 4 4 2 M1 M1 R M1
Where I = M.I. of disc-mass system about PQ Therefore, the point of contact of the cylinder with the
ground move towards left. Hence friction force acts towards
10 R 6R 1 1 2
mg × + mg = mgR + I w 2 Þ 3mgR = Iw right on the cylinder.
4 4 2 2 Note : You can assume any direction of friction at the point
6mgR of contact and solve the problem. If the value of friction
Þ w= ... (i) comes out to be positive, our assumed direction is correct
I
otherwise the direction of friction is opposite. The above
(I)PQ = (Idisc)PQ + (Imass)PQ activity is done so that if only the direction of friction is
é mR 2 asked, an approach may be developed.
æ Rö ù
2 2
æ 5R ö Applying Newton's law on plank, we get
=ê + M ç ÷ ú + mç ÷
êë 4 è 4ø ú è 4ø F – f = m2a2 ... (i)
û
Also, a2 = 2a1 ... (ii)
1 Because a2 is the acceleration of topmost point of cylinder
[Q M.I. of disc about diameter = MR 2 ]
4 and there is no slipping.
Applying Newton's law on cylinder
mR 2 [4 + 1 + 25] 15mR 2 M1a1 = f + f ' ... (iii)
= = ... (ii)
16 8 The torque equation for the cylinder is
From (i) and (ii)
1 æa ö
f × R – f ' × R = Ia = M1 R 2 ´ ç 1 ÷
6mgR ´ 8 16 g 2 è Rø
w= 2
=
15mR 5R
1
Let v be the velocity of mass m at the lowest point of rotation [Q I = M1R 2 and Ra = a1]
2
æ Rö 16 g 5R 1 1
v = wç R + ÷ \ v = ´ = 5 gR \ (f – f ') R = M1Ra1 Þ f + f ' = M1a1 ... (4)
è 4ø 5R 4 2 2
17. The man applies a force F in the horizontal direction on the Solving equation (iii) and (iv), we get
plank as shown. Therefore the point of contact of the plank 3
with the cylinder will try to move towards right. Therefore f = M 1a1 ... (5)
4
the friction force F will act towards left on the plank. To
1
each and every action there is equal and opposite reaction. and f ' = M1a1 ... (6)
Therefore a frictional force f will act on the top of the cylinder 4
From (i) and (iii)
towards right.
Rotational Motion P-S-61

3 19. (a) The mass B is moving Y


F – f = 2m2a1 Þ F – M 1a1 = 2m2a1 in a circular path centred at A
4 o X
A. The centripetal force 60
F'
4F 8F (m l w2 ) required for this F'
\ a1 = \ a2 =
3M1 + 8m2 3M1 + 8m2 circular motion is provided by
From (v) and (vi) F '. Therefore a force F ' acts Fnet
on A (the hinge) which is l l
3 4F 3FM1
f= M1 ´ = equal to m l w2. The same is
4 3M1 + 8m2 3M1 + 8m2 the case for mass C. F' F'
Therefore the net force on the
1 FM 1
And f ' = M1 ´ a1 = hinge is B l C
4 3M1 + 8m2
18. Ic = 1.2 kg -m2 Fnet = F '2 + F '2 + 2 F ' F 'cos 60°
A 1
Fnet = 2 F '2 + 2 F '2 ´ = 3F ' = 3 mlw 2
2

m Q (b) The force F acting on B will provide a torque to the


0.5 y c.m system. This torque is

P
B l 3
F× = Ia
m = 30kg 2
3l
F× = (2ml 2 ) a l Ö3
2 c.m 2
Let y be the distance of c.m. from line AB.
Applying parallel axis theorem of M.I. we get 3 æ F ö
Þ a= ´
M.I. of laminar sheet about AB 4 çè m l ÷ø
F
IAB = Ic.m. + my2 The total force acting on the system along x-direction is
IAB = 12 + 30y2 ... (i) F + (Fnet)x
The angular velocity of the laminar sheet will change after This force is responsible for giving an acceleration ax to the
every impact because of impulse. system.
Impulse = Change in linear momentum
6 = 30 (Vf – Vi) Therefore,
6 = 30 × y (wf – wi) ... (ii) F + (Fnet)x = 3m (ax) c.m.
Also, change in angular momentum = Moment of Impulse
\ IABwf – IABwi = Impulse × distance F æ 3 F l Fö
IAB (wf – wi) = 6 ×0.5 = 3 = 3m çèQ a x = ar = 4 ml ´ = ÷
4m 3 4ø
3 3
\ wf = + wi = + ( -1) ... (iii) 3F F
I AB 1.2 + 30 y 2 = \ (Fnet)x = -
4 4
Note : Minus sign with wi because the direction of laminar
plate towards the obstacle is taken as – ve (assumption). (Fnet)y remains the same as before = 3 mlw 2 .
From (ii) and (iii) ur
r dL
é 3 ù 20. We know that t =
6 = 30 × y ê - 1 + 1ú dt
2 r ur
ëê1.2 + 30 y ûú Þ t ´ dt = d L
r r
é 3 ù When angular impulse ( t ´ dt ) is zero, the angular
1 = 5y ê 2ú momentum is constant. In this case for the wooden
ëê1.2 + 30 y ûú log-bullet system, the angular impulse about O is constant.
\ 1.2 + 30 y2 = 5y [+ 3] = 15y Therefore,
\ 30y2 – 15y – 1.2 = 0 [angular momentum of the system]initial
On solving, we get y = 0.1 or 0.4 = [angular momentum of the system]final
\ wf = 1 rad/s if we put y = 0.1 in eq. (ii) Þ mv × L = I0 × w ... (i)
And wf = 0.5 rad/s if we put y = 0.4 in eq. (ii) where I0 is the moment of inertia of the wooden log-bullet
(Not valid as per sign convention) system after collision about O
Now, since the lamina sheet comes back with same angular I0 = Iwooden log + Ibullet
speed as that of incident angular speed, the sheet will swing 1
in between P and Q infinitely. = ML2 + ML2 ... (ii)
3
From (i) and (ii)
EBD_7036
P-S-62 Topic-wise Solved Papers - PHYSICS

mv ´ L L
w= Þ Mg × cos q + f ´ L sin q = NL cos q
é1 2 2ù 2
êë 3 ML + mL úû MgL
NL cos q - cos q Mg
mv 3mv Þ f=
2 = N cot q - cot q
Þ w= = L sin q 2
é ML ù (M + 3m) L
êë 3 + mL úû éæ 2N + m ö Mg ù
Þ f = êç ÷ g- cot q
21. Applying Fnet = ma in X-direction ëè 2 ø 2 úû
Y
mg sin q – f = ma ... (i) é gù
The torque about O will be Þ f = ê( M + m) ú cot q
N X ë 2û
t=f×R
= Ia ... (ii)
O
23. KEY CONCEPT Since the plate is held horizontal therefore
R f
As the case is of rolling
q
net torque acting on the plate is zero.
\ a = aR mg sinq mg cosq
m
mg

a F
Þa = ... (iii) 3b/4
R
q
Ia
From (ii) and (iii), f =
R2 a
Substituting this value in (i), we get
Ia
mg sin q – = ma
R2 b/2
Mg
mg sin q mg sin q 2 m'
Þ a= = = g sin q
I 1 mR 2 3 b 3b
m+ 2 m+ Þ Mg × =F´ ... (i)
R 2 R 2 2 4
dp b
é 1 2 ù F= n (Area) = n × (2mv) × a × ... (ii)
êëQ I = 2 mR for solid cylinder úû dt 2
From (i) and (ii)
22. The various forces acting on the ladders are shown in the
figure. b b 3b
Mg × = n × (2mv) × a × ×
Since the system is in equilibrium, therefore 2 2 4
SFy = 0
3´ 2
Þ Mg + mg + Mg = N + N Þ 3 × 10 = 100 × 2 × 0.01 × v × 1 ×
Y
4
Þ v = 10 m/s
P
X
F. Match the Following
mg
1. A ® (p,t) ; B ® (q,s,t) ; C ® (p,r,t) D ® (q, p)
N N
Mg Mg N
A f f B f
v
(2M + m) g
Þ N= ... (i) q
2
Considering the rotational equilibrium of one ladder as mgsinq
shown in figure. Calculating torques about P mgcosq
Q mg
O
q
P q

m2
mg As the velocity is constant
f = mg sin q ..... (i)
l/2 But f = mN = mmg cos q ..... (ii)
N Mg
From (i) and (ii)
q
A f M mmg cos q = mg sin q Þ m = tan q
Mg × PQ + f × PM = N × OP
Rotational Motion P-S-63

The force by X on Y is the resultant of f and N and is (t) Sphere Y is moving with terminal velocity. Therefore,
equal to the net force on Y is zero i.e.
B Fv
Mg = B + Fv
f 2 + N 2 = m 2 N 2 + N 2 = m 2 + 1N
where B = buoyant force and Fv =
= ( tan 2 q + 1 ) mg cos q = sec qmg cos q = mg viscous force.
B + Fv are exerted by X on Y.
= weight of Y. Therefore, option (a) is correct.
Therefore statement (a) is correct. The gravitational potential energy of X
Now, due to the presence of frictional force between Y Mg
is continously increasing because as Y
and X, the mechanical energy of the system ( X + Y ) moves down, the centre of mass of X
decreases continously as Y slides down. moves up.
Therefore statement (c) is correct. Option [b] is correct.
(q) As the lift moves up, X also The mechanical energy of the system (X + Y) is
moves up and therefore the P continously decreasing to overcome the viscous
gravitational energy of X is forces.
continously increasing. Option (c) is correct.
Z
option (b) is correct. V
The torque of the weight of Y G. Comprehension Based Questions
about P is zero as the Y 1. (c) For disc A
perpendicular distance of the Mg 1 2 1
line of action of force from the kx1 = I (2w )2
point P is zero. X 2 2
2w
Option (d) is correct. Þ kx12 = 2 I w 2 ... (i)
The force exerted by X on Y will be equal to For disc B A
Mg + Mg = 2mg where Mg is wt. of Y and Mg is the 1 2 1
kx2 = ´ 2 I w 2
force on Y due to Z. 2 2
Option (a) is incorrect. Þ kx22 = I w2 ... (ii) w
(r)
On dividing (i) and (ii), we get B
P
Y
k x12 2I w 2
x
= Þ 1 = 2
Xm g kx22 Iw 2 x2
0 v
2. (a) When disc B is brought in contact with disc A
Let w' be the final angular velocity of both the disc
rotating together. Apply conservation of angular
In this case the force exerted by X on Y is same as the momentum for the two disc system.
force exerted by Y on X. The force on X due to Y is 4
I (2w) + 2I (w) = (I + 2I) w' Þ w' = w
R = ( Mg ) 2 + (m0 + M ) g ]2 ¹ Mg 3
Torque on disc A
Mg 4
I ´ w - I ´ 2w
DLA L f - Li 3 -2 I w
m0g tA = = = =
t t t 3t
R Note : The negative sign represents that the torque
Mg creates angular retardation.
Therefore, option (a) is incorrect. 3. (b) Loss in kinetic energy = (K.E.)initial – (K.E.)final
The mechanical energy of the system (X + Y) is
ù é1 æ4 ö ù
2
é1 1
continously decreasing as the system is coming down = ê I (2w )2 + (2I ) w 2 ú - ê ( I + 2 I ) ç w ÷ ú
and its potential energy is decreasing, the kinetic ë2 2 û ëê 2 è3 ø ú
û
energy remaining the same.
Therefore, option (c) is correct and (b) is incorrect. 8 I w2
= 3I w 2 - I w 2 =
The torque of the weight of Y about P is not zero. 3 3 y
(s) The force on Y by X is equal to the wt. of liquid 4. (d) When the disc is at a
displaced which cannot be equal to Mg as the density distance x from the mean kx x
of Y is greater than density of X (As Y is sinking) position (equilibrium x
Therefore, option (a) is in correct. position), the forces
The gravitational potential energy of X increases acting on the disc are
continously because as Y moves down, the centre of shown in the figure C R
mass of X moves up. \ –2kx + f = –Mac ...(1) Mean
where ac = acceleration of center Position kx f
Therfore option (b) is correct.
EBD_7036
P-S-64 Topic-wise Solved Papers - PHYSICS
of mass. Also the torque acting Therefore the statement 1 is false.
on the disc about its center of Statement 2 : The linear momentum of an isolated
mass C is t = f × R = I × ac system remains constant. This statement is true.
1 2. (d) The acceleration of a body rolling down an inclined
Ia
MR 2 a plane is given by
2 ´ c
\f= = g sin q
R R R a=
I
1 1+
[Q I= MR2 and ac = Rac for rolling without slipping] MR 2
2
1
For hollow cylinder I MR 2
\f= Ma c ...(ii) = =1
2 MR 2 MR 2
From (i) & (ii)
1
1 MR 2
I 1
–2kx + Mac= – Mac
2
For solid cylinder =2 =
2 2 2
MR MR
3 4kx Þ Acceleration of solid cylinder is more than hollow
Þ Mac= 2 kx Þ Mac =
2 3 cylinder and therefore solid cylinder will reach the
Þ Net external force acting on the disc when its centre of bottom of the inclined plane first.
mass is at displacement x with respect to the equilibrium \ Statement -1 is false
• Statement - 2
4kx In the case of rolling there will be no heat losses.
position = directed towards the equilibrium.
3 Therefore total mechanical energy remains conserved.
5. (d) As derived in ans 4. The potential energy therefore gets converted into
4k kinetic energy. In both the cases since the initial
|Fnet| = x potential energy is same, the final kinetic energy will
3
For S.H.M. |Fnet| = Mw2x also be same. Therefore statement -2 is correct.

4k 4k I. Integer Value Correct Type


\Mw2 = Þ w= ...(iii)
3 3M 1. 6
6. (c) From (i) & (ii) Let the center of mass of the binary star system be at the
2k origin. Then
Þ –2kx + f = –2 f Þ f= ×x A B
3 x dx
We see that the frictional force depends on x. As x 2.2 M s c.m 11 M s
increases, f increases. Also, the frictional force is origin
maximum at x = A where A is the amplitude of S.H.M. 2.2M s (- x) + 11M s (d - x )
Therefore the maximum frictional force 0=
2.2 M s + 11M s
2k
fmax= ×A 5d
3 Þ 0 = 2.2 Ms (– x) + 11Ms (d – x) Þ x =
The force should be utmost equal to the limiting friction 6
(mMg) for rolling without slipping. For a binary star system, angular speed w about the centre
of mass is same for both the stars.
2k 2 2
\ mMg = ×A ....(iv) æ 5d ö ædö
3 2.2 M s ç ÷ w + 11 M s ç ÷ ´ w
For S.H.M. Velocity amplitude = Aw \ Vo = Aw L è 6ø è 6ø
\ Total = 2
=6
3μMg LB ædö
\ Vo = w from (iv) 11 M s ç ÷ ´ w
2k è 6ø
2. 4
3μMg 4k Under the influence of the force of stick (2N), the point of
\ Vo = ´ from (iii)
2k 3M contact O of the ring with ground tends to slide. But the
frictional force f2 does not allow this and creates a torque
3M which starts rolling the ring. A friction force f1 also acts
\ Vo = μg
k between the ring & the stick.
7. (a) Axis of rotation is parallel to z-axis. Applying Fnet = ma in the horizontal direction. We get
8. (d) Since the body is rigid, w is same for any point of the
body.
C
H. Assertion & Reason Type Questions R 2N
1. (d) Statement 1 : For velocity of centre of mass to remain R f1
constant the net force acting on a body must be zero. f2
O
Rotational Motion P-S-65

2 – f2 = 2 × 0.3 \ f2 = 1.4 N
Applying t = Ia about C we get 1
´ 50´ 0.4 ´ 0.4´10
a = 2 40
(f2 – f1) R = Ia = I [Q For rolling a = Ra] 1 = = 8 rad / s
R ´ 50´ 0.4´ 0.4 + 2[2´ 6.25´ 0.2´ 0.2] 4 + 1
2
0.3 6. (4) By conservation of angular momentum
\ [1.4 – m × 2] × 0.5 = 2 × (0.5)2 × [Q I = MR2] 2 (mvr) = Iw
0.5
\ m = 0.4 1
2 × 0.05 × 9 × 0.25 = × 0.45 × (0.5)2 × w
P 2
Given m = \ P=4 \ w = 4 rad s–1
10
3. 9 é 1ù
Let the four spheres be A, B, C, & D 7. (2) 3 êF ´ r ´ ú = I a
ë 2û
IXY = I1 + IB + IC + ID = 2 IA + 2IB
1 1
é2 2 2ù é2 2ù 3 ´ 0.5 ´ 0.5 ´ = ´ 1.5 ´ 0.5 ´ 0.5 ´ a
= 2 ê MR + Ma ú + 2 ê MR ú 2 2
ë5 û ë5 û Þ a = 2 rad s–1
X D w = w0 + at Þ w = 0 + 2 × 1 = 2 rad s–1
C
1 2 1 2
8. (7) Total kinetic energy of a rolling disc = mv + Iw
2 2
4 cm
1 æ 1 mR 2 ö æç v ö
2
1 ÷
a = 4 22 = mv2 + ç ÷ç 2 ÷
2 2 è2 øè R ø
A B
Y
3 2
K.E = mv
é8 2 4
2 2ù
= 4 ´ MR 2 + 2Ma 2 = M ê R + 2(a) ú For surface AB
5 ë 5 û k.Ei + loss in gravitational potential energy = K.Ef
é æ 2 ù 3 3
8 5ö ú ´10-4 m (3)2 + mg(30) = m VB2 ...(i)
= 0.5 ê ´ ç ÷ + 2 ´ 8 4 4
ê 5 çè 2 ÷ø ú
ë û For surface CD
= 0.5 [ 2 + 16] ´ 10 -4 = 9 ´ 10-4 3 3
m (v2)2 + mg (27) = m VD2 ...(ii)
\ N=9 4 4
4. 3 Let s be the surface mass density. Then Given VB = VD. Therefore from (i) and (ii)
1 3 3
2
IO = s[p(2R) ] ´ (2R) -
2 m (3)2 + mg × 30 = m (v2)2 + mg × 27
2 4 4
\ V2 = 7
é1 2 2 2 2ù
êë 2 (s p R ) + s (p R ) ´ R úû R
9. (6) I = ò (dm)r 2 R
13 0
= psR4 R
2
IP = 8 p s R4 + sp(2R)2 × (2R)2 \ I = ò r ´ 4 pr 2 dr ´ r 2 r dr
0
é1
( (2R) + R ) úûú

2 2 2 2 2 R
ê s (pR )R + s(pR )
ëê 2 \ I = 4 p ò r r 4 dr
0
= 24 p s R4 – 5.5 s p R4 = 18.5 p s R4
R R
r 4 pK
IP 18.5 psR 37 4
\ I A = 4p ò k ´ r 4 dr = òr
5
dr
\ = = »3 R R
IO 13 4 13 0 0
psR
2 4pK æ R6 ö R5
5. 8 Applying conservation of angular momentum = ç ÷ = 4pK
R ç 6 ÷ 6
I1w1 = I2w2 è ø
R 5 10 5
ærö
1 I B = 4p ò K ç ÷ r 4 dr = 4pK ´ R = 4pK R
MR 2 ´ ω1 èRø
I1ω1 2 0 R5 10 10
\ ω2 = =
I2 ì1 2 2 ü IB 6
í MR + 2[2mr ]ý \ = Þn=6
î2 þ I A 10
EBD_7036
P-S-66 Topic-wise Solved Papers - PHYSICS

Section-B JEE Main/ AIEEE


1. (c) When two small spheres of mass m are attached gently,
the external torque, about the axis of rotation, is zero mass( M )
8. (d) We know that density (d ) =
and therefore the angular momentum about the axis of volume(V )
rotation is constant.
\ M = d ´ V = d ´ ( pR 2 ´ t ) .
I
\ I1w1 = I 2 w 2 Þ w 2 = 1 w1
I2 1
The moment of inertia of a disc is given by I = MR 2
1 2 1 2 2 2
Here I1 = MR and I 2 = MR + 2mR
2 2 1 pd
\ I= ( d ´ pR 2 ´ t ) R 2 = t ´ R4
1 2 2
MR 2
2 M
\ w2 = ´ w1 = w1 I X t X RX4 t ´ R4 1
1 M + 4m \ = = =
MR 2 + 2mR 2 t
2 IY tY RY4 ´ (4 R )4
64
2. (b) For negotiating a circular curve on a levelled road, the 4
maximum velocity of the car is vmax = mrg 1 L
Here m = 0.6, r = 150 m, g = 9.8 9. (a) K .E . I w 2 , but L = Iw Þ I =
2 w
\ vmax = 0.6´150´9.8 ; 30m / s
3. (b) The velocity of efflux is given 1L 1
\ K .E. = ´ w 2 = Lw
v = 2gh 2w 2
Where h is the height of the free surface of liquid from K .E L´w K .E L´w L
the hole \ = Þ = \ L' =
K .E ' L ' ´ w ' K .E L '´ 2w 4
\ v = 2 ´10 ´ 20 = 20 m / s
2
4. (c) The velocity of centre of mass of two particle system is
r r r
given by 10. (d) We know that t = r ´ F
m m r
m v + m2 v2 The angle between t
vc = 1 1 r
m1 + m2 2v v and r is 90° and the t r
r
m(2v ) + m(-v ) v angle between t and
= = r
m+m 2 F is also 90°. We also
5. (d) This is a case of sliding (the plane being frictionless) know that the dot F
and therefore the acceleration of all the bodies is same product of two vectors
(g sin q). which have an angle of
6. (a) M. I of a circular wire about an axis nn' passing through 90° between them is
the centre of the circle and perpendicular to the plane
zero. Therefore (d) is the
of the circle = MR2
correct option.
Z
n Y
11. (b) Angular momentum will remain the same since external
torque is zero.
12. (a) The moment of inertia of solid sphere A about its diameter
2
MR 2 .
X
IA =
5
n'
The moment of inertia of a hollow sphere B about its
As shown in the figure, X-axis and Y-axis lie in the
plane of the ring . Then by perpendicular axis theorem 2
diameter I B = MR 2 . \ I A < IB
IX + IY = IZ 3
Þ 2 IX = MR2 [Q IX = IY (by symmetry) and IZ = MR2] 13. (a) Does not shift as no external force acts. The centre of
1 2 mass of the system continues its original path. It is only
\ IX = MR the internal forces which comes into play while breaking.
2
7. (d) Angular momentum (L) 14. (c) The disc may be assumed as combination of two semi
= (linear momentum) × (perpendicular distance of circular parts.
the line of action of Let I be the moment of inertia of the uniform semicircular
momentum from the axis of rotation) disc
= mv × r [Here r = 0 because the line of
= mv × 0 action of momentum passes 2Mr 2 Mr 2
Þ 2I = ÞI=
=0 through the axis of rotation] 2 2
Rotational Motion P-S-67

20. (b) Let the mass per unit area


l
15. (d) be s. 2R
A B
(0, 2 l) Then the mass of the
complete disc
O R
F
= s [p(2 R) 2 ] = 4psR 2
P
2l
(0, l)
y The mass of the removed disc = s (pR 2 ) = psR 2
Let us consider the above situation to be a complete
(0, 0) disc of radius 2R on which a disc of radius R of negative
C mass is superimposed. Let O be the origin. Then the
uur above figure can be redrawn keeping in mind the
To have linear motion, the force F has to be applied concept of centre of mass as :
at centre of mass. 2 R
i.e. the point ‘P’has to be at the centre of mass 4ps R
2
m1 y1 + m2 y2 m ´ 2 l + 2m ´ l 4l O –ps R
y=
m1 + m2
=
3m
=
3
xc.m =
(4psR ) ´ 0 + ( -psR ) R
2 2

16. (c) Initially, 4psR 2 - psR 2


-psR 2 ´ R R 1
\ xc.m = 2
\ xc.m = - Þa=
3psR 3 3
m ( - x ) + m2 x2
0= 1 1 Þ m1 x1 = m2 x2 ...(1) 21. (b) This is a standard formula and should be memorized.
m1 + m2
Finally, g sin q
a=
The centre of mass is at the origin I
1+
x1– d x2– d ¢ MR 2
d d¢ uur
uur d Lc
m1 O m2 22. (d) We know that t c =
dt
uur
m1 (d - x1 ) + m2 ( x2 - d ') where tc torque about the center of mass of the body
\0 =
m1 + m2 uur
and Lc = Angular momentum about the center of mass
m1 of the body. Central forces act along the center of mass.
Þ 0 = m1d - m1 x1 + m2 x2 - m2 d ' Þ d ' = m d Therefore torque about center of mass is zero.
2 uur uur
[From (1).] When t c = 0 then Lc = constt.
17. (c) l 23. (d) By the theorem of perpendicular axes,
D C
n Iz = Ix + Iy or, Iz = 2 Iy
O
2 (Q Ix = Iy by symmetry of the figure)
l/
A B I Z
\ I EF = z ...(i) Y
2 D F C
n' Again, by the same theorem
Iz = IAC + IBD = 2 IAC
Inn' = M.I due to the point mass at B + (\ IAC = IBD by symmetry of the figure) X
M.I due to the point mass at D + O
M.I due to the point mass at C. Iz
\ I AC = ...(ii)
2 2
æ l ö From (i) and (ii), we get IEF = IAC. A E B
I nn ' = 2 ´ m ç + m( 2l) 2
è 2 ÷ø 24. (a) · When n = 0, x = k where k is a constant. This means
that the linear mass density is constant. In this case
= ml 2 + 2ml 2 = 3ml 2 the centre of mass will be at the midelle of the rod ie at
ur ur uur L/2. Therefore (c) is ruled out
18. (b) Torque t = r ´ F = ( ˆj - iˆ) ´ (- Fkˆ) = – F (iˆ + ˆj ) · n is positive and as its value increases, the rate of
19. (d) Applying conservation of angular momentum I'w' = Iw increase of linear mass density with increase in
x increases. This shows that the centre of mass will
(mR 2 + 2MR 2 )w ' = mR 2 w shift towards that end of the rod where n = L as the
value of n increases. Therefore graph (b) is ruled out.
é m ù
Þ w' = wê æ xö
n
ë m + 2 M úû
x
· The linear mass density l = k çè ÷ø Here £1
L L
With increase in the value of n, the centre of mass shift
EBD_7036
P-S-68 Topic-wise Solved Papers - PHYSICS
towards the end x = L such that first the shifting is at a from principle of conservation of angular momentum,
higher rate with increase in the value of n and then the angular speed, first increases then decreases.
rate decreases with the value of n. 29. (a) F = 20t – 5t2
These characteristics are represented by graph (a). FR dw
\ a= = 4t - t 2 Þ = 4t - t 2
L L L n I dt
æ xö
ò x dm ò x (l dx) ò k çè L ÷ø .xdx w t

xCM = 0 = 0 = 0 Þ ò dw = ò (4t - t ) dt
2

L L L n 0 0
æxö
ò dm ò l dx ò k çè L ÷ø dx Þ w = 2t 2 -
t3
(as w = 0 at t = 0, 6s)
0 0 0
3
n+ 2 L
é x ù q 6 æ
t3 ö
kê nú ò d q = ò ç 2t 2 - ÷ dt
ëê (n + 2) L ûú0 L(n + 1) 0 0 è

= =
é kx n +1 ù
L n+2 36
Þ q = 36 rad Þ n = <6
ê nú 2p
ëê (n + 1) L ûú 0 30. (c) From conservation of angular momentum about any fix
L point on the surface, r
For n = 0, xCM = ; n = 1, o
2 mr 2 w 0 = 2mr 2 w o
2L 3L
xCM = ; n = 2, xCM = ;.... w0 r
3 4 Þ w = w0 / 2 Þ v =
2
[Q v = rw ]
n m
1 ma 2 31. (c) Torque working on th e bob of mass m is,
25. (d) Inn' = m(a 2 + a2 ) = A
12 6 t = mg ´ l sin q . (Direction parallel to plane of rotation
D
DB 2a a of particle)
Also, DO = = =
2 2 2 ur
O As t is perpendicular to L , q
According to parallel axis theorem
direction of L changes but l l
2
æ a ö B magnitude remains same.
Imm ' = I nn ' + m ç ÷
C
è 2ø m mg
n' m1 ò ydm
ma 2
ma 2
ma + 3ma 2
2 2 32. (d) ycm =
=
6
+
2
=
6
= ma 2
3
ò dm y a
26. (c) The moment of inertia O h
2 r h
of the rod about O is ò pr dyr ´ y 3h
1 2 =0 =
ml . The maximum 1 2 R
3 4
C. M pR hr
angular speed of the h 3
rod is when the rod is C. M Reference
level for P.E. 2
i n s t a n t a n e ou sl y 33. (a) Here a = R
vertical. The energy 3
of the rod in this 4 a
1 pR 3
M
condition is I w 2 where I is the moment of inertia of Now, = 3 3
2 M¢ a
the rod about O. When the rod is in its extreme portion,
4 3
its angular velocity is zero momentarily. In this case, pR
3 2M
the energy of the rod is mgh where h is the maximum = 3 3
= p. M¢ =
height to which the centre of mass (C.M) rises æ 2 ö 2 3p
1 2 1 æ 1 2ö 2 çè R ÷ø
l 2 w2 3
\ mgh = I w = 2 çè 3 ml ÷ø w Þ h = Moment of inertia of the cube about the given axis,
2 6g
27. (b) For translational motion, 2
m 2M æ 2 ö
mg – T = ma .....(1) ´ç R÷
R M¢ a 2
3p è 3 ø 4MR 2
For rotational motion, I= = =
a
T 6 6 9 3p
T.R = I a = I ....(2) 34. (c) As shown in the diagram, the B D
R m
Solving (1) & (2), mg normal reaction of AB on
mg mg 2 mg 2 g roller will shift towards O.
a= = = =
æ I ö mR 2 3m 3
This will lead to tending of
çè m + 2 ÷ø m+ O
R 2R 2 the system of cones to turn
28. (c) As insect moves along a diameter, the effective mass
and hence the M.I. first decreases then increases so left.
A C
7
Gravitation P-S-69

Gravitation

Section-A : JEE Advanced/ IIT-JEE


4G
A 1. 1.23 × 10–3 rad/s 2. 8.48 hr 3. v= ( M1 + M 2 ) 4. h=R
d
B 1. F
C 1. (c) 2. (a) 3. (b) 4. (c) 5. (c) 6. (d)
7. (c) 8. (a) 9. (b) 10. (b)
D 1. (b) 2. (a, c, d) 3. (a, b) 4. (a, c) 5. (b, d) 6. (b)

Gm a3
E 1. (i) -p ´ 104 km/hr (ii) 3 ´ 10 -4 rad/s 2. , 2p
a 3Gm

3 5GM
3. (i) 6400 km (ii) 7.92 km/s 4. 5. 99.5 R
2 a
H 1. (a)
I 1. 3 2. 2 3. 7

Section-B : JEE Main/ AIEEE


1. (c) 2. (b) 3. (d) 4. (a) 5. (c) 6. (c)
7. (c) 8. (d) 9. (c) 10. (b) 11. (c) 12. (d)
13. (c) 14. (d) 15. (c) 16. (b) 17. (d) 18. (c)
19. (d) 20. (a) 21. (d) 22. (d) 23. (b)

Section-A JEE Advanced/ IIT-JEE


A. Fill in the Blanks This increase in P.E. is at the expense of K.E. of mass m
1. We know that g' = g – Rw2 cos2 f 1 2 Gm
At equator, f = 0, Therefore g' = g – Rw2 \ mv = ( M1 + M 2 )
2 d /2
g where v is the velocity with which mass m is projected.
Here g' = 0 \ w= = 1.23 ´ 10-3 rad/s
R 4G
Þ v= (M1 + M 2 )
2. KEY CONCEPT : According to Kepler's law T µ R 2 3 d
1 2 æ GMm ö GMm
T12 R13 4. mv + ç - ÷ =- ... (ii)
= . Here R1 = R + 6 R = 7 R 2 è R ø ( R + h)
T22 R23
From (i) and (ii)
and R2 = 2.5 R + R = 3.5 R
GMm GMm GMm
- =-
24 ´ 24 7´7´7´R 3 2R R R+h
Þ = Þ T2 = 8.48 hr
T22 3.5 ´ 3.5 ´ 3.5 ´ R 3 1 1
or, - =- Þ R + h = 2R
3. Increase in P.E. of system 2R R+h
= {(P.E.)i – (P.E.)f} or, h=R
ì é GM 1M 2 GM 1m GM 2 m ù é GM 1M 2 ù ü B. True/ False
= – í ê- - - - - úý
îë d d /2 d / 2 úû êë d ûþ
1. False
Gm New Delhi is not on the equatorial plane and geostationary
= ( M1 + M 2 ) satellite is launched on the equatorial plane.
d /2
EBD_7036
P-S-70 Topic-wise Solved Papers - PHYSICS
C. MCQs with ONE Correct Answer 2
\ mv = m ´ é GM ù where M is the total mass of the
GM GM r ê 2 ú
ë r û
1. (c) g = 2 and g' = spherical system.
R (0.99 R) 2
1
2 \ vµ
g' æ R ö r
\ =ç ÷ Þ g' > g
g è 0.99 R ø For r < R Again F ' = m Eg'
GMm 2
2. (a) Ui = - = Initial potential energy of the system. \ mv = m é GM ´ r ù
R r ê 3 ú
ë R û
GMm Þ vµr
Uf = - = Final P.E. of the system.
2R 8. (a) Let us consider a circular elemental area of radius x and
\ DU = Uf – Ui B thickness dx. The area of the shaded portion = 2pxdx.
Let dm be the mass of the shaded portion.
é 1 1 ù GMm R
= – GMm ê - = ...(i) P
ë 2R R úû
Mass M
2R \ =
area p ( 4 R 2 ) - p ( 3R )
A 2
GM 4R
But g = 2
R R dm
=
\ GM = gR 2 ... (ii) 2 pxdx
From (i) and (ii) O x dx
2M
\ dm = xdx
gR 2 m mgR 7R2
DU = = The gravitational potential of the mass dm at P is
2R 2
T12 R3 -G dm G 2M
= 1 dV = =- ´ 2 xdx
3. (b) According to Kepler's law
T22 R23 ( 4R ) + x 2
2 2
16 R + x 2 7R

R -2GM xdx
Here T1 = 365 days ; T2 = ?; R1 = R ; R2 = = 2
(1)
2 7R 16 R 2 + x 2
3/ 2 3/ 2 Suppose 16R + x2 = t2
2
æ R2 ö é R /2ù
Þ T2 = T1 ç ÷ = 365 ê = 129 days Þ 2xdx = 2tdt Þ xdx = tdt
èR ø 1 ë R úû Also for x = 3R, t = 5R
4. (c) Note : A satellite revolving near the earth's surface has and for x = 4R, t = 4 2R
a time period of 84.6 min.
We know that as the height increases, the time period On integrating equation (1), taking the above limits,
increases. Thus the time period of the spy satellite we get
should be slightly greater than 84.6 minutes. 4 2R
2GM -2GM 4 2R
\ Ts = 2 hr V =- ò dt = [t ]5 R
7R2 7R2
5. (c) The components of 5R

acceleration are as shown ar -2GM é


r r = 4 2 R - 5R ùû Þ V = -2GM 4 2 - 5
( )
a = ar + at
a
7R2 ë 7R
The resultant of transverse and WP¥
radial component of the at Now = V¥ - VP = -VP [ Q V¥ = 0]
r 1
acceleration is represented by a
6. (d) Note : The gravitational force of attraction between \ WP¥ =
2GM
7R
(
4 2 -5 )
the stars will provide the necessary centripetal forces. 9. (b) V is the orbital velocity. If VC is the escape velocity
In this case angular velocity of both stars is the same.
2p ö then Ve = 2 V . The kinetic energy at the time of ejection
æ
Therefore time period remains the same. ç w = ÷ . 1 1
è T ø KE = mVe 2 = m( 2 V )2 = mV 2
2 2
Re
10. (b) Rp = = 6 ´ 105 m Rp
RA 10 Rp
MB MA The mass of the wire = 10 –3
RB 10 5
× 1.2 × 105 = 120 kg M
Let gpM be the acceleration Rp
due to gravity at point M
7. (c) For r ³ R which is the mid point of the
Force on the test mass m is F = m × | Eg |
Rp
Where Eg is the gravitational field intensity at the point wire and is at a depth of .
of observation 10
Gravitation P-S-71

Let gp be the acceleration due to gravity at the surface centre of the earth. Net torque of this gravitational force
of the planet. F about centre of earth is zero. Therefore, angular
4 momentum (both in magnitude and direction) of S about
4
g p = prGRP ; g = prGRE centre of earth is constant throughout. Since the force
3 e
3 F is conservative in nature, therefore mechanical
g p Rp 1 energy of satellite remains constant. Speed of S is
\ = = maximum when it is nearest to earth and minimum when
ge RE 10
it is farthest.
10
\ gp = = 1 ms –2 5. (b, d)Let the mass of P be m.
10 3/ 2
4 3 4 éAù
é R p /10 ù Then m = r´ pr = r´ p ê ú
ú = 1[1 - 0.1] = 0.9 ms
-2 3 3 ë 4p û
and g pM = g p ê1 -
êë R p úû 3/2
4 é 4A ù
\ Force = mass of wire × gpM = 120 × 0.9 = 108 N The mass of Q = r ´ p =8m
3 êë 4p úû
D. MCQs with ONE or MORE THAN ONE Correct \ The mass of R = 9 m
If the radius of P = r
1. (b) KEY CONCEPT : The centripetal force is provided by Then the radius of Q = 2r
the gravitational force of attraction
é æ 4A ö
3/2
æ Aö
3/2 ù
mRw2 = GMmR–5/2 êQ rQ = ç = 2ç ú
÷ ÷
2 êë è 4p ø è 4p ø úû
Þ mR ´ 4p = GMm Þ T2 µ R7/2 1/3
and radius of R = 9 r
T2 R5 / 2
2. (a,c,d) The gravitational field intensity at the point O is zero éQ MR = MP + MQ ù
(as the cavities are symmetrical with respect to O). Now the ê 3 3 3 3
ú
force acting on a test mass m0 placed at O is given by ëê rR = r + (2r ) = 9r ûú
F = m0 E = m0 × 0 = 0 2GMP 2Gm
=
Now, y2 + z2 = 36 represents the equation of a circle with Now, vP = Rp r
centre (0, 0, 0) and radius 6 units the plane of the circle is
perpendicular to x-axis. 2GMQ 2G (8m)
Y vQ = = = 2vP
RQ 2r
2G (9 m)
vR = 1/3
= 91/3 vP
9 r
A B 1 é GMm ù GM
(-2,0,0)
O (2,0,0) 6. (b) mv 2 = 2 ê ú Þv=2
X 2 ë L û L
4
The potential energy is a combined property of the three
mass system. The kinetic energy of mass m is only its energy
Z which decreases as it moves.
Note : Since the spherical mass distribution behaves as if (b) is the correct option.
the whole mass is at its centre (for a point outside on the
sphere) and since all the points on the circle is equidistant
from the centre of the sphere, the circle is a gravitational
equipotential.
The same logic holds good for option (d).
GM
3. (a,b) For r > R, the gravitational field is F =
r2 E. Subjective Problems
2
GM GM F r 1. (i) According to Kepler's third law
\ F1 = and F2 = Þ 1 = 2
2
r12 r2 F 2 r12 T12 R3 T2
GM = 1 Þ R23 = R13 ´ 2 v2
For r < R, the gravitational field is F = 3 ´ r T22 R23 T12 S2
R v1

( ) 82
GM GM 3
\ F1 = 3 ´ r1 and F2 = 3 ´ r2 \ R23 = 104 ´ = 64 ´ 1012 S1
R R 2
1 P
F1 r1 Þ R2 = 4 × 104 km. R1
Þ =
F2 r2 Linear speed of satellite S1
4. (a, c)Force on satellite is always towards earth, therefore, 2pR1 2p ´ 104 R2
acceleration of satellite S is always directed towards v1 = = = 2p ´ 104 km/hr
T1 1
EBD_7036
P-S-72 Topic-wise Solved Papers - PHYSICS
Linear speed of satellite S2, 4. MB=16M MS=M

2pR2 (2p) (4 ´ 104 ) 2a


v2 = = = p ´ 104 km/hr P a
T2 8
The speed of satellite S2 w.r.t. S1 Bigger Smaller
star x star
= v2 – v1 = p × 104 – 2p × 104 = – p × 104 km/hr
(ii) Angular speed of S2 w.r.t. S1 10a
The force of attraction is zero at say x from the bigger star.
vr v -v 3.14 ´ 104 ´ 5 /18 Then force on mass m due to bigger star = Force on mass m
= = 2 1 = = 3 × 10–4 rad/s
Rr R2 - R1 3 ´ 104 ´ 103 due to small star
GM B m GM S m 16 M M
2 2 a2 a = Þ = Þ x = 8a
2
2. The radius of the circle r = a - = x 2
(10 a - x) 2 x (10a - x) 2
3 4 3
If we throw a mass m from bigger star giving it such a velocity
Let v be the velocity given. The centripetal force is provided
that is sufficient to bring it to P, then later on due to greater
by the resultant gravitational attraction of the two masses. force by the star MS it will pull it towards itself [without any
FR = F 2 + F 2 + 2 F 2 cos 60° m
external energy thereafter].
m´m The energy of the system (of these masses) initially
= 3 F = 3G 2 = Final energy when m is at P
a GM B M S GM B m GM S m 1 2
m2 mv 2 - - - + mv
\ 3G = a/ Ö 3
10a 2a 8a 2
a2 r F
FR GM B M S GM B m GM S m
m m =- - -
æ mv 2 ö F a 10a 8a 2a
ç = centripetal force÷ v
[Q M B = 16M ; M S = M ]
è r ø
3Gmr 3 G ma 3 5GM
Gm \ v=
v2 = = Þ v= 2 a
a2 a2 ´ 3 a
5. Total energy at A = Total energy at B
Time period of circular motion (K.E.)A + (P.E.)A = (P.E.)B
2pr 2 p a / 3 a3 1 2GM é -GMm ïì 2 æ 99 R ö ïü ù
2
GMm
T= = = 2p +ê
v Gm 3Gm Þ m´ í3R - ç ÷ ýú = -
2 R 3
êë 2 R îï è 100 ø ï ú R+h
þû
a On solving we get h = 99.5 R.
3. (i) KEY CONCEPT : Since the satellite is revolving in a
B
circular orbit, the centripetal force is provided by the
gravitational pull. ve
v R
mv 2
GMm m A 100
A
= 99R
( R + h) ( R + h)2 100
B h R

GM M R
\ v2 =
R+h E

1 1 2GM
But v = ve = H. Assertion & Reason Type Questions
2 2 R
1 æ 2GM ö GM 1. (a) The normal force exerted by the astronaut on orbiting
\ ç ÷= space station is zero (until the astronaut exerts some
4è R ø R+h
muscular force). Therefore the apparent weight of
Þ 2R + 2h = 4R Þ h = R = 6400 km.
astronaut in an orbiting space station is zero. Astronaut
(ii) KEY CONCEPT : When the satellite is stopped, its is called in a state of weightlessness. This is because
kinetic energy is zero. When it falls freely on the Earth, its astronaut as well as space -ship are freely falling bodies.
potential energy decreases and converts into kinetic energy. Statement - 1 is true, statement - 2 is true and statement
\ (P.E.)A – (P.E.)B = K.E. - 2 is the correct explanation of statement - 1.
-GMm æ -GMm ö 1 2
Þ -ç = mv I. Integer Value Correct Type
2R è R ÷ø 2
GM 1. We know that v = 2 gR
Þ v= = gR = 9.8 ´ 6.4 ´ 106
R vp gp Rp
= 7920 m/s = 7.92 km/s \ = ´ ...(i)
v g R
Gravitation P-S-73

gp Now applying conservation of energy for the throw


6
Given = ...(ii) Loss of kinetic energy = Gain in gravitaional potential energy
ge 11
1 2 GMm æ GMm ö
g p rp Rp \ –ç–
R ÷ø
4 mv = –
Also g = p G rR \ = ´ 2 2R è
3 g r R
GM
6 2 Rp é rp 2 ù \ v= ...(ii)
\ = ´ êQ = (given)ú R
11 3 R ë r 3 û Comparing (i) & (ii) N=2
Rp 3 6 3. (7) For the tension in the rod to be zero, the force on both
\ = ...(iii) the masses m and m should be equal in magnitude and
R 22
direction. Therefore
vp 6 3 6 3´ 6 3 M
From (i), (ii) & (iii) = ´ = =
v 11 22 11 ´ 22 11 m m
3 3
\ v p = ´ v = ´11km/s = 3 km/s
11 11
2. (2) Let h be the height to which the bullet rises 3l l
–2 GMm Gmm GMm Gmm
æ hö + = –
then, g1 = g ç1 + ÷ (4l)2 l2 (3l)2 l2
è R ø
–2 é1 1 ù
g æ hö \ 2m = M ê – ú
Þ = g ç1 + ÷ ë 9 16 û
4 è Rø 7M
Þ h=R \ m=
288
2GM K=7
We know that ve = = v N (given) ...(i)
R

Section-B JEE Main/ AIEEE


1 Force = mass × acceleration.
1. (c) K. E = m ve2 where ve = escape velocity = 2gR 9R
2
1 xM x5M
R 2R
\ K.E = m ´ 2 gR = mgR
2 12R
2. (b) Due to inertia of motion it will move tangentially to the
original orbit in the same velocity. 1
For same force, acceleration µ
3. (d) Energy required = (Potential energy of the Earth -mass mass
system when mass is at distance 3R) – (Potential energy a5 M M 1
\ = = .....(i)
of the Earth -mass system when mass is at distance 2R) aM 5M 5
-GMm æ -GMm ö -GMm GMm Let t be the time taken for the two masses to collide and
-ç +
=
3R è 2R ÷ø = 3R 2R
x5M, xM be the distance travelled by the mass 5M and
M respectively.
-2GMm + 3GMm GMm For mass 5M
= =
6R 6R u = 0, S = x5M , t = t , a = a5M
1 1
2GM S = ut + at 2 \ x5 M = a5M t 2 ....(ii)
4. (a) Escape velocity, ve = 2 gR = Þ Ve µ m0 2 2
R
For mass M
Where M, R are the mass and radius of the planet u = 0, s = xM, t = t, a = aM
respectively. In this expression the mass of the body
(m) is not present showing that the escape velocity is 1 2 1 2
\ s = ut + at Þ xM = aM t … (iii)
independent of the mass. 2 2
5. (c) According to Kepler’s law of planetary motion T2 µ R3 Dividing (ii) by (iii)
3 3 1
æR ö 2
é 4R ù 2
x5 M a5 M t 2
\ T2 = T1 ç 2 ÷ =5´ ê ú = 5 ´ 23 = 40 hour = 2 a 1
èR ø ëRû = 5M =
1 xM 1
a t 2 aM 5 [From (i)]
6. (c) The gravitational force acting on both the masses is 2 M
the same. We know that \ 5 x5M = xM ....(iv)
EBD_7036
P-S-74 Topic-wise Solved Papers - PHYSICS
From the figure it is clear that 4
G ´ r ´ pR 3
x5M+ xM= 9R ....(v) GM Gr ´ V 3
14. (d) g = = Þg=
Where O is the point where the two spheres collide. R2 R2 R 2
From (iv) and (v) 4
g = rpG. R where r ® average density
xM 3
+ xM = 9 R
5 2GM p
45 Rp
\ 6 xM = 45R \ xM = R = 7.5 R (ve ) p Mp Re
6 15. (c) = = ´ =
(ve )e 2GM e Me Rp
7. (c) ve = 2 gR
Re
The escape velocity is independent of the angle at
which the body is projected. 10M e Re
= ´ = 10
8. (d) Gravitational force provides the necessary centripetal Me R e /10
force.
\ (ve ) p = 10 ´ (ve )e = 10 ´ 11 = 110 km / s
mv 2 GmM GM 16. (b) Gravitational flux through a closed surface is given by
\ = also g = 2 uuur r
( R + x) ( R + x) 2
R
1/ 2
ò E g dS = -4pGM
gR 2 æ gR 2 ö where, M = mass enclosed in the closed surface
2
\v = Þ v=ç ÷ 1
R+ x è R + xø This relationship is valid when | E g | µ 2 .
r
mv 2 GmM
9. (c) We have, = g' R2
R + x ( R + x )2 17. (d) We know that =
g ( R + h) 2
x = height of satellite from earth surface
2
m = mass of satellite g /9 é R ù
\ =ê \ h = 2R
GM GM g ë R + h úû
Þ v2 = or v = 18. (c) Let the gravitational field at P, distant x from mass m,
( R + x) R+x
be zero.
2p ( R + x ) 2 p ( R + x )
T= = Gm
=
4Gm r
v GM \ x2 (r - x )2
Þ x=
3
R+x m P 4m
which is independent of mass of satellite x
-GmM GmM GmM r
10. (b) \ DU = + ; DU = Gm 4Gm 9Gm
2R R 2R Gravitational potential at P, V = - r - 2r = -
r
GM GM 1 3 3
Now 2 = g ; \ = gR \ DU = mgR
R R 2 19. (d) The required energy for this work is given by
11. (c) F = KR–n = MRw2 Þ w 2 = KR - ( n +1) GMm
= mgR
- ( n +1) R
= 1000 × 10 × 6400 × 103
or w = KR 2
= 6.4 × 1010 Joules
- (n +1) + ( n +1)
2p 20. (a) Applying energy conservation
µR 2
\T µ R 2
T æ GMm ö 1 2 æ GMm ö
k +ç- ÷ = mv0 + ç - ÷
é 2h ù è R ø 2 è R+h ø
12. (d) Variation of g with altitude is, gh = g ê1 - ú ;
ë Rû GMm 1 æ Gm ö GMm
k- = mç ÷-
variation of g with depth is, gd = g éê1 - ùú
d R 2 è R + 2R ø R + 2R
ë Rû 5GMm
\k=
Equating g h and gd , we get d = 2h 6R
Mv 2
13. (c) Workdone, W = DU = U f - U i = 0 - é -GMm ù 21. (d) 2 F cos 45° + F ' = (From figure)
êë R úû R
GM 2 GM 2
6.67 ´10 -11 ´ 100 10 Where F = and F ' =
W= ´ = 6.67 × 10–10 J ( 2 R) 2 4R 2
0.1 1000
Gravitation P-S-75

Solid
F sphere
M M
F'
F
R
o P
Cavity

M M
Due to cavity part potential at point P
2 ´ GM 2 GM 2 Mv 2 GM
Þ + =
2( R 2)2 4 R2 R 3 8 3GM
Vcavity = - =-
GM 2 é 1 1 ù 2 R 8R
2
Þ + ú = Mv 2
R êë 4 2û So potential at the centre of cavity
Gm æ 2 + 4 ö 1 Gm 11GM æ 3 GM ö -GM
\ v= ÷= (1 + 2 2) = Vsphere - Vcavity = - - ç- =
ç
R çè 4 2 ÷ø 2 R 8R è 8 R ÷ø R
22. (d) Due to complete solid sphere, potential at point P 23. (b) For h << R, the orbital velocity is gR

- GM é 2 æ R ö

Escape velocity = 2gR
Vsphere = ê3R - ç ÷ ú
2R 3 êë è2ø úû \ The minimum increase in its orbital velocity
-GM æ 11R ö 2
GM = 2gR – gR = gR ( 2 – 1)
= 3 ç 4 ÷
= -11
2R è ø 8R
EBD_7036
8 Mechanical PropertiesTopic-wise
of Solids and
P-S-76 Solved Papers - PHYSICS

Fluids
Section-A : JEE Advanced/ IIT-JEE

YAx 2 Mg
A 1. 2. 3. ( g 2 - g 1 ) DT 4. 500 Pa.
2L 3 Ak
B 1. F 2. F 3. F 4. F
C 1. (c) 2. (a) 3. (b) 4. (a) 5. (a) 6. (d)
7. (d) 8. (a) 9. (a) 10. (a) 11. (b) 12. (a)
13. (c) 14. (d)
D 1. (a) 2. (b, c) 3. (c) 4. (c) 5. (c) 6. (a, d)
7. (b, c) 8. (a, b) 9. (a, d)
1 YA d L t1
E 1. 2.95 cm 2. 3. l = 10 cm 4. Fall 5. 45° 6. (a) d - d
2p mL L
(b) no (c) remains same
5d æ 3H L ö 3H - 4h 3H 3H
7. (a) (i) (ii) P0 + ç + ÷ dg (b) (i) g (ii) (3H - 4h)h (iii) , 8. 29.025 × 103 J/
4 è 2 4ø 2 8 4

4T p 1 2 é d4 ù 4
m3 ; 29.4 × 103 J/m3 9. (a) zero (b) 0.25 cm (c) g/6, ­ 10. 11. 8Ql ´ 2 rv0 ê1 - 4 ú ´ a
rv 2 êë D úû

lag w2 L2
12. 2m 13. 14. H =
2y 2g
F 1. (c)
G 1. (c) 2. (b) 3. (a) 4. (c) 5. (a) 6. (b)
7. (c) 8. (a)
H 1. (a)
I 1. 6 2. 6 3. 4 4. 3
Section-B : JEE Main/ AIEEE
1. (b) 2. (d) 3. (b) 4. (a) 5. (a) 6. (c) 7. (a) 8. (c)
9. (a) 10. (d) 11. (c) 12. (c) 13. (b) 14. (c) 15. (c) 16. (c)
17. (d) 18. (c) 19. (d) 20. (None) 21. (a)

Section-A JEE Advanced/ IIT-JEE


A. Fill in the Blanks R 3 - [ R 3 - 3 R 2 dR ] Mg dR Mg
Þ 3
= Þ =
1 1 x 2
YAx 2 R AK R 3 AK
1. W= ´ Y ´ (strain) 2 ´ Yd = ´ Y ´ 2 ´ AL = 3. KEY CONCEPT
2 2 L 2L
-DP Using the relation for floatation, vdHgg = Vdmg
2. K= Fraction of volume of metal submerged in mercury
DV / V
Mg DV Mg v dm
where DP= \ - = = = = K1 (say)
A V AK V d Hg
(V f - Vi ) Mg Vi - V f Mg vdHgg v'd'Hgg
Þ - = Þ =
Vi AK Vi AK
V V'
4 3 4
pR - p( R - dR )3 v v'
3 3 Mg
Þ =
4 3 AK Vdmg V'd'mg
pR
3
T T +D T
Mechanical Properties of Solids and Fluids P-S-77

In second case, when temperature is increased by DT. C. MCQs with ONE Correct Answer
v'd'Hgg = V'd'm g
1. (c)
v' d'
Þ = m = Fraction of volume of metal submerged
V ' d 'Hg A (dx)
D
in mercury = K2 (say)
K 2 d 'm ´ d Hg d 'm ´ d 'Hg (1 + g 2 DT ) (1 + g 2 DT )
\ = = =
K1 d 'Hg ´ d m d 'Hg ´ d 'm (1 + g 1DT ) (1 + g 1DT )
= (1 + g2 DT) (1 + g1 DT)–1
= (1 + g2 DT) (1 – g1 DT) = 1 + ( g2 – g1) DT B C
Note : If g2 – g1 then k2 > k1 Let us consider a small dotted segment of thickness dx
i.e., metal block will get immersed deeper for observation.
If g2 < g1 then k2 < k1 Since, this segment is accelerated towards right, a net
i.e. metal block will rise a bit as compared to its previous force is acting in this segment towards right from the
position. liquid towards the left of ABCD. According to Newton's
K2 K - K1 third law, the segment ABCD will also apply a force on
- 1 = ( g 2 - g 1 ) DT Þ 2 = ( g 2 - g 1 ) DT the previous section creating a pressure on it which
K1 K1 makes the liquid rise.
4. KEY CONCEPT
T/A T ´l T l
Applying equation of continuity at cross section 1 and 2 2. (a) Y = Þ Dl = = ´
Dl / l A´Y Y A
A1v1 = A2v2 Þ 10 × 1 = 5 × v2 Þ v2 = 2m/s
T l
Applying Bernoulli's theorem Here, is constant. Therefore, Dl µ .
Y A
1 1
P1 + rv12 = P2 + rv22 l
2 2 is largest in the first case.
P1=2000Pa A
1 3. (b) Pressure in limb I at B = Pressure in limb II at A
Þ 2000 + ´1000 ´12 P2
2 hr1g = hr2g
Þ r1 = r2
–1
1 v1 =1ms v2
2
= P2 + ´1000 ´ 2 A2=5cm2 h P1 II h P2 I
2
Þ P2 = 500 Pa A1=10cm2 A B

B. True/ False
1. When the man drinks some water from the pond, his weight
increases and therefore the boat will sink further. The further
sinking of the boat will displace the same volume of water in 4. (a) Weight of cylinder = Upthrust due to upper liquid +
pond as drunk by man. Therefore, there will no change in Upthrust due to lower liquid.
the level of water in the pond.
2. Pressure P1= P2 = 1 atm = hrg æA ö æ Aö æ 3 ö æ Aö æ L ö
D ç ´ L ´ g ÷ = d ç ÷ ç L ÷ g + 2d ç ÷ ç ÷ ´ g
Note : è5 ø è 5ø è 4 ø è 5 ø è 4ø
On changing the temperature, g will not change and 5d
atmospheric pressure will not change. \ D=
4
\ h × r = constant. 5. (a) Equating the rate of flow, we have
When temperature is increased, the density of Hg decreases
and hence, h increases. (2 gy ) ´ L2 = (2 g ´ 4 y ) pR 2

[Flow = (area) × (velocity), velocity = 2gx ]


h where x = height from top
P2 L
P1 Þ L2 = 2pR2 Þ R =
2p
6. (d) KEY CONCEPT :
3. When water is heated at end A, the density decreases and According to Archimedes principle
the water moves up. This is compensated by the movement Upthrust = Wt. of fluid displaced P1 h
of water from B to A i.e., in clockwise direction. Fbottom – Ftop = Vrg
4. When the block of ice melts, the lead shot will ultimately \ Fbottom = Ftop + Vrg
V
sink in the water. When lead shot sinks, it will displace water = P1 × A + Vrg
equal to its own volume. But when lead shot was embedded = (hrg) × (pR2) + Vrg
in ice, it displaced more volume of water than its own volume
because dlead > dwater. Therefore, level of water will fall. = rg [pR2 h + V]
EBD_7036
P-S-78 Topic-wise Solved Papers - PHYSICS
7. (d) l decreases as the block moves up. h will also decreases Dl 2
F / p (2R)2 F / p R 2 \ =2
because when the coin is in water it will displace a 13. (c) Y= = Dl1
volume of water, equal to its own volume, whereas Dl1 / 2L Dl 2 / L
when it is on the block it displaces more volume than
to own volume (because density of coin is greater than 14. (d) C
density of water). R a
2
F Dl F l = 20 ´ 1 q+a q
8. (a) Y= = . 11 2
-6 -4 = 2 ´ 10 N / m . M
A l A Dl 10 ´ 10 b 2 P
9. (a) The square of the velocity of efflux
2 gh 2 ´ 10 ´ 2.475
v2 = or,, v2 = = 50 m2/s2
2 2
æ aö 1 - (0.1) h
1- ç ÷
è Aø
h = 3 - 0.525 = 2.475 m
Dp (1.165 ´ 105 - 1.01 ´ 105 ) aö b
10. (a) B = = = 1.55 × 105 Pa æ
DV / V 0.1 In DPCM cos ç q + ÷ = ...(1)
11. (b) We know that excess pressure in a soap bubble is è 2ø R
inversely proportional to its radius. The soap bubble æ 2S ö
at end 1 has small radius as compared to the soap bubble Also ç P0 - ÷ + hrg = P0
è R ø
at end 2 (given). Therefore excess pressure at 1 is more.
2S 2S
h= = cos ( q + a 2 )
Rrg brg

2 1 D. MCQs with ONE or MORE THAN ONE Correct


1. (a) The whole system falls freely under gravity
As the value is opened, air flows from end 1 to end 2
Upthrust = weight of fluid displaced
and the volume of soap bubble at end 1 decreases.
= (mass of fluid displaced) × g
12. (a) Let V be the volume of the material of which the cylinder
is made. The cylinder is half immersed in water. For a freely falling body, g = 0
Therefore the volume of water displaced because of \ Upthrust = 0.
2. (b,c) When the block of mass m is
V
the material of the cylinder is . Let h be the total arranged as shown in the
2 A
figure, an upthrust FT will act
height of the cylinder. As the cylinder is half submerged on the mass which will
therefore buoyant force decrease the reading on A.
A Note :
According to Newton's third F F T T

law, to each and every action, B


h there is equal and opposite
V rw g hArw g
B= + h' h reaction.
2 2 2 So FT will act on the liquid of the beaker which will
increase the reading in B.
where A is the area of cross-section of the cylinder 3. (c) Weight of sphere
The weight of the cylinder W = Vrc g = Upthrust due to Hg + Upthrust due to oil
The weight of the water inside the cylinder V V
= h'Arw g Vdg = d Hg g + doil ´ g
2 2
For equilibrium, dHg + doil 13.6 + 0.8
V rw g Ahrw g Þ d= = = 7.2g / cm3
+ = V rc g + h ' Arw g 2 2
2 2 Dl
Here rw = 1 4. (c) D l = l a DT Þ = a DT
h V l
\ h' = + [1 - 2rc ] Stress = Y × strain \ Stress = Ya DT
2 2A For first rod stress = Y1a1D T
h For second rod stress = Y2a2D T
If rc < 0.5 then h ' >
2 Since, stresses are equal
h \ Y1a1D T = Y2a2D T
and if rc > 0.5 then h ' <
2 Y1 a 2 3
or, = =
h Y2 a1 2
if rc = 0, h ' =
2
Mechanical Properties of Solids and Fluids P-S-79

5. (c) KEY CONCEPT : The strain of P is more than Q therefore P is more


The equation of continuity is : v1A1 = v2A2 ductile. Therefore (B) is a correct option.
where v and A represent the speed of water stream and stress
its area of cross section, respectively. We are given Y=
that strain
v1 = 1.0 ms–1 For a given strain, stress is more for Q. Therefore
A1 = 10–4 m2 YQ > YP.
v2 = velocity of water stream at 0.15 m below the tap 8. (b, c) Let us consider an elemental mass dm shown in the
A2 = ? shaded portion.
For calculating v2 Here
u = 1 m/s; s = 1.5 m, a = 10 m/s2 and v = ? P 4pr2 – (P + dP) 4pr2
v2 – u2 = 2as dr
GMr r
v2 – 1 = 2 × 10 × 0.15 Þ v = 2 m/s = r (4pr2) dr
R3
v1 A1 1 ´ 10-4 p + dp p
Hence, A2 = = = 5 ´ 10 -5 m2 P r
v2 2 GM r
\ – ò dp = ò rdr
6. (a, d) Consider the equilibrium of the system of both spheres 0 R3 R
and the spring.
GM r é 2 2 ù
\ P= R -r
r 2 R3 ë û
é 2 9R 2 ù 2
2r êR – ú 7R
P (r = 3R/ 4) êë 16 úû 63
\ = = 16 =
P (r = 2 R/ 3) é 2 4 R 2 ù 5R 2 80
3r êR – ú
ëê 9 úû 9

The weight of system é 2 9 R2 ù


êR – ú
4 3 4 3 é4 3 ù P (r = 3R/ 5) ëê 25 ûú 16
= pR (3r)g + pR rg = 4 ê p R rg ú and = =
3 3 ë3 û P (r = 2 R/ 5) é 2 4R 2 ù 21
This is to be balanced by the buoyant force. Fs B êR – ú
This can be possible only when the light êë 25 úû
sphere is completely submerged. In this way B and C are correct options.
the buoyant force 9. (a, d) From the figure it is clear that
éæ 4 ö ù é4 ù (a) s2 > s1
B = êç p R 3 ÷ ´ 2ú ´ (2r) ´ g = 4 ê p R 3 rg ú W
ëè 3 ø û ë3 û (b) r2 > s2 [As the string is taut]
Now considering the equilibrium of the heavy sphere (c) r1 < s1 [As the string is taut]
Fs + B = W \ r1 < s1 < s2 < r2
\ Fs = W – B When P alone is in L2
4 3 4 3
\ Kx = pR (3r)g - pR (2r)g 2pr 2 (r1 – s 2 ) g
3 3 VP = is negative as r1 < s2
4 R 3 rg 9h2
\x = p
3 K Where r is radius of sphere.
7. (a, b) The maximum stress that P can withstand before When Q alone is in L1
breaking is greater than Q. Therefore (A) is a correct
option. 2pr 2 (r2 – s1 ) g
VQ = is positive as r2 > s1
Strain 9h1
ur ur
Max Therefore V P . V Q < O option (d) is correct
(strain P)

s1
Max
(strain Q) T
T

s2

Max
stress VP r1 – s 2 h1
Stress of P Also = ´ ...(i)
Maximum VQ r2 – s1 h2
stress of Q
EBD_7036
P-S-80 Topic-wise Solved Papers - PHYSICS
For equilibrium of Q From fig. (c)
4 3 4 Restoring force
T+ pr s2 g = pr 3 r 2 g ...(ii) é YA(l + x ) YA l ù
3 3 = – [T' – mg] = – ê - [from (iii)]
For equilibrium of P ë L L úû
-YAx
4 3 4 =
T+ pr r1 g = pr 3 s1 g ...(iii) L
3 3 On comparing this equation with F = – mw2x, we get
(iii) – (ii) gives
YA YA 2p YA
r1 – s2 = s1 – r2 ...(iv) mw2 = Þ w= Þ =
L mL T mL
From (i) and (iv)
1 1 YA
VP h VP h Frequency f = =
=– 1 \ = 1 T 2p mL
VQ h2 VQ h2 3. Let the edge of cube be l. When mass is on the cube of
\ A is also a correct option wood
E. Subjective Problems 200 g + l3 d wood g = l3 d water g
1. M is the mid-point of tube AB. Þ l3 d wood = l3 d water - 200 ... (i)
At equilibrium When the mass is removed
p1× A + mg = p2 × A
l3 d wood = (l - 2) l2 d water ... (ii)
p1 × A + 10 × A × dHgg = p2 × A
Þ p1 + 10dHg × g = p2 ... (i) From (i) and (ii)

A A l3 d water - 200 = (l - 2) l 2 d water


45 cm
But d water = 1
p1 P
x 10 cm
P' \ l3 - 200 = l 2 (l - 2) Þ l = 10 cm
M
M 4. KEY CONCEPT :
10 cm
When the stones were in the boat, the weight of stones
p2 Q' Q were balanced by the buoyant force.
mg 45 cm Vsds = Vl dl
B B Vl , Vs = volume of liquid and stone respectively
For air present in column AP dl , ds = density of liquid and stone respectively
p × 45 × A = p1 × (45 + x) × A
Since, ds > dl \ Vs < Vl
45
Þ p1 = ´ 76 d Hg ´ g ... (ii) Therefore when stones are put in water, the level of water
45 + x
falls.
For air present in column QB
p × 45 × A = p2 × (45 – x) × A 5. (a) For equilibrium Fnet = 0 and tnet = 0
45
Þ p2 = ´ 76d Hg ´ g ... (iii)
45 - x
x
From (i), (ii) and (iii)
45 ´ 76 ´ d Hg g 45 F1 q
1m

+ 10 d Hg ´ g = ´ 76 ´ d Hg ´ g
45 + x 45 - x 0.5m
45 ´ 76 45 ´ 76 q mg
Þ + 10 =
45 + x 45 - x
O
x = 2.95 cm.
Taking moment about O
2. From fig. (b), due to equilibrium
T = mg ... (i) l æ l - xö
mg ´ sin q = FT ç sin q ... (i)
T/A 2 è 2 ÷ø
But Y = L
l/ L Also FT = wt. of fluid displaced = [ (l - x) A] ´ rw g... (ii)
YAl And m = (l A) 0.5rw ... (iii)
Þ T= ... (ii) l
T
L x T' Where A is the area of cross section of the rod.
From (i) and (ii) mg
mg From (i), (ii) and (iii)
YAl l
mg = ... (iii)
Fig. (a) Fig. (b) Fig. (c)
æ l - xö
(l A) 0.5rw g ´ sin q = [(l - x ) A]rw g ´ ç sin q
L 2 è 2 ÷ø
Mechanical Properties of Solids and Fluids P-S-81

Here, l = 1 m
\ (1 – x)2 = 0.5 Þ x = 0.293 m
d A/5
From the diagram F H/2
D
0.5 0.5 3L
cos q = = Þ q = 45° 4
1 - x 0.707 mg
L
6. (a) Let the ball be dropped from a height h. During fall 4 H/2
t 2v 2d
A
V = ut + at = 0 + g 1 Þ t1 =
2 g
A A L A 3L
In the second case the ball is made to fall through the ´ L ´ D ´ g = ´ ´ 2d ´ g + ´ ´d ´g
5 5 4 5 4
same height and then the ball strikes the surface of
2d 3d 5d
liquid of density dL. When the ball reaches inside the Þ D= + =
4 4 4
liquid, it is under the influence of two force (i) Vdg, the (ii) Total pressure at the bottom of the cylinder =
weight of ball in downward direction (ii) VdLg, the Atmospheric pressure + Pressure due to liquid of
upthrust in upward direction. density d + Pressure due to liquid of density 2d +
Note : Pressure due to cylinder [Weight/Area]
The viscous forces are absent. (given) A
´L´D´g
H H
Since, dL > d P = P0 + dg + ´ 2d ´ g + 5
the upward force is greater and the ball starts retarding. 2 2 A
For motion B to C æ 3H L ö é 5d ù
A Þ P = P0 + ç + ÷ dg êQ D =
u = V , v = 0, t = t, a = – a h è 2 4ø ë 4 úû
v = u + at Þ 0 = v + (- a ) t (b) KEY CONCEPT :
Applying Bernoulli's theorem
v
Þ t= éH æH ö ù
a B P0 + ê ´ d ´ g + ç - h÷ 2d ´ g ú
u=V ë2 è 2 ø û
Fnet
Now, a = VdLg 1 (3H - 4h)
m = P0 + (2d )v 2 Þ v = g
a=? 2 2
Vd g - Vdg (d L - d ) g
= L = t=t Horizontal Distance x
Vd d Vdg dL ux = v; t = t; x = vt .... (i)
vd For vertical motion of liquid falling from hole
Þ t= ... (iii) v=0 C
(d L - d ) g uy = 0, Sy = h, ay = g, ty = t
Therefore, 1
S = ut + at2
2 dv 2
t2 = t1 + 2t = t1 +
(d L - d ) g 1 2 2h
Þ h = gt Þ t = ... (ii)
2d t1 g é d ù 2 g
= t1 + = t1 ê1 + ú
(d L - d ) g 2 Po
ë dL - d û
d t
Þ t2 = L 1
dL - d H
2
(b) Since the retardation is not proportional to
displacement, the motion of the ball is not simple H -h
2
harmonic. v Po
(c) If d = dL then the retardation a = 0. Since the ball strikes Reference h
the water surface with some velocity, it will continue level for P.E
x
with the same velocity in downward direction (until it From (i) and (ii)
is interrupted by some other force).
7. (a) (i) 2h g 2h
x = vy × = (3H - 4h) ´
KEY CONCEPT : g 2 g
Since the cylinder is in equilibrium in the liquid therefore
Weight of cylinder = upthrust = (3H - 4h) h ... (iii)
For finding the value of h for which x is maximum, we
mg = FT1 + FT2 where
differentiate equation (iii) w.r.t. t
FT1 and FT2 = upthrust due to lower and upper liquid dx 1
= [3H – 4h)h]–1/2 {3H – 8h}
respectively dt 2
EBD_7036
P-S-82 Topic-wise Solved Papers - PHYSICS

dx 1
Putting = 0 for finding values h for maxima/minima Þ P + rgH = r(v02 - v12 )
dt 2
1 3H But according to equation of continuity
[(3H – 4h)] –1/2 [3H – 8h] = 0 Þ h = Av
2 8 v1 = 2 0
A1
é æ 3H ö ù 3H
\ xm = ê3H - 4 çè 8 ÷ø ú 8 [From (iii)]
1 é 2 æ A2 ö ù
2
ë û
\ P + rgH = r ê v -
0 ç v0÷
ú
12H 3H 6H 3H 2 ê è A1 ø ú
= ´ = = ë û
8 8 8 4
1 2 é æ A2 ö ù
2
8. Given that ê
P + rgH = r v0 1 - ç ÷ ú
r = 1000 kg/m3, h1 = 2m, h2 = 5 m 2 ê è A1 ø ú
ë û
A1 = 4 × 10–3m2, A2 = 8 × 10–3 m2, v1 = 1 m/s Here, P + rgH = DP
Equation of continuity According to Poisseuille's equation
A1v1
= 0.5 m/s p( DP)a 4 p( DP)a 4
A1 v1 = A2 v2 \ v2 = Q= Þ h=
A2 8hl 8Ql
According to Bernoulli's theorem,
p ( P + rgH ) a 4 p 1 é æ A ö2ù
1 \ h= = ´ rv02 ê1 - ç 2 ÷ ú ´ a 4
(p1 – p2) = rg (h2 – h1) – r (v22 - v12 ) 8Ql 8Ql 2 ê è A1 ø ú
2 ë û
Where (p1 – p2) = work done/vol. [by the pressure] A2 d 2
rg (h2 – h1) = work done/vol. [by gravity forces] Where =
A1 D 2
Now, work done/vol. by gravity forces
= rg (h2 – h1) = 103 × 9.8 × 3 = 29.4 × 103 J/m3. p 1 2 é d4 ù 4
h= ´ rv0 ê1 - 4 ú ´ a
8Ql 2 ëê D ûú
1 1 é1 ù 3
And r (v22 - v12 ) = ´ 103 ê - 1ú = - ´ 103 J/m3 12. From law of continuity A1v1 = A2v2
2 2 ë4 û 8
3
= – 0.375 × 10 J/m 3 Given A1 = p × (4 × 10–3 m)2, A2 = p × (1× 10–3 m)2
\ Work done / vol. by pressure v1 A2
= 29.4 × 103 – 0.375 × 103 J/m3 = 29.025 × 103 J/m3. A1 v2
9. (a) As the pressure exerted by liquid A on the cylinder is
radial and symmetric, the force due to this pressure h
cancels out and the net value is zero.
(b) For equilibrium, Buoyant force = weight of the body
Þ hArAAg + hBrB Ag = (hA + h + hB) A rCg x
(where rC = density of cylinder)
v1 = 0.25 m/s
æ hAr A + hB rB ö
h= ç ÷ø - (hA + hB ) = 0.25cm p ´ (4 ´ 10 -3 ) 2 ´ 0.25
è r C \ v2 = 2 = 4 m/s
FBuoyant - Mg p ´ (1 ´ 10-3 )
(c) a = 1 2 2h
M gt Þ t =
Also, h =
é hAr A + rB (h + hB ) - (h + hA + hB ) rC ù 2 g
= ê ú g 2 ´ 1.25
ë rC (h + hA + hC ) û 2h
Horizontal range x = v2 × t = v2 = 4´ = 2m
g g 10
= upwards 13. The free body diagram of wire is given below. If l is the
6
10. KEY CONCEPT : When the force due to excess pressure in length of wire, then for equilibrium 2F sin q = W.
the bubble equals the force of air striking at the bubble, the F=S× l F F
bubble will detach from the ring. q
or, 2S × l × sin q = l × l × g y
4T 4T
\ rAv2 = ´A Þ R= 2 q
R rv lg q
or, S =
11. KEY CONCEPT : When the tube is not there, using 2sin q
W
Bernoulli's theorem
lg alg é yù
= êëQ sin q = a úû
1 1 \ S=
P + P0 + rv12 + rgH = rv02 + P0 2y / a 2y
2 2
Mechanical Properties of Solids and Fluids P-S-83
d
w
14. Weight of liquid of height H
G. Comprehension Based Questions
2 1. (c) Consider the equilibrium of wooden block.
pd
= ´ H ´ r ´ g ... (i) Forces acting in the downward direction are
4 H
(a) Weight of wooden cylinder P1 ´ p(2r ) 2
Let us consider a mass dm
situated at a distance x from A dx 2 r
x = p (2 r ) ´ h ´ ´ g
as shown in the figure. The 3
A dm B
centripetal force required for L
hr
the mass to rotate = (dm) xw2 = p ´ 4r 2 g
3 P1 P0 W P2
\ The total centripetal force required for the mass of (b) Force due to pressure (P1) created by liquid of height
length L to rotate h1 above the wooden block is
= P1 × p (2r)2 = [P0 + h1rg] × p (2r)2
L pd 2
= ò0 ( dm) xw 2 where dm = r ´ ´ dx Force acting on the upward direction due to pressure
4 P 2 exerted from below the wooden block and
Læ ö atmospheric pressure is
\ Total centripetal force = ò 0 çè r ´
pd 2
4
´ dx÷ ´ xw 2
ø
( ) = P2 ´ p é(2r )2 - r 2 ù + P0 ´ p (r ) 2
ë û
pd 2 L = [ P0 + ( h1 + h )rg ] ´ p ´ 3r 2 + P0 pr 2
= r´ ´ w 2 ò x dx
4 0 At the verge of rising

pd 2 L2 [ P0 + (h1 + h)rg ] ´ ( p ´ 3r 2 ) + pr 2 P0
= r´ ´ w2 ´ ... (ii)
4 2 p ´ 4r 2 hrg 5h
= [ P0 + h1rg ] ´ 4pr 2 + or, h1 =
This centripetal force is provided by the weight of liquid of 3 3
height H. 2. (b) KEY CONCEPT :
Considering equilibrium of wooden block.
From (i) and (ii) Total downward force = Total force upwards
Wt. of block + force due to atmospheric pressure =
pd 2 pd 2 w 2 ´ L2 w 2 L2 Force due to pressure of liquid + Force due to
´ H ´r´ g = r´ ´ Þ H=
4 4 2 2g atmospheric pressure
r
p (16r 2 ) ´ g + P0 p ´ 16 r 2
F. Match the Following 3
= [h2rg + P0] p [(16 – 4)r2] + P0 × 4r2
1. (c)
4
Þ h = h2
9
3. (a) When the height h2 of water level is further decreased,
h2 then the upward force acting on the wooden block
v = 2gh2 decreases. The total force downward remains the same.
This difference will be compensated by the normal
reaction by the tank wall on the wooden block. Thus
h1 the block does not moves up and remains at its original
position.
4. (c) The vertical force due to surface q
tension r
2gh2 × 2h1 = (Tcosq) × 2pr q
g R
ærö 2 pr 2 T
= 2 h1h2 = T ç ÷ ´ 2pr = .
è Rø R
If geff > g
geff = g 5. (a) When the drop is about to detach from the dropper
geff < g Weight = vertical force due to surface tension
4 3 2pr 2T
pR rg =
In all the three cases d = 2 h1h2 = 1.2 m 3 R
æ 3 r 2 ö
T 3 (5 ´ 10 -4 ) 2 ´ 0.11
If geff = 0, then no water leaks out \R4 = ç ÷ = ´ = 4.12 ´ 10 -12
è 2 rg ø 2 1000 ´ 10
\ R = 1.42 × 10–3 m
EBD_7036
P-S-84 Topic-wise Solved Papers - PHYSICS

U 4T 4 ´ 0.04 2
6. (b) We know that, T = PB - 8 = = = 4 Þ PB = 12 N / m
a RB 0.04
\ U = T × a = T × 4pR2 According to ideal gas equation
22 4
= 0.11 ´ 4 ´ ´ (1.42 ´ 10 -3 ) 2 = 2.7 ´ 10 -6 J 3
PBVB = nB RTB Þ 12 ´ p (0.04) = n B RTB … (ii)
7 3
7. (c) From principle of continuity, Dividing (ii) by (i) we get
a1v1 = a2v2 4
12 ´ p(0.04)3
pr12 v1 = pr22 v2 n
3 = B [Q TA = TB ]
4 nA
( 20 )2 ´ 5 = (1)2 ´ v2 16 ´ p(0.02)3
3
\ v2= 2000 mms–1 = 2 ms–1 nB
\ =6
8. (a) A B nA
2. Initially, the pressure of air column above water is P1 = 105
C
Nm–2 and volume V1 = (500 - H ) A , where A is the area of
cross-section of the vessel.
1 Finally, the volume of air column above water is
PA - PB = ra va2 300 A. If P2 is the pressure of air then
2
1 P2 + rgh = 105
PC - PB = rl vl2
2 \ 200
P2 + 103 ´ 10 ´ = 105
But PC = PA 1000
1 1 ra P1 = 105 N / m2
\ rl vl2 = ra va2 Þ vl = ´ va V1 = (500 - H ) A
P2
2 2 rl

500 mm
V2 = 300 A
ra
\ Volume flow rate µ H
rl

200 mm
H. Assertion & Reason Type Questions
1. (a) We know that volume flow rate (V) of an incompressible Initially Finally
fluid in steady flow remains constant. \ P2 = 9.8 ´ 10 N / m 4 2
V=a×v
As the temperature remains constant, according to Boyle’s law
where a = area of cross-section and v = velocity
Þ If v decreases a increases and vice - versa. PV1 1 = P2V2
When stream of water moves up, its speed (v) decreases
and therefore 'a' increases i.e. the water spreads out as \ 105 ´ (500 - H ) A = (9.8 ´ 10 4 ) ´ 300 A Þ H = 206 mm
a fountain. When stream of water from hose pipe moves \ The fall of height of water level due to the opening of
down, its speed increases and therefore area of cross- orifice = 206 – 200 = 6 mm
section decreases. K
Therefore statement-1 is true and statement-2 is the 3. We know that w = ...(i)
m
correct explanation of statement-1.
FL æ YA ö
Here Y = ÞF =ç ÷l
I. Integer Value Correct Type Al è Lø
Comparing the above equation with F = kl we get
1. For bubble A :
If PA is the pressure inside the bubble then æ YA ö
K =ç ÷ ...(ii)
è Lø
B YA
A From (i) & (ii), w =
ml
2 cm 4 cm
n ´ 109 ´ 4.9 ´ 10-7
8 Nm
–2
\ 140 = \ n=4
0.1 ´ 1
4T 4 ´ 0.04 2 2r12 (s - r1 ) g
PA - 8 = = = 8 Þ PA = 16 N / m
RA 0.02 VP 9h1 r 2 (s - r1 ) h2
4. 3 = 2 = 12 ´
According to ideal gas equation, VQ 2 r2 (s - r 2 ) g r2 (s - r 2 ) h1
4 9h2
PAVA = n A RTA Þ 16 ´ p(0.02)3 = n A RTA … (i)
3
For bubble B : 12 [8 - 0.8] 2
= ´ =3
If PB is the pressure inside the bubble then (0.5) [8 - 1.6] 3
2
Mechanical Properties of Solids and Fluids P-S-85

Section-B JEE Main/ AIEEE


1. (b) Small amount of work done in extending the spring by 10. (d) From the figure it is clear that liquid 1 floats on liquid 2.
dx is The lighter liquid floats over heavier liquid. Therefore
dW = k x dx we can conclude that r1 < r2
0.15 Also r3 < r2 otherwise the ball would have sink to the
800 é
\W= k ò x dx =
2 ë
(0.15)2 - (0.05)2 ù = 8 J
û
bottom of the jar.
Also r3 > r1 otherwise the ball would have floated in
0.05
liquid 1. From the above discussion we conclude that
2. (d) Work done by constant force in displacing the object
by a distance l. r1 < r3 < r2.
11. (c) In case of water, the meniscus shape is concave
1 Fl upwards. Also according to ascent formula
= ´ Force × extension =
2 2 2T cosq
h=
3. (b) From Stoke's law, rrg
viscous force F = 6phrv The surface tension (T) of soap solution is less than
hence F is directly proportional to radius & velocity. water. Therefore rise of soap solution in the capillary
tube is less as compared to water. As in the case of
2T water, the meniscus shape of soap solution is also
4. (a) p0 – pi =
R concave upwards.
\ P1 > P2 12. (c) l

hence air moves from smaller bubble to bigger bubble. A Y 3A Y


5. (a) Energy stored per unit volume,
Wire (1)
1 l/3
E = ´ stress ´ strain Wire (2)
2
As shown in the figure, the wires will have the same
1 stress 1 S2 Young’s modulus (same material) and the length of the
\ E= ´ stress ´ = .
2 Y 2 Y wire of area of cross-section 3A will be l/3 (same volume
as wire 1).
6. (c) Water fills the tube entirely in gravity less condition
i.e., 20 cm. For wire 1,
7. (a) Case (i) F/A
Y= ...(i)
At equilibrium, T = W D x/l
For wire 2 ,
W/A T
Y= .....(1) F '/ 3 A
l/L Y= ...(ii)
T T
Dx /( l / 3)
Case (ii) At equilibrium T = W F l F' l
From (i) and (ii) ,´ = ´ Þ F ' = 9F
W/A W/A A Dx 3 A 3Dx
\Y = ÞY = W 13. (b) Oil will float on water so, (2) or (4) is the correct option.
l/2 l/L W W But density of ball is more than that of oil,, hence it will
L/2
sink in oil.
Þ Elongation is the same. 14. (c)
2 r 2 ( d1 - d 2 ) g
8. (c) Terminal velocity, vT = 30°
9h
T
vT (10.5 - 1.5) 9 T cos 15°
2
= Þ vT = 0.2 ´ = 0.1 m/s
0.2 (19.5 - 1.5) 2 18
9. (a) The condition for terminal speed (vt) is Fe
Weight = Buoyant force + Viscous force T sin 15°

Vg (r1 - r2 )
\ V r1 g = V r2 g + kvt2 \ vt = mg
k
Fe = T sin15° ; mg = T cos15°
EBD_7036
P-S-86 Topic-wise Solved Papers - PHYSICS

Fe sLAg
Þ tan 15° = ...(1) Mg -
mg Þ x0 = 2 = Mg æ1 - LAs ö
ç ÷
k k è 2M ø
In liquid , Fe ' = T 'sin15° ...(A)
19. (d) When radius is decrease by DR,
mg = FB + T 'cos15°
4pR 2 DRrL = 4pT[R 2 - (R - DR) 2 ]

30° Þ rR 2 DRL = T[R 2 - R 2 + 2RDR - DR 2 ]


FB T¢
Þ rR 2 DRL = T2RDR [ DR is very small]
T¢ cos 15°
2T
F¢e ÞR=
T¢ sin 15° rL
20. (None) None of the given option is correct.
mg When the bubble gets detached,
FB= V (d - r) g = V (1.6 - 0.8) g = 0.8 Vg Buoyant force = force due to surface tension
m 0.8 mg mg
= 0.8 g= =
d 1.6 2
mg
\ mg = + T 'cos 15°
2
mg R
Þ = T 'cos 15° ...(B) q
2
2 Fe'
From (A) and (B), tan 15° = … (2)
mg rq
T×dl
From (1) and (2)
Fe 2 Fe' Fc Force due to excess pressure = upthrust
= Þ Fe = 2 Fe' Þ Fc¢ =
mg mg 2 2T
Access pressure in air bubble =
15. (c) W = T × change in surface area R
W = 2T4p[(52) – (3)2] × 10–4 2T 4p R 3
= 2 × 0.03 × 4p [25 – 9] × 10–4 J = 0.4p × 10–3 J (p r 2 ) = rw g
R 3T
= 0.4p mJ
16. (c) From Bernoulli's theorem, 2 R 4 rw g
Þ r2 =
1 1 3T
P0 + rv12rgh = P0 + rv22 + 0
2 2 2rw g
Þ r = R2
v2 = v12 + 2 gh = 0.16 + 2 ´ 10 ´ 0.2 = 2.03 m/s 3T
From equation of continuity
A2v2 = A1v1
21. (a) (54–x)
D2 D2 54 cm P
p 2 ´ v2 = p 1 v1
4 4 8 cm x
Þ v1 3.55 × 10–3 m
D1 = D2 =
v2
17. (d) At equilibrium,
2Tl = mg Hg
mg 1.5 ´ 10-2 1.5
T= = - 2
= = 0.025 N/m = 0.025Nm
2l 2 ´ 30 ´ 10 60
18. (c) From figure, kx 0 + FB = Mg Length of the air column above mercury in the tube is,
L KX0 P + x = P0
kx 0 + s Ag = Mg Þ P = (76 – x)
2 FB
Þ 8 × A × 76 = (76 – x) × A × (54 – x)
[Q mass = density × volume]
\ x = 38
L Mg Thus, length of air column = 54 – 38 = 16 cm.
Þ kx 0 = Mg - s Ag
2
9
Heat & Thermodynamics and Gases P-S- 87

Heat & Thermodynamics and Gases

Section-A : JEE Advanced/ IIT-JEE


A 1. 4 cal. 2. Partly solid and partly liquid. 3. 2T 4. 0°C 5. 5803K

4pR 2 KT P´t
6. 1.71rrc 7. t= 8. Lfusion = 9. 300 K
P m
10. 0.628 11. 60°C 12. 192º 13. 5.5 min.
B 1. F 2. F 3. F 4. F 5. T 6. T
7. F 8. F
C 1. (d) 2. (c) 3. (b) 4. (a) 5. (b) 6. (a) 7. (b) 8. (d)
9. (c) 10. (c) 11. (d) 12. (b) 13. (d) 14. (c) 15. (d) 16. (a)
17. (c) 18. (a) 19. (b) 20. (b) 21. (a) 22. (b) 23. (a) 24. (a)
25. (a) 26. (a) 27. (c) 28. (c) 29. (b) 30. (b) 31. (b) 32. (c)
33. (d) 34. (b) 35. (a) 36. (d) 37. (d) 38. (a) 39. (c) 40. (b)
41. (c) 42. (d) 43. (a) 44. (d) 45. (d) 46. (a) 47. (d) 48. (a)
49. (b) 50. (c) 51. (a)
D 1. (a) 2. (b) 3. (a) 4. (c) 5. (a, b, c, d) 6. (d) 7. (b)
8. (a, b, d) 9. (a, b) 10. (b) 11. (b) 12. (d) 13. (b, c) 14. (d)
15. (b, d) 16. (c) 17. (c, d) 18. (a) 19. (a, c, d) 20. (b, d) 21. (b, d)
22. (a, b) 23. (a, c, d) 24. (a, b, c, d) 25. (a, b, c) 26. (b, c)
w2 - w1 b( w0 - w1 )
E 1. + 2. TB = 30°C, TC = TD = 20°C 3. Same
( w0 - w2 )(t2 - t1 ) ( w0 - w2 )
5. 817 mmHg 7. 12.96 m/s 8. –973.1 J
9. hollow sphere 10. 1.97 × 1027, 35.6 m/s 11. 12.9 T0, 2.25 T0, –15.58T0
12. 83.75 cm Hg 13. 75.4 cm 14. 675 K, 3.6 × 106 N/m2
15. (ii) 113 l, 0.44 × 105 N/m2 (iii) 12450 J 16. 800 K, 720 J 17. (i) 5 (ii) 1.25 PV
18. (b) 0.58 RTA, 0.58 RTA 19. (a) 1153 J (b) 1153J (c) Zero 20. (i) 1870 J (ii) –5298 J (iii) 500 K
21. (i) 765 J (ii) 10.82% 22. Mass of Neon = 4gm, mass of Argon = 24gm
-5
23. (a) 2 mole (b) 400.03 m/s (c) 1/6 (d) -8.27 ´10 V 24. (i) 189 K (ii) –2767 J (iii) 2767 J
25. 0.0122 Kg 26. TB = 909 K, TD = 791 K, 61.4% 27. 6.67 × 10–5 per °C
é -2 KAt1 ù
5 P0V0 25 P0V0
28. (a) P0V0 (b) - P0V0 , 3P0V0 (c) (d) 29. ëê 300 + 12.5e CL ûú Kelvin
2 2 8R

3 é æ V ö2/3ù 3 é æ V ö 2/3ù 5 / 3 -2 / 3
1 1 ê1 - ç
1 ú , Q - PV
1 1 ê1 - ç
1 ú PV1 1 V2 Q
30. (b) 2 PV ê è V2 ø ÷ ú 2 ê è V2 ø ÷ ú , + 31. (a) 1200R (b) –2100R, 831.6 R
ë û ë û 2R 3R
32. 0.495 Kg 33. (a) 600 K (b) 1500R, 831.8R, –900R, –831.8R (c) 600R
34. (a) 160K (b) 3.312 × 10–21 J (c) 0.3012 gm

mv02 4 2/5
m, 400 æç ö÷ K
4
35. 36. (a) 595 W/m2 (b) 419.83 K 37. 38. Rate of heat produced µ r 5
3R 3 è3ø

é 4eslTs3 ù
39. ê1 + ú 40. g = 2a 41. 69.99°C, 0.0499J, 19999.95 J 42. 273K or 0°C
êë K úû

F 1. (A)-(q); (B)-(p, s); (C)-(s); (D)-(q, r) 2. (A)-(q); (B)-(p, r); (C)-(p, s); (D)-(q, s)
3. (A)-(p, r, t); (B)-(p, r); (C)-(q, s); (D)-(r, t) 4. (a)
EBD_7036
P-S- 88 Topic-wise Solved Papers - PHYSICS

G 1. (a) 2. (d) 3. (c) 4. (d) 5. (b) 6. (b) 7. (d) 8. (d)


H. 1. (b)
I. 1. 9 2. 8 3. 9 4. 4 5. 3 6. 2
7. 2 8. 9
Section-B : JEE Main/ AIEEE
1. (a) 2. (b) 3. (b) 4. (a) 5. (a) 6. (c) 7. (d) 8. (c)
9. (c) 10. (a) 11. (a) 12. (d) 13. (c) 14. (b) 15. (d) 16. (d)
17. (c) 18. (d) 19. (b) 20. (a) 21. (d) 22. (b, c) 23. (d) 24. (b)
25. (d) 26. (a) 27. (b) 28. (c) 29. (a) 30. (b) 31. (c) 32. (d)
33. (b) 34. (a) 35. (a) 36. (a) 37. (b) 38. (a) 39. (a) 40. (a)
41. (c) 42. (b) 43. (c) 44. (a) 45. (d) 46. (a) 47. (d) 48. (a)
49. (a) 50. (c) 51. (b) 52. (c) 53. (a) 54. (c) 55. (c) 56. (d)
57. (d) 58. (a) 59. (a) 60. (c) 61. (c) 62. (d)

Section-A JEE Advanced/ IIT-JEE


A. Fill in the Blanks 6. The energy emitted per second when the temperature of the
copper sphere is T and the surrounding temperature T0
n1Cv1 + n2 CV2
1. Cv =
n1 + n2
= s (T4 – T04 ) × A = s .T4 A [QT0 = 0] ...(i)
We know that
3 5
1´ R + 1´ R dQ dT
= 2 2 = 2R dQ = mcdT Þ = mc ... (ii)
1+1 dt dt
2. AB represent a process when physical state changes from From (i) and (ii)
solid to liquid and the temperature remains unchanged. dT
Since P is a point between A and B, therefore the material is sT4A = mc
dt
partly solid and partly liquid.
3. PV = RT (Ideal gas equation) 4 3
p r cdTr´
mcdT 3 é 4 3ù
Þ P=
RT Þ dt = = êëQ m = r ´ 3 pr úû
V
... (i) sT 4 A s T 4 ´ 4p r 2
Given that VP2 = const ... (ii)
rrc dT
From (i) and (ii) Þ dt =
3s T 4
T2
\ = const. Integrating both sides
V
100
T12 T22 V2 2V t rrc 100 dT rrc é 1 ù
\ =
V1 V2
Þ T2 = T1
V1
=T
V
= 2T
ò0 dt =
3s ò 200 T 4 3s êë 3T 3 úû 200
= -

4. The heat required for 100 g of ice at 0° C to change into


water at 0°C = mL = 100 × 80 ×4.2 = 33,600 J ... (i) rrc é 1 1 ù
The heat released by 300g of water at 25°C to change its t= - ê - ú
9s ëê (100) 3
(200)3 ûú
temperature to 0°C = mcDT = 300 × 4.2 × 25 = 31,500 J ... (ii)
Since the energy in eq. (ii) is less than of eq. (i) therefore the
7rrc 7rrc
final temperature will be 0°C. t= » = 1.71rrc
5. The energy received per second per unit area from Sun at a (72 ´ 10 ) s 6
72 ´ 10 (5.67 ´ 10 -8 )
6

distance of 1.5 × 1011 m is 1400 J/sm2. The total energy 7. KEY CONCEPT :
released by Sun/per second. When the spherical shell is thin, t << R. In this case,
= 1400 × 4p × (1.5 × 1011)2. The rate of flow of heat from the sphere to the surroundings
\ The total energy released per second per unit surface
area of the Sun K (4pR 2 )T
P=
1400 ´ 4p ´ (1.5 ´ 1011 ) 2 t
= R
4 p ´ (7 ´ 108 ) 2 where T is the temperature difference t
This energy E is also equal to E = sT4 and t is the thickness of steel then P
1
é 1400 ´ 4 p ´ (1.5 ´ 1011 ) 2 ù4 4pR 2 KT K
\ T= ê -8
ú » 5803 K t=
8 2
êë 4 p ´ (7 ´ 10 ) ´ 5.67 ´ 10 úû P
Heat & Thermodynamics and Gases P-S- 89

8. Since P joules per second of heat is supplied to keep the If t is the time taken for the ice to melt, we will have
substance in molten state, it means that the substance in é Eù
the molten state at its melting point releases P Joule of heat (280)t = 9.24 × 104 J êQ P = t ú
ë û
in one second.
9.24 ´ 104
t= s = 330s º 5.5min
280
P Joule/sec.
B. True/ False
P Joule/second 3RT
M 1. KEY CONCEPT : c =
M
1
At the same temperature c µ
M
i.e., dependent on molar mass and hence rms speed c will be
different for different ideal gases.
1
The power is turned off then the heat input becomes zero. 2. For a particular temperature T, V µ
But heat output continues. It takes t seconds for the P
substance to solidify (given). Therefore total heat released Volume is greater for pressure P1
in t seconds = P × t = mLfusion \ p1 < p2
1
P ´t 3. For a particular termperature Crms µ
Lfusion = M
m
i.e., Crms will have different values for different gases.
9. In this expansion, no work is done because the gas expands
in vacuum. Therefore DW = 0 g 1 RT
As the process is a adiabatic, Q = 0. From first law of (C H 2 )1 M1 g1 M 2
thermodynamics, DU = 0 i.e. temperature remains constant. 4. = = ´
10. For isothermal expansion (C H e ) 2 g 2 RT g 2 M1
M2
P
P × V = Pi × 2V Þ Pi =
2 7/5 4 7 3 42
For adiabatic expansion = ´ = ´ ´2 =
5/3 2 5 5 25
P P 5. The slope of P-V curve is more for adiabatic process than
PVg = Pa × (2V)g Þ Pa = g
=
1.67 for isothermal process. From the graph it is clear that slope
2 2
for B is greater than the slope for A.
Pa P 2 2 6. Cp – Cv = R
\ = ´ = = 0.628
Pi 2 1.67 P 2 1.67 7. We know that
11. The heat transferred through A per second 3RT 3R (2T )
Q1= K1 A (100 – t) v= then v' =
M M /2
The heat transferred through B per second
\ v' = 2v
Q2= K2 A (t – 0)
8. Energy radiated per second by the first sphere
At steady state K1 A (100 – t) = K2A (t – 0)
E1 = esT 4A = es (4000)4 × 4p × 11 × 1
Þ 300 (100 – t) = 200 (t – 0) Þ 300 – 3t = 2t Þ t = 60° C
12. The movable stopper will adjust to a position with equal = 1024 × p × 1012 × es
pressure on either sides. Applying ideal gas equation to the Energy radiated per second by the second sphere
two gases, we get E2 = es × (2000)4 × 4p × 4 × 4
m m = 1024 p × 1012 × es
PV1 = n1RT = RT , PV2 = n2RT = RT E1 = E2
M1 M2
V2 M1 32 8 C. MCQs with ONE Correct Answer
Hence, = = =
V1 M 2 28 7 1. (d) Note : At constant volume, Charle's law is used.
360° 2. (c) W1 = mg – Vda g
a= ´ 8 = 192°
(8 + 7) da g
13. Solar power received by earth = 1400 W/m 2 W2 = mg – V'd'a g = mg – V (1 + 50 gb)
(1 + 50 g a )
Solar power received by 0.2 m2 area
= (1400 W/m2) (0.2 m2) = 280 W é 1 + 50 g b ù
Mass of ice = 280 g = 0.280 kg = mg – V dag ê ú
Heat required to melt ice ë1 + 50 g a û
= (0.280) (3.3 × 105) = 9.24 × 104 J Given gb < ga
EBD_7036
P-S- 90 Topic-wise Solved Papers - PHYSICS

1 + 50 g b or, mc DT = esAT4 Dt
\ 1 + 50 gb < 1 + 50 ga \ <1
DT esAT 4 esT 4 é æ 3m ö
2/3ù 1/ 3
1 + 50 g a æ 1ö
or, = = êp ú= kç ÷
\ W2 > W1 or W1 < W2 Dt mc mc ê èç 4 prø÷ úû è mø
ë
3. (b) q A - q B = 36°C (Given) 1/ 3 1/ 3
KA = 2KB (Given) DT1 / Dt1 æ m2 ö æ 1ö
\ = =ç ÷
DT2 / Dt2 èç m1 ø÷ è 3ø
KA KB
q + q 9. (c) Average translational kinetic energy of an ideal gas
l A l B
qC = 3
KA KB
+ molecule is kT which depends on temperature only..
l l 2
Therefore, if temperature is same, translational kinetic
qA qC qB
energy of O2 and N2 both will be equal.
KA KB nRT
10. (c) PV = nRT or P = or P µ T
V
l l (Q V and n are same.)
Therefore, if T is doubled, pressure also becomes two
2q A + q B 2q A + q A - 36 3(q A - 12) times, i.e.., 2P.
\ qC = = =
3 3 3 11. (d) The energy radiated per second by a black body is
\ q A - qC = 12 given by Stefan's Law
4. (a) The work done during the cycle = area enclosed in the E
= sT 4 ´ A , where A is the surface area.
curve t
n1 + n 2 n n
5. (b) = 1 + 2 E
g -1 g1 - 1 g 2 - 1 = sT 4 ´ 4pr 2 ( Q For a sphere, A = 4 p r2)
t
2 5
Here, n1 = n2 = 1, g1 = ,g = E
5 2 3 Case (i) : = 450, T = 500 K, r = 0.12 m
6. (a) For an ideal gas PV = nRT t
Þ Coefficient of volume expansion \ 450 = 4ps (500)4 (0.12)2 ... (i)
æ DV ö nR E
Case (ii) : = ?, T = 1000 K, r = 0.06 m
ç ÷ = = Constant t
è DT øp P
E
Note : Average translation K.E. for O2 is
3
kT \ = 4 p s (1000)4 (0.06) 2 ... (ii)
2 t
(Three degrees of freedom for translational motion). Dividing (ii) and (i), we get
Now decrease in pressure increases the volume.
E / t (1000)4 (0.06) 2 2 4
Þ It increases mean free path of the molecules. Also = = =4
average K.E. does not depend on the gas, so molecules 450 (500)4 (0.12)2 22
of each component of mixture of gases have same
average translational energy. E
Þ = 450 × 4 = 1800 W
7. (b) Heat flow from B to A, A to C and C to B (for steady t
state condition, DQ Dt is same) 12. (b) When a enclosed gas is accelerated in the positive
x-direction then the pressure of the gas decreases along
DQ k A DT A the positive x-axis and follows the equation
Where = T
Dt l DP = – r a dx
2a
a where r is the density and a the acceleration of the
æ DT ö container.
For sides AC and CB ç ÷
è 2a ø AC TC The result will be more pressure on the rear side and
B a C
less pressure on the front side.
æ DT ö 2T
13. (d) The internal energy of n moles of a gas is
=ç ÷
è a øCB
1
U= nFRT
T - Tc T - 2T 2
Þ = c Þ T - Tc = 2 Tc - 2T
2a a where F = number of degrees of freedom.
Tc 3 Internal energy of 2 moles of oxygen at temperature T is
Þ 3T = Tc ( 2 + 1) Þ =
T 2 +1 1
U1 = ´ 2 ´ 5RT = 5RT [F = 5 for oxygen molecule]
8. (d) According to Stefan's law 2
DQ = esAT4 Dt Internal energy of 4 moles of argon at temperature T is
also, DQ = mc DT
Heat & Thermodynamics and Gases P-S- 91

1 Dividing eq. (ii) by eq. (i)


U2 = ´ 4 ´ 3RT = 6RT
2 q
2= Þ q = 60°
Total internal energy = 11 RT 90 - q
21. (a) From the first law of thermodynamics
VN g N 2 M He 7/5´ 4 = 3
14. (c) = = dQ = dU + dW
VHe g He M N 2 5 / 3 ´ 28 5 Here dW = 0 (given)
15. (d) Here TVg –1 = constant \ dQ = dU
Now since dQ < 0 (given)
5 \ dQ is negative
As g = , hence TV2/3 = constant
3 Þ dU = – ve Þ dU decreases.
Þ Temperature decreases.
Now T1 L12 / 3 = T2 L22 / 3 (Q V µ L);
22. (b) For adiabatic process PV g = constant
2/3
T æL ö Cp
Hence, 1 = ç 2 ÷ Also for monoatomic gas g = = 1.67
T2 è L1 ø CV
16. (a) for diatomic gas g = 1.4
Temp.

100o 100o
Water Steam
P
0o 0o
Ice Water 1
2
–10o Heat supplied
Ice
1. The temp. of ice changes from –10°C to 0°C. V
2. Ice at 0°C melts into water at 0°C.
3. Water at 0°C changes into water at 100°C. Since, gdiatomic < gmono atomic
4. Water at 100°C changes into steam at 100°C. \ Pdiatomic > Pmono atomic
17. (c) We know that V/T = constant Þ Graph 1 is for diatomic and graph 2 is for mono
atomic.
V + DV V DV 1 23. (a) For equilibrium in case 1 at 0° C
\ = or =
T + DT T V DT T Upthrust = Wt. of body
18. (a) Work done is equal to area under the curve on PV \ K1Vd2g = Vd1g
diagram.
d1
19. (b) According to Wien's law, lT = constant Þ K1 = ... (i)
From graph l1 < l3 < l2 d2
\ T1 > T3 > T2. K1Vd2g K2Vd
' '2g
20. (b) Let q°C be the temperature at B. Let Q is the heat
d1 d '1
flowing per second from A to B on account of V V'
temperature difference.
KA (90 - q ) d2 Vd1g d'2 V'd'1g
\ Q= ... (i)
l At 0oC At 60oC
A
For equilibrium in case 2 at 60° C
90oC Note : When the temperature is increased the density
Q will decrease.
D B \ d1' = d1 (1 + gFe × 60)
qoC and d2' = d2 (1 + gHg × 60)
0oC 2Q 0oC
Again upthrust = Wt. of body
Q
\ K2V'd2'g = V'd1'g
C
90oC é d2 ù d1
By symmetry, the same will be the case for heat flow \ K2 ê ú=
êë 1 + g Hg ´ 60 úû 1 + g Fe ´ 60
from C to B.
\ The heat flowing per second from B to D will be
é 1 + g Fe ´ 60 ù d K 1 + g Fe ´ 60
KA (q - 0) \ K2 ê ú= 1 Þ 1 =
2Q = ... (ii) êë1 + g Hg ´ 60 úû d2 K 2 1 + g Hg ´ 60
l
EBD_7036
P-S- 92 Topic-wise Solved Papers - PHYSICS
24. (a) For cyclic process; lA< lB < lC
Qcyclic = WAB + WBC + WCA = 10 J + 0 + WCA = 5 J So, TA > TB > TC
Þ WCA = – 5 J ì C C C ü
25. (a) PV = constant. Differentiating, íQ TA = ,T =
-7 B
,T =
-7 C ý
î 3 ´ 10 4 ´ 10 5 ´ 10 -7 þ
PdV æ 1 ö æ dV ö æ 1 ö Q = esAT4
= -V ; b = - ç ÷ ç = Þ b×P=1
dP è V ø è dP ÷ø çè P ÷ø e = 1 black body
\ Graph between b and P will be a rectangular \ Q = sAT4
hyperbola. C4
26. (a) Note : According to Kirchoff's law, good absorbers are \ QA = s.p (2 × 10–2)2 ×
27 ´ 10-28
good emitters as well.
At high temperature (in the furnace), since it absorbs C2
QB = s.p (4 × 10–2)2 ×
more energy, it emits more radiations as well and hence 64 ´ 10 -28
is the brightest.
C2
27. (c) The graph shows that for the same temperature and QC = s.p (6 × 10–2)2 ×
difference (T2 – T1), less time is taken for x. This means 625 ´ 10 -28
the emissivity is more for x. According to Kirchoff's From comparison QB is maximum.
law, a good emitter is a good absorber as well. 32. (c) Q = mc DT
T Þ Q = mc (T – t0) ... (i)
\ From 50 K to boiling temperature, T increases
T2 linearly.
During boiling, equation is
y Q = mL
x Temperature remains constant till boiling is complete
T1
After that, again eqn. (i) is followed and temperature
increases linearly.
33. (d)
t
28. (c) The lengths of each rod increases by the same amount o
100 C K A
o
0C
l
\ Dl a = Dl s K A
Þ l1a a t = l 2 a s t
l 2 aa l a
Þ = Þ 2 +1 = a +1
l1 a s l1 as o
100 C K A
o
0C
l l
l 2 + l1 a a + a s l1 as
Þ = Þ =
l1 as l1 + l 2 a a + a s
29. (b) If we study the P – T graph we find AB to be a K 2 A (100)
q1 =
isothermal process, AC is adiabatic process given. Also l
for an expansion process, the slope of adiabatic curve
is more (or we can say that the area under the P – V A (100) KA (100)
q2 = =
graph for isothermal process is more than adiabatic l l 2
+
process for same increase in volume). K K
Only graph (b) fits the above criteria. q2 KA (100) l 1
30. (b) Heat required to convert 5 kg of water at 20°C to 5 kg of \ = ´ =
water at 0°C q1 2l K 2 A(100) 4
= mCw DT = 5 × 1 × 20 = 100 kcal 34. (b) In the first process W is + ve as DV is positive, in the
Heat released by 2 kg. Ice at – 20°C to convert into 2 kg second process W is – ve as DV is – ve and area under
of ice at 0°C the curve of second process is more
= mCice DT = 2 × 0.5 × 20 = 20 k cal. \ Net Work < 0 and also P3 > P1.
How much ice at 0°C will convert into water at 0°C for
giving another 80 kcal of heat
Q = mL Þ 80 = m × 80 P3
Þ m = 1 kg
Therefore the amount of water at 0°C P1
= 5 kg + 1kg = 6 kg
Thus, at equilibrium, we have, [6 kg water at 0°C + 1kg P2
ice at 0°C].
31. (b) We know that
lmT = Constant V1 V2
Heat & Thermodynamics and Gases P-S- 93

35. (a) According to Wein's displacement law V


lm × T = constant = constt Þ V = k T3 where k = constant
T3
Here, l m3 < l m2 < l m1 dV dT
Þ =3
Þ T3 > T2 > T1 V T
æ 3ö
Þ dv = ç ÷ VdT ...(i)
è Tø
We know that change in volume due to thermal
2
3
expansion is given by dV = V g dT ...(ii)
1
T1 T2
T3
where g = coefficient of volume expansion.
From (i) and (ii)
The temperature of Sun is higher than that of welding æ 3ö 3
arc which in turn is greater than tungsten filament. V g dT = ç ÷ VdT Þ g =
è Tø T
36. (d) Heat transfer of glass bulb from filament is through 42. (d) A real gas behaves as an ideal gas when the average
radiation. A medium is required for convection process. distance between the gas molecules is large enough
As a bulb is almost evacuated, heat from the filament is so that (i) the force of attraction between the gas
transmitted through radiation. molecules becomes almost zero (ii) the actual volume
37. (d) In this question the given options are wrong as all the of the gas molecules is negligible as compared to the
four options contain e in place of s. occupied volume of the gas.
When a spherical body is kept inside a perfectly block The above conditions are true for low pressure and
body then the total heat radiated by the body is equal high temperature.
to that of the black body. 43. (a) Initially
38. (a) 1 Calorie is the amount of heat required to raise
V1 = 5.6l, T1 = 273K, P1 = 1 atm,
temperature of 1 gm of water from 14.5°C to 15.5°C at
760 mm of Hg. 5
g = (For monoatomic gas)
39. (c) As shown in the figure, the net 3
heat absorbed by the water to 160 J/s
raise its temperature 5.6l 1
The number of moles of gas is n = =
= (1000 – 160) = 840 J/s 22.4l 4
Now, the heat required to raise 2l Finally (after adiabatic compression)
the temperature of water from 27° V2 = 0.7l
C to 77°C is
Q = mc Dt = 2 × 4200 × 50 J For adiabatic compression T1V1g-1 = T2V2 g-1
Therefore the time required g -1 5
1000 J/s -1
Q 2 ´ 4200 ´ 50 æ V1 ö æ 5.6 ö 3
t= = = 500 sec = 8 min 20 sec \ T2 = T1 ç ÷ = T1 ç ÷ = T1 (8)2/3 = 4T1
840 840 è V2 ø è 0.7 ø
é hrg ö ù We know that work done in adiabatic process is
40. (b) The force is ê æç P0 + ÷ø ´ (2 R ´ h)ú - 2 RT
ë è 2 û nRDT 9
W= = RT1
g -1 8
P0
2R vrms (helium) Margon 40
44. (d) = = = 10 » 3.16
FT vrms (argon) Mhelium 4
h/2
h 45. (d) The heat is supplied at constant pressure.
P0 + rg
2 Therefore,
Q = n Cp Dt
5 5
= 2 éê R ùú ´ Dt = 2 × × 8.31 × 5 = 208 J
ë2 û 2
Note : In the first part the force is created due to
æ 5 ö
pressure and in the second part the force is due to çQ Cp = R for mono-atomic gas ÷
surface tension T. è 2 ø
\ Force = 2P0Rh + Rrgh2 – 2RT 46. (a) Given HI = HII
41. (c) PT2 = constant (given)
ADT (A + A) DT
PV k1 ´ t1 = k 2 ´ t2
Also for an ideal gas = constt l+l l
T
From the above two equations, after eliminating P. k t
\t2 = 1 ´ 1 ...(i)
k2 4
EBD_7036
P-S- 94 Topic-wise Solved Papers - PHYSICS
Where k1 and k2 are the equivalent conductivities in
25 - 100 75 ´ 3
configuration I and II respectively. \ W = ( 3 - 5 ) / 3 = 2 = 112.5J
For configuration I :
l+l l l 2 3 From first law of thermodynamics q = DU + w \ DU = – w
= + \ =
k1 k 2k k1 2k \ DU = – 112.5 J
4k Now applying first law of thermodynamics for process
\ k1 = ...(ii) 1 & 2 and adding q1 + q2 = DU + Pi(Vf – Vi)
3
For configuration II : = – 112.5 + 105 (8 – 1) × 10– 3 = 587.55
k2(A+A) = kA + 2kA 51. (a) The heat flow rate is same
3k T
\ k2 = ...(iii) M 2K
Q
2 P K
10°C 400°C
4k R S
9 x H H
From (i) , (ii) and (iii) t 2 = 3 ´ = 2sec dx
3k 4
2 KA ( 400 - T ) 2KA ( T - 10 )
option (a) is correct \ =
47. (d) P1M1 = d1RT ; P2M2 = d2RT l l
\ T = 140°C
P M d
\ 1´ 1 = 1 The temperature gradient access Pd is
P2 M 2 d 2
dT 140 - 10
4 2 d1 d 8 = \ dt = 130 dx
´ = \ 1 = dx 1
3 3 d2 d2 9 Therefore change temperature at a cross-section M
48. (a) In steady state distant ‘x’ from P is
Energy lost = Energy gained DT = 130 x

( )
s T 4 - T04 ´ 4pR 2 = I pR 2 ( ) Extension in a small elemental length ‘dx’ is
dl = dxa DT = dx a (130x )
\ 5.7 ´ 10 -8 éT - ( 300 ) ´ 4 = 912
4 4ù
ëê úû 1

\ T = 330 K \ ò dl = 130a ò xdx


0
mc D T V rc DT
49. (b) Pheater – Pcooler = = 1
t t \ Dl = 130 × 1.2 × 10–5 × = 78 × 10 – 5 m
2
0.12 ´ 1000 ´ 4.2 ´103 ´ 20
\ (3000 – Pcooler) = D. MCQs with ONE or MORE THAN ONE Correct
3 ´ 60 ´ 60
\ Pcooler = 2067W 3RT 3 ´ 8.314 ´ 298
5 1. (a) M = = ´ 1000 = 2gm
C2rms 1930 ´ 1930
50. (c) P3V5 = constant Þ PV5/3 = constant Þ g =
3 \ The gas is H2
Þ monoatomic gas
Q2 nCv DT 1 Q
For adiabatic process 2. (b) = = Þ Q2 = 1
Q1 nC p DT g g
P PV 1
i i
70
Þ Q2 = = 50 cal
1.4
2 3. (a) Heat lost by steam = Heat gained by
adiabatic (water + calorimeter)
mL + m × c × (100 – 80) = 1.12 × c × (80 – 15)
PfVf m [540 + 1 × 20] = 1.12 × 1 × 65
m = 0.13 kg
4. (c) Total transfer of heat per second through the composite
V = Heat transfer per second from material with thermal
1 conductivity K1 + Heat transfer per second from
´105 ´ 8 ´10-3 - 105 ´ 10-3
Pf Vf - Pi Vi 32 material with thermal conductivity K2.
W= =
1- g 5
1-
3
Heat & Thermodynamics and Gases P-S- 95

Þ P2V0 – PP0V0 = – P0RT ... (ii)


K2 Relation between P and T is the equation of a parabola.
R RT
K1 Also, P =
2R V
From (i) and (ii)
l P P0 2 P0
RT
= - 0 V + P0 Þ T = - V + V
KADT K1 A1DT K 2 A2 DT V V0 V0 R R
= +
l l l Note :The above equation is of a parabola
or, Kp (2R)2 = K1p R2 + K2p [(2R)2 – R2] (between T and V)
K + 3K 2 Differentiating the above equation w.r.t.V, we get
or, K= 1
4 dT P P
5. (a, b, c, d) = - 0 ´ 2V + 0
dV V0 R R
(a) For all thermal processes.
V0
DU = nCv DT where DT = (T2 - T1 ) For
dT
=0, V =
(b) According to first law of thermodynamics. dV 2
DQ = DU + DW d 2T -2P0
Also, = = – ve
In an adiabatic process DQ = 0. dV 2 V0 R
or, 0 = DU + DW
V0
or, DU = DW Þ V= is the value for maxima of temperature
2
(c) In the isothermal process, DT = 0. Also, PA VA = PB VB
Q DU = 0 Þ TA= TB (From Boyle's law)
(d) In the adiabatic process, DQ = 0. Þ In going from A to B, the temperature of the gas first
V0
DU nCv DT Cv 1 1 5 increase to a maximum (at V = ) and the decreases and
6. (d) = = = = = 2
Q p nC p DT C p g 7 / 5 7 reaches back to the same value.
7. (b) Note : All three vessels are at same temperature.
According to Maxwell's distribution of speed, average 9. (a, b)
Energy emitted per second by body A = eAs TA4 A
speed of molecules of a gas v µ T .
\ The velocity of oxygen molecules will be same in Energy emitted per second by body B = eBs TB4 A
A as well as C.
8. (a, b, d) Given that power radiated are equal eAs T A4 A = eBs TB4 A
The work done by the gas in the process A to B exceeds the 1/ 4
æ eA ö
work that would be done by it if the system were taken from Þ TB = ç ÷ ´ TA = 1934 K
A to B along the isothermal line. This is because the work è eB ø
done is the area under the P-V indicator diagram. As shown 1
by the diagram the area under the graph in first diagram will According to Wein's displacement law l m µ
T
be more than in second diagram. When we extrapolate the Since temperature of A is more therefore (lm)A is less
graph shown in figure (i). Let P0 be the intercept on P-axis \ (lm)B – (lm)A = 1 × 10–6 m (given) ... (i)
and V0 be the intercept on V-axis. The equation of the line Also according to Wein's displacement law
AB can be written as (lm)A TA = (lm)B TB
P0 (l m ) A TB 5802
P= - V + P0 ... (i) [Q y = mx + c ] Þ = = ... (ii)
V0 (l m ) B TA 1934
On solving (i) and (ii), we get
P
P lB = 1.5 × 10–6 m.
Po
isothermal
P A (Vrms )1 T V 120
= 1 Þ
process
P 10. (b) =
(Vrms )2 T2 (V )
rms 2 480
p B p
2 V 1
2
Þ = Þ (Vrms)2 = 2V
V 2V Vo V
(Vrms )2 2
V 2V V
Fig (i) Fig (ii) 11. (b) For an isothermal process; PV = constant
On differentiating, we get ; PdV + VdP = 0
P0 RT RT dP
\ P= - ´ + P0 [Q V = ] Þ P= =K (Bulk modulus)
V0 P P dV / V
EBD_7036
P-S- 96 Topic-wise Solved Papers - PHYSICS
12. (d) A is free to move, therefore heat will be supplied at 16. (c) Container A Container B
constant pressure mA mB
\ DQA = nCpDTA ... (i) PAV = RT PBV = RT
B is held fixed, therefore heat will be supplied at constant M M
volume. mA mB
\ DQB = nCvDTB ... (ii) P'A (2V) = RT P'B (2V) = RT
M M
But DQA = DQB (given)
æ Cp ö m A RT m A RT
Þ PA – P'A = -
\ nCpDTA = nCvDTB \ DTB = ç ÷ DTA MV M (2V )
è Cv ø
= g (DTA) [g = 1.4 (diatomic)] m A RT
= (1.4) (30 K) Þ DP = ... (i)
2 MV
\ DTB = 42 K
13. (b, c) mB RT mB RT
and PB – P'B = -
There is a decrease in volume during melting of an ice slab MV M (2V )
at 273 K. Therefore, negative work is done by ice-water
system on the atmosphere or positive work is done on the mB RT
1.5 DP = ... (ii)
ice-water system by the atmosphere. Hence, option (b) is 2 MV
correct. Dividing (i) and (ii)
NOTE : Secondly heat is absorbed during melting (i.e. dQ is 1.5DP mB 3 mB
positive) and as we have seen, work done by ice-water = Þ = Þ 3mA = 2mB
system is negative (dW is negative.) Therefore, from first DP MA 2 mA
law of thermodynamics dU = dQ – dW 17. (c, d)
change in internal energy of ice-water system, dU will be We know that
positive or internal energy will increase.
14. (d) According to Wien's 8 RT 3 RT 2RT
v= ; vrms = and vp =
displacement law, U2 pM M M
lmT = 2.88 × 106 nmK El From these expressions, we can conclude that
The wavelength at the vp < v < vrms
peak of th e spectrum U1 U3
Also the average kinetic energy of gaseous molecules is
becomes
1 2 1 æ3 ö 3
2.88 ´ 106 nmK lm
E = mvrms = m ç v 2p ÷ = mv 2p
lm = = 103 nm l 2 2 è2 ø 4
2880 K 18. (a) NOTE : The law of equipartition of energy states that
NOTE : Thus, the maximum energy is radiated for 103 nm
'For a dynamical system in thermal equilibrium, the
wavelength. It follows that the energy radiated between 499 nm
energy of a system is equally distributed among its
to 500 nm will be less than that emitted between 999 nm to
1000 nm, i.e., U1 < U2 or U2 > U1. various degrees of freedom and the energy associated
15. (b, d) 1
with each degree of freedom per molecule is k.T. In
Co-efficient of linear expansion of brass is greater than that 2
of copper i.e., a B > a C . Now,, this case, O2 and N2 both have two degrees of rotational
kinetic energy and since the temperature is also same,
LB = L0 (1 + a B DT ) the ratio of the average rotational kinetic energy is 1 : 1.
Brass
or, 19. (a, c, d)
Copper
( R + d ) q = L0 (1 + a B DT ) d Since sun rays fall on the black body, it will absorb more
radiation and since, its temperature is constant it will emit
Again, LC = L0 (1 + a C DT ) more radiation. The temperature will remain same only when
or, Rq = L0 (1 + a C DT ) R energy emitted is equal to energy absorbed.
q 20. (b, d)
(R + d )q 1 + a B DT
\ = C p - Cv = R for all gases
Rq 1 + aC DT
R+d 5 5
or, = (1 + a B DT ) (1 - a C DT ) , by binomial For monoatomic gas : Cv =
3
R ; Cp = R ; g =
R 2 2 3
theorem.
15
or, 1 + d = 1 + ( a B - aC ) DT - a B aC (DT ) 2 C p .Cv = ; C p + Cv = 4
R 4
d d 5 7 7
or, = ( a B - a C ) DT or R = For diatomic gas : Cv = R ; Cp = R ; g =
R ( a B - a C ) DT 2 2 5
1 1 35
\ Rµ and R µ . and C p .Cv = ; C p + Cv = 6
DT a B aC
- 4
Heat & Thermodynamics and Gases P-S- 97

21. (b, d) Now, DT = QR


In case of an isothermal process we get a rectangular As RE is least, DTE is also smallest ie since the resistance to
hyperbola in a P-V diagram. Therefore option (a) is wrong. heat flow is least for slab E, the temperature difference across
TD < TB. Therefore in process B ® C ® D, DU is negative. is smallest.
PV decreases and volume also decreases, therefore W is \ Option (c) is the correct answer.
negative. From first law of thermodynamic, Q is negative Also
i.e., there is a heat loss option (b) is correct. DT DTC
QC = C = = 2 KZ ( DTC )
WAB > WBC. RC 1/ 2 KZ
Therefore work done during path A ® B ® C is positive, DTB DTC 3KZ ( DTC )
option (c) is wrong. QB = = = [Q DTB = DTC]
RB 4 / 3KZ 4
Work done is clockwise cycle in a PV diagram is positive.
Option (d) is correct. DTD DTC 5KZ ( DTc )
QD = = = [Q DTD = DTC ]
22. (a, b) RD 4 / 5KZ 4
Process A to B 3 KZ ( DTC ) 5 KZ ( DTC )
As the temperature remains the same, this process is QB + QD = +
4 4
isothermal. Therefore there is no change in the internal 8 KZ ( DTC )
energy. Option (a) is correct. = = 2 KZ ( DTC ) = QC
Also P0 V0 = PB × 4 V0 V 4
\ (d) is the correct option.
P0 4V0 B 24. (a, b, c, d)
Þ PB =
4 We know that dQ = m C dT in the range 0 to 100K
Work done From the graph, C increases linearly with temperature
V0 A
therefore the rate at which heat is absorbed varies linearly
4V0 C with temperature. Option (a) is correct
W = nRT0 loge
V0 T
As the value of C is greater in the temperature range 400-
T0 500K, the heat absorbed in increasing the temperature from
= P0V0 log e 4 [Q P0V0 = nRT0 ] 0 - 100K is less than the heat required for increasing the
The process BC is not clear. Therefore no judgement can be temperature from 400 - 500K option (b) is correct.
made for point C. From the graph it is clear that the value of C does not
23. (a, c, d) change in the temperature range 400-500K, therefore there
It is given that heat Q flows only from left to right through is no change in the rate of heat absorption in this range.
the blocks. Therefore heat flow through A and E slabs are Option (c) is correct.
the same. As the value of C increases from 200-300K, the rate of heat
\ [a] is correct option absorption increases in the range 200-300K. Option (d) is
also correct.
B 25. (a, b, c)
A 3K QB RB E
2K 6K 3 5
C RC Total energy = RT + RT = 4RT
2 2
4K QC
QA QE 4RT
\ Average energy per mole = = 2RT
2
RA D RE
5K QD RD
gRT
QA = QB + QC + QD = QE We know that Vsound =
M
Since heat flow through slabs A and E is same,
[b] is not correct. n1 + n2 n n
= 1 + 2
g mi x –1 g1 - 1 g 2 – 1
l
We know that resistance to heat flow is R =
KA
2 1 1
Let the width of slabs be Z. Then Þ = +
g mix - 1 5 7
L 1 4L 4 -1 -1
RA = = ,R = = 3 5
2 K (4L) Z 8KZ B 3K ( LZ ) 3KZ
2 3 5
4L 1 4L 4 = +
RC = = ,R = = g mix - 1 2 2 =4
4 K (2 LZ ) 2 KZ D 5K ( LZ ) 5 KZ
L 1 1 3
RE = = \ g mix - 1 = \ g mix =
6 K (4 LZ ) 24 KZ 2 2
EBD_7036
P-S- 98 Topic-wise Solved Papers - PHYSICS

(Vs )mix g mix M He 1 P1 A V1 1


= ´ \ Energy = ´ = PV
1 1
(Vs ) He M mix g He 2 2 A 4
\ A is correct
3 Now
´4
2 é 1´ 2 + 1´ 4 ù æ kx ö kx
=
5 êQ M mix = 2
= 3ú W = ò PdV = ò ç P1 + ÷ dV = ò P1dV + ò dV
3´ ë û è Aø A
3 kx
\ W = ò P1dV + ò ´ (dx) A
A
6
=
5 kx 2
\ W = P1 (V2 – V1 ) +
2
3RT
We know that Vrms = é PV P2V2 4 P1 ù
M 1 1
ê Here on applying T = T we get P2 = 3 ú
M H2 ê 1 2 ú
(Vrms ) He 2 1 ê 2V1 ú
\ (V ) = M HE
=
4
=
2 ê and V2 = V1 + Ax Þ x = [QV2 = 3V1 ] ú
rms H 2 ë A û
\ options [A], [B] and [C] are correct.
1 P1 A 2V1 7
\ W = 2P1V1 + ´ ´ = PV
1 1
26. (b, c) 2 3 A 3
Applying combined gas law C is correct option
Heat supplied
PV
1 1 PV
= 2 2 Q = W + DU
T1 T2
7 PV
1 1 + 3 ( P V – PV )
If V2 = 2 V1 and T2 = 3T1 then = 2 2 1 1
3 2
PV
1 1 P ´ 2V1 2
= 2 Þ P1 = P2 7 PV
1 1 3 é4 ù 41
T1 3T1 3 = + ê P1 3V1 – PV
1 1ú = PV
1 1
3 2 ë3 û 6
Now change in internal energy
f f
E. Subjective Problems
DU = [nR (T2 – T1)] = [P V – P1V1]
2 2 2 2 1. W0 – W1 = V ´ d l ´ g ... (i)
For monoatomic gas f = 3
W0 – W2 = V '´ d 'l ´ g ... (ii)
3 é3 ù Also, V' = V (1 + b DT) ... (iii)
DU = ê P1 ´ 2V1 – PV 1 1 ú = 3P1 V1
2 ë2 û and dl = dl ' (1 + g l DT ) ... (iv)
\ (b) is the correct option. From (ii), (iii) and (iv)
Now assuming that the pressure on the piston on the right V (1 + b DT ) ´ d l
hand side (not considering the affect of spring) remains the W0 – W2 = ´ g ... (v)
1 + g l DT
same throughout the motion of the piston then,
Dividing (i) and (v), we get
kx kx
Pressure of gas = P1 + Þ P2 = P1 + W0 - W1 Vd l g (1 + g l D T )
A A =
where k is spring constant and A = area of piston W0 - W2 V (1 + bDT ) d l g

1 2 W0 - W1 1 + g l DT W0 - W1 1 + g l (t2 - t1 )
Energy stored = kx Þ = Þ =
2 W0 - W2 1 + bDT W0 - W2 1 + b (t2 - t1 )
kx 3 kx Þ (W0 – W1) [1 + b (t2 – t1) = (W0 – W2) [1 + g l (t2 - t1 )]
P2 = P1 + Þ P1 = P1 +
A 2 A W2 - W1 b (W0 - W1 )
Þ gl = +
P1 kx (W0 - W2 ) (t2 - t1 ) (W0 - W2 )
=
2 A 2. KX = 0.92 cal/sec-cm-°C
PA KY = 0.46 cal/sec-cm-°C
\ kx = 1 C [T ]
2 C
Also, X X
V2 = V1 + Ax X
B
V1 = Ax A E
[60oC] Y [TB] [10oC]
V
\ x= 1 Y Y
A [TD]
D
Heat & Thermodynamics and Gases P-S- 99

NOTE THIS STEP : The heat flow through AB is divided 8. Work don in an adiabatic process is
into two path BC and BD. Symmetry shows that no heat will 1
flow through CD. Therefore W= [ P V - PV ]
1- g 2 2 1 1
KY A (60 - TB ) K X A (TB - 10) KY A (TB - 10)
= + Here, P1 = 105 N/m2, V1 = 6 l = 6 × 10–3 m3
l 2l 2l
On solving the above equation, we get g
æV ö
TB = 30°C P2 = P1 ç 1 ÷ , V2 = 2 l = 2 × 10–3
As C is a point at the middle of BE therefore temperature at è V2 ø
C is 20°C.
Similarly temperature at D is also 20°C. 3
Given that Cv = R
3. PV = nRT 2
When P, T are same n µ V 5
As volumes are same, both samples will have equal number \ Cp = R [Q C p - Cv = R]
2
of molecules
4. (i) Region AB : Heat is absorbed by the material at a Cp
constant temperature called the melting point. \ g= = 1.67
Cv
The phase changes from solid to liquid.
Region CD : Heat is absorbed by the material at a 1.67
é6ù
constant temperature called the boiling point. The \ P2 = 105 êë 2 úû = 105 × (3)1.67 = 6.26 × 105 N/m2
phase changes from liquid to gas.
(ii) Latent heat of vaporisation = 2 (latent heat of fusion) 1
(iii) Q = mcg DT. \ W= [6.26 × 105 × 2 × 10–3 – 105 × 6 × 10–3]
1 - 1.67
DT 1
The slope DE = = 1 652
Q mcg W= [1252 – 600] = - = – 973.1 J
-0.67 0.67
NOTE : The slope DE indicates that the temperature of the
Work done is negative because the gas is compressed.
solid begins to rise.
9. NOTE : Since the temperature and surface area is same,
(iv) The reciprocal of heat capacity in solid state is greater
therefore the energy emitted per second by both spheres is
than the reciprocal of heat capacity in liquid state
same.
æ 1 ö æ 1 ö We know that Q = mcDT
çè ÷ø >ç ÷ Þ (mc)liquid > (mc)solid
mc solid è mc ø liquid Since Q is same and c is also same (both copper).
Þ cliquid > csolid 1
5. P1 = 830 – 30 = 800 mm Hg ; P2 ? \ mµ
DT
V1 = V ; V2 = V ; T1 = T ; T2 = T – 0.01 T = 0.99 T Mass of hollow sphere is less;
P1V1 P2 V2 P1T2 800 ´ 0.099T \ Temperature change will be more.
=
T1 T2 \ P2 = T = T
= 792 mmHg \ Hollow sphere will cool faster.
1 10. (i) F = P × A = 105 × 1 = 105 N
\ Total pressure in the jar = 792 + 25 = 817 mm Hg
A ® B A straight line between A and P A Dp
6. B F= Þ Dp = F ´ Dt = 105 ´ 1 = 105 .. (i)
B in V-T graph indicates V µ T Dt
Þ Pressure is constant. Now, momentum change per second
(Dp) = n × 2mv ...(ii)
B ® C Volume is constant. Since the Where n is the number of collisions per second per
C
temperature is decreasin g, the square metre area
pressure should also decrease. V From (i) and (ii)
C ® A The temperature is constant but volume decreases.
The process is isothermal. 105
n × 2mv = 105 \ n=
7. Lead bullet just melts when stopped by an obstacle. Given 2mv
that 25% of the heat is absorbed by the obstacle. Therefore Root mean square velocity
75% heat is used in melting of lead. Initial temp. = 27°C
M.P. = 300°C 3RT 3 ´ 8.314 ´ 300
v= = = 483.4 m/s
(0.75) K.E. = Heat utilised in increasing the temperature and M 32 /1000
heat utilised to melt lead at 300°C According to mole concept 6.023 × 1023 molecules will
1 have mass 32 g
(0.75) × Mv 2 = Mc DT + ML 32
2 \ 1 molecule will have mass g
1 2 6.023 ´ 1023
(0.75) × v = (0.03 × 300 + 6) × 4.2
2 105 ´ 6.023 ´ 1023
\ n= = 1.97 ´ 1027
[4.2 to convert into S.I. system] 2 ´ 32 ´ 483.4
v = 12.96 m/s
EBD_7036
P-S- 100 Topic-wise Solved Papers - PHYSICS
(ii) The kinetic energy of motion of molecules will be 12. Let x moles shift from high temperature side to low
converted into heat energy. temperature side.
1 2 for left bulb PV = nRT
K.E. of 1 gm mole of oxygen = Mv 0 ... (i) 76 × V = nR × 273 Initially
2
where v0 is the velocity with which the vessel was P' × V = (n + x) R × 273 Finally
moving. Dividing, we get
The heat gained by 1 gm mole of molecules at constant P' n + x
volume for 1°C rise in temperature = ... (i)
= nCv DT = 1 × Cv × 1 = Cv ... (ii) 76 n
From (i) and (ii) V V
1 R n n n+x n-x
Mv02 = Cv But, Cv =
2 g -1 Initially 0OC 62OC
1 R Finally
Mv02 =
2 g -1 For right bulb
76 × V = nR × 273 Initially
2R 2 ´ 8.314 P' × V = (n – x) R × 335 Finally
\ v0 = = = 35.6 ms–1
M( g - 1) 32 On dividing,
´ (1.41 - 1)
100 P ' n - x 335
= ´ ... (ii)
[ Q g = 1.41 for O2 (diatomic gas)] 76 x 273
11. For the left chamber From (i) and (ii)
P0V0 P0 ´ 243 n + x n - x 335
= ´ V1 = ´
T0 32 ´ T1 n n 273
243 V1T0 608
Þ T1 = ´ ... (i) Þ n= x. ... (iii)
32 V0 62
For the right chamber for adiabatic compression Substituting the value of (iii) in (i), we get
243 P' 62
We get, P0 V0g = P0 ´ ´ V2g = 1+
32 76 608
3/ 5
V2 æ 32 ö 8 670
Þ =ç ÷ Þ V2 = V0 Þ P' = ´ 76 = 83.75 cm Hg
V0 è 243 ø 27 608
But V1 + V2 = 2V0 13. Let A be the area of cross-section of the tube.
Since temperature is the same, applying Boyle's law on the
8 46 side AB
\ V1 = 2V0 – V2 = 2V0 – V0 = V0 ... (ii)
27 27 P × (x × A) = P2 × (x2 × A) ... (i)
243 46 ´ V0 Applying Boyle's law in section CD
From (i) and (ii) T1 = ´ ´ T0 P × (x × A) = P1 × (x1 × A) ... (ii)
32 V0 ´ 27
From (i) and (ii)
207 P1 × (x1 × A) = P2 × (x2 × A)
or, T1 = T0 = 12.9T0 (approx.) Þ P1x1 = P2x2
16 where P2 = P1 + Pressure due to mercury column
To find the temperature in the second chamber (right), we
apply x2
m
g 1-g 5c P1
æ T1 ö æ P2 ö x D
çè T ÷ø = çè P ÷ø x x1
2 1 C
P P P2
1- 5 / 3
B
5/3 30
æ T0 ö æ 243 P0 ö
O
A B C D
Þ çè T ÷ø =ç Þ T2 = 2.25 T0 5cm A
2 è 32 P0 ø÷
Pressure due to mercury column
Work done in right chamber (adiabatic process)
F mg sin 30° Vdg sin 30°
1 P= = =
W= ( P2V2 - P0V0 ) A A A
1- g
(A ´ 5) ´ dg sin 30°
= = 5 sin 30° cm of Hg
3 é 243 8 ù A
=- ê P0 ´ V0 - P0V0 ú
2 ë 32 27 û P2 = P1 + 5 sin 30° = P1 + 2.5
Substituting this value in (iii)
3æ9 ö 15
= - ç - 1÷ P0V0 = - ´ RT0 = –15.8 T0 P1 × x1 = [P1 + 2.5] × x2
2è4 ø 8
Heat & Thermodynamics and Gases P-S- 101

P1 × 46 = [P1 + 2.5] × 44.5 1 1


44.5 ´ 2.5 é T ù g -1 é 600 ù 5 -1
\ P1 = \ V3 = V2 ê 2 ú = 40 ê ú = 113l
1.5 ë T3 û ë 300 û 3
Substituting this value in (ii) 5
[Q g = for mono atomic gas]
44.5 ´ 2.5 3
P×x = ´ 46 Now, P3V3 = nRT3
1.5
nRT3 2 ´ 8.3 ´ 300
é 46 + 44.5 ù 44.5 ´ 2.5 Þ P3 = = = 0.44 × 105 N/m2
Þ P× ê
ë 2 úû = 1.5
´ 46 V3 113 ´ 10 -3

(NOTE : T3 = T1 given)
x1 + x 2 (iii) W = W12 + W23
[Q x= ] Þ P = 75.4 cm
2 nR
= P1 (V2 – V1) + (T - T )
14. We know that PV = nRT g -1 2 3
PV 1.6 ´ 106 ´ 0.0083 16 W12 = work done at constant pressure
\ n= = = = 5.33 moles W23 = work done in adiabatic condition
RT 8.3 ´ 300 3
2 ´ 8.3
5R 3R = 2.49 × 105 (40 – 20) 10–3 + (600 - 300)
Cp = Þ Cv = 5
2 2 -1
3
When 2.49 × 104 J of heat energy is supplied at constant
= 4980 + 7470 = 12450 J
volume then we can use the following relationship to find
16. KEY CONCEPT : The final pressure on the gas
change in temperature.
= atm pressure + pressure due to compression of spring
Q = nCv DT
kx
Q 2.49 ´ 104 P2 = Patm +
\ DT = = = 375 K A
nCv 3
5.33 ´ ´ 8.3 8000 ´ 0.1
2 Þ P2 = 105 + = 2 × 105 N/m2
Therefore, the final temperature 8 ´ 10 -3
= 300 + 375 = 675 K
Applying Gay Lussac's Law, to find pressure. T1=300K Atm pressure = 105N/m2
P1 P2 Ideal
= Heater Monoatomic
T1 T2 Gas
k = 8000N/m
PT 1.6 ´ 106 ´ 675
Þ P2 = 1 2 = = 3.6 × 106 Nm–2
T1 300
The final volume,
15. (i) P – V diagram is drawn below.
V2 = V1 + xA
P = 2.4 × 10–3 + 0.1 × 8 × 10–3 = 3.2 × 10–3 m3
(P1,V1)
P1=P2
(P2,V2) PV PV P2V2T1
1 2 Applying 1 1
= 2 2 Þ T2 =
T1 T2 PV
1 1

(P3,V3) 2 ´ 105 ´ 3.2 ´ 10-3 ´ 300


P3 Þ T2 = = 800 K.
3 105 ´ 2.4 ´ 10 -3
20l 40l V NOTE : Heat supplied by the heater is used for expansion
of the gas, increasing its temperature and storing potential
(ii) P1V1 = nRT1 energy in the spring.
\ P1 × 20 × 10–3 = 2 × 8.3 × 300 \ Heat supplied
P1 = 2.49 × 105 Nm–2 1
PV PV = P DV + nCv DT + k x 2
Applying 1 1 = 2 2 2
T1 T2 PV 1
For 1 ® 2 = 105 [0.8 × 10–3] + 1 1
Cv DT + kx 2
RT1 2
20 40
= Þ T2 = 600 K 105 ´ 2.4 ´ 10 -3 3 1
300 T2 = 80 + ´ ´ 2 ´ 500 + ´ 8000 × 0.1
2 ® 3 is adiabatic expansion. 2 ´ 300 2 2
= 720 J
T2V2g -1 = T3V3g -1
EBD_7036
P-S- 102 Topic-wise Solved Papers - PHYSICS
17. (i) Let pressure = P, Volume = V and Temperature = T be Let the system initially be at point A at pressure PA and temp
the initial quantities and Pressure = P', Volume = 5.66 V TA and volume VA.
Temperature = T/2 be the final quantities. Process A to B
For adiabatic process The system is isothermally expanded and reaches a new
T state B (PB, 2VA, TA) as shown in the figure.
TV g -1 = (5.66 V ) g -1 Þ 2 = (5.66)g –1 Process B to C
2
Taking log on both sides, log 2 = (g – 1) log 5.66 The system is the compressed at constant pressure to its
Þ g = 1.4 original volume to reach at state C (PB, VA, TC)
Process C to A
2 2 Finally at constant volume, the pressure is increased to its
But g = 1 + Þ 1.4 = 1 +
original pressure to reach the state A again.
f f
(b) The total work done
2 W = WA® B + WB® C + WC® A
Þ f= =5
0.4
VB
Thus degrees of freedom of gas molecules = 5 = nRTA loge + nR (TC – TA) + 0
(ii) For adiabatic process the pressure-volume relationship VA
is
2VA æT ö
g g = 2.303 × 3 × R × TA log10 + 3 R ç A - TA ÷
PV
1 1 = P2V2 VA è 2 ø

P P 3
Þ P2 = = = 2.08 RTA – RTA = 0.58RTA
1.4 11.32 2
(5.66)
Work done for adiabatic process NOTE : The total work done is equal to the heat exchanged
as the process is cyclic.
æ P ö 19. Let us find out the work done in the cycle
PV - ç (5.66 V )
PV - P V è 11.32 ÷ø P
W= 1 1 2 2 = = 1.25 PV A B
g -1 1.4 - 1 2atm

18. (a) Process A to B (isothermal expansion)


PAVA = PBVB 1atm
D
C
Þ PAVA = PB× 2VA
P
PA
Þ PB = A
2 (PA,VA,TA)
300K 400K T
Process B to C (isobaric
Work done from A to B (Isobaric process)
compression)
,T ) WAB = nR (TB – TA)
(PB,VA C (PB,2VA,TA)
VB VC = nR × 100 = 2 × 200 × 8.32 = 1664 J
= C B
Work done from B to C (Isothermal process)
TB TC
PB
2V A VA T WBC = 2.303nRT log10
Þ = PC
TA TC
TA 2
Þ TC = = 2.303nR × 400 log10 = 277.2 nR
2 1
Process C to A [volume is constant] = 554.4 × 8.32 = 4612.6
Work done from C to D (Isobaric process)
PC PA P P WCD = nR (TD – TC) = nR (300 – 400)
= Þ B = A
TC TA TC TA = – 100nR = – 200 × 8.32 = – 1664 J
T Work done from D to A (Isothermal process)
PA / 2 PA
Þ = Þ TC = A PD 1
TC TA 2 WDA = 2.303nRT log10 = 2.303nR × 300 log10
PA 2
P = – 207.9nR
PA A (P ,V ,T ) = – 415.8 × 8.32 = – 3459.5 J
A A A
The total work done = WAB + WBC + WCD + WDA
= 1153 J
(a) DU = Q – W
B For complete cycle DU = 0
C (PB,2VA,TA) \ Q = W = 1153 J
(b) W = 1153 J
O VA 2VA (c) DU = 0. Since, the process is cyclic.
V
Heat & Thermodynamics and Gases P-S- 103

20. Given TA = 1000 K P g -1 g


A æ PC PA ö æT T ö
2
çè P P ÷ø = ç C ´ A ÷ .... (v)
PB = PA D B è TD TB ø
3 B
1 Multiplying (ii) and (iv)
PC = PA D
3 æ PA PC ö æ TC TA ö
(i) WAB (adiabatic expansion) C çè P P ÷ø = çè T T ÷ø ... (vi)
B D B D
nR[TA - TB ]
WAB = V From (v) and (vi)
g -1
g -1 g
Here, n = 1, R = 8.31 J mol–1 k–1, TA = 1000 K æ TC TA ö æT T ö T T
çè T T ÷ø = ç C A ÷ Þ A C =1
5 B D è TBTD ø TB TD
g= (For mono atomic gas)
3
TATC 1000 ´ 425
To find TB, we use Þ TD = = = 500K
TB 850
g -1 g
g 1-g æP ö æT ö 21. (i) The process is cyclic, therefore DU = 0
TA PA = TBg PB1-g Þ ç A÷ = ç A ÷ ... (i)
èP ø B èT ø
B Now, DQ = DU + DW
Þ DQ = DW
1-g 1-5 / 3 Þ Q1 + Q2 + Q3 + Q4 = W1 + W2 + W3 + W4
éP ù g é 3ù 5/3
Þ TB = TA ê A ú = 1000 ê ú = 850 K Þ 5960 – 5585 – 2980 + 3645 = 2200 – 825 – 1100 + W4
ë PB û ë 2û Þ W4 = 765 J
1 ´ 8.31[1000 - 850] Work done
\ WAB = = 1870 J (ii) Key Concept: h =
5/ 3-1 Heat supplied
(ii) Heat Lost B ® C
Q = nCv DT = nCv (TB – TC) W1 + W2 + W3 + W4
= Q1 + Q 4
3
Here, n = 1, Cv = R (For mono atomic gas),
2 1040
TB = 850 K Þ = = 10.82%
9605
PB PC 22. The total pressure exerted by the mixture P = 105 Nm–2
To find TC, we use = (volume constant)
TB TC Temperature T = 300 K ; Volume = 0.02 m3
Let there be x gram of Ne. Then mass of Ar will be 28 – x.
PC TC
Þ = ..(ii) x
PB TB Number of moles of Neon = ;
20
é 1 ù 28 - x
PC 1 ê PC 3 PA 1 ú Number of moles of Argon =
Þ TC = ´ TB = ´ 850 = 425 K êQ = = ú 40
PB 2 ê PA 2 PA 2 ú Partial pressure due to Neon;
ë 3 û
( x / 20) RT
3 p1 =
\ Q = 1× × 8.31 [425 – 850] = – 5298 J V
2
Partial pressure due to Argon
(iii) Temperature TD : C to D is adiabatic compression
[(28 - x ) / 40] RT
g -1 g p2 =
æ PC ö æT ö V
çè P ÷ø =ç C ÷ ...(iii)
D èT ø
D But according to Dalton's law of partial pressure
P = p1 + p2
PD PA
D to A is isochoric process = x RT (28 - x ) RT
TD TA 105 = +
20V 40V
PA TA
Þ = ... (iv) 105 ´ 40 ´ 0.02
PD TD Þ = x + 28 Þ x = 4g
8.314 ´ 300
Multiplying (i) and (iii) Þ Mass of Neon = 4g
\ Mass of Argon = 24g
EBD_7036
P-S- 104 Topic-wise Solved Papers - PHYSICS

5 + 7nB 19 -dP
23. (a) = Þ nB = 2 mol. Þ = -g P
3 + 5nB 13 dV / V
We know that \ Bulk Modulus B = g P
1 1
nA + nB n n \ Compressibility K = =
= A + B B gP
g m -1 g A - 1 g B - 1
where gm = Ratio of specific heats of mixture 1 1
\ K1 = and K2 =
Here, nA = 1, g A = 5 3, g B = 7 5 gP1 gP2
According to the relationship 1 1 1æ 1 1ö
DK = K2 – K1 = - = -
19
19 gP2 gP1 g çè P2 P1 ÷ø
PV 13 = constant, we get gm =
13
(b) On substituting the values we get nB = 2 mol. Since the process is adiabatic, P2V2g = PV
1 1
g

We know that velocity of sound in air is given by the g g


relationship æ V1 ö æ V1 ö
\ P2 = P1 ç ÷ = P1 ç = P1 5g
è V2 ø è V1 / 5 ÷ø
gP m
v= where d = density =
d V 1æ 1 1ö 1 æ 1 ö
\ DK = ç - ÷= - 1÷
( n A + nB ) g è P1 5g P1 ø gP1 çè 5g ø
Also, PV = (nA + nB) RT Þ PV = RT
V (n A + nB ) RT (1 + 2) ´ 8.31 ´ T 24.93T
P1 = = =
g (n A + nB ) RT g (n A + nB ) RT V V V
\ v= =
m m 1 æ 1 ö
V´ Þ DK = ç - 1÷
V 19 T è519 /13 ø
´ 24.93 ´
Mass of the gas, m = nAMA + nBMB = 1 × 4 + 2 × 32 13 V
= 68 g/mol = 0.068 kg/mol = – 8.27 × 10–5 V [Q T = 300 K]
5
19(1 + 2) ´ 8.314 ´ 300 24. (i) T1 = 27 + 273 = 300 K; g = (for monoatomic gas)
\ v= = 400.03 ms–1 3
13 ´ 0.068 V1 = V
(c) Velocity of sound, V2 = 2V
T2 = ?
g RT g R(T + DT ) Since the gas expands adiabatically.
v= and v + Dv =
M M g -1 g -1
T1V1 = T2V2
1/ 2
v + Dv T + DT æ DT ö g -1 5/ 3-1
Þ = = ç1 + ÷ æV ö éV ù
v T è T ø Þ T2 = T1 ç 1 ÷ = 300 ê ú = 189 K
èV ø 2 ë 2V û
DT
When DT << T then << 1
T -nR (T2 - T1 ) -2 ´ 8.31(189 - 300)
(ii) W = =
g -1 5/ 3 -1
Dv 1 DT
\ 1+ = 1+
v 2 T + 8.31 ´ 111 ´ 3
= = + 2767J
Dv 1 DT 2
Percentage change ´ 100 = ´ ´ 100 Change in internal Energy
v 2 T According to first law of thermodynamics
Dv 1 1 1 DQ = DU + DW But D Q = 0
´ 100 = ´ 100 = %
v 2 300 6 (as the process is adiabatic)
(d) PVg = Const. \ DU = – DW = – 2767 J
Differentiating the above equation (iii) W = 2767 J
Vg (dP) – P (gVg –1 dV) = 0 25. Heat lost by steam = Heat gained by water
Þ Vg dP = gPVg–1dV msLfus = mwc DT
dP gPV g -1 gP mwc DT 0.1 ´ 4200 ´ 66
Þ = g
= gPV g -1-g = Þ ms = = = 0.0122 kg
dV V V L fus 540 ´ 103 ´ 4.2
Heat & Thermodynamics and Gases P-S- 105

26. n = 1, for diatomic gas, P 5R


B C = ´ 1027
2 7 2
g = 1+ = = 1.4
5 5
5R 5R
A ® B, adiabatic compression Wnet = 909 R + (1027 – 609) = 909 R + × 418
B ® C, isobaric expansion 2 2
D
C ® D, adiabatic expansion = 909 R + 1045 R = 1954 R
D ® A, isochoric process A 1954 R
V \ Efficiency = 100 × (Wnet/Q1) = 100 × = 61.4%
V V 3182 R
Given A = 16, C = 2
VB VB 27. Let the pressure at point O be P0. Since the liquid is at
TA = 300 K, TB = ?, TD = ?, h = ? equilibrium at M
For adiabatic compression process A ® B PA + h1r95°g = P0 + h r5° g
Þ P0= PA + h1r95° g – h r5° g ... (i)
g -1 g -1
TAVA = TBVB or Since the liquid is at equilibrium at N
Þ PA + h2r5° g = P0 + h r95° g
g-1
æV ö Þ P0 = PA + h2r5° g – h r95° g ... (ii)
TB = ç A ÷ TA = (16)2 / 5 ´ 300 = 909 K From (i) and (ii)
è VB ø
PA + h1r95° g – h r5° g PA
\ For isobaric process B ® C : According to Charles' law = PA + h2r5° g – h r95° g PA
h1
VB VC æV ö r o
h2
As = or TC = TB ç C ÷ = 909 [2] = 1818 K 5 O
TB TC è VB ø Þ = 1.018 ... (i) A
r
95o
o
95C oB C oD
For adiabatic expansion process C ® D : 5C h o
95C 5C
We know that
V V M N
V r0 = r1 (1 + g DT)
As A = 16 and C = 2; hence A = 8
VB VB VC Applying the above formula, we get
According to Poisson's law, r0 = r95° (1 + g × 95)
r0 = r5° (1 + g × 5)
g -1
TCVC = TDVDg -1 r5° 1 + 95 g
\ = ... (ii)
g -1 2/5 r95° 1 + 5 g
éV ù é1ù 1818
\ TD = TC ê C ú = 1818 ê 8 ú = = 791K From (i) and (ii)
ë VD û ë û (64)1/ 5
1 + 95 g
For B ® C process : Heat absorbed Þ g = 2.002 × 10–4
Q1 = nCp (TC – TB) 1 + 5 g = 1.018
gR (7 / 5) R But g = 3a
=n (TC - TB ) = 1 (1818 - 909)
g -1 (2 / 5) g 2.002 ´ 10-4
Þ a= = = 6.67 × 10–5 °C –1
7R 3 3
= ´ 909 @ 3182 R
2 5R 3R
For D ® A process : Heat released 28. n = 1, For monoatomic gas : Cp = , Cv =
2 2
R Cyclic process
Q2 = nCv (TD – TA) = n (T - TA ) A ® B Þ Isochoric process
g -1 D
C ® A Þ Isobaric compression
R 5R (a) Work done = Area of closed curve ABCA during cyclic
= 1. (791 - 300) = ´ 491
(2 / 5) 2 process. i.e. DABC
(Q No heat is exchanged in adiabatic processes). 1 1
DW = × base × height = V0 × 2P0 = P0V0
nR 2 2
Now, WAB = - (T - TA ) (b) Heat rejected by the gas in the path CA during isobaric
g -1 B
compression process
R 5R DQCA = nCp DT = 1 × (5R/2) (TA – TC)
=- (900 - 300) = - ´ 609
(2 / 5) 2 2 P0V0 PV
TC = , TA = 0 0
WBC = – nR (TC – TB) = 1× R (1818 – 909) = 909 R I ´R I´R
nR R 5 R é P0V0 2 P0V0 ù 5
WCD = - (TC - TD ) = + (1818 - 791) DQCA = - = - P0V0
g -1 (2 / 5) 2 êë R R úû 2
EBD_7036
P-S- 106 Topic-wise Solved Papers - PHYSICS
Heat absorbed by the gas on the path AB during
dT é KA ù
isochoric process Þ - = K '+ (T - TA ) (for t > t1]
DQAB = nCvDT = 1 × (3R/2) (TB – TA) dt êë CL úû
Where K = coefficient of thermal conductivity of the rod.
3R é 3P0V0 P0V0 ù
= - = 3P0V0
2 êë 1 ´ R 1 ´ R úû Þ
- dT é
= ê K '+
KA ù
dt
(c) As DU = 0 in cyclic process, hence, T - TA ë CL úû
DQ = DW On integrating, we get
DQAB + DQCA + DQBC = DW 3t1 æ
T dT KA ö
PV PV -ò = ò ç K '+ ÷ø dt
DQBC = P0V0 – 0 0 = 0 0 350 T - T
A t1 è CL
2 2
æ KA ö 3t1
NOTE : As net heat is absorbed by the gas during - [ log e (T - TA )] 350 = ç K '+ ÷ [t ]
T
Þ
path BC, temp. will reach maximum between B and C. è CL ø t1
é 2P ù 350 - 300 æ KA ö
(d) Equation for Line BC is P = – ê 0 ú V + 5P0, Þ loge = ç K '+ ÷ 2t = 2 K ' t1
ë V0 û T - 300 è CL ø
RT 50 2KA
P= [For one mole] Þ loge = 2(log e 2) + t1
V T - 300 CL
2 P0 2 {log e 4 +
2 KA
\ RT = – V + 5P0V ... (i) 50 t1}
V0 =e CL
T - 300
dT 2P
For maximum; = 0, - 0 × 2V + 5P0 = 0; -2 KAt1
dV V0 Þ T – 300 = 50 e -[log e 4] ´ e CL

5V0 -2 KAt1
\ V= ... (ii) Þ T = [300 + 12.5 e CL ] Kelvin
4
Hence from equation (i) and (ii) 30. n = no. of moles = 2,
2 (A) The complete process is shown on P-V diagram in the
-2 P0 æ 5V0 ö æ 5V ö figure.
RTmax = ´ç ÷ + 5 P0 ç 0 ÷
V0 è 4 ø è 4 ø
P 3 C
25 25 P0V0 25 Heating at constant
= – 2P0V0 × + = P0V0 Volume V2
16 4 8 P2
2
B
Adiabatic
25 P0V0 compression
\ Tmax =
8 R P1
1
A
29. Case (i)
According to Newton's law of cooling
V
V2 V1
dT dT
= – K' (T – TA) Þ = – K'dt
dt T - TA (B) (i) Total work done
On integrating, we get
1 1 - P2V2 )
( PV
350 dT W = WAB + WBC = +0
t1 ( g - 1)
ò400 T - TA = K ò0 dt
[ Q WBC = PDV = P × 0 = 0]
350 t1 g
– [loge (T – TA) ] 400 = K' [t ] 0 æV ö
According to Poisson's law, P2 = P1 ç 1 ÷
350 - 300 èV ø 2
Þ - log e = K ' t1
400 - 300 g
100 1 é æ V1 ö ù
Þ loge = K ' t1 or K't1 = loge 2 ... (i) \ W= ê PV - P1ç V2ú
è V2 ø÷
1 1
50 g -1 ê ú
ë û
Case (ii)
NOTE : When the body X is connected to a large box Y. In g -1
1 é V1 æ V1 ö ù
this case cooling occurs by Newton's law of cooling as well = ê PV - P V
. . . ú
V2 èç V2 ø÷ ú
1 1 1 2
as by conduction g -1 ê
ë û
dT KA(T - TA ) For monoatomic gas,
\ - = K '(T - TA ) +
dt CL
Heat & Thermodynamics and Gases P-S- 107

2 5 æ PC ö
g=1+ = ,
3 3 DT = 0 and QC ® A = WC ® A = nRTC ln çè P ÷ø
A

3é æ V1 ö
2/3ù
3 é æ V ö 2/3ù
æ 2P1 ö
1 1 ê1 - ç ú
ê PV ú 1
W= - PV = PV
ê è V2 ÷ø
1 1ç
è V2 ø÷ = nR(2T1) ln çè P ÷ø = (2) (R)(600) ln(2) = 1200R×0.6932
1 1
2ê ú 2 ú
ë û ë û 1
(ii) DU = DUAB + DUBC = Q – W QC ® A = 831.6 R (absorbed)

3 é æ V ö 2/3ù 32. Here the equilibrium temperature is 273 + 27 = 300 K


1 1 ê1 - ç ú
1
=Q– PV Also according to the principle of calorimetry
2 ê è V2 ÷ø ú
ë û Heat lost by container = Heat gained by ice.
[according to first law of thermodynamics] Heat lost by container :
NOTE : Since specific heat is variable, we need to take the
é B ® C Q = DU BC + 0 ù help of calculus to find the heat lost by the container.
ê A ® B Q = DU + W ú
ë AB û Let dQ be the heat lost when the temperature decreases by
(iii) For process BC : DUBC = nCv DT = Q dT at any instant when the temperature of the container
[Q WBC = 0 ] is T.
\ dQ = mc dT
R 3 where m is the mass of the container and C = A + BT is
For monoatomic gas Cv = = R,
g -1 2 specific heat at that temperature
\ dQ = m (A + BT) dT
3R
\ DUBC = Q = 2 × .DT On integrating, we get
2
300
Q 300 é BT 2 ù
Hence DT = . Q = ò500 m ( A + BT ) dT = m ê AT + ú
3R êë 2 úû
500
According to Poission's Law : = – 21600 m calorie (heat lost)
g -1 g -1 Heat gained by ice
For the process AB, TAVB = TBVB
This heat is to be divided into two parts
g -1 g -1 (i) 0° ice ® 0° water
æV ö 1 1 æ V1 ö
PV
or TB = TA ç 1 ÷ = (ii) 0° water ® 27° water
è V2 ø nR çè V2 ÷ø Q1 = mL Q2 = mcDT
= 0.1 × 80,000 = 0.1 × 103 × 27
5 / 3 -2 / 3
P1 g 1-g PV V2 = 8000 cal = 2700 cal
\ TB = .V1 .V2 = 1 1
2R 2R \ Q1 + Q2 = 8000 + 2700 = 10,700 cal ... (i)
5 / 3 -2 / 3
Heat lost = heat gained
PV
1 1 V2 Q 21600 m = 10,700
Hence, TC = TB + DT = +
2R 3R Þ m = 0.495 kg
31. For PVx = Constt., Molar heat capacity 33. (a) Since AB is a straight line in V-T graph
R R R R V
C = + = + \ = Constant (Isobaric process)
g -1 1 - x 5 1 T
-1 1-
3 2 VA VB
\ =
2 1/2
Here P V = constant or PV = constant TA TB
1 VB é VB ù
\x =
2 TB = ´ TA = 2 × 300 = 600 K êQ = 2ú
VA ë VA û
Þ C = 3.5R (b) (i) A to B is a isobaric process
QA ® B = nC DT = 2(3.5 R) (300 – 600) = – 2100 R 5
Process B – C: Process is isobaric therefore \ Q = nCpDT = 2 × R ´ 300
2
æ5 ö é 5 ù
QB ® C = nCp DT = (2) çè R ÷ø (TC – TB)
2 = 1500 R êQ C p = 2 R for monoatomic gas ú
ë û
æ5 ö NOTE : Heat is absorbed as Q is positive.
= 2 çè R ÷ø (2T1 – T1) = (5R) (600 – 300) = 1500R (ii) B to C is an isothermal process.
2
Since the temperature is not changing
Heat is absorbed
\ Internal energy change dU = 0
Process C – A: Process is isothermal
\ From first law of thermodynamics dQ = dW
EBD_7036
P-S- 108 Topic-wise Solved Papers - PHYSICS

Vf Here n = number of moles of gas present in the container.


\ Q = 2.303 × nRT log10 The kinetic energy at a given temperature for a monatomic
Vi
3
= 2.30. × 2 × R × 600 × log10 2 gas is = ´ nRT
= 2763.6 × log10 2 × R = 831.8 R 2
NOTE : Heat is absorbed since temperature is same 3
but volume increases. \ Change in kinetic energy = ´ nR ( DT ) .... (ii)
2
(iii) C To D is a isochoric process
where DT = Change in temperature
\ dW = 0
From (i) and (ii)
\ Q = nCv DT = n æç R ö÷ ( TA - TB )
3
è2 ø 3 1 mv02
nR (DT ) = (nm)v02 \ DT =
3 2 2 3R
= 2 × R × (– 300) = – 900 R 36. (a) The rate of heat loss per unit area per second due to
2
Volume is constant as heat is released. radiation is given by Stefan's-Boltzmann law
(iv) D to A is an isothermal process
E = e s (T4 – T04 )
Vf
\ Q = 2.303 × nRT log10 17
Vi = 0.6 × ´ 10-8 [(400)4 - (300)4 ] = 595 watt/m2
3
æVf ö (b) Let Toil be the temperature of the oil.
= 2.303 × 2 R × 300 × log ç ÷ = – 831.8 R Then rate of heat flow through conduction = Rate of
è Vi ø heat flow through radiation
Heat is released as Q is positive. KA(Toil - T )
(c) Total work done = 595 × A
l
= QA ® B + QB ® C + QC ® D + QD ® A
where A is the area of the top of lid
= (1500 R + 831.8 R) – (900 R + 831.8 R) = 600 R
34. The distance travelled by an 595 ´ l 595 ´ 5 ´ 10 -3
ÞToil = + T = + 400 = 419.83 K
1 k 0.149
atom of helium in sec (time
500 T1 T2
between two successive 37. At constant pressure, we have =
V1 V2
collision) is 2m. Therefore root
mean square speed also, V = A × h
1m T1 T
\ = 2
distance 2 Ah1 Ah2
Crms = = = 1000 m/s
time 1/ 500 T h 400 4
Þ h2 = 2 1 = ´1 = m
3RT T1 300 3
(a) But Crms =
M when the gas is compressed without heat exchange, the
process is adiabatic
3 ´ 25 / 3 ´ T g -1
Þ 1000 = Þ T = 160 K æV ö æ 4ö
2/5
4 ´ 10-3 \ T '1 = T2 ç 2 ÷ = 400 ç ÷ K
èV ø 1 è 3ø
(b) Average kinetic energy of an atom of a monoatomic
38. Rate of heat produced = F × v
3
gas = kT = (6 p h r v) v [Q Viscous F = 6 p h r v]
2
2
é 2 (s - r)r 2 g ù
3
2
3
2
(
\ Eav = kT = ´ 1.38 ´ 10-23 ´ 160 ) = (6 p h r) ê
êë 9 h
ú
úû
= 3.312 × 10–21 Joules
é 2 (s - r)r 2 g ù
m
RT êQ Terminal velocity = ú
(c) From gas equation PV = 9 h
M ëê ûú
Þ Rate of heat produced µ r5
PVM 100 ´ 1 ´ 4
m= = Þ m = 0.3012 gm 39. From the figure it is clear that emission takes place from the
RT 25 / 3 ´ 160 surface at temperature T2 (circular cross section). Heat
35. When container is stopped, velocity decreases by v0. conduction and radiation through lateral surface is zero.
1 Heat conducted through rod is
Therefore change in kinetic energy = (nm)v02 ... (i)
2 KA(T1 - T2 ) Dt
Q=
l
Heat & Thermodynamics and Gases P-S- 109

Energy emitted by surface of rod in same time Dt, is Energy used to raise the temperature = mcDT
4 4 = 1 × 400 × (T – 20) ... (i)
E = e s A (T2 - Ts ) Dt where T°C is the final temperature of the cylinder.
Since rod is at thermal equilibrium therefore E = Q Energy used for work done
KA(T1 - T2 )Dt = Patm (V2– V1) = 105 (V2 – V1) ... (ii)
hence, = e s A (T24 - Ts4 ) Dt The final volume V2 = V1 [1 + g (T – 20)]
l Þ V2 – V1 = V1g (T – 20) ... (iii)
e s (T24 - Ts4 ) l From (ii) and (iii),
Þ T1 – T2 = Energy used for work done = 105 V1g (T – 20)
K
1 é m 1 ù
Here T2 - Ts = DT and Ts >> DT = 105 × ´ 9 ´ 10 -5 (T - 20) êQV1 = = úû
9000 ë d 9000
esl é
T1 - ( DT + Ts ) = ë ( DT + Ts )4 - Ts 4 ùû = 0.001 (T – 20) ... (iv)
K \ Heat supplied to the cylinder
= 400 (T – 20) + 0.001 (T – 20)
esl éæ DT ö4 ù 20,000 = 400.001 (T – 20)
T1 - ( DT + Ts ) = ´ Ts 4 êç1 + - 1ú
K êëè Ts ÷ø úû Þ T = 69.99°C » 70°C
(b) Work done = 0.001 (69.99 – 20) = 0.0499 J
esl é 4D T ù (c) Change in internal energy = 20,000 – 0.0499 = 19999.95 J.
T1 - ( DT + Ts ) = ´ Ts 4 ê1 + - 1ú
K ë Ts û 42. (1) Heat lost by steam at 100°C to change to 100°C water
mLvap = 0.05 × 2268 × 1000 = 1,13,400 J
4es l 3 (2) Heat lost by 100°C water to change to 0°C water
or T1 – (Ts + DT) = Ts DT
K = 0.05 × 4200 × 100 = 21,000 J
(3) Heat required by 0.45 kg of ice to change its temperature
æ 4 e s l Ts3 ö from 253 K to 273 K
or T1 – Ts = ç + 1÷ DT
ç K ÷ = m × Sice × DT = 0.45 × 2100 × 20 = 18,900 J
è ø
(4) Heat required by 0.45 kg ice at 273 K to convert into
æ 4 esl Ts3 ö 0.45 kg water at 273 K
\ The proportionality constant = çç 1 + ÷ = mLfusion = 0.45 × 336 × 1000 = 151,200 J
è K ÷ø From the above data it is clear that the amount of heat
40. Initially, at temperature T required by 0.45 kg of ice at 253 K to convert into 0.45 kg of
FB = mg water at 273 K (1,70,100 J) cannot be provided by heat lost
Axrl g = ALrb g by 0.05 kg of steam at 373 K to convert into water at 273 K.
Therefore the final temperature will be 273 K or 0°C.
FB At T
F'B
F. Match the Following
r
o
1. (A) ® (q) : JK is a isovolumic process. Therefore work done
x x is zero. But there is decrease in pressure. Now DQ = DU +
DW. Therefore DQ = DU. In this case DU = nCvDT and P µT.
mg mg
Since pressure has decreased means temperature has
rl
decreased. Therefore DU is negative and so is DQ.
(B) ® (p, s) : KL is a isobaric process. Pressure is constant.
Þ xr l = Lrb ... (i) The volume is increasing therefore DW > 0. Also there is an
At temperature T + DT increase in temperature. For both the case heat is absorbed.
FB ' = mg Therefore DQ > 0.
(C) ® (s) : LM is a isovolumic process. Therefore work
A ' xr 'l g = ALrb g [mg remains the same as above] done is zero. The process is accompanied by increases in
Now, A' = A (1 + 2aDT) pressure. In this case, the temperature has increased and
r 'l = rl (1 - gDT ) therefore DU > 0. Therefore DQ > 0.
(D) ® (q, r) : The process MJ is accompained with decrease
\ A(1 + 2a DT) x rl (1 - gDT ) g = ALrbg in volume. Therefore DW < 0. Also from the graph we can
Þ xrl (1 + 2a DT )(1 - gDT ) = Lrb conclude that the temperature in the process decreases.
Therefore DU is also negative
Þ xrl (1 + 2a DT ) (1 - gDT ) = xrl [From (i)]
Þ DQ < 0.
Þ 1 + 2a DT – g DT = 1 2. (A)-(q) : As the ideal gas expands in vacuum, no work is
Þ g = 2a done (W = 0). Also the container is insulated therefore no
41. (a) Heat supplied to the cylinder = Energy used to raise heat is lost or gained (Q = 0). According to first law of
the temperature of cylinder + Energy used for work
thermodynamics
done by the cylinder.
EBD_7036
P-S- 110 Topic-wise Solved Papers - PHYSICS
DU = Q + W From these two interpretation we can comfortably conclude
\ DU = 0 that the gas gains heat during the process.
Þ There is no change in the temparature of the gas (s) is the correct option.
(B)-(p, r) : Given PV2 = constant ....(i) 3. A-p,r,t; B-p,r; C-q,s; D-r,t
PV (A) Process A ® B
Also for an ideal gas = constant This is an isobaric process in which the volume of the gas
T decreases. Therefore work is done on the gas.
From (i) & (ii) V × T = constant W = P (3V – V) = 2PV
As the gas expands its volume increases and temperature Also temerature at B is less than temperature at A
decreases \ Heat is lost & internal energy decreases.
\ (p) is the correct option (p, r, t) are correct matching
To find whether heat is released or absorbed let us find a (B) Process B ® C
relationship between Q and change in temperature DT. This is an isovolumic/isochoric process in which the pressure
We know that Q = n C D T ...(i) decreases
where C = molar specific heat Here temperature at B is less than temperature at C.
Also for a polytropic process we have
\ Heat is lost and internal energy decreases.
R (p, r) are correct matching.
C =Cv + and PVn = constant (C) Process C ® D
1- n
Here PV2 = Constant. Therefore n = 2 This is isobaric expansion where temperature at D is greater
than temperature at C. Therefore internal energy increases
R and heat is gained.
\ C = Cv + = Cv – R
1- 2 (q, s) are correct matching
3 (D) D ® A
For monoatomic gas Cv = R This is a process in which volume decreases. Therefore
2
work is done on the gas.
3 R Applying PV = nRT
\C= R – R =
2 2 9 PV
Substituting this value in (1) we get for D P (9V) = 1 RTD \ TD =
R
R
Q=n × × DT.. 9 PV
2 for A 3P (3V) = 1RTA \ TA =
In this case the temperature decreases i.e. DT is negative. R
Therefore Q is negative. This in turn means that heat is lost Þ TA = TD \DU = 0
by the gas during the process. (r) is the correct option. Now, DQ = DU + W \ DQ = W.
(C)-(p, s) : Proceeding in the same way we get in this case The energy obtained by the gas by work done on it is lost to
V1/3× T = constant the surroundings as D U = 0.
Þ As th e gas expands and volume increases, the \ (r, t) are correct matching.
temperature decreases. Therefore (p) is the correct option 4. (a) WGE = P0 (V0 – 32 V0) = – 31 P0V0
4 WGH = P0 (8V0 – 32V0) = – 24 P0V0
In this process, x = .
3 P0 (8V0 ) - 32P0 (V0 )
(WFH)adiabatic = = 36P0 V0
R 3 3R 3 -3R 5
\C = Cv + = R+ = R–3R = 1-
4 2 -1 2 2 3
1-
3
32V0
(WFG) isothermal = 1(32 P0V0) loge
æ -3R ö V0
\Q=nç ÷ Δt
è 2 ø = 32 P0V0 loge 25
As DT is negative, Q is positive. This in turn means that = 160 P0V0 loge2
heat is gained by the gas during the process (s) is the correct (a) is the correct option
option.
D ( PV) G. Comprehension Based Questions
(D)-(q, s) : Also DT = 1. (a) NOTE : When the piston is pulled out slowly, the
nR
Here D(PV) is positive \ DT is positive pressure drop produced inside the cylinder is almost
\ temperature increase s (q) is the correct option instantaneously neutralised by the air entering from
From the graph it is clear that during the process the pressure outside into the cylinder (through the small hole at the
top).
of the gas increases which shows that the internal energy
Therefore, the pressure inside the cylinder is P0
of the gas has increased. Also the volume increases which
throughout the slow pulling process.
means work is done by the system which needs energy.
Heat & Thermodynamics and Gases P-S- 111

2. (d) KEY CONCEPT : The condition for equilibrium of the 6. (b) Buoyancy force = weight of fluid displaced
piston is Seal = (mass of fluid displaced) g
Mg = (P0 – p) pR2 = Vrl g ...(ii)
y 2L
p where V = Volume of fluid displaced
- Mg = Volume of the bubble.
Þ p= + P0
pR 2 P0 Now, PV = nRT
NOTE : Since the cylinder is thermally conducting, the nRT nRT
temperature remains the same. ÞV = =
P P0 + ( H - y )rl g
Therefore
Where P is pressure of the bubble at an arbitrary
P0 location distant y from the bottom.
P0 × (2L × pR2) = py × pR2 Þ y = ´ 2L
p Substituting the value of tempertaure from equtaion (i)
we get
P0 é P ´ pR 2 ù 2
= ´ 2L = ê 0 2 ú ´ 2L
é Mg ù êë P0 pR - Mg úû nR T [ P + ( H - y )rl g ]5
P - V= ´ 0 0
ê 0 ú [ P0 + ( H - y )rl g ] 2
ë pR 2 û
[ P0 + H rl g ]5
3. (c) At equilibrium, p = P
Þ p = P0 + (L0 – H) rg ... (i) nRT0
= ...(iii)
Also P0 × (pR2 L0) = p [pR2 (L0 – H)] 3 2

L0 P0
[ P0 + ( H - y )rl g] 5 [ P0 + H rl g ] 5
Þ p= ... (ii) P0
From (ii) and (iii)
L0 - H
p nRT0rl g
From (i) and (ii) Buoyancy force =
L0 3 2
L0 P0
= P0 + ( L0 - H ) r g P
P H
[ P0 + ( H - y )rl g ]5 [ P0 + H rl g ] 5
L0 - H
7. (d) Heat lost by monatomic gas at constant volume
Þ L0P0 = P0 (L0 – H) + (L0 – H)2 r g = Heat gained by diatomic gas at constant pressure
Þ rg (L0 – H)2 + P0 (L0 – H) – L0P0 = 0
\ nCv1 ( 700 - T ) = nC p2 (T - 400)
4. (d) The forces acting besides buoyancy force are
(a) Force of gravity (vertically downwards) 3 7
R ( 700 - T ) = R (T - 400)
(b) Viscous force (vertically downwards) 2 2
Force due to pressure of the liquid is the buoyant force. Þ 2100 - 3T = 7T - 2800
5. (b) It is given that the bubble does not exchange any heat Þ 10T = 4900
with the liquid. This means that while the bubble moves \ T = 490 K
up and expand, the process is adiabatic. 8. (d) In this case both the gases are at constant pressure.
For adiabatic expansion the pressure -temperature
relationship is \ nC p1 ( 700 - T ) = nC p2 (T - 400 )
1-g 5 7
R ( 700 - T ) = R (T - 400 )
éP ù g 2 2
T2 = T1 ê 1 ú
ë P2 û
3500 – 5T = 7T – 2800
Þ 12T = 6300
Here T1 = T0, P1 = P0 + H rl g , \ T = 525 K
5 Applying first law of thermodynamics
P2 = P0 + ( H - y )rl g , g =
3 DW1 + DU1 = DQ1
5
1- 5/3 and DW2 + DU 2 = DQ2
é P0 + H rl g ù 3
\ T2 = T0 ê ú As the gas two system is thermally insulated, therefore
ë P0 + ( H - y )rl g û
DQ1 + DQ2 = 0
-2 3
´
é P0 + H rl g ù3 5 - ( DW1 + DW2 ) = DU1 + DU 2
= T0 ê ú
ë P0 + ( H - y )rl g û = nCv1 ( 525 - 700 ) + n2 Cv2 ( 525 - 400 )
2 3R 5R
é P + ( H - y )rl g ù 5 = -2 ´ ´ 175 + 2 ´ ´125
2 2
T2 = T0 ê 0 ú
ë P0 + H rl g û = –525R + 625R = – 100 R
Therefore, total work done = –100 R
EBD_7036
P-S- 112 Topic-wise Solved Papers - PHYSICS
H. Assetion & Reason Type Questions
g -1
1. (b) Statement 1 : The total kinetic energy of n moles of gas æV ö
T1V1g -1 = T2V2 g -1
Þ T1 = T2 ç 2 ÷
3 èV ø 1
is K = nRT
2
But PV = nRT V1
Here g = 1.4 (for diatomic gas). V2 = , T1 = Ti , T2 = aTi
\ K = 1.5 PV 32
Statement one is true. 1.4 -1 0.4
Statement 2 : The molecules of a gas collide with each é1ù é1ù aTi
\ Ti = aTi ê ú = aTi ê 5 ú = \a=4
other and the velocities of the molecules change due ë 32 û ë2 û 4
to collision. 5. (3) We know that
But statement 2 is not a correct explanation of
statement 1. mg / A mg l
Y= = ....(1)
Dl / l ADl
I. Integer Value Correct Type Also Dl = l a DT ...(2)
A 400°C B From (1) and (2)

1. lx mg l mg
0°C 100°C Y= =
Al a DT A a DT
P
For heat flow from P to 0 YA a DT 1011 ´ p(10-3 )2 ´ 10-5 ´10
\ m= = =p»3
dm1 KA 400 g 10
Lf= ..... (i)
dt lx 6. (2) Applying first law of thermodynamics to path iaf
For heat flow from P to B Qiaf = DUiaf + Wiaf
dm2 KA 300 é dm1 dm2 ù 500 = DUiaf + 200 \ DUiaf = 300 J
Lvap = ..... (ii) ê Given =
dt 10 x - lx ë dt dt úû Now,
On solving (i) and (ii), we get l = 9 Qibf = DUibf + Wib + Wbf
2. Heat supplied = Heat used in converting m grams of ice = 300 + 50 + 100
from –5°C to 0°C + Heat used in converting 1 gram of ice at Qib + Qbf = 450 J ...(1)
0°C to water at 0°C Also Qib = DU ib + Wib
2100 1 ´ 3.36 ´ 105 \ Qib = 100 + 50 = 150 J ...(2)
Þ 420 = m ´ ´5+
1000 1000
84 Qbf 300
Þ 420 = m ´ 10.5 + 336 \m= = 8 grams From (1) & (2) = =2
10.5 Qib 150
T2
PA AA TA4 A l4
T1 7. (2) = = A´ B
PB AB TB4 AB l 4A
r1= 6cm r2= 18cm 1 1 1
3. l éA é R 2 P ù 4 é 400 ´ 400 ù 4
PB ù 4
\ A =ê A ´ ú =ê A ´ B ú =ê ú
lB ë AB PA û
2
êë RB PA úû ë 104 û
A
l
lm = 500 nm
B \ A =2
1
lm =1500 nm lB
2

Rateof total energy radiated by A 8. (9) Here P µ T 4 or P = P0T 4


Rateof total energy radiated by B
P
sT14 (4pr12 )
æT ö æ r ö
4 2 \ log 2 P = log 2 P0 + log 2 T 4 \ log 2 = 4log 2 T
= ç 1÷ ´ç 1÷ P0
=
sT2 (4pr2 ) è T2 ø è r2 ø
4 2

P
æ lm ö æ r ö 2
4
é T1 l m2 ù At T = 487°C = 760 K, log 2 P = 4log 2 760 = 1 … (1)

2
÷ ç ÷
1
êQ = by Wein 's law ú 0
è l m1 ø è r2 ø êë T2 l m1 úû At T = 2767°C = 3040K,
2
æ 1500 ö æ 6 ö
4
r
=ç loge = 4log 2 3040 = 4 log 2 (760 ´ 4)
è 500 ÷ø çè 18 ÷ø
=9 r0
4. For an adiabatic process, the temperature-volume
relationship is = 4 éëlog 2 760 + log 2 22 ùû
= 4log 2 760 + 8 = 1 + 8 = 9
Heat & Thermodynamics and Gases P-S- 113

1. (a) All reversible engines working for the same temperature relationship is given by
of source and sink have same efficiencies. If the
g
temperatures are different, the efficiency is different. ....(ii)
1-g
2. (b) Heat required for raising the temperature of the whole P1-g T g = constant Þ PT = constt.
body by 1ºC is called the thermal capacity of the body.
3. (b) Pyrometer is used to detect infra-red radiation. g 3
From (i) and (ii) 1 - g = -3 Þ g = -3 + 3g Þ g =
4. (a) Black board paint is quite approximately equal to black 2
bodies. 13. (c) Work is a path function. The remaining three parameters
5. (c) Since pressure and volume are not changing, so are state function.
temperature remains same.
6. (c) When water is cooled to form ice, energy is released T2 (273 + 27) 300 1 2
14. (b) h = 1- = 1- = 1- = 1- =
from water in the form of heat. As energy is equivalent T1 (273 + 627) 900 3 3
to mass therefore when water is cooled to ice, its mass
decreases. W W 2 2 2
But h = \ = Þ W = ´ Q = ´ 3 ´ 106
8RT Q Q 3 3 3
7. (d) vrms =
pM 6 6
TH 2 TO2 = 2 ´ 106 cal = 2 ´ 10 ´ 4.2 J = 8.4 ´ 10 J
For vrms to be equal M = 15. (d) Wein’s law correctly explains the spectrum
H2 M O2
Here M H 2 = 2; M O2 = 32 ; dQ
16. (d) - µ (Dq)
dt
TO 2 = 47 + 273 = 320 K
n1 + n2 n n
TH 2 17. (c) = 1 + 2
320 g -1 g1 - 1 g 2 - 1
\ = Þ TH 2 = 20 K
2 32
1 +1 1 1 3
T Þ = + Þ g=
8. (c) h = 1- 2 g -1 5 7 2
T1 -1 -1
3 5
For h = 1 or 100 %, T2 = 0 K. 18. (d) 2
E = s AT 4 ; A µ R \ E µ R T
2 4
The temperature of 0 K (absolute zero) can not be
obtained . E R22 T24
9. (c) If n1 moles of adiabatic exponent g1 is mixed with n2 \ 2 =
E1 R12 T14
moles of adiabatic exponent g 2 then the adiabatic
component of the resulting mixture is given by E2 (2R)2 (2T )4
Þ = = 64
n1 + n2 n n E1 R 2T 4
= 1 + 2
g -1 g1 - 1 g 2 - 1 19. (b) Internal energy and entropy are state function, they
do not depend upon path taken.
1 +1 1 1 20. (a) Here Q = 0 and W = 0. Therefore from first law of
= + 2 5 3
g -1 7 5 \ = + =4 thermodynamics DU = Q + W = 0
-1 -1 g -1 2 2
5 3 \ Internal energy of the system with partition =
6 3 Internal energy of the system without partition.
\ 2 = 4g - 4 Þ g = = n1Cv T1 + n2 Cv T2 = (n1 + n2 )Cv T
4 2
10. (a) The energy radiated per second is given by E = esT 4 A n1T1 + n2T2
\T =
For same material e is same. s is stefan's constant n1 + n2

E1 T14 A1 T14 4pr12 (4000)4 ´12 1 PV


1 1 PV
\ = = = = But n1 = and n2 = 2 2
E2 T24 A2 T24 4pr22 RT1 RT2
(2000)4 ´ 42 1
11. (a) This is a statement of second law of thermodynamics PV
1 1 ´ T + P2V2 ´ T
1 2
RT1 RT2 T T ( PV + P V )
12. (d) P µ T 3 Þ PT -3 = constant ....(i) \T= = 1 2 11 2 2
PV P V
1 1+ 2 2 1 1T2 + P2V2T1
PV
But for an adiabatic process, the pressure temperature
RT1 RT2
EBD_7036
P-S- 114 Topic-wise Solved Papers - PHYSICS
21. (d) The thermal resistance
16 16
x 4x 3x +
+ = 4 32 = 16 / 4 + 16 / 32
KA 2 KA KA 5
r -1 1.4 - 1
-1
dQ DT (T2 - T1 ) KA 1 ì A(T2 - T1 ) K ü 3
\ = = = í ý
dt 3x 3x 3î x þ \ g = 1.62
KA
27. (b) Total power radiated by Sun = sT 4 ´ 4pR 2
1
\f =
3 sT 4 ´ 4pR2
The intensity of power at earth's surface =
22. (b, c)First law is applicable to a cyclic process. Concept of 4pr 2
entropy is introduced by the second law.
sT 4 R 2
23. (d) T-dT Total power received by Earth = 2
(pr02 )
dr r
28. (c) Heat lost by He = Heat gained by N2
T1 r1 r
n1Cv1 DT1 = n2Cv2 DT2
T2
r2
3 é7 ù 5 é 3
ù
Consider a shell of thickness (dr) and of radius (r) and
R T - Tf
2 êë 3 0 úû = 2 R ëT f - T0 û Þ T f = 2 T0
the temperature of inner and outer surfaces of this shell
be T, (T – dT) nRDT 1000 ´ 8.3 ´ 7
29. (a) W= Þ -146000 =
KA[(T - dT ) - T ] - KAdT 1- g 1- g
H= =
dr dr 58.1 58.1
or 1 - g = - Þ g = 1+ = 1.4
2 dT 146 146
H = -4pKr (Q A = 4pr 2 ) Hence the gas is diatomic.
dr
r2 T2 30. (b) For path iaf, DU = Q - W = 50 – 20 = 30 cal.
1
Then, ( H ) ò dr = -4pK ò dT For path ibf W = Q – DU = 36 – 30 = 6 cal.
r1 r2 T1 31. (c) The efficiency (h) of a Carnot engine and the coefficient
of performance (b) of a refrigerator are related as
é1 1 ù
H ê - ú = -4pK [T2 - T 1 ] 1
ë r1 r2 û 1-
1- h 1 10
-4pKr1r2 (T2 - T1 )
b= Here, h = \ b= = 9.
or H = h 10 æ 1ö
(r2 - r1 ) çè 10 ÷ø
24. (b) Change in internal energy do not depend upon the
Q2
path followed by the process. It only depends on initial Also, Coefficient of performance (b) is given by b =,
and final states i.e., DU1 = DU2 W
where Q2 is the energy absorbed from the reservoir.
1 3
25. (d) Q1 = T0 S 0 + T0 S0 = T0 S 0
2 2
T or, 9 = Q2 \ Q2 = 90 J.
Q2 = T0(2S0 – S0) 10
= T0S0 32. (d) T1 l1 T l2 T2
2T0
and Q3 = 0 Q1
Q3 A
T0 K1 K2
Q2
S K1 A(T1 - T ) K2 A(T - T2 ) ,
S0 2S 0 =
Q2 TS 1 l1 l2
h= 1 - = 1- 0 0 =
Q1 3 3
T S K1T1l 2 + K 2T2 l1 K l T + K 2 l1T2
2 0 0 \ T = = 1 2 1 .
K 2 l1 + K1l 2 K1l 2 + K 2 l1
n1 + n2 n n 33. (b) According to Mayer's relationship CP – CV = R
26. (a) = 1 + 2
r -1 r1 - 1 r2 - 1 CP CV R
\ - = Here M = 28.
M M M
Heat & Thermodynamics and Gases P-S- 115

2
gRT
34. (a) The speed of sound in a gas is given by v = \ T = (32) 5 .T Þ T1 = 4T2
M 1 2

vO2 g O2 T2 T2 1 3
M 1.4 4 Now, efficiency = 1 - = 1- = 1 - = = 0.75.
\ = ´ He = ´ = 0.3237 T1 4T2 4 4
vHe M O2 g He 32 1.67
43. (c) Here, work done is zero.
vO 2 460 So, loss in kinetic energy = change in internal
\ vHe = = = 1421 m / s energy of gas
0.3237 0.3237
35. (a) Same as A . 20 1 2 R
mv = nCv DT = n DT
36. (a) The heat flow rate is given by 2 g -1
dQ kA(q1 - q)
= 1 2 m R Mv 2 ( g - 1)
dt x mv = DT \ DT = K
2 M g -1 2R
x dQ x dQ
Þ q -q = Þ q = q1 - n1
1 kA dt 44. (a) Number of moles of first gas = N
kA dt
A
where q1 is the temperature of hot end and q is
n2
temperature at a distance x from hot end. Number of moles of second gas = N
The above equation can be graphically represented by A

option (a) . n3
37. (b) A to B is an isobaric process. The work done Number of moles of third gas = N
A
W = nR (T2 - T1 ) = 2R (500 - 300) = 400 R
If there is no loss of energy then
38. (a) Work done by the system in the isothermal process P1V1 + P2V2 + P3V3 = PV
PD n1 n n
DA is W = 2.303 nRT log10 RT1 + 2 RT2 + 3 RT3
PA NA NA NA

n1 + n2 + n3 n T + n T + n3T3
1 ´ 105 = RTmix T = 1 1 2 2
= 2.303 ´ 2 R ´ 300 log10 = – 414 R NA mix n1 + n2 + n3
2 ´ 105
45. (d) Efficiency of engine
Therefore work done on the gas is + 414 R.
1 T
39. (a) The net work in the cycle ABCDA is = 1 - 2 and h = 1 - T2 - 62 = 1
6 T1 2
W = W AB + WBC + WCD + WDA T1 3

5
PB \ T1 = 372 K and T2 = × 372 = 310 K
= 400 R + 2.303nRT log + (-400 R) - 414R 6
PC
46. (a) DU = DQ = mcDT = 100 × 10–3 × 4184 (50 – 30) » 8.4 kJ

2 ´ 105 F/S FL
= 2.303 ´ 2R ´ 500log - 414 R 47. (d) Y= Þ DL =
1 ´ 105 DL / L SY
= 693.2 R – 414 R = 279.2 R FL
\ LaDT = [Q DL = LaDT ]
mass 1 SY
40. (a) Volume = = m3
density 4 \ F = SY aDT
\ The ring is pressing the wheel from both sides,
5 5 1 \ Fnet = 2F = 2YSaDT
K.E = PV = ´ 8 ´ 104 ´ = 5 ´ 104 J
2 2 4
48. (a) Heat given to system = (nCV DT ) A® B + (nC p DT ) B®C
41. (c) (The relation R = R0 (1 + a Dt ) is valid for small values
of Dt and R0 is resistance at 0°C and also (R – R0) é3 ù é5 ù
= ê (nR DT )ú + ê (nR DT )ú
should be much smaller than R0. So, statement (1) is ë2 û A®B ë 2 û B ®C
wrong but statement (2) is correct.
é3 ù é5 ù
= ê ´ V0 DP ú + ê ´ 2 P0 ´ V0 ú
42. (b) T1V g-1 = T2 (32V )g-1 Þ T1 = (32) g-1.T2 ë2 û A® B ë 2 û
7 2 13
For diatomic gas, g = \ g -1 = = P0V0
5 5 2
EBD_7036
P-S- 116 Topic-wise Solved Papers - PHYSICS

and W0 = P0V0
100°C
Work PV Copper
h= = 0 0 ´ 100 = 15.4%
heat given 13 P V
0 0 T
2
B Brass
49. (a) Newton's law of cooling Steel
0°C 0°C
dq
= - k(q - q0 ) Þ dq = - kdt
dt (q - q0 ) If the junction temperature is T, then
QCopper = QBrass + QSteel
Integrating
0.92 ´ 4(100 - T ) 0.26 ´ 4 ´ (T - 0)
Þ log(q - q0 ) = -kt + c = +
46 13
Which represents an equation of straight line.
Thus the option (a) is correct. 0.12 ´ 4 ´ (T - 0)
T2 T 12
50. (c) 0.4 = 1 - and 0.6=1– 2 Þ 200 – 2T = 2T + T
500 T1
Þ T = 40°C
on solving we get T2 = 750 K
51. (b) Same as in A -51 0.92 ´ 4 ´ 60
\ QCopper = = 4.8 cal/s
52. (c) According to Newton’s law of cooling, the temperature 46
goes on decreasing with time non-linearly. 56. (d) In cyclic process, change in total internal energy is
zero.
stress DUcyclic = 0
53. (a) Young's modulus Y =
strain
stress = Y ´ strain 5R
DUBC = nCv DT = 1 ´ DT
Stress in steel wire = Applied pressure 2
Pressure = stress = Y × strain Where, Cv = molar specific heat at constant volume.
For BC, DT = –200 K
DL \ DUBC = –500R
Strain = = α D T (As length is constant)
L 57. (d) The entropy change of the body in the two cases is
= 2 × 1011 × 1.1 × 10–5 × 100 same as entropy is a state function.
= 2.2 × 108 Pa 1æUö
58. (a) As, P = ç ÷
54. (c) Pressure at interface A must be same from both the 3èVø
sides to be in equilibrium.
U
But = KT 4
V
1 4
So, P = KT
3
R d2 Rsina
a uRT 1
or = KT 4 [As PV = u RT]
a Rcosa R V 3
Rsin a – Rsin a
A 4 3 3
d1 pR T = constant
3
1
\ ( R cos a + R sin a )d2 g = ( R cos a - R sin a )d1 g Therefore, Tµ
R
d1 cos a + sin a 1 + tan a 1
Þ = = 59. (a) t =
d 2 cos a - sin a 1 - tan a æ N ö 3RT
2pd2 ç ÷
èVø M
55. (c) Rate of heat flow is given by,
V
KA(q1 - q 2 ) tµ
Q= T
l
As, TVg–1 = K
Where, K = coefficient of thermal conductivity So, t µ Vg + 1/2
l = length of rod and A = Area of cross-section of rod g +1
Therefore, q =
2
Heat & Thermodynamics and Gases P-S- 117

60. (c) The equation for the line is


dT 3P0 V0 - 2P0 V
= =0
dv nRV0
P

3Po 3V0 3P0


V= \ p= [From (i)]
2 2
c
2Po q 9Po Vo
Po \ Tmax = [From (iii)]
q 4nR
Po
Vo 1
V 61. (c) Time lost/gained per day = µ Dq ´ 86400 second
Vo 2Vo 2
1
12 = a (40 – q) ´ 86400 .... (i)
- P0 2
P = V V + 3P
0 1
4 = a (q – 20) ´ 86400 ....(ii)
-P0 2
[slope = , c = 3P0]
V0 40 – q
On dividing we get, 3 =
PV0 + P0V = 3P0V0 ...(i) q – 20
But pv = nRT 3q – 60 = 40 – q
nRT 4q = 100 Þ q = 25°C
\p= ...(ii) 62. (d) For a polytropic process
v
R R
nRT C = Cv + \ C - Cv =
From (i) & (ii) V0 + P0V = 3P0V0 1- n 1- n
v
\ nRT V0 + P0V2 = 3P0V0 R R
\ 1- n = \ 1- =n
...(iii) C - Cv C - Cv
dT
For temperature to be maximum =0 C - Cv - R C - C v - Cp + Cv
dv \ n= =
C - Cv C - Cv
Differentiating e.q. (iii) by 'v' we get
dT C - Cp
nRV0 + P0(2v) = 3P0V0 = (Q C p - C v = R )
dv C - Cv

dT
\ nRV0 = 3P0V0 – 2 P0V
dv
EBD_7036
Simple Harmonic Motion
10
P-S- 118 Topic-wise Solved Papers - PHYSICS

(Oscillations)
Section-A : JEE Advanced/ IIT-JEE
A 1. 0.06
C 1. (d) 2. (d) 3. (a) 4. (a) 5. (a) 6. (b) 7. (d) 8. (c) 9. (d)
10. (b) 11. (a)
D 1. (c) 2. (b, c) 3. (b) 4. (d) 5. (b) 6. (a) 7. (a, c) 8. (a, b, c) 9. (a, d)
10. (b, d) 11. (a,b,c)

k m1 g 1 2p 1.803R
E 1. 1.6 kg 2. , 3. 2.83 sec. 4. (a) q = tan -1 (b)
m2 k 5 g

1 -1 3(d 2 - d1 ) g g
5. (i) s (ii) 0.02p m/s (iii) 3.95 × 10–4 J 6.
2d1 L 8. y* =
p w2

F 1. (A) ® p; (B) ® q, r; (C) ® p; (D) ® q, r 2. (A) ® p; (B) ® q, r, s; (C) ® s; (D) ® q


G 1. (c) 2. (b) 3. (d) 4. (d) 5. (c) 6. (b)

Section-B : JEE Main/ AIEEE


1. (c) 2. (b) 3. (b) 4. (c) 5. (c) 6. (d) 7. (c) 8. (a) 9. (a) 10. (b) 11. (a) 12. (b)
13. (c) 14. (b) 15. (c) 16. (b) 17. (a) 18. (a) 19. (a) 20. (a) 21. (b) 22. (b) 23. (a) 24. (a)
25. (d) 26. (c) 27. (d) 28. (c) 29. (c) 30. (d) 31. (c) 32. (d) 33. (d)

Section-A JEE Advanced/ IIT-JEE


A. Fill in the Blanks U(x)
1. x = 0.04 m, K.E. = 0.5 J and P.E. = 0.4 J
T.E. = (0.5 + 0.4) J = 0.9 J
1 1 C 1 B
Now, T.E. = mw 2 a 2 = m ´ 4 p 2 v 2 a 2
2 2 A
1 2 25 25 x=0 x
Þ 0.9 = ´ 0.2 ´ 4p ´ ´ ´ a 2 Þ a = 0.06m From the graph it is clear that the potential energy is
2 p p
minimum at x = 0. Therefore, x = 0 is the state of stable
C. MCQs with ONE Correct Answer equilibrium. Now if we displace the particle from x = 0
then for displacements the particle tends to regain the
1. (d) Both the bodies oscillate in simple harmonic motion,
2kx
for which the maximum velocities will be position x = 0 with a force F = . Therefore for small
Given that v1 = v2 Þ a1w1 = a2w2 x2
e
values of x we have F µ x.
2p 2p 3. (a) As shown in the figure, gsina is the pseudo acceleration
\ a1 × = a2 ´
T1 T2 applied by the observer in the accelerated frame
Y
m
2p
a1 T1 k1 k2 g sin a X
Þ = = = g sin a sin a
a
a2 T2 m k1 g cos a sin a
2p
k2
g
2. (d) Let us plot the graph of the mathematical equation
a
dU 2
U (x) = K éê1 - e - x ùú , F = -
2
= 2kxe - x
ë û dx ax = gsinacosa
Simple Harmonic Motion (Oscillations) P-S- 119

ay = g – gsin2a = g(1 – sin2 a) = gcos2a 8. (c)


Figure shows the rod at an angle lq
a= ax2 + a 2y q with respect to its equilibrium 2
position. Both the springs are
= g 2 sin 2 a cos 2 a + g 2 cos 4 a lq q
stretched by length . l/2
= g cosa 2 2
sin a + cos a = g cos a 2 M
The restoring torque due to the
L springs
\ T = 2p t = – 2 (Restoring force) ×
g cos a
perpendicular distance lq
NOTE : Whenever point of suspension is accelerating 2
æ lq ö l l 2
L t = -2k ç ÷ ´ = - k q ... (i)
use T = 2p è 2 ø 2 2
geff If I is the moment of inertia of the rod about M then
4. (a) The velocity of a body executing S.H.M. is maximum at d 2q
its centre and decreases as the body proceeds to the t = Ia = I … (ii)
extremes. Therefore if the time taken for the body to go dt 2
From (i) & (ii) we get
from O to A/2 is T1 and to go from A/2 to A is T2 then
obviously d 2q l 2q d 2q k l2 -k l2
I = -k Þ = - q = q
T1 < T2 dt 2 2 dt 2 I 2 M l 2 /12 2
5. (a) NOTE : In S.H.M., at extreme position, P.E. is maximum
when d 2q 6k
Þ 2
=- q
t = 0, x = A. dt M
i.e., at time t = 0, the particle executing S.H.M. is at its Comparing it with the standard equation of rotational
extreme position. SHM we get
Therefore P.E. is max. The graph I and III represent the d 2q
above characteristics.
2
= -w2q Þ w 2 = 6k Þ w = 6k
6. (b) y = kt2 dt M M
2 1 6k
dy d y 6k
\ = 2 kt or = 2k Þ 2 pv = Þ v=
dt dt 2 M 2p M
or ag = 2m / s 2 (Q k = 1 m/s2 given) 9. (d) k1 k2

l P M
We know that T = 2p Case (i)
g x1

T12 g2 T12 12 6 A
\ = Þ = = M
T22 g1 T22 10 5 P
Case (ii)
[Q g1 = 10 m/s2 and g2 = g + 2 = 12 m/s2] In case (ii), the springs are shown in the maximum
7. (d) From the graph it is clear that the amplitude is 1 cm and compressed position. If the spring of spring constant
the time period is 8 second. Therefore the equation for k1 is compressed by x1 and that of spring constant k2
the S.H.M. is is compressed by x2 then
æ 2p ö æ 2p ö p x1 + x2 = A … (i)
x = a sin ç ÷ ´ t = 1sin ç ÷ t = sin t
èT ø è 8ø 4 k1 x1
The velocity (v) of the particle at any instant of time ‘t’ and k1 x1 = k2 x2 Þ x2 = …(ii)
is k2
From (i) & (ii)
dx d é æ p ö ù p æ pö
v= = sin ç ÷ t = cos ç ÷ t k x k A
dt dt êë è 4 ø úû 4 è 4ø x1 + 1 1 = A Þ x1 = 2
The acceleration of the particle is k2 k2 + k1
2 10. (b) Two sinusoidal displacements have amplitude A each,
d2x æ pö æ pö
= - ç ÷ sin ç ÷ t p
dt 2 è 4ø è 4ø with a phase difference of 2 . It is given that
4 3
At t = s we get sinusoidal displacement x3(t) brings the mass to a
3
2 2 complete rest. This is possible when the amplitude of
d x æ pö p 4 -p2 p
= - ç ÷ sin ´ = sin p
dt 2 è 4ø 4 3 16 3 third is A and is having a phase difference of 4 with
3
- 3p 2
= cm / s 2 respect to x1 (t) as shown in the figure.
32
EBD_7036
P-S- 120 Topic-wise Solved Papers - PHYSICS
5. (b) Let us consider the wire also as a spring. Then the case
2p /3 becomes that of two spring attached in series. The
equivalent spring constant is
A A 1 1 1
= +
keq k k '
2p /3 where k' is the spring constant of the wire
4p /3
A F/A F L
Now, Y = = ´
DL / L A D L
2V sin q 2V sin 45º
11. (a) T = \ 1= \ v = 50 ms -1 F YA
g g or = or, k ' = YA
DL L L
We know that time period of the system
D. MCQs with ONE or MORE THAN ONE Correct
m é1 1 ù
1. (c) NOTE : During one complete oscillation, the kinetic T = 2p = 2p m ê + ú
keq ë k k 'û
energy will become maximum twice.
Therefore the frequency of kinetic energy will be 2f. é1 L ù m (YA + kL )
2. (b, c) The total energy of the oscillator Þ T = 0 2p m ê + ú = 2p
ë k YA û kYA
1 2 6. (a) U (x) = k | x |3
= kA = Max. K.E.
2
[U ] ML2T -2
1 6 2 \ [k] = = = ML-1T -2
= ´ 2 ´ 10 ´ (0.01) = 100 J 3
[x ] L 3
2
As total mechanical energy = 160 J Now time period may depend on T µ (mass) x
The P.E. at equilibrium position is not zero. (amplitude)y (k)z
P.E. at mean position = (160 – 100) J = 60 J \ [M0L0T] = [M]x [L]y [ML–1 T–2]z
\ Max P.E. = (100 + 60 ) J = 160 J. = [Mx + z Ly – z T – 2z]
Extreme Mean Equating the powers, we get
position position – 2z = 1 or z = – 1/2
y – z = 0 or y = z = – 1/2
K.E. = 0 K.E. = 100J Hence T µ (amplitude) –1/2 µ a–1/2
P. E. = 160J P. E. = 60J 7. (a, c) From superposition principle
3. (b) If x is the displacement then,
y = y1 + y2 + y3
\ M w 2 x = [ rAg + k ] x = a sin wt + a sin (wt + 45°) + a sin (wt + 90°)
1/2 1/2
= a[sin wt + sin (wt + 90°] + a sin (wt + 45°)
é rAg + k ù 1 é rAg + k ù = 2a sin (wt + 45°) cos 45° + a sin (wt + 45°)
Þ w= ê Þ v=
ë M û
ú 2p êë M úû = ( 2 + 1 ) a sin (wt + 45°)
4. (d) NOTE : When a force is applied on cubical block A in = A sin (wt + 45°)
the horizontal direction then the lower block B will get
Therefore resultant motion is simple harmonic of
distorted as shown by dotted lines and A will attain a
new position (without distortion as A is a rigid body) amplitude A = ( 2 + 1 ) a
as shown by dotted lines. and which differ in phase by 45° relative to the first.

Energy in SHM µ (amplitude)2 é Q E = mA2 w 2 ù


1
A ê 2 ú
F ë û
L
Eresultant æ A ö 2
L \ = ç ÷ = ( 2 + 1) 2 = (3 + 2 2)
DL Esingle è aø
L
B Fixed \ Eresultant = (3 + 2 2) Esingle
For cubical block B 8. (a, b, c) The given equation is
F/A F L F L F x = A sin2 wt + B cos2 wt + C sin wt cos wt
h= = ´ = 2´ =
NOTE THIS STEP
DL / L A DL L DL L ´ DL
Þ F = hLDL Rearranging the equation in a meaningful form (for
hL is a constant interpretation of SHM)
Þ Force F µ DL and directed towards the mean position, A B C
2 2
oscillation will be simple harmonic in nature. x = (2sin wt ) + (2 cos wt ) + (2sin wt cos wt )
Here, Mw2 = hL 2 2 2
A B C
hL 2p hL M = [1 - cos 2wt ] + [1 + cos 2wt ] + [sin 2w t ]
Þ w= Þ = Þ T = 2p 2 2 2
M T M hL
Simple Harmonic Motion (Oscillations) P-S- 121

C 11. (a, b, d)
(a) For A = 0 and B = 0, x = sin (2wt )
2 Case (i) : Applying conservation of linear momentum.
C
The above equation is that of SHM with amplitude
2 MV1 = (M + m) V2 – (1) \ V2 = æç M ö÷ V1
and angular frequency 2w. Thus option (a) is correct. èM+mø
(b) If A = B and C = 2B then x = B + B sin 2wt
This is equation of SHM. The mean position of the
particle executing SHM is not at the origin. v1 v2
Option (b) is correct.
(c) A = – B, C = 2B; Therefore From (1)
x = B cos 2wt + B sin 2wt M (A1 × w1) = (M+m) (A2×w2)
Let B = X cos f = X sin f then
x = X sin 2wt cos f + X cos 2wt sin f K K
\ MA1 × = (M+m) A2 ×
This represents equation of SHM. M M+m
(d) A = B, C = 0 and x = A. This equation does not represents
SHM. M
\ A2 = = A1
9. (a,d) M+m
Applying t = I a 1
Also E1 = MV12
æl ö 2
For case A : mg ç sin q÷ + Mg (l sin q) = I A a A
è2 ø 1 1
and E2= (M+m)V22 = (M+m)
2 2
l/2 q M 2 V12 2
1æ M ö 2
×
(M + m)2
= ç ÷ V1
2èM+mø
l/2 sin q Clearly E2 < E1
mg m+M
l/2
The new time Period T2 = 2
lsin q
K
m+M
Case (ii) : The new time Period T2 = 2
K
Mg Also A2 = A1
æl ö
For case B : mg çè sin q÷ø + Mg (l sin q) = I B a B Here E2 = E1
2 The instantaneous value of speed at X o of the
The restoring torque in both the cases is same. combined masses decreases in both the cases.
Also I A > I B \ a A < a B
E. Subjective Problems
\ wA < wB
(a) and (d) are correct options. 1. Key Concept:
10. (b, d)
Maximum linear momentum in case 1 is (p1)max = mnmax The time period T of the spring is T = 2p M
k
b = m [aw1] ...(i)
Maximum linear momentum in case 2 is (p2)max = mnmax M
R = m [Rw2] or, 2 = 2 p .... (i)
k
\ 1 = mw2 ...(ii)
In the second case
b aw1
Dividing (i) & (ii) = M +2
1 w2 3 = 2p ... (ii)
w1 b 1 k
\ w =a= 2 \ B is a correct option. From (i) and (ii)
2 n
1 2 M 4 M
Also E1 = m w12 a 2 = Þ = Þ M = 1.6 kg.
2 3 M +2 9 M +2
1 2.
E2 = m w 22 R 2
2
E1 w12 a 2 w12 1 w12 w2 w1
\ E = 2 ´ = 2´ = 2´w =w
2 w2 R 2 w2 n w2 1 2
E1 E 2
\ w = w D is the correct option m 1

1 2 m 2
m 2
EBD_7036
P-S- 122 Topic-wise Solved Papers - PHYSICS

NOTE 4.
When mass m1 is removed then the equilibrium will get
disturbed. There will be a restoring force in the upward
direction. The body will undergo S.H.M. now. O
R
Let x1 be the extension of the spring when ( m1 + m2 ) are M o
45
q Q
°-q
suspended and x2 be the extension of spring when m1 is 45
K
removed. P N m2g

( m1 + m2 ) g m1g
\ kx1 = ( m1 + m2 ) g or x1 = At equilibrium, taking torque of liquids about O
k
m2 g æ Torque due to ö æ Torque due to ö
and, kx2 = m2 g or x2 = çè liquid of density r ÷ø = çè liquid of density1.5 r ÷ø
k
Amplitude of oscillation = x1 - x2 m2g × QM = m1g × PN
\ m2g R sin (45° + q ) = m1gR sin (45° – q )
( m1 + m2 ) g - m2 g
m1 g VrgR sin (45° + q ) = 1.5VrgR sin (45° – q )... (i)
or A = =
k k sin (45 + q)
Let at any instant the mass m2 be having a displacement x Þ = 1.5
sin (45 - q)
from the mean position. Restoring force will be
sin 45° cos q + cos 45° sin q 3
k Þ =
F = – k x or m2a = – k x Þ a = – x sin 45° cos q - cos 45° sin q 2
m2
1
Comparing this with a = – w2x , Þ tan q =
5
k k NOTE THIS STEP : Let us now displace the liquids in
we get w2 = Þ w= anticlockwise direction along the circumference of tube
m2 m2
through an angle a.
3. The mass will strike the right spring, compress it. The K.E. The net torque
of the mass will convert into P.E. of the spring. Again the
spring will return to its natural size thereby converting its t = m2gR sin (45° + q + a) – m1gR sin (45° – q – a)
P.E. to K.E. of the block. The time taken for this process will = VrgR sin (45° + q + a) – 1.5VrgR (45° – q – a)
= VrgR sin ( q + 45°) cos a + VrgR cos (45° + q )
T m sin a – 1.5 VrgR sin (45° – q) cos a
be , where T = 2p .
2 k + 1.5 VrgR cos(45° – q) sin a
60 cm Using eq. (i) we get
k1 k2
v t = V rgR éëcos ( 45° + q ) sin a + 1.5cos ( 45° - q ) sin aùû
A C m D B t = VrgR [cos (45° + q ) + 1.5 cos (45° – q )] sin a
when a is small (given)
T m 0.2 \ sin a » a
\ t1 = =p =p = 0.785 sec
2 k2 3.2 t = VrgR [cos (45° + q ) + 1.5 cos (45° – q )]a
The block will move from A to B without any acceleration. Since, t and a are proportional and directed towards mean
The time taken will be position.
\ The motion is simple harmonic.
60 Moment of inertia about O is
t2 = = 0.5
120 I = VrR2 + 1.5 VrR2
Now the block will compress the left spring and then the I
spring again attains its natural length. The time taken will be T = 2p
C
m 0.2
t3 = p =p = 1.05 sec. (V r ´ 2.5R 2 )
k1 1.8 = 2p
[cos(45 + q) + 1.5cos(45 - q)]V rgR
Again the block moves from B to A, completing one
oscillation. The time taken for doing so 1.803R 1
= 2p (using the value tan q = )
60 g 5
t4 = = 0.5
120 5. (i) As both the balls are displaced by an angle q = p / 6
\ The complete time of oscillation will be radian with respect to the diameter PQ of the circle and
= t1 + t2 + t3 + t4 released from rest. It results into compression of spring
= 0.785 + 0.5 + 1.05 + 0.5 in upper segment and an equal elongation of spring in
= 2.83 (app.) lower segment. Let it be x. PB and QA denote x in the
figure.
Simple Harmonic Motion (Oscillations) P-S- 123

L
B \ t = LSg (d2 – d1) × sin q
A 2
p = Rq when q is small, sin q » q
q q Q
P R L2 Sg
\ t= (d 2 - d1 )q. Since, t µ q
2
\ Motion is simple harmonic.

Compression = Rq = elongation = x On comparing it with t = C q


\ Force exerted by each spring on each ball = 2 kx L2 Sg
\ Total force on each ball due to two springs = 4 kx We get C= (d 2 - d1 )
2
\ Restoring torque about origin O = - ( 4kx ) R
L2 Sg
\ t = -4k ( Rq) R, where q = Angular displacement Þ Iw2 = (d 2 - d1 ) ... (i)
2
or t = -4 kR 2 q The moment of inertia I of the rod about P
Since torque (t) is proportional to q, each ball executes 2
angular SHM about the centre O. 1 æ Lö
= ML2 + M ç ÷
12 è 2ø
Again, t = -4 kR 2 q
1 1
or I a = -4 kR 2q where a = angular acceleration I= ML2 = LSd1L2 ... (ii)
3 3
or ( ) æ 4k ö
mR 2 a = -4kR 2q or a = - ç ÷ q
è mø From (i) and (ii)

1 a L3 L2 Sg
\ Frequency f = w2 ´ Sd1 = (d2 - d1 )
2p q 3 2
1 4k 3Sg (d 2 - d1 ) 3(d2 - d1 ) g
\ Frequency of each ball = Þ w= Þ w=
2p m 2 LSd1 2d1 L
1 4 ´ 0.1 1
= = sec -1 ..(ii) 7. If a small mass is attached to one end of a vertically hanging
2p 0.1 p spring then it performs SHM.
(ii) Let velocity at the mean position be vmax.
Angular frequency = w, Amplitude = a
Loss in elastic potential energy = Gain in kinetic energy
é1 æ pö ù
2
é1 2 ù Under SHM, velocity v = w a 2 - y 2
2 ê K ç 2 R ÷ ú = 2 ´ ê mvmax ú
êë 2 è ø
6 úû ë2 û After detaching from spring, net downward acceleration of
the block = g.
K Rp
\ vmax = ´ = 0.02 p m/s \ Height attained by the block = h
m 3
é1 2 ù
(iii) Total energy = 2 ê mvmax úû \ h= y+
v2
or h = y +
w 2
(a 2
- y2 )
ë2 2g 2g
é1 2ù dh
= 2 ê ´ 0.1(0.02p) ú For h to be maximum, = 0, y = y *.
ë2 û dy
= 3.95 × 10–4 J
w2 2
( -2 y *) or 0 = 1 - 2w y *
6. Let q be the angle made by the rod at any instant t. dh
\ = 1+
The volume of the rod = LS dy 2g 2g
Weight of the rod = LS d1g
Upthrust acting on rod = LS d2g w2 y * g
Since, d2 > d1 (given). Therefore LSd 2g or = 1 or y * =
g w2
net force acting at the centre of
mass of the rod at tilted position is Since aw 2 > g ( given )
(LS d2g – LSd1g) M
q
Taking moment of force about P q g
t = (LSd2g – LSd1g) × PN L/2
\ a> \ a > y *. y * from mean position < a.
LSd 1g w2
where PN = perpendicular
distance of line of action of force P N g
from P
d2 Hence y* = .
w2
EBD_7036
P-S- 124 Topic-wise Solved Papers - PHYSICS
F. Match the Following G. Comprehension Based Questions
1. A®p 1. (c) If the energy is zero, the particle will not perform
2 2
oscillations. Therefore E should be greater than zero.
Reason : For a simple harmonic motion v = a w - x . On Further if E = V0, the potential energy will become
constant as depicted in the graph given. In this case
comparing it with v = c1 c2 - x 2 we find the two also the particle will not oscillate.
comparable. Therefore E should be greater than zero but less
B ® q, r than V0.
Reason : v = – k x 2. (b) We can get the answer of this question with the help of
dimensional analysis.
when x is positive; v is – ve, and as x decreases, v decreases.
Therefore kinetic energy will decreases. When x = 0, v = 0. Given potential energy = ax4
Therefore the object does not change its direction.
Potential energy ML2T -2
When x is negative, v is positive. But as x decreases in a= = = [ ML-2T -2 ]
magnitude, v also decreases. Therefore kinetic energy x4 L4
decreases. When x = 0, v = 0. Therefore the object does not
1 m 1 M
change its direction. Now = =T
A a L ML-2T -2
C®p
Therefore option (b) is correct.
Reason : When a = 0, let the spring have an extension x.
Then k x = mg. -dV ( x)
3. (d) F=
When the elevator starts going upwards with a constant dx
acceleration, as seen by the observer in the elevator, the
As V (x) = constant for x > X0
object is at rest.
\ ma + mg = k x' \F=0 for x > X0
Þ ma = k (x' – x) (Since a is constant) Since F = 0, a = 0
D ® q, r 4. (d) When the ball is thrown upwards, at Momentum
the point of throw (O) the linear
The speed is 2 times the escape speed. Therefore the momentum is in upwards direction (and
object will leave the earth. It will therefore not change the has a maximum value ) and the position Position
direction and its kinetic energy will keep on decreasing. is zero. As the time passes, the ball 0 A
2. A® p moves upwards and its momentum
goes on decreasing and the position
When simple pendulum is displaced from the mean position becomes positive. The momentum
towards any of the extreme position, its potential energy becomes zero at the topmost point (A).
increases. In case of a S.H.M. we get a parabola for potential
As the time increases, the ball starts moving down
energy versus displacement
with an increasing linear momentum in the downward
B ® q, r, s direction (negative) and reaches back to its original
S = ut for zero acceleration. Therefore we get a straight line position.
passing through the origin. Therefore option (q) is correct. These characteristics are represented by graph (d).
1 5. (c) We know that the
S = ut + at2 for constant positive acceleration. In this
2 total mechanical energy µ (Amplitude)2
case we get a part of parabola as a graph line between s
versus t as shown by graph (s). \ E1 µ (2a)2
(p) is ruled out because if a is –ve and v is positive. & E2 µ a2
R E1
\ =4
E2
C®s
u 6. (b) As the mass is osicillating in water its amplitude will go
on decreasing and the amplitude will decrease with
u 2 sin 2q time. Options (c) and (d) cannot be true.
R= Þ R ¥ u2 (for consant q and g)
g When the position of the mass is at one extreme end in
T2 the positive side (the topmost point), the momentum is
l zero. As the mass moves towards the mean position
D ® q T = 2π \ T2¥ l the momentum increases in the negative direction.
g These characteristics are depicted in option (b).
l
Simple Harmonic Motion (Oscillations) P-S- 125

Section-B JEE Main/ AIEEE


1 2 2 1 2 l l
1. (c) K.E = k(A - x ) ; U = kx 9. (a) t = 2p ; t 0 = 2p
2 2 g eff g
At the mean position x = 0
Buoyant
1 2 force
\ K.E. = kA = Maximum and U = 0 1000 Vg
2
2. (b) Let the spring constant of the original spring be k.
4
m ´1000 Vg
3
Then its time period T = 2p where m is the mass of
k Weight
oscillating body. mgeff = mg – B = my – V × 100 × g
When the spring is cut into n equal parts, the spring
constant of one part becomes nk. Therefore the new \ geff = g - 100 g = g - 1000 g = g
( m / v) 4 4
time period, ´ 1000
3
m T l
T ' = 2p = \ t = 2p
nk n t = 2t0
g/4
l m
3. (b) KEY CONCEPT : The time period T = 2p where 10. (b) For first spring, t1 = 2p ,
g k1
l = distance between the point of suspension and the m
centre of mass of the child. This distance decreases For second spring, t 2 = 2 p
when the child stands k2
k1k 2
\ T ¢ < T i.e., the period decreases. when springs are in series then, keff =
kl + k 2
M
4. (c) T = 2p m ( kl + k 2 )
k \ T = 2p
k1k 2
M + m 5T
T ' = 2p = 2 2
k 3 m m t2 t
\ T = 2p + = 2p + 1
M +m 5 M m 16 k 2 k1 (2p)
2
(2p)
2
\ 2p = ´ 2p Þ =
k 3 k M 9 2 2 2
Þ T = t1 + t 2
k where x is the displacement from the mean position
5. (c) Maximum velocity during SHM = Aw = A
m 11. (a) At any instant the total energy is
é k ù 1 2
ê\ w = ú kA = constant, where A = amplitude
ë mû 2
hence total energy is independent of x.
Here the maximum velocity is same and m is also same 12. (b) Equation of displacement is given by
\ A1 k1 = A2 k 2
A1 k2 x = A sin(wt + f)
\ =
A2 k1 F0
where A = 2 2
l 1.21l m (w 0 - w )
6. (d) T = 2p and T ' = 2p
g g 13. (c) The maximum of amplitude and energy is obtained when
the frequency is equal to the natural frequency
(Q l ' = l + 21% of l) (resonance condition)
\ w1 = w2
T '- T 1.21l - l
% increase = ´ 100 = ´100 = 10% dy1 æ pö
T l 14. (b) v1 = = 0.1 ´ 100p cos ç100pt + ÷
dt è 3ø
æ sin pt cos pt ö
7. (c) x = 4(cos pt + sin pt ) = 2 ´ 4 ç + ÷ dy æ pö
è 2 2 ø v2 = 2 = - 0.1p sin pt = 0.1p cos ç pt + ÷
dt è 2ø
x = 4 2 sin(p t + 45°) p p 2 p - 3p p
\ Phase diff. = f1 - f 2 = - = =–
1 2 2 2 3 2 6 6
8. (a) K.E. = mw (a - x )
2 1 1
15. (c) y = - cos 2wt
1 2 2 2 2
When x = 0, K.E is maximum and is equal to mw a .
2
EBD_7036
P-S- 126 Topic-wise Solved Papers - PHYSICS
16. (b) Centre of mass of combination of liquid and hollow 22. (b) Here, x = 2 × 10–2 cos p t
portion (at position l ), first goes down ( to l + D l) and dx
when total water is drained out, centre of mass regain \ v= = 2 × 10–2 p sin p t
dt
its original position (to l ), For the first time, the speed to be maximum,
l p
T = 2p sin p t = 1 or, sin p t = sin
g 2
\ ‘T’ first increases and then decreases to original value. p 1
Þ pt = or,, t = = 0.5 sec.
2 2
23. (a) Here,
x = x0 cos (wt – p / 4 )
c
dx æ pö
\ Velocity, v = = - x0 w sin ç wt - ÷
dt è 4ø
2
d x 2 Acceleration,
17. (a) 2 = -ax = -w x
dt dv 2 æ pö
2p 2p a= = - x0 w cos ç wt - ÷
= dt è 4ø
Þ w = a or T =
w a
é æ pö ù
18. (a) Maximum velocity, = x0 w 2 cos ê p + ç wt - ÷ ú
2p ë è 4øû
vmax = a w , vmax = a ´
T æ 3p ö
-3 = x0 w 2 cos ç wt + ÷ ...(1)
2pa 2 ´ 3.14 ´ 7 ´ 10 è 4ø
ÞT = = » 0.01 s
vmax 4.4 Acceleration, a = A cos (wt + d) ...(2)
19. (a) K.E. of a body undergoing SHM is given by, Comparing the two equations, we get
1 1 3p
2 2 2
K .E. = ma w cos wt , T .E. = ma w
2 2 A = x0w2 and d = .
2 2 4
Given K.E. = 0.75 T.E. 24. (a) For an SHM, the acceleration a = -w2 x where w 2 is
2 p a
Þ 0.75 = cos wt Þ wt = a constant. Therefore is a constant. The time period
6 x
aT
p p´2 1 T is also constant. Therefore is a constant.
Þt= Þt= Þt= s x
6´w 6 ´ 2p 6 25. (d) For X0 + A to be the maximum seperation y one body is
20. (a) The two springs are in parallel. at the mean position, the other should be at the extreme.
1 K1 + K 2 26. (c) The net force becomes zero at the mean point.
f= ....(i) Therefore, linear momentum must be conserved.
2p m
\ Mv1 = (M + m)v2
1 4 K1 + 4 K 2
f '= k k æ k ö
2p m MA1 = ( M + m) A2 \ çV = A M ÷
M m+M è ø
1 4( K1 + 4K 2 ) æ 1 K1 + K 2 ö
= = 2ç A1 m+ M
2p m è 2p m ÷ø A1 M = A2 M + m \ A =
2 M
=2 f from eqn. (i) 27. (d) The equation of motion for the pendulum, suffering
21. (b) KEY CONCEPT : The instantaneous kinetic energy of retardation
a particle executing S.H.M. is given by
Ia = –mg(lsin q) – mbv (l) where I = ml2
1 2 2 2 and a = d2q/dt2
K= ma w sin wt
2
1 d 2q g bv
\ average K.E. = < K > = < mw2 a2 sin2 wt > \ =– tan q +
2 dt 2 l l
1 bt
= mw a <sin2 wt >
2 2 - sin(wt +f)
2 On solving we get q = q0 e 2

1 æ 1ö æ 2 1ö
= mw2a2 çè ÷ø çè Q < sin q > = ÷ø
-bt
2 2
q0
2 According to questions = q0 e 2
1 2 2 1 e
= mw a = ma2 (2pn)2 (Q w = 2pn )
4 4 2
2 2 2 \ t=
or, < K > = p ma n b
Simple Harmonic Motion (Oscillations) P-S- 127

-
bt
A - 3a 2 A2 + 2a 2 - 4 Aa - A2
2m Þ =
28. (c) Q A = A0 e (where, A0 = maximum amplitude) A A2
According to the questions, after 5 second, 2 2 2
Þ A – 3aA = A + 2a – 4Aa
-
b(5) Þ 2a2 = aA Þ A = 2a
0.9A 0 = A 0e 2m … (i)
a 1
Þ =
After 10 more second, A 2
b(15) Now, A – a = A coswt
-
A = A0 e 2m …(ii) A- a 1
Þ cos wt = Þ cos wt =
From eqns (i) and (ii) A 2
A = 0.729 A0 \ a = 0.729 2p p
or, t= Þ T= 6t
Mg T 3
29. (c) = P0 P0V0 g = PV g
A l
Mg = P0A … (1) P0 Ax0 g = PA( x0 - x ) g 31. (c) As we know, time period, T = 2p
g
g When additional mass M is added then
P0 x0
P=
( x0 - x )g l + Dl
TM = 2p
Let piston is displaced by distance x g
æ P xg ö l + Dl æ TM ö
2
l + Dl
Mg - ç
0 0
÷ A = Frestoring TM = or ç =
l ÷
çè ( x - x) g ÷ø
0 T
è T ø l
2
Piston æ TM ö Mg é Mgl ù
or, ç
è T ø÷ =1+ êQ Dl = Ay ú
Ay ë û
1 éæ TM ö
2 ù
x A
\ = êç ÷ - 1ú
è
y êë T ø úû Mg
x0 Cylinder
containing
1
ideal gas
32. (d) K.E = k ( A2 - d 2 )
2
æ x0g ö
P0 A ç1 - ÷ = Frestoring 1 2
çè ( x - x )g ÷ø [ x0 - x » x0 ] and P.E. = kd
0 2
gP0 Ax At mean position d = 0. At extremes positions d = A
F=- w
x0 33. (b) We know that V = A2 - x 2
\ Frequency with which piston executes SHM. 2
w A2 æ 2A ö
gP0 A 2 Initially V= -ç ÷
1 1 gP0 A è 3 ø
f = =
2px0 M 2p MV0 2
w A' 2 æ 2A ö
30. (d) In simple harmonic motion, starting from rest, Finally 3v = -ç ÷
At t = 0 , x = A è 3 ø
x = Acoswt ..... (i) 2A
Where A'= final amplitude (Given at x = , velocity
When t = t , x = A – a 3
When t = 2 t , x = A –3a to trebled)
From equation (i) 2
æ 2A ö
A – a = Acosw t ......(ii) A' 2 - ç ÷
3 è 3 ø
A – 3a = A cos2w t ......(iii) On dividing we get =
1 æ 2A ö
2
As cos2w t = 2 cos2 w t – 1…(iv) A2 - ç ÷
From equation (ii), (iii) and (iv) è 3 ø
A - 3a æ A-a ö
2 é 2 4A 2 ù 4A 2 7A
= 2ç ÷ -1 9 ê A - 9 ú = A'2 – \ A' =
A è A ø ëê ûú 9 3
EBD_7036
11
P-S- 128 Topic-wise Solved Papers - PHYSICS

Waves

Section-A : JEE Advanced/ IIT-JEE


A 1. A (2pv), A (2pv)2 2. 0.125 m 3. 1: 1 4. 240 Hz 5. 0.5 ms–1
6. f 7. 6 Hz
B 1. F 2. T 3. F
C 1. (c) 2. (c) 3. (a) 4. (c) 5. (a) 6. (a)
7. (a) 8. (d) 9. (d) 10. (b) 11. (b) 12. (c)
13. (b) 14. (a) 15. (b) 16. (b) 17. (b) 18. (c)
19. (d) 20. (a) 21. (a) 22. (a) 23. (a) 24. (b)
25. (a) 26. (b)
D 1. (a,b,c,d) 2. (b) 3. (a, c) 4. (a) 5. (a, c) 6. (c)
7. (a,b,d) 8. (b, c) 9. (b, c) 10. (b) 11. (a, b, c) 12. (b)
13. (b, c) 14. (a, c) 15. (b, c, d) 16. (a,b,c,d) 17. (a, b, c) 18. (a,c,d)
19. (a,d) 20. (b, d) 21. (b, c) 22. (a, b) 23. (d) 24. (a, c, d) 25. (a,b,c)
E 1. 1650 Hz 2. 70 m/s 3. 3.33 cm; 163 Hz 4. 8 5. 27.04 N
æ px ö æ px ö
6. 0.75 m/s 7. 11 Hz 8. (i) 3.46 cm (ii) 0, 15, 30 (iii) zero (iv) 2sin ç 96pt + ÷ and -2sin ç 96pt - ÷
è 15 ø è 15 ø
(2 n + 1) p (2 n + 1) p
9. 1.5 m/s 10. (i) z1 and z2; where n = 0, 1, 2, ... (ii) z1 and z3;
2K K
11. (i) 599 Hz (ii) 0.935 km, 621 Hz 12. 438.7 Hz, 257.3 Hz
2p b
13. (a) , (b) y = –0.8Acos(ax – bt) (c) 1.8 Ab, 0
a 2p
é ( -1)n ù p 2p
ê n + ú , –X direction 14. (i) p
(d) y = -1.6 A sin ax sin bt + 0.2 A cos(ax + bt ) sec . (ii) ´ 10 -3 sec .
êë 2 úû a 103 2

(v + vm ) ´ 2vb f
15. 403.3 Hz to 484 Hz 16.
v 2 - vb2

15 DP0
17. (a) m (b) (c) equal to mean pressure (d) P0 + DP0 , P0 - DP0 18. (a) 0.14s (b) 2.0 cm, 1.5 cm
16 2
-dH 400 3
19. = (1.11 ´ 10 -2 ) H , 43 sec. 20. (a) 1.007 × 105 Hz, (b) 1.03 × 105 Hz 21. (a) (b)
dt 189 4

p 2Ta 2 é 3 ù
22. 336 m/s 23. 24. 30 m/s 25. y = 0.1 sin ê30t ± x ± f ú
4l ë 2 û
F 1. A ® q; B ® p; C ® r 2. A ® p, t; B ® p, s; C ® q, s; D ® q, r
G 1. (a) 2. (c) 3. (d) 4. (b) 5. (a) 6. (a)
I. 1. 5 2. 7 3. 5 4. 3

Section-B : JEE Main/ AIEEE


1. (b) 2. (c) 3. (b) 4. (b) 5. (b) 6. (a)
7. (a) 8. (c) 9. (b) 10. (d) 11. (c) 12. (c)
13. (a) 14. (a) 15. (b) 16. (a) 17. (b) 18. (a)
19. (d) 20. (a) 21. (a) 22. (b) 23. (c) 24. (d)
25. (a) 26. (b)
Waves P-S- 129

Section-A JEE Advanced/ IIT-JEE


A. Fill in the Blanks (The driver behaves
as an observer)
1. Since y = A sin (wt – k x)
Displacement amplitude = A (Max displacement) vo =5ms-1 vs=5ms–1
dy
Particle velocity, v = = A w cos (wt – k x) (The image of the bus
dt formed by the
wall behaves as source)
\ Velocity amplitude = Aw =2pnA
Particle acceleration
The frequency of sound reflected from the wall
dv
Acc = = – A w2 sin (wt – kx) é v + v0 ù é 342 + 5 ù
dt n' = n ê ú Þ v' = 200 êë 342 - 5 úû » 206 Hz.
\ Acceleration (Max acc) amplitude = Aw2 = 4p2n2A ë v - vs û
c 330 \ Frequency of beats = n' – n = 6 Hz.
2. c = vl \ l= = = 0.5m
v 660 B. True/ False
The rarefaction will be at a distance of
1. The intensity of sound at a given point is the energy per
l 0.5
= = 0.125 m second received by a unit area perpendicular to the direction
4 4 of propagation.
3. y1 = 10 sin (3pt + p/4) ... (i) 1
I = r V w 2 A2
y2 = 5 sin 3pt + 5 3 cos 3pt ... (ii) 2
Also intensity varies as distance from the point source as
é1 3 ù
\ y2 = 5 × 2 ê sin 3pt + cos3pt ú = 10 sin (3pt +p/3) 1
ë2 2 û Iµ 2
The ratio of amplitudes is 10 : 10 = 1 : 1 r
NOTE : None of the parameters are changing in case of a
1 50.7 ´ 8 clear night or a clear day.
n1
4. = l
2 m
Þ n2 = n1
43.2
= 260
43.2
= 240 Hz.
Therefore the intensity will remain the same.
n2 1 43.2 ´ g 50.7 50.7 2. Speed of sound waves in water is greater than in air.
2l m 3. NOTE : If the sound reaches the observer after being
reflected from a stationary surface and the medium is also
1
5. As y = stationary, the image of the source will become the source
(1 + x )2 of reflected sound.
At t = 0 and x = 0, we get y = 1. Thus in both the cases, one sound coming directly from the
Also at t = 2 and x = 1, again y = 1 source and the other coming after reflection will have the
The wave pulse has travelled a distance of 1m in 2 sec. same apparent frequency (Since velocity of source w.r.t.
observer is same in both the cases). Therefore no beats will
1 be heard.
\ v= = 0.5 ms -1
2
C. MCQs with ONE Correct Answer
6. In figure (i)
l l v v
=l Þ l=2l l/2 1. (c) Case (i) Here =l \ f= = ... (i)
2 2 l 2l
c c l' l v v
and f = = l
Case (ii) Here = Þ l' = 2l \ f ' = = = f
l 2l 4 2 l ' 2l
In figure (ii) 2. (c) NOTE : Stationary wave is produced when two waves
l' l travel in opposite direction.
Þ = Þ l ' = 2l
4 2 Now, y = a cos (k x – wt) – a cos (k x + wt)
Fig (i) Fig (ii)
c c \ y = 2a sin kx sin wt is equation of stationary wave
and f ' = = =f
l ' 2l which gives a node at x = 0.
7. The first frequency that driver of bus hears is the original 3. (a) In air : T = mg = rVg
frequency of 200 Hz. The second frequency that driver hears 1 rVg
is the frequency of sound reflected from the wall. The two \ f= ... (i)
2l m
frequencies of sound heard by driver is In water : T = mg – upthrust
(a) Original frequency (200 Hz.) V Vg
(b) Frequency of sound reflected from the wall (n') = Vrg – rw g = (2r - rw )
2 2
EBD_7036
P-S- 130 Topic-wise Solved Papers - PHYSICS

Vg æ gRT ö 1
(2r - rw ) 10. (b) KEY CONCEPT : Crms = ç Here Crms µ ;
\ f '=
1 2
=
1 Vgr (2r - rw ) è M ÷ø m
2l m 2l m 2r
Crms1 æm ö
f' 2r - rw æ 2r - rw ö
1/ 2 \ = ç 2÷
= f '= f ç Crms2 è m1 ø
f 2r è 2r ø÷
11. (b)
1/ 2
é 2r - 1 ù
= 300 ê ú Hz
ë 2r û
4. (c) Comparing it with After two seconds pulses will overlap each other.
y (x, t) = A cos (wt + p/2) cos k x NOTE : According to superposition principle the string
If k x = p/2, a node occurs; will not have any distortion and will be straight.
\ 10p x = p/2 Þ x = 0.05 m Hence there will be no P.E. The total energy will be
If k x = p, an antinode occurs only kinetic.
Þ 10px = p Þ x = 0.1 m 12. (c) E µ A2n2 where A = amplitude and n = frequency.
Also speed of wave Also w = 2pn Þ w µ n
w 50 p In case 1 : Amplitude = A and n1 = n
= = = 5 m/s and l = 2p/k = 2p/10p = 0.2 m In case 2 : Amplitude = A and n2 = 2n
k 10p
T E2 A2 n22
5. (a) Velocity of sound by a stretched string v = \ = =4 Þ E2 = 4E1
m E1 A2 n12
v T T' 1.5x æ v + vö
= v' = v =v = 1.22 v 13. (b) Using the formula n ' = n ç A
v' T' T x è v ÷ø
6. (a) For both end open
v A + v 5.5 V +V 6 v
2l1 = and B = Þ B =2
= l Þ l1 = 2l v 5 V 5 vA
4
l
c c 5 9g 3 Mg
n1 = = ... (i) 14. (a) f0 = =
l1 2l 2l µ 2l µ
For one end closed Þ M = 25 kg
NOTE : Using the formula of a vibrating string,
3l 2 4l
For third harmonic = l Þ l2 =
4 3 p T
f = where p = number of loops.
c 3c 2l m
n2 = = ... (ii)
l 2 4l In each case, the wire vibrates, in resonance with the
Given n2 – n1 = 100 same tuning fork. Frequency of wire remains same while
From (i) and (ii) p and T change.
n2 3/ 4 3 p1 T1 p T2
= = \ = 2 or p1 T1 = p2 T2
n1 1/ 2 2 2l m 2l m
On solving, we get n1 = 200 Hz. T2 p
dy or = 1
7. (a) v = = - Aw cos (kx - wt ) \ vmax = Aw T1 p2
dt
M ´g 5
340 10 = or M = 5 ´ 5 ´ 9 or M = 25 kg.
8. (d) n1 = n0 = n0 ; 9´g 3 3´3
340 - 34 9
15. (b) f1= frequency of the police car heard by motorcyclist,
340 20 n1 10 19 19
f2 = frequency of the siren heard by motorcyclist.
n2 = n0 = n0 ; n = 9 ´ 20 = 18
340 - 17 19 2 330 - v 330 + v
f1 = ´ 176; f2 = ´ 165;
1 æ T ö 1 æ T ö 330 - 22 330
9. (d) n1 = and n = Q f1 – f2 = 0 Þ v = 22 m/s
ç ÷
2l è 4pr 2rø 2 4l çè pr 2rø÷
l
16. (b) l1 + x = or, l = 4 (l1 + x )
v 1 T l 4
n= = [where = length of string] 3l 4
l l m 2 (l 2 + x ) = or l = (l 2 + x)
4 3
n1 1 é mass r ´ A ´ length ù
\ = 2 ´ = 1 êQ m = = = rA ú
\ n1 =
v
=
v
\ n2 =
v
=
3v
n2 2 ë length length û l1 4(l1 + x ) l 2 4 (l 2 + x )
Waves P-S- 131

Given n1 = n2 Here n is a odd number. From (i) and (ii)


n 5
f2 = f1 , For first resonance, n = 5, f2 = f1
0.1m=l1 l1 +x 0.35m=l2 l2 +x 4 4
1 TAB
20. (a) Frequency of Ist harmonic of AB =
2l m
1 TCD
Frequency of 2nd harmonic of CD =
l m
Given that the two frequencies are equal.
1 TAB 1 TCD TAB
\ = Þ = TCD
2l m l m 4
v 3v Þ TAB = 4TCD ... (i)
or, = or, x = 0.025 m
4 (l1 + x ) 4 (l 2 + x) A B
17. (b) Frequency of first overtone in closed pipe,
3v P
n= ... (i) l AB l CD = l
4 l1 r1 =l
2
Frequency of first overtone in open pipe, TAB TCD

1 P O
n' = ... (ii) D
l2 r2 B x L–x
From equation (i) and (ii)
m
4 r L
Þ l 2 = l1 1 For rotational equilibrium of massless rod, taking torque
3 r2 about point O.
18. (c) TAB × x = TCD (L – x) ... (ii)
l1 +e For translational equilibrium,
l2 +e
TAB + TCD = mg ... (iii)
On solving, (i) and (iii), we get
mg 4mg
TCD = \ TAB =
5 5
Substituting these values in (ii), we get
4mg mg L
For first resonance For second resonance ´x= ( L - x) Þ x =
5 5 5
l 3l 21. (a) As shown in the figure, the fringes of the tuning fork
l1 + e = l2 + e = are kept in a vertical plane.
4 4
But v = nl
4 3v
\ v = n (l 2 + e ) Þ l2 + e = ...(i)
3 4v
v
\ v = n 4( l1 + e) Þ l1 + e = ... (ii)
4n
Subtracting (i) and (ii),
v = 2n (l 2 - l1 ) \ Dv = 2n ( Dl 2 + Dl1 ) 22. (a) Since the wave is sinusoidal moving in positive x-axis
= 2 × 512 × (0.1 + 0.1) cm/s = 204.8 cm/s the point will move parallel to y-axis therefore options
19. (d) (c) and (d) are ruled out. As the wave moves forward in
positive X-direction, the point should move upwards
nth harmonic i.e. in the positive Y-direction. Therefore correct option
is a.
l l 23. (a) The frequency (v) produced by the air column is
l=l given by
v v v
\ f1 = = ... (i) l× v=vÞ v=
l l l
v nv 3l
\ f2 = = ... (ii) = l = 75cm = 0.75 m
l 4l Also,
4
EBD_7036
P-S- 132 Topic-wise Solved Papers - PHYSICS
4 ´ 0.75 340 ´ 3 é dy ù
\l= Þ v= = 340 Hz or, ê dt ú = y0 ´ 2pf
3 4 ´ 0.75 ë û max
\ The frequency of vibrating string = 340. Since this Given that the maximum particle velocity is equal to four
string produces 4 beats/sec with a tuning fork of times the wave velocity (c = f × l)
frequency n therefore n = 340 + 4 or n = 340 – 4. With p y0
increase in tension, the frequency produced by string \ y0 × 2pf = 4(f × l) \ l =
2
increases. As the beats/sec decreases therefore n = 3. (a,c) The wavelengths possible in an air column in a pipe
340 + 4 = 344 Hz. which has one closed end is
24. (b) Frequency of 2nd harmonic of string = Fundamental
frequency produced in the pipe 4l
l= So, c = nl
(2n + 1)
4l
300 = 264 ×
0.5 m 2n + 1
n = 264 Hz as it is in resonance with a vibrating turning fork
of frequency 264 Hz.
330 ´ (2n + 1)
0.8 m
l=
é 1 Tù v 264 ´ 4
\ 2´ê ú= For n = 1, l = 0.3125 m = 31.25 cm
ë 2l1 m û 4l2 For n = 2, l = 0.9375 m = 93.75 cm
1 50 320
\ =
0.5 m 4 ´ 0.8
\ m = 0.02 kg m–1 4. (a)
The mass of the string = m l1
l l
= 0.02 × 0.5 kg = 10g
é v + vo ù l
= l (Fundamental mode) \ l = 4l
25. (a) f ¢ = f ê v - v ú 4
ë sû
l'
Here v = 320 m/s (given) = l (Fundamental mode) \ l ' = 2l
2
5
vo = vs = 36 ´ = 10 m/s \ n= =
c c
= 512 Hz (given)
18
l 4l
é 320 + 10 ù 33
f ¢ =8ê ú = 8 ´ » 8.5 kHz c c æ cö
ë 320 - 10 û 31 and n' = = = 2 ç ÷ = 2 × 512 = 1024 Hz.
l ' 2l è 4l ø
26. (b) Considering the end correction [e = 0.3 D], we get 5. (a,c) For wave motion, the differential equation is
l l
L+e= ÞL= –e ¶2 y æ w2 ö ¶ 2 y
4 4 = ç constant ÷
336 ´ 100 é vù ¶t 2 è k 2 ø ¶x 2
= - 0.3 ´ 4 = 15.2 cm êQ l = ú
512 ´ 4 ë uû ¶2 y ¶2 y
or = v2 ....(i)
D. MCQs with ONE or MORE THAN ONE Correct ¶t 2 ¶x 2
NOTE : The wave motion is characterized by the two
1. (a,b,c,d) y = 10–4 sin (60t + 2x) conditions
Comparing the given equation with the standard wave f ( x, t ) = f ( x , t + T ) ....(ii)
equation travelling in negative x-direction
y = a sin (wt + k x) f ( x, t ) = f ( x + l , t ) ....(iii)
we get amplitude a = 10–4m v
30 6. (c) n1 = for first harmonic
Also, w = 60 rad/s \ 2pf = 60 Þ f= Hz 4l1
p
2p l1
Also, k = 2 Þ = 2 Þ l= pm
l 3v
n2 = for third harmonic
30 2l 2
We know that v = f l = ´ p = 30 m/s
p l 2

é xù v 3v l1 1
2. (b) y = y0 sin 2p ê f t - ú Q n1 = n2 = \ Þ =
ë lû 4l1 2l 2 l2 6
dy é æ x öù v 3v 5v
\ = ê y0 cos 2p ç f t - ÷ ú ´ 2pf 7. (a,b,d) \ n = , , ,.... = 80, 240, 400....
dt ë è l øû 4l 4l 4l
Waves P-S- 133

8. (b,c) y = A sin (10 px + 15 pt + p/3) u ( x, y ) =0 at x = L, y = L


The standard equation of a wave travelling in – X direction
u ( x, y ) ¹ 0 at x =0, y =0
é 2p ù
is y = A sin ê (vt + x) + (f)ú Option (b) :
ël û
é 2p v 2p ù u ( x, y ) = 0 at x = 0, y = 0 [Q sin 0 = 0]
Þ y = A sin ê t+ x + fú
ë l l û u ( x, y ) = 0 at x = L, y = L [Q sin p = 0]
Comparing it with the given equation, we find Option (c) :
2p v 2p u ( x, y ) = 0 at x = 0, y = 0 [Q sin 0 = 0]
= 15p and = 10p
l l u ( x, y ) = 0 at x = L, y = L [Q sin p = 0, sin 2p = 0]
1 15p 1
Þ l = = 0.2 m and v = ´ = 1.5 m/s Option (d) :
5 2p 5
u ( x, y ) = 0 at y = 0, y = L [Q sin 0 = 0, sin p = 0]
1 T
9. (b,c) As, f = \ fµ T u ( x, y ) ¹ 0 at x = 0, x = L [Q cos0 = 1, cos 2p = 1]
2p m
14. (a,c)
Given that T1 > T2 \ f1 > f 2 NOTE : For a transverse sinusodial wave travelling on a
string, the maximum velocity is aw.
Initially beat frequency ( f1 - f2 ) = 6.
v 10
The beat frequency remains unchanged which is possible But maximum velocity is = = 1m/s
10 10
when f2 increases and f1 decreases. Thus T2 increases and \ aw = 1 Þ 10–3 × 2pn = 1
T1 decreases.
1 103
ætö æ t ö Þ n= = Hz
10. (b) y = 4 cos2 çè ÷ø sin (1000 t) = 2 ç 2cos2 sin1000t ÷ 2p ´ 10-3 2p
2 è 2 ø
v 10
= 2 [cos t + 1] sin 1000t
\ l= = 3 = 2p ´ 10-2 m
= 2 cos t sin 1000t + 2 sin 1000t n 10 / 2p
= sin 1000t + sin 999t + 2 sin 1000t 0.8 0.8
15. (b,c,d) y = = ... (1)
æ c + vö (4x + 5t) + 5 2
é 5 ù
2
11. (a, b, c) Frequency of reflected wave is f '' = f ç
è c - v ÷ø 16 ê x + t ú + 5
4 û
ë
2v We know that equation of moving pulse is
Þ Beat freq. = f '' – f = .
c -1 y = f (x + vt) ... (2)
c c (c - v) On comparing (1) and (2), we get
Wavelength of reflected wave = =
f '' f (c + v) 5 -1 2.5 -1
12. (b) KEY CONCEPT : The time required for constructive v= ms = ms
4 2
interference equal to the time period of a wave pulse. So, the wave will travel a distance of 2.5 m in 2 sec.
y
For a string frequency f = 1 F 0.8
2l m Q y=
( 4 x + 5t )2 + 5
\ Time period, T = 2l m At
F 0.8
-2 x = 0, t = 0, y = = 0.16 m
mass 10 5 –x O x
F = 1.6 N, m = = = 2.5 ´ 10-2
length 0.4 \ maximum displacement is 0.16 m
The graph for the given equation is drawn. This is symmetric
2.5 ´ 10-2 about y-axis.
\ T = 2 ´ 0.4 = 0.1 sec.
1.6 16. (a,b,c,d) In the wave motion y = a ( kx - wt ) , y can represent
13. (b,c) Due to the clamping of electric and magnetic fields in electromagnetic waves and
the square plate at the edges, displacement and pressure in sound waves.
its displacements along the x y
17. (a,b,c) Standing waves are produced by two similar waves
and y axes will individually be superposing while travelling in opposite direction.
zero at the edges. Only (0, L) R Q (L, L) This can happen in case (a), (b) and (c).
th e choices (b) and (c) 18. (a,c,d) For a plane wave, intensity (energy crossing per unit
predict these displacements area per unit time) is constant at all points.
But for a spherical wave, intensity at a distance r from a
correctly. This is because
P P
sin 0 = 0. x point source of power (P), is given by I =
Option (a) : (0, 0) (L, 0) 4p r 2
EBD_7036
P-S- 134 Topic-wise Solved Papers - PHYSICS
For monatomic gas g =1.67
1
Þ Iµ 2 gRT 10
r v= = 100 gRT ´
-3 M
But the total intensity of the spherical wave over the M ´ 10
spherical surface centered at the source remains constant 10 10
at all times. = 167RT ´ = 640
M M
1 e
NOTE : For line source I µ 10 7
r For Neon M = 20 and =
19. (a, d) 20 10
At second resonance the length of air column 7
\ v = 640 ´ = 448 ms–1
is more as compared to first resonance. Now, 10
longer the length of air column, more is the \ (a) is incorrect
absorption of energy and lesser is the
10 17
intensity of sound heard. For Argon M = 36, =
36 32
As shown in the figure, the length of the air column at the 17
\ v = 640 ´ = 340 ms–1
1 32
first resonance is somewhat shorter than th of the
4 \ (d) is the correct option.
wavelength of the sound in air due to end correction. For diatomic gas g = 1.4
l l
l+e = \ l = -e 10 10
4 4 v = 140RT = 590 ´
20. (b, d) M M
When sound pulse is reflected through a rigid boundary 10 9
(closed end of a pipe), no phase change occurs between the For Oxygen =
incident and reflected pulse i.e., a high pressure pulse is 32 16
reflected as a high pressure pulse. 9
When a sound pulse is reflected from open end of a pipe, a \ v = 590 ´ = 331.87 ms–1
16
phase change of a radian occurs between the incident and
\ (c) is incorrect
the reflected pulse. A high pressure pulse is reflected as a
low pressure pulse. 10 3
21. (b, c) For Nitrogen =
28 5
y = [0.01 sin (62.8x)] cos (628 t).
3
M \ v = 590 ´ = 354 ms–1
5
l 1 \ (b) is incorrect
Length of string = 5 ´ = 5´ = 0.25 m
24. (a, c, d)
2 20
Clearly in the given situation a displacement node is present
é 2p ù
êëQ l = 62.8úû at x = 0 and a displacement antinode is present at x = 3 m.
The midpoint M is an antinode and has the maximum Therefore, y = 0 at x = 0 and y = ± A at x = 3 m.
displacement = 0.01 m w
v ω/k The velocity v = = 100 ms–1.
The fundamental frequency = = = 20Hz k
2l 2l a, c and d are the correct options which satisfy the above
22. (a,b) Source Observer
conditions.
S O
u u 25. (a,b, c)
f1 f2
Wind blowsfromobserver tosource P
Case1: \ f2 > f1 vccos q q vc
é (V - w) + u ù A
f2 = f1 ê ú a Vx
ë (V - w) - u û 118Hz c
121Hz
Wind blowsfromsource toobserver 10 Q
Case2 : M N
é (V + w) + u ù \f2 > f1
f2 = f1 ê ú
ë (V + w) - u û R
(a) and (b) are correct options é v + v cos q ù
n P = 121 - 118 = ê úû
v gRT 1 ë v
23. (d) Here, n = = ´ Þ v = n × 4l
4l -3 4l nq = 121 - 118 = 3
M ´ 10
Þ v = 336.7 m/s to 346.5 m/s
Waves P-S- 135

é v - vc cos q ù Speed of transverse wave is given by


n R = (121 –118) ê úû
ë v F é Alr ù
v= ê where m = mass per unit length = l = Arú
\ n P + n R = 2 n Q Þ ( A ) is correct option m ë û
In general when the car is passing through A YAaDq Y aDq
= =
é v + vc cos a ù Ar r
n = 3ê úû …(i)
ë v 1.3 ´ 1011 ´ 1.7 ´ 10 -5 ´ 20
= = 70 m/s
dn é v sin a ù d n 9 ´ 103
\ = -3 ê c is max when sin a = 1
da ë v úû d a 3. Tube open at both ends :
i.e., a =90° (at Q) v 320
Þ (b) is correct option. (a) v = \ 320 =
2(l + 0.6 D) 2(0.48 + 0.6 ´ D)
d n 3vc da
From (i) = (– sin a ) … (ii) 0.48 + 0.6 D = 0.5 Þ 0.6 D = 0.02
dt v dt
0.02
10 2 da 10 dx Þ D= ´ 100 cm = 3.33 cm
Also tan a = \ sec a =- 2 60
x dt x dt Tube closed at one end :
da -10v v 320
\ = 2 …(iii) v= =
dt x sec 2 a 4(l + 0.3D) 4(0.48 + 0.3 ´ 0.033)
From (ii) & (iii) » 163 Hz
dn 3v æ -10v ö 30Vc sin a 4.
= - c sin a ´ ç 2 ÷=
dt v è x sec 2 a ø x 2 sec 2 a v0 = 5 m/s v0 = 5 m/s
dn 30vc sin a
\ dt = = 0.3vc sin 3 a . At a = 90° S I
(10 cot a ) 2 sec 2 a (Observer) (Source)
dn
= max NOTE : If the sound reaches the observer after being
dt reflected from a stationary surface and the medium is also
\ (c) is the correct option stationary, the image of the source in the reflecting surface
E. Subjective Problems will become the source of the reflected sound.
é c - v0 ù
1. It is given that C acts as a node. This implies that at A and B n' = n ê ú
antinodes are formed. Again it is given that the frequencies ë c - vs û
are same. v0, vs are + ve if they are directed from source to the observer
and – ve if they are directed from observer to source.
p1 harmonic p2 harmonic
é 330 - (-5) ù
n' = 256 ê = 264 Hz
H2 O2 ë 330 - 5 úû
Antinode Node Antinode \Beat frequency = 264 – 256 = 8
A C B
0.5m 0.5m 40cm=l
v1 v p v 3 l/4
Þ ´ p1 = 2 ´ p2 or 1 = 1 =
4l 4l p2 v2 11 25cm
5. l
or, 11p1 = 3p2
This means that the third harmonic in AC is equal to 11th
harmonic in CB.
Now, the fundamental frequency in AC First Overtone
v1 1100
= = = 550 Hz 2.5 ´ 10-3
4l 4 ´ 0.5 Mass of string per unit length = = 0.01 kg/m
and the fundamental frequency in CB 0.25
v2 300 1 T 1 T
= = = 550 Hz \ Frequency, ns = = ... (i)
4l 4 ´ 0.5 l m 0.25 0.01
\ Frequency in AC = 3 × 550 = 1650 Hz Fundamental frequency
and frequency in CB = 11 × 150 = 1650 Hz l
2. (a) Using the formula of the coefficient of linear expansion \ = 0.4 Þ l = 1.6 m
4
of wire, Dl = laDq we get c 320
F = YAaDq \ nT = = = 200 Hz ... (ii)
lT 1.6
EBD_7036
P-S- 136 Topic-wise Solved Papers - PHYSICS
Given that 8 beats/second are heard. The beat frequency æ pö
decreases with the decreasing tension. This means that beat = 4sin ç ÷ [- 96 p sin (24p)] = 0
è 2ø
frequency decreases with decreasing ns. So beat frequency
is given by the expression. æ p xö
(iv) y = 4sin ç ÷ cos[96 p t ]
n = ns – nT è 15 ø
1 T é æ p xö ù
\ 8= - 200 Þ T = 27.04 N = 2 ê 2 sin ç
è ÷ø cos (96 p t ) ú
0.25 0.01 ë 15 û
6. KEY CONCEPT : The velocity of wave on the string is é æ p xö æ p xöù
= 2 êsin ç 96 p t + ÷ - sin ç 96 p t - ÷
given by the formula ë è 15 ø è 15 ø úû
T æ p xö æ p xö
v= = 2sin ç 96 p t + ÷ - 2sin ç 96 p t - ÷
m è 15 ø è 15 ø
where T is the tension and m is the mass per unit length. = y1 + y2
p xö
Since the tension in the string will increase as we move up where y1 = 2 sin æç 96 p t + ÷
the string (as the string has mass), therefore the velocity of è 15 ø
wave will also increase. (m is the same as the rope is uniform) æ p xö
and y2 = – 2 sin ç 96 p t - ÷
v1 T 2 ´ 9.8 1 è 15 ø
\ = 1 = = \ v2 = 2v1 9. The apparent frequency from tuning fork T1 as heard by the
v2 T2 8 ´ 9.8 2
observer will be
Since frequency remains the same v v
\ l2 = 2l1 = 2 × 0.06 = 0.12 m
7. KEY CONCEPT : Using the formula of the coefficient of
linear expansion,
Dl = la ´ Dq T2 T1
Observer
stress T A T A c
Also, Y= = = \ T = YA
A a Dq n1 = ´n ... (i)
strain Dl l aAq c-v
where c = velocity of sound
The frequency of the fundamental mode of vibration. v = velocity of turning fork
1 T 1 YA a Dq The apparent frequency from tuning fork T2 as heard by the
n = = observer will be
2l m 2l m
c
n2 = ´n ... (ii)
1 2 ´ 1011 ´ 10 -6 ´ 1.21 ´ 10 -5 ´ 20 c+v
= = 11 Hz Given n1 – n2 = 3
2 ´1 0.1
é 1 1 ù c ´ n ´ 2v
\ c×n ê - ú = 3 or, 3 = 2
æ p xö ëc - n c + nû c - v2
8. (i) Here amplitude, A = 4 sin ç ÷
è 15 ø c ´ n ´ 2v
At x = 5 m Since, v < < c \ 3=
c2
æ p ´ 5ö 3 ´ 340 ´ 340
A = 4sin ç = 4 × 0.866 = 3.46 cm \ v= = 1.5 m / s
è 15 ÷ø 340 ´ 340 ´ 2
10. (i) KEY CONCEPT : When two progressive waves having
(ii) Nodes are the position where A = 0
same amplitude and period, but travelling in opposite
æ px ö direction with same velocity superimpose, we get standing
\ sin ç ÷ = 0 = sin np \ x = 15 n
è 15 ø waves.
where n = 0, 1, 2 x = 15 cm, 30 cm, 60 cm, .... The following two equations qualify the above criteria and
hence produce standing wave
æ p xö z1 = A cos (k x – wt)
(iii) y = 4sin ç ÷ cos (96p t )
è 15 ø z2 = A cos (k x + wt)
The resultant wave is given by z = z1 + z2
dy æ p xö Þ z = A cos (k x – wt) + A cos (k x + wt)
v= = 4sin ç [– 96p sin (96 pt)]
dt è 15 ÷ø = 2A cos k x cos wt
At x = 7.5 cm, t = 0.25 cm The resultant intensity will be zero when
2A cos k x = 0
æ p ´ 7.5 ö
v = 4sin ç [– 96p sin (96 p × 0.25)] (2n + 1)
è 15 ÷ø Þ cos k x = cos p
2
Waves P-S- 137

2n + 1 (2 n + 1) p A1 w=10rad/s
Þ kx= p Þ x=
2 2k
where n = 0, 1, 2, ...
(ii) The transverse waves A2 6m 6m
z1 = A cos (k x – wt) A B C D
z3 = A cos (k y – wt)
Combine to produce a wave travelling in the direction making
an angle of 45° with the positive x and positive y axes. A3
The resultant wave is given by z = z1 + z3 Þ When the detector is at C moving towards D, the source
z = A cos (k x – wt) + A cos (ky – wt) is at A1 moving leftwards. It is in this situation that the
( x - y) é k ( x + y ) - 2wt ù frequency heard is minimum
Þ z = 2A cos cos ê úû
2 ë 2 é v - v0 ù (340 - 60)
The resultant intensity will be zero when n' = n ê ú = 340 ´ = 257.3Hz
ë v + vs û (340 + 30)
k ( x - y) k ( x - y) Again when the detector is at C moving towards B, the
2 A cos = 0 Þ cos =0
2 2 source is at A3 moving rightward. It is in this situation that
k ( x - y) 2n + 1 (2 n + 1) the frequency heard is maximum.
Þ = p Þ (x – y) = p
2 2 k é v + v0 ù (340 + 60)
11. (i) The frequency of the whistle as heard by observer on the n'' = n ê ú = 340 ´ = 438.7 Hz
hill ë v - vs û (340 - 30)
x 13. (a) KEY CONCEPT : Use the equation of a plane
é v + vm ù progressive wave which is as follows.
n' = n ê ú A B
ë v + vm - vs û y = A cos ç
æ 2p ö
x + 2pn t÷
1km
Hill è l ø
é 1200 + 40 ù The given equation is
= 580 ê = 599 Hz
ë1200 + 40 - 40 úû y1 = A cos (ax + bt)
(ii) Let echo from the hill is heard by the driver at B which 2p 2p
is at a distance x from the hill. On comparing, we get = a Þ l=
l a
The time taken by the driver to reach from A to B Also, 2pn = b
1- x b
t1 = ... (i) Þ n=
40 2p
The time taken by the echo to reach from hill (b) Since the wave is reflected by an obstacle, it will suffer
t2 = tAH + tHB a phase difference of p. The intensity of the reflected wave
1 x is 0.64 times of the incident wave.
t2 = + ... (ii) Intensity of original wave I µ A2
(1200 + 40) (1200 - 40)
Intensity of reflected wave I ' = 0.64 I
where tAH = time taken by sound from A to H with velocity Þ I ' µ A'2 Þ 0.64 I µ A'2
(1200 + 40) Þ 0.64 A2 µ A'2 Þ A' µ 0.8A
tHB = time taken by sound from H to B with velocity 1200 – 40 So the equation of resultant wave becomes
From (i) and (ii) y2 = 0.8A cos (ax – bt + p) = – 0.8 A cos (ax – bt)
1- x 1 x (c) KEY CONCEPT : The resultant wave equation can be
t1 = t2 Þ = +
40 1200 + 40 1200 - 40 found by superposition principle
Þ x = 0.935 km y = y1 + y2
The frequency of echo as heard by the driver can be = A cos (ax + bt) + [– 0.8 A cos (ax – bt)]
calculated by considering that the source is the acoustic The particle velocity can be found by differentiating the
image. above equation
é ( v - vm ) + v s ù dy
n'' = n ê ú v= = – Ab sin (ax + bt) – 0.8 Ab sin (ax – bt)
ë ( v - vm ) - v0 û dt
= – Ab [sin (ax + bt) + 0.8 sin (ax – bt)]
é (1200 - 40) + 40 ù = – Ab [sin axcos bt + cos ax sin bt
= 580 ê ú = 621 Hz
ë (1200 - 40) - 40 û + 0.8 sin ax cos bt – 0.8 cos ax sin bt]
12. The angular frequency of the detector = 2pn v = – Ab [1.8 sin ax cos bt + 0.2 cos ax sin bt]
5 The maximum velocity will occur when sin ax = 1 and
= 2p ´ = 10 rad / s cos bt = 1 under these condition cos ax = 0 and sin bt = 0
p
The angular frequency of the detector matches with that of \ | vmax | = 1.8 Ab
the source. Also, | vmin | = 0
EBD_7036
P-S- 138 Topic-wise Solved Papers - PHYSICS
(d) y = [A cos (ax + bt)] – [0.8 A cos (ax – bt)] 1 é ( w - w 2 )t ù 1
or, cos p ( n1 - n2 ) t ³ cos ê 1
or,
= [0.8 A cos (ax + bt) + 0.2 A cos (ax + bt)] 2 2 ú³
ë û 2
– [0.8 A cos (ax – bt)]
= [0.8 A cos (ax + bt) – 0.8 A cos (ax – bt)] é (w - w 2 )t ù
The detector lies idle when the values of cos ê 1 ú
+ 0.2 A cos (ax + bt)] ë 2 û
é ì ( ax + bt ) + (ax - bt ) ü 1
= 0.8 A ê -2 sin í ý is between 0 and
ë î 2 þ 2
ì ( ax + bt ) - (ax - bt ) ü ù (w1 - w 2 )t p p
sin í ý ú 0.2 A cos (ax + bt)] \ is between and
î 2 þû 2 2 4
Þ y = – 1.6 A sin ax sin bt + 0.2 A cos (ax + bt) p p
where (– 1.6 A sin ax sin bt) is the equation of a standing \ t1 = and t2 =
w1 - w 2 2(w1 - w 2 )
wave and 0.2 A cos (ax + bt) is the equation of travelling
\ The time gap = t1 – t2
wave.
The wave is travelling in –x direction. p p p
= – =
NOTE : Antinodes of the standing waves are the positions w1 - w 2 2(w1 - w 2 ) 2(w1 - w 2 )
where the amplitude is maximum,
p -3
é pù = ´ 10 sec
i.e., sin ax = 1 = sin ê np + (-1)n ú 2
ë 2û 15. The whistle which is emitting sound is being rotated in a
é ( -1) ù p
n circle.
Þ x = ên + ú vs=rw =30m/s
ëê 2 ûú a
A
14. Let the two radio waves be represented by the equations
y1 = A sin 2pn1t r= 1.5m
y2 = A sin 2pn2t
The equation of resultant wave according to superposition O
principle w=20rad-1 vo=0
y = y1 + y2 = A sin 2pn1t + A sin 2pn2t
= A [sin 2 pn1t + sin 2pn2t] B v =rw
s
(2pn1 + 2pn 2 )t (2pn1 + 2pn 2 )t =30m/s
= A × 2 sin cos
2 2 r = 1.5 m (given); w = 20 rads–1 (given)
= 2A sin p (n1 + n2) t cos p (n1 – n2) t We know that
where the amplitude A' = 2A cos p (n1 – n2) t v = rw = 1.5 × 20 = 30 ms–1
When the source is instantaneously at the position A, then
Now, intensity µ (Amplitude)2
the frequency heard by the observer will be
Þ I µ A'2
é v ù é 330 ù
I µ 4A2 cos2p (n1 – n2) t n' = n ê ú = 440 êë 330 - 30 úû = 484 Hz
The intensity will be maximum when ë v - vs û
cos2p (n1 – n2) t = 1 When the source is instantaneously at the position B, then
the frequency heard by the observer will be
or, cos p (v1 – v2) t = 1
or, p (n1 – n2) t = np é v ù é 330 ù
n'' = n ê ú = 440 ê 330 + 30 ú = 403.3Hz
ë v + vs û ë û
(w1 - w 2 ) 2np
Þ t = n p or,, t = Hence the range of frequencies heard by the observer is
2 w1 - w 2
403.3 Hz to 484 Hz.
\ Time interval between two maxima 16. KEY CONCEPT : Motorist will listen two sound waves.
2 np 2(n - 1)p 2p 2p One directly from the sound source and other reflected from
or, - or, = sec
w1 - w 2 w1 - w 2 w1 - w 2 103 the fixed wall. Let the apparent frequencies of these two
Time interval between two successive maximas is waves as received by motorist are f 'and f '' respectively.
For Direct Sound : Vm will be positive as it moves towards
2p × 10–3 sec
the source and tries to increase the apparent frequency. Vb
(ii) For the detector to sense the radio waves, the resultant will be taken positive as it move away from the observer and
intensity > 2A2 hence tries to decrease the apparent frequency value.
\ Resultant amplitude > 2A v + vm
f'= f ... (1)
v + vb
or, 2 A cos p ( n1 - n2 ) t ³ 2 A
Waves P-S- 139

18. (a) Mass per unit length of PQ


f1 0.06
m1 = kg / m
Vm Reflected 4.8
wave Fixed
Vb Wall 0.06 kg 0.2 kg
P 4.8 m Q 2.56 m R
O S 0.2
Mass per unit length of QR, m2 = kg / m
For reflected sound : 2.56
Velocity of wave in PQ is
For sound waves moving towards stationary observer (i.e.
wall), frequency of sound as heard by wall T 80
v1 = = = 80 ms -1 [Q T = 80 N given]
v m1 0.06 / 4.8
f1 = f
v - vb Velocity of wave in QR is
After reflection of sound waves having frequency f1 fixed T 80
wall acts as a stationary source of frequency f1 for the moving v2 = = = 32 m/s
m2 0.2 / 2.56
observer i.e. motorist. As direction of motion of motorist is
opposite to direction of sound waves, hence frequency f '' \ Time taken for the wave to reach from P to R
of reflected sound waves as received by the motorist is = tPQ + tQR
v + vm v + vm 4.8 2.56
f '' = f1 = f ... (2) = + = 0.14s
v v - vb 80 32
(b) When the wave which initiates from P reaches Q
Hence, beat frequency as heard by the motorist
(a denser medium) then it is partly reflected and partly
æ v + vm v + vm ö transmitted.
Df = f '' – f ' = ç - f
è v - vb v + vb ÷ø In this case the amplitude of reflected wave
2vb ( v + vm ) f æ v2 - v1 ö
Df = Ar = ç Ai ... (i)
or,
v 2 - vb 2 è v2 + v1 ÷ø
17. (a) For second overtone as shown, where Ai = amplitude of incident wave.
Also amplitude of transmitted wave is
5l 4l
= l \ l= æ 2v2 ö
4 5 At = ç Ai ... (ii)
Also, v = nl è v1 + v2 ÷ø
4l 15 From (i), (ii)
Þ 330 = 440 × Þ l = m.
5 16 Therefore, At = 2 cm and Ar = – 1.5 cm.
19. Speed of sound, v = 340 m/s.
l
(Pressure (Pressure antinode) Let l 0 be the length of air column corresponding to the
node) or fundamental frequency. Then
displacement displacement node
antinode v
x=L MID POINT x=O = 212.5
Pressure variation x=x Pressure variation 4l 0
is zero is max or min
v 340
(b) KEY CONCEPT : At any position x, the pressure is or l0 = = = 0.4 m.
given by 4(212.5) 4(212.5)
DP = DP0 cos kx cos wt NOTE : In closed pipe only odd harmonics are obtained.
2p Now, let l1, l 2 , l 3 , l 4 , etc. be the lengths corresponding
Here amplitude A = DP0 cos kx = DP0 cos x
l to the 3rd harmonic, 5th harmonic, 7th harmonic etc. Then
15 15 æ v ö
For x = = m (mid point) 3ç = 212.5 Þ l1 = 1.2 m;
2 ´ 16 32 è 4l1 ÷ø
é 2p 15 ù DP
Amplitude = DP0 cos ê ´ ú= 0 æ v ö
ë (330 / 440) 32 û 5ç = 212.5 Þ l 2 = 2.0 m
è 4l 2 ÷ø
2
(c) At open end of pipe, pressure is always same i.e. equal
æ v ö
to mean pressure Q DP = 0, Pmax = Pmin = P0 7ç = 212.5 Þ l 3 = 2.8 m;
è 4l 3 ÷ø
(d) At the closed end :
Maximum Pressure = P0 + DP0 æ v ö
9ç = 212.5 Þ l 4 = 3.6 m
Minimum Pressure = P0 – DP0 è 4l 4 ÷ø
EBD_7036
P-S- 140 Topic-wise Solved Papers - PHYSICS
0.4m

B 2.088 ´ 109

1.2m
Velocity of sound in water c = =
103
r

2.0m

2.8m
c = 1445 m/s; vm = + 2 m/s; v0 = 0; vs = 10 m/s
3.2m

é 1445 + 2 - 0 ù
3.4m
\ n' = n ê = n[1.007]
ë1445 + 2 - 10 úû

1.6m

0.8m
or heights of water level are (3.6 – 0.4) m, (3.6 – 1.2) m, c 1445
Now, n = = = 105 Hz
(3.6 – 2.0) m and (3.6 – 2.8) m. l 14.45 ´ 10-3
Therefore heights of water level are 3.2 m, 2.4 m, 1.6 m and \ n' = 1.007 × 105 Hz
0.8 m. (b) Situation 2.
Let A and a be the area of cross-sections of the pipe and
g RT
hole respectively. Then In air c = = 344 m/s
A = p (2 × 10–2)2 = 1.26 × 10–3 m–2 M
vm = 5ms-1
and a = p (10–3)2 = 3.14 × 10–6 m2
S
Velocity of efflux, v = 2 gH O
Continuity equation at 1 and 2 gives, 10m/s (Receiver)
æ - dH ö
a 2 gH = A ç
è dt ÷ø Applying formula (1)
Therefore, rate of fall of water level in the pipe, é 344 - 5 - 0 ù
n' = n ê ú = 1.03 × 105 Hz
æ -dH ö a ë 344 - 5 - 10 û
çè ÷= 2 gH
dt ø A 21. (a) Second harmonic in pipe A is
Substituting the values, we get
é v ù 1 g A RT
-6 2 ( n0 ) A = 2 ê ú =
-dH 3.14 ´ 10
= 2 ´ 10 ´ H ë 2l û l M A
dt 1.26 ´ 10 -3 Third harmonic in pipe B is
- dH évù 3 g B RT
Þ = (1.11 × 10–2) H 3 ( n0 ) B = 3 ê ú =
dt ë 4l û 4l MB
Between first two resonances, the water level falls from
3.2 m to 2.4 m. Given n A = nB
dH 1 g A RT 3 g B RT
\ = -1.11 ´ 10-2 dt =
H l MA 4l MB
2.4 dH t 2
Þ ò = -(1.11 ´ 10-2 )ò dt M A g A æ 4ö 5 / 3 16 400
3.2 H 0 or, = ´ç ÷ = ´ =
M B g B è 3ø 7/5 9 189
Þ 2[ 2.4 - 3.2] = -(1.1 ´ 10 -2 ).t A B
Þ t » 43second Gas (Monoatomic) Gas (Diatomic)
MA MB
20. KEY CONCEPT : The question is based on Doppler's effect
where the medium through which the sound is travelling is l l
also in motion.
By Doppler's formula (v0 ) A g A MB 3
Now, = ´ =
(v0 ) B g A MB 4
é c + v m ± v0 ù
n' = n ê ú ... (1) 22. KEY CONCEPT : In the fundamental mode
ë c + vm ± vs û
NOTE : Sign convention for Vm is as follows : l v
(l + 0.6 r ) = = Þ v = 4f (l + 0.6 r ) = 336 m/s.
If medium is moving from s to O then + ve and vice versa. 4 4f
Similarly v0 and vs are positive if these are directed from S to 2p 2p p
l =
O and vice versa. 23. Here l = or l = 2l Since, k = =
2 l 2l l
(a) Situation 1.
10m/s
The amplitude of vibration at a distance x from x = 0 is given
2m/s by A = a sin k x
S Mechanical energy at x of length dx is
(Transmitter) O
(Receiver) 1 2 2 1 2 2
dE = (dm) A w = (µdx)(a sin k x ) (2pn)
2 2
Waves P-S- 141

= 2p2µn2a2 sin2 kx dx 2. A-p,t; B-p,s; C-q,s; D-q,r


But v = nl (A) Pipe closed at one end
v v2 T / µ Waves produced are longitudinal (sound waves)
\ n= Þ n2 = 2 = 2 éQ v = T / m ù L
l l 4l ë û
lf
T /µ ìæ p ö ü
=L
\ dE = 2p2µ 2 a2 sin 2 íç ÷ x ý dx 4
4l îè l ø þ \ lf = 4 L lf
\ Total energy of the string (p, t) are correct matching 4
l 2 T /µ 2 2 æ p xö (B) Pipe open at both ends
E = ò dE = ò 2p µ 2 a sin ç ÷ dx
0 4l è l ø waves produced are longitudinal (sound waves)
p 2Ta 2 lf L
= =L
4l 2
24. Let the speed of the train be vT
While the train is approaching \ lf = 2L lf
Let n be the actual frequency of the whistle. Then (p, s) are correct matching. 2
vS (c) Stretched wire clamped at both ends
n' = n Waves produced are transverse in nature. (waves on string)
v S - vT
where vS = Speed of sound = 300 m/s (given) lf
n' = 2.2 K Hz. = 2200 Hz (given) =L
2
300 lf
\ 2200 = n ... (i) \ lf = 2L
300 - vT 2
(q, s) are correct matching.
While the train is receding
(D) Stretched wave clamped at both ends & mid point
vS Waves produced are transverse in nature (waves on string)
n'' = n
v S + vT
Here, v' = 1.8 KHz = 1800 Hz (given)
300 lf = L
\ 1800 = n ... (ii) lf
300 + vT
Dividing (i) and (ii) (q, r) are correct matching.
2200 300 300 + vT
= ´ Þ vT = 30 m/s G. Comprehension Based Questions
1800 300 - v T 300
25. KEY CONCEPT : The wave form of a transverse harmonic 1. (a) 2. (c) 3. (d)
disturbance is The equations are y 1 = A cos (0.5 px – 100 pt) and
y = a sin (wt ± kx ± f) y2 = A cos (0.46 px – 92 pt) represents two progressive wave
Given vmax = aw = 3 m/s ... (i) travelling in the same direction with slight difference in the
Amax = aw2 = 90 m/s2 ... (ii) frequency. This will give the phenomenon of beats.
Velocity of wave v = 20 m/s ... (iii) Comparing it with the equation
Dividing (ii) by (i) y = A cos (kx – wt), we get
w1 = 100 p Þ 2pf1 = 100 p Þ f1 = 50 Hz and
aw 2 90
= Þ w = 30 rad/s ... (iv) 2p
aw 3 K1 = 0.5 p Þ = 0.5p Þ l1 = 4 m
Substituting the value of w in (i), we get l1
3 w
a= = 0.1m ... (v) Wave velocity = l1f1 = 200 m/s [Alternatively use v = ]
30 K
2p 2p w 30 3 w2 = 92 p Þ 2pf2 = 92 p Þ f2 = 46 Hz
Now, k = = = = = ... (vi) Therefore beat frequency = f1 – f2 = 4 Hz and
l v / n v 20 2
From (iv), (v) and (vi) the wave form is 2p 200
K2 = 0.46 p Þ = 0.46p Þ l2=
3 l2 46
y = 0.1 sin [30t ± x ± f]
2 200
Wave velocity = ×46 = 200 m/s
F. Match the Following 46
NOTE : Wave velocity is same because it depends on the
1. (A) Pitch q. frequency
medium in which the wave is travelling.
(B) quality p. waveform Now, at x = 0,
(C) loudness r. intensity y1 + y2 = (A cos 10 pt) + (A cos 92 pt) = 0
EBD_7036
P-S- 142 Topic-wise Solved Papers - PHYSICS
Þ cos 100 pt = – cos 92 pt = cos (– 92 pt) l 10
= = = 5cm
2n + 1 2 2
= cos [(2n + 1)p – 92 pt Þ t = 2. Let v be the speed of sound and vc and f0 the speed and
192
1 191 frequency of car.
when t = 0, n = - and when t = 1, n = = 95.2 The frequency of sound reflected by the car is
2 2
Þ net amplitude is zero for n = 96 times (the nearest é v + vc ù
\ f1¢ = f 0 ê ú
answer). ë v - vc û
4. (b) The speed of sound depends on the frame of reference Differentiating the above equation w.r.t. vc ,we get
of the observer.
é d d ù
5. (a) Since all the passengers in train A are moving with a
d f1¢ ê (v - vc ) dv (v + vc ) - (v + vc ) dv (v - vc ) ú
velocity of 20 m/s therefore the distribution of sound = f0 ê c
2
c ú
intensity of the whistle by the passengers in train A is dvc ê ( v - vc ) ú
uniform. êë úû

é v - v0 ù é 340 - 30 ù 31 df ¢ é 2v ù 2v
6. (a) n ' = n1 ê ú = 800 ê 340 - 20 ú = 800 ´ 32 \ 1 = f0 ê 2ú
= f 0 2 (Q vc << v)
ë v - vs û ë û dvc ëê (v - vc ) ûú v
é v - v0 ù 31 df1¢ 2
n '' = n2 ê ú = 1120 ´ 32 \ ´ 100 = ´ dvc
ë v - vs û f0 v
31 31 2 ´ dvc
\ n ''- n ' = (1120 – 800) × = 320 ´ = 310 Hz. \ 0.012 = \ dvc = 0.198 m/s » 7 km/h
32 32 330
2 2
I. Integer Value Correct Type 3. Resultant amplitude, A = A1 + A2 + 2 A1 A2 cos f
p
T 0.5 = 42 + 32 + 2 ´ 4 ´ 3 ´ cos = 16 + 9 + 0 = 5
1. We know that, n = = = 10m/s 2
-
m 10 3 / 0.2 é p 2p ù
The wavelength of the wave established 4. (3) y = I 0 êsin O + sin + sin + sin pú
ë 3 3 û
v 10
l= = = 0.1m = 10cm é 3 3ù
f 100 y = I0 ê + ú = 3 I0
\ The distance between two successive nodes ë 2 2 û
\ Ir = y2 = 3I0 Þ n=3

Section-B JEE Main/ AIEEE


1. (b) This will happen for fundamental mode of vibration as 2 beats/sec. When a little wax is placed on the unknown
shown in the figure. S1and S2are rigid support fork, its frequency decreases and simultaneously the
l beat frequency decreases confirming that the frequency
Here = 40 \ l = 80 cm of the unknown fork is 292 cps.
2
4. (b) To form a node there should be superposition of this
wave with the reflected wave. The reflected wave
S1 S2
should travel in opposite direction with a phase change
of p. The equation of the reflected wave will be
40 cm
2. (c) KEY CONCEPT : The fundamental frequency for y = a sin (wt + kx + p)
Þ y = – a sin (wt + kx)
v 5. (b) KEY CONCEPT : The frequency of a tuning fork is
closed organ pipe is given by uc = and
4l given by the expression
v m2 k Y
For open organ pipe is given by u0 = f=
2l 4 3 pl 2 r
u0 v 4l 2 As temperature increases, l increases and therefore f
\ u = 2l ´ v = 1 decreases.
c
3. (b) A tuning fork produces 4 beats/sec with another tuning æ pö
6. (a) y = 10-4 sin ç 600t - 2x + ÷
fork of frequency 288 cps. From this information we è 3ø
can conclude that the frequency of unknown fork is
288 + 4 cps or 288 – 4 cps i.e. 292 cps or 284 cps. When But y = A sin ( wt - kx + f)
a little wax is placed on the unknown fork, it produces On comparing we get w = 600; k=2
Waves P-S- 143

w 600 æI ö æI ö
v= = = 300 ms -1 or, 20 = 10 log ç 1 ÷ or, 2 = log ç 1 ÷
k 2 è I2 ø è I2 ø
7. (a) KEY CONCEPT : For a string vibrating between two
rigid support, the fundamental frequency is given by I1 I
or, = 102 or, I2 = 1 .
I2 100
1 T 1 10 ´ 9.8
n= = = 50 Hz Þ Intensity decreases by a factor 100.
2l m 2 ´ 1 9.8 ´ 10-3
As the string is vibrating in resonance to a.c of v v
15. (b) For first resonant length n = = (in winter)
frequency n, therefore both the frequencies are same. 4l1 4 ´ 18
8. (c) A tuning fork of frequency 256 Hz makes 5 beats/
For second resonant length
second with the vibrating string of a piano. Therefore
the frequency of the vibrating string of piano is 3v ' 3v ' v 3v'
n' = = (in summer) \ =
(256 ± 5) Hz ie either 261Hz or 251 Hz. When the tension 4l 2 4x 4 ×18 4× x
in the piano string increases, its frequency will
increases. Now since the beat frequency decreases, v' v'
\ x = 3 ´ 18 ´ \ x = 54 ´ cm
we can conclude that the frequency of piano string is v v
251Hz v' > v because velocity of light is greater in summer as
9. (b) From equation given,
compared to winter (v µ T )
w 100
w = 100 and k = 20, v = = = 5m / s \ x > 54cm
k 20
10. (d) No. of beats heard when fork 2 is sounded with fork 1 16 (a) y(x, t) = 0.005 cos (ax - bt) (Given)
= Dn = 4 Comparing it with the standard equation of wave
Now we know that if on loading (attaching tape) an
unknown fork, the beat frequency increases (from 4 to y(x, t) = a cos (kx - wt) we get
6 in this case) then the frequency of the unknown fork k = a and w = b
2 is given by,
n = n 0 - Dn = 200 – 4 = 196 Hz 2p 2p
\ = a and =b
l T
é vù
é v + v0 ù êv + 5 ú é6ù 2p 2p
11. (c) n' = nê ú = nê ú = nê ú \ a= = 25p and b = =p
ë v û ë v û ë5û 0.08 2
17. (b) Maximum number of beats = ( n + 1) – ( n – 1) = 2
n' 6 n' - n 6 - 5 2
= ; = ´ 100 = 20% u=0 a = 2m/s vm
n 5 n 5 18. (a)
Electric s Motor
é v ù é 300 ù siren cycle
12. (c) n' = n ê ú Þ 10000 = 9500ê ú
ë v - vs û ë 300 - v û v 2m - u 2 = 2as Þ v2m = 2 ´ 2 ´ s \ vm = 2 s
-1
Þ 300 - v = 300 ´ 0.95 Þ v = 300 - 285 = 15 ms According to Doppler’s effect
v
13. (a) Given nv = 315 and ( n + 1) = 420 é 330 - 2 s ù
2l 2l 0.94v = v ê ú Þ s = 98.01 m
ë 330 û
n + 1 420
Þ = Þn =3
n 315 é æ t ö x ù
19. (d) y = 0.02(m)sin ê 2p ç ÷ - ú
Hence 3 ´
v
= 315 Þ
v
= 105 Hz ë è 0.04( s) ø 0.50(m) û
2l 2l
But y = a sin(wt - kx )
Lowest resonant frequency is when n = 1
Therefore lowest resonant frequency = 105 Hz. 2p 1
\ w = 0.04 Þ n = 0.04 = 25 Hz
æI ö æ I2 ö
14. (a) We have, L1 = 10 log ç 1 ÷ ; L2 = 10 log ç ÷
è I0 ø è I0 ø 2p
k= Þ l = 0.5 m
0.50
æI ö æI ö
\ L1 – L2 = 10 log ç 1 ÷ - 10log ç 2 ÷ \ velocity, v = nl = 25 × 0.5 m/s = 12.5 m/s
è I0 ø è I0 ø Velocity on a string is given by
æI I ö æI ö T
or, DL = 10 log ç 1 ´ 0 ÷ or, DL = 10log ç 1 ÷ v= 2 = (12.5)2 × 0.04 = 6.25 N
è I0 I 2 ø è I2 ø m \ T = v ´m
EBD_7036
P-S- 144 Topic-wise Solved Papers - PHYSICS

(a) Given wave equation is y(x,t) = e(- ax +bt 2 + 2 ab xt ) (2n - 1) ´ 340


2
20. £ 1250
Þ
0.85 ´ 4
-[( ax ) 2 + ( b t ) 2 + 2 a x . b t ] -( ax + bt ) 2
=e =e Þ 2n – 1 < 12.5 » 6
2
æ ö é v ù 320
- çè x +
b ÷
tø 24. (d) f1 = f ê ú= f´ Hz
= e a
ë v - vsû 300
It is a function of type y = f (x + vt)
é v ù 320
f2 = f ê ú= f ´ Hz
b
Þ Speed of wave = ë v + vs û 340
a
21. (a) The fundamental frequency of open tube æ f2 ö æ 300 ö
çè f - 1÷ø ´ 100 = çè 340 - 1÷ø ´ 100 ; 12%
v 1
n0 = … (i) 25. (a) We know that velocity in string is given by
2l0
That of closed pipe T
v= ...(I)
v m
nc = … (ii)
4lc
m mass of string
where m = =
l0 l length of string
According to the problem lc =
2
m
The tension T = ´ x´g ..(II)
v v l
Thus nc = Þ nc … (iii)
l0 /2 2l
From equations (i) and (iii) From (a) and (b)
l
n 0 = nc dx T
= gx
dt
Thus, nc = f ( Q n0 = f is given) x
22. (b) Fundamental frequency,
l l
v 1 T 1 T é T mù x -1/2 dx = g dt \ ò x -1/2dx - g ò dt
f = = = êQ v = and m = ú 0 0
2l 2l m 2l Ar ë m lû
l 20
Tl T Y Dl 1 gDl 2 l = g´t \ t = 2 =2 =2 2
Also, Y = Þ = Þ f = ....(i) g 10
ADl A l 2l lr
f
Dl 26. (b)
Putting the value of l, , r and g in eqn. (i) we get,
l
l l
2 103
f = ´
7 3 or, f » 178.2 Hz
(a) (b)
23. (c) Length of pipe = 85 cm = 0.85m
Frequency of oscillations of air column in closed organ v
pipe is given by, The fundamental frequency in case (a) is f =
2l
(2n - 1)u The fundamental frequency in case (b) is
f =
4L
v u
(2n - 1)u f'= = =f
f = £ 1250 4(l / 2) 2l
4L
12
Electrostatics P-S- 145

Electrostatics

Section-A : JEE Advanced/ IIT-JEE

e0 A 2e 0 A 1 Q2 3V
A 1. ´V ; ´V 2. B 3. 180°, ´ 4.
d d 4pe 0 4 L2 k+2

1 q´q
5. –qEa 6. –8 7. 4pe 0 L2
B 1. F 2. T 3. T 4. T 5. F 6. F
C 1. (b) 2. (b) 3. (b) 4. (d) 5. (d) 6. (b) 7. (b)
8. (c) 9. (a) 10. (b) 11. (b) 12. (c) 13. (c) 14. (c)
15. (d) 16. (c) 17. (a) 18. (d) 19. (b) 20. (c) 21. (c)
22. (a) 23. (b) 24. (a) 25. (a) 26. (d) 27. (c) 28. (d)
29. (c) 30. (a) 31. (c) 32. (d) 33. (c) 34. (c)
D 1. (d) 2. (a, d) 3. (b) 4. (b, d) 5. (a) 6. (a) 7. (a, c, d)
8. (b) 9. (c) 10. (d) 11. (b, c) 12. (d) 13. (a, c) 14. (a, c)
15. (c, d) 16. (a,b,d) 17. (a) 18. (a, d) 19. (a, b, c, d) 20. (c, d) 21. (a, c, d)
22. (a, b, c) 23. (d) 24. (b, d) 25. (c, d) 26. (c) 27. (a, d) 28. (c)
29. (d) 30. (d)

4 3q q2
E 1. (i) Move towards centre; (ii) Q = , 3(2 + 3) K 2
9 a

qq KQ( R + r ) 3
2. (i) 60°; (ii) mg = ± k 12 2 ; (iii) N1 = 3 mg ; N2 = mg 3. 2 2 4. 0.628 sec. 5.
l R +r 5

p ML
6. 8.48 m 7. 3.16 × 10–9 C 8.
2 2qE

s s æ a2 ö s æ a 2 - b 2 + c2 ö
9. (i) Î ( a - b + c ) , Î ç b - b + c ÷, ç ÷ (ii) c = a + b
0 0è ø Î0 è c ø

é 1 2 ù 1 æ Qq ö
10. (a) 4a, (5a, 0) (b) KQ ê - (c) ç ÷
ë 3a - x 3a + x úû 4p Î0 è 2 ma ø

3Q 2 3GM 2 32
Q2
11 (a) (b) , 1.5 ´ 10 J (c)
20p Î0 R 5R 8p Î0 R

lq
12. (i) 2 × 10–9 F, 1.21 × 10–5 J (ii) 4.84 × 10–5 J (iii) 1.1 × 10–5 J 13. 2 Î0 m

K1 K 2 A Î0 K
14. 4.425 × 10–9 A. 15. log 1
d ( K1 - K 2 ) K2
16. (i) 90 × 10–6 C, 210 × 10–6 C, 150 × 10–6 C (ii) 4.74 × 10–2 J, 1.8 × 10–2 J
EBD_7036
P-S- 146 Topic-wise Solved Papers - PHYSICS

2
1 1 ìï QR é æ R ö n ù üï Q2 R a
4a
17. (a) ´ ´í ê1 - ç ú
÷ ý (b) 18. (a) (b)
2 4p Î0 R ï r ê è R + r ø ú ï 2(4pe 0 ) r 2 3 3
î ë ûþ

1 q2 4
19. 3 m/s, 3 × 10–4 J 20. . (3 3 - 3 6 - 2)
4p Î0 a 6

1 pQ 1 2 pQ ˆ q0 ( s1 - s 2 ) a æ aö
1/ 3
21. (a) 4p Î 2 (b) 4p Î i 22. 23. V ç ÷
0 d 0 d
3 2 Î0 è 3t ø
F 1. (A)-(p, r, s): (B)-(r, s); (C)-(p, q, t); (D)-(r, s) 2. (a)
G 1. (a) 2. (b) 3. (c) 4. (c) 5. (d)
H 1. (a)
I 1. 2 2. 3 3. 6 4. 6
Section-B : JEE Main/ AIEEE
1. (a) 2. (b) 3. (b) 4. (d) 5. (a) 6. (a) 7. (b) 8. (c) 9. (d)
10. (b) 11. (d) 12. (d) 13. (d) 14. (b) 15. (c) 16. (b) 17. (a) 18. (b)
19. (c) 20. (c) 21. (c) 22. (a) 23. (c) 24. (c) 25. (a) 26. (a) 27. (a)
28. (b) 29. (c) 30. (a) 31. (c) 32. (d) 33. (a) 34. (b) 35. (c) 36. (a)
37. (d) 38. (c) 39. (c) 40. (c) 41. (b) 42. (a) 43. (d) 44. (c) 45. (a)
46. (d) 47. (a,b) 48. (c) 49. (a) 50. (c)

Section-A JEE Advanced/ IIT-JEE

A. Fill in the Blanks KC


1. On the plate 1 there is +ve charge V'
e0 A
q = CV = ´V V'
d 2C
On the plate 4 the charge is
Finally charge on capacitance C = q1' = KCV '
-2 e 0 A
–2q = ´V Charge on capacitance 2C = q2' = 2CV '
d
Total charge will remain conserved
2. It is greatest at point B because at B the equipotential 3V
\ CV + 2C V = KCV ' + 2CV ' or, V ' =
surfaces are closest. K+2
3. There is no gravitational force acting. Only electrostatic 5. NOTE : Since electric field is conservative in nature, the
force of repulsion is acting which will take the two balls as work done by the field along PQRS will be same as along
far as possible. The angle between the two strings will be PMS
180°. The tension in the string will be equal to the ur uuuur
electrostatic force of repulsion Work done from P to M = F . PM
O = F (PM) cos 90° = 0
Q 180 Q
Y
T T E
P (a,b,o)
1 Q´Q 1 Q2
T= ´ = ´ 0,0
) Q
4 p e 0 (2 L)2 4 p e 0 4L2 (0, S a M X

4. Initially charge on capacitance C = q1 = CV R


Charge on capacitance C = q2 = 2CV
Electrostatics P-S- 147
ur uuur
Work done from M to S = F . MS If we take another diametrically opposite length dl, the
= F (MS) cos 180° [Q F = qE] charge on it being dq. Then the force on charge (– q) by this
= – qEa small charge dq will be
6. Electric potential V = 4x2 volts qdq
The electric potential changes only along x-axis, dF = k
We know that 5 R2
Again resolving this force, we find dFsin q components of
-dV d (4 x 2 )
Ex = Þ Ex = - = - 8x the two forces cancel out and dFcos q component adds up.
dx dx
\ The total force
The electric field at point (1, 0, 2) will be (here x = 1)
Ex = – 8 volt/m. 2 pR 2 pR kqdq 2R
7. F= ò0 dF cos q = ò
0 5R 2
´
5R
Charge on length 2pR = Q
Q dl
\ Charge on length d l = = dq
2 pR
2 pR 2kq Q dl
\ F= ò0 5 5R 2
´
2 pR
Force on (–q) due to charge at D will get cancelled out by 2kQq 2 kQq
force on (–q) due to change on A. Similarly force on –q due = ´ 2 pR =
3
to charge at E will get cancelled out due to charge on B. So 5 5 ´ 2pR 5 5 R2
This is not an equation of simple harmonic motion.
1 q2
the net force will be because of charge on C F = 6. For a particle to move in circular motion, we need a centripetal
4p Î0 L2 force which is not available.
directed from O to C. The statement is false.
B. True/ False C. MCQs with ONE Correct Answer
1. Electrostatic force is conservative in nature, therefore work 1. (b) The potential at the surface of a hollow or conducting
done is path independent. sphere is same as the potential at the centre of the
2. The metallic sphere which gets negatively charged gains sphere and any point inside the sphere.
electrons and hence its mass increases. 2. (b) The two charges form an electric dipole. If we take a
The metallic sphere which gets positively charged loses point M on the X-axis as shown in the figure, then the
electrons and hence its mass decreases. net electric field is in –X-direction.
3. When high energy X-ray beam falls, it \ Option (a) is incorrect.
If we take a point N on Y- Y
will knock out electrons from the small
metal ball making it positively charged. axis, we find net electric
field along + X direction. N E
Therefore the ball will be deflected in
the direction of electric field. The same will be true for +q M E –q E
any point on Y-axis. (b) is X
(–d, 0) (d, 0) M
a correct option. p
4. The electric field produced between the parallel plate
NOTE : For any point on
capacitor is uniform. The force acting on charged particle
the equatorial line of a
placed in an electric field is given by F = qE.
dipole, the electric field is
In the case of two protons, q and E are equal and hence
opposite to the direction
force will be equal.
of dipole moment.
5. KEY CONCEPT : Force on charge (– q) due to small charge
(b) W¥ – 0 = q (V¥–V0) = q (0 – 0) = 0
dq situated at length d l is given by \ (c) is incorrect. The direction of dipole moment is
from –ve to + ve. Therefore (d) is incorrect.
3. (b) C and 2C are in parallel to each other.
\ Resultant capacity = (2C + C)
qdq CR = 3C
dF = k Net potential = 2V – V
5 R2
VR = V

Resolving this force into two parts dFcos q and dFsin q as 1 1 3


\ Final energy = C R (VR )2 = (3C )(V )2 = CV 2
shown in figure. 2 2 2
EBD_7036
P-S- 148 Topic-wise Solved Papers - PHYSICS
4. (d) Within the capacitor, d d d
\ = +
Q1 Q KAe0 Ae 0 ( K1 + K 2 ) 2 Ae 0 K 3
E1 = ; E2 = 2 ;
2 e0 A 2e 0 A
1 1 1
where A = area of each plate or = + .
K K1 + K 2 2 K3
d = separation between two plate
8. (c) Electric field lines do not form closed loops. Therefore
1 options (a), (b) and (d) are wrong.
E = E1 – E2 = (Q1 - Q2 )
2e 0 A Option (c) is correct. There is repulsion between similar
charges.
Hence, V = Ed 9. (a) When S is closed, there will be no shifting of negative
1 d Q - Q2 charge from plate A to B as the charge – q is held by the
= (Q1 - Q2 ) = 1 charge + q. Neither there will be any shifting of charge
2 e0 A 2C
from B to A.
5. (d) With the closing of switch S3 and S1 the negative 10. (b) NOTE : As we move along the direction of electric
charge on C2 will attract the positive charge on C2 field the potential decreases.
thereby maintaining the negative charge on C1. The \ VA > VB
negative charge on C1 will attract the positive charge y
on C1. No transfer of charge will take place. Therefore
p.d across C1 and C2 will be 30 V and 20 V.
(0, 1) C
Qq q 2 Qq
6. (b) Here we have + + =0 E
a a a 2
(0, 0) (1, 0)
q 2 2q x
A
\ Q= - =- B
2 +1 2+ 2
11. (b) Initial energy
2Qq q Q q
7. (b) Ui =
4pe 0 a x = –a 0 x=+a
Initially
q Q q
Final energy x

a+x a–x
Finally

2Qqa
Qq é 1 1 ù =

Let C1 = Capacity of capacitor with K1


Uf =
4pe 0 êë a + x + a - x úû (
4 pe0 a 2 - x 2 )
C2 = Capacity of capacitor with K2 é ù
2Qq ê 1 a ú
C3 = Capacity of capacitor with K3 U i - Uf = -
æ A ö e ´ 2 Ae 0 K1 êë (
4pe 0 ê a a 2 - x 2 ú
úû )
\ C1 = K1 ç ÷ 0 =
è 2ø d d é ù
2Qq ê a 2 - x 2 - a 2 ú -2Qqx2
= =
\
æ A ö e ´ 2 Ae 0 K 2
C2 = K 2 ç ÷ 0
è 2ø d
=
d
êë (
4pe 0 ê a a 2 - x 2 ú
úû )
4pe 0 a3

when x << a then x 2 is neglected in denominator..


e0 ´ 2 2 Ae0 K3
\ C3 = K3 ( A) =
æ -Qq ö 2
d d
C1 and C2 are in parallel Ui – Uf = ç 3÷
x
è 2pe 0 a ø
Ae 0 12. (c) Initially we know that
\ Ceq =
d
( K1 + K2 )
1 C1C2
DU = (V1 – V2 )2
Ceq and C3 are in series 2 C1 + C2

1 d d 1 C ´C
\ = + DU = ´ (V1 – V2 )2
C Ae 0 ( K1 + K 2 ) 2 Ae 0 K3 2 2C
C
KAe 0 DU = (V1 - V2 )2
But C = for single equivalent capacitor 4
d
Electrostatics P-S- 149

13. (c) Electric field everywhere inside the metallic portion of 18. (d) When a positive point charge is
shell is zero. placed outside a conducting net charge = 0
Hence options (a) and (d) are incorrect. sphere, a rearr angement of
Electric field lines are always normal to a surface. Hence charge takes place on the surface. - +
- +
option (b) is incorrect. Only option (c) represents the But the total charge on the - +
sphere is zero as no charge has +q -- +
correct answer. +
left or entered the sphere.
ur Kq 19. (b) Let us consider a uniformly
14. (c) | E |= 2 charged solid sphere without
r
any cavity. Let the charge per
Electric field due to P on O unit volume be s and O be the
is cancelled by electric field P- +Q centre of the sphere. Let us O
due to S on O. E consider a uniformly charged a
Similarly Electric field due - E sphere of negative charged P O¢
to Q on O is cancelled by U
r +
R density s having its centre at
E O E
electric field due to T and O. O'. Also let OO' be equal to a.
E Let us consider an arbitrary point P in the small sphere.
The electric field due to R E
- The electric field due to charge on big sphere
on O is in the same +T S
direction as that of U on O. ur s uuur
E1 = OP
Therefore the net electric 3e 0
ur
field is 2E . Also the electric field due to small sphere
15. (d) The flux through the Gaussian surface is due to the ur s uuur
E2 = PO ' \ The total electric field
charges inside the Gaussian surface. But the electric 3e 0
field on the Gaussian surface will be due to the charges
present in side the Gaussian surface and outside it. It ur ur ur s uuur uuur s uuur
E = E1 + E 2 = [OP + PO] = OO '
will be due to all the charges. 3e 0 3e 0
16. (c) Figure shows the electric fields due to the sheets 1, 2 Thus electric field will have a finite value which will be
and 3 at point P. The direction of electric fields are uniform.
according to the charge on the sheets (away from 20. (c) The charges make an electric dipole. A and B points lie
positively charge sheet and towards the negatively on the equatorial plane of the dipole.
charged sheet and perpendicular). Therefore, potential at A = potential at B = 0
W = q (VA – VB) = q × 0 = 0
1
Z = 3a P -s
E1
E2
E3 2
Z=a -2s

3
Z = –a -s
21. (c) The electric field due to A and B at O are equal and
The total electric field opposite producing a resultant which is zero. The
ur ur ur ur
E = E1 + E 2 + E 3 electric field at O due to C is
1 2q / 3 q
= E1 ( -k$ ) + E2 ( -k$ ) + E3 ( - k$ ) E= = .
4π Îo R 2 6 π Îo R 2
é s 2s s ù $ 2s $ \ Option [A] is not correct. The electric potential at O is
=ê + + ú (-k ) =- k
ë 2e 0 2e 0 2e 0 û e0 q
17. (a) When a charge density is given to the inner cylinder, B 3
30 º
the potential developed at its surface is different from - 2q C R
that on the outer cylinder. This is because the potential 3 60º
O
decreases with distance for a charged conducting R
q A
cylinder when the point of consideration is outside the
cylinder. 3
But when a charge density is given to the outer cylinder, é +q / 3 ù é +q / 3 ù é -2q / 3 ù
it will change its potential by the same amount as that VO= K ê ú +K ê ú +K ê =0
of the inner cylinder. Therefore no potential difference ë R û ë R û ë R úû
will be produced between the cylinders in this case. \ Option [D] is wrong
EBD_7036
P-S- 150 Topic-wise Solved Papers - PHYSICS
+ +
AC AB + – –(Q 1+ Q 2)
In D ABC = sin 30º ÞAC= =R –
(Q1+ Q 2) +
– +
AB 2 + – Q 1+ Q 2+ Q 3
– + –
– – +
BC 3AB + +
Also =sin 60º Þ BC = = 3R + –

–Q1
AB 2 – +
Potential energy of the system + – +
+ R Q1 – +
+ –
é( q / 3)( 2/ 3) ù é ( q/3)( -2q / 3) ù é( q/3)( -2q / 3) ù – +
Kê ú+Kê ú +Kê ú –
ë 2R û ë R û ë 3R û –
+
2R +

+ –
– + +
2 –
kq é 1 2 ù + 3R –
= ê -2- ú ¹0
+
– – +
9R ë 2 3û + +
\ Option [B] is wrong Q1 Q1 + Q2 Q1 + Q2 + Q3
= = = x(say)
Magnitude of force between B and C is 4pR 2
4p (2 R ) 2
4 p(3R ) 2
1 ( 2q / 3)( q / 3) q2
\ Q1 = 4pR 2 x
F= =
4π Îo
( 3R ) 2 54π Îo R 2
Q1 + Q2 = 4p(2R )2 k = 4[4pR 2 x ]
22. (a) Let the level of liquid at an instant of time 't' be x. Then
dx
Þ Q2 = 4[4pR 2 x ] - Q1
v=– Þ dx = –vdt
dt = 4[4pR 2 x ] - 4pR 2 x = 3[4pR2x]
x t
Also Q1 + Q2 + Q3 = 4p(3R)2 x = 9[4pR 2 x]
Þ ò dx = – v ò dt
d /3 0 \ Q3 = 9[4pR 2 x] - Q1 - Q2 = 9[4pR 2 x ] - [4pR 2 x ]
®

d
d
Þx – x -3[4pR 2 x] = 5[4pR 2 x ]
®®

=– vt
®

3
Þ Q1 : Q2 : Q3 = 1: 3 : 5
d
Þ x= – vt 24. (a) From the figure it is clear that the charge enclosed in
3
Also the capacitance can be considered as an equivalent the cubical surface is 3C + 2C – 7C = –2C. Therefore
of two capacitances in series such that the electric flux through the cube is
1 1 1
= + Y
Ceq C1 C 2 Z

1 1 1 d- x x
Þ = + = +
Ceq Îo A Îo AK Îo A Îo AK
d –x x
qin -2C
f= = X
Î0 AK e0 e0
\ Ceq = 2C
Kd + x(1 - K) 3C
–7C
d
But A = 1, K = 2 and x = – vt
3
25. (a) The electrostatic pressure at a point on the surface of
Î0 ´1 ´ 2 6 Î0
\ Ceq = =
éd ù 5d + 3vt s2
2d + ê - vt ú (1 - 2 ) a uniformly charged sphere =
ë 3 û 2 Î0

6R Î0
\ Time constant t = RCeq = s2
5d + 3vt \ The force on a hemispherical shell = ´ pR 2
2 Î0
23. (b) The charges on the surfaces of the metallic spheres are
shown in the diagram. It is given that the surface charge 26. (d) When the electric field is on
densities on the outer surfaces of the shells are equal. Force due to electric field = weight
Therefore
qE = mg
Electrostatics P-S- 151

Here, C1 = 2mF, C2 8 mF, V1= V, V2 = 0


4 3 4pR 3rg
qE = pR rg \q = ...(i) 1 2 ´8 4
3 3E (V - 0)2 = V 2
\ DU = ´
When the electric field is switched off 2 2+8 5
Weight = viscous drag force DU
The percentage of the energy dissipated = ´ 100
mg = 6phRvt Ui

9hvt 4 2
4 3 V
pR rg = 6phRvt \ R = ...(ii)
3 2rg = 5 ´ 100 = 80%
V2
3 29. (c) The pattern of field lines shown in option (c) is correct
4 é 9hvt ù 2 rg because
From (i) & (ii) q = p ê ´
3 ë 2rg úû E (a) a current carrying toroid produces magnetic field
lines of such pattern
3 (b) a changing magnetic field with respect to time in
4 é 9 ´ 1.8 ´ 10-5 ´ 2 ´ 10-3 ù 2 900 ´ 9.8 ´ 7 a region perpendicular to the paper produces
= ´pê ú ´
3 2 ´ 900 ´ 9.8 81p ´ 105 induced electric field lines of such pattern.
ë û
30. (a) The frequency of SHM performed by wooden block is
= 7.8 × 10–19 C E
r 1 k
27. (c) Given E = Eo xˆ n0 = +Q
2p m
This shows that the electric field acts along + x direction
and is a constant. The area vector makes an angle
of 45° with the electric field. Therefore the electric when electric field is switched on, the value of k and m
flux through the shaded portion whose area is is not affected and therefore the frequency of SHM
r r remains the same. But as an external force QE starts
a´ 2a = 2 a 2 is f = E. A = EA cos q= E0 ( 2 a 2 ) cos
acting on the block towards right, the mean position of
2 1
45° = E0 ( 2 a ) ´ = E0a2 QE
2 SHM shifts towards right by
z k
correct option is (a).
Note : In SHM if a constant additional force is applied
then it only shift the quilibrium position and does not
(a,0,a) (a,a,a) q = 45°
2a
change the frequency fo SHM.
E 31. (c) The two forces acting on the
q A proton just after the release are qE
y shown in the figure. In this +
(0,0,0) (0,a,0)
q
situation
v
qE = mg [\ q = 45º]
x mg
28. (d) When S and 1 are connected æV ö
The 2mF capacitor gets charged. The potential \ q ç ÷ = mg
èdø
difference across its plates will be V.
The potential energy stored in 2 mF capacitor mgd 1.67 ´ 10 -27 ´ 10 ´ 10 -2
\ V= = -19
= 10 -9 V
1 1 q 1.6 ´ 10
U i = CV 2 = ´ 2 ´ V 2 = V 2
2 2 32. (d) For a thin uniformly positive charged spherical shell
When S and 2 are connected (i) Inside the shell at any point
The 8mF capacitor also gets charged. During this 1 q
charging process current flows in the wire and some E = O and V = = constt.
4 p Î0 R
amount of energy is dissipated as heat. The energy
where q = charge on sphere
loss is
R = Radius of sphere
1 C1 C2 2 (ii) Outside the shell at any point at any distance r
DU = 2 C + C (V1 - V2 )
1 2 1 1
from the centre E µ and V µ
2 r
r
EBD_7036
P-S- 152 Topic-wise Solved Papers - PHYSICS
33. (c) The total charge on plate A will be – 80 µC. If qB and qC (i) P.d. = V0 P.d. = V0
be the charges on plate B and C then Capacitance = C Capacitance = KC
qB + qC = 80 µC ...(1) (ii) Q0 = CV0 [K is the dielectric constant of
Also 2µF and 3µF capacitors are slab K > 1]
+80 mC New charge = KCV0
in parallel. Therefore,
4mF (iii) Potential Energy New potential energy
qB qC –80 mC A
= 1 1
2 3 = CV02 = KCV02
+qB B +qC C 2 2
80 - qC qC 3mF
\ = 2mF V0 V0
2 3 –qB –qC (iv) E = E=
d d
240 3. (b) q has to be negative for equilibrium.
\ 240 – 3qC = 2qC \ qC = = 48µC
5
This charge will obviously be positive. 1 F12 2 F23 3
F13 +Q F21 –q +Q
1 Q
34. (c) E1 = · ; l l
4pÎ0 R 2
Considering equilibrium of 1
1 2Q 1 Q/2 F13 = F12
E2 = · E3 = ·
4pÎ0 R 2 ; 4pÎ0 R 2 KQ ´ Q KQ ( - q ) Q
2
= 2 \q= -
Clearly E2 > E1 > E3 (2l) l 4
where Q/2 is the charge enclosed in a sphere of radius 4. (b, d)
R concentric with the given sphere. Charge on plate is q
e0 A
é ù C=
ê 4Q Q' ú d
ê4 = ú
ê p ( 2 R )3 4 pR 3 ú q = CV Þ V =
q
,
ë3 3 û C
1
D. MCQs with ONE or MORE THAN ONE Correct U= q ´V
2
1. (d) Let us consider the positive charge Q at any instant of Charge on plate is q
time t at a distance x from the origin. It is under the e0 A
r r C' = Þ C' < C,
influence of two forces F1 (= F) and F2 (= F). On d'
resolving these two forces we find that F sin q cancels q
out. The resultant force is V' = Þ V' > V
C'
1
FR = 2F cos q U' = qV' Þ U' = U
2
5. (a) The potential inside the shell will be the same
kQq x everywhere as on its surface. As we add – 3Q charge
=2× ´ on the surface, the potential on the surface changes
(x2 + a2 ) x + a2
2
by the same amount as that inside. Therefore the
2k Q q x potential difference remains the same.
= 6. (a) The equivalent capacitance
( x + a 2 )3 / 2
2
1 1 1 1 11 10
Since FR is not proportional to x, the motion is NOT = + + = Þ Ceq = µF
simple harmonic. The charge Q will accelerate till the Ceq 2 2 2 ´ 5 10 11
origin and gain velocity. At the origin the net force is
zero but due to momentum it will cross the origin and
more towards left. As it comes on negative x-axis, the
force is again towards the origin.
2. (a, d)

vo vo
Electrostatics P-S- 153

7. (a, c, d) q (Q1 - Q2 ) æ 2 - 1ö
=
e0 K e0 A 4pe 0 R çè 2 ø
÷
As C = A C' =
d d 9. (c) Let l be the charge per unit length. Let us consider a
Gaussian surface (dotted cylinder).
Applying Gauss's law
- -
V ur uur l l
f = Ñò E.ds =
+ +
K V'=
K e0 - -
+ +
For the flat portions of - -
l + +
Q = CV Gaussian surface, the
angle between electric - + r + -
e0 A e 0 AV field and surface is 90°.
= ´V Q= = C 'V ' + +
d d Hence flux through flat - -
Q will remain same as no charge is leaving or entering the portions is zero.
plates during the process of slab insertion NOTE : By symmetry, the electric field on the curved surface
Now, Q = C ' V ' = C ' E 'd is same throughout.
ur uur
e 0 AV The angle between E and ds is 0° (for curved surface)
Q 1 V ll ll
E'= = d ´ = Þ E ò ds = Þ E × 2 pr l =
C ' d K e 0 A d Kd e0 e0
d l 1
Work done is the change in energy stored Þ E= Þ Eµ
2 pe 0r r
1 1 10. (d) The electric lines of force cannot enter the metallic
W= CV 2 - C 'V '2
2 2 sphere as electric field inside the solid metallic sphere
2
is zero. Also, the origination and termination of the
1 e0 AV 2 1 K e0 A æ V ö é Vù electric lines of force from the metallic surface is
=
2 d
-
2 d
´ç ÷
èKø êëQ V ' = E ' d = K úû normally (directed towards the centre).
11. (b, c)
1 e0 A 2 é 1 ù r
W= V ê1 - ú In region I and III, there will be electric field E0 directed
2 d ë Kû from positive to negative. In region II, due to orientation of
8. (b) The work done in moving a charge from A to B ur
dipoles, there is an electric field E k present in opposite
ur ur
direction of E 0 . But since E 0 is also present, the net electric
r r ur
field is E0 - Ek in the direction of E 0 as shown in the
diagram. (Q E0 > Ek)

Ek
Eo Enet= Eo – Ek Eo
W = (T.P.E.)A – (T.P.E.)B where T.P.E. = Total Potential Energy _ +
DIELECTRIC
SLAB
éæ æ ö ù
Q1 ö Q2
(T.P.E.)A = ê ç ´ q + ç ÷ qú
ê è 4pe 0 R ø÷ çè 4pe R 2 + R 2 ÷ø ú
ë 0 û
x=0 d d d x = 3d
q é Q ù
= ê Q1 + 2 ú E
4pe 0 R ë 2û d 2d 3d
0 x
éæ æ ö ù
ê Q2 ö Q1 E0
(T.P.E.)B = ç q + ç ÷ qú
ê è 4pe 0 R ÷ø çè 4pe R 2 + R 2 ÷ø ú V
ë 0 û C
B
q é Q ù A
= ê Q2 + 1 ú
4 pe 0 R ë 2û
x
O d 2d 3d
q é Q Q ù
\ W= ê Q1 + 2 - Q2 - 1 ú NOTE : When one moves opposite to the direction of electric
4pe 0 R ë 2 2û field, the potential always increases. The stronger the electric
field, the more is the potential increase. Since in region II,
EBD_7036
P-S- 154 Topic-wise Solved Papers - PHYSICS
the electric field is less as compared to I and III therefore the Option (a) and (b) are dependent on the curvature which is
increase in potential will be less but there has to be increase different at points A and B.
in potential in all the regions from x = 0 to x = 3d. Also where 16. (a,b,d)
V
(a) The whole charge
dV Q will be enclosed in a
E is uniform, = constt.
dx sphere of diameter 2R0.
12. (d) Potential at origin will be given by (b) Electric field E =
zero inside the sphere. R0 r
q é1 1 1 1 ù Hence electric field is + +
V= ê - + - + ....ú
discontinued at r = R0. ++
+ + ++
4pe 0 ë x0 2 x0 3x0 4 x0 û +
(c) Changes in V and

++
++++++
q
V= ln (2) E are continuously

+ +++++
R0
4pe 0 x0 presen t for
13. (a, c) r > R0. Option (c) is
incorrect.

++
Let Q be the charge on the ring, the negative charge – q is +
released from point P (0, 0, Z0). The electric field at P due to (d) For r < R 0 , the +
+++++
++
the charged ring will be along positive z-axis and its potential V is constant
magnitude will be and the electric E
intensity is zero.
1 QZ 0 Obviously, the
E= .
4pe 0 ( R + Z 02 )3 / 2
2 electrostatic energy is
zero for r < R0.
Therefore, force on charge P will be towards centre as 17. (a) The situation is shown in R0 r
shown, and its magnitude is the figure which is similar v
1 Qq to a planet revolving
Fe = qE = . 2 .Z ... (1) around sun. The distance –q
4pe0 ( R + Z02 )3 / 2 0
of – q from + Q is
Similarly, when it crosses the origin, the force is again towards changing, therefore, force
centre O. +Q
between
Thus the motion of the particle is periodic for all values of
the charges will change.
Z0 lying between 0 and ¥.
The speed of the charge – q will be greater when the
Secondly if Z0 << R, (R2 + Z 02 )3/2 » R3
charge is nearer to + Q as compared to when it is far.
1 Qq Therefore, the angular velocity of charge – q is also
Fe = . . Z0 [From equation 1]
4pe 0 R3 variable. The direction of the velocity changes
continously, therefore, linear momentum is also variable.
i.e. the restoring force Fe µ – Z0. Hence the motion of the
The angular momentum of (– q) about Q is constant
particle will be simple harmonic. (Here negative sign implies because the torque about + Q is zero.
that the force is towards its mean position).
18. (a,d)
14. (a, c)
The electric field lines are orginating from Q1 and terminating
KEY CONCEPT : The expressions of the electric field inside
on Q2. Therefore Q1 is positive and Q2 is negative.
1 Qq As the number of lines associated with Q1 is greater than
the sphere (r < R) E = . r ; outside the sphere
4pe 0 R 3 that associated with Q2, therefore |Q1| > |Q2|.
Option (a) is correct.
At a finite distance on the left of Q1 , the electric field
intensity cannot be zero because the electric field created
by Q1 will be greater than Q2. This is because the magnitude
1 Q é Qù
(R < r < ¥) E = .
4pe 0 r 2 of Q1 is greater and the distance smaller ê E µ 2 ú
ë r û
At a finite distance to the right of Q2, the electric field is
zero. Here, the electric field created by Q2 at a particular
Hence, E increases for r < R and decreases for R < r < ¥.
15. (c, d) point will cancel out the electric field created by Q1.
When two points are connected with a conducting path in 19. (a, b, c, d)
electrostatic condition, then the potential of the two points Electric field inside a spherical metallic shell with charge on
is equal. Thus potential at A = Potential at B the surface is always zero. Therefore option [a] is correct.
(c) is the correct option. When the shells are connected with a thin metal wire then
Option (d) is a result of Gauss's law electric potentials will be equal, say V.
q 1 QA 1 QB
Total electric flux through cavity =
Î0 \ 4p Î R = 4 p Î R = V
0 A 0 B
Electrostatics P-S- 155

As RA > RB therefore QA > AB. option [b] is also correct. 1


VO = [2q + q + q – q – q – 2q] = 0
QA 4p e 0 L
2 2 2 PR is perpendicular bisector (the equatorial line) for the
s A 4 pR A RB QA RB 4 pÎ0 R AV
As = = ´ = ´
2 QB R 2 4 p Î0 RBV electric dipoles AB, FE and BC. Therefore the electric
s B QB RA A
2
4 pRB potential will be zero at any point on PR.
At any point ST, the electric field will be directed from S to T.
s A RB The potential decreases along the electric field line.
\ s = Option (c) is also correct
B RA 23. (b, d)
s s Electric field E1 due to smaller sphere at P is
Also E A = A & EB = B
Î0 Î0 4
r1 ´ p R 3
E A s A RB 1 3
= = <1 \ E <E E1 =
EB s B RA A B 4pe 0 (2R)2
Option (d) is also correct
20. (c, d) + + + + +
(a) is not correct because it is valid only when E µ r–2 + + + + + + +
+ +
(b) is not correct + + + +
® ®
E2 P E1 + + + +
(c) is correct as between two point charges we will get a + + + + + + + +
point where the electric field due to the two point + + + 2R R + + + +
+ +
charges cancel out each other. r1
+ + + + + +
(d) is correct when the work done is without accelerating + + + + +
the charge. r2
21. (a, c, d)
1 r pR rR
a E1 = ´ 1 = 1
The electric flux passing through x = + , 4p Î0 3 4 Î0 ´3
2
Electric field E2 due to bigger sphere at P is
a a
x= - , z = + is same due to symmetry.. r R
E2 = 2
2 2 3 Î0
The net electric flux through the cubical region is
rR r R r1
-q + 3q - q
=
q . As E1 = E 2 \ 1 = 2 Þ r =4
4 Î0 ´3 3 Î0 2
e0 e0
Option (d) is correct.
24. (b, d)
+q F E
–q
22. (a, b, c) Step 1 : When S1 is pressed. The capacitor C1 gets charged
such that its upper plate acquires a positive charge + 2 CV0
+ 2q –2q and lower plate – 2 CV0.
A D
Step 2 : When S2 is pressed (S1 open). As C1 = C2 the
charge gets distributed equal. The upper plates of
C1 and C2 now take charge + CV0 each and lower plate
+q –q – CV0 each.
B C (b) and (d) are correct option.
EB +EE
25. (c, d)
Let us consider a point P on the overlapping region. The
electric field intensity at P due to positively charged sphere
r
rr1
=
60º 3 Î0
O 60º EA ED P The electric field intensity at P due to negatively charged
r
rr 2
sphere = . The total electric field,
3 Î0
EF +EC ur ur
uuur uuur ur rr1 rr2 r r r
|EA | uur uuur |ED | uuur uuur E= + = é r1 + r 2 ù
Here =|EB |=|EC | = =| EE |=|EF | = K 3Î 3Î 3Î ë
0 0 0
û
2 2 ur r r
\ EO = EA + ED + (EF + EC) cos 60º + (EB + EC) cos 60º E= r
3 Î0
1 1
= 2K + 2K + (K + K) × + (K + K) × = 6K Therefore the electric field is same in magnitude and
2 2 direction option (c) and (d) are correct.
The electric potential at O is
EBD_7036
P-S- 156 Topic-wise Solved Papers - PHYSICS

27. (a, d) C1
K
P
r1 A/3 Q1 E1
r2
+r –r
r
E2
2A/3 Q2
C2 = C – C1
26. (c) d
r0 This is a combination of two capacitors in parallel.
1 Q
+Q E1 = Therefore
4pÎ0 r 2 C = C1 + C2 \ C2 = C – C1
0

+ r0 1 2l kA
E2 = 2A
+ 4pÎ0 r0 where C1 = and C - C1 =
+ 3Î0 d 3Î0 d
r0 s
++ E3 = C - C1 2
=
++ 2Î0 \ C1 k
++
C 2
E1 = E2 (Given) \ -1 =
C1 k
1 Q 1 2l
= C 2
4pÎ0 r0 4p Î0 r0
2
\ = +1
C1 k
\ Q = 2lr0 ...(1)
E2 = E3 (Given) C 2
= +1
1 2l s l C1 k
= Þ r0 =
4pÎ0 r0 2Î0 sp \ (d) is a correct option.
\ (b) is incorrect
kA
E1 = E3 (Given) Now, Q1 = C1V = ×V
3Î0 d
1 Q s
\ = Þ Q = 2psr02
4pÎ0 r0 2Î0
2 2A
and Q2 = (C – C1)V = ´V
3Î0 d
\ (a) is incorrect
1 4Q Q1 k
Now E1 (r0/2) = \ = \ (c) is incorrect
4p Î0 r02 Q2 2
Also V = E × d
1 4 ´ 2lr0 1 8l
= ´ = V
4p Î0 2
r0 4 p Î0 r0 \ E= = E1 = E2 \ (a) is a correct option
d
é 1 4l ù 1 8l 28 (c) Force on change q when it is given a small displacement
and 2E2 (r0/2) = 2 ê ú= x is Fnet = F1 – F2
ë 4pÎ0 r0 û 4 pÎ0 r0 l l
\ (c) is correct 1 l 1 l
Fnet = 2pe d – x – 2pe d + x
1 2l 1 4l l 0 0
E2 (r0/2) = = = F1 +q F2
4p Î0 r0 / 2 4p Î0 r0 pÎ0 r0 l éd + x – d + x ù x
\ Fnet = 2pe ê 2 2 ú
4s 2s 2 l 0 ë d -x û x d–x
4E3 (r0/2) = = = ´
2 Î0 Î0 Î0 pr0 l 2x d

\ (d) is incorrect. \ Fnet = 2pe 2 2


0 d –x
2d
line charge 1 line charge 2
When x << d then
Electrostatics P-S- 157

l 2e 0 s 4 e 0 s
Fnet = pe x and is directed towards the mean position ´
0 C1 ´ C2
+ C3 = d d + e0 s
Ceq = 6e 0 s
therefore the charge +q will execute SHM. C1 + C2 d
l l d

4 e 0 s e0 s
= +
–q 3 d d
F2
F1 7 e0 s 7 é e0 s ù
\ Ceq = = C1 êQ C1 = d ú
3 d 3 ë û

E. Subjective Problems
line charge 1 line charge 2
In case of charge (–q) 1. (i) The force on charge q kept at A due to charges kept at
B and C
F2 > F1 therefore the charge –q continues to move in the F1 = 2F cos 30°
direction of its displacement.
[C] is the correct option. æ q2 ö
F1 = 3 ´ ç 9 ´ 109 2 ÷
29. (d) Assume the cavity to contain similar charge distribution è a ø
of positive and negative charge as the rest of sphere.
The force on q due to charge (– q) kept at D
Electric field at M due to uniformly distributed charge
of the whole sphere of radius R1 q2 9 æ q2 ö
F2 = 9 × 109 = ´ ç 9 ´ 109 ´ ÷
ur r r M
(2a / 3) 2 4 è a2 ø
E= r P
r
3e Clearly the two forces are not equal. Also as F2 > F1 the
a
Electric field at M due the charges will move towards the centre.
negative charge distribution O
A q
in the cavity
2a 2F cos 30°
ur r uuur 3
E2 = MP a a F F
D 30° 30°
3e
Q
\ The total electric field at M is Aq
ur ur ur r r r uuur B
q a q
C
E = E1 + E 2 = r+ MP
3e 3e (ii) For charges to remain stationary
ur r r r r r r uuur r
\ E= r+
3e 3e
( ë )
a – r éQ r + MP = a ù
û 2× K
3 q2
=
9
´´K´
q2Q
Þ
4 3q
=Q
ur r r a 2 2 4 a 2 9
\ E= a The charge Q should be negative.
3e
(d) is the correct option The potential energy of the system is
30. (d) é q2 q2 ù é 4 3 q´q ù
d/2 = 3 êK 2 + K 2 ú + 3 êK ´ ú
êë a a úû êë 9 (2a / 3) 2 úû

C1 C2 q2 q2
q2
e2 = 4 s/2 = 6K × +3 3K
= 3(2 + 3) K
e1 = 2

a2 a2 a2
This is the amount of work needed to move the charges to
+ C3 infinity.
e1 = 2
s/2 2. Because of equilibrium of charge q1
N1 = mg sin 60° + (T – F) sin a ... (i)
and (T – F) cos a = mg cos 60° ... (ii)
2 e0 s/2 4 e0 s/2
C1 = C2 =
d/2 d/2 N1
(T–F) cos a
a
T –F
60° mg sin 60°
mg cos 60°
[(T–F) sin a ]
mg
2 e0 s/2 e0s 30°
C3 = =
d d
EBD_7036
P-S- 158 Topic-wise Solved Papers - PHYSICS
Because of equilibrium of charge q2 Potential at O due to inner sphere
(T – F) sin a = mg cos 30° ... (iii) æ Qr 2 ö
and N2 = (T – F) cos a + mg sin 30° ... (iv) q =K
Vi = K ç ÷
r r è R2 + r2 ø
Qr
Vi = K 2 2
R +r
Potential at O due to outer sphere
é 2 ù
(Q - q) = K êQ - Qr ú
V0 = K
R R êë R 2 + r 2 úû

é 2 2 2ù
K ëQR + Qr - Qr û K ( QR )
2 KQR

) ( )
From (i) and (iii) =
N1 = mg sin 60° + mg cos 30°
=
R 2 2
(R + r ) R R2 + r 2
= R2 + r 2
(
æ 3 3ö The total potential at the common centre
= mg ç + = 3 mg
è 2 2 ÷ø KQr KQR KQ( R + r )
V = Vi + V0 = + =
From (ii) and (iv) R +r 2 2
R +r 2 R2 + r 2
2

4. KEY CONCEPT : The electric field due a uniformly charged


æ 1 1ö ring of radius r at a point distant x from its center on its axis
N2 = mg cos 60° + mg sin 30° = mg ç + ÷ = mg
è 2 2ø is given by
qq Qx
Also, F = k 1 22 E= k
l ( r + x 2 )3 / 2
2
Now from eqn. (ii) and (iii), we get
(T – F)2 cos2 a + (T – F)2 sin2 a r = 1m
= m2g2 cos2 60° + m2g2 cos2 30° Q = 10–5 C
é1 3ù mass of particle m = 0.9 × 10–3 kg
Þ (T – F)2 = m2g2 ê + ú = m2g2
ë4 4û charge on particle q = – 10–6 C
Þ T – F = ± mg ... (v) \ Force on the negative charge q will be F = qE
q1q2
Þ T = mg + F = mg + k ... (vi) - kQq -k Q q
l2 \ F= 2 2 3/ 2
´x or, mA = ´x
[Taking positive sign] (r + x ) ( r + x 2 )3 / 2
2

From (ii) and (v)


Qq
mg cos a = mg cos 60° Þ cos a = cos 60° or, A = – k ´x
\ a = 60° m ( r + x 2 )3 / 2
2

when the string is cut, T = 0


\ From (vi) kQq
For x << r A = - ´x
2 r3
qq mg l
mg = ± k 1 22 Þ q1q2 = ± Þ The motion is simple harmonic in nature.
l k
Now the charges should be unlike for equilibrium. Comparing the above equation with A = -w 2 x we get
3. Let q be the charge on the inner sphere and (Q – q) be the kQq kQq
charge on outer sphere. \ w2 = or w =
mr 3
mr 3
Given that surface charge densities are equal.
2p kQq mr 3
q Q-q æ qö \ = Þ T = 2p
\ = çè Surface charge density,s = ÷ø T mr 3 kQq
4pr 2
4pR 2 A
or, qR2 = (Q – q) r2 or, qR2 = Qr2 – qr2 1/ 2
é 0.9 ´ 10-3 ´ 13 ù
T = 2 × 3.14 ê 9 -5 -6
ú
Qr 2 ëê 9 ´ 10 ´ 10 ´ 10 ûú
\ q=
R2 + r 2 = 6.28 [0.01]1/2 = 6.28 [0.1]
T = 0.628 sec
Electrostatics P-S- 159

5. The potential difference across each capacitor is V.


Total Energy = Energy in A + Energy in B
1 1
= CV 2 + CV 2 = CV2
2 2
When the switch opened and a dielectric is inserted between
the plates of capacitors, the new capacitance is 3C.
1 2 3
Energy in A = (3C )V = CV 2 (V is the same)
2 2
1 q2 1 (CV ) 2
Energy in B = = ´ Let T make an angle q with the vertical
2 KC 2 3C
2
CV 2 OC = (0.03)2 - (0.015)2 = 0.0173 m
= (charge on capacitor B remains same when 3
6 \ OM = 0.9997
switch is opened) NOTE THIS STEP : Resolving T in the direction of mg and
Total Energy = Energy in A + Energy in B F and applying the condition of equilibrium, we get
3 2 1 2 5 2 T cos q = mg; T sin q = F
\ Total Energy = CV + CV = CV ... (ii)
2 6 3 F
\ tan q = ... (i)
Total Energy initially CV 2
3 mg
= =
Total energy finally 5 5
CV 2 F= 2
FCA 2
+ FCB + 2 FCA FCB cos a
3
6. Total energy of the system of three charges when the charge 2 2 2 1
–q is at C \ F= FCA + FCA + 2 FCA ´
2
= P.E. + K.E.
kq 2
é Kq ´ q K (q)( - q) Kq (- q) ù F= 3FCA = 3 ´ ... (ii)
=ê + + +4 ... (i) (CA) 2
ë 6 5 5 úû
[where FCB = Force on C due to B
FCA = Force on C due to A
ur ur
| F CB | = | F CA | and a = 60° ]
OC 0.0173
Also, tan q = = .... (iii)
OM 0.9997
From (i), (ii) and (iii)
Final energy of the system of three charges when – q is at D 0.0173 3 ´ 9 ´ 109 ´ q 2
and momentarily at rest =
0.9997 (0.03) 2 ´ 10 -3 ´ 9.8
= P.E. + K.E.
On solving, we get q = 3.16 × 10–9 C.
é Kq ´ q Kq (- q ) Kq ( - q ) ù 8. Time for the dipole to align along the direction of electric
=ê + + ú +0
êë 6 x 2 + 32 x 2 + 32 úû field will be
Kq ´ q 2Kq(- q) T 2p ML p ML
= + ... (ii) t= = = .
6 x 2 + 32 4 4 2qE 2 2qE
By the principle of conservation of energy from (i) and (ii), 9. Charge on Shell A = qA = s (4pa2)
we get Charge on Shell B = qB = s (4pb2)
kq ´ q 2kq (- q ) kq ´ q 2kq (- q ) Charge on Shell C = qC = s (4pc2)
+ +4= + The potential of shell A
6 5 6 x 2 + 32
kq A kqB kqC
é1 ù VA = + +
1 a b c
2 = kq2 ê - ú
êë 5 x 2 + 32 úû k s (4 pa 2 ) k ( -s ) (4 pb ) k s (4 pc 2 )
2
\ x2 + 9 = 81 \ x = 8.48 m = + +
a b c
7. Each mass will be in equilibrium under the act of three force
namely tension of string, weight, resultant electrostatic force 1 4 pa 2 1 (4pb 2 ) 1 (4 pc 2 )
= ´s´ - s + ´s
of the two other charges out of these three forces F and mg 4pe0 a 4pe 0 b 4pe 0 c
are perpendicular.
EBD_7036
P-S- 160 Topic-wise Solved Papers - PHYSICS

s kq A kqB kqC 1
= [a - b + c] Similarly,, VB = + + • V (x) varies in general.
e0 b b c x

kq A kq B kqC V
and VC = + +
c c c
sé a2 ù s é a2 - b2 + c 2 ù
VB = ê - b + c ú and VC = ê ú –3a
e0
êë b úû e 0 êë c úû x
a 3a 9a
Given that VA = VC

s s é a 2 - b2 + c 2 ù
(a - b + c ) = ê ú
e0 e 0 ëê c ûú (c) Applying Energy Conservation
or ac – bc + c2 = a2 – b2 + c2 or c = a + b (K.E. + P.E.)centre = (K.E. + P.E.)circumference
10. (a) Let P be a point in the X-Y plane with coordinates (x, y)
at which the potential due to charges – 2Q and + Q é Qq 2Qq ù 1 2 é Qq 2Qq ù
0+K ê - ú = mv + K ê -
placed at A and B respectively be zero. ë 2a 8a û 2 ë 6a 12a úû
Y
P(x, y)
1 2 KQq KQq 1 æ Qq ö
y y mv = , v= = ç ÷
2 4a 2 ma 4 pe 0 è 2ma ø
V=0
A -2Q B a,0 5a ,0 9a,0x 11. (a) Let us consider a shell of the thickness dx at a distance
X' (-3a,0) O x +Q C X 0 4a x from the centre of a sphere
(3a,0) (5a,0)
(3a +x)
(3a-x)
Y'
K (2Q ) K (+Q)
\ =
(3a + x) 2 + y 2 (3a - x) 2 + y 2

Þ 2 (3a - x)2 + y 2 = (3a + x) 2 + y 2


Þ (x – 5a)2 + (y – 0)2 = (4a)2
This is the equation of a circle with centre at (5a, 0) and
radius 4a. Thus C (5a, 0) is the centre of the circle. 4 é 4 ù
(b) For x > 3a The vol. of the shell = p ê( x + dx )3 - p x 3 ú
3 ë 3 û
To find V (x) at any point on X-axis, let us consider a point
(arbitrary) M at a distance x from the origin. 4 é
= p ( x + dx)3 - x 3 ù
3 ë û

4 3 é æ dx ö ù
3
= p x ê çè 1 + ÷ø - 1ú
3 ëê x ûú
The potential at M will be
K ( -2Q) K (+Q) 1 4 3 é 3dx ù
V (x) = + where k = = p x ê1 + - 1ú
x + 3a ( x - 3a) 4pe0 3 ë x û
4 3 3dx
é 1 2 ù =
px ´ = 4px2dx
\ V (x) = KQ ê - for | x | > 3a 3 x
ë x - 3a x + 3a úû
Let r be the charge per unit volume of the sphere
Similarly, for 0 < | x | < 3a \ Charge of the shell = dq = 4px2rdx ... (i)
é 1 2 ù Potential at the surface of the sphere of radius x
V (x) = KQ ê -
ë 3a - x 3a + x úû 4
r ´ px 3 é qù
1
êëQ V = k r úû
Since circle of zero potential cuts the x-axis at (a,0) and = ´ 3
(9a, 0) 4pe 0 x
Hence, V (x) = 0 at x = a, at x = 9a \ Potential at the surface of the sphere of radius x =
• From the above expressions
V (x) ® ¥ at x ® 3a and V (x) ® – ¥ at x ® – 3a rx 2
• V (x) ® 0 as x ® ± ¥ 3e 0
Electrostatics P-S- 161

Work done in bringing the charge dq on the sphere of radius x The combined capacitance is

rx 2 rx 2 +
dW = × dq Þ dW = × 4p x2 r dx 110V + + + +
3e 0 3e 0
Therefore the work done in accumulating the charge Q over A - - - +
A/2 A/2
a spherical volume of radius R meters
-
R
R 4pr2 4pr2 é x5 ù 4pr2 R5 C = C1 + C2
ò0
4
W= x dx = ê ú =
3e 0 3e 0 êë 5 úû 0 3e 0 5 ( A / 2) e0 ( A / 2) e 0 er A e 0
= + = [1 + e r ]
d d 2 d
2
4 p æ Q ö R5 3Q 2
=
3e 0 çè 4 / 3pR3 ÷ø 5
=
20 pe0 R
0.4 ´ 8.85 ´ 10-12
= [1 + 9] = 2 ×10–9 F
This is also the energy stored in the system. 2 ´ 8.85 ´ 10-4
(b) The above energy calculated is 1 1
2 2
\ Energy stored = CV 2 = × 2 × 10–9 × (110)2
3Q 3KQ 1 2 2
E= = where K = –5
= 1.21 × 10 J
5 ´ (4pe 0 ) R 5R 4pe 0
(ii) Work done in removing the dielectric slab = (Energy
NOTE : In case of earth and gravitational pull, K may be stored in capacitor without dielectric) – (Energy stored in
replaced by G. Therefore the energy required to disassemble
capacitor with dielectric).
the planet earth against the gravitational pull amongst its
NOTE : While taking out the dielectric, the charge on the
constituent particle is the work required to make earth from
its constituent particles. capacitor plate remains the same.

3GM 2 q2 q2
\ E= [ Q Q is replaced by M] \ W= - Here, C = 2 × 10–9 F,
5R 2C ' 2C

GM GM 2 Ae0 0.04 ´ 8.85 ´ 10 -14


But g = Þ gMR = C'= = = 0.4 × 10–9 F
R2 R d 8.85 ´ 10-4
Kq1q2 G m1m2 q = CV = 2 × 10–9 × 110 = 2.2 × 10–7 C
F= ;F=
r2 r2 (2.2 ´ 10-7 )2 é 1 1 ù
\ W= ê -9
- - 9 ú
3 3 2 ë 0.4 ´ 10 2 ´ 10 û
\ E= gMR = ´ 10 ´ 2.5 ´ 1031 = 1.5 ´ 1032 J
5 5 = 4.84 × 10–5 J
(c) During the charging process, let at any instant the
spherical conductor has a charge q on its surface. e 0 e r AB
(iii) The capacitance of B =
d
1 q
The potential at the surface = ´
4pe 0 R 8.85 ´ 10-12 ´ 9 ´ 0.02
dq =
q 8.85 ´ 10-4
Small amount of work done in
increasing charge dq more on CB = 1.8 × 10–9 F
the surface will be The charge on A, qA = 2.2 × 10–7 C gets distributed into two
parts.
1 q R
dW = ´ ´ dq \ q1 + q2 = 2.2 × 10–7 C
4pe 0 R also the potential difference across A = p.d. across B
\ Total amount of work done in bringing charge Q on the
q1 q
surface of spherical conductor. = 2
C A CB
Q
1 Q 1 é q2 ù Q2
4pe 0 R ò0
W= qdq = ê ú =
4pe 0 R ëê 2 ûú (8pe 0 R) CA 0.4 ´ 10-9
0 Þ q1 = q2 = q = 0.22 q2
12. (i) NOTE : The capacitor A with dielectric slab can be
CB 1.8 ´ 10-9 2
considered as two capacitors in parallel, one having dielectric \ 0.22 q2 + q2 = 2.2 × 10–7
slab and one not having dielectric slab. Each capacitor has 2.2
Þ q2 = ´ 10-7 = 1.8 × 10–7 C
A 1.22
an area of .
2 Þ q1 = 0.4 × 10–7 C
EBD_7036
P-S- 162 Topic-wise Solved Papers - PHYSICS
Differentiating the above quation w.r.t. time
q2 q2
Total energy stored = 1 + 2 dC e 0 dx e
2C A 2C B = (k - 1) = 0 (k - 1)v
dt d dt d

0.4 ´ 0.4 ´ 10-14 1.8 ´ 1.8 ´ 10-14 dx


= + where v=
2 ´ 0.4 ´ 10-9 2 ´ 1.8 ´ 10-8 dt
= 0.2 × 10–5 + 0.9 × 10–5 = 1.1 × 10–5 J dq dC
We know that q = CV, =V
13. Potential energy can be found at the initial point A and final dt dt
point O. The difference in potential energy has to be From (iii) and (iv)
provided by the K.E. of the charge at A.
e0
I= V (k - 1)v
d

500 ´ 8.85 ´ 1012


I= (11 – 1) × 0.001
0.01
= 4.425 × 10–9 Amp.
15. Case (i) When no dielectric :
e0 A
Given C=
d
Case (ii) When dielectric is filled : A small dotted element
of thickness dx is considered as shown in the figure.
1 Q
V ( x) = . , at A.
4pe 0 R2 + x 2

1 2 pR l
VO = . , at O. or V = l
4pe 0 R O
2e 0
1 2 pR l l
VP = =
4 pe0 4e 0
R2 + ( 3R ) 2

Potential difference between points O and P = V


\ V = VO - VP The small capacitance of the dotted portion
1 1 1
l l l = + where dC1 = capacitance of capacitor
or V= - or V = dC dC1 dC2
2 e 0 4 e0 4e 0
with dielectric K1
The kinetic energy of the charged particle is converted into
its potential energy at O. dC2 = capacitance of capacitor with dielectric K2.
\ Potential energy of charge (q) = qV Let l,b the length and breadth of the capacitor plate.
1 2 Therefore l ´ b = A .
Kinetic energy of charged particle = mv
2 K1 (bdx ) e 0
For minimum speed of particle so that it does not return to P, dC1 =
d'
1 2 2qV 2q ´ l
mv = qV or v 2 = = d é xù
= d ê1 - ú
2 m m ´ 4e 0 d' = d – x
l ë lû

or v =
ql K1b ( dx) e 0 K1bl ( dx) e0 K A e ( dx)
2 e0 m \ dC1 = - = 1 0
é xù d (l - x ) d (l - x )
d ê1 - ú
14. The adjacent figure is a case of parallel plate capacitor. The ë lû
combined capacitance will be
C = C1 + C2 K 2 e 0(bdx ) K e bdx
Similarly, dC2 = = 2 0
k e 0 ( x ´ 1) e0 [(1 - x ) ´ 1) d -d' xd
d -d +
= + l
d d
K 2 e 0b.l.dx K 2 e 0Adx
e0 =
C= [kx + 1 - x] ... (i) xd xd
d
Electrostatics P-S- 163

Amount of electrostatic energy stored finally


1 d (l - x ) xd
\ = +
dC K1 Ae 0 ( dx ) K 2 Ae 0 ( dx ) 1 q12 1 q22 1 q32
Uf = + +
2 C A 2 CB 2 CC
K1K 2 A e 0dx
Þ = dC
K 2 ld + d ( K1 - K 2 ) x 1 (90 ´ 10 -6 )2 1 (210 ´ 10 -6 ) 2 1 (150 ´ 10-6 ) 2
= + +
To find the capacitance of the whole capacitor, we integrate 2 3 ´ 10-6 2 2 ´ 10-6 2 2 ´ 10 -6
the above equation.
1 (150 ´ 10-6 )2
l K1 K 2 Ae 0 dx + = 1.8 × 10–2 J
2 2 ´ 10-6
C= ò0 K 2 ld + d ( K1 - K 2 ) x
17. Limiting value of energy as n ® ¥.
Let us calculate qn when n tends to ¥.
l dx
= K1K2Ae0 ò0 K2 ld + d ( K1 - K2 ) x For G.P., S¥ =
a
where r1 = common ratio
1 - r1
l
= K1K2Ae0 ê log[ K 2 ld + d ( K1 - K 2 ) x] ú
é ù QR é 1 ù QR
d ( K1 - K 2 ) q¥ = ê ú or q¥ =
ë û0 \ R+r R r
ê1 - ú
ë R+r û
K1 K2 Ae 0 K
C= log 1 2 2
d ( K1 - K 2 ) K2 q¥ æ QR ö 1
\ U¥ = =ç ÷ ´
16. (i) KEY CONCEPT : Use charge conservation to solve 2C è r ø 2 ´ ( 4pe 0 ) ´ ( R )
this problem.
INITIALLY : Q2 R 2 Q2 R
or U¥ = or U ¥ =
Charge on capacitor A r 2 ´ 2 ´ 4pe0 R 2 ( 4pe0 ) r 2
qA = 3 × 10–6 × 100 = 3 × 10–4C A B
+ + 18. (a) KEY CONCEPT : The K.E. of the particle, when it
Charge on capacitor B + +
+ + reaches the disc is zero.
qB = 2 × 10–6 × 180 = 3.6 × 10–4C 3mF 2mF Given that a = radius of disc, s = surface charge density,
FINALLY : q1 q2
q/m = 4e0g/s
Let the charge on capacitor A, 100V 180V
Potential due to a charged disc at any axial point situated at
C and B be q 1 , q 2 and q 3 a distance x from O is,
respectively. q2 C(2mF)
s
By charge conservation. + V (x) = [ a 2 + x2 - x]
The sum of charge on +ve plate + 2 e0
+
of capacitor A and C should be
s
equal to qA Hence, V (H) = [ a2 + H 2 - H ]
\ q1 + q2 = 3 × 10–4C... (i) 2e 0
A B
Similarly the sum of charge on + +
+ + sa
–ve plates of capacitor C and and V(O) =
+ + 2e 0
B will be equal to qB 3mF 2mF NOTE : According to law of conservation of energy,
\ – q2 – q3 = – 3.6 × 10–4C q q3
1 loss of gravitational potential energy = gain in electric
Þ q2 + q3 = 3.6 × 10–4C ... (ii)
potential energy
Applying Kirchoff's law in the closed loop, we get
mgH = qDV
q1 q2 q3 = q [V(0) – V (H)]
-6
- -6
+ =0
3 ´ 10 2 ´ 10 2 ´ 10-6 s 2 2
mgH = q [a – { (a + H ) - H }] ... (1)
Þ 2q1 – 3q2 + 3q3 = 0 ....(iii) 2e 0
On solving (i), (ii) and (iii), we get
q1 = 90 × 10–6 C, q2 = 210 × 10–6 C, sq
and q3 = 150 × 10–6 C, From the given relation : = 2mg
2e 0
(ii) Amount of electrostatic energy in the system initially
Putting this in equation (1), we get,
1 2 1
U i = U A + U B = C A (VA ) + C B (VB )
2
2 2 mgH = 2mg [a - { (a 2 + H 2 ) - H }]
1 1 4a
=
2
× 3 × 10–6 (100)2 + × 2 × 10–6 (180)2
2 or H= [Q H = O is not valid]
3
–2
= 4.74 × 10 J
EBD_7036
P-S- 164 Topic-wise Solved Papers - PHYSICS
(b) Total potential energy of the particle at height H
U (x) = mgx + qV (x) 5
Þ x=±
2
qs
= mgx + ( a 2 + x2 - x) This means that we need to move the charge from – ¥ to
2e 0
5
2
= mgx + 2mg [ (a + x ) - x ] 2
....(2) . Thereafter the attractive forces will make the charge
2
move to origin.
U( A) = mgH + 2mg é a 2 + H 2 - H ù
ëê ûú 5
The electric potential of the four charges at x = is
= mg é 2 a2 + H 2 - H 2 ù ....(3)
2
êë úû
dU 2 ´ 9 ´ 109 ´ 8 ´ 10 -6 2 ´ 9 ´ 109 ´ 10 -6
For equilibrium : =0 V= -
dH 5 27 5 3
+ +
a 2 2 2 2
This gives : H = \ U min = 3 mga
3
é8 1ù 4
From equation (2), graph between U(x) and x is as shown = 2 × 9 × 109 × 10–6 ê - ú = 2.7 ´ 10 V
above. ë 4 2û
Kinetic energy is required to overcome the force of repulsion
5
from µ to x = .
2
The work done in this process is W = q (V)
5
where V = p.d between ¥ and x = .
2
\ W = 0.1 × 10–6 × 2.7 × 104 = 2.7 × 10–3 J
1
By energy conservation mV02 = 2.7 × 10–3
19. Let the particle at some instant be at a point P distant x from 2
the origin. As shown in the figure, there are two forces of
repulsion acting due to two charges of + 8 mC. The net force 1
Þ ´ 6 ´ 10-4 V02 = 2.7 ´ 10-3
is 2F cos a towards right. 2
Similarly there are two forces of attraction due to two charges Þ V0 = 3 m/s
of – 1 mC. The net force due to these force is 2F' cos b
K.E. at the origin
towards left.
Potential at origin
2 ´ 9 ´ 109 ´ 8 ´ 10 -6 2 ´ 9 ´ 109 ´ 10-6
Vx = 0 = -
27 3
2 2
= 2.4 × 104
Again by energy conservation

K.E. = q éVx= 5 - Vx = 0 ù
êë 2
úû
\ K.E. = 0.1 × 10–6 [2.7 × 104 – 2.4 × 104]
The net force on charge 0.1 µC is zero when = 0.1 × 10–6 × 0.3 × 104
2F cos a = 2F' cos b = 3 × 10–4 J
K ´ 8 ´ 10 -6 ´ 0.1 ´ 10-6 x 1 q 2 é -3 3 1 ù
´ 20. Wexternal = DPE = + - ú´4
æ 2 27 ö
2 27 4pe 0 a êë 1 2 3û
x2 +
ç x + 2 ÷ 2
è ø 1 q2 4
= . [3 3 - 3 6 - 2]
4pe 0 a 6
K ´ 1 ´ 10-6 ´ 0.1 ´ 10 -6 x
= ´ 21. (a) Potential energy of the dipole-charge system
2 3
æ 2 3ö x2 + Ui = 0 (since the charge is far away)
ç x + 2÷ 2
è ø
Electrostatics P-S- 165

1 p 1 4pe 0V ´ a
Uf = – Q × [at a point (d, w) or, Vdroplet = ´ [From (i) and (ii)]
4pe 0 d 2 4pe 0 [3a 2t ]1/ 3
1 pQ 1/ 3
\ K.E. = | Uf – Ui | = éaù
4pe 0 d 2 or, Vdroplet = V ê ú
ë 3t û
(b) Electric field at origin due to dipole
ur 1 2p $ F. Match the Following
E= i
4pe 0 d 3 1. + – Q
Thus, force on charge Q is given by
ur ur 2 pQ $ – +
F = QE = i M
4pe 0 d 3
P+ –
s
22. Electric field due to S1, E1 = 1 The electric field at M due to the charges at the corners of
e0 regular hexagon is as shown
s2 E–
Electric field due to S2, E2 = E+
e0 120°
\ E = E1 – E2 E– 120°
E+ 120° E+
s1 - s 2
=
e0
(Q s1 > s2 ) E–
Work done by electric field Here | E+ | = | E- | . The symmetry of the situation shows
a that E = 0 at M.
W = ( q0 E ) a cos 45o = q0 E × Therefore (A) is the correct option.
2
The electric potential due to all the charges at M is zero.
q0 (s1 - s 2 ) a Therefore (B) is incorrect option.
\ W=
2 e0 When the system of charges is rotated about line PM, the
23. LIQUID BUBBLE : The potential of the liquid bubble is V. net current will be zero.
Therefore the magnetic field at M is zero.
1 q
Þ V= ... (1) (C) is the correct option.
4pe 0 a When magnetic field is zero, then m = 0
where q is the charge on the liquid bubble. (D) is incorrect option.
(q) P

– + – + – +
M
Q
The electric field due to the inner most positive and negative
LIQUID DROPLET é qù
The volume of liquid droplet = Volume (of the liquid) charges at M is E1 = 2 ê k ú towards left. The electic field
in liquid bubble. ë r2 û
due to the next positive and negative charges at M is
4 3 4 4
p r = p ( a + t )3 - p a 3 é
3 3 3 q ù
E2 = 2 ê k towards right. The electric field due to
or, r3 = a3 + t3 + 3a2t + 3at2 – a3 2ú
or, r3 = 3a2t ëê (2r ) ûú
(Q t is very small as compared to a) the outermost positive and negative charges at M is
or, r = [3a2t]1/3 ... (iii) é q ù
E3 = 2 ê k ú towards left. Clearly the vector sum of
êë (3r )2 úû
NOTE : By charge conservation we can conclude that
charge on liquid bubble is equal to charge on liquid droplet these three electric field is not zero.
Let charge on liquid droplet is q. (A) is incorrect option.
\ Potential on liquid droplet The electric potential due to the charges at M
1 q é +q -q q q q q ù
Vdroplet = =kê + - + - =0
4pe 0 r ë r r 2r 2r 3r 3r úû
EBD_7036
P-S- 166 Topic-wise Solved Papers - PHYSICS
(B) is incorrect option. When the system of charges rotates about PQ, the net
The net current due to the innermost positive and negative current is zero due to symmetrical arrangement of charges.
charges is zero. Similarly the net current due to other charges Therefore B = 0 and m = 0
in pairs is zero. Therefore the magnetic field at M is zero. (C) is the correct option.
Also the magnetic moment is zero. (D) is the incorrect option.
(C) is the correct option 18. (a) If Q1, Q2, Q3 and Q4 are all positive, then the force will
(D) is incorrect option.
be along + y-direction.
(r) + Q
The net electric field due to negative – If Q1, Q2 are positive and Q3, Q4 are negative the force
charges in the inner circle is zero. will act along + x-direction.

Similarly the net electric field due to M – + If Q1, Q4 are positive and Q2, Q3 are negative then
positive charges in the outercircle is attractive force will dominate repulsive force and the
zero. P
+ force will be along – y direction.
(A) is the correct option.
The electric potential due to negative charges at M is
different from the electric potential due to positive charges
G. Comprehension Based Questions
at M. Therefore the electric potential at M is not equal to 1. (a) When the point of observation is on the surface of
zero. sphere then the whole charge inside the sphere (when
(B) is the correct option. distributed symmetrically about the centre) behaves as
When the system of charges rotate, we get a current I1 due a point charge on the centre. Therefore until the charge
to negative charges and another current I due to positive distribution is symmetrical about the centre it does not
charges. The magnitude of the magnetic at M due to the matter what is the ratio a/R. The electric field remains
currents is different. Therefore B ¹ 0 and m ¹ 0. constant and is equal to
(C) is incorrect option 1 Ze
(D) is the correct option. E= . .
4π Îo R
(s) 2a
– + – P
a R
M a
P Q
– + –
The electric field at M due to all the charges is zero because
the electric field due to different charges cancel out in pairs.
(A) is the correct option.
The potential at M due to the charges is 2. (b) For a = 0, the graph is as shown. The equation for the
é +q q æ q öù graph line is
V =kê + -4 ¹0
a/2 a/2 ç 5a ÷ ú r
ê ç ÷ú r = d– dr
êë è 4 ø úû R
(B) is the correct option. r(r)
When the whole system is set into rotation with a constant
angular velocity about the line PQ we get three loops in d
which current is flowing. r dr
The magnetic field due to these currents produce a resultant
magneic field at M which is not equal to zero. Therefore a
net magnetic dipole moment will be produced.
(C) is an incorrect option. r Fig (2)
O R
(D) is correct option. Fig (1)
P The charge in the dotted element shown in Fig (2) is
+ – dq = r × 4pr2dr
(t) + + R R
M– – æ ö d 4πd 3
\ dq = çè d - r÷ø 4pr2dr Þ Ze = ò 4πdr dr – ò
2
r dr
R R
Q o o
There will be a net electric field due to the arrangement of
R 3 4πd R 4
charges at M towards the right side. Ze = 4πd -
(A) is an incorrect option. 3 R 4
The electic potential at M will can out in pairs by positive Ze. 1 1 1 3Ze
and negative charges, due to symmetrical arrangement of \ – =
3
= \d=
4πdR 3 4 12 πR 3
charges.
(B) is an incorrect option. 3. (c) If the volume charge density is constant then E µ r.
Electrostatics P-S- 167

4. (c) After colliding the top plate, the ball will gain negative kq
2
2
charge and get repelled by the top plate and bounce kq
2
back to the bottom plate. 2a a
2

Q kq
2
5. (d) Iav µ ...(i)
t 2
Here Q µ V0 ...(ii) a
1 2
Also S = ut + at
2
1 QE 2 1 æ Q ´ 2V0 ö 2
h= t = ç ÷´ t As the system of charges and planar film is in equlibrium,
2 m 2 è mh ø therefore
1
\ tµ – (iii) [Q Q µ V0] q2
V0 ´ constant = ga ´ constant
From (i), (ii) and (iii) a2
V0 1/ 3
æ q2 ö
Iav µ 1 = Iav µ V02 \ a =kç ÷ \ N =3
V0 ç g ÷
è ø
3. 6 We suppose that the cavity is filled up by a positive as
H. Assertion & Reason Type Questions well as negative volume charge of r. So the electric
1. (a) Both the statements are true and statement-2 is the field now produced at P is the superposition of two
correct explanation of statement-1 electric fields.
(a) The electric field created due to the infinitely long
I. Integer Value Correct Type solid cylinder is
1. 2 Let us consider a spherical rR
E1 = directed towards the +Y direction
shell of radius x and thickness 4e 0
dx. The volume of this shell dx
(b) The electric field created due to the spherical
x
is 4px 2 (dx). The charge negative charge density
enclosed in this spherical
rR
shell is E2 =
96 e 0 directed towards the –Y direction.
dq = (4 p x 2 ) dx ´ kx a
\ The net electric field is
\ dq = 4pkx 2 + a dx .
1 é 23rR ù
For r = R : E = E1 - E2 = ê ú
The total charge enclosed in the sphere of radius R is 6 ë 16e 0 û
R 4. (6)
R 3+ a cylinder
ò0
2+ a
Q = 4 pk x dx = 4pk
3+ a .
\ The electric field at r = R is
1 4pkR 3+ a 1 4pk 1+ a line charge
E1 = = R +
4pe 0 (3 + a ) R 2 4 pe 0 3 + a
For r = R/2 : q q
The total charge enclosed in the sphere of radius R/2 is 3a
R/2 3+ a 2
4 pk ( R / 2)
Q' = ò0 4pk x 2 + a dx =
3+ a
a/2 a/2

a/2 1
\ The electric field at r = R/2 is tanq = =
1+ a
3a / 2 3
1 4pk ( R / 2)3+ a 1 4 pk æ R ö \ q = 30°
E2 = = ç ÷
4 pe 0 3 + a ( R / 2) 2 4pe 0 3 + a è 2 ø The flux through the dotted cylinder by Gauss’s law is
1 q in lL
Given, E2 = E1 f cylinder = =
8 e0 e0
1+ a
1 4 pk æ R ö 1 1 4 pk 1+ a
\ ç ÷ = ´ R lL
4pe0 (3 + a ) è 2 ø 2 3 4 pe 3 + a
0 \ For 360° angle the flux is e
Þ 1+a=3 Þ a=2 0
2. 3 lL
\ For 60° angle the flux will be
kq 2 é kq 2 1 ù q 2 6e0
Felectric = + 2ê ´ ú = ´ constant Therefore n = 6
2a 2 ëê a
2 2 ûú a 2
EBD_7036
P-S- 168 Topic-wise Solved Papers - PHYSICS

Section-B JEE Main/ AIEEE


\ The net electric potential at point P is
WAB
1. (a) We know that = VB - V A 1 2Q 1 q
q V = V1 + V2 = +
4pe o R 4pe o R
2J
\ VB - V A = = 0.1J/C = 0.1V 9. (d) The work done is stored as the potential energy. The
20 C potential energy stored in a capacitor is given by
2. (b) The equivalent capacitance of n identical capacitors of
( )
-18 2
capacitance C is equal to nC. Energy stored in this 1 Q 2 1 8 ´ 10
capacitor U= = ´ = 32 ´ 10 –32 J
2 C 2 100 ´ 10-6
1 1
E= (nC )V 2 = nCV 2 qq
2 2 10. (b) Force on charge q1 due to q2 is F12 = k 1 2
3. (b) Both the charges are identical and placed symmetrically b2
about ABCD. The flux crossing ABCD due to each
q1q3
1é q ù Force on charge q1 due to q3 is F13 = k
a2
charge is 6 ê Î ú but in opposite directions. Therefore
ë 0û The X - component of the force (Fx) on
the resultant is zero. q1 is F12 + F13 sin q
4. (d) For equilibrium of charge Q
q1q2 q1q2
Q´Q Qq \ Fx = k +k sin q F12
K +K =0 Þ q =-Q b 2
a2
2 2
(2x) x 4 F13 sin q
x x q2 q3 q
\ Fx µ + sin q
Q q Q b2 a 2
F13cos q F13
5. (a) For an isolated sphere, the capacitance is given by 11. (d) Rf = n2Ri
Here n = 2 (length becomes twice)
1 -10 \ Rf = 4Ri
C = 4p Î0 r = ´ 1 = 1.1 ´ 10 F
9 ´ 109 New reresistance = 400 of Ri
6. (a) The flux entering an enclosed surface is taken as \ Increase = 300%
negative and the flux leaving the surface is taken as QAQC
positive, by convention. Therefore the net flux leaving 12. (d) Fµ
x2
the enclosed surface = f2 - f1
x is distance between the spheres. After first operation
\ the charge enclosed in the surface by Gauss’s law is
Q
q =Î0 (f 2 - f1 ) charge on B is halved i.e and charge on third sphere
2
7. (b) The capacitance of a parallel plate capacitor in which a
metal plate of thickness t is inserted is given by Q
becomes . Now it is touched to C, charge then
e A e A 2
C = o . Here t ® 0 \ C = o equally distributes them selves to make potential same,
d -t d
hence charge on C becomes æç Q + ö÷ =
8. (c) Electric potential due to charge Q placed at the centre Q 1 3Q
.
of the spherical shell at point P is è 2ø 2 4
1 Q 1 2Q
V1 = = æ 3Q ö æ Q ö
4pe o R / 2 4pe o R çè ÷ø çè ÷ø
QC' QB' 4 2 3 Q2
\ Fnew µ = =
x2 x2 8 x2
R/2
Q P 3
or Fnew = F
8
R q
1 2 kQq 1 kqQ r
Electric potential due to charge q on the surface of the 13. (d) mv = Þ m(2v )2 = Þ r'=
2 r 2 r' 4
spherical shell at any point inside the shell is
14. (b) Net field at A should be zero
1 q
V2 = 2 E1 + E2 = E3
4pe o R
Electrostatics P-S- 169

kQ ´ 2 kQ kq 19. (c) P
\ 2
+ 2
= 2
a ( 2 a) æ a ö T cos q
çè ÷ q
2ø T
s
F = Eq = q
E3 –Q –Q B e0 K
E1 T sin q
A
E2
2 E1 mg
E A –Q –Q

Q 2 Q Q s
Þ + = 2q Þ q = (2 2 + 1) T sin q = .q .... (i)
1 2 4 e0 K
15. (c) At equilibrium, electric force on drop balances weight T cos q = mg .... (ii)
of drop.
Dividing (i) by (ii),
mg 9.9 ´ 10 -15 ´ 10
. qE = mg Þ q = = = 3.3 ´ 10-18 C sq
E 3 ´ 10 4 tan q = \ s µ tan q
e 0 K . mg
- K 2q K 8q 1 4 20. (c) Applying conservation of energy,
16. (b) + =0 Þ =
2 2 2 2
( x - L) x ( x - L) x 2m. s. DT
1
CV 2 = m. s DT ; V=
1 2 2 C
or = Þ x = 2 x - 2 L or x = 2L
x-L x
q q +q
F1
17. (a) 21. (c) E1
A B F2
R R E2
1 2 –q
d
The electric field will be different at the location of the
two charges. Therefore the two forces will be unequal.
VA = Vself + Vdue to (2)
This will result in a force as well as torque.

1 éq q ù 1 2
Þ VA = ê - ú 22. (a) eV = mv
4pe 0 2
êë R R 2 + d 2 úû
VB = Vself + Vdue to (1) 2eV 2 ´ 1.6 ´ 10-19 ´ 20
Þv= =
m 9.1 ´ 10 -31
1 é-q q ù
Þ VB = ê + ú
4pe 0 ëê R = 2.65 ´10 6 m / s
R 2 + d 2 ûú
23. (c) After connection, V1 = V2
DV = V A - VB +Q2
Q Q +Q1
éq q ù ÞK 1 =K 2
1 q q r1 r2
= ê + - - ú r1 r2
4pe 0 êë R R R2 + d 2 R2 + d 2 úû
Q Q
Þ 1= 2 A B
éq ù r1 r2
1 q
= ê - ú The ratio of electric fields
2pe 0 ëê R R 2 + d 2 ûú
Q1
18. (b) As n plates are joined, it means (n – 1) capacitor joined K
in parallel. E1 r12 Q1 r22
= = ´
\ resultant capacitance = (n – 1) C E2 Q 2 Q
K 2 r1 2
2
r2
EBD_7036
P-S- 170 Topic-wise Solved Papers - PHYSICS
27. (a) The potential energy of a charged capacitor before
E r ´ r2 E r 2
Þ 1 = 1 2 Þ 1 = 2 =
2
E2 r1 ´ r2 E2 r1 1 Q2
removing the dielectric slat is U = .
Since the distance between the spheres is large as 2C
compared to their diameters, the induced effects may The potential energy of the capacitor when the
be ignored. dielectric slat is first removed and the reinserted in the
24. (c) Y Q2
gap between the plates is U =
2C
A(Ö2,Ö2) There is no change in potential energy, therefore work
® done is zero.
r1
28. (b) Electronic charge does not depend on acceleration due
to gravity as it is a universal constant.
O X
(0,0) ®
r2 B (2,0)
So, electronic charge on earth
= electronic charge on moon
The distance of point A ( )
2, 2 from the origin, \ Required ratio = 1.
ur 2 2
OA = | r1 | = ( 2) + ( 2) = 4 = 2 units.
The distance of point B(2, 0) from the origin,
uur 29. (c)
2 2
OB = | r2 | = (2) + (0) = 2 units.

1 Q
Now, potential at A, VA = . The given capacitance is equal to two capacitances
4p Î0 (OA)
connected in series where
1 Q
Potential at B, VB = .
4p Î0 (OB)
\ Potential difference between the points A and B is and
zero.
25. (a) As shown in the figure, the resultant electric fields
before and after interchanging the charges will have
the same magnitude, but opposite directions. The equivalent capacitance Ceq is
Also, the potential will be same in both cases as it is a 1 1 1 d d 2d
scalar quantity. = + = + =
q q -q
Ceq C1 C2 9 Î0 A 9 Î0 A 9 Î0 A
-q
A B A ® B
E 9 Î0 A 9
\ Ceq = = ´ 9 pF = 40.5 pF
2 d 2
30. (a) The electric field inside a thin spherical shell of radius
® R has charge Q spread uniformly over its surface is
E
D C D C zero.
-q -q q q
Q +
20 ++ + +
+

26. (a) Here, V(x) = volt R


+

x2 - 4
+ + +
+++

Q
dV d æ 20 ö E=k
We know that E = - =- ç 2 r2
dx è x - 4 ÷ø
++

+ +

dx E=0
+

40 x + + +
++
or, E = + 2 2
( x - 4)
At x = 4 mm , Q
Outside the shell the electric field is E = k 2 . These
40 ´ 4 r
160 10
E= + 2 2 =+ = + volt / mm. characteristics are represented by graph (a).
(4 - 4) 144 9
r
Positive sign indicates that E is in +ve x-direction.
Electrostatics P-S- 171

Electric field at O due to dq is


WPQ
31. (c) = (VQ - VP ) 1 dq 1 q
q dE = . = . dq
4p Î0 r 2 4 p Î0 pr 2
Þ WPQ = q (VQ - VP ) The component dE cos q will be counter balanced by
= (– 100 × 1.6 × 10–19) (– 4 – 10) another element on left portion. Hence resultant field at O is
= +2.24 × 10–16J the resultant of the component dE sinq only.
32. (d) Let F be the force between Q and Q. The force between p
q
q and Q should be attractive for net force on Q to be \ E = ò dE sin q = ò 4 p2 r 2 Î sin qd q
zero. Let F ¢ be the force between Q and q . For 0 0
equilibrium Q q q
=
2 2
[ - cos q]p0 = q
(+1 + 1)
2 F ' = -F 4p r Î0 2 2
4p r Î0
R = 2F' q
Qq Q2 l F¢
2 ´ k 2 = -k 2
= 2 2
2p r Î0
l ( 2 l)
The direction of E is towards negative y-axis.
Q
= -2 2 Q r q
Þ q \ E=- 2 2 ˆj
q F¢ 2p r Î0
33. (a) Statement 1 is true. F
36. (a) Let us consider a spherical shell of radius x and
Statement 2 is true and is the correct explanation of (1)
thickness dx.
34. (b) Charge on this shell
R
æ5 x ö 2
dq = r.4px 2 dx = r0 ç 4 - R ÷ .4px dx
è ø
r dr \ Total charge in the spherical region from centre to r
(r < R ) is
r
æ5 xö
q = ò dq = 4 pr0 ò ç - ÷ x 2 dx
Let us consider a spherical shell of thickness dx and 0
è4 Rø

radius x. The volume of this spherical shell = 4pr 2 dr .


The charge enclosed within shell
dx
Qr
= 4
[4pr 2 dr ] x
pR
The charge enclosed in a sphere of radius r1 is
r1 r1
4Q 4Q é r 4 ù Q 4
ò
3
= r dr = ê ú = 4 r1
R4 4 4
R êë úû 0
R
0
é 5 r3 1 r 4 ù
- . ú = pr0r 3 æç - ö÷
\ The electric field at point p inside the sphere at a 5 r
= 4 pr0ê .
distance r1 from the centre of the sphere is ëê 4 3 R 4 ûú è3 Rø
é Q 4ù 1 q
r
1 êë R 4 1 úû = 1 Q r 2 \ Electric field at r, E = .
E= 1 4p Î0 r 2
4p Î0 r12 4p Î0 R 4
1 pr0 r 3 æ 5 r ö r r æ5 r ö
35. (c) Let us consider a differential element dl. charge on this = . 2 ç - ÷= 0 ç - ÷
element. j 4pÎ0 r è 3 R ø 4 Î0 è 3 R ø

æ q ö 37. (d) At any instant


+

dq = ç ÷ dl
+

dl T cosq = mg ....(i)
+

+
è pr ø + dq T sin q = Fe ....(ii)
+
q q
(rd q) (Qdl = rd q) + dE
+
cos q
= i sin q Fe
pr O Þ = Þ Fe = mg tan q
cos q mg
dE
æqö
= ç ÷dq dE sin q
è pø
EBD_7036
P-S- 172 Topic-wise Solved Papers - PHYSICS

kq 2 C C
Þ = mg tan q Þ q2 µ x2 tan q + 1– – 2+
x2 41. (b)
x 120 V 200 V
sin q = For potential to be made zero, after connection
2l
For small q, sin q » tan q \ q2 µ x3
é qù
120 C1 = 200 C2
êëQ C = v úû
Þ 3C1 = 5C2 y
l 42. (a) Þ Fnet = 2F cosq
l
F q F
Tcosq
æ qö x
2kq ç ÷ q a a q
è 2ø y
q Fnet = .
( )
Tsinq 2
y2 + a2 y + a2
2

æ qö
2kq ç ÷ y
è 2ø kq2 y
Fnet = Þ
Þ q
dq
µ x2
dx ( y 2 + a 2 )3/ 2 a3
dt dt
dq So, F µ y
\ = const.
dt 43. (d) L L
3/2 2
\ q µ x2.v
Þ x ax .v [Q q2 µ x3 ]
Þ v µ x–1/2 O A B
dx
38. (c) Electric field
df Electric potential is given by,
E=- = – 2ar ....(i)
dr
æ qö
By Gauss's theorem dx
1 çè L ÷ø
2L 2L q
kdq = ln(2)
E=
1 q
....(ii)
V= ò x
= ò 4pe 0 x 4pe 0 L
4pe 0 r 2 L L
From (i) and (ii), 44. (c) Potential difference between any two points in an
q = –8 pe0ar3 electric field is given by,
Þ dq = – 24pe0ar2 dr ur uur
dV = – E.dx
dq
Charge density, r = = – 6e0a VA 2
4pr 2dr
ò dV = - ò 30 x 2 dx
39. (c) Ein µ r VO 0

1 VA - VO = -[10 x3 ]20 = -80 J / C


Eout µ
r2 45. (a) Electric field in presence of dielectric between the two
40. (c) The electric field inside a uniformly charged sphere is plates of a parallel plate capacitor is given by,
r.r s
E=
3 Î0 K e0
The electric potential inside a uniformly charged sphere
Then, charge density
rR é 2
r2 ù
= ê3 - 2 ú s = K e0 E
6 Î0 ë R û
= 2.2 × 8.85 × 10–12 × 3 × 104 » 6 × 10–7 C/m2
\ Potential difference between centre and surface
Q1 1mF
rR 2 rR 2 46. (d)
= [3 - 2] =
6 Î0 6 Î0 Q
C
qrR2
DU = Q2 2mF
6 Î0
Electrostatics P-S- 173

2 2
From figure, Q2 = Q = Q 3µF C1 = 4µF 12µF = CP
2 +1 3 49. (a) 4µF

æC ´ 3ö 9µF
Q = E çè ÷ Þ
C + 3ø
2µF
2 æ 3CE ö 2CE
\ Q2 = çè ÷ = 8v 8v
3 C + 3ø C + 3
Therefore graph d correctly dipicts. é æ 12 ö ù
Charge on C1 is q1 = ê ç
è ÷ø ´ 8ú ´ 4 = 24mc
ë 4 + 12 û
Charge
4
The voltage across CP is VP = ×8 = 2v
4 + 12
\ Voltage across 9mF is also 2V
\ Charge on 9mF capacitor = 9 × 2 = 18mC
\ Total charge on 4 mF and 9mF = 42mc
C KQ 42 ´ 10-6
1mF 3mF \E = = 9 × 109 × = 420 Nc–1
r2 30 ´ 30
Kq 50. (c) Applying Gauss's law
47. (a,b) We know, V0 = = Vsurface
R
uur uur Q
Now, Vi =
Kq 2 2
(3R – r ) [For r < R] Ñò S E. ds = Î0
2R 3
At the centre of sphare r = 0. Here
Q + 4par 2 - 4pAa 2
3 \ E × 4pr2 =
V = V0 Î0
2
5 Kq Kq dr r dr
Now, = (3R 2 – r 2 ) r=
4 R 2R 3 dv Qa Gaussiam
R Q = r4pr2 surface
R2 = b
2 .
A
Q = ò 4pr dr =4pA [r2 – a2]
2
3 Kq Kq r
= 3 a
4 R R
1 Kq Kq 1 é Q - 4pAa 2 ù
= E= ê + 4 pA ú
4 R R4 4p Î0 ëê r2 ûú
R4 = 4R For E to be independent of 'r'
Also, R1 = 0 and R2 < (R4 – R3) Q – 2pAa2 = 0
48. (c) Field lines originate perpendicular from positive charge Q
and terminate perpendicular at negative charge. Further \ A=
this system can be treated as an electric dipole. 2pa 2
EBD_7036
13
P-S- 174 Topic-wise Solved Papers - PHYSICS

Current Elec tricity

Section-A : JEE Advanced/ IIT-JEE

A 1. 20 2. R/2 3. 0
B 1. F 2. F 3. T
C 1. (d) 2. (b) 3. (a) 4. (b) 5. (c) 6. (d) 7. (b)
8. (a) 9. (c) 10. (b) 11. (c) 12. (a)
13. (d) 14. (a) 15. (a) 16. (a) 17. (b) 18. (c) 19. (b)
20. (a) 21. (a) 22. (a) 23. (b) 24. (b) 25. (a) 26. (c)
27. (c) 28. (d) 29. (c) 30. (c) 31. (b) 32. (c) 33. (c)
D 1. (b, d) 2. (c) 3. (d) 4. (a, b, d) 5. (a, d) 6. (a, b, c, d) 7. (b,d)
8. (a, b, d) 9. (b) 10. (c,d) 11. (a,b,c,d)
E 1. 5W 2. 0.2% 3. 2W 4. VAB = 25 V, VBC = 75 V
5. Positive to negative terminal, 2.5 V
6. 22.5 V 7. (i) 2V, 1A, 0A, 1A (ii) 1A, 2A, 1A; 2A 8. 0.9A

2 21 19
9. (i) V (ii) V, V 10. 8 × 10–4 J 11. (ii) 1.5A 12. 1.5 A, 1.44 × 10–5 J 13. 6.67 V
13 13 13

14. (a) CV (1 - e -2t /3RC ); (b) V - V e -2t /3RC ; V 15. (a) No (b) 8W
2 2R 6R 2R
16. Battery connected across A and B. Output across A and C or B and C.
18. R2

CVR2 R1 + R2
19. Q0 = R + R ; a = CR R
1 2 1 2

F 1. A ® s; B ® q; C ® p, q; D ® q, r
G 1. (a) 2. (b)
H 1. (d)
I 1. 4 2. 2 3. 5 4. 5 5. 1

Section-B : JEE Main/ AIEEE

1. (c) 2. (b) 3. (b) 4. (b) 5. (b) 6. (c) 7. (d)


8. (a) 9. (c) 10. (d) 11. (b) 12. (c) 13. (a) 14. (c)
15. (b) 16. (c) 17. (a) 18. (a) 19. (d) 20. (c) 21. (b)
22. (b) 23. (a) 24. (c) 25. (a) 26. (a) 27. (c) 28. (b)
29. (d) 30. (d) 31. (d) 32. (d) 33. (b) 34. (b) 35. (b)
36. (c) 37. (a) 38. (d) 39. (b) 40. (a) 41. (c) 42. (c)
43. (c) 44. (d) 45. (a) 46. (c) 47. (d) 48. (d)
49. (c) 50. (b) 51. (a) 52. (a)
Current Electricity P-S- 175

Section-A JEE Advanced/ IIT-JEE


A. Fill in the Blanks Resistance is less in case of T1 and more in T2.
NOTE : For a metallic wire, resistance increases with
2 20W R
V temperature, therefore T2 > T1
1. We know that P =
R C. MCQs with ONE Correct Answer
2
V 100 ´ 100 1. (d) R1 = R0 (1 + a t1) Þ 1 = R0 [1 + 0.00125 × 27]
\ R= = = 20 W
P 500 200V R2 = R0 (1 + a t2) Þ 2 = R0 [1 + 0.00125 × t2]
NOTE : For the bulb to deliver 500 W, it should have a p.d. On solving we get
of 100 V across it. This would be possible only when T2 = 854°C = 1127 K
R = 20 W is in series with the bulb because in that case both 2. (b) KEY CONCEPT : The heat produced is given by
resistances will share equal p.d. æ 2ö
rl \ H = V 2 ç p r ÷
2
V
2. The given circuit may be redrawn as shown in the figure. H= and R =
R pr 2 è rl ø
Thus, the resistances 2R, 2R and R are in parallel.
1 1 1 1 2 æ pV 2 ö r 2
Hence, = + + = or H = ç ÷
R AB 2 R 2 R R R è r ø l
2R Thus heat (H) is doubled if both length (l) and radius
(r) are doubled.
A 2R B 1 1 0
3. (a) I µ 2 ;V µ ;V µ r
D C r r
R 4. (b) Since RAB = 2RCD therefore, current in AB will be half
R as compared to current in CD.
Hence, RAB =
2 P4 (i / 2) 2 4 1 10
3. Let a current I flow through the circuit. Net emf of the circuit = = Þ P4 = = 2 cal/s
P5 i2 ´ 5 5 5
= 8 (5V) = 40 V
Here P4 = Power dissipation in 4W
Net resistance in the circuit = 8 (0.2 W) = 1.6 W
P5 = Power dissipation in 5W
Current flowing through the circuit,
5. (c) BC and AC are in series
40 V \ RBCA = 30 + 30 = 60 W
I= = 25 A
1.6 W Now BA and DC are in parallel.
The voltmeter reading would be 1 1 1 90
V = E – IR = (5V) – (25A) (0.2 W) = + =
Req 30 60 30 ´ 60
= 5V – 5V = 0
Req = 20 W ; V = IR
B. True/ False 2
Þ I= = 0.1 Amp.
1. NOTE : An electrolyte solution is formed by mixing an 20
electrolyte in a solvent. The electrolyte on dissolution 6. (d) Copper is a metal whereas Germanium is Semi-conductor.
furnishes ions. The preferred movement of ions under the NOTE : Resistance of metal decreases and
semiconductor increases with decrease in temperature.
influence of electric field is responsible for electric current.
7. (b) The equivalent circuits are shown in the figure.
2. NOTE : Billions of electrons in a conductor are free and
have thermal velocities. The electrons have motion in random
R
R 2R
directions even in the absence of potential difference. R
3. For a given voltage, current is more in case of T1. E R 6R R 6R
4W
Since, V = IR 2R 4R
R 4R
V
\ R= E 4W Balanced
I Wheat stone
Bridge
T1 The circuit represents balanced Wheatstone Bridge.
I Hence 6R W resistance is ineffective
T2 R 2R 3R
2R 4R 6R

4W
E
V E 4W
EBD_7036
P-S- 176 Topic-wise Solved Papers - PHYSICS

1 1 1 (3R)(6 R) Let R' be the resistance of the second wire


= + , Req = = 2R 2l
Req 3R 6 R (3R) + (6R)
For Max. Power
External Resistance = Internal Resistance
2R = 4 W \ R = 2 W
8. (a) Since the opening or V V n times V
closing the switch does not Þ R' = 2R (Q length is twice)
affect the current through G, A ( NV ) 2
it means that in both the \ Energy released in t-seconds = ´t
P Q
2R
cases there is no current Also Q' = m'cDT = (2m) CDT
passing through S. Thus
potential at A is equal to S N 2V 2
2 mcDT = ´t .... (ii)
potential at B and it is the 2R
case of balanced R G
Dividing (i) by (ii)
wheatstone bridge.. B mcDT 9V 2 ´ t / R 1 9´ 2
= 2 2 or,
, =
IP = IQ and IR = IG 2mc DT N V t / 2 R 2 N2
or, N 2 = 18 × 2 \ N = 6
9. (c) There will be no current flowing in branch BE in steady 11. (c) Since current I is independent of R6, it follows that the
condition. resistance R1, R2, R3 and R4 must form the balanced
Let I be the current flowing in the loop ABCDEFA. Wheatstone bridge.\ R1 R4 = R2R3
Applying Kirchoff's law in the loop moving in 12. (a) The circuit is symmetrical about the axis POQ.
anticlockwise direction starting from C. The circuit above the axis POQ represents balanced
+ 2V – I (2R) – I (R) – V = 0 wheatstone bride. Hence the central resistance 2R is
\ V = 3IR ineffective. Similarly in the lower part (below the axis
POQ) the central resistance 2R is ineffective.
Þ I = V/3R ... (1)
2R 2R
2R 2R
Applying Kirchoff's law in the circuit ABEFA we get on
2R
moving in anticlockwise direction starting from B r O r
R P Q P r r Q
A F 2R
V I
2R 2R 2R 2R
Therefore the equivalent circuit is drawn.
B E
V C 1 1 1 1 r + r + 2r
\ = + + =
RPQ 4 R 4 R 2r 4 Rr
C D
2V 2R I 2 Rr
RPQ =
+ V + Vcap – IR – V = 0 (where Vcap is the p.d. across R+r
capacitor). V2
13. (d) KEY CONCEPT : R =
æV ö V P
\ Vcap = IR = ç ÷ ´ R = V2 V2
è 3R ø 3 \ R1 = , R2 = = R3 ;
10. (b) Let R be the resistance of wire. 100 60
R1 R2
l
V1 V2
V1= VR 1 V2= VR 2
R 1+R 2 R 1+R 2

V V V R3

(3V )2 250V
Energy released in t second = ´t 2
(250)
2
R W3= V =
R3 R3
9V 2
\ Q= ´t
R V = 250V
2
9V V12 V 2 R1 V22 V 2 R2
\ mcDT = ´ t (Q But Q = mcDT) .... (i) W1 = = , W2 = =
R R1 ( R1 + R2 ) 2 R2 ( R1 + R2 )2
Current Electricity P-S- 177

V2 æ 2R ö
and W3 = Power dissipation, P2 = I2 çè ÷ø = 0.67I2R
R3 3

(250) 2 (250) 2 R2 (250)2 R


W3 : W2 : W1 = : R2 : R1
R3 ( R1 + R2 )2 ( R1 + R2 ) 2 I R I R/3
or W3 : W2 : W1
(250) 2 (250) 2 V2 (250)2V 2 R
= ´ 60 : ´ :
V2 é 1 1ù 4
2 60 é 1 2
1ù 4 Power dissipation, P3 = I2 ( R / 3) = 0.33I 2 R
êë100 + 60 úû V êë100 60 úû V ´ 1000
+

or
W3 : W2 : W1 R

100 ´ 100 ´ 60 ´ 60 100 ´ 100 ´ 60 ´ 60 I R I 3R/2


= 60 : :
160 ´ 160 ´ 60 160 ´ 160 ´ 100 R
= 64 : 25 : 15
æ3 ö
14. (a) In ohm's law, we check V = IR where I is the current Power dissipation, P4 = I 2 çè R÷ø = 1.5 I2R
2
flowing through a resistor and V is the potential
difference across that resistor. Only option (a) fits the 17. (b) Total external resistance will be the total resistance of
above criteria. whole length of box. It should be connected between A
NOTE : Remember that ammeter is connected in series and D.
with resistance and voltmeter parallel with the net 18. (c) For various combinations equivalent resistance is
resistance. maximum between P and Q.
19. (b) KEY CONCEPT : The current in RC circuit is given by
A A
I = I0e–t/RC
V V
t æ -t ö
or lnI = lnI0– or lnI = çè ÷ + lnI 0
RC RC ø
æ -t ö æE ö
lnI = ç + ln ç 0 ÷
è RC ÷ø è Rø
15. (a) On comparing with y = mx + C
æE ö 1
Intercept = ln ç 0 ÷ and slope = –
R1 G R2 è Rø RC
x When R is changed to 2R then slope increases and
A R3 C R4 B current becomes less. New graph is Q.
20. (a) The current in 2W resistor will be zero because it is not
At null point
a part of any closed loop.
R1 R3 x
= =
R2 R4 100 - x
10V 5W
If radius of the wire is doubled, then the resistance of 20V
AC will change and also the resistance of CB will 10W

R1 R
change. But since does not change so, 3 should
R2 R4 2W
21. (a) KEY CONCEPT : At any instant of time t during
also not change at null point. Therefore the point C
does not change. charging process, the transient current in the circuit is
16. (a) given by
III < II < IV < I. V -t / RC
I= 0 e
R
I R R R I 3R \ Potential difference across resistor R is
Power dissipation, P1 = I 2 (3R) = 3I 2 R
é V0 - t / RC ù
VR = ê e ú´R
R R ëR û
I = V0e - t / RC ...(i)
I 2R
3 \ Potential diff. across C
R Vc = V0 – V0e–t/RC = V0 (1– e–t/RC) ...(ii)
EBD_7036
P-S- 178 Topic-wise Solved Papers - PHYSICS
25. (a) Given X is greater than 2W when the bridge is balanced
Q Vc = 3VR (given)
R X
Q ( )
V0 1 - e -t / RC = 3V0 .e - t / RC =
l 100 - l
Þ 1– e–t/RC = 3e–t/RC
Þ 1 = 4e–t/RC R = 2W X
X R = 2W
Taking log on both sides
æ t ö
loge 1 = 2loge2 + ç -
è RC ÷ø
l 100-l
l+20 80-l
t G
Þ 0 = 2 × 2.303 log10 2 – G
RC
Þ t = [2 × 2.303 log10 2] × 2.5 × 106 × 4 × 10–6 B Switch
= 13.86 sec. Case (i)
22. (a) KEY CONCEPT : or, 100 R – R l = l X or,, 200 – 2 l = l X
IgG = (I – Ig) S
200
Here, Ig = 100 × 10–6 A; G = 100 W; S = 0.1 W or, l=
X +2
Ig
G When the resistances are interchanged the jockey shifts
I 20 cm. Therefore
I-Ig X 2
=
l + 20 80 - l
S
80 X – l X = 2 l + 40
æG ö -6 æ 100 ö
\ I = Ig ç + 1÷ = 100 ´ 10 çè + 1÷ or, 80 X = l (X + 2) + 40
èS ø 0.1 ø
= 100 × 10–6 × 1000.1 = 100.01 mA æ 200 ö
or, 80 X = çè ÷ ( X + 2) + 40
23. (b) The heat supplied under these conditions is the change X + 2ø
in internal energy 240
Q = DU or, X = = 3W.
80
The heat supplied Q = i2RT 26. (c) The total charge enclosed in the dotted portion when
= 1 × 1 × 100 × 5 × 60 = 30,000 J = 30 kJ the switch S is open is zero. When the switch is closed
24. (b) A 2r B and steady state is reached, the current I coming from
C the battery is
I r 9 = I (3 + 6) Þ I = 1A
l/2
l/2 3 mF 6 mF
l +Q1 –Q1 +Q2 –Q2
r
VAB I AB R AB R 2[p ´ 4r 2 ] 1
(a) = = AB = l
= 3W 6W
VBC I BC RBC RBC r 4
2[pr 2 ]
[IAB = IBC, wire is of same material]
Therefore option (a) is incorrect. 9V
l \ Potential difference across 3W resistance = 3V and
2 r potential difference across 6W resistance = 6V
PBC I RBC 2[p ´ 4r 2 ] 1
(b) = 2 = = \ p.d. across 3 µF capacitor = 3V
PAB I RAB l 4
r 2
2[pr ] and p.d. across 6 µF capacitor = 6V
\ PAB = 4PBC; Therefore (b) is correct. \ Charge on 3 µF capacitor Q1 = 3 × 3 = 9 µC
I Charge on 6 µF capacitor Q2 = 6 × 6 = 36 µC
J AB p ´ 4r 2 1 The total charge enclosed in the dotted portion =
(c) = = ; Therefore (c) is incorrect.
J BC I 4 \ Charge passing the switch = 36 – 9 = 27 µC
2
p´r V2
éV AB ù 27. (c) We know that P =
R
E AB êë l / 2 úû 1 For constant value of potential difference (V) we have
(d) = = ; Therefore (d) is incorrect.
E BC é VBC ù 4 1
ê l/2 ú Pµ
ë û R
Current Electricity P-S- 179

Case (i) (iv) Ammeter is connected in series with the test resistor
RT.
(v) A variable voltage source V is connected in series with
1W 1W
1W 1W the test resistor RT.
2W 2W 1W
1W 1W l
1W 30. (c) We know that R = ρ
1W 1W a
Where l is the length of
the conductor through
This is a case of balanced Wheatstone bridge R1 = 1W which the current flows
Case (ii) and a is the area of cross
L
1W 1W section.
2W Here l = L and a = L × t t
1W 2W
1W rL r L
\ R= =
1W 1W L´t t
Clearly the equivalent resistance (R2) will be less than \ R is independent of L
1W. 31. (b) At Null point
Case (iii)
X 10W
1W 1W
2W 2W

1W
A B
1W 1W 52 cm 48 cm
X 10
Thus R3 = 2W =
l1 l 2
Since, R2 < R1 < R3
\ P2 > P1 > P3 Here l1 = 52 + End correction = 52 + 1 = 53 cm
V2 l 2 = 48 + End correction = 48 + 2 = 50 cm
28. (d) We know that P =
R X 10 53
For a given potential difference at a particular \ 53 = 50 Þ X = = 10.6W
temperature 5
32. (c) In case of a meter bridge
1
Pµ R X
R = Here X = 90 W, l = 40.0 cm
It is given that the powers of the bulbs are in the order l 100 - l
100W > 60 W > 40W Xl
\ R=
1 1 1 100 - l
\ > > For finding the value of R
R100 R60 R40
90 ´ 40
29. (c) The following points should be considered while making R= = 60W
the circuit : 60
(i) An ammeter is made by connecting a low resistance R2 For finding the value of DR
in parallel with the galvanometer G2. DR Dl D (100 - l )
= +
G2 R l 100 - l
DR 0.1 0.1
\ = +
60 40 60
R2 \ DR = 0.25W
(ii) A voltmeter is made by connecting a high resistance R1
Therefore, R = ( 60 ± 0.25 ) W
in series with the galvanometer G1.
R1 I dV / dR
33. (c) J= = … (i)
G1 2pr l 2 pr l
dr 1 dr
(iii) Voltmeter is connected in parallel with the test resistor dR = r = ´ … (ii)
RT. 2pr l s 2 pr l
EBD_7036
P-S- 180 Topic-wise Solved Papers - PHYSICS

dV q0 V
Now E = - I= = (R is same for C1 and C2)
dr RC R
l Also, q = q0e–t/RC
\ dV = –Edr = – dr … (iii)
2p Î r q q
When q = 0 then 0 = q0 e–t/RC
2 2
or e + t/RC = 2. Þ \ t = RC loge 2 \ t µ C.
+ t1 C1 1
+
+
V \ t = C = = 0.5
+ 2 2 2
+ I or, t1 = 0.5 t2
+ V + dV
l + Therefore time taken for the first capacitor (1 µF) for
+
+ discharging 50% of initial charge will be less.
+ r 2. (c) NOTE : The conductivity of a semiconductor increases
+
+ dr with increase in temperature i.e. the resistivity decreases
+
+ with increase in temperature.
+ In a conducting solid, the collisions become more
+
+ frequent with increase of temperature.
+ 3. (d) The net resistance of the circuit is 9W as shown in the
+
From (i), (ii), and (iii) following figures.
3W 2W 2W 3W 2W
1 é ldr s 2pr l ù ls
J= ê ´ ú = … (iv)
2pr l ë 2l Î r dr û 2p Î r 9V 8W 8W 4W
9V
8W 8W 8W
dV -l s ´ 2pr l -lsl
Also I = = dr ´ = …(v) 2W 2W 2W 2W 2W
dR 2p Î r dr Î
Here negative sign signifies that the current is 3W 3W 2W
decreasing
d (q ) d (l l) dl 9V
8W 8W
9V
8W 4W
But I = = =l … (vi)
dt dt dt
2W 2W 2W
From (v) and (vi)
dl lsl d l -s 3W
l =- Þ = dt
dt Î l Îl
4W 9V 9W
On integrating 9V
l t
dl s 2W
ò l
= - ò dt
Î0 V 9V
l0 I= = = 1.0 A
R 9W
l st s
- t
\ loge l = - Î \ l = l0 e Î The flow of current in the circuit is as follows.
0 1ampere 3W 0.5A
Substituting this value in (iv) we get 0.5A
s 9W 8W 8W
sl0 - Î t 9V 9V
J= e
2p Î r 2W

D. MCQ's with ONE or MORE THAN ONE Correct 1A 2W 2W 0.25A 3W 2W 0.25A


3W 0.25A
1. (b, d) KEY CONCEPT : During decay of charge in R.C. 8W 8W 4W 8W 8W 8W
9V 9V
circuit,
I = I0e–t/RC 2W 2W 2W 2W 2W
C1 C2 NOTE : The current divides into two equal parts if passes
through two equal resistances in parallel.
q0
where I0 = Thus current through 4W resistor is 0.25 A.
RC 4. (a,b,d) At 0K an insulator does not permit any current to
V V
flow through it. Option (a) is correct.
q0
When t = 0, I = I0 = At 0K a semiconductor behaves as an insulator. Option (b)
RC is correct.
Since potential difference between the plates is same initially
In reverse biasing at 300 K, a very small current.
therefore I is same in both the cases at t = 0 and is equal to
flows through a p-n junction diode. Option (d) is correct.
Current Electricity P-S- 181

In case of metal, the current flowing will be very-very high 6. (a, b, c, d)


because a metal becomes super conductor at 0K. Option (c) The given circuit is an extension of wheatstone bridge,
is incorrect therefore points P and Q are at the same potential and point
5. (a, d) S and T are also at the same potential. Therefore no current
2k W
passes through PQ and ST and the circuit reduces to as
R1 R1
I I shown
24 V 6W

RL = 1.5 kW
24 V
6k W R2 Rp I2

I1 – I2
R ´ RL 6 ´ 1.5 9 12 W
Rp = 2 = = kW
R2 + RL 6 + 1.5 7.5 I1
24 24 ´ 7.5
\ I= mA = = 7.5 mA
9 24 12V
+2
7.5 12 é 6 ´ 12 ù
Þ option (a) is correct.
I1 =
4 êëQ Req = 6 + 12 úû
The potential difference accros RL = potential = 3A
difference accros Rp é 12 ù
\ I2 = 3 ê = 2A
æ 9 ö ë 6 + 12 úû
= (7.5 mA) ç k W÷ = 9V As P and Q are equipotential and potential at S is less
è 7.5 ø
than the potential at P (potential drops across a
Þ option (b) is incorrect.
resistance as current passes through it), therefore VS <
(15)2 VQ.
Power dissipation across R1 V2
Now, = 2 7. (b, d) H = ´4 ...(i)
Power dissipation across R2 (9)2 R
6 4rl
where R =
15 ´ 15 6 pd 2
= ´ = 8.33
2 9´9 When resistances are connected in series
Þ option (c) is incorrect. é 4rl ù R R
The magnitude of power dissipated accross R 2 is Total resistance = R1 + R2 = 2 ê 2ú
= 2´ =
ë 4pd û 4 2
(9)2 V2
. \ H= ´ t2 ...(ii)
1.5 R/2
Now when R1 and R2 are interchanged the equivalent From (i) and (ii) t2 = 2 min. Therefore (b) is correct.
2 ´ 1.5 3 When resistance are connected in parallel
resistance between R1 and RL = = kW
2 + 1.5 3.5 R1R 2 R2 R / 4 R
Total resistance = = 1 = =
R1 + R 2 2R1 2 8

R2 = 6 k W V2
\ H= ´ t2 ...(iii)
R /8
From (i) and (iii) t2 = 0.5 min
RL = 1.5 k W

24 V
6kW R1 \ (d) is correct
8. (a, b, d)
Applying KVL in MNOPQAM O P
\ Potential drop across this equivalent resistance i
V1 - iR1 + V2 - iR 3 = 0
3
V1 R1
3.5 3 V1 + V2
= ´ 24 = ´ 24 = 3V \i= ... (i) R2
3 24 R1 + R 3
+6 N Q
3.5 Applying KVL in NOPQN
32 1 é 92 ù V V2
\ Potential difference accros RL = = ê ú v1 - iR1 = 0 \ i = ...(ii)
1.5 9 ëê1.5 ûú R1 R3 i

\ The magnitude of the power dissipation in RL will M A


V1 V1 + V2
decrease by a factor 9 if R1 and R2 are interchanged. From (i) & (ii) R = R + R
(d) is the correct option. 1 1 3
EBD_7036
P-S- 182 Topic-wise Solved Papers - PHYSICS
\ V1R1 + V1R3 = V1R1 + V2R1 (b) is the correct option
Þ V1R3 = V2R1
io éQ i = i e - t t ù
If V1 = V2 then R1 = R3 = R2\ (a) is correct option. At t = 1sec, i =
e ëê o
ûú
If V1 = V2 then R1 = R3 = 2R2\ (b) is correct option.
(R2 can have any value as there is no current flowing \ (c) is the correct option
through d) After a long time no current flows since both capcitor
If V1 = 2V2 then 2R3 = R1 \ (c) is incorrect option. and voltmeter do not allow current to flow.
If 2V1 = V2 then R3 = 2R1 = R2 \ (d) is correct option. \ (d) is the correct option.

rFe ´ lFe 10–7 ´ 50 ´10 –3 25 E. Subjective Problems


9. (b) RFe = = = ´10–4
AFe 4 ´ 10 –6 2 1. The resistance of the heater is
V 2 100 ´ 100
r Al ´ l Al 2.7 ´ 10 –8 ´ 50 ´ 10 –3 2.7 ´ 50 R= = = 10 W
RAl = = = × 10–5 P 100
AAl (49 – 4) ´ 10 –6 45 The power on which it operates is 62.5 W
= 0.3 × 10–4 \ V = R ´ P ' = 10 ´ 62.5 = 625 = 25
Since the voltage drop across the heater is 25V hence
RFe ´ RAl 12.5 ´10 –4 ´ 0.3 ´ 10–4 voltage drop across 10W resistor is (100 – 25) = 75V.
Rtotal = = » 29µW
RFe + RAl 12.8 ´10 –4 V 75
\ The current in AB = I = = = 7.5 A
10. (c, d) R 10
With the use of filament and the evaporation involved, the This current divides into two parts. Let I1 be the current
filament will become thinner thereby decreasing the area of that passes through the heater. Therefore
cross-section and increasing the resistance. Therefore the 25 = I1 × 10
filament will consume less power towards the end of life. I1 = 2.5 A
As the evaporation is non-uniform, the area of cross-section Thus current through R is 5A.
will be different at different cross-section. Therefore A B Heater
temperature distribution will be non-uniform. The filament R
will break at the point where the temperature is maximum.
æ 1ö
When the filament temperature is higher ç l n µ ÷ , it emits 100 V
è Tø
Applying Ohm's law across R, we get
light of lower wavelength or higher band of frequencies.
25 = 5 × R
11. (a, b, c, d) Þ R = 5W
At t=0, Capacitors act as short circuit and voltmeter display
–5v lf li
r -r
At t = µ, Capacitor acts as open circuit and no current R f - Ri Af Ai
flows through voltmeter ( Q very high resistance of 2. ´ 100 = ´ 100
Ri l
voltmeter) r i
Ai
so it display +5V. (a) is the correct option

( ),
t
-t V - 2CR lf li
q1 = 2CV 1 - e 2CR
i1 = e -
R Af Ai
( ), = ´ 100
t
-t V - 2CR li ... (i)
q2 = CV 1 - e 2CR
i2 = e
2R Ai
2C R Let the initial length of the wire be 100 cm, then the new
i1 0.1
length is 100 + ´ 100
V 100
i2 2R lf = 100.1 cm ...(ii)
Let Ai and Af be the initial and final area of cross-section.
C
Then
100 ×Ai = 100.1 Af
100
5V Þ Af = A ... (iii)
100.1 i
CV1 é -t
ù From (i), (ii) and (iii)
\ D V = – i2 × 2R + = V ê1 - 2e 2CR ú =0
2C ë û
Current Electricity P-S- 183

400 R
(100.1)2 100 R'=
- (400 + R )
R f - Ri 100 Ai Ai
´ 100 = ´ 100 As the potential difference of 60 V is equally shared between
Ri 100
the 300 W and 400 W resistance. R' should be equal to 300 W.
Ai Thus
400 R
(100.1)2 - (100)2 200.1 ´ 0.1 300 =
= ´ 100 = ´ 100 (400 + R )
(100)2 100 ´ 100
which gives R = 1200W, is the resistance of the voltmeter.
= 0.2 %
When the voltmeter is connected across the 300W resistance,
Thus the resistance increases by 0.2%.
their combined resistance is
DR DA Dl
Alternatively for small change = + 300R 300 ´ 1200
R A l R '' = = = 240W
(300 + R ) (300 + 1200)
3W 3W 3W \ Total resistance in the ciruit = 400 + 240 = 640 W
3W
3. 3W 3W 6W 3W \ Current in the circuit is
6W 6W 6W 6W 3W
6W Þ Þ
3W 6W 3W 3W 3W 60V 3
6W 6W I= = A
3W 3W 3W 640W 32
3W 6W 3W 3W 6W
6W
3W \ Voltmeter reading
3W 6W
3W 3W
3W
= Potential difference across 240 W resistance
6W
3W 3W 3W
3W 3
6W
Þ = ´ 240 = 22.5V
2W 32
3W

4. Applying Kirchoff's law in loop AQBRC E1 E2 E3


+ +
q q SE / r r1 r2 r3
– - + 100 = 0 7. (i) VAB = =
6 2 1 1 1 1
S + +
Þ q = 150 µC r r1 r2 r3
150
\ Potential difference between AB = = 25V
6 (ii) r1
\ Potential difference between BC P Q
I1 E1
= 100 – 25 = 75V
A R U I2 E2 R B
3 mF 1 mF 3 mF 1 mF
I3
3 mF 1 mF
3 mF 1 mF T r3 E3 S
I1+I2+I3
1 mF
1 mF
10 W 10 W

20 W 100 V 20 W 100 V Applying Kirchoff's law in PQRUP starting from P moving


6 mF 2 mF
clockwise
B
Q +q -q +q -q R I1r1 – E1 + E2 = 0 or I1 – 3 + 2 = 0
or I1 = 1 amp
A C
100 V Applying Kirchoff's law in URSTU starting from U moving
D E clockwise
14mF
– E2 + E3 – I3r3 = 0
5. NOTE : The current will flow from the positive terminal to or – 2 + 1 – I3 = 0
the negative terminal inside the battery. or I3 = – 1 amp
During charging the potential difference NOTE : The – ve sign of I3 indicates that the direction of
V = E + Ir = 2 + 5 × 0.1 = 2.5 V current in branch UTSR is opposite to that assumed.
6. Potential difference across the 400 W resistance = 30 V. Applying Kirchoff's law in AURBA starting from A moving
Therefore, potential difference across the 300 W resistance clockwise.
= 60 – 30V = 30 V. Let R be the resistance of the voltmeter.
(I1 + I2 + I3) R – E2 = 0 or (1 + I2 – 1) R = 2
As the voltmeter is in the parallel with the 400 W resistance,
their combined resistance is or I2 = 2 amp
Current through R is I1 + I2 + I3 = 2A
EBD_7036
P-S- 184 Topic-wise Solved Papers - PHYSICS
NOTE : No current flows through capacitor at steady state.
2´3
8. (Req)AB = = 1.2 W Moving the loop along MNO to P
2+3
\ VM – 5 × i1 – 1 × I1 – 2 × i2 = VP
Total current through the battery \ VM – VP = 6i1 + 2i2 = 6 × 3 + 2 × 1 = 20 V
6 3 \ V = 6 × 3 + 2 × 1= 20 V
= = 1.5 A \ I1 = ´ 1.5 = 0.9 A
1.2 + 2.8 5 Energy stored in the capacitor
9. Let I2 current flow through the branch DCB 1 1
\ By Kirchoff's junction law, current in branch DB will be = CV 2 = × 4 × 10–6 × 20 × 20 = 8 × 10–4 J
2 2
I2 – I1 as shown in the figure.
11. (i) Let the effective resistance between points C and D be
I2 2V R then the circuit can be redrawn as shown
A
E
B The effective resistance between A and B is
2W 2´R
Req = 1 + .
R+2
1V 1V
1W
2W

1W
H
F
A 1W C 1W 1W
A 1W
I2 3W I1
6V 2W 2W 2W to¥ 6V 2W R
I2-I1
D G C B
3V I1 D B

Applying Kirchoff's law in loop BDAB This resistance Req can be taken as R because if we add one
+ 2 (I2 – I1) + 1 + 1 × I2 – 2 + 2I2 = 0 identical item to infinite items then the result will almost be
Þ 2I1 – 5I2 = – 1 ... (i) the same.
Applying Kirchoff's law in loop BCDB, we get 2´R
– 2(I2 – I1) + 1 + I1 – 3 + 3I1 = 0 \ 1+ =R
Þ 3I1 – I2 = 1 ... (ii) R+2
Solving (i) and (ii), we get I1 = 6/13 amp Þ R + 2 + 2R = R2 + 2R Þ R2 – R – 2 = 0
Þ R2 – 2R + R – 2 = 0
5
and I2 = amp Þ R (R – 2) + 1 (R – 2) = 0
13 Þ [R + 1] [R – 2] = 0 Þ R = 2W.
(i) To find the p.d. between B and D, we move from B to D
6
é5 6ù 2 (ii) RAB= 1W + 1W = 2W \ IAB = = 3 Amp.
VB + ê - ú ´ 2 = VD \ VB – BD = volt 2
ë13 13 û 13 Further, iCD = iCF as resistances RCD = RCF
\ iCD = iCF = 1.5 A
6 39 - 18 21
(ii) p.d. across G = 3 - ´3= = volt
13 13 13 12. R 1=6W
G
E1=6V
A
[Q the cell is in discharging mode]
i1
6 19 5mF=C i2 i1
i2+i1
p.d. across H = 1 + 1 × = volt E B
13 13 R =2W F R3 =4W
i2
2V=E2 2
[Q cell is in charging mode] i2
10. Applying Kirchoff's first law at junction M, we get the current D E =3V C
R4=3W i2
3
i1 = 3A
Applying Kirchoff's law in ABFGA
1A 6 – (i1 + i2) 4 = 0 ... (i)
3W Applying Kirchoff's law in BCDEFB
4V 3W
M i1 5W N i2 × 3 – 3 – 2 + 2i2 + (i2 + i1) 4 = 0 ... (ii)
2amp
C 4mF Putting the value of 4 (i1 + i2) = 6 in (ii)
1W 3i2 – 5 + 2i2 + 6 = 0
3V 1W
2A P i2 2W O 4W 1
3W \ i2 = - A
5
1A
Substituting this value in (i), we get
Applying Kirchoff's first law at junction P, we get current æ 1ö
i2 = 1A i1 = 1.5 – ç - ÷ = 1.7 A
è 5ø
Current Electricity P-S- 185

Therefore current in R3 (a) Applying Kirchoff's second law in the loop MNABM
= i1 + i2 = 1.7 – 0.2 = 1.5 A V = (i – i1) R + iR
To find the p.d. across the capacitor or V = 2iR – i1R ... (i)
VE – 2 – 0.2 × 2 = VG Similarly, applying Kirchoff's second law in loop MNSTM,
\ VE – VG = 2.4 V we have
or V = 24 V
Q
V = i1 R + + iR ...(ii)
1 2 C
\ Energy stored in capacitor = CV
2 Eliminating i from equation (1) and (2), we get
1 2Q 2Q
= × 5 × 10–6 × (2.4)2 = 1.44 × 10–5 J V = 3i1R + or 3i1R = V –
2 C C
13. We can redraw the circuit as. 1 æ 2Q ö dQ 1 æ 2Q ö
E or i1 = çèV - ÷ or = çV - ÷
3R C ø dt 3R è Cø
200W 100W
I/2 dQ dt Q dQ t dt
G
M 400W 100W B or
æ
=
2Q ö 3 R
or ò0 æ 2Q ö

0 3R
çè V - V-
I/2 ÷
C ø èç C
÷
ø
V 100W
D CV
400W This equation gives Q = (1 – e–2t/3RC)
2
dQ
(b) i1 =
dt
10V
The equivalent resistance between G and D is d é CV
i1 = ê
dt ë 2
( )
1 - e-2t / 3RC ú
ù
û
400 ´ 400
RGD = = 200 W CV 2 V -2t / 3RC
400 + 400 = ´ ´ e -2t / 3 RC = e
2 3 RC 3R
RGE REB From equation (i)
Since, =
RGD RDB V
V + e-2t /3RC
\ It is a case of balanced wheatstone bridge. V + i1R 3
i= =
The equivalent resistance across G and B is 2R 2R
300 ´ 300 \ Current through AB
RGB = = 150 W
300 + 300 V
V + e -2t / 3 RC
3 V
V 10 1 i2 = i – i1 = - e -2t / 3 RC
\ Current I = = = Amp. 2R R
RGB 150 15
V V -2t / 3 RC
NOTE : Since RGEB = RGDB the current is divided at G into i2 = - e
2R 6R
æ Iö I V
two equal parts çè ÷ø . The current further divides into i2 = as t ® ¥
2 2 2R
two equal parts at M. 15. (a) No. There are no positive and negative terminals on
Therefore the potential difference across the voltmeter the galvanometer.
I 1 400 20 NOTE : Whenever there is no current, the pointer of the
= ´ 400 = ´ = Volt = 6.67 V galvanometer is at zero. The pointer swings on both side of
4 15 4 3 zero depending on the direction of current.
14. Let at any time t charge on capacitor C be Q. Let currents are
R 0.6 r 12 W
as shown in fig. Since charge Q will increase with time 't' (b)Q Bridge is balanced AJ = =
RJB 0.4 r X
dQ
therefore i1 = Þ x=8W
dt where r is the resistance per unit length.
A i1 R
S
N i B X C
i-i1=i2 A JÝ D
12W
Q+
V R C G
R 16. Battery should be connected across A and B. Output can be
M B T taken across the terminals A and C or B and C.
EBD_7036
P-S- 186 Topic-wise Solved Papers - PHYSICS

C F. Match the Following


1. A®s
A B Reason : Bimetallic strip is based on thermal expansion of
R
solids.
B®q
17. For the experimental verification of Ohm's law, ammeter and Steam engine is based on energy conversion.
voltmeter should be connected as shown in the figure. C ® p, q
A voltmeter is a high resistance galvanometer (106W) which Incandescent lamp is based on energy conversion and
is connected in parallel with the main resistance of 100W. radiation from a hot body.
An ammeter is a low resistance galvanometer (10–3W) which D ® q, r
is connected in parallel with the main resistance. Electric fuse is based on melting point of the fuse material
100W Ammeter which is turn depends on the heating effect of current.
–3
10 W
G. Comprehension Based Questions
6
10 W 1. (a) From the given graph it is clear that with increase of
Voltmeter the magnitude of magnetic field (B), the critical
temperature TC (B) decreases.
18. KEY CONEPT : At all null points the wheatstone bridge will Given B2 > B1. Therefore for B2, the temperature at
be balanced which the resistance becomes zero should be less. The
X R above statement is true for graph (a).
\ =
r1 r2 2. (b) We know that as B increases, TC decreases but the
r exact dependence is not known.
Þ X= R 1 Given at B = 0, TC = 100 K
r2 X R
where R is a constant and at B = 7.5T, TC = 75 K
G
r1 and r2 are variable. r1 r2
\ At B = 5T, TC should be between 75 K and 100 K.
The maximum fractional
error is M A B C N H. Assertion & Reason Type Questions
R=R1 R=R2 R=R3
DX D r1 D r2 1. (d) When the temperature of metal increases; its resistance
= +
X r1 r2 increases.
Here, Dr1 = Dr2 = y (say) then Therefore statement - 2 is correct.
DX
For to be minimum r1 × r2 should be max
X (Unknown resistance) (Standard resistance)
[Q r1 + r2 = c (constt.)] X S
Let E = r1 × r2 N
Þ E = r1 × (r1 – c)
dE
\ = (r1 – c) + r1 = 0
dr1 l 100 l
c c N
Þ r1 = Þ r2 = Þ r1 = r2 G
2 2
Þ R2 gives the most accurate value.
19. Given Q = Q0[1 – e–at]
Here Q0 = Maximum charge and For a meter bridge when null point N is obtained we get
1 1 X S
a= t = C =
c Re q l 100 - l
Now the maximum charge When the unknown resistance is put inside an
enclosure, maintained at a high temperature, then X
Q0 = C [V0 ] where V0 = max potential difference across C increases. To maintain the ratio of null point l should
é V ù also increase. But if we want to keep the null point at
= C ê R + R ´ R2 ú the initial position (i.e., if we want no change in the
ë 1 2 û
value of l) there to maintain the ratio, S should be
and t c = C Req increased.
é RR ù 1 R1 + R2 Therefore statement - 1 is false.
=Cê 1 2 ú \ a = =
R
ë 1 + R2û t c CR1 R2
Current Electricity P-S- 187
6v
I. Integer Value Correct Type –3 – 2i – i + 6 = 0 1W
\ 3i = 3 i
1. 4 Cells connected in series
\ i = 1 Amp
R R A B

2W i
I 3v
Now let us travel in the circuit from A to B through battery
r
of 6V, we get
r
E E 2E 2r VA – 6 + 1 × 1 = VB
æ 2E ö
2 \ VA – VB = 5 volt.
2
J1 = I R = ç .R
è 2r + R ÷ø
...(1)
æ I - Ig ö V
Cells connected in parallel 4. 5 ç ÷S= -R
I
è g ø I g
R
R
1.5 - 0.006 2 n 30
´ = - 4990
I 0.006 249 0.006
E r
\ n»5
E r 5. 1 The equivalent resistance of balanced wheatstone
r
E 2 bridge is
2 3´ 6
æ E ö
2 Req = = 2W
J2 = I R = ç ´R 3+ 6
r÷ ...(2)
çR+ ÷
è 2ø
Given J1 = 2.25 J2 2W 1W
Balanced
2W
(2 E )2 E2 I wheatstone
. R = 2.25 .R 6W 8W
2W bridge
(2r + R )2 r
( R + )2 6.5V 4W
2
10W
4 2.25
\ = 12W 4W
(2r + R )2 æ rö
2

çè R + ÷ø
2
The equivalent resistance of balanced wheat stone
2
\ 4[ R + 0.5] = 2.25[2 + R] 2
[Qr = 1W] bridge is

\ 2 (R + 0.5) = 1.5 (2 + R) 6 ´ 18 9
Req= = W
\ R = 4W 24 2
2. 2 The equivalent circuit is shown in the figure. 6.5
\ I= = 1A
R = 1MW, C = 4mF 2 + 4.5
1MW 4m F
2W Balanced
wheatstone
I bridge
6W 2W
6.5V 10W
10V
\ The time constant t = RC = 4 sec 12W 4W
The potential across 4mF capacitor at any time ‘t’ is given as

é -t ù é t ù
Þ t = 2 sec
ê
V = V0 ë1 - e ût ú 4 = 10 ê
ë1 - e 4úû
3. 5
Let i be the current flowing in the circuit. Apply Kirchhoff’s
law in the loop we get
EBD_7036
P-S- 188 Topic-wise Solved Papers - PHYSICS

Section-B JEE Main/ AIEEE


1. (c) KEY CONCEPT : To convert a galvanometer into a L = length of potentiometer wire
voltmeter we connect a high resistance in series with El 30E
the galvanometer. V= =
L 100
The same procedure needs to be done if ammeter is to
be used as a voltmeter. E
R
i
2 l
V i
2. (b) Case 1 : P1 =
R
V r
Case 2 : The wire is cut into two equal pieces. Therefore V
R NOTE In this arrangement, the internal resistance
the resistance of the individual wire is . These are
2 of the battery E does not play any role as current is not
connected in parallel passing through the battery.
R/2 8. (a) Let q be the smallest temperature difference that can
R/4
be detected by the thermocouple, then
R/2
R/2 R I × R = (25 × 10–6) q
\ Req = = =
2 4 where I is the smallest current which can be detected
V V by the galvanometer of resistance R.
\ 10–5 × 40 = 25 × 10 – 6 × q
V2 æ V2 ö \ q = 16°C.
P =
\ 2 R/4 = 4 ç ÷ = 4P1
è R ø 9. (c) According to Faraday’s first law of electrolysis
m = Z×q
3. (b) When current is passed through a spring then current For same q, mµ Z
flows parallel in the adjacent turns.
NOTE : When two wires are placed parallel to each mCu ZCu
\ =
other and current flows in the same direction, the wires mZn Z Zn
attract each other.
Similarly here the various turns attract each other and Z Cu 31.5
Þ mCu = ´ mZn = ´ 0.13 = 0.126 g
the spring will compress. Z Zn 32.5
2´R 10. (d) ig × G = (i – ig) S
4. (b) The equivalent resistance is Req =
2+ R ig ´ G 1 ´ 0.81
\ S= = = 0.09W
i - ig 10 - 1
V2
\ Power dissipation P =
Re q 3 ´ 6 18
11. (b) Rp = = = 2W
3+ 6 9
15 ´ 15 15 3
\ 15 0 = R \ Req = = V 3
eq 10 2 \ V = IR ÞI = = = 1.5 A
R 2
2R 3
Þ = Þ 4 R = 6 + 3R Þ R = 6W
2+ R 2 3W 3W
3V
5. (b) According to Faraday's first law of electrolysis 3V 3W
m = ZIt Þ m µ It
6W
q q
6. (c) qn = i + c . 3W
2 2
Vrated (220)2
7. (d) From the principle of potentiometer, V µ l 12. (c) We know that R = =
Prated 1000
V l
Þ = ; where When this bulb is connected to 110 volt mains supply
E L
we get
V = emf of battery, E = emf of standard cell.
Current Electricity P-S- 189

18. (a) DQ = mC ´ DT
V 2 (110)2 ´ 1000 1000
P= = = = 250W = 1 × 4180 × (40 – 10) = 4180 × 30
R (220)2 4
( \ DQ = heat supplied in time t for heating 1L water
from 10°C to 40°C)
6V
4180 ´ 30
also DQ = 836 ´ t Þ t = = 150 s
2W 836
1.5W
13. (a) 19. (d) Neutral temperature is the temperature of a hot junction
at which E is maximum.
6W 3W
dE -a
Þ = 0 or a + 2bq = 0 Þ q = = -350
dq 2b
6V 6V
Neutral temperature can never be negative hence no q
is possible.
3/2W 3/2W 3W 20. (c) The mass liberated m, electrochemical equivalent of a
metal Z, are related as
3W 3W m = Zit
Þ m = 3.3 ´ 10 -7 ´ 3 ´ 2 = 19.8 ´ 10 -7 kg
6
hence Req = 3/2; \I = = 4A
3/ 2 i i
R1 R2 F m 0 i1 i2 m 0 i 2 F
14. (c) 21. (b) = =
l 2 pd 2 pd
R1 R2
S = R1 + R2 and P =
R1 + R2
(attractive as current is in the same direction)
n( R1R2 )
S = nP Þ R1 + R2 = V 2t
( R1 + R2 ) 22. (b) H =
R

2 R11 + R22 + R1R2 R


Þ ( R1 + R2 ) = nR1 R2 Þ n = Resistance of half the coil =
R1 R2 2
\ As R reduces to half, ‘H’ will be doubled.
R1 R2
n= + +2 500W Zero
R2 R1 A
Arithmetic mean > Geometric mean
i i
Minimum value of n is 4 2V
R
23. (a) 12V
R1 i1

R2 i2
15. (b)
1
V iR = 2 = 12 – 500 i \ i =
50
i1R1 = i2R2 (same potential difference)
1
\ ´R = 2
i R l r12 3 4 1
50
\ 1 = 2 = 2´ = ´ = (same r)
i2 R1 l1 r22 4 9 3 \ R = 100 W
24. (c) KEY CONCEPT : Resistance of Galvanometer,
X 20 1 Current sensitivity 10
16. (c) In the first case = = G= Þ G= = 5W
Y 80 4 Voltage sensitivity 2

In the second case 4 X = l 150


Þ l = 50 Here i g = Full scale deflection current = = 15 mA
Y 100 - l 10
17. (a) Thermistors are usually made of metaloxides with high V = voltage to be measured = 150 volts
temperature coefficient of resistivity. (such that each division reads 1 volt)
EBD_7036
P-S- 190 Topic-wise Solved Papers - PHYSICS
30. (d) NOTE : Kirchhoff's first law is based on conservation
150
Þ R= - 5 = 9995W of charge and Kirchhoff's second law is based on
15 ´ 10 -3 conservation of energy.
31. (d) rB = 2rA
25. (a)
R1 R2 dB = 2dA
I
rB l B r A l A
RB = RA Þ A = A
B A

l B r A d B2 r 4d 2
\ = ´ 2 = A ´ 2A = 2
l A rB d A 2r A d A

R 32. (d) At cold junction, current flows from Antimony to


Bismuth (because current flows from metal occurring
2e later in the series to metal occurring earlier in the
I= thermoelectric series).
R + R1 + R 2
33. (b) The network of resistors is a balanced wheatstone
Potential difference across second cell = V = e – IR2 = 0
bridge. The equivalent circuit is
2e
e – .R2 = 0 10W 20W
R + R1 + R 2 30W
R + R1 + R 2 - 2R 2 = 0 10W

R + R1 - R 2 = 0 \ R = R 2 - R1
15W
26. (a) Mass deposited
5W 10W
1 Þ Z1 = q2 5V 5V 5V
m = Zq Þ Zµ .... (i)
q Z 2 q1 15 ´ 30 V 5
Req = = 10 W Þ I = = = 0.5 A
Also q = q1 + q2 .... (ii) 15 + 30 R 10
34. (b) R1 = R0 [1 + a × 100] = 100 ....(1)
q q
Þ = 1 +1 (Dividing (ii) by q2 ) R2 = R0 [1 + a × T] = 200 ....(2)
q2 q2
On dividing we get
q 200 1 + aT 1 + 0.005 T
Þ q2 = q
.... (iii) = Þ2= Þ T = 400°C
1+ 1 100 1 + 100a 1 + 100 ´ 0.005
q2
NOTE : We may use this expression as an approximation
q because the difference in the answers is appreciable.
From equations (i) and (iii), q2 = For accurate results one should use R = R0eaDT
Z
1+ 2
Z1 P R SS
35. (b) = where S = 1 2
27. (c) The internal resistance of the cell, Q S S1 + S2

æl -l ö 240 - 120
r= ç 1 2÷ ´R = ´ 2 = 2W V 2 (220) 2
è l2 ø 120 36. (c) The resistance of the bulb is R = =
P 100
The power consumed when operated at 110 V is
V2
28. (b) P = Vi =
R (110)2 100
P= = = 25 W
2 2 4
V 200 ´ 200 (220) /100
Rhot = = = 400 W
P 100 37. (a) Required ratio
400 1
Rcold = = 40 W CV 2
10 Energy stored in capacitor 2
= =
E Workdone by the battery Ce 2
29. (d) I = , Internal resistance (r) is
R+r where C = Capacitance of capacitor
E V = Potential difference,
zero, I = = constant.
R e = emf of battery
Current Electricity P-S- 191

1 2 é 5 - 10i1 ù
Ce 1
From (i) and (ii) 11 i1 + 2 - ê ú=0
=2 = ( Q V = e) ë 2 û
Ce 2 2
1
Þ i1 = A from P2 to P1
38. (d) KEY CONCEPT : We know that 32
Rt = R0 (1 + at ),
q2
where Rt is the resistance of the wire at t ºC, 43. (c) Initial energy of capacitor, E1 = 1
R0 is the resistance of the wire at 0ºC 2C
2
and a is the temperature coefficient of resistance. æ q1 ö
1 q12
Þ R50 = R0 (1 + 50 a ) ... (i) Final energy of capacitor, E2 = E1 = =ç 2÷
2 4C ç ÷
R100 = R0 (1 + 100 a ) ... (ii) è 2C ø
1
From (i), R50 – R0 = 50 aR0 ... (iii) \ t1 = time for the charge to reduce to of its initial value
2
From (ii), R100 – R0 = 100 aR0 ... (iv)
1
Dividing (iii) by (iv), we get and t2 = time for the charge to reduce to of its
4
R50 - R0 1 initial value
=
R100 - R0 2 We have, q2 = q1e-t / CR
Here, R50 = 5W and R100 = 6W 1 ö -t1
Þ ln æç q2 ö÷ = - t \ ln æç ÷= ...(1)
5 - R0 1 è q1 ø CR è 2 ø CR
\ =
6 - R0 2 æ 1 ö -t2
and ln ç ÷ = ...(2)
or, 6 – R0 = 10 – 2 R0 or, R0 = 4W . è 4 ø CR
39. (b) According to the condition of balancing
æ 1 ö ln æç ö÷
1
ln ç ÷
55 R t 1 è2ø 1
= Þ R = 220W By (1) and (2) , 1 = è 2 ø = =
2ln æç ö÷
2 1 4
ln æç ö÷
20 80 t2 1
40. (a) Let j be the current density. è4ø è 2 ø

Then j ´ 2pr 2 = I Þ j =
I
\ E = rj =
rI 44. (d) R1 = R0 [1 + a1Dt ] ; R2 = R0 [1 + a 2 D t ]
2pr 2 2 pr 2
In Series, R = R1 + R2
'
a r uur a
rI
Now, DVBC =- ò E. dr = - ò 2
dr é æ a1 + a 2
= R0 [ 2 + (a1 + a 2 )Dt ] = 2 R0 ê1 + ç
ö ù
÷ Dt ú
a +b a + b 2 pr
ë è 2 ø û
a
rI é 1 ù rI rI a1 + a 2
=- - = -
2p êë r úû a + b 2pa 2p (a + b) \ a eq =
2
On applying superposition as mentioned we get
1 1 1 1 1
' rI rI In Parallel , = + = R 1 + a Dt + R 1 + a Dt
DVBC = 2 ´ DVBC = -
pa p(a + b)
R R1 R2 0 [ 1 ] 0 [ 2 ]

rI 1 1 1
41. (c) As shown above E= Þ = +
2pr 2 R0 R0 (1 + a1Dt ) R0 (1 + a 2 Dt )
(1 + aeq Dt )
42. (c) Applying kirchoff's loop law in AB P2P1A we get 2
-2i + 5 - 10 i1 = 0 .....(i) a1 + a 2
2(1 - a eq Dt ) = (1 - a1Dt )(1 - a 2 Dt ) \ a eq =
B i P2 i–i1 C 2
i1 45. (a) Resistance of wire
10W 2V
5V 1W rl rl 2
R= = (where Al = C)
2W A C
\ Fractional change in resistance
A P1 D
DR Dl
Again applying kirchoff's loop law in P2 CDP1P2 we =2
R l
get, 10 i1 + 2 – i + i1= 0 .....(ii)
\ Resistance will increase by 0.2%
EBD_7036
P-S- 192 Topic-wise Solved Papers - PHYSICS
46. (c) The current upto which bulb of marked 25W -220V, will Voltage across bulb after heater is switched on,
W1 25 48
not fuse I1 = V = 220 Amp V2 = ´120 = 106.66 volt
1 54
W2 100 Hence decrease in voltage
Similarly, I2 = V = 220 Amp V1 – V2 = 117.073 – 106.66 = 10.04 Volt (approximately)
2
48. (d) Statements I is false and Statement II is true
The current flowing through the circuit
IgG
B1 B2 For ammeter, shunt resistance, S =
I – Ig
Therefore for I to increase, S should decrease, So
R1 r2 additional S can be connected across it.
49. (c) Total power consumed by electrical appliances in the
building, Ptotal = 2500W
440V
Watt = Volt × ampere
440 Þ 2500 = V × I
I= , Reff = R1 + R2
Reff Þ 2500 = 220 I
2500
V 2 (220)2 V 2 (220)2 Þ I= = 11.36 » 12A
R1 = 1 = ; R2 = 2 = 220
P1 25 P 100
(Minimum capacity of main fuse)
440
=
440 l
I= 2 2 50. (b) V = IR = (neAvd )r
(220) (220) é1 1 ù A
+ (220)2 ê +
25 100 ë 25 100 úû
V
\ r = V lne
40 d
I= Amp
220 Here V = potential difference
l = length of wire
æ 25 ö æ 40 ö æ 100 ö
Q I1 ç = A÷ < I ç = A ÷ < I2 ç = A÷ n = no. of electrons per unit volume of conductor.
è 220 ø è 220 ø è 200 ø
e = no. of electrons
Thus the bulb marked 25W-220 will fuse. Placing the value of above parameters we get resistivity
5
r= -19
6W 28
8 ´ 10 ´ 1.6 ´ 10 ´ 2.5 ´ 10-4 ´ 0.1
(Lead) Bulb = 1.6 × 10–5Wm
47. (d) 51. (a) From KVL
– 6 + 3I1 + 1 (Ii – I2) = 0
6V P 2W
120 V

Power of bulb = 60 W (given)


9V
é V ù 2 1W
120 ´120
Resistance of bulb = = 240W êQ P = ú
60 ë R û q 4W
3W
Power of heater = 240W (given)
6 = 3 I 1 + I1 – I2
120 ´120 4I1 – I2 = 6 ...(1)
Resistance of heater = = 60W
240 – 9 + 2I2 – (I1 – I2) + 3I2 = 0
Voltage across bulb before heater is switched on, – I1 + 6I2 = 9 ...(2)
On solving (1) and (2)
240
V1 = ´ 120 = 117.73 volt I1 = 0.13A
246
Direction Q to P, since I1 > I2.
Current Electricity P-S- 193

Alternatively Considering potential at P as 0V and at


Q as x volt, then 2W
p 6V O
3
V x-6 x-0 x+9 P
6V 9V 8
1W 1W + + =0
3 1 5 1W
9V
3W 5W 15 W I
8
2
\ x=
23
d
x-0 2 3W Q 3W
E1 E 2 6 9 \ i= = =0.13A
+ 1 23
-
r1 r2 3 5 3 From Q to P
Eq = 1 1 = 1 1 =
8V
+ +
r1 r2 3 5 r P
52 (a)
3
8 = 3
\ I = 15 23 = 0.13A T T
+1
8 Metal (for limited Samiconductor
–Eg
range of temperature) KT B

qq
EBD_7036
14
P-S- 194 Topic-wise Solved Papers - PHYSICS

Moving Charges and Magnetism

Section-A : JEE Advanced/ IIT-JEE


iL2 m0I é 1 1 ù
A 1. D, B 2. 3. 1.25 × 10–23 Am2 4. ê - ú
4p 4 ë R1 R 2 û
5. IlB; +Z direction 6. evB; ABCD
B 1. T 2. T 3. F 4. F
C 1. (c) 2. (d) 3. (a) 4. (d) 5. (a) 6. (c) 7. (b)
8. (c) 9. (c) 10. (d) 11. (b) 12. (c) 13. (b) 14. (a)
15. (d) 16. (b) 17. (b) 18. (a) 19. (b) 20. (a) 21. (c)
22. (a) 23. (b) 24. (d)
D 1. (a) 2. (a,b,d) 3. (c) 4. (b) 5. (c) 6. (a,b,d) 7. (b, c)
8. (b) 9. (a, c) 10. (a) 11. (d) 12. (c, d) 13. (a,c) 14. (a, c, d)
15. ( b, d) 16. (b, d) 17. (c, d) 18. (a, c) 19. (a, d) 20. (a, b, c) 21. (a,c)
E 1. 25.98 N 3. 0.1T, Directed perpendicular to the plane of paper inwards.
4. (i) 0.1414 m, 45° (ii) 4.71 × 10–8 sec.
5. 0.012 m, 0.044 m 6. (i) 3A, upward direction (ii) 1.3 × 10–6 T (iii) 28.8 × 10–7 N
7. 10–4 tesla; directed towards the reader perpendicular to the plane of paper.
8. (i) 4 A. (ii) r = 1m where r is the distance from R
0.11m 0 IQv
9. (a) directed 30° with the negative X-axis (b) zero, 0.614 BIa2 ĵ
ma
m0 2 é a 2 + L2 ù
10. F= 2 I êloge ú directed toward –Z direction, zero.
4p êë a 2 úû
11. 4.737 × 10–3 T 12. 0.2 sec.
he heB uur
13. (i) M = (ii) directed perpendicular to the plane containing n̂ and B .
4 pm 8pm
m0 ˆ pm
( )
d i 2 3 I0 B 2 mv0
14. (i) ± (ii) n = 15. (a) I 0 L B ˆj - î (b) Dt 16. (a) L = (b) -v0i,
3 2 pd pl 2 4 M 2qB qB0
æ -m qv I ö uur uur
17. (a) ç 0 0 ÷ kˆ (b) F1 = 2 BIRiˆ, F2 = 2 BIRiˆ, 4 BIRiˆ
è 4R ø
18. (a) 6.54 × 10–5T (b) 0, Force on arc AC = 0, 8.1 × 10–6N
DT0 1 2Ni0 AB NABp
19. w = 2 20. 21. (a) k = NAB (b) (c) Q
BQr 2 π 2 Ii0
F 1. A-p; B-q, s; C-q, s; D-q, r, s 2. A-q; B-r, s; C-s; D-p, q, r 3. A-q, r; B-p; C-q, r; D-q, s
G 1. (d) 2. (d) 3. (b) 4. (c) 5. (b) 6. (a, d) 7. (a,c)
H 1. (c)
I 1. 7 2. 6 3. 5 4. 3
Section-B : JEE Main/ AIEEE
1. (a) 2. (a) 3. (c) 4. (a) 5. (b) 6. (a) 7. (b)
8. (a) 9. (d) 10. (a) 11. (b) 12. (b) 13. (c) 14. (a)
15. (b) 16. (b) 17. (d) 18. (c) 19. (d) 20. (b) 21. (b)
22. (b) 23. (a) 24. (d) 25. (d) 26. (b) 27. (b) 28. (c)
29. (c) 30. (b) 31. (a) 32. (a) 33. (a) 34. (d) 35. (a)
36. (b) 37. (b) 38. (b) 39. (c) 40. (d) 41. (a) 42. (b)
43. (c) 44. (b)
Moving Charges and Magnetism P-S- 195

Section-A JEE Advanced/ IIT-JEE


A. Fill in the Blanks directed towards reader perpendicular to the plane of paper.
1. According to Fleming's left hand rule, the force on electrons I B
will be towards right (D).
Also, by the same rule we find that the force on proton and
a-particle is towards left. Now since the magnetic force will I A
behave as centripetal force, therefore
mv 2 S R Q P
\ = qvB
r Magnetic field due to current in semi-circular arc
mv m 1 é µ0 I ù
\ =r or rµ SBP = 2 ê 2 R ú
qB q ë 2û
1 4 directed away from reader perpendicular to the plane of
For proton r µ = 1 ; For a-particle r µ = 2
1 2 paper.
\ radius will be more for a-particle
\ a-particle will take path B. 1 é m0 I ù 1 é m0 I ù
\ Net Magnetic field = ê ú- ê ú
2. L 2 ë 2 R1 û 2 ë 2 R2 û
(directed towards the reader perpendicular to plane of paper).

r
m0 I é 1 1 ù
= ê - ú.
4 ë R1 R2 û
5. We may assume current to be flowing in segment EB in
both directions.
Wire of length L is bent in the form of a circle. Then the Net force on the loop EDCBE will be zero. Also force due to
perimeter of the circle segment FE and BA will be zero. Force due to segment EB
ur
L F = I [ Li$ ´ B $j ] = ILBk$
2pr = L Þ r = ur r ur
2p
6. F = q(v ´ B) = ( - e) ( - vi$ ´ B $j ) = evB k$
pL2
L2 NOTE : The direction of flow of electrons is opposite to
\ Area of the circle = pr2 = =
4p 2 4p that of current.
Magnetic moment of a loop in which current i flows is given B. True/ False
by 1. A current carrying coil is a magnetic dipole. The net force
iL2 on a magnetic dipole placed in uniform magnetic field is
M = iA = . zero.
4p
2. NOTE : The magnetic force acts in a direction perpendicular
q ne 1016 to the direction of velocity and hence it cannot change the
3. i= = = × 1.6 × 10–19 = 1.6 × 10–3 A.
t t 1 speed of the charged particle.
M = i × A = i × pr2
æ 1 ö
= 1.6 × 10–3 × 3.14 × 0.5 × 10–10 × 0.5 × 10–10 Therefore, the kinetic energy ç = mv2 ÷ does not change.
= 1.25 × 10–23 Am2 è 2 ø

e 3. The velocity component v2 will be responsible in moving


the charged particle in a circle.

B
+
v
v2 = vsinq q
)

v1= v cos q

4. The magnetic field at C due to current in PQ and RS is zero.


Magnetic field due to current in semi-circular arc QAR The velocity component v1 will be responsible in moving
1 é µ0 I ù the charged particle in horizontal direction. Therefore the
= ê ú charged particle will travel in a helical path.
2 ë 2 R1 û
EBD_7036
P-S- 196 Topic-wise Solved Papers - PHYSICS
4. When a charged particle passes through a uniform magnetic µ0 I 2
field perpendicular to the direction of motion, a force acts B2 = ( 2p - q) (Directed downwards)
4p r
on the particle perpendicular to the velocity. This force acts
Therefore net magnetic field at the centre
as a centripetal force
m 0 I1 q m 0 I 2
B= - (2p - q)
mv 2 4 p r p 4p r
\ = qvB
r E E EA
Also, I1 = = =
R1 rl1 / A rr q
2 mK é P2 ù
r= ê Where K = ú E E EA
qB ëê 2m úû and I2 = = =
R2 rl 2 / A rr (2p - q)
m
\ rµ [for const. K.E. and B] m0 é EA q EA (2p - q) ù
q \ B= ê ´ - ´ ú=0
4p ë r r q r rr (2p - q) r û
Here, q is same for electron and proton
\ rµ m m
3. (a) KEY CONCEPT : r µ
Radius of proton will be more. q

C. MCQs with ONE Correct Answer 1 2 4


\ rp : rd : ra = : : = 1: 2 :1
1. (c) The magnetic field is perpendicular to the plane of the 1 1 1
paper. Let us consider two diametrically opposite Þ ra = rp < rd
4. (d) The magnetic lines of force created due to current will
elements. By Fleming's left hand rule, on element AB
be in such a way that on x – y plane these lines will be
the direction of force will be leftwards and the
perpendicular. Further, these lines will be in circular
magnitude will be loops. The number of lines moving downwards in x – y
dF = I ( d l ) B sin 90° = I ( d l ) B plane will be same in number to that coming upwards
of the x – y plane. Therefore, the net flux will be zero.
X X X X X X X One such magnetic line is shown in the figure.
y
X X B X X CX X X
dl
dF dl dF Magnetic line of
A D
X X X X X X X force
I
X X X X X X X
x
On element CD, the direction of force will be towards
right on the plane of the paper and the magnitude Current loop
will be
dF = I ( d l ) B.
5. (a) FE = qE (Force due to electric field)
These two forces will cancel out.
NOTE : Similarly, all forces acting on the diametrically FB = evB sin q = qvB sin 0 = 0
opposite elements will cancel out in pair. The net force (Force due to magnetic field)
acting on the loop will be zero. E
D
2. (d)
+q
I2 B
2 p-q
Force due to electric field will make the charged particle
A B
released from rest to move in the straight line (that of
q
electric field). Since the force due to magnetic field is
zero, therefore, the charged particle will move in a
C
I1
straight line.
6. (c) The angular momentum L of the particle is given by
I
E
L = mr2w where w = 2pn.
w wq
Magnetic field at the centre due to current in arc \ Frequency n = ; Further i = q × n =
2p 2p
ABC is wq
Magnetic moment, M = iA = ´ pr 2 ;
m 0 I1 2p
B1 = q (Directed upwards)
4p r wqr 2 M wqr 2 q
Magnetic field at the centre due to current in arc \M= So, = =
2 2
L 2 mr w 2 m
ADB is
Moving Charges and Magnetism P-S- 197

7. (b) The wires at A and B are perpendicular to the plane of Case of negatively charged particle.
paper and current is towards the reader. Let us consider Two forces are acting on the negatively charged particle
certain points. (a) due to electric field in the negative X-direction.
Point C (mid point between A and B) : The magnetic (b) due to magnetic field
ur
field at C due to A ( B CA ) is in upward direction but ur r ur
F = - q(v ´ B)
magnetic field at C due to B is in downward direction.
ur
F = - q[v (-$i ) ´ B (k$ )]
Net field is zero.
Point E : Magnetic field due to A is upward and
ur ur
F = - qvB [i$ ´ k$ ] , F = qvB (- $j )
magnetic field due to B is downward but
ur ur
| B EA | < | B EB | . Same direction as that of positive charge.
\ Net magnetic field is in downward direction. (c) is the correct answer.
ur ur
Point D : | B DA | > | B DB | . Net field upwards. Similarly,, 10. (d) NOTE : If we take individual length for the purpose of
other points can be considered. calculating the magnetic field in a 3-Dimensional figure
then it will be difficult.
BDA Here a smart choice is divide the loop into two loops.
BCA BGB
One loop is ADEFA in y-z plane and the other loop will
BEA
be ABCDA in the x – y plane.
BGA
FB A D C E B G
FB
Z
BDB Y
BFA BCB E

I
BEB I
D C
8. (c) Case 1 : Magnetic field at M due to PQ and QR is F
1 é m0 I ù m I X
H1 = ê ú +0= 0 I
2 ë 2 pR û 4pR
A B
M
We actually do not have any current in the segment
R AD. By choosing the loops we find that in one loop we
to µ to µ have to take current from A to D and in the other one
P I Q S from D to A. Hence these two cancel out the effect of
each other as far as creating magnetic field at the
concerned point P is considered.
to µ The point (a, 0, a) is in the X-Z plane.
R
The magnetic field due to current in ABCDA will be in
Case 2 : When wire QS is joined. + ve Z-direction.
H2 = (Magnetic field at M due to PQ) + (magnetic field
NOTE : Due to symmetry the y-components and x-
at M due to QR) + (Magnetic field at M due to QS)
components will cancel out each other.
1 é m0 I ù 1 é m I / 2 ù 3m 0 I H 2 Similarly the magnetic field due to current in ADEFA
= ê ú +0+ ê 0 ú = \ 1 =
2 ë 2pR û 2 ë 2pR û 8pR H2 3 will be in x-direction.
NOTE : The magnetic field due to an infinitely long \ The resultant magnetic field will be
wire carrying current at a distance R from the end point ur 1
is half that at a distance R from the middle point. B= ($i + k$ ).
9. (c) Case of positively charged particle : 2
Two forces are acting on the positively charged particle 11. (b) KEY CONCEPT : When a charged particle is moving
(a) due to electric field in the positive x-direction. at right angles to the magnetic field then a force acts
(b) Force due to magnetic field.
ur r ur on it which behaves as a centripetal force and moves
F = q(v ´ B) the particle in circular motion.
ur ur
Þ F = q(vi$ ´ Bk$ ) Þ F = qvB ( - $j ) m Av 2A m Av A
\ = q.v A B \ = qB
This forces will move the positively charged particle rA rA
towards Y-axis.
EBD_7036
P-S- 198 Topic-wise Solved Papers - PHYSICS

mB vB y x m I
= qB sin q = , cos q = B= 0
rB B r r 2 pr
y
ur m0 I
Þ
m A v A m B vB
= \ B=
2pr 2
(
yiˆ - xjˆ ) P(x, y)
rA rB r q ®
A B
Since rA > rB
or B=
(
ur m 0 I yiˆ - xjˆ
. O×
) q
x
y
x
Þ mAvA > mBvB B

12. (c) Let us consider a thickness dx of wire. Let it be at a (


2p x 2 + y 2 )
distance x from the centre O. 15. (d) NOTE : Magnetic lines of force form closed loops.
Inside a magnet, these are directed from south to north
pole.
dx 16. (b) The velocity at P is in the X-direction (given).
r
x Let v = ki$.
a After P, the positively charged particle gets deflected
O
in the x – y plane toward – y direction and the path is
b
non-circular.
ur r ur
Now, F = q(v ´ B)
ur
Þ F = q [ki$ ´ (ckˆ + ai$)] for option (b)
N
Number of turns per unit length = = q [kci$ ´ kˆ + kai
ˆ $ ´ $i ] = kcq ( - $j )
b-a
ur
N Since in option (b), electric field is also present E = ai$,
\ Number of turns in thickness dx = dx therefore it will also exert a force in the + X direction.
b-a
Small amount of magnetic field is produced at O due to The net result of the two forces will be a non-circular
thickness dx of the wire. path.
Only option (b) fits for the above logic. For other option,
m 0 NI dx we get some other results.
\ dB =
2 (b - a) x 17. (b) KEY CONCEPT : Use Fleming's left hand rule. We find
On integrating, we get, that a force is acting in the radially outward direction
throughout the circumference of the conducting loop.
b m 0 NI dx m NI uur ur
B = òa = 0
18. (a) U = - M . B = – MB cos q
2 b-a x 2 (b - a )
In case I, q = 180°, U = + MB
b dx m NI
òa x
= 0
2 (b - a )
[loge x] ba In case II, q = 90°, U = 0
In case III, q = acute, U = + ve (less than + MB)
m 0 NI b In case IV, q = obtuse, U = – ve
B= loge
2 (b - a) a \ I > III > II > IV.
19. (b) The force acting on electron will be perpendicular to
13. (b)
the direction of velocity till the electron remains in the
q X magnetic field. So the electron will follow the path as
x=a v x=b given.

y X B
Width of the magnetic field region (b – a) < R; where 'R'
is its radius of curvature inside magnetic field, e
u x
mv (b - a) qB
\ R= ³ (b – a) Þ vmin =
qB m
14. (a) The wire carries a current I in the negative z-direction.
We have to consider the magnetic vector field
ur 20. (a) Use the vector form of B and v in the formulae
B at (x, y) in the z = 0 plane. ur r ur
ur F = q(v ´ B) to get the instantaneous direction of
Magnetic field B is perpendicular to OP.
ur force at x = a and x = 2a.
\ B = B sin qiˆ - B cos qˆj
Moving Charges and Magnetism P-S- 199

21. (c) Let us consider an elemental length dl subtending an


angle dq at the centre of the circle. Let FB be the
magnetic force acting on this length. Then

T cos dq T cos dq R
2 2 r P
R
dq dq 2
2 2
T Tsin dq T
2 Tsin dq
2
I R I
dq dq | B|
2 2

R R r
FB = BI (dl) directed upwards as shown
2
é arc(dl ) ù
= BI (Rdq) êQ angle (dq) = ú
ë radius R û
D. MCQs with ONE or MORE THAN ONE Correct
æ Lö é Lù ur
= BI ç ÷ d q êQ 2pR = L Þ R = 2p ú 1. (a) The force on north pole = mB1
è 2p ø ë û ur
Let T be the tension in the wire acting along both ends The force on south pole = mB 2
of the elemental length as shown. On resolving T, we
æ dq ö B
find that the components. T cos ç ÷ cancel out and
è 2 ø
æ dq ö N
the components. T sin ç ÷ add up to balance FB.
è 2 ø
æ dq ö L S
At equilibrium 2T sin ç ÷ = BI dq
è ø2 2p Since the forces will be unequal and are not having same
dq L é dq ù line of action therefore, the magnetic needle experiences a
Þ 2T
2
= BI
2p
dq êëQ 2 = small úû force as well as a torque.
2. (a,b,d)
BIL There is no change in velocity. It can be possible when
ÞT =
2p • Electric and magnetic fields are absent, i.e., E = 0, B = 0
• Or when electric and magnetic fields are present but
22. (a) same as Q.12 (above) force due to electric field is equal and opposite to the
23. (b) The magnetic moment of a current carrying loop is magnetic force, (i.e., E ¹ 0, B ¹ 0).
r r
given by M = NIA • Or when E = 0. B ¹ 0 provided
2 F = qvB sin q = 0
æ aö é pù
Here N = 1, A = a2 + 2p ç ÷ = a2 ê1 + ú , the direction sin q = 0, i.e., q = 0 Þ v and B are in the same direction.
è2ø ë 2û 3. (c) AB part of the rectangular loop will get attracted to the
is towards positive z-axis. long straight wire as the currents are parallel and in the
r p same direction whereas CD part will be repelled. But
\ M = Ia 2 é1 + ù kˆ
êë 2 úû
1
since this force F µ where r is the distance between
R r
24. (d) For r < , B=0
2 the wires. Therefore, there will be a net attractive force
R on the rectangular loop. Force on BC is equal and
For £ r < R, opposite to that on AD.
2
B I C
µ é R2 ù
B= 0 êr - úJ
2 êë 2r úû I

µ0 i A D
For r > R, B=
2pr
EBD_7036
P-S- 200 Topic-wise Solved Papers - PHYSICS
4. (b) The magnetic field due to current 8. (b) Let us consider any point P
in wire 1 in the region of wire 2 1 2
inside the thin walled pipe. Let
will be us consider a circular loop and P
m 0 2i apply Ampere's circuital law,
i i
B1 =
4p b ur uur
Since wire 2 having current i is Ñò B.d l =µ0I
placed in a magnetic field B1, it Since current inside the loop is zero.
will experience a force given by ur
b \ B =0
F = i (l B1 sin 90°) 9. (a, c)
2 KEY CONCEPT : When the charged particles enter a
F m 2i m i
\ force per unit length =i´ 0 ´ = 0 magnetic field then a force acts on the particle which will act
l 4p b 2p b as a centripetal force. We know that when kinetic energy
é m 0 2i ù and magnetic field are equal then
êQ B = 4 p ´ b ú
ë û m

1 2 q
5. (c) K.E. of first particle = m1v1 = qV ... (i)
2
1 4 16
1 2 r + µ ;r +µ ; r ++ µ
K.E. of second particle = m2 v2 = qV ... (ii) H 1 He 1 O 2
2
NOTE : After entering the magnetic field, a magnetic force Þ r µ 1; r µ 2; r µ2
H+ He + O+ +
acts on the charged particle which moves the charged particle + ++
in circular path of radius He and O will be deflected equally.
H+ will be deflected the most since its radius is smallest.
2m K
R= 10. (a)
qB w
Here, K, q, B are equal
\ R2 µ m
m1 R12 R R
Þ = From (i) and (ii)
m2 R22 (q,m) (q,m)
6. (a, b, d)
Considering the activity from P to Q (Horizontal) æ wö qw
Current, i = (frequency) (charge) = çè ÷ø (2q) =
u1 = v, v1 = 2v, s1 = 2a, Acc = A 2p p
Þ 4v2 – v2 = 2A(2a) Magnetic moment,
3v 2 æ qw ö 2 2
Þ A= M = (i) (A) = çè ÷ø (pR ) = (qwR )
4a p
v E
Force acting in the P Angular momentum, L = 2 Iw = 2(mR2) w
horizontal direction is B
F = qE = mA M qw R 2 q
a \ = =
2
L 2( mR ) w 2m
mA 3 é mv 2 ù
Þ E= = ê ú 11. (d) Net magnetic field due to the wires will be downward
q 4 ëê qa úû r
as shown
ur below in the figure. Since angle between v
Rate of doing work at P 2a Q and B is 180°,
2v
3 é mv3 ù v Z
Power = F × v = mA × v = ê ú
4 êë a úû Y
Rate of doing work by the magnetic field is throughout zero.
The rate of doing work by electric field is zero at Q. Because
at Q, the angle between force due to electric field and I -I
displacement is zero. X
7. (b, c) B Wires are in X-Y plane
For V = Ig (G + R) = 5 × 10–5 [100 + 200,000] = 10V d and velocity in Z-direction
ur r ur
æG ö é100 ù Therefore, magnetic force F m = q(v ´ B) = 0
For I = Ig ç + 1÷ = 5 ´ 10-5 ê + 1ú = 5mA.
èS ø ë 1 û
Moving Charges and Magnetism P-S- 201

12. (c, d) • For the particle to enter region III, r > l (path shown by
Out of the given options only induced electric field and dotted line) Region I Region III
Region II
magnetostatic field form closed loops of field lines.
13. (a, c) Þ
mv
> l Þv>
ql B ××××
Net force on the loop :
qB m ××××
Force on AB : The magnetic field due to current I1 is • For maximum path length in
region II, r = l
× r× × ×
along AB. × ×r × ×
dF = I (d l ´ B ´ sin 0°) = 0 mv qlB
\l =
qB
Þv =
m
××××
C • The time taken by the particle
××××
B to move in region II before ××××
I2
coming back in region I is × v× ×
O
given by
× ×l × ×
O' pm
I1 t= which is independent of v..
qB
A 15. (b, d)
D
When current I is switched on in both the solenoids in
identical manner, eddy currents are setup in metallic rings A
Force on CD : Similarly the magnetic field due to current I1 and B in such a way that rings A and B are repelled.
is along DC. Because q = 180° here, therefore force on DC is
zero.
hA
B1 rA r B hB
I2 C A B

dl
B
Force on BC : Consider a small element d l.
dF = I 2 d l B1sin 90° Þ dF = I 2 d l B1
By Fleming's left hand rule, the direction of this force is Given hA > hB . This shows that eddy currents produced in
perpendicular to the plane of the paper directed outwards. A are greater than in B. This is possible when r A < rB (the
Force on AD : dF = I 2 d l B1sin 90° = I 2 d l B1 rate of change of flux is same in both the rings, therefore
induced emf is same).
B1 16. (b,d)
D r
Figure shows that the megnetic field B is present on the
dl right hand side of AB. The electron (e) and proton (p) moving
on straight parallel paths with the same velocity enter the
A I2 region of uniform magnetic field.
By Fleming's left hand rule, the direction of this force is The entry and exit of electron & proton in the magnetic field
perpendicular to the plane of paper directed inwards. Since makes the same angle with AB as shown.
the current elements are located symmetrical to current I1, Therefore both will come out travelling in parallel paths.
A
therefore force on BC will cancel out the effect of force on
AD. q × × × × ×
Þ Net force on loop ABCD is zero.
Net Torque on the loop : The force on BC and AD will create B
2q × × × × ×
a torque on ABCD in clockwise direction about OO' as seen
by the observer at O.
14. (a, c, d) q
× × × × ×
As the particle enters the magnetic field, a force acts on it p
due to the magnetic field which moves the particle in a circular e– q
×2q × × × ×
path of radius
q
mv
r= × × × × ×
qB
B
EBD_7036
P-S- 202 Topic-wise Solved Papers - PHYSICS
The time taken by proton 19. (a, d)
In the region O < r < R, the magnetic field is present due to
distance arc angle × radius 2q ´ R p
tp = = = = current in solenoid.
speed speed speed v

2q æ m p v ö 2q m p
= v ´ çç eB ÷÷ = eB
è ø
The time taken by electron is
(2p - 2q) R e (2p - 2q) æ me v ö (2p - 2q) me
te = = çè ÷=
v v eB ø eB
In the region r > 2 R, the magnetic field is present due to the
clearly te is not equal to tp as mp >> me
current in the cylinder.
\ (b) , (d) are correct options
For the region R < r < 2R, the magnetic field is neither along
17. (c, d)
When q = 0º, the charged the common axis, nor tangential to the circle of radius r. (a)
particle is projected along x- and (d) are correct options.
y
ur 20. (a, b, c)
axis, due to B the charged ur uur ur
particle will tend to move in E B ( )
F = I éë ò dl ´ B ùû
a circular path in y-z plane
ur r ur
but due to force of electric If B is along z then F = I éë(2 L + 2R )$i ´ B x$ ùû
field, the particle will move
in a helical path with option [A] is correct
x ur r ur
If B is along x then F = I éë(2L + 2R )i$ ´ Bi$ùû = 0
increasing pitch. Therefore
options (A) and (B) are
incorrect. ur ur ur
When q = 10º, we can resolve velocity into two rectangular If B is along y then F = I éë(2L + 2R)$i ´ ˆj ùû
components. One along x-axis (v cos 10º) and one along y- Option (b) and (c) are also correct
axis (v sin 10º). Due to v cos 10º, the particle will move in 21. (a, c)
circular path and due to v sin 10º plus the force due to The range of voltmeter ‘V’ is given by the expression
electric field, the particle will undergo helical motion with its V = Ig [Rc + (Rc + R + R)]
pitch increasing. V is max in this case as RHS is maximum. Thus (a) is correct.
If q = 90º, the charge is moving along the magnetic field. The range of ammeter I is given by the expression
Therefore the force due to magnetic field is zero. But the
force due to electric field will accelerate the particle along y- Ig R c 1 1 1 1
axis. I = R + Ig Where = + +
eq R eq R c R R
18. (a, c)
Here Req is minimum and therefore I is maximum. Thus (c) is
The magnetic field should be in the –z direction (Fleming’s
the correct option.
left hand rule) y
O z x E. Subjective Problems
u2 = v
vy 2 q
tan q = = × × × × ×
vy = 2
× 1. 2l = 0.25 m
vx 2 3 × × × × q × ×
× × × × vx = ×
2 3 × 14.4
Also, m × 2l = 14.4 Þ m= = 57.6 A-m2
p × × × × × × 0.25
\q = Q,M×+ × × × × × Torque due to magnetic field
6 ui = u = 4iˆ
× × × × × ×
× × × × × × 3
= pm × B × sin 60° = 14.4 × 0.25 ×
arc speed ´ time 2
Angle = =
radius radius The torque due to the force = F × 0.12

p 4 ´ 10 ´ 10-3 é Mv ù 3
\ = êQ radius = QB ú For equilibrium F × 0.12 = 14.4 × 0.25 ×
2
Þ F = 25.98 N
6 M ´ 4 / QB ë û
If the force F is removed, the torque due to magnetic field
50p M will move the bar magnet. It will start oscillating about the
\B = ur
3Q mean position where the angle between p m and
(a) and (c) are the correct options ur
B is 0.
Moving Charges and Magnetism P-S- 203

2. v2 1014
\ = 1015 Þ r = 15 = 0.1m.
r 10
N
H H Therefore EF = 0.141 m.
W E If the magnetic field is in the outward direction and the
particle enters in the same way at E, then according to
F F S Fleming's left hand rule, the particle will turn towards
clockwise direction and cover 3/4th of a circle as shown in
the figure.

3. The force on electron will be O

45
towards the left plate due to B E
electric field and will be equal + x x x x -
to
x x x x
Fe = eE O
NOTE : For the electron to E
move undeflected between the x x x x 45
O

45O
plates there should be a force F
(magnetic) which is equal to x x x x
the electric force and opposite v
in direction. The force should + x x x x - 3 é 2p r ù
\ Time required = ´ = 4.71 × 10–8 sec.
be directed towards the right e 4 êë v úû
as the electric force is towards
the left.
On applying Fleming's left hand rule we find the magnetic 5.
field should be directed perpendicular to the plane of paper
inwards. Therefore,
Force due to electric field = Force due to magnetic field.
eE = evB
E V /d é Vù
\ B= = êQE= ú v1 is responsible for horizontal motion of proton
v v ë dû
v2 is responsible for circular motion of proton
where V = p.d. between plates
d = distance between plates mv22
\ = qv2 B
r
600 / 3 ´ 10-3 600
\ B= 6
= -3
2 ´ 10 3 ´ 10 ´ 2 ´ 106 mv2 1.76 ´ 10-27 ´ 4 ´ 105 ´ 3
r= = = 0.012 m
B = 0.1 tesla qB 1.6 ´ 10-19 ´ 0.3 ´ 2
4. m = 1.6 × 10–27 kg, q = 1.6 × 10–19 C
B=1T Pitch of helix = v1 × T
v = 107 m/s 2p r 2p r
F = q . v B sin a where T= =
v2 v sin q
(acting towards O by Fleming's left hand rule)
Þ F = qvB [Q a = 90°] 2p r
But F = ma Þ Pitch of helix = v cos q ´
v sin q
q ´ ´
qvB = 2p r cot q = 2 × 3.14 × 0.012 × cot 60° = 0.044 m
\ qvB = ma \ a = F ´ ´
m 6. (i) The magnetic field at P due to current in wire A.
´ ´
-19 7
1.6 ´ 10 ´ 10 ´ 1 O
´ ´
m 0 2I A m0 2 ´ 9.6
= -27 V ´ ´ BA = = ´ (Direction P to M) ...(i)
1.6 ´ 10 V2= ¾
Ö2 4p rAP 4p æ 10 ö
´ o
45
´ çè 2 + ÷ø
11
= 1015 m/s2 E´
V
V 1= ¾
Ö2
´
NOTE : The current in wire B should be in upward direction
Ð OEF = 45° (Q OE act as a radius) so as to cancel the magnetic field due to A at P. (By right
By symmetry Ð OFE = 45° hand Thumb rule)
\ Ð EOF = 90° (by Geometry) The magnetic field at P due to current in wire B
This is the centripetal acceleration
EBD_7036
P-S- 204 Topic-wise Solved Papers - PHYSICS
A ´
µ0 i
B1 =
4p (OR)

1.
6m
10 m directed towards the reader perpendicular to the plane of
2m S 11 paper.
M Magnetic field due to current carrying conductor Q at point
1.2m O is directed towards the reader perpendicular to the plane
BA P BB N
of paper.
B
µ0 i
m0 2I B B2 =
BB = ´ ... (ii) 4p (OS )
4p æ 10 ö
çè ÷ø Magnetic field due to current carrying conductors L and M
11
at O is zero.
From (i) and (ii) \ Resultant magnetic field at O
m0 2 ´ 9.6 m 2I B = B1 + B2
´ = 0´ B
4p æ 10 ö 4 p æ 10 ö (directed towards the reader perpendicular to the plane of
çè 2 + ÷ø çè ÷ø paper)
11 11
m0 i m i m é 1 1 ù
9.6 ´ 11 I B ´ 11 96 Þ B= + 0 = 0 iê +
Þ = Þ IB = = 3A 4p OR 4p OS 4p ë OR OS úû
32 10 32
(ii) The dimensions given shows that é 1 1 ù
SA2 + SB2 = AB2 Þ ÐASB = 90° = 10–7 × 10 × ê + –4
ú = 10 tesla.
ë 0.02 0.02 û
Magnetic field due to A at S
8. (i) The magnetic field (due to current in wire P) at R
m 0 2I A m0 2 ´ 9.6
BSA = . = ´ (Directed S to B)
4p rSA 4p 1.6 m 0 2I p m0 2 ´ 2.5
= ´ = ´
Magnetic field due to B at S 4p rPR 4p 5
m0 2 I B m0 2 ´ 3 µ0
BSB = . = (Directed S to A) = [in the plane of paper downwards]
4p rSB 4p 1.2 4p
The resultant magnetic field 2.5 amp I amp 5
× × e v = 4X10 m/s
2 2 µ0 æ 9.6 ö 2 æ 3 ö 2 P Q R +X
B= BSA + BSB = ç ÷ + çè ÷
4p è 0.8 ø 0.6 ø 2m

= 10–7 × 13 = 1.3 × 10–6 T rPR=5m


(iii) Force per unit length on wire B
Similarly, the magnetic field (due to current is wire Q) at R
m0 2I A I B
= µ0 2 ´ I µ0
4p rAB = ´ = I
4p 2 4p
[in the plane of paper downwards]
10-7 ´ 2 ´ 9.6 ´ 3
= = 28.8×10–7 N/m The total magnetic field at R [due to P and Q]
2
µ 0 µ0 µ
This force will be repulsive in nature. B= + I = 0 (1 + I )
4p 4 p 4p
¥
[in the plane of paper downwards]
7. i
The force experienced by the electron
Q
¥ L R M F = qvB sin q
i O S i ¥
P µ0
i = evB sin 90° = 1.6 × 10–19× 4 × 105 × (1 + I)
4p
But F = 3.2 × 10–20 N (Given)
to¥
\ 3.2 × 10–20 = 1.6 × 10–19 × 4 × 105 × 10–7 (1 + I)
\ Magnetic field due to current carrying conductor P at Þ I = 4 amp.
point O is
Moving Charges and Magnetism P-S- 205

(ii) Let us consider a position between Q and R. The Let Ð BPO = ÐAPO = q Y
magnetic field produced should be equal to 5 × 10–7 T in the ur µ ur
2I
plane of paper acting upwards. | B1 | = 0 = | B2 | B
4p a 2 + x 2

Öa
For this let the wire having current 2.5 amp be placed at a

2
a

+x
xq

2
distance r from R and current flowing outwards the plane of On resolving B1 and B2 we get P
B1sinq
X
paper. that the sin q components O
B 2sinq C
q
q
cancel out and the cos q -a
µ0 2 ´ 2.5
\ 5 × 10–7 = ´ or r = 1 m components add up. B2
B2cosq
B1
4p r Therefore, the total magnetic
Z A
B1cosq
Let us consider another position beyond R collinear with P, field at P is
Q and R. Let it be placed at a distance r' from R, having B = 2B1 cos q
current in the plane of paper. 2µ0 2I x m0 4 Ix
= 4p ´ =
m0 2 ´ 2.5 2
a +x 2
a +x 2 4 p
2 (a + x 2 )
2
\ 5 × 10–7 = ´ or r' = 1 m
4p r' (towards – Y direction)
Let us consider a small portion of wire OC at P of length dx.
ur m 2I 3 $ ur m 2pI $ The small amount of force acting on that small portion
9. (a) B1 = 0 ( -k ); B2 = 0 k ur ur ur
4p a 4p 3a dF = I (dx ´ B) \ dF = I dx B sin 90°
ur ur ur m I é2 ù -m 0 2 I m0 4 Ix
B = B1 + B 2 = 0 ê - 2 3 ú k$ = (1.4) (k$ ); Þ dF = I dx × ´
4p a ë 3 û 4p a 4p ( a 2 + x 2 )
r
v = v cos 60$i + v sin 60 $j Þ dF =
m0 2 xdx
4I
4p (a 2 + x 2 )
ur ur ur év 3 ˆù é -m0 2.8I ˆ ù
F = Q (V ´ B ) = Q ê $i + vjú ´ k The total force
ë2 2 û êë 4p a úû m0 L xdx
F= ´ 4 I2ò
ur 4p 0 (a + x 2 )
2
r F
Now apply a = L
m m0 é1 ù
= ´ 4 I 2 ê log e (a 2 + x 2 ) ú
(b) KEY CONCEPT : The torque acting on the loop in the 4p ë2 û0
magnetic field is given by
r uur ur m0 é a2 + L2 ù
t=M ´B where M = IA Þ F= ´ 2 I 2 êloge ú
4p ëê a2 ûú
A = (area of PMQNP) – (area of triangle PMN)
To find the direction of force we can use Fleming's left hand
1 1 ur
= (p a 2 ) - ´ MN ´ PS rule. The direction of F is towards – Z direction.
3 2
When the current in wire B is reversed, the resultant magnetic
field at any arbitrary point P on OC will be in the X-direction.
p a2 1 a ép 3ù
= - ´ 3a ´ = a 2 ê - ú Since the current is also in X-direction, therefore force acting
3 2 2 ë3 4 û will be zero (F = I l B sin q and q = 180°).
11. (a) Let us resolve the velocity into two rectangular
ur 2 ép 3ù $ components v 1 (= vcos 60°) and v 2 (= vsin 60°). v 1
A = a ê - úk
ë3 4 û component of velocity is responsible to move the charge
particle in the direction of the magnetic field whereas v2
r ép 3ù $ $ component is responsible for revolving the charged particle
\ t = Ia 2 ê - ú k ´ iB in circular motion. The overall path is helical. The condition
ë3 4 û for the charged particle to strike S with minimum value of B
is that Pitch of Helix = GS
r æp 3ö $ 2$
t = BIa 2 ç - ÷ j = 0.614 BIa j 2p m S B
è3 4 ø T × v1 = GS Þ × v cos 60° = 0.1
qB
The force acting on the loop is zero.
2 p mv cos 60° r = 0.1m
10. The magnetic field produced at different points on OC will B= v1 = v cos 60°
be different. Let us consider an arbitrary point P on OC q ´ 0.1
v
which is at a distance x from the origin. Let the magnetic 1 2E G
field due to currents in A and B at P be B1 and B2 respectively, But mv2 = E Þ v = v2 = v sin 60o
2 m
both being in the X-Y plane.
EBD_7036
P-S- 206 Topic-wise Solved Papers - PHYSICS

2pm 2E 2p g
\ B= ´ ´ cos 60° Þ =
q ´ 0.1 m T r

2p 2 ´ 3.14 Þ T = 2p
r 0.01
= ´ 2mE ´ cos 60° = = 2p = 0.2sec
q ´ 0.1 1.6 ´ 10-19 ´ 0.1 g 9.8
1 13. (i) KEY CONCEPT : Orbital magnetic dipole moment M = IA
= 2 ´ 9.1 ´ 10 -31 ´ 2 ´ 103 ´ 1.6 ´ 10 -19 ´ where I is the current due to orbital motion of electron and A
2 is the area of loop made by electron.
149.8 e ew
= -19
×0.316 × 10–23 = 47.37 × 10–4 Þ M= ´ pR 2 Þ M = ´ pR 2
10 T 2p
= 4.737 × 10–3 T
1 R
12. When AB is steady, Þ M= ewR 2 w
Weight per unit length = Force per unit length 2
But according to Bohr's postulate e-
m0 2 I1I 2
Weight per unit length = ... (i) nh nh
4p r mRw2 = Þ Rw2 =
NOTE : When the rod is depressed by a distance x, then the 2p 2 pm
force acting on the upper wire increases and behaves as a e nh nhe eh
restoring force Þ M= ´ = = (Q n = 1 for ground state)
2 2 pm 4 pm 4 pm
Fmag NOTE : The direction of magnetic momentum is same as
A B the direction of area vector, i.e., perpendicular to the plane
I1 = 20A of orbital motion.
x
A' (ii) KEY CONCEPT : We know that torque
r = 0.01m mg B' r uur ur
t = M ´ B Þ t = MB sin q
where q is the angle between M and B
C I2 = 30A D
B
m 0 2 I1 I 2 m0 2 I1 I 2 n̂ t
Restoring force/length = - 30
o
4p r - x 4p r
m0 é 1 1ù
= 2 I1I 2 ê - ú
4p ë r - x rû
m0 é r - (r - x ) ù e–
Þ Restoring force/length = 2 I1I 2 ê ú
4p ë ( r - x) r û
m0 2I1I2 x
= he heB
4p r (r - x) Þ t= ´ B sin 30° =
4pm 8pm
When x is small i.e., x << r then r = x » r
NOTE : The direction of torque can be found by right hand
m 0 2 I1I 2
Restoring force/length F = x thumb rule.
4p r 2 The direction of torque is perpendicular to the plane
Since, F µ x and directed to equilibrium position. ur
\ The motion is simple harmonic containing n$ and B as shown.
14. (i) KEY CONCEPT : Magnetic field due to an infinitely
µ0 2I1 I 2
\ = (mass per unit length) w2 ... (ii) long current carrying wire at distance r is given by
4p r 2
m0 æ 2i ö
m 0 2I1I 2 B= ç ÷
From (i), (Mass per unit length) × g = 4p è r ø
4p r
The direction of B is given by right hand palm rule.
m 0 2I1 I 2 Hence, in case of three identical wires, resultant field can be
Mass per unit length = ... (iii)
4p rg zero only if the point P is between the two wires, otherwise
From (ii) and (iii) field B due to all the wires will be in the same direction and
m 0 2 I1I 2 m0 2I1I 2 so resultant B cannot be zero. Hence, if point P is at a
g
= ´ w2 Þ w = distance x from the central wire as shown in figure, then,
4p r 2 4p rg r
Moving Charges and Magnetism P-S- 207
ur ur ur ur ur
B P = B PA + B PB + B PC 15. (a) As the magnetic field B is in x – y plane and subtends
ur an angle of 45° with the x-axis, hence,
where B PA = magnetic field at P due to A
ur Bx = B cos 45° = B / 2 and
B PB = Magnetic field at P due to B
By = B sin 45° = B / 2
ur
B PC = Magnetic field at P due to C. Y
Io
S R
ur m é 1 1 1 ù $
B P = 0 2i ê + - (- k ).
4p ë d + x x d - x úû B
ur t
A 45o
For B P = 0 , we get x = ± d 3 X
(ii) KEY CONCEPT : The force per unit length between
two parallel current carrying wires is given by
m 0 2i1i2 P Q
= f (say)
4p r So, in vector from
and is attractive if currents are in the same direction. ur æ B ö $æ B ö uur
B = i$ ç + jç 2$
Y
C
è 2 ÷ø è 2 ÷ø and M = I = I 0 L k
i
2
A B
r uur ur æ B $ B $ ö = I0 L B ˆ ˆ
P So, t = M ´ B = I0 L2 k$ ´ ç
è 2
i+ j÷ ( j -i)
O x
X 2 ø 2

Z i.e., torque has magnitude I 02 L2 B and is directed along


d d
line QS from Q to S.
d d (b) According to the theorem of perpendicular axes,
A B
X
moment of inertia of the frame about QS.
r 1æ4
1 ö 2
f
Z
f IQS = I z = ç ML2 ÷ = ML2
q q 2 2è3 ø 3
B Also t = Ia,
Z
t I 0 L2 B ´ 3 3 I 0 B
So, when the wire B is displaced along Z-axis by a small \ a= = =
F
I 2ML2 2 M
distance Z, the restoring force per unit length on the Here a is constant, therefore we can apply
l
wire B due to wires A and C will be 1
q = w 0t + a t 2 with w0 = 0, we have
F m 2i i z é zù 2
= 2 f cos q = 2 0 1 2 ´ êë as cos q =
l 4p r r r úû 1 2 1 æ 3I 0 B ö
q= at = ç ( Dt ) 2
2 2 è 2 M ÷ø
F m0 4i 2 z
or = . [as I1 = I2 and r2 = d2 + z2]
l 4p ( d 2 + z 2 ) 3 I0B
or q= ( Dt ) 2
2 4 M
F m æ 2i ö 16. KEY CONCEPT : This question involves a simple
or = - 0 ç ÷ z [as d >> z and F is opposite to z] ...(1)
l 4p è d ø understanding of the motion of charged particle in a magnetic
Since F µ – z, the motion is simple harmonic. field.
Comparing eq. (1) with the standard equation of S.H.M. (a) × × × × ×
which is A × × × × ×
× × × × vo
F m 30°
F = – mw2z i.e., = - w2 z M × × × × ×
P
l l × × × × × B = B0 kˆ
= – l w2 z, we get × × × × ×
B X
O × × × × ×
µ0 4i 2 µ0i 2 × × × × ×
2
lw = ´ Þ w=
4p d 2 pd 2 l × × × × ×
× × × × ×
i m0 i m0 × × × × ×
Þ 2pn= Þ n= × × × × ×
d pl 2pd pl X=0 X=L
EBD_7036
P-S- 208 Topic-wise Solved Papers - PHYSICS
Let the particle emerge out from the region of magnetic field 18. For finding the magnetic field produced by this circuit at the
r centre we can consider it to contain two semicircles of radii,
at point P. Then the velocity vector v 0 makes an angle 30°
with x-axis. The normal to circular path at P intersects the r1 = 0.08 m and r2 = 0.12 m. Since current is flowing in the
negative y-axis at point A. same direction, the magnetic field created by circular arcs
Hence, AO = AP = R = radius of circular path, which can be will be in the same direction and therefore will be added.
found as
m0 i m0 i m 0i é 1 1 ù
mv02 mv0 \ B1 = and B2 = 4r \ B = 4 ê r + r ú
= B0 qv0 Þ R = qB ... (i) 4r1 2 ë 1 2û
R 0
\ B = (6.54 × 10–5) T Directed outwards.
R (Right hand thumb rule)
In DAPM, R sin 30° = L Þ =L ... (ii)
2 (b) Force acting on a current carrying conductor placed in
a magnetic field is given by
mv0 ur r ur
From (i) and (ii), L = 2qB . F = I (l ´ B) = I lB sin q
0
(i) For force acting on the wire at the centre
(b) - v0 iˆ In this case q = 180°
\ F=0
(ii) On arc AC due to current at the centre
ur m0 I
| B | on AC will be B = 2 p r
1
v0 iˆ X
2.1mv0 /2qB0 The direction of this magnetic field on any small
segment of AC will be tangential
As the new region of magnetic field is 2.1 L \ q = 180° Þ F = 0
2.1R (iii) On segment CD.
= which is obviously > R.
2 Force on a small segment dx distant r from O
Thus, the required velocity = -v0 $i.
Since the time period for complete revolution = 2pm/qB0.
The time taken by the particle to cross the region of magnetic r2= 0.12m D D
dx
field = pm/qB0.
ur
17. (a) Magnetic field ( B) at the origin = Magnetic field due O
to semicircle KLM + Magnetic field due to other semicircle r1= 0.08m C B
KNM. x
ur m 0 I m I
Therefore, B = ( -i$) + 0 ( $j )
4R 4R
ur m I m I m I dF = I dxB
Þ B = - 0 $i + 0 $j = 0 ( -i$ + $j )
4R 4R 4R m0 I 5m 0 I dx
ur = 10 × dx × =
[NOTE : The magnetic field B due to a circular current 2p x p x
m0 I On integrating
carrying loop is \ For semicircle it is half]
2R 5 µ0 I r2 dx 5 m0 I
Therefore, magnetic force acting on the particle. \ F=
p òr1 x
\ F=
p
[ log e x ] r2
r
1
ur r ur ì m Iü
F = q (v ´ B) = q í( - v0 i$) ´ (-$i + $j ) ´ 0 ý 5 m0 I r 5m ´ 10 æ 0.12 ö
î 4R þ log e 2 = 0 log e ç
\ F= p r1 p è 0.08 ÷ø
-m 0 qv0 I $
= k = 8.1 × 10–6 N
4R directed downwards (By Fleming left hand rule).
ur ur ur
(b) F KLM = F KNM = F KM 19. When the ring is not rotating
ur Wt. of ring = Tension in string
and F KM = BI (2 R)$i = 2 BIRi$
mg = 2T0
ur ur
Therefore, F1 = F 2 = 2BIRi$ or total force on the loop, mg
ur ur ur ur \ T0 = .... (i)
F = F 1 + F 2 Þ F = 4B I R iˆ 2
Moving Charges and Magnetism P-S- 209
d

mv 2p mv p m
evpB = or rp = Þ ra =
rp eB eB
T1 T2
2eV 1 2mV 1 4mV
w0 = and ra =
P m B e B e
O
B rp 1
\ =
ra 2
When the ring is rotating, we can treat it as a current carrying 21. (a) The torque acting on a rectangular coil placed in a
loop. The magnetic moment of this loop
uniform magnetic field is given by,
Q Q r uur ur
M = iA = × pr2 = w × pR2 t = M ´ B Þ t = MB sin q
T 2p
This current carrying loop will create its own magnetic field But M = N i A and q = 90° (for moving coil galvanometer)
which will interact with the given vertical magnetic field in \ t = N i A B sin 90°
such a way that the tensions in the strings will become Þ t=NiAB
unequal. Let the tensions in the strings be T1 and T2. But t = k i (given)
For translational equilibrium \ k i = NiAB
T1 + T2 = mg ... (ii)
Þ k = NAB
Torque acting on the ring about the centre of ring
r uur ur (b) The torsion constant is given by
t= M ´B
t = M × B × sin 90° t NiAB
C= =
q q
Q QwBR 2
= w ´ pR 2 ´ B = Here given that when i = i0, q = p / 2
2p 2
NOTE : For rotational equilibrium, the torque about the 2 N i0 A B
centre of ring should be zero. \ C= ... (i)
p
D D QwBR 2 (c) We know that angular Impulse
\ T1 × - T2 ´ =
2 2 2
= ò tdt = ò NiAB dt = NAB ò i dt
QwBR 2
Þ T1 – T2 = ... (iii) = NABQ ... (ii)
D This angular impulse creates an angular momentum
On solving (ii) and (iii), we get
mg QwBR 2 ò t dt = I w ... (iii)
T1 = +
2 2D From (ii) and (iii)
3T0 NABQ
But the maximum tension is I w = NABQ Þ w =
2 I
3T0 Qw max BR 2 é mg ù This is the instantaneous angular momentum due to which
\ = T0 + êëQ T0 = 2 úû the coil starts rotating. Let us apply the law of energy
2 2D
conservation to find the angle of rotation.
DT0 Rotational kinetic energy of coil
\ wmax =
BQR2
1 2 1 I N 2 A2 B 2Q 2 N 2 A2 B 2Q 2
20. KEY CONCEPT : = Iw = =
2 2 I2 2I
1 2 1
eV = mv p and eV = mva2
2 2 1 2 N 2 A2 B 2Q 2
V is the potential difference C qmax =
vp = velocity of proton 2 2I
va = velocity of a-particle
m = mass of proton, mass of a-particle = 4 m N 2 A2 B 2Q 2 N 2 A2 B 2Q 2
Þ q2max = = ´p
2eV 2eV CI 2 Ni0 ABI
Þ vp = , va =
m 4m
p NAB Q 2 NAB p
Now when the particles enter in magnetic field, the force on Þ q2max = Þ qmax = Q .
proton is 2i0 I 2 Ii0
EBD_7036
P-S- 210 Topic-wise Solved Papers - PHYSICS
F. Match the Following D : q, s
1. (A) Charge on ring will create electric field which is time Reason : The direction of magnetic field at P due to current
independent. in loop A is perpendicular to the plane of paper directed
(B) The rotating charge is like a current. This will create a vertically upwards.
magnetic field and a magnetic moment. The direction of magnetic field at P due to current in loop B
(C) Since net charge is zero there will be no time is perpendicular to the plane of paper directed vertically
independent electric field. The current produces downward.
magnetic field and magnetic moment. Since the current are in opposite direction the wires repel
(D) A changing magnetic field will be produced. This will each other. But net force on each wire is zero.
create a induced electric field. Also a changing magnetic
moment will be produced. G. Comprehension Based Questions
2. A: q 1. (d) The magnetised coils running along the track repel large
Reason : When a charged capacitor is connected to the
magnets on the train's under carriage.
ends of the wire, a variable current (decreasing in magnitude
with time) passes through the wire (shown as resistor) and 2. (d) Initial cost will be more.
thermal energy is generated. The potential difference across 3. (b) The magnetic force will pull the vehicle.
the wire also decreases with time. The charge on the
capacitor plate also decreases with time. 4. (c) B
B : r. s
Reason : e = Blv B2 sin q
B1 sin q
When B, l, v are constant, e is constant
B2 q q B1
Þ A constant potential difference develops across the
ends of the wire and charges of constant magnitude appear (90º – q)
at the ends of the wire. B1cos q + B2 cos q
C:s 2 2
Reason : The free electrons move under the influence of a +h
h
electric field opposite to the direction of electric field. This
movement of e– continues till the electric field inside the q
wire is zero. Wire 1 a a Wire 2
Þ Changes of constant magnitude appear at the ends of
the wire. Magnetic field due to current carrying loop =
D : p, q, r Magnetic field due to straight wires
Reason : Since, E, R are constant, a constant current flows B = B1cos q + B2cos q = 2 B1cos q
in the wire. Due to heating effect of current, thermal energy
is generated in the wire. Also a constant potential difference m0 Ia 2 é m0 I ù a
= 2ê ú´
3.
develops between the ends of the wire.
A : q, r (
2 a2 + h )
2 3/ 2
ëê 2p a + h úû
2 2
a2 + h2

Reason : The magnetic field at P due to current flowing in


Þ h » 1.2a
AB is perpendicular to the plane of paper acting vertically
The current is from P to Q in wire 1 and from R to S in
downward. And the magnetic field at P due to current flowing
wire 2.
in CD is perpendicular to the plane of paper acting vertically
5. (b) We know that torque
upwards.
® ® ®
Therefore, q is correct. t = M´ B
As P is the mid point, the two magnetic fields, cancel out t = MB sin q
each other. Therefore, r is correct.
B:p
Reason : The magnetic field at P due to current in loop A is
( é m Iù
)
= I ´ pa 2 ´ ê2 ´ 0 ú sin 30°
ë 2 pd û
along the axial line towards right. Similarly, the magnetic
field at P due to current in loop B is also along the axial line m0 I 2a2
=
towards right. 2d
C : q, r 6. (a, d) When megnetic force balances electric force
Reason : The magnetic field due to current in loop A at P is FB = FE
equal and opposite to the magnetic field due to current in q vd B = q E
loop B at P.
Moving Charges and Magnetism P-S- 211

V In DPRM,
\ vd B = [Q V = E × w] 16x2 = PM 2 + (5x – a)2 … (iii)
w
Þ 2
é I ù 7 x 2 = 25 x 2 - 10 xa Þ 10xa = 18x
\ V = wvdB = w ê ú´B Þ a = 1.8 x … (iv)
ë newd û
From (ii) & (iv),
é I I ù
êvd = neA = newd ú 9 x 2 = PM 2 + (1.8 x)2
ë û
I 2
\ V = ´B Þ PM = 9 x 2 - 3.24 x 2 = 5.76 x = 2.4 x … (v)
ned
1 a 1.8 x
\ Vµ Þ V1d1 = V2d2 Also cos q1 = = = 0.6 … (vi)
d 3x 3x
when d1 = 2d2, V2 = 2V1
5 x - a 5 x - 1.8 x 3.2
and when d1 = d2, V2 = V1 cos q2 = = = = 0.8 …(vii)
(a), (d) are correct options 4x 4x 4
7. (a,c) From (i), (v), (vi) and (vii),
Here
m0 I m I ém I ù

B Vn V n
Þ 11= 2 2 B= ´ [ 0.6 + 0.8] = 4p0 ´ 2.4 x ´ 1.4 = 7 ê 480px ú
n B1 B2 4 p 2.4 x ë û
If B1 = B2 and n1 = 2n2 Þ V2 = 2V1
and of B1 = 2B2 and n1 = n2 Þ V2 = 0.5V1 é m I ù
Comparing it with B = k ê 0 ú , we get, k = 7.
A and C are the correct options. ë 48p x û

H. Assertion & Reason Type Questions 2. (6) Let us consider an amperian loop ABCD which is a
rectangle as shown in the figure.
θ NBA Applying ampere’s circuital law we get
1. (c) Statement-1 is true. Sensitivity = = . If B
I C r uur
Ñò B.d l = mo × (current passing through the loop)
θ
increases, increases. Statement-2 is wrong because r uur æIö
Ñò
I B.d l = mo ç ÷ ´ l
\
soft iron can be easily magnetised and de magnetized. è Lø
D C
L
I. Integer Value Correct Type I
\ B × l = mo ´l l
1. (7) L
A B
The right angled triangle is shown in the figure. Let us drop
a perpendicular from P on QR which cuts QR at M.
mo I mo
The magnatic field due to currents in PQ and RP at P is zero. \ B= = Io cos (300 t)
L L
The magnetic field due to current in QR at P is
The magnetic moment of the loop
= (current in the loop) × pr2
Q a
q1 1 æ dfö 2
M = ç - ÷ ´ pr
R è dt ø
I (5x – a)
1 éd ù p 2 r 4 dB
3x =- ê ( B ´ pr 2 ) ú ´ pr 2 = -
R ë dt û R dt
q2
P R é p2 r 4 mo ù
4x
= ê R ´ L I o sin(300 t ) ú ´ 300
ëê ûú
m0 I Comparing it with the expression given in the
B= (cos q1 + cos q2 ) …(i)
4p PM question we get
In DPQM,
300p2 r 4 1 300(3.14)2 ´ (0.1)4
9x2 = PM2 + a2 … (ii) N= ´ = =6
R L 0.005 ´10
EBD_7036
P-S- 212 Topic-wise Solved Papers - PHYSICS

current I I' mv
3. (5) Current density J = = = 4. (3) R=
area p(2a) 2
( )
pa 2 qB

R1 B2
I = [... m, q, v are the same]
Þ I' = R2 B1
4
Let us consider the cavity to have current I' flowing in
µ0 é 1 1 ù
both the directions. ´ 2I ê + ú
The magnetic field at P due to the current flowing R1 4p ë X1 X 0 - X1 û
=
through the cylinder R2 µ0 é 1 1 ù
´ 2I ê - ú
4p ë X1 X 0 - X1 û
µ0 2 I
B1 =
4p a
X 0 - X1 + X1 X0
The magnetic field at P due to the current (I’) flowing = X - X - X = X - 2X
in opposite direction is 0 1 1 0 1

µ 0 3I ' µ 2( I /4) µ0 I X0
B2 = = 0 =
4 p 3 a /2 4 p 3 a /2 4 p 3a R1 X1 3
\ = = =3
\ The net magnetic field is R2 X 0 3- 2
-2
X1
µ0 I é 1ù µ I 5
B = B1 – B2 = ê 2- ú = 0 ´
4p a ë 3 û 4p a 3

µ0 J p a 2 5 5 Ja
\ B= ´ = µ0
4p a 3 12

Section-B JEE Main/ AIEEE

1. (a) KEY CONCEPT : We know that the magnetic field 3. (c) Magnetic field due to
current in wire 1 at point q i2
produced by a current carrying circular coil of radius r
P distant r from the wire i1
r P
m0 I is
at its centre is B = ´ 2p dl
4p r m 0 i1
B= [ cos q + cos q]
4p r q
m I m0 2I
Here B A = 0 ´ 2p and BB = ´ 2p m 0 i1 cos q
4p R 4p 2 R
B= (directed perpendicular to the plane
2p r
BA of paper, inwards)
Þ =1
BB The force exerted due to this magnetic field on current
element i2 dl is
2. (a) KEY CONCEPT : When a charged particle enters dF = i2 dl B sin 90°
perpendicular to a magnetic field, then it moves in a
é m0 i1 cos q ù m 0
circular path of radius. \ dF = i2 dl ê ú= i1 i2 dl cos q
ë 2p r û 2pr
p 4. (a) KEY CONCEPT : The time period of a charged particle
r=
qB
(m, q) moving in a magnetic field (B) is T = 2pm
where q = Charge of the particle qB
p = Momentum of the particle The time period does not depend on the speed of the
B = Magnetic field particle.
5. (b) The workdone, dW = Fds cosq
Here p, q and B are constant for electron and proton,
The angle between force and displacement is 90° .
therefore the radius will be same. Therefore work done is zero.
Moving Charges and Magnetism P-S- 213

6. (a) The situation is shown in the figure.


FE= Force due to electric field
FB = Force due to magnetic field
It is given that the charged particle remains moving
along X-axis (i.e. undeviated). Therefore FB = FE N S

E 104
Þ qvB = qE Þ B = = = 103 weber/m2
v 10

Y
Z' 10. (a) KEY CONCEPT : The temperature above which a
ferromagnetic substance becomes paramagnetic is
B
called Curie’s temperature.
11. (b) Using Ampere’s law at a distance r from axis, B is same
E
from symmetry.
FB

v
X
ò B.dl = m0i i.e., B ´ 2pr = m 0i
E
Here i is zero, for r < R, whereas R is the radius
\ B=0
Z
12. (b) KEY CONCEPT : Magentic field at the centre of a
m0i
7. (b) KEY CONCEPT : The time period of a rectangular circular coil of radius R carrying current i is B =
magnet oscillating in earth’s magnetic field is given by 2R
Given : n ´ (2pr ') = 2pR
I
T = 2p Þ nr ' = R ...(1)
mBH
n.m 0i
where I = Moment of inertia of the rectangular B' = ...(2)
2r '
magnet
m = Magnetic moment nm 0 i.n
from (1) and (2), B ' = = n2 B
BH = Horizontal component of the earth’s magnetic 2pR
field 13. (c) The magnetic field at a point on the axis of a circular
loop at a distance x from centre is,
I 1 2
Case 1 : T = 2p where I = M l m 0i a 2 m i
mBH 12 B= B' = 0
2 2 3/ 2 2a
2( x + a )
Case 2 : Magnet is cut into two identical pieces such
that each piece has half the original length. Then
B.( x 2 + a2 )3/ 2
\ B' =
I' a3
T ' = 2p
m ' BH
54(53 )
Put x = 4 & a = 3 Þ B ' = = 250 µT
1 æ M ö æ lö
2
I m 3´ 3´ 3
where I ' = çè ÷ø çè ÷ø = and m ' = 14. (a) Force between two long conductor carrying current,
12 2 2 8 2
m0 2I1I 2
F= ´l
T' I' m I /8 m 1 1 4p d
\ = ´ = ´ = =
T m' I m/2 I 4 2 µ0 2(2 I1 ) I 2 F ' -2
F'= - l \ =
8. (a) W = MB (cos q1 - cos q2 ) 4p 3d F 3

I 1
1
= MB (cos 0° - cos 60°) = MB(1 - ) =
MB 15. (b) T = 2p where I = ml 2
2 2 M ´B 12
When the magnet is cut into three pieces the pole
MB strength will remain the same and
\ t = MB sin q = MB sin 60° = 3 = 3W
2 2
1 æ m öæ l ö I
9. (d) As shown in the figure, the magnetic lines of force are M.I. (I¢) = ´3 =
12 çè 3 ÷ø çè 3 ÷ø 9
directed from south to north inside a bar magnet.
EBD_7036
P-S- 214 Topic-wise Solved Papers - PHYSICS
We have, Magnetic moment (M)
æ aö
= Pole strength (m) × l amperean path formed at a distance r1 ç = ÷
è 2ø
\ New magnetic moment,
æ lö T 2 æ p r2 ö
M ' = m ´ ç ÷ ´ 3 = ml = M \ T ' = = s. = ç 12 ÷ ´ I , where I is total current
è 3ø 9 3 èpa ø
16. (b) NOTE : Electro magnet should be amenable to
magnetisation & demagnetization. \ Magnetic field at P1 is
\ retentivity should be low & coercivity should be m0 ´ current enclosed
low B1 =
Path
17. (d) The magnetic field due to circular coil 1 and 2 are
æ p r2 ö
µ i m 0 i1 m ´ 3 ´ 102 m 0 ´ ç 12 ÷ ´ I
B1 = 0 1 = = 0
2r 2 (2 p ´ 10 -2 ) 4p è pa ø m ´ I r1
(1) Þ B1 = = 0
2p r1 2p a 2 P2
m0 i2 m ´ 4 ´ 10 2 ®
B2 a
B2 = = 0 Now, magnetic field at point P2, a
2(2p ´ 10 -2 ) 4p ® 2
B1 (2) m0 I m I
m B2 = . = 0 . P1
B= B12 + B22 = 0 . 5 × 102 2p (2a) 4pa
4p
B1 m 0 Ir1 4pa
Þ B = 10 - 7 ´ 5 ´ 10 2 Þ B = 5 × 10–5 Wb / m2 \ Required ratio = = ´
B2 2pa 2 m 0 I
18. (c) Equating magnetic force to centripetal force,
a
mv 2 2 r1 2 ´ 2
= qvB sin 90º = =
r = 1.
a a
Time to complete one revolution,
25. (d) There is no current inside the pipe. Therefore
2pr 2pm ur uur
T= = oò B . d l = µo I
v qB
I=0
19. (d) A magnetic needle kept in non uniform magnetic field
experience a force and torque due to unequal forces \ B=0
r r
acting on poles. 26. (b) Here, E and B are perpendicular to each other and
20. (b) Due to electric field, it experiences force and decelerates r
the velocity v does not change; therefore
i.e. its velocity decreases.
E
21. (b) Ferromagnetic substance has magnetic domains qE = qvB Þ v =
whereas paramagnetic substances have magnetic B
dipoles which get attracted to a magnetic field. Also,
r r
Diamagnetic substances do not have magnetic dipole E´B E B sin q E B sin 90° E r
but in the presence of external magnetic field due to = = = = |v| = v
2 2 2 B
B B B
their orbital motion of electrons these substances are
repelled. 27. (b) NOTE : When a charged particle enters a magnetic
22. (b) The charged particle will move along the lines of electric field at a direction perpendicular to the direction of
field (and magnetic field). Magnetic field will exert no motion, the path of the motion is circular. In circular
force. The force by electric field will be along the lines motion the direction of velocity changes at every point
of uniform electric field. Hence the particle will move in (the magnitude remains constant).
a straight line. Therefore, the tangential momentum will change at
every point. But kinetic energy will remain constant as
i 1
B2 m0 n2i2 100 ´ it is given by mv 2 and v 2 is the square of the
= B2 3
23. (a) 2
B1 m 0 n1i1 Þ =
6.28 ´ 10 -2 200 ´ i magnitude of velocity which does not change.
28. (c) Clearly, the magnetic fields at a point P, equidistant
6.28 ´ 10-2 from AOB and COD will have directions perpendicular
Þ B2 = = 1.05 ´ 10-2 Wb / m2
6 to each other, as they are placed normal to each other.
24. (d) Here, current is uniformly distributed across the cross-
\ Resultant field, B = B12 + B22
section of the wire, therefore, current enclosed in the
Moving Charges and Magnetism P-S- 215

m 0 I1 m I The component dB cos q, of the field is cancelled by


But B1 = and B2 = 0 2 another opposite component.
2pd 2pd P Therefore,
A D
B1 B2
2
æ m ö
\ B= ç 0 ÷
è 2pd ø (I
2
1 + I 22 ) I1 d I2

(
m0 2
)
1/ 2
or, B = I1 + I 22
2pd C B dB
29. (c) The magnetic field is m0 I
p
m0I
m 2I
W N Bnet = ò dB sin q = ò
2p 2 R 0
sin qd q =
p2 R
B= 0
4p r 100A 35. (a) The magnetic field due a disc is given as
2 ´ 100 h0 w Q 1
= 10 -7 ´ B= i.e., B µ
4 2 pR R
–6
= 5 × 10 T 4m
2mv m
According to right hand S E 36. (b) r= Þr ´v
qB q
palm rule, the magnetic Ground
field is directed towards B Thus we have, ra = rp < rd
south.
30. (b) For a diamagnetic material, the value of µr is less than 37. (b) Given : M 1 = 1.20 Am 2 and M 2 = 1.00 Am2
N
one. For any material, the value of Îr is always greater
20 BH
than 1. r= cm = 0.1m B1
2 S S
31. (a) The magnetic field at O due to current in DA is B2
Bnet = B1 + B2 + BH
O
m I p N N
B1 = o ´ (directed vertically upwards) m ( M1 + M 2 ) r r
4p a 6 Bnet = 0 + BH
4p r3 S
The magnetic field at O due to current in BC is
m I p 10 -7 (1.2 + 1)
B2 = o ´ (directed vertically downwards) = + 3.6 ´ 10 -5 = 2.56 ´ 10 -4 wb/m2
4p b 6 (0.1) 3

The magnetic field due to current AB and CD at O is 38. (b) Work done in moving the conductor is,
zero. 2
W = ò Fdx
Therefore the net magnetic field is 0
B = B1 - B2 (directed vertically upwards) 2
= ò 3.0 ´ 10 -4 e -0.2 x ´ 10 ´ 3dx
0
mo I p m o I p m I æ 1 1ö m I
= - ´ = o ç - ÷ = o (b - a)
4 p a 6 4p b 6 24 è a b ø 24ab
r r r
32. (a) KEY CONCEPT : F = I ( l ´ B) l=3m
The force on AD and BC due to current I1 is zero. This I = 10 A
uur z
is because the directions of current element I d l and
r
magnetic field B are parallel. x
33. (a) The magnetic field varies inversely with the distance
1 -3 -0.2 x 2
for a long conductor. That is, B µ . According to the = 9 ´ 10 ò e dx
d 0
magnitude and direction shown graph (1) is the correct
one. 9 ´ 10-3
= [-e -0.2 ´ 2 + 1] B = 3.0 ´ 10-4 e-0.2 x
dq 0.2
34. (d) Current in a small element, dI = I
(By exponential function)
p
Magnetic field due to the element
9 ´ 10-3
m 2dI = ´ [1 - e -0.4 ]
dB = 0 0.2
4p R
EBD_7036
P-S- 216 Topic-wise Solved Papers - PHYSICS
= 9 × 10–3 × (0.33) = 2.97 × 10–3J r r
Power required to move the conductor is, 41. (a) For stable equilibrium M || B
r r
W For unstable equilibrium M || (–B)
P=
t
2.97 ´ 10-3
P= = 2.97 W 42. (b) Case (a) :
(0.2) ´ 5 ´ 10-3
39. (c) Magnetic field in solenoid B = m0n i
B µ0 I µ I
= ni BA = ´ 2p = 0 ´ 2p (2pR = l)
Þ m0 4p R 4p l / 2p

(Where n = number of turns per unit length)


µ0 I
= ´ (2p)2 45°
B Ni
= 4p l a BB
Þ
m0 L
Case (b) : a/2
100i
Þ 3 ´ 103 =
10 ´ 10-2 µ0 I
Þ i = 3A BB = 4 × [sin 45° + sin 45°]
4p a / 2
40. (d) Let us consider 'l' length of current carrying wire.
At equilibrium
T cos q = lgl µ0 I 2 µ0 I 32
= 4´ ´ ´ = ´ 2 [4a = l]
4p l / 8 2 4p l
q L 43. (c) Ig G = ( I – Ig)s
T \ 10–3 × 100 = (10 – 10–3) × S
T cos q \ S » 0.01 W
Lsin q Lsin q 44. (b) Graph [A] is for material used for making permanent
FB
T sin q magnets (high coercivity)
(ll)g Graph [B] is for making electromagnets and
m 0 I ´ Il é FB m0 2 I ´ I ù transformers.
and T sin q = êQ =
4p 2l sin q úû
2 p 2L sin q ë l
plgL
Therefore, I = 2sin q
u 0 cos q
Electromagnetic Induction and
15
Electromagnetic Induction and Alternating Current P-S- 217

Alternating Current
Section-A : JEE Advanced/ IIT-JEE
A 1. 0.3 × 10–4 sec, 10 A 2. Left to right
B 1. T 2. F 3. T
C 1. (b) 2. (d) 3. (c) 4. (d) 5. (b) 6. (b) 7. (d)
8. (a) 9. (d) 10. (b) 11. (a) 12. (c) 13. (a) 14. (b)
15. (d) 16. (b)
D 1. (a, b, c) 2. (d) 3. (a, c, d) 4. (b) 5. (a, b, c, d) 6. (b) 7. (a, c)
8. (a, c) 9. (a) 10. (c, d) 11. (a,d) 12. (a,b)
E 1. Clockwise 2. 1 mV 3. Yes, opposite direction of A 4. 0.02 m/s, clockwise direction
-1
1 Br 2 w æ R + 2lx ö 2lmI 2 é 2 lmI ( R + 2lx ) ù
5. (i) (ii) anticlockwise 6. (i) V = ç ÷ I , F = BId + ( R + 2lx) , (ii) ê1 + ú
2 R è Bd ø ( Bd ) 2 ë B3 d 3 û

7 6 1
7. (a) –5V, 24.5 W (b) (i) 0.6 amp. (ii) 1.386 × 10–3 sec., 4.5 × 10–4 J 8. amp , amp , amp
22 22 22

Br 2 w 2é - ç ÷t ù
æRö
B 2 r 4 w mgr
B wr ê
9. 1 m/s, 0.47 W, 0.3 W 10. (a) (b) I = ë1 - e è L ø úû , t = + cos wt 11. 3.466 sec
2 2R 4R 2

mgR g B0av(t ) B 2 a 2 v (t )
12. (i) (ii) 13. (a) , anticlockwise (b) - 0 , upward
B 2 L2 2 R R

é - B02 a 2t ù
(c) mgR êë1 - e mR úû ; mgr
2 2
B0 a B02 a 2

14. 12e–5tV, 3e–10tA, clockwise

mg df di 1 é m 0 I 0l ù L T
15. (a) P to Q (b) IbB0 (3kˆ - 4iˆ) (c) I = 16. (a) = iR + L (b) ê loge 2ú - i1 (c)
6aB0 dt dt R ë 2p û R 2 log e 2

m 0 aI 0 w 2C ln(2) p 4 µ0na 2 Ldi0w cos w t


17. (a) 18. 20A, 19. (r1 - r2 ) 20. I=
p 4 3 2rR
F 1. A-r, s, t; B-q, r, s, t; C-p, q; D-q, r, s, t
G 1. (b) 2. (d) 3. (c) 4. (a) 5. (b) 6. (b) 7. (b)
H 1. (a)
I 1. 4 2. 7 3. 8

Section-B : JEE Main/ AIEEE


1. (b) 2. (d) 3. (d) 4. (b) 5. (b) 6. (d) 7. (c)
8. (a) 9. (a) 10. (d) 11. (b) 12. (b) 13. (a) 14. (b)
15. (c) 16. (d) 17. (a) 18. (b) 19. (a) 20. (b) 21. (c)
22. (d) 23. (b) 24. (a) 25. (c) 26. (a) 27. (d) 28. (c)
29. (b) 30. (b) 31. (d) 32. (c) 33. (a) 34. (b) 35. (d)
36. (d) 37. (a) 38. (c) 39. (c) 40. (b) 41. (c) 42. (c)
43. (b)
EBD_7036
P-S- 218 Topic-wise Solved Papers - PHYSICS

Section-A JEE Advanced/ IIT-JEE


A. Fill in the Blanks Since currents are in opposite direction, the loop B is
repelled by loop A.
1. The coil is broken into two identical coils.
L/2´ L/2 L
Leq = = = 0.45 × 10–4 H,
L/2+ L/2 4
R/2´ R/2 R
Req = = = 1.5 W
R/2+ R/2 4 B A
Leq 0.45 ´ 10 -4 4. (d) KEY CONCEPT : Using I = I0 (1 –e–t/t)
Time constant = = = 0.3 × 10–4s V L
Re q 1.5 But I 0 = and t =
E 15 R R
Steady current I = = = 10 A. V 12 -3
Req 1.5 \ I = (1 - e - Rt / L ) = é1 - e -6t / 8.4´10 ù
R 6 ëê úû
2. NOTE : As the source is switched off, the current
decreases to zero. The induced current opposes the cause = 1 (given)
as per Lenz's law. Therefore, the induced current will direct \ t = 0.97 ×10 – 3 s » 1ms
from left to right. ur uur d f d ur ur d dB
5. (b) Ñò E.d l = = ( B. A) = ( BA cos 0°) = A
B. True/ False dt dt dt dt
dB
1. True. A copper wire consists of billions and billions of free Þ E (2 pr ) = pa 2 for r ³ a
electrons. When the wire is at rest, the average velocity of dt
each electron is zero. But when the wire is in motion, the
a 2 dB 1
electrons have a net velocity in the direction of motion. Þ E= ÞEµ
NOTE : A charged particle moving in a magnetic field 2r dt r
r r ur 6. (b) KEY CONCEPT : The magnetic field at the centre of
experiences a force given by F = q( n ´ B). the coil
Here also each electron experiences a force and therefore, B(t) = µ0nI1.
electrons will move towards one end creating an emf As the current increases, B will also increase with time
between the two ends of a straight copper wire. till it reaches a maximum value (when the current
2. NOTE : For induced emf to develop in a coil, the magnetic becomes steady).
flux through it must change. The induced emf in the ring
But in this case the number of magnetic lines of force through
df d ur ur d
the coil is not changing. Therefore the statement is false. e=- = - ( B. A) = - A ( µ0 nI1)
3. NOTE : When conduction rod AB B dt dt dt
moves parallel to x-axis in a uniform Fm – – \ The induced current in the ring
magnetic field pointing in the | e | µ0 nA dI1
e v I 2 (t ) = =
positive z-direction, then R R dt
according to Fleming’s left hand dI1
rule, the electrons will experience + + [NOTE : decreases with time and hence I2 also
a force towards B. Hence, the end A dt
A will become positive. decreases with time.]
Where I1 = Imax (1 – e –t/t)
The relevant graphs are
C. MCQs with ONE Correct Answer
1. (b) The current induced will be
|e| 1 df dq
i= Þ i= But i =
R R dt dt
dq 1 d f 1 BA
Þ = Þ ò dq = ò d f Þ q =
dt R dt R R
2. (d) Induced emf produced across MNQ will be same as the
induced emf produced in straight wire MQ.
\ e = Bvl = Bv × 2R with Q at higher potential.
3. (c) When the current in the loop A increases, the magnetic
lines of force in loop B also increases as loop A is 7. (d) NOTE : Electric field will be induced, as ABCD moves,
placed near loop B. This induces an emf in B in such a in both AD and BC. The metallic square loop moves in
direction that current flows opposite in B (as compared its own plane with velocity v. A uniform magnetic field
to A).
Electromagnetic Induction and Alternating Current P-S- 219

is imposed perpendicular to the plane of the square 12. (c) KEY CONCEPT : Initially, fB increases as magnet
loop. AD and BC are perpendicular to the velocity as approaches the solenoid
well as perpendicular to applied. \ e = – ve and increasing in magnitude. When
magnet is moving inside the solenoid, increase in fB
slow down and finally fB starts decreasing
\ emf is positive and increasing.
Only graph (c) shows these characteristic.
13. KEY CONCEPT : For a current to induce in cylindrical
conducting rod,
(a) the conducting rod should cut magnetic lines of force
which will happen only when the conducting rod is
8. (a) Clearly the flux linkage is maximum in case (a) due to moving. Since conducting rod is at rest, no current will
the spatial arrangement of the two loops. be induced.
(b) the magnitude/direction of the magnetic field changes.
A changing magnetic field will create an electric field
which can apply force on the free electrons of the
conducting rod and a current will get induced.
But since the magnetic field is constant, no current will
be induced.

9. (d) When switch S is closed, a magnetic field is set-up in


the space around P. The field lines threading Q
increases in the direction from right to left. According
to Lenz's law, IQ1 will flow so as to oppose the cause
and flow in anticlockwise direction as seen by E.
Reverse is the case when S is opened. IQ2 will be
clockwise.
10. (b) KEY CONCEPT : 14. (b) KEY CONCEPT :
2 Time constant of R – C circuit is t = Req Ceq
E 2 pr 2 æ d f ö pr 2 é d ù
P= = çè ÷ø = ê ( NBA)2 ú (i) t1 = (2 + 1) (2 + 4) = 18 µs
R rl dt rl ë dt û
æ 2 ´1 öæ 2 ´ 4 ö 8
2 2 2 2 (ii) t2 = ç ÷ç ÷ = µs
pr æ dB ö N r è 2 + 1 øè 2 + 4 ø 9
= N 2 A2 ç ÷ Þ Pµ
rl è dt ø l
æ 2 ´1 ö
(iii) t3 = ç ÷ ´ (4 + 2) = 4µs
N 2r 2 (4 N )2 (r / 2) 2 è 2 +1 ø
Case 1 : P1 µ , Case 2 : P2 µ 15. (d) The magnetic field is increasing in the downward
l 4l
NOTE : When we decrease the radius of the wire, its direction. Therefore, according to Lenz’s law the current
length increases but volume remains the same] I1 will flow in the direction ab and I2 in the direction dc.

P1 1 Vrms
Þ = 16. (b) I rms =
2
P2 1 æ 1 ö
R2 + ç
\ Power remains the same. è wC ÷ø
11. (a) NOTE : Since current leads emf (as seen from the
As w increases, Irms increases. Therefore the bulb glows
graph), therefore, this is an R – C circuit.
brighter.
XC - XL
tan f = D. MCQs with ONE or MORE THAN ONE Correct
R
Here f = 45° 1. (a,b,c)
\ XC = R [XL = 0 as there is no inductor]
1
1 , R/L and 1/ LC have the dimensions of frequency..
= R Þ RCw = 1 RC
wC 2. (d) NOTE : Since the rate of change of magnetic flux is
1 -1 zero, hence there will be no net induced emf and hence
\ RC = s no current flowing in the loop.
100
EBD_7036
P-S- 220 Topic-wise Solved Papers - PHYSICS
\ f2 µ I1 Þ f = MI1where M = Mutual Conductance
3. (a, c, d)
2 2 µ0 I1 2
´l
Rate of change of current = di1 = m( say) f
\ M2 = 2 = pL =
2 2 µ0
´ l2
dt I1 I1 pL
di1
= - 8 ´ 10 -3 ´ m 2
Þ M µl .
Induced emf V1 = - L1
dt
L
V2 1 5. (a, b, c, d)
\ =
V1 4 f weber
Power P = V1 i1 = 8 × 10 – 3 × m ×i1 (a) L= or henry =
i ampere
di2
Rate of change of current = = m (given) æ di ö
dt (b) e = - Lç ÷
è dt ø
di2
Induced emf, V2 = - L2 = – 2 × 10 – 3 × m e volt - second
dt \ L= - or henry =
Power P = V2 i2 = 2 × 10 –3 × m × i2 (di / dt ) ampere
Since Power is equal 1 2
\ 8 × 10 – 3 × mi1 = 2 ×10 – 3 mi2 (c) U= Li
2
i1 1 2U joule
Þ = ...(i) \ L= 2 or henry =
i2 4 i (ampere)2
1 2 1 -3 2 1 2 2
Energy W1 = L1i1 = ´ 8 ´ 10 ´ i1 (d) U = Li = i Rt
2 2 2
\ L = R.t or henry = ohm-second.
W2 10 -3 ´ i22 1
\ = = ´ 4 ´ 4 = 4 [from (i)]
W1 4 ´ 10 -3 ´ i12 4 6. (b) A B
4. (b) KEY CONCEPT : The magnetic field due to a current
flowing in a wire of finite length is given by
µ0 I
B= (sin a + sin b) l
v
4 pR
Applying the above formula for AB for finding the field
at O, we get
µ0 I1 B
B= (sin 45° + sin 45°) = µ0 I1
4p( L / 2) 2 pL A motional emf, e = Blv is induced in the rod. Or we can say
acting perpendicular to the plane of paper upwards a potential difference is induced between the two ends of
\ The total magnetic field due to current flowing the rod AB, with A at higher potential and B at lower potential.
through ABCD is Due to this potential difference, there is an electric field in
the rod.
2
2 æ 1 ö
7. (a,c) We know that Z = R +ç
è WC ÷ø
The capacitance in case B is four times the capacitance in
case A
\ Impedance in case B is less then that of case A (ZB < ZA)
4µ0 I1 2 2 µ0 I1 V
B = 4 B1 = = Now I =
2 pL pL Z
The total flux passing through the square EFGH \I RA < I RB . option (a) is correct.
2 2 µ0 I1 2
f2 = B ´ l 2 = ´ l ... (i) \VRA < VRB .
pL
[Q l > L and therefore, B can be assumed Þ VCA > VCB
constant for l2] [Q If V is the applied potential difference access
The flux through small square loop is directly
proportional to the current passing through big square series R-C circuit then V = VR2 + VC2 ]
loop.
\ (c) is the correct option.
Electromagnetic Induction and Alternating Current P-S- 221

8. (a,c) Impedance across AB


I
2
æ 1 ö 2
Z1 = Xc2 + R12 = ç ÷ + R1
è wC ø
®
= (100)2 + (100)2 = 100 2 B

V 20 ®
\ I1 = = [leads emf by f1]
Z1 100 2 B

R 100 1 Circular
where cos f1 = = = Þ q = 45º
Z1 100 2 2 loop

When the current is decreasing at a steady rate then


100 µF
the change in the flux (decreasing inwards) on the right
100 W half of the wire is equal to the change in flux (decreasing
A B outwards) on the left half of the wire such that Df
I1
through the circular loop is zero.
10. (c, d) I = cos 500 t
0.5 H 50 W
C D
I2 I

~ 1
20V
7p
6w
Impedance across CD is p t
p 2p
w w
Z2 = X L2 + R22 = (wL)2 + R22 2w

–1
= (0.5 ´ 100)2 + (50)2 = 50 2 W

V 20 B D
\ I2 = = [leads emf by f2]
Z2 50 2 A

50 V
I1 C = 20 m F

45º R = 10 W
45º I 20V 7p p
Till t = , the charge will be maximum at
6w 2w
I2 p / 2w p / 2w
é sin 500t ù
Q¢ = ò cos 500t dt = ê
ë 500 û 0
ú
R 50 1 0
where cos f2 = = = Þ f2 = 45º
Z2 50 2 2
1 æ p ö 1
\ The current I from the circuit is = sin ç 500 ´ ÷= C
500 è 2 ´ 500 ø 500
I = I1 + I2 ; 0.3 A
9. (a) Emf will be induced in the circular wire loop when flux \ (a) is incorrect
through it changes with time. 7p
From the graph it is clear that just before t = , the
Df 6w
e =-
Dt current is in anticlockwise direction.
when the current is constant, the flux changing through \ (b) is incorrect
it will be zero.
EBD_7036
P-S- 222 Topic-wise Solved Papers - PHYSICS

7p B2 L2
At t = , the charge on the upper plate of capacitor V –V0 = – x
6w mR
is
B2 L2 x
\ V= V0 – – (iii)
7p mR
6w [V decreases from x=0 to x=L, remains constant
1 æ 7p ö
ò cos 500t dt = 500 sin çè 500 ´ 6 ´ 500 ÷ø for x = L to x= 3L again decreases from x=3L to x=4L]
0 From (i) and (iii)

1 1 BL é B2 L2 x ù
=- ´ = -10 -3 C i= R ê V0 - ú
500 2 ë mR û
Now applying KVL (when A is just connected to D) [i decreases from x=0 to x=L i becomes zero from x = L to x = 3L
i changes direction and decreases from x = 3L to x = 4L]
10-3 These characteristics are shown in graph (a) and (b) only.
50 + - i ´10 = 0 Þ i = 10 A
20 ´10-6 E. Subjective Problems
\ (c) is the correct option.
1. The magnetic lines of force due to current flowing in wire
The maximum charge on C is Q = CV = 20 × 10–6 × 50 AB is shown.
=10–3C NOTE : As the current increases, the number of magnetic
Therefore, the total charge flown = 2 × 10 –3 C lines of force passing through the loop increases in the
\ (d) is the correct option. outward direction. To oppose this change, the current will
11. (a, d) flow in the clockwise direction.
The flux passing through the triangular wire if i current flows
through the inifinitely long conducting wire
0.1 A B
m0 i
= ò 2px
´ 2pdx
x
0
10 cm dx
x x
m i
f = 0 = Mi
10p 2. KEY CONCEPT : This is based on motional emf.
m0 dI
\M = = 10 AS -1 180 km/h=50m/s
10p dt

di m0 m
Induced emf in the wire = M = ´ 10 = 0
dt 10 p p
As the current in the triangular wire is decreasing the induced 1m
current in AB is in the same direction as the current in the
hypotenuse of the triangular wire. Therefore force will be
repulsive.
e = vBl = 50 × 0.2 × 10–4 × 1 = 10–3 volt = 1 milli volt
BLv
12. (a, b) i = – (i) [Counter clockwise direction 3.
R
while entering, Zero when completely
inside and clockwise while exiting]
B 2 L2 v
F = iLB = – (ii) [Toward left while entering
R
and exiting and zero when completely inside] NOTE : When the coil A moves towards B, the number of
magnetic lines of force passing through B changes.
dv B 2 L2 v Therefore, an induced emf and hence induced current is
\ - mV =
dx R produced in B.
v x The direction of current in B will be such as to oppose the
B2 L2
\ ò dV = -
v0 mR ò0
dx field change in B and therefore, will be in the opposite
direction of A.
Electromagnetic Induction and Alternating Current P-S- 223

Magnitude of curr ent remains constant at


4.
I = Bwr 2 / 2 R .
I
Bw r 2
2R

T/2 T 3T/2 2T t

For next half rotation, when the loop comes out of the
field, current of the same magnitude is set up clockwise.
Anticlockwise current is supposed to be positive. The
I-t graph is shown in the figure for two periods of
rotation.
6. (i) A variable force F is applied to the rod MN such that as
NOTE : The network behaves like a balanced wheatstone the rod moves in the uniform magnetic field a constant
bridge. current flows through R. Consider the loop MPQN.
The free electrons in the portion MN of the rod have a Let MN be at a distance x from PQ.
velocity v in the right direction. Applying Fleming’s left hand Length of rails in loop = 2x
rule, we find that the force on electron will be towards N.
Hence, M will be + ve and N will be negative. Current will
flow in clockwise direction.
The induced emf developed is given by
e = vBl = v × 2 ×0.1 = 0.2v ...(i)
Now, e = IR
e = 10 – 3 × 4 = 4 ×10–3 amp ...(ii)
From (i) and (ii),
0.2 v = 4 ×10 –3
4 ´10-3 \ Resistance of rails in loop = 2xl
\ v= = 0.02 m / s
0.2 \ Total resistance of loop = R + 2lx
5. (i) Induced emf Induced emf = Bvd
Bvd
df d \ Induced current (I) =
E =- = - ( B ´ A) R + 2 lx
dt dt So for constant I,
d é æ1 2 öù ( R + 2lx )
=- ê B ç r q÷ ú = - 1 Br 2 d q = - 1 Br 2 w v=
Bd
I ...(i)
dt ë è 2 øû 2 dt 2
Furthermore, as due to induced current I the wire will
E 1 Br 2 w 1 Br 2 w
experiencea force FM = BId opposite to its motion, the
\ I= =- Þ| I |= equation of motion of the wire will be
R 2 R 2 R F – FM = ma i.e., F = FM + ma
But as here FM = BId and from equation (i)
dv 2 lI dx 2 lIv 2 lI 2
a= = = = ( R + 2l x )
dt Bd dt Bd ( Bd ) 2
2 lmI 2
So, F = BId + ( R + 2 lx )
( Bd ) 2
(ii) As the work done by force F per sec.
dW é 2lmI 2 ù é R + 2lx ù
= P = Fv = ê BId + ( R + 2l x ) ú ê .Iú
dt 2
(ii) The loop is entering in the magnetic field and hence ëê ( Bd ) ûú ë Bd û
magnetic lines of force passing through the loop is é 2lmI 3 ù
increasing in the downward direction. Therefore, current i.e., P = ê I 2 ( R + 2lx ) + ( R + 2lx )2 ú
3 3
will flow in the loop in such a direction which will ëê B d ûú
oppose the change. The current will flow in the anti- and heat produced per second, i.e., joule heat
clockwise direction. H = I 2 ( R + 2lx )
(iii) Graph between induced emf and period of rotation:
-1
For first half rotation, (t = T/2), when the loop enters H é 2 lmI ( R + 2lx) ù
So, f = = 1+
the field, the current is in anticlockwise direction. P êë B3 d 3
ú
û
EBD_7036
P-S- 224 Topic-wise Solved Papers - PHYSICS
7. (a) (i) In this case S and I are connected.
I0 I é - tù
R
When I = , then 0 = I 0 ê1 - e L ú
2 2 ê ú
ë û
-R R
1 t - t 1
Þ = 1- e L Þ e L =
2 2
Taking log on both sides
R
- t 1
log e e L = log e
2
R L 0.6930 ´ 10 ´10-3
Þ t = 0.693 Þ t = 0.693 =
L R (2 + 3)
when R = R2 + R4
Þ t = 1.386 × 10 –3 sec. Thus this much time is required
for current to reach half of its steady value.
The energy stored by the inductor at that time is given
2
1 2 1 æ 0.6 ö
Using Kirchhoff’s law in ABCDGA by E = LI = ´ 10 ´ 10-3 ´ ç ÷ = 4.5 ´ 10 -4 J
2 2 è 2ø
+ 3 – I2 × 2 – 12 + I1× 2 = 0
8. The equivalent circuit is drawn in the adjacent figure.
2 I1 – 2I2 = 9 ...(i)
Applying Kirchhoff ’s law in DEFGD
– 2I1 + 12 – (I1 + I2) 2 = 0
Þ 2I1 + I2 = 6 ...(ii)
21
From (i) and (ii) I1 = amp.
6
\ From (ii) I2 = –1 amp.
To find potential difference between A and B
VA + 3 – (– 1) × 2 = VB Þ VA – VB = – 5V
The rate of production of heat in R1 NOTE : As the magnetic field increases in the downward
2 direction, an induced emf will be produced in the AEFD as
æ 21ö well as in the circuit EBCF such that the current flowing in
= I12 R1 = ç ÷ ´ 2 = 24.5W
è 6ø the loop creates magnetic lines of force in the upward
(b) (i) When the switch is put in position 2 then the active direction (to the plane of paper).
circuit will be as shown in the figure. Thus, the current should flow in the anticlockwise direction
in both the loops.
Induced emf in loop AEFD
df d dB
e=- = - BA = - A = -1 ´ 1 = -1volt
dt dt dt
Induced emf in loop EBCF
df d dB
e= - = - BA ' = - A ' = – 0.5 × 1 = – 0.5 volt
dt dt dt
When the steady state current is reached then the
inductor plays no role in the circuit Let the current flowing in the branch EADF be i1 and the
E2 = I (R2 + R4) current flowing in the branch FCBE be i2. Applying junction
law at F, we get current in branch FE to be (i1 – i2)
3 Applying Kirchhoff ’s law in loop EADFE
Þ I= = 0.6 amp.
5 – 1 × i1 – 1× i1 + 1 – 1 × i1 – 1 (i1 – i2) = 0
(ii) KEY CONCEPT : The growth of current in L–R Þ 4i1 – i2 = 1 ... (i)
circuit is given by the expression Applying Kirchhoff 's law in loop EBCFE
+ 0.5i1 – 0.5 + 1i2 + 0.5 i2 – 1(i1 – i2) = 0
I = I0 é - tù
R
– i1 + 3i2 = 0.5 ... (ii)
êë1 - e L úû
Solving (i) and (ii)
11 i1 = 3.5
Electromagnetic Induction and Alternating Current P-S- 225

7 Y
Þ i1 = 3.5/11 = A
22
Also 11i2 = 3 w A
6 S x dx
Þ i2 = 3/11 A = A q
22 O
R X
7 B
\ Current in segment AE = i1 = A
22
6 L
Current in segment BE = i2 = A
22
1
Current in segment EF = (i1 - i2 ) = A
22 é x2 ù
r
r Br 2 w
9. KEY CONCEPT : We can R1 e = ò Bxw dx = Bw ê ú =
ëê 2 ûú 0 2
understand the direction of flow A C 0
i1
of induced currents by imagining
B(i1+i2)l (b) The above diagram can be reconstructed as the
a fictitious battery to be attached adjacent figure. e is a constant. O will accumulate
between E and F. The direction of e positive charge and A negative. When the switch S is
induced current can be found with E L closed, transient current at any time t, when current I is
i1+i2 F
the help of Lenz’s law. mg flowing in the circuit,
i2 I = I0 (1– et / t )
NOTE : P.d. across parallel B D
combinations remains the same R2 Here,
Also, P1 = ei1 = 0.76 W e B wr 2
and P2 = ei2 = 1.2 W I0 =
= I
R 2R S
i1 1.76 1.76 L O
\ = Þ i1 = i2 … (ii) and t = R
i2 1.2 1.2 R A
The horizontal metallic bar L moves with a terminal velocity. æ Rö
Bwr 2 -ç ÷ t
This means that the net force on the bar is zero. Therefore, I = [1 - e è L ø ] L
\ B (i1 + i2 ) = mg 2R
(ii) In steady state,
mg 0.2 ´ 9.8 49
Þ i1 + i2 = = = amp. ......(iii)
Bl 0.6 ´ 1 15 B wr 2 æ Rö
I= [Q t has a large value and - çè L ÷ø t ® 0]
From (ii) and (iii) 2R e
1.76 49
NOTE : When current flows in the circuit in steady state,
i2 + i2 = there is a power loss through the resistor.
1.2 15 Also since the rod is rotating in a vertical plane, work needs
19 0.76 to be done to keep it at constant angular speed.
Þ i2 = 2 amp. Þ i1 = amp. Þ e = = 0.6V Power loss due to current I will be
15 19 /15
The induced emf across L due to the movement of bar L in a æ Br 2 w ö
2 t=t
2
magnetic field P=I R=ç ÷ R
è 2R ø r/2
e 0.6 q
e = BvTL Þ vT = = = 1m/s r t =0
BL 0.6 ´ 1
If torque required for this power 2 cosq

Also from (i), is t1 then mg


P = t1w
e 0.6 e 0.6
R1 = = = 0.47W and R2 = = = 0.3 W
i1 19 /15 i2 2 B 2 r 4w
Þ t1 =
10. (i) (a) Let us consider a small length of metal rod dx at a 4R
distance x from the origin. Small amount of emf (de) Torque required to move the rod in circular motion against
induced in this small length (due to metallic rod cutting gravitational field
magnetic lines of force) is r
de = B (dx) v … (i) t 2 = mg ´ cos q
2
where v is the velocity of small length dx
v = xw … (ii) The total torque
t = t1 + t2 (Clockwise)
\ The total emf acoss the whole metallic rod OA is
EBD_7036
P-S- 226 Topic-wise Solved Papers - PHYSICS

B 2 r 4 w mgr vT
t= + cos wt For the case when velocity is
4R 2 2
The required torque will be of same magnitude and in
anticlockwise direction. The second term will change signs
as the value of cos q can be positive as well as negative.
11. KEY CONCEPT : Let I0 be the current at steady state. The
magnetic energy stored in the inductor at this state will be

L=10H R=2W

vT mgR
10V v= =
2 2 B 2 L2
1 2
E= LI 0 … (i) Substituting this value of velocity in eq. (2) we get
2
This is the maximum energy stored in the inductor. The B 2 L2 1 mgR 1 g
a=g- ´ =g- g=
current in the circuit for one fourth of this energy can be mR 2 B L 2 2 2 2
found as 13. Suppose at t = 0, y = 0 and t = t, y = y
1 1 ur ur
´ E = LI 2 … (ii) (a) Total magnetic flux = B . A
4 2
ur ur B0 y
Dividing equation (i) and (ii) where A = a 2 k$ and B = k$
a
1 2
LI 0 I
E B0 y 2
=2 ÞI = 0 \ f= .a = B0 ya
E/4 1 2 2 a
LI
2
df dy
Also, V = I0 R Net emf., e = - = - B0 a = - B0 av(t )
dt dt
V 10 I0 5 As total resistance = R
Þ I0 = = = 5 amp. \ I = = = 2.5 amp.
R 2 2 2 | e | B0 av(t )
The equation for growth of current in L-R circuit is \|i|= =
R R
I = I0 [1 – e – Rt/L]
X
1
Þ 2.5 = 5 [1– e – 2t/10] Þ = 1 – e-t / 5 y
F1
2
Þ t = 5 loge 2 = 2 × 2.303 × 0.3010 = 3.466 sec. y+a
12. KEY CONCEPT : If v is the velocity of the rod at any time F2
t, induced emf is BvL and so induced current in the rod F4
Induced e.m.f. BvL
I= = F3
R R Y
Due to this current, the rod in the field B will experience a
NOTE : Now as loop goes down, magnetic flux linked with
force
it increases, hence induced current flows in such a direction
B 2 L2 v so as to reduce the magnetic flux linked with it. Hence,
F = BIL = (opposite to its motion) … (1) induced current flows in anticlockwise direction.
R
So, equation of motion of the rod will be, (b) Each side of the cube will experience a force as shown
T – F = 0 × a, i.e., T = F [as rod is massless] (since a current carrying segment in a magnetic field
experiences a force).
T B 2 L2v
mg – T = ma Þ a = g - =g- … (2) ur r ur æ B y ö
m mR F 1 = i ( l ´ B ) = i ç - aiˆ ´ 0 kˆ÷ = B0 y (iˆ ´ ˆj );
è a ø
So rod will acquire terminal velocity when its acceleration is
zero i.e., ur æ B ( y + a ) ˆö
2 2 F 3 = i ç + aiˆ ´ 0 k ÷ = iB0 ( y + a ) ˆj
B L vT mgR è a ø
g- = 0 i.e. vT = 2 2 ;
mR B L
Electromagnetic Induction and Alternating Current P-S- 227
ur ur
NOTE : F 2 = - F 4 and hence will cancel out each other.. Also we know that the emf (V) produced across the inductor
ur ur ur ur ur B02 a 2v (t ) df d dI
Net force, F = F 1 + F 2 + F 3 + F 4 = -iB0 ajˆ = - ˆj V=– = - [ LI 2 ] = - L 2
R dt dt dt

ur é B 2 a 2 v (t ) ù ˆ é æ - R2 ö ù
t
d êE
(c) Total net force = mg ˆj + F = ê mg - 0 ú j; = -L ç 1- e L ÷ ú
ëê R ûú dt ê R2 çè ÷ø ú
ë û
dv B 2 a 2v (t ) R2
\ m = mg - 0 t
dt R V = – E e - L . Here the negative sign shows the opposition
to the growth of current.
v dv t
Integrating it, we get, ò0 B02 a 2 v (t )
= ò dt
0 -
2
t
g- \ V = 12 e 400 ´10 -3 = 12 e–5t volt
mR DECAY OF CURRENT : When the switch is opened, the
branch AG is out of the circuit. Therefore, the current decays
(v )t
é B02 a 2v (t ) ù through the circuit CBFDC (in clockwise direction).
log ê g - ú Applying Kirchhoff 's law C
ëê mR ûú
0
=t I
æ L dI ö
- B02 a 2 I (R1 + R2) – ç - =0
è dt ÷ø L
mR R1
dI æ R + R2 ö
\ = -ç 1 dt
é I è L ÷ø
B02 a2 v(t ) ù
êg - ú 2 2 \ On integrating, R2
or log ê mR ú = - B0 a t
ê ú I dI (R + R ) t
ê
g
ú
mR
òI 0 I = - 1 L 2 ò0 dt
ë û
( R1 + R2 )t
-

or 1-
B02 a 2 v (t )
=e
(
- B02 a 2t ) mR \ I = I0 e L

mgR R + R2 2+2
Here, 1 = = 10
L 400 ´ 10-3
or 1- e
( )
- B02 a 2t mR
=
B02 a 2
v (t ); E 12
mgR and I0 = = = 3A
R1 + R2 4

\
mgR é
v (t ) = ê1 - e
- B02a 2t mR ù
ú
( ) \ I = 3e–10t A, clockwise.
Alternatively, you may directly find the time constant
B02 a 2 ëê úû
L
When terminal velocity is attained, v (t) does not depend t= and use the equation i = i0 e - t / t where i0 = 6A
R1 + R2
on t
15. Let us consider the current in the clockwise direction in
mgR loop PQRS. Force on wire QR,
\ v (t) = ur r ur
B02 a2 F = I (l ´ B) = I [(ai$) ´ (3i$ + 4k$ ) B ]
QR 0
14. This is a question on growth and rise of current.
GROWTH OF CURRENT : Let at any instant of time t the = I B0 [3ai$ ´ i$ + 4ai$ ´ k$ ] = I B0 [0 + 4a ( – $j )] = -4aB0 I $j
current be as shown in the figure. Z
Applying Kirchoff’s law in the loop ABCDFGA
we get, starting from G moving clockwise B
dI A B I2 C
E - L 2 - I 2 R2 = 0 I
>
3i

dt Q
+

>
P
4k

I1 Y
>

L 4k
12V E

R1 X
E éê - tù
R
S R
or I 2 = 1- e L ú R2
R2 ê ú Force on wire PS
ë û

G F D
EBD_7036
P-S- 228 Topic-wise Solved Papers - PHYSICS
ur r ur (b) Let AB be at a distance x from the long straight wire at
F PS = I (l ´ B ) = I [a( -$i ) ´ (3$i + 4k$ ) B0 ] = 4aB0 I $j
any instant of time t during its motion. The magnetic
Thus we see that force on QR is equal and opposite to that field at that instant at AB due to long straight current
on PS and balance each other. carrying wire is
The force on RS is
ur r ur m 0 I0
F RS = I (l ´ B ) = I [b (- $j ) ´ (3$i + 4k$ ) B0 ] B=
2p x
= I bB0 [3k$ - 4i$] ... (i) The change in flux through ABCD in time dt is
The torque about PQ by this force is
df = B (dA) = B l d x
r r ur $ $ $
t RS = r ´ F = (i a) ´ (3k - 4i ) IbB0 Therefore, the total flux change when metal bar moves
from a distance x0 to 2x0 is
= I abB0 (3 $j ) ... (ii)
2 x0 2 x0 m0 I0 m I l 2x
The torque about PQ due to weight of the wire PQRS is Df = ò x0 B ldx = l ò
x0 2p x
dx = 0 0 [log e x] x 0
2p 0
æ aö
t = mg ç ÷ ... (iii) m0 I0l
è 2ø = log e 2 ... (ii)
2p
For the wire loop to be horizontal, we have to equate (ii) and
The charge flowing through resistance R in time T is
a
(iii) 3IabB0 = mg T T 1é di ù
2 q= ò0 idt = ò0 ê

Einduced - L ú dt [from eq. (i)]
dt û
mg
Þ I= ... (iv) 1 T L i1
6bB0 1 L
R ò0
= Einduced dt - ò d i = ( D f) - i1
R 0 R R
Therefore,
(a) The direction of current assumed is right. This is 1 é m0 I0 l ù L
because torque due to mg and current are in opposite q= ê log e 2ú - i1 from eq. (ii)
R ë 2p û R
directions. Therefore, current is from P to Q.
ur (c) When the metal bar AB is stopped, the rate of change
(b) From (i), F RS = IbB0 (3k$ - 4$i )
of magnetic flux through ABCD becomes zero.
mg From (i),
(c) From (iv), I =
6aB0 di
iR = – L
16. (a) KEY CONCEPT : As the metal bar AB moves towards dt
the right, the magnetic flux in the loop ABCD increases
2T L i1 / 4 di
in the downward direction. By Lenz's law, to oppose òT dt =
R òi1 i
this, current will flow in anticlockwise direction as
shown in figure.
L i /4 L T
T= - log e 1 Þ =
R i1 R 2 log e 2
A
D
17. (a) Let us consider a small strip of thickness dx as shown
in the figure.
R The magnetic field at this strip
i B = BA + BB
Io
v (Perpendicular to the plane of paper directed upwards)
L m0 I m0 I
= +
2p x 2p (3a - x )

C BA = Magnetic field due to current in wire A


B
x
m0 I é 1 1 ù
= +
Applying Kirchhoff 's loop law in ABCD, we get 2p ë x 3a - x úû
ê

df di BB = Magnetic field due to current in wire B


= iR + L ... (i)
dt dt
Electromagnetic Induction and Alternating Current P-S- 229

V 2
dx V0 = Vrms 2 \ I0 = rms = 20 A
Z
R 1 p
cos f = = \ f=
x Z 2 4
I
\ graph is given by.

Small amount of magnetic flux passing through the strip V = V0 sin w t


V0
of thickness dx is
m 0 Ia ´ 3a dx I0
df = B × adx = O
2 p x(3a - x ) I= I0 sin(w t- p /4) t
Total flux through the square loop
2a m0 I ´ 3a 2 dx m Ia
f= òa 2p x (3a - x)
= 0 ln 2
p
19. NOTE : After a long time capacitor will be fully charged,
m 0 a ln (2) hence no current will flow through capacitor and all the
= (I0 sin wt) current will flow from inductor. Since current is D.C.,
p
The emf produced resistance of L is zero.

df m aI w æR ö 1 3R
e= - = 0 0 ln (2) cos wt \ Req = ç + R÷ ´ + r1 + r2 = + r1 + r2
dt p è2 ø 2 4
Charge stored in the capacitor R
R L
m 0 aI 0 w A B R
q=C×e=C× ln (2) cos wt ... (i)
p
\ Current in the loop r1 r2 R
R C
R
dq C ´ m0 aI 0 w 2
i= = ln (2)sin wt
dt p S

m 0 aI 0 w 2 C ln (2) e+e 2e 2e
\ imax = I= Þ I= =
p Req Req (3R / 4) + r1 + r2
(b) From (i), the graph between charge and time is
Potential drop across A is
Q 2e
Q0
e – I r1 = 0 Þ e = r1
(3R / 4) + r1 + r2

4
p/w 2p/w t Þ r1 = r2 + 3R/4 (r1 - r2 )
or R=
3
p/2w 3p/2w 20. KEY CONCEPT : The magnetic field in the solenoid is given
by
B = µ0 ni
– Q0 R
a

C ´ m 0 aI0 w ln (2) d
Here, q0 =
p
18. Given, Vrms = 220 V L
n = 50 Hz, L = 35 mH, R = 11W
Impedance

Z= (wL) 2 + R 2 = 11 2 W
V0 Þ B = µ0 ni0 sin wt [Q i = i0 = sin wt (given)]
also, I0 =
Z The magnetic flux linked with the solenoid
EBD_7036
P-S- 230 Topic-wise Solved Papers - PHYSICS
ur ur G. Comprehension Based Questions
f = B . A = B A cos 90° = (µ0 ni0 sin wt) (pa2)
1. (b) For charging of R – C circuit, Q = Q0 [1 – e–t/t] when
\ The rate of change of magnetic flux through the solenoid
the charging is complete, the potential difference
df between the capacitor plates will be V. The charge
= p m 0 na 2i0 w cos wt stored in this case will be maximum.
dt
Therefore, Q0 = CV.
The same rate of change of flux is linked with the cylindrical
shell. By the principle of electromagnetic induction, the é -2 t ù
induced emf produced in the cylindrical shell is When t = 2t, Q = CV ëê1 = e t ûú
= CV[1 – e–2]
V

x
x xx R
C S1

I
TOP VIEW
S2
df
e=- = -p m0 na 2i0 w cos wt ... (i) L
dt 2. (d) The instantaneous charge on plates at any time t during
The resistance offered by the cylindrical shell to the flow of discharging is
induced current I will be Q = Q0 cos wt
\ Instantaneous current,
l
R= r dQ
A I= = Q0 w sin wt
dt
Here, l = 2 p R and A = L × d
_ C I
+
2p R
\ R= r ... (ii)
Ld
The induced current I will be S2

| e | [p µ0 na 2i0 w cos wt ] ´ Ld
I= =
R r ´ 2 pR
L
m na2 Ld i0 w cos wt \ The magnitude of maximum current
Þ I= 0
2 rR Imax = Q0w

F. Match the Following 1


Here Q0 = CV and w =
LC
1. A-r,s,t; B-q,r,s,t; C-p,q; D-q,r,s,t
The following are the important concepts which are applied 1 C
in the given situation. \ Imax = CV × =V
LC L
(i) For DC circuit, in steady state, the current I through
3. (c) Apply Kirchhoff 's law in the circuit
the capacitor is zero. In case of L-C circuit, the potential
difference across the inductor is zero and that across Q dI Q dI
-L =0 Þ =L
the capacitor is equal to the applied potential difference. C dt C dt
In case of L-R circuit, the potential difference across
inductor is zero across resistor is equal to the applied d æ dQ ö d 2Q
Þ Q = LC ç - ÷ = - LC
voltage. dt è dt ø dt 2
(ii) For AC circuit in steady state, Irms current flows
Ns Vs 10 V
through the capacitor, inductor and resistor. The 4. (a) For step up transformer = Þ = s
N p Vp 1 4000
potential difference across resistor, inductor and
capacitor is proportional to I. \ Vs = 40,000 V
(iii) For DC circuit, for changing current, the potential Np Vp
difference across inductor, capacitor or resistor is For step down transformer =
Ns Vs
proportional to the current.
Electromagnetic Induction and Alternating Current P-S- 231

I. Integer Value Correct Type


40,000 200
= =
200 1 1. (4) Time constant = RC
(a) is the correct option. 2
5. (b) We know that P = V × I 2 æ 1 ö
Impedance = R +ç
è wC ÷ø
P 600 ´ 1000
\ I= =
V 4000 Given impedance = R 1.25
\ I = 150 A
Total resistance = 0.4 × 20 = 8 W æ 1 ö
2
\ R 1.25 = R2 + ç
\ Power dissipated as heat = I2 R = (150)2 × 8 è wC ÷ø
= 180,000W = 180 kW
180 2 2
\ % loss = ´ 100 = 30% \ RC = = ´ 1000 ms
600 w 500
(b) is the correct option. \ RC = 4 ms
® ®
2. (7) The magnetic field due to current carrying wire at the
- df d dB
6. (b) ò E .dl = = - (BpR 2 ) = -pR 2 location of square loop is
dt dt dt
µ0 2piR2 µ i
= - pR B2 B= 2 2 3/2
= 0
4p (R + 3R ) 16R
\ E × 2pR = – pR2B
The mutual induction
- BR
\E= N f 2 é µ0 i 2 ù
2 M= = ê ´ a cos 45º ú
(b) is the correct option. i i ë 16R û
7. (b) Given M = g L
\ M = g mwR2 µ0 a2
\ M= 7
\ M = g m (Dw) R2 ...(1)
22 R
Q×B
But Dw = ...(2)
2m 1mH 3W
3. (8)
2
æ QB ö 2 - gBQR
From (1) and (2) DM = -g m çè ÷ R = 2mH 4W
2m ø 2
The negative sign shows that change is opposite to
the direction of B. 12W
(b) is the correct option.

H. Assertion & Reason Type Questions


1. (a) As shown in the figure the horizontal component of 5V
the magnetic field interacts with the induced current
5
produced in the conducting ring which produces an At t = 0 Imin =
12
average force in the upward direction. (Fleming's left
hand rule). 5 5 10
At t = ¥ Imax = R = 3 / 2 = 3
eq
F I
I
é 1 1 1 1 8ù
BH ê R = 3 + 4 + 12 = 12 ú
ë eq û

Conducting ring I max 10 12


\ I = ´ =8
min 3 5
EBD_7036
P-S- 232 Topic-wise Solved Papers - PHYSICS

Section-B JEE Main/ AIEEE


When half energy is with the capacitor in the form of
1. (b) The impedance triangle for resistance (R) and inductor
electric field between the plates of the capacitor we get
(L) connected in series is shown in the figure.
E 1 Q '2
2
2 = where Q ' is the charge on one plate of
L 2 2 C
2 +w the capacitor
R XL= w L
f 1 1 Q 2 1 Q '2 Þ Q ' = Q
\ ´ =
R 2 2 C 2 C 2
8. (a) Laminated core provide less area of cross-section for
R the current to flow. Because of this, resistance of the
Power factor cosf = core increases and current decreases thereby
R2 + w 2 L2
decreasing the eddy current losses.
2. (d) The induced emf is 9. (a) D.C. ammeter measure average current in AC current,
r r
-d f d ( B. A) - d ( BA cos 0º ) average current is zero for complete cycle. Hence
e= =- = reading will be zero.
dt dt dt
10. (d) Since the phase difference between L & C is p,
× × × × \ net voltage difference across LC = 50 – 50 = 0
× × × ×
Df (W2 - W1 )
× ×l × × V 11. (b) =
Dt t
× × × ×
x Rtot = ( R + 4 R )W = 5R W
× × × ×
nd f -n(W2 - W1 )
dA d (l ´ x ) dx i= =
\ e = –B = -B = - Bl = - Blv Rtot dt 5 Rt
dt dt dt
3. (d) These three inductors are connected in parallel. The (QW2 & W1 are magnetic flux)
equivalent inductance Lp is given by r r
12. (b) f = B. A ; f = BA cos wt
1 1 1 1 1 1 1 3
= + + = + + = =1 df wBA
L p L1 L2 L3 3 3 3 3 e=- = wBA sin wt ; i = sin wt
dt R
\ Lp = 1
2
4. (b) Np = 140, Ns = 280, Ip = 4A, Is = ? æ wBA ö
Pinst = i 2 R = ç ´ R sin 2 wt
è R ÷ø
Is N p
For a transformer I = N
p s T T
2
ò Pinst ´ dt 2 ò sin wtdt
I s 140
= Pavg = 0 =
(wBA) 0 1 ( wBA) 2
Þ Þ Is = 2 A T R T =
4 280 2 R
5. (b) Mutual conductance depends on the relative position ò dt ò dt
0 0
and orientation of the two coils.
Df -D ( LI ) DI é pr 2 ù
6. (d) e=- = = -L (w B pr 2 ) 2 êA = ú
Dt Dt Dt \ Pavg = 2 ûú
8R ëê
DI 4 13. (a) For resonant frequency to remain same
\ | e |= L Þ 8= L ´
Dt 0.05 LC should be const. LC = const
8 ´ 0.05 L
Þ L= = 0.1H Þ LC = L' × 2C Þ L ' =
4 2
7. (c) When the capacitor is completely charged, the total
14. (b) l = 1m, w = 5 rad/s, B = 0.2 ´ 10 -4 T
energy in the L.C circuit is with the capacitor and that
energy is Bwl 2 0.2 ´10-4 ´ 5 ´1
e= = = 50mV
1 Q2 2 2
E=
2 C
Electromagnetic Induction and Alternating Current P-S- 233

15. (c) Relative velocity = v + v = 2v 24. (a) Initially, when steady state is achieved,
\ emf. = B.l (2v) E
16. (d) For maximum power, X L = X C , which yields i=
R
1 1 Let E is short circuited at t = 0. Then
C= = E
2 2
(2 pn) L 4 p ´ 50 ´ 50 ´ 10 At t = 0, i0 =
R
\ C = 0.1 ´ 10-5 F = 1mF Let during decay of current at any time the current

æ pö flowing is - L di - iR = 0
17. (a) Phase difference for R–L circuit lies between ç 0, ÷ dt
è 2ø
i t
R 12 4 di R di R
18. (b) Power factor = cos f =
=
Z 15 5
= = 0.8 Þ
i
= - dt Þ
L ò i
= ò - dt
L
i0 0
19. (a) KEY CONCEPT : The charging of inductance given
R
æ - t
- ö i R
Rt
by, i = i0 ç1 - e L ÷ Þ log e = - t Þ i = i0 e L
i0 L
çè ÷ø
R -100 ´10-3
Rt Rt E - t
100 100 ´10 -3 1
i0 - - 1 Þi= =e L e =
= i0 (1 - e L ) Þ e L = R 100 e
2 2 25. (c) KEY CONCEPT : We know that power consumed in
Taking log on both the sides,
a.c. circuit is given by, P = Erms .I rms cos f
Rt
- = log1 - log 2 Here, E = E0 sinwt
L
æ pö
300 ´ 10 -3
I = I0 sin ç wt - ÷
L è 2ø
Þ t = log 2 = ´ 0.69
R 2
p
Þ t = 0.1 sec. which implies that the phase difference, f =
2
f BA
20. (b) Mutual inductance = = p p
I I æ ö
\ P = Erms .I rms .cos
2
=0 çè Q cos = 0÷
ø
2
[MT -1Q -1 L2 ]
[Henry] = = ML2 Q -2
[QT -1 ] æ - tö
R
26. (a) KEY CONCEPT : I = I o 1 - e L ÷
ç
V 100 çè ÷ø
21. (c) Across resistor, I = = = 0.1 A
R 1000 (When current is in growth in LR circuit)
At resonance,
æ - ´2 ö
5
Eæ - tö
R
1 1 5
X L = XC = = = 2500 = ç1 - e L ç
÷ = 1- e 10 ÷ = (1 – e–1)
wC 200 ´ 2 ´ 10-6 R çè ÷ø 5 çè ÷ø
Voltage across L is
I X L = 0.1 ´ 2500 = 250 V m0 N1 N 2 A 4p ´ 10-7 ´ 300 ´ 400 ´ 100 ´ 10-4
ur ur 27. (d) M= =
l 0.2
df d ( N B. A)
22. (d) e = - =-
dt dt = 2.4p ´ 10 -4 H
d 28. (c) Growth in current in LR2 branch when switch is closed
= -N ( BA cos wt ) = NBAw sin wt is given by
dt
R2t
Þ e max = NBAw E di E R2 - R2t / L E - L
i= [1 - e - R2t / L ] Þ = . .e = e
R2 dt R2 L L
23. (b) f = 10t 2 - 50t + 250
Hence, potential drop across
df 2t
e=- = - (20t - 50) æ E -R t / L ö -
dt -R t / L -3
L = çè e 2 ÷ø L = Ee 2 = 12e 400 ´10
L
et = 3 = -10 V = 12e–5tV
EBD_7036
P-S- 234 Topic-wise Solved Papers - PHYSICS
29. (b) Due to the movement of resistor R, an emf equal to Blv
1 2 1 q2
will be induced in it as shown in figure clearly, \ Li =
2 2 C
I = I1 + I 2
1
Also, I1 = I2 Also q = q0 cos wt and w =
Solving the circuit, P LC
we get l Blv p
On solving t = LC
Blv 4
I1 = I 2 = 34. (b) We have, V = V0 (1 – e–t/RC)
3R RW v RW
RW

and I = 2 I1 =
2 Blv I (
Þ 120 = 200 1 - e - t / RC )
3R I2
Þ t = RC in (2.5)
I1 Q
Þ R = 2.71 × 106 W
30. (c) At t = 0 , no current will flow through L and R1 35. (d) Because of the Lenz's law of conservation of energy.
V 36. (d) Here, induced e.m.f.
\ Current through battery = R
2 w l
2l
At t = ¥ ,
dx
R1R2 x
effective resistance, Reff =
R1 + R2 [(3l) 2 – (2l) 2 ] = 5 Bl w
3l 2

V V ( R1 + R2 )
e= ò (wx) Bdx = Bw 2 2
2l
\ Current through battery = R = R1R2
eff 37. (a) As we know, Magnetic flux, f = B. A
31. (d) When capacitance is taken out, the circuit is LR.
m0 (2)(20 ´ 10 -2 ) 2
wL ´ p(0.3 ´ 10 -2 ) 2
2 2
\ tan f = 2[(0.2) + (0.15) ]
R
On solving
1 200 = 9.216 × 10–11 = 9.2 × 10–11 weber
Þ wL = R tan f = 200 ´ =
3 3 38. (c) Charge on he capacitor at any time t is given by
Again , when inductor is taken out, the circuit is CR. q = CV (1– et/t)
at t = 2t
1
\ tan f = q = CV (1 – e–2)
w CR
39. (c) Applying Kirchhoff's law of voltage in closed loop
1 1 200 V
Þ = R tan f = 200 ´ = –VR –VC = 0 Þ R = -1
wc 3 3 VC
VR
2 A C R
2 æ 1 ö
Now, Z = R + ç - wL ÷
è wC ø
L VL
2
æ 200 200 ö
= (200) 2 + ç - ÷ = 200 W
è 3 3ø B

Power dissipated = Vrms I rms cos f

Vrms R æ Rö 15 ´ 100
I(0) = = 0.1A
= Vrms . . çQ cos f = ÷ 40. (b)
0.15 ´ 103
Z Z è Zø
I (¥) = 0
V 2rms R (220)2 ´ 200 220 ´ 220 –t
= = 2 = = 242 W I (t) = [I (0) – I (¥)] e L / R + i( ¥ )
2 (200) 200
Z
32. (c) Induced emf = vBH l = 1.5 × 5 × 10–5 × 2 = 15 × 10–5 –t R
= 0.15 mV I(t) = 0.1 e L / R = 0.1 e L
1 2 0.15 ´1000
33. (a) Energy stored in magnetic field = Li I(t) = 0.1 e
2 0.03 = 0.67mA
1 q2
Energy stored in electric field =
2 C
Electromagnetic Induction and Alternating Current P-S- 235

41. (c) From KVL at any time t


d2x b dx k
Double differential equation 2
+ + x=0
di dt m dt m
R L
+ dt –
Rt Rt
i Qmax = Q o e - Þ Q 2max = Qo2e -
2L L

Hence damping will be faster for lesser self inductance.


r r
42. (c) F1 = F2 = 0
+ – because of action and reaction pair
q c
43. (b) Here

q di e e e
- iR - L = 0 i= = =
2
c dt R + X 2L 2
R +w L 2 2
R + 4 p2 v 2 L2
2

dq q dq Ld 2q 220 V 80
i=- Þ + R+ 2 =0 10 = [Q R = = = 8]
dt c dt dt 2
64 + 4p (50) L 2 I 10

d 2q R dq q On solving we get
2
+ + =0 L = 0.065 H
dt L dt Lc
From damped harmonic oscillator, the amplitude is
dt
given by A = Ao e -
2m
EBD_7036
16
P-S- 236 Topic-wise Solved Papers - PHYSICS

Ray and Wave Optics


Section-A : JEE Advanced/ IIT-JEE
A 1. 2 × 108 m/s, 0.4 × 10–6 m 2. d = +15 cm 3. 4000Å, 5 × 1014 Hz 4. 2
25
5. 60 cm 6. 7. 30 cm 8. 1.5 9. zero 10. smaller
9

me
11. 12. 5 × 1014 Hz, 4000Å 13. 0.125 m, 0.5 m 14. 15°
m0 e 0
B 1. T 2. T 3. F 4. T 5. T 6. F 7. T
C 1. (c) 2. (a) 3. (d) 4. (a) 5. (c) 6. (c) 7. (d)
8. (c) 9. (c) 10. (a) 11. (c) 12. (c) 13. (a) 14. (d)
15. (a) 16. (d) 17. (d) 18. (b) 19. (a) 20. (a) 21. (c)
22. (b) 23. (b) 24. (d) 25. (c) 26. (b) 27. (c) 28. (a)
29. (b) 30. (b) 31. (b) 32. (b) 33. (c) 34. (b) 35. (d)
36. (c) 37. (a) 38. (c) 39. (a) 40. (c) 41. (b) 42. (c)
43. (b) 44. (b) 45. (c) 46. (a) 47. (b) 48. (c) 49. (b)
50. (c) 51. (b) 52. (d) 53. (a) 54. (b) 55. (c) 56. (a)
57. (c)
D 1. (b, d) 2. (a) 3. (a, c) 4. (b, d) 5. (d) 6. (a)
7. (d) 8. (c) 9. (a, b, c, d) 10. (c) 11. (b, c) 12. (b)
13. (c, d) 14. (d) 15. (d) 16. (a) 17. (a, b) 18. (c, d) 19. (a, b, c)
20. (a,c) 21. (a,b,c) 22. (b) 23. (a,d) 24. (a,c,d) 25. (b,d)
E 1. 11 cm, Real 2. 1.732 3. 1.41 4. m1 < m2 5. 7.67 cm 6. (i) 15 cm (ii) 1.15 cm
7. 13.9 8. 75 cm 9. (i) 1.17 × 10–3 m, (ii) 1.56 × 10–3 m 11. (i) 2 (ii) No
1
12. (i) –6 mm, –5 mm 13. 7 × 10–6 W 14. 15. (a) 0.8 × 10–14 m2 (b) 27.2° (c) 9I
49
4
dy 2 æ xö
16. 2 17. (i) 10–3 m (ii) increase 19. (a) = cot i (b) y = k ç ÷ (c) (4m, 1m)
dx è 4ø

20. (i) sin -1 ê


é 1
ë 2
{ } ù
n2 - n12 - n1 ú (ii) 72.94°
û
æ 3ö
21. (i) 6.3 × 10–4 m (ii) 1.575 × 10–6 m 22. 0.4m, 0.6m 23. 9.3 × 10–6m 24. (a) 600 nm (b) sin–1 çè ÷ø
4
25. (a) ± 0.26 m, ±1.13 m (b) 1.13 m, 0.26 m
1 4
26. (a) 4.33 × 10–3 m (b) 0.75 (c) 0.65 × 10–6 m, 0.433 × 10–6 m 27. é3iˆ + 4 ˆj - 5kˆù 28.
5 2ë û 3

1
29. (a) 15 cm, (b) 90 nm 30. 4°, – 0.04° 31. 2 cm, 1.0016 32. 1.6 33. (a) circular (b) 1/16 (c) 300 nm
2
m3 R
34. 35. (a) zero (b) 125 nm 36. 6.056 m 37. 3.5 mm 38. 0.09 m/sec, 0.35 s–1
m 3 - m1
39. 60º 40. (a) 60º, (b) 60º, anticlockwise
F 1. A-p; B-r; C-r; D-p, q, s 2. A-p, q, r, s; B-q; C-p, q, r, s; D-p, q, r, s 3. A-p, s; B-q; C-t; D-r, s, t
4. A-p, r; B-q, s, t; C-p, r, t; D-q, s 5. (d) 6. (b)
G 1. (a) 2. (c) 3. (b) 4. (c) 5. (b) 6. (a,c) 7. (d)
H 1. (c)
I 1. 6 2. 6 3. 3 4. 2 5. 3 6. 7 7. 2
Ray and Wave Optics P-S- 237

Section-B : JEE Main/ AIEEE


1. (b) 2. (a) 3. (a) 4. (d) 5. (a) 6. (b) 7. (d) 8. (c)
9. (b) 10. (b) 11. (c) 12. (d) 13. (b) 14. (a) 15. (a) 16. (b)
17. (b) 18. (d) 19. (a) 20. (b) 21. (a) 22. (a) 23. (c) 24. (d)
25. (c) 26. (a) 27. (b) 28. (c) 29. (d) 30. (b) 31. (d) 32. (a)
33. (a) 34. (b) 35. (a) 36. (b) 37. (d) 38. (d) 39. (c) 40. (c)
41. (d) 42. (c) 43. (b) 44. (b) 45. (d) 46. (d) 47. (b) 48. (c)
49. (a) 50. (b) 51. (c)

Section-A JEE Advanced/ IIT-JEE


A. Fill in the Blanks
1 æ 1 1 ö
8 and = (1.125 - 1) ç - ÷
3 ´ 10 f2 è R1 R2 ø
1. V2 = = 2 ´ 108 m / s ;
1.5 On dividing we get
V2 2 ´ 108 f2 1.5 - 1 0.5
l2 = = = 0.4 ´ 10-6 m = =
15 1.125 - 1 0.125
=4 \ f2 = 60 cm
v 5 ´ 1014
2. I1 A12
6. = ...(i)
I 2 A22
Parallel
beam
1 I r2
But I µ Þ 1 = 2 ...(ii)
r2 I 2 r12
From (i) and (ii),
5 cm A1 r2 25
= = .
20 cm A2 r1 9
From the diagram it is clear that the focus of both the lenses 7. For refraction at APB
should coincide as shown in the diagram. µ µ µ - µ2
Therefore d = 15 cm. - 2+ 1= 1
3. KEY CONCEPT : u v R

Speed of light in med1 nl a l a -2 1 1 - 2


a Þ + = Þ v = – 30 cm
mm = = = -15 v -10
Speed of light in med 2 nl m l m
Þ Image of O will be formed at 30 cm to the right at P.
[Q v does not change with the medium] 8. Since the image formed is real and elongated, the situation
la 6000 is as shown in the figure. Since the image of B is formed at B'
\ lm = = = 4000Å itself
a
mm 1.5

ca 3 ´ 108
\ na = = = 5 ´ 1014 Hz B' A'
l a 6000 ´ 10 - 10
P A B
4. For coherent sources, for constructive interference
f/3
The amplitude at the mid point = A + A = 2A
Þ I1 µ (2A)2 Þ I2 µ 4I0 ... (i)
NOTE : For incoherent sources, the intensity add up \ B is situated at the centre of curvature that is at a
normally (no interference). distance at 2f from the pole.
Therefore, the total intensity I2 = 2I0 ... (ii) f 5f
From (i) and (ii) \ PA = 2 f - =
3 3
I1 4 I 0 5f
= =2 Let us find the image of A. For point A, u = - ,v = ?
I2 2I0 3
1 1 1 1 1 1
m gm1.5 Applying, + = Þ + =
5. gm = = = 1.125 u v f - 5 f v -f
mm 43 3
1 æ 1 1ö 1 1 3
= (1.5 - 1) ç - ÷ [Lensmaker's formula] Þ =- + Þ v = – 2.5f
15 è R1 R2 ø v f 5f
EBD_7036
P-S- 238 Topic-wise Solved Papers - PHYSICS
Image length = 2.5 f – 2f = 0.5f B. True/ False
\ Magnification = 0.5 f = 1.5 1. This is due to atmospheric refraction. The light coming from
f 3 sun bends towards the normal. Therefore, sun appears
æ A + dm ö æ 60 + dm ö higher.
sin ç ÷ sin ç
è 2 ø è 2 ÷ø 2. KEY CONCEPT : Formula for intensity of a line source of
9. m= , 2=
sin A 2 sin 60 2 power (P) at a distance r from the source is
60 + dm P
\ = 45° Þ dm = 30° I=
2 2 prl
Þ The condition is for minium deviation. In this case the
ray inside the prism becomes parallel to base. Therefore the 3. The image formed by the convex lens at the focus of the
angle made by the ray inside the prism with the base of the concave lens. Therefore I will act as a virtual object for
prism is zero. concave lens and angle will be formed at infinity.
10. KEY CONCEPT :The resolving power of a microscope
device is inversely proportional to the wavelength used.
Þ The resolving power of an electron microscope is
higher than that of an optical microscope because the
wavelength of electrons is smaller than the wavelength of I
visible light.
Virtual
1 object
11. Velocity of light in vacuum c = for concave
lens
µ0 e 0 0.75m 0.25m
1
and the velocity of light in a medium v =
µe 4. NOTE : For the light to split, the material through which the
light passes should have refractive index greater than 1.
Velocity light in vacuum c 1/ µ0e0 µe
n= = = = Since the prism is hollow, we get no spectrum. The thickness
Velocity light in medium v 1/ µe µ0e0 of glass slabs through which the prism is made can be
12. Frequency remains the same neglected.
c 3 ´ 108 5. When the two slits of Young's double slit experiment are
f = = = 5 ´ 1014 Hz illuminated by two different sodium lamps, then the sources
l 6000 ´ 10 -10
are not coherent and hence sustained interference pattern
l 6000Å
and l 2 = 1 = = 4000Å will not be achieved. It will change so quickly that there will
µ 1.5 be general illumination and hence interference pattern will
13. P1 + P2 = 10 m– 1 not be observed .
P1 + P2 – (0.25) P1P2 = 6m – 1 6. In Young's double slit experiment if source is of white light
From these two expressions, we get than the central fringe is white with coloured fringes on
P1 P2 = 16m –2 either side.
P1 - P2 = ( P1 + P2 ) 2 - 4 P1 P2 1 1 1
7. = +
F f1 f 2
= (10-1 )2 - 4(16-1 ) = 6m -1
\ P1 = 8m – 1 and P2 = 2 m – 1, Hence 1 1 1 -2 + 1
Þ = + = Þ F = – 30 cm.
F -15 30 30
1 1 2 1
f1 = = m = 0.125 m and f 2 = = m = 0.5m This combination behaves as a concave lens of focal length
P1 8 P2 2 30 cm.
14. Using Snell's law for the refraction at AC, we get Since Fv < Fr.
µ sin i = (1) sin r \ One sees coloured pattern with violet colour at the
outer edge.
2 sin 30° = sin r Þ r = 45°
Angle of deviation at face AC C. MCQs with ONE Correct Answer
= 45° – 30° = 15° 1. (a) NOTE :
A
When the ray enters a glass slab from air, its frequency
o
30 remains unchanged.
Since glass slab in an optically denser medium, the
velocity of light decreases and therefore we can
o
60 conclude that the wavelength decreases.
30
o r = 45o (Q n = nl)
2. (a) The phenomenon of total internal reflection takes place
during reflection at P.
B C
Ray and Wave Optics P-S- 239

1 9. (c) NOTE : A convex mirror and a concave lens always


sin q = produce virtual image.
w ...(i)
gµ Therefore, option (b) and (d) are not correct. The image
by a convex lens is diminished when the object is placed
l
beyond 2f.
B A Let u = 2f + x
q
1 1 1 1 1 1
q - =
v u f Þ v - -(2 f + x) = f
Using
R P

1 1 1 2f + x- f ( f + x)
Þ = + = =
v f 2f +x f (2 f + x ) f (2 f + x )
C But u + v = 1 (given)
a f (2 f + x)
w gm 1.5 1 8 (2 f + x ) + £1
Now, gµ = = = 1.125 \ sin q = = f +x
a 4/3 1.125 9

é f ù (2 f + x) 2
2 f + x ê1 + £1 Þ £1
\
8
sin q should be greater than . ë f + x úû f +x
9
Þ (2f + x)2 £ f + x. The above is true for f < 0.25 m.
lD l (2 D) lD 10. (a) Here fo = 2 cm and fe = 3 cm.
3. (d) b= , b' = =4 = 4b
d d /2 d Using lens formula for eye piece
æ 1ö -1 1 1
-1 Þ + = Þ ue = – 3 cm
4. (a) C = Sin ç1 ÷ ..... (i) ue µ 3
è 2µø But the distance between objective and eye piece is
Applying Snell's law at P, we get 15 cm (given)
\ Distance of image formed by the objective
1 sin r ' sin (90 - r )
m= = [Q i = r; r' + r = 90°] = v = 15 – 3 = 12 cm.
2 sin i sin r Let u be the object distance from objective, then for
1 cos r objective lens
\ m= ...(ii)
2 sin r 1 1 1 -1 1 1
- + = or + =
From (i) and (ii) u0 v0 f 0 u 12 2
C = sin –1 (tan r)
5. (c) Let I1 = I and I2 = 4I -1 1 1 5 12
Þ = - = , u = - = -2.4 cm
u 2 12 12 5
( ) =( ) = (3 I )
2 2 2
I max = I1 + I 2 I + 4I = 9I 11. (c) Path difference between the opposite edges is l.
For a phase difference of 2p we get a path diff. of l.
( ) =( )
2 2
I min = I1 - I 2 I - 4I =I 12. (c) We know that
6. (c) Spherical aberration occurs due to the inability of a d 2pd tan q
I (q) = I0 cos 2 where d =
lens to converge marginal rays of the same wavelength 2 l
to the focus as it converges the paraxial rays. This can
be done by using a circular annular mask over the lens. æ pd tan q ö 2 æ p ´ 150 ´ tan q ö
7. (d) The distance between the first dark fringe on either I (q) = I 0 cos 2 ç ÷ = I 0 cos çè ÷
è l ø 3 ´ 108 /106 ø
side of the central maximum = width of central maximum

2 Dl 2 ´ 2 ´ 600 ´ 10-9 æp ö
= = I0 cos 2 ç tan q÷
= = 2.4 × 10 – 3 m = 2.4 mm è2 ø
a 10-3
8. (c) Applying Snell's law at P, 30°
sin r
m=
sin 30° P r y
d S1
1.44 30°
d/2
sin r = = 0.72 q
2
d/2 D
\ d = r – 30 ° = sin – 1 ( 0.72) – 30° S2
\ The rays make an angle of
2d = 2 [sin – 1(0.72) – 30 °] with each other..
EBD_7036
P-S- 240 Topic-wise Solved Papers - PHYSICS

æ p ö x d /2
For q = 30°; I(q) = Io cos2 çè \ = or x = d
÷ 2L L
2 3ø
So, required distance = d + d + d = 3d.
For q = 90° ; I(q) = Io cos2 (¥)
For q = 0° 18. (b) 2
n1 a/
I (q) = I0 a/2
I (q) is not constant.
n2
Alternatively, when q is zero the path difference r
between wave originating from S1 and that from S2 will a/2
be zero. This corresponds to a maxima.
a /2
13. (a)
1
f
= ( m
)
g µ -1
æ 1 1 ö
çè R - R ÷ø Now, g µ =
m gµ
=
1.5
The incident and emergent ray of a glass slab are
1 2 m µ 1.75 parallel therefore, the angle remains the same.
For concave lens as shown in figure in this case 19. (a) See figure. The ray will come out from CD if it suffers
R1= – R and R2 = + R total internal reflection at surface AD, i.e., it strikes the
surface AD at critical angle C ( the limiting case).
gm P D
mm A
C n n2
1
R1 R2 Q 90°–C
MEDIUM
a
mm
B C
1 æ 1.5 ö æ 1 1ö 0.25 ´ 2 Applying Snell's law at P
\ =ç - 1÷ ç - - ÷ = +
f è 1.75 ø è R R ø 1.75 R n2
n1 sin C = n2 or sin C =
Þ f = + 3.5 R n1
NOTE : The positive sign shows that the lens behaves
Applying Snell's law at Q
as convergent lens.
n2 sin a = n1 cos C
14. (d) For diffraction pattern to be observed, the dimension
of slit should be comparable to the wave length of n1 ïì æ n ö ïü
rays. The wavelength of X-rays (1 – 100 Å) is less than Þ sin a = cos ísin -1 ç 2 ÷ ý
n2 ïî è n1 ø ïþ
0.6 mm.
15. (a) Locus of equal path difference are lines running parallel
én ìï æ n ö üïù
to axis of the cylinder. Hence straight fringes will be or a = sin -1 ê 1 cos ísin -1 ç 2 ÷ ýú
observed. ëê n2 îï è n1 ø þïûú
1 æ µ2 ö æ 1 1ö 20. (a) When slits are of equal width.
16. (d) =ç - 1÷ ç - ÷
f è µ1 ø è R1 R2 ø m 1 m 2 m3 Imax µ (a + a)2 ( = 4a2)
If m2 > m1 , the concave lens Imin µ (a – a)2 ( = 0)
maintains its nature otherwise the When one slit's width is twice that of other
nature of th e lens will be
reversed. I1 W1 a 2 W a2
= = Þ = Þ b = 2a
So, the lens should be filled with L2 and immerse in L1. I 2 W2 b 2 2W b2
17. (d) From the ray diagram.
\ Imax µ (a + 2a )2 = (5.8 a2)
M
A 2L X N Imin µ ( 2a – a)2 = (= 0.17 a2)
d/2 21. (c) NOTE : The intermediate image in compound
D L B
microscope is real, inverted and magnified.
d/2
22. (b) I = I1 + I2 +2 I1 I 2 cos f ...(1)
X
Applying eq. (1) when phase difference is p/2
In DANM and DADB Ip/2 = I + 4I Þ Ip/2 = 5I
ÐADB = ÐANM = 90° Again applying eq. (1) when d phase difference is p
ÐMAN = ÐBAN (laws of reflection)
Also ÐBAN = ÐABD Þ ÐMAN = ÐABD I p = I + 4I + 2 I 4 I cos p
\ DANM is similar to DADB \ Ip = I \ Ip/2 – Ip = 4I
Ray and Wave Optics P-S- 241

PM2 = 4h2 + h2 = 5h2


12l1 D k l 2 D 12 ´ 600
23. (b) = , k= = 18
d d 400 h 1
\ sin i = = ...(ii)
24. (d) Applying Snell's law at P, 5h 5
1 sin i µ2 From (i) and (ii)
µ2 = = ...(1)
sin r1 µ1
1 5
µ=
m1 m2 m3 m4
2 2

r1 r2 i 27. (c) Since both surfaces have same radius of curvature on


r2 R the same side, no dispersion will occur.
Q
i1 P r1 28. (a) Path difference = (µ – 1) t = nl;
For minimum t, n = 1; \ t = 2 l

29. (b) Maximum number of reflection = 2 3


x
Applying Snell's law at Q,
where x = 0.2 tan 30° = 0.2 3.
2 sin r1 m3
m3 = = ...(2)
sin r2 m 2 l
x
Again applying Snell's law at R

3 sin r2 m 4
m4 = = ...(3)
sin i m3 0.2m 30° 30°
Multiplying (i), (ii) and (iii), we get
µ4 = µ1
NOTE : If the emergent ray is parallel to incident ray d
after travelling a number of parallel interfaces then the 30. (b) In D OPM, OP =
cos q
refractive index of the first and the last medium is always
same. d cos 2q
In DCOP, OC =
cos q
25. (c) Q
O M
Q R
q q
P
C
R
d

There will be no refraction from P to Q and then from Q


q
to R (all being identical). Hence the ray will now have
the same deviation. P

26. (b) For the image of point P to be seen by the observer, it


should be formed at point Q. Path difference between the two rays reaching P is
In DQNS,
l d cos 2q d l
NS = QS = 2h = CO + OP + = + +
N 2 cos q cos q 2
\ ÐNQS = 45°
\ r = 45° d l l
r = (cos 2q + 1) + = 2 d cos q +
Now in DQMA, cos q 2 2
M 2h
ÐMQA = 45° For constructive interference, path difference should
i be nl
\ MA = QA = h 45°
Q S 2h l (2n - 1) l
A \ 2d cos q + = nl Þ cos q =
1 sin r sin 45° h 2 4 d
m= = ...(i)
2 sin i sin i
l
In D PMB, P B For n = 1, cos q =
2h 4d
EBD_7036
P-S- 242 Topic-wise Solved Papers - PHYSICS
31. (b) Convex lens forms the image at I1. I1 is at the second 36. (c) NOTE : Frequency does not change with change of
focus of convex lens. Size of I1 = 2 cm. medium.
I1 acts as virtual object for concave lens. Concave lens 37. (a) The rays coming from the point object fall on the glass-
forms the image of I1 and I2. air interface normally and hence pass undeviated.
Therefore if we retrace the path of the refracted rays
backwards, the image will be formed at the centre only.
I1
I2 P

pdsin q A
4 cm 38. (c) I = Imax cos 2
26 cm l q
d
30 cm in
q
B ds
For concave lens,
I max æ pd sin q ö
1 1 1 1 1 1 4 1 Þ = I max cos 2 ç ÷
- =- or = - + = = 4 è l ø
v 4 20 v 20 4 20 5
or v = 5 cm = Distance of I2 from concave lens.
v size of image 5 pd sin q 1 pd sin q p
\ cos = \ =
\ Magnification = = = l 2 l 3
u size of object 4
æl ö
size of image \ q = sin -1 ç ÷
or = 1.25 è 3d ø
2
or size of image due to concave lens = 2.5 cm | f1 | 2
39. (a) =
32. (b) µg sin i = m air sin 90° | f2 | 3
f 1 : focal length of convex lens.
1
mg = 1 1 1 1 1 2
sin i = - Þ = -
f f1 f 2 30 f1 3 f1
1 1
34. (c) KEY CONCEPT : sin C = andµ µ f1 = 10 cm, f2 = –15 cm
µ l 40. (c) The image I ' for first refraction (i.e., when the ray comes
\ sin C µ l out of liquid) is at a depth of
For higher value of l, the angle C also increases
33.25 é Re al depth ù
= = 25 cm êQ Apparent depth = ú
l increases 1.33 ë m û
Air
(Green light) Now, reflection will occur at concave mirror. For this I'
V I B G Y O R C behaves as an object.
Glass \ u = – (15 + 25) = – 40 cm

34. (b) NOTE : For minimum deviation, incident angle is equal é 25 ù


and v = - ê15 +
to emerging angle and QR is parallel to base. ë 1.33 úû
35. (d) At the area of total darkness minima will occur for both
the wavelengths. 25
Where is the real depth of the image.
1.33
(2 n + 1) (2 m + 1)
\ l1 = l 2 Þ (2n + 1)l1 = ( 2m +1) l2 Using mirror formula we get
2 2
1 1 1
(2n + 1) 560 7 = + , f = – 18.31 cm
or = = or 10n = 14m + 2 f v u
(2m + 1) 400 5 41. (b) The focal length f of the equivalent mirror is
by inspection for m = 2, n =3 and for m = 7, n = 10, the
distance between them will be the distance between 1 2 1 2 1 15
= + = + Þ f= cm
such points. f f1 f m 15 ¥ 2
Since f has a positive value, the combination behaves
Dl1 ì (2n2 + 1) - (2 n1 + 1) ü
i.e., Ds = í ý as a converging mirror.
d î 2 þ
15
put n2 = 10, n1 = 3 Here u = – 20cm, f = - cm , v = ?
2
On solving we get, Ds = 28mm.
Ray and Wave Optics P-S- 243

1 1 1
According to mirror formula + = Movement towards left
v u f
1 1 1 F
Þ - = Þ v = – 12 cm
v -20 -15 / 2 I C A P
Negative sign indicates that the image is 12 cm in front
of mirror.
42. (c) We know that in case of a convex lens when object is
placed at C ', the image is obtained at C. This situation 45. (c) The ray is partly reflected and partly refracted
is represented in the graph by the point corresponding ÐMOB = 180 – 2q
to u = –10 cm, v = 10 cm. ed ray
M e fr act
t ly r P
R P ar
Therefore R = 10 cm Þ = 5cm = f
2 O
180–2q
q q

Pa
31

rtl
ay

y
v cm

tr

re
fle
en
cid

c te
30

In

d
ray
A B
But the angle between refracted and reflected rays is
ÐPOB. Clearly is ÐPOB is less than ÐMOB.
10
46. (a)
60°
0 (–9, +9)
u cm –31 –30 –20 –10 60°
iv r=
Lens formula is 30
º

1 1 1 Df Dv Du
= - Þ = 2 + 2 Medium 1 Medium 2
f v u 2
f v u
(for maximum error in f) 60°
Df 0.1 0.1 60°
Þ = + [Du = Dv = 0.1 from graph) ir
r=
25 (10)2 (10) 2 30º

Þ Df = 25 × 0.1 × 2 × 0.01 = 0.05


Therefore, the focal length = (5.00 ± 0.05) cm. Medium 1 Medium 2

AB For minimum deviation the ray in the prism is parallel to


43. (b) From the figure in D ABC, tan b = the base of the prism. This condition does not depend
AC
Þ AB = AC tan b, 2r = f tanb on the colour (or wave length) of incident radiation. So
Þ Area of image = pr2µ f 2 in both the cases, by geometry, r = 30º. So (a) is correct
option.
Parallel 47. (b) For refraction at parallel interfaces
Rays from n0 n0
Focal n0
object placed n0 sinq = sin a = sinb = sin 90º
plane 2 6 8
at infinity
f
A a b b
on focal plane

2r C
Image formed

1 b
\ sinq = a
B 8 a
q

Biconvex lens 48. (c) Consider the activitiy A to B


Applying v 2 - u 2 = 2 as
44. (b) As shown in the figure, when the object (A) is placed
between F and C, the image (I) is formed beyond C. It v 2 - 0 2 = 2 ´ 10 ´ 7.2 Þ v = 12 m/s
is in this condition that when the student shifts his The velocity of ball as perceived by fish is
eyes towards left, the image appears to the right of the
4
object pin. (Image distance > object distance) v ' = wm ´ v = ´ 12 = 16 m/s
3
EBD_7036
P-S- 244 Topic-wise Solved Papers - PHYSICS

é1 1 ù 1
A u=0 = (1.2 - 1) ê - úû = 70
a = 10 m/s
2
ë ¥ -14
s = 7.2 m The focal length F of the combination is
20 m
B 1 1 1 1
= + =
v F f1 f 2 20
12.8 m Applying lens formula for the combination of lens
1 1 1 1 1 1
- = Þ - =
V U F V -40 20
fish
Þ V = 40 cm
49. (b) Focal length of the biconvex lens is 15 cm. A small lD
52. (d) We know that b =
object is placed at a distance of 30cm from the lens i.e. d
at a distance of 2f. Therefore the image should form at Now, lR > lG > lB
30cm from the lens at I1. \ bR > bG > bB

æ ˆi 3 ˆö æ ˆi 3 ˆö
ç 2 + 2 j÷ .ç 2 - 2 j÷
è ø è ø
O I3 I2 I1 53. (a) cos(180° - 2a ) =
2 2 2
æ 1ö æ 3ö æ 1ö æ 3ö
çè ÷ø + ç ÷ çè ÷ø + ç - ÷
10cm 2 è 2 ø 2 è 2 ø
But since the ray strike the plane mirror before reaching
1
I1, the image I1 acts as the virtual object for reflection \ cos(180° - 2a ) = - 180°-2a
2
on plane mirror kept at a distance of 20 cm from it.
\ 180° – 2a = 120° a a
It should produce an image I2 but as the ray encounters
\ a = 30°
the lens, it gets refracted and the final image is formed option (a) is correct
at I3. For the last refraction from the biconvex lens, 54. (b) The intensity I is given as
u = 10 cm.
f
I = Io cos 2 where Io is the peak intensity
1 1 1 2
Applying lens formula - =
v u f Io I f p
Here I = , \ o = I o cos 2 \ f = (2n + 1)
2 2 2 2
1 1 1 1 1 1 25 For a phase difference of 2p the path difference is l
Þ - = Þ = + =
v 10 15 v 15 10 150 p
\ For a phase difference of (2n+1) the path
Þ v = 6 cm. 2
Therefore a real image is formed at a distance of 16 cm difference
from the plane mirror. l
is (2n + 1) . option (b) is correct.
50. (c) When the light is incident on glass - an interface at an 4
angle less than critical angle a small part of light will be 55. (c)
reflected and most part will be transmitted. n2
When the light is incident greater than the critical angle, 11.54 mm
it gets completed reflected (total internal reflection) 2
These characteristics are depicted in option (c).
51. (b) The focal length (f1) of the lens with n = 1.5 is given by P
q
1 é 1 1 ù 10 mm
= (n1 - 1) ê - ú n1 q
f1 ë R1 R2 û
é 1 1ù 1
= (1.5 - 1) ê - ú = Applying Snell’s law at point P
ë 14 ¥ û 28 n1 sin q = n2sin 90°
The focal length (f 2 ) of the lens with n = 1.2 is
11.54 / 2
given by \ n2 = 2.72 ´
2
(10 )2 + æç
é 1 11.54 ö
1 1 ù ÷
= (n2 - 1) ê - ú è 2 ø
f2 ë R1 R2 û
\ n2 = 1.36
Ray and Wave Optics P-S- 245

56. (a) Applying Snell's law at A D. MCQs with ONE or MORE THAN ONE Correct
1 × sin 45° = 2 ´ sin r1
\ r1 = 30° … (i) 1.
I
(b, d) max =
( I1 + I 2 )2 = 9
I min
( I1 - I2 )
2 1

q I1 a2 a
\ =4= 2 Þ =2
90° – q I2 b b
45° M
A r1 90°
+q 1 1 1 1 1
c B 2. (a) P= = + = + = -1.5 dioptre.
f f1 f 2 0.4 -0.25

n= 2 lD ld
3. (a, c)Here y = (2n - 1) = (2n - 1)
2 d 2b
(Q d = b and D = d)

Applying Snell's law at B b


But y =
2
2 sin C = 1 × sin 90°
\ C = 45° … (ii)
In D AMB, 90° + q + r1 + (90° – C) = 180° (From fig.) b ld
\ = (2n - 1)
\ q = 15° 2 2b
1 1 1
57. (c) For lens - =
Þ
b2
v u f l= when n = 1, 2
(2n - 1)d
1 1 1
- =
v -50 30 b2 b2
l= , ,...
\ v = 75 cm d 3d
4. (b, d) The image formed will be complete because light rays
from all parts of the object will strike on the lower half.
Principal axis of mirror
I¢ But since the upper half light rays are cut off, the intensity
will reduce.
1 1 1 dv du
5. (d) + = \- 2
- =0
50 cm v u f v u2
30° 2
dv - v 2 æ f ö
\ = 2 = -ç
30° du u è u - f ÷ø
I A M
2
Principal
image length = çæ
f ö
axis of \ ÷ ´b
èf -uø
lens
6. (a) For total internal reflection
MIRROR
1 1
m= = = 1.414
Therefore object distance for mirror is 25 cm and object sin C sin 45°
is virtual i.e. for an angle of incidence of 45°, that colour will suffer
total internal reflection for which the refractive index is less
1 1 1 1 1 1 than 1.414.
For minor + = \ + = \ v = – 50 cm A
v u f v 25 50
The image I would have formed as shown had the mirror
been straight. But here the mirror is tilted by 30°.
Therefore the image will be tilted by 60° and will be i=45o
formed at A.
Here MA = 50 cos 60° = 25 cm B
and I¢A = 50 sin 60° = 25 3 cm Therefore, red light will be refracted at interface AB whereas
blue and green light will suffer total internal reflection.
EBD_7036
P-S- 246 Topic-wise Solved Papers - PHYSICS
7. (d) In an astronomical telescope when the object and final
f 2 ( f1 - d )
image are at infinity, M and L are given as shown:
v f + f -d f2
Angular magnification M = fo/fe m= 2 = 1 2 =
Seperation between lenses, L = fo +fe u2 f1 - d f1 + f 2 - d

fo h2 D ´ f2
\ = 5 or f = 5 f . ... (i) Also m = Þ h2 =
fe o e
D f1 + f 2 - d
fo + f e = 36 or 5 f e + f e = 36 \ The y-coordinate y = D – h2
or fe = 6 cm ... (ii) Df 2 D ( f1 - d )
= D- =
\ f o = 5 f e or fo = 30 cm f1 + f 2 - d f1 + f 2 - d
Hence fo = 30 cm, fe = 6 cm 11. (b, c) NOTE : Concave lens and convex mirror are diverging
8. (c) The angle of deviation for the first prism P1 in nature. Therefore the refracted/reflected rays do not meet.
d1= (µ1 – 1) A1 These rays are produced backwards to make them meet.
The angle of deviation for the second prism P2 Therefore the image formed is virtual and erect.
d2= (µ2 – 1) A2 12. (b) Spherical aberation is smaller when the curved surface
Since total deviation is to be zero is facing the object because the total deviation is shared
\ d1+ d2= 0 between the two surfaces.
Þ (µ1 – 1) A1 + (µ2 – 1) A2= 0 13. (c, d) KEY CONCEPT : For total internal reflection to take
place :
-(1.54 - 1)
Þ A2 = 4° = -3° Angle of incidence i > critical angle, qc
(1.72 - 1)
9. (a, b, c, d) In case of an astronomical telescope the distance 1 1 1
or sin 45° > or > or n > 2 or n > 1.414.
between the objective lens and eyepiece lens n 2 n
= f0 +fe = 16 + 0.02 = 16.02 m Therefore, possible values of n can be 1.5 or1.6 in the
given options.
f objective -16
The angular magnification = – = = -800 14. (d) For first minima the path difference between the rays
f eye piece 0.02 coming from the two edges should be l which
corresponds to a phase difference of 2p.
NOTE : The image seen by the astronomical telescope is
15. (d) The ray diagram is shown in figure. Therefore, the
inverted. Also the objective lens is larger than eye piece
image will be real and between C and O.
lens.
10. (c) The image I1 of parallel rays formed by lens 1 will act as Normal at P will
virtual object. pass through C
2 C C
I
1 Image

P
O O
I1
I 16. (a) The formula for spherical refracting surface is
- µ1 µ2 µ2 - µ1
d + =
f1 u v R
Applying lens formula for lens 2
m1 m2
1 1 1 f 2 ( f1 - d ) Glass
Air
Þ - = Þ v2 =
v2 f1 - d f 2 f 2 + f1 - d
P O C Q (Real Image)
\ The horizontal distance of the image I from O is u v

f 2 ( f1 - d ) f f + d ( f1 - d )
x=d+ = 1 2
f 2 + f1 - d f1 + f 2 - d Here u = –x, v = + x, R = + R, µ1 = 1, µ2 = 1.5

To find the y-coordinate, we use magnification formula for -1 1.5 1.5 - 1


+ = Þ x = 5R
lens 2 -x x R
Ray and Wave Optics P-S- 247

17. (a,b) The condition to obtain maxima in the phenomenon For (78, 39) when u = 78 then
observed in young's double slit experiment is
1 1 1
d sin q = nl where n is an integer + =
v -78 -24
Þ v = – 34.67
The absoluate error is 39 – 34.67 = 4.33 which is greater
than 0.2 cm.
19. (a,b,c)
4I Applying Snell’s law at P
q
n1 sin 60° = n 2 sinq
d
Þ sin 60° = 3 sin q
inq Þ q = 30°
I ds
In quadrilateral BCQP,
B
Screen C
60° 60°
When d = l
P 30°= q 135°
l sin q = nl
Þ sin q = n i
When n = 0, q = 0 45° 45°
30°
When n = 1, q = 90° (This will be a point on the screen
75°
which will be at infinity and therefore not practical)
A aM D
Other values of n are invalid as –1 £ sin q £ 1 .
ÞThe screen will have only one maxima. 60° + (90° + 30°) + 135° + ÐPQC = 360°
When λ < d < 2λ
Þ ÐPQC = 45° Þ i = 45°
nl é nl ù The critical angle for prism - air pair of media is
Þl <
sin q
< 2l êëQ d = sin q úû
-1 æ 1 ö
C = sin çè ÷ø which is less than 45°.
n 3
Þ1< <2 Therefore total internal reflection takes place at face
sin q
CD.
The possible values of n are 0, + 1, –1. option (a) is correct.
ÞThere is at least one more maxima (besides the central In DQDM, ÐQMD = 180° – (45° + 75°) = 60°
maxima, option [B] is correct. Therefore the angle of incidence of ray QM on AD is
We know that 30°.
This angle is less than the critical angle. Therefore the
( ) ( )
2 2
Imax = I1 + I2 , Imin = I1 - I2 ray emerges out of face AD.
Option (b) is correct.
Initially I1 = 4 I and I2 = I Applying Snell’s law at M, we get
\ Imax = 9 I and Imin = I
3
When I1 = I2 = I then Imax = 4 I and Imin= 0 3 sin 30° = 1´ sin a Þ a = sin -1 = 60°
i.e., when the intensities become equal, Imin reduces to 2
In quadrilateral PQMN, ÐPNM = 360° – [60° + 90°
zero. Options [C] and [D] are incorrect.
+120°] = 90°
18. (c, d)Given f = – 24 cm \ The angle between the incident ray and the emergent
1 1 1 ray is 90°
Applying + = Option (c) is correct.
v u f 20. (a, c) There will be no effect of the transparent thin film of
For (66, 33) uniform thickness and refractive index n 1 = 1.4
1 1 1 1 1 -66 + 24 -42 n1 n 2 n 2 - n1
Þ = - = + = = Therefore, - + = [For case (i)]
v f u -24 66 24 ´ 66 24 ´ 66 u v R
1 1.5 1.5 - 1
24 ´ 66 Þ - + = Þ f1 = 3R
Þ v=- = -37.7 ¥ f1 R
42
\ (a) is a correct option
But the value of v = 33. The absolute error is 37.7 – 33
= 4.7 cm which is greater than 0.2 cm. Therefore a wrong n1 n 2 n 2 - n1
Again applying - + = [For case (ii)]
reading. u v R
EBD_7036
P-S- 248 Topic-wise Solved Papers - PHYSICS
24. (a, c, d)
1.5 1 1 - 1.5
- + = Þ f 2 = 2R n1 sin qi = n2 sin q f [Q 1 and 2 interfaces are parallel]
¥ f2 -R
l depends on the refractive index of transparent slab but not
\ (c) is a correct option. on n 2 . In fact qf depends on n2.
21. (a, b, c) 25. (b,d)
lD Path difference at O = d = 0.6003mm
We know that b = Now, = mm = 300×10–6 mm
d
For n = d we get n = 2001
As l2 > l1 \ b2 > b1 \ (a) is correct option. As n is a whole number, the condition for minima is satis-
Therefore m1 > m2 \ (b) is correct option. fied.
l2 D ( 2 ´ 5 - 1) l1 D Therefore ‘O’ will be dark.
As 3 ´ = Also, as the screen is perpendicular to the plane containing
d 2 d the slits, therefore fringes obtained will be semi-circular (Top
3 × 600 = 4.5 × 400 \ (c) is correct option. half of the screen is available)
l E. Subjective Problems
The angular width = \ (d) is incorrect option.
d
1. The focal length of the equivalent mirror is
22. (b) For refraction in S1 1 2 1
= +
n1 n2 n2 – n1 1.5 1 1 - 1.5 F f fm
– + = Þ– + =
u v R –50 V –10 2 2 1 1 21
= + = + =
Þ v = 50 cm. 20 22 10 11 110
For refraction in S2
110
Þ F=
n n n – n1 21
– 1+ 2 = 2
u v R NOTE : Since the focal length is positive it is a converging
mirror
1 1.5 1.5 –1
– + = 1 1 1 1 1 1
–(d – 50) µ 10 Now, + = Þ + =
u v f -10 v -110 / 21
1 1
\ = 1 1 21
d - 50 20 Þ = - Þ v = – 11cm
v 10 110
\ d = 70 cm. NOTE : The negative sign indicates the image is real.
B is the correct option. 2. The situation can be shown as in the figure.
23. (a, d)
For lens A

1 1 1 1 1 1
- = Þ - =
v u f v -30 f
d
i e
v v r r'
Also m = Þ -2 =
u u
B C
On solving we get f = + 20 cm and v = 60 cm.
n Here, i = 60°, A = 30°, d = 30°, e = ?
For reflection
We know that, A + d = i + e ....(1)
1 1 1 2 1 1 2 Also, A = r + r' ....(2)
+ = = Þ + = Þ R = 30 cm
v u f R 10 -30 R From (1),
e = A + d – i = 30° + 30° – 60° = 0
The image formed by convex side is faint erect and virtual.
As the angle of emergence (e) is 0, hence the emergent ray
By lens maker formula
is normal to the face from which it emerges.
1 æ nl öæ 1 1 ö When e = 0, r' = 0
= ç - 1÷ ç - ÷ \ From (2), A = r = 30°.
f è ns ø è R1 R2 ø From Snell's law, refractive index of prism,
1 æ nl öæ 1 ö sin i sin 60° 3/2
\ = ç - 1÷ ç ÷ \ n = 2.5 µ= = = = 3 = 1.732.
20 è 1 ø è 30 ø l sin r sin 30° 1/ 2
Ray and Wave Optics P-S- 249

3. A C 20 æ 1 ö 20 23
Vertical + t ç1 - ÷ , +1 = = 7.67 cm
side
3 è µø 3 3
C
6. (i) KEY CONCEPT :The given silvered concavo-convex
90–C
lens behaves like a mirror whose focal length can be
q 2m
B D
M 1m 1 2 1
calculated by the formula f = f + f
1 2
For a grazing incident ray at BD for which i » 90° the angle of f1 = focal length of concave surface.
refraction (90 – C) is maximum. For this C is least. Let C is f2 = focal length of concave mirror
greater than the critical angle.
Applying Snell's law at M
1 sin 90° 1 1
µ= Þ µ = ...(i) R1=60 cm
2 sin(90 - C ) 2 cos C
1 1 R 2=20 cm
Also µ= ...(ii)
2 sin C
When C is the critical angle.
1 1 1 2 1 4
From (i) and (ii), = Þ C = 45° \ = + =-
cos C sin C f -30 -10 30
\ f = – 7.5 cm
1 1 Using mirror formula
\ µ= = 2 = 1.41
2 sin 45°
1 1 1 1 1 1
4. For case (i), there is no refraction. Therefore µ1 = µ = + Þ = +
NOTE : Here the convex lens behaves as a diverging lens. f v u -7.5 - x - x
Therefore, µ < µ2. x = 15 cm
5. The rays originating from A (the point object) suffer (ii) Let the object distance be u. When water is poured
refraction before striking the concave mirror. over the concave surface the apparent object distance will
For the mirror the rays are coming from A' be v then
m1 m 2 m 2 - m1
- + =
u v R

O For flat surface R = ¥


A' A
µ1 µ2
\ - + =0
u v
æ 1ö m 1 4
such that AA ' = shift = t ç1 - ÷ Þ v =u 2 =u´ m = u´
è µø m1 2 3
Therefore the object distance Since the ray enters the lens from water into glass
æ 1ö
u = OA ' = OA - AA ' = 21 - t ç1 - ÷ -m w m g m g - m w
è mø + =
u v R
æ 1 ö
= 21 - 3 ç1 - ÷ = 20cm -4 / 3 1.5 1.5 - 4 / 3
è 1.5 ø Þ + = Þ u = – 13.85 cm
4 -20 -60
uf 20 ´ 5 20 u
v= = = cm = 6.67cm 3
\
u - f 20 - 5 3 \ Downward shift = 15 – 13.85 = 1.15 cm.
The reflected rays again pass through the glass slab. The 7. The total intensity at point P will be
image should have formed at B is the absence of glass slab. = IA + IB + I C
C
But. due to its presence the image is formed at B'. (Illumination power) ´ cos q 20W
IA =
4pr 2
90 ´ cos 0 60O
= P
O
B B' 4p´ 32 A 90W
O
60
10
= watt / m 2
4p
Therefore image distance = OB + BB'
B 180W
EBD_7036
P-S- 250 Topic-wise Solved Papers - PHYSICS

180 ´ cos 60° 10 critical angle


IB = = watt / m 2 Case (i) : When n3 < n1
4p ´ (1.5) 2 p Obviously n3 < n2 and the angle q is greater than the critical
IC = 20 cos 60° = 10 angle required for the ray passing from medium II to medium
10 10 III. Therefore total internal reflection will also take place
\ Ip = + + 10 = 13.9W / m 2 when a ray strikes with the same angle at the interface of
4p p
8. Here R = µ i.e., plane surface is the refracting surface medium II and medium III.
Case (ii) : n3 > n1 but n3 < n2
The ray will get refracted in medium III as the angle q will
now be less than the critical angle required for medium II
v and medium III pair.
u
I
Med I R
O
Med III
µ µ µ -µ r
µ µ
- 1+ 2 = 2 1 Þ - 1 + 2 =0 r
u v R -4 -3
µ2 3 G P Med II F
\ = ...(i)
µ1 4 q
Again applying
sin q n3
µ µ µ -µ 1 µ /µ (µ / µ ) - 1 \ = (Applying Snell's law at P)
- 1+ 2 = 2 1 Þ - + 2 1= 2 1 sin r n2
u v R u v R
n2
1 3/4 3 / 4 -1 \ sin r = sin q
Þ - + = n3
-4 -25 / 8 R
As n2 > n3 So, r > q
When the refracted ray PR meets the boundary DE, it is
travelling from a denser medium to a rarer medium. Therefore
the ray will be totally internally reflected at DE if its angle of
u v
I
incidence r is more than the critical angle for med III and I.
O
n1
sin i'' = n
3
On solving we get R = – 25cm.
Applying Len's maker formula, n1
Since, sin r > n Þ sin r > sin i'' Þ r > i''
1 æ 1 1ö 3
= (m - 1) ç - ÷ = æç - 1öæ
4 1 1ö
- \ f = 75cm Therefore ray PR will be totally internal reflected along RQ.
f è R1 R2 ø è 3 øè 25 µ ÷ø
֍
On reaching Q, the ray will be refracted in med II. Thus, the
9. (i) The distance of the nth bright fringe from the central ray will ultimately be reflected back in medium II.
maxima is given by the expression 11. (i) Let x is the incident angle for reflection at AC. For total
internal reflection x > iC (critical angle)
n lD
yn = , For 3rd bright fringe n = 3
d
3 ´ 6500 ´ 10-10 ´ 120 ´ 10-2 B
\ y= = 1.17 × 10–3 m A x y
2 ´ 10-3 x
90–x y 90–y
(ii) Let nth bright fringe of wavelength 6500 Å coincide y
M
with mth bright fringe of wavelength 5200Å. Their N
distance will be same from the central bright. Therefore, C
nl1 D ml 2 D n 5200 4 Let y be the incident angle of the ray on face CB. For total
= \ = = internal reflection
d d m 6500 5 y > iC
i.e., at the least distance 4th bright fringe of 6500 Å will \ x + y > 2iC
coincide with 5th bright fringe of 5200 Å. Its distance But x = ÐA and y = ÐB (from geometry)
from the central maxima will be \ x + y = 90°
Þ 90 > 2iC Þ iC < 45°
4 ´ 6500 ´ 10-10 ´ 120 ´ 10-2
yn = = 1.56 ´ 10 -3 m The refractive index of the medium for this to happen.
2 ´ 103 1 1
10. KEY CONCEPT : For total internal reflection, the conditions µ= = = 2
sin iC sin 45°
are
· The object should be in the denser medium. 5
· The angle of incidence should be greater than the (ii) µ =
3
Ray and Wave Optics P-S- 251

1 1 3 14. As shown in the figure, the interference will be between


Þ sin iC' = = = Þ iC' = 37° 0.25 I = I1 and 0.14 I = I2
µ 5/3 5 Glass Glass
Plate Plate I
y = 30° (Given) \ x = 60°
x > iC' but y < iC'
( )
Þ Total internal reflection will take place on face AC but 2
I max I1 + I 2 0.75I 0.25I
not on CB. =
12. (i) Initially the object is in denser medium and
u = ¥ using the formula of refraction at a spherical
I min
( I1 - I2 )
2

surface for AB
0.1875I
µ µ µ - µ2 -4 / 3 1 1 - 4 / 3 [ 0.25 I + 0.14 I ]2 49
- 2+ 1= 1 Þ + = = = . 0.14I
u v R -¥ v 2 [ 0.25 I - 0.14 I ]2 1
Þ v = – 6 mm
NOTE : This is the position of the image due to refraction at
the first surface. This image will behave as a virtual object 15. (a) l1 = 4000 Å and l2 = 5000 Å
for the refraction at the second surface.
u = – 6 – 4 = – 10 mm q
Again using the formula of refraction at a spherical surface
for CD l1 M
q
4 d q
µ1 µ2 +µ2 - µ1 -1 N r
- + = 1
, - + 4 / 3 3
u ' v' R = l2
Q q=i
i
d
10 v ' -2
Þ v' = – 5 mm.
The is the position of final image. For total internal reflection to take place, q should be
(ii) Ray Diagram. greater than C. For smaller values of C, the values of m
should be high or in other words the value of l should
A be small.
C Therefore, total internal reflection will be given by
V' l1 = 4000 Å
I'
V
I 2mm 2mm Here, sin q = 0.8 (given) Þ q = 53.1°
AIR
BUBBLE
1 1
B \ µ= = = 1.25
WATER D sin q 0.8
13. The power transmitted through A b
\ µ = 1.2 + = 1.25
é æ 10 ö ù (4000 ´ 10 -10 ) 2
= ê10% of ç ÷ ú ´ p (0.001) 2 = 10–6 W
ë è p øû Þ b = 0.8 × 10–14 m2
The power transmitted through B (b) Applying Snell's law at N for wavelength l2

é æ 10 ö ù 2 sin r 0.8 ´ 10 -14


= ê10% of ç ÷ ú ´ p ´ (0.002) = 4 × 10–6 W µ= where µ = 1.5 + = 1.232
ë è p øû sin i (5000 ´ 10-10 ) 2
Let Df be the phase difference introduced by film
sin r
2p Þ 1.232 = Þ r = 80.3°
\ Df = (path difference introduced by the film) 0.8
l From the figure it is clear that the deviation,
2p 2p d = r – i = 80.3° – 53.1° = 27.2°
= ´ (µ - 1) t = [1.5 –1]×2000×10–10 (c) The intensities of transmitted beams are 4I and I
l 6000 ´ 10-10 respectively.
p
=
radian
3
A r
The power received at F
i N
P = P1 + P2 + 2 P1 P2 cos D f i r

p M B
= 10–6 + 4 × 10–6 + 2 10-6 ´ 4 ´ 10 -6 cos
3
= 7 × 10–6 W.
Path diff = m( MB) - AN
EBD_7036
P-S- 252 Topic-wise Solved Papers - PHYSICS

sin r
= ( AB sin i ) - AB sin q
sin i u v
= 0 L1
S1
Since both the radiations are mutually coherent and A
while coming to focus these travel equal paths,
therefore, these two beams will arrive in phase at focus. O1 d
\ Resultant Intensity S O2 O
0.3m
( ) =( )
2 2
I max = I1 + I 2 4I + I
S2
2 L2
= (3 I ) = 9I. 0.15m
D
16. The light entering the rod does not emerge from the curved 1.3m
surface of the rod when the angle (90° – b) is greater than
the critical angle. For lens L1
The object is S
u = – 0.15 m, v = ?, f = + 0.1 m
b 90 – b 1 1 1 1 1 1 1 1
a - = Þ = + = +
v u f v f u 0.1 -0.15
Þ v = 0.3 m
1 D SO1O2 and D SS1S2 are similar. Also the placement of O1
i.e., µ < where C is the critical angle.
sin C and O2 are symmetrical to S
Here, C = 90 – b S1S2 u + v
1 \ =
1 O1O2 u
Þ µ < sin (90° - b) Þ µ <
cos b
(u + v) (O1O2 ) (0.15 + 0.3)
1 Þ S1S2 = = × 0.5 × 10–3
As a limiting case, µ = ... (i) u (0.15)
cosb Þ S1S2 = d = 1.5 × 10–3 m \ D = 1.3 – 0.3 = 1m
Applying Snell's law at A The fringe width
sin a sin a
µ= Þ sin b = ... (ii) lD 500 ´ 10-9 ´ 1 1
sin b µ b= = = ´ 10-3 m
d -3 3
NOTE : The smallest angle of incidence on the curved 1.5 ´ 10
p \ Therefore,
surface is when a = . This can be taken as a limiting case
2 1
OA = 3b = 3 × ´ 10-3 m = 10–3
for angle of incidence on plane surface. 3
From (ii) (ii) If the gap between L1 and L2 i.e., O1O2 is reduced.
sin p / 2 1 Then d will be reduced. Then the fringe width will
sin b = Þ µ = sin b ... (iii)
µ increase and hence OA will increase.
From (i) and (ii), sin b = cos b 18. (i) Since Y is below of optic axis, therefore the image is
Þ b = 45° real and inverted.
1 1 (i) STEPS OF CONSTRUCTION OF DIAGRAM.
Þ µ= = Þ µ= 2 For convex lens
cos 45° 1/ 2
(1) Join XY. This represents the ray originating from the
This is the least value of the refractive index of rod for light
entering the rod and not leaving it from the curved surface. source and meeting the image Y. Since the ray is
17. (i) In this case, the two identical halves of convex lens undeviated after passing through the lens, therefore O
will create two seperate images S1 and S2 of the source is the optical centre of the lens. Draw Y 1 OY 2
S. These Images (S1 and S2) will behave as two perpendicular to AB.
(2) Draw a ray from X, parallel to AB. It strikes Y1OY2 at M.
coherent sources and the further dealing will be in
Join MY. It cuts AB at F. This is the focus of the convex
accordance to Young's double slit experiment. lens.
Ray and Wave Optics P-S- 253

Y1 n sin i = constant
M Let us consider the rectangular state to be made up of
X parallel layers such that as we move in the + Y direction,
the refractive index increases as given by the
F relationship
A B n (y) = [ky3/2 + 1]1/2 ...(iv)
Applying Snell's law at O, we get 1 × sin 90° = constant
Y = 1.
Y2 Again applying Snell's law at B, we get
(ii) For concave mirror n sin i = const. = 1 (from above equation)
As the image is real and inverted, the concave mirror 2
has to be placed towards the left of X. To find the exact 1 2 æ dy ö
\ n = = cosec
= i 1 + cot= i 1+ ç ÷ ,
position of the concave mirror, we draw a line YY' sin i è dx ø
perpendicular to AB such that BY = BY'
2
æ dy ö
N Y' ky 3 / 2 +=
1 1+ ç ÷ from (iv)
è dx ø
M X dy
dy 1/ 2
F Þ = [ky 3 / 2 ]1/ 2 Þ 3 / 4 = k dx = dx (Q k = 1)
A O C B dx y
dy
Y
Þ ò y3 / 4 = ò dx
Join Y'X and extend the line to meet AB at O. If the concave Þ 4y1/4 = x + C where C is an integration constant.
mirror is placed at O then after reflection at O, this line will But at x = 0, y = 0
meet Y. 4
To find the radius of curvature of the mirror æ xö
\ C=0 \ 4y1/4 =
x Þ y= ç ÷
Join X and Y. Let it cut AB at C. This C should be the centre è 4ø
of curvature of the concave mirror. With OC as radius, draw (c) CO-ORDINATES (x1, y1) OF THE POINT P
a part of sphere. This is the concave mirror.
At P, y = 1m \ x = 4 y1/ 4 = 4
To find the focus of the concave mirror
Draw XM parallel to the principal axis. Join M to Y. Let it cut The coordinates of P are (4m, 1m)
AD at F. Therefore, F is the focus of concave mirror. (d) The refractive index at P
19. (a) SLOPE AT P np = [ky3/2 + 1]1/2 = [1 (1)3/2 + 1]1/2 = 2
To find the slope at B, we draw a tangent to the trajectory If ip is angle of incidence at P then according to Snell's
at B. The trajectory is such that as the ray passes law,
through the rectangular transparent medium, the ray
continuously deviates towards the normal. The tangent 1
np sin ip = 1 Þ sin ip =
at B makes an angle q with the x-axis. Therefore, the 2
Also by Snell's law, nair sin rp = np sin ip
dy
slope at point B is tan q = ... (i) 1 p
dx 1 sin rp = 2´ Þ sin rp = 1 Þ rp =
2 2
Y
Þ After emerging from the rectangular glass slab, the light
AIR
ray becomes parallel to slab length.
P 20. (i) The ray incident on AB at M makes an angle of
incidence i. It gets refracted at M. The angle of
B(x,y) refraction is r. Applying Snell's law at M
i TRANSPARENT
q MEDIUM sin i
O Q M AIR n= ...(i)
X sin r
(Origin)
i is the angle of incidence at B then according to DBQM A
p 45
o

i+q + = p ... (ii)


2
Substituting the value of q from (ii) in (i) P
90°–r
æ p ö dy dy n1
tan ç - i÷ = rC
è 2 ø dx Þ dx = cot i ... (iii)
i M N n
o
(b) EQUATION OFTRAJECTORY 45
According to Snell's law, when light propagates B C
through a series of parallel layers of different media, From fig
then
EBD_7036
P-S- 254 Topic-wise Solved Papers - PHYSICS
ÐAPM = 180° – (45° + 90° – r) = 45° + r The shift has to be such that the minima shifts to the axis.
and C = 90° – (45° + r) = 45° – r
The ray after refraction at M enter the prism and strikes its b
For this the shifting of the fringes should be where b is
diagonal face AC making an angle C with the normal at P. 2
Here C is the critical angle, therefore, the ray after refraction fringe width.
at P makes angle of refraction 90°
Applying Snell's law at P t
n sin 90° n1 m
= Þ sin C = ... (ii)
n1 sin C n d
From (i), sin i = n sin r = n sin (45° – C) b /2
= n [sin 45° cos C – cos 45° sin C] Minima

n D
= [ 1 - sin 2 C - sin C ]
2
Dt é g m -m m ù b
n é n2 n ù \ ê ú=
sin i = ê 1- 1 - 1 ú [From (ii)] d ë mm û 2
2ê n2 n2 ú
ë û
b dm µ 6.3 ´ 10 -4 ´ 10 -3 ´ 1.33
é 1 ù Þ t= =
Þ i = sin–1 ê { n 2 - n12 - n1}ú 2( g µ - m µ) ´ D 2 (1.53 - 1.33) ´ 1.33
ë 2 û
= 15.75 × 10–7 m = 1.575 × 10–6 m
(ii) Angle of incidence at AB for which the refracted ray 22.
passes through the diagonal face undeviated. For this to u 2– d v 2+ d
happen, the angle of incidence of ray MP on diagonal face 1
should be zero. It means that the ray should strike normal to
AC.
A O
45°
P'
d
45° n1 2 Image
M u2
r=45°
n v2
i'
B C
Given u2 + v2 = 1.8 m ... (i)
sin i '
Applying Snell's law at M, we get n = The magnification of lens (1) is 2
sin r
v2 + d
Since ÐAP'M = 90° Ð AMP = 45 Þ r = 45º \ 2= ... (ii)
u2 - d
1.352 From (i) and (ii)
\ sin i ' = n sin r = n sin 45° = = 0.956
2
u2 = 0.6 + d , v2 = 1.2 – d
Þ i ' = 72.94° Applying lens formula
l l 1 1 1
21. (i)
a
mm = a Þl = a + =
lm m a v2 + d u2 - d f .... (iii) for lens (1)

1 1 1
la D 6300 ´ 10-10 ´ 1.33 + = ... (iv) for lens (2)
\ Fringe width = = -3 v2 u2 f
a
m µd 1.33 ´ 10
From (iii) and (iv)
= 6.3 × 10–4m
1 1 1 1
(ii) KEY CONCEPT : The shift of fringes when one slit is + = +
covered with thin glass sheet is v2 + d u2 - d v2 u2

Dt é g m ù 1 1 1 1
Þ + = +
= d ê m - 1ú 1.2 - d + d 0.6 + d - d 1.2 - d 0.6 + d
ëm û
On solving, we get
where, t = thickness of glass sheet.
Ray and Wave Optics P-S- 255

Þ d = 0.6 m From (1) and (2),


Substituting this value in (iv)
æ 3ö
1 1 1 the angle of incidence is i = sin–1 çè ÷ø
+ = 4
1.2 - 0.6 0.6 + 0.6 f 25. (a) The path difference (Dx) from the ray starting from S1
\ f = 0.4 m and S2 and reaching a point P will be
2p 2p D x = d sin q
23. The phase difference f = Dx = (5l + D )
l l P

æ fö
We know that I (f) = Imax cos2 ç ÷ y
è 2ø S1
q q
3 f f p d Q
Þ I max = I max cos 2 Þ = 30° = M O
4 2 2 6 S2

2p 2p l
Þ = (5l + D ) Þ D x = = 0.3 t D
6 l 6 (=1m)
–6
Þ t = 9.3 × 10 m We know that the path difference for minimum intensity is
E l
(2m – 1) where m = 1, 2, 3...
2
D l
C 70° \ d sin q = (2m – 1)
2
n2
n1 (2 m - 1)l (2 m - 1)0.5 2 m - 1
Þ sin q = = =
60° 2d 2 ´ 1.0 4
i
24. r = 30° Also -1 £ sin q £ 1 . Therefore, possible values of m are ± 1,
20

± 2, 0
°

60° 60° From D POQ


A B
For Minimum Deviation D sin q
y = D tan q = ... (i)
(a) The rays of wavelength l0 incident at any angle on the 1 - sin 2 q
interface BC will pass through without bending, Positions of minima
provided the refractive indices n1 and n2 have the same
1
value for the wavelength l0. Equating the expressions For m = +1, sin q = and y = 0.26
of n1 and n2, we get 4
3
10.8 ´ 10-4 1.80 ´ 10-4 m = – 1, sin q = - and y = -1.13 m
1.20 + = 1.45 + 4
l 20 l 02 3
m = + 2, sin q = \ y = + 1.13 m
(where l0 is in nm) 4
1/ 2 1
æ 9.0 ´ 104 ö m = 0, sin q = - \ y = – 0.26 m
or l0 = ç ÷ = 600 nm 4
è 0.25 ø (b) WHEN THE INCIDENT BEAM MAKES AN ANGLE
OF 30° WITH X-AXIS
(b) For the wavelength 600 nm, the combination of prism Two cases arise as shown by the following two figures.
acts as a single prism shaped like an isosceles triangle
(ABE). At the minimum deviation, the ray inside the
prism will be parallel to the base. Hence, the angle of P
refraction on the face AC will be r = 30°. S1 y
n Na d q
q
Now sin i = n sin r = n sin 30° = ... (1)
2 aQ M
1
The value of n at 600 nm is S2
4
10.8 ´ 10 2
n = 1.20 + = 1.50 ... (2)
(600) 2
EBD_7036
P-S- 256 Topic-wise Solved Papers - PHYSICS
Path difference between ray 1 and 2 reaching P = S2M – NS1
2p æ m g ö 2 ´ 3.14 æ 1.5 ö
\ Dx1 = d sin q - d sin a (Case 1) f= ç - 1÷ t = çè - 1÷ (10.4 ´ 10 -6 )
l è mm ø 6 ´ 10 -7 4/3 ø
= 6.8 rad
f I
S1 We know that I = I0 cos2 \ 2
I 0 = cos (6.8) = 0.75
M
2
Na d (c) For maximum at O
q
Qq
a
1 æ mg ö
y Again path difference = ç - 1÷ t
S2
è mm ø
P
2 We know that for maxima, path difference = nl
æ mg ö
Path difference between ray 1 and 2 reaching P = NS1 + S1M \ nl = ç - 1÷ t
Dx2 = d sin a + d sin q (Case 2) è mm ø
Position of Central maxima : Path difference should be -6
æ mg ö t æ 1.5 ö 10.4 ´ 10
zero. Therefore Dx1 = 0 or Dx2 = 0 Þ l= ç - 1÷ = ç - 1÷
Þ d sin a = d sin q è mm ø n è 4 / 3 ø n

1 1.3 ´ 10-6 m
Þ sin q = [Q a = 30°] =
2 n
From equation (i), y = 0.58m Putting different values of n for find the wave length in the
range of 0.4 × 10–6 m to 0.7 × 10–6 m we get
l l = 0.65 × 10–6 m and 0.433 × 10–6 m
For first minima; d sin q + d sin a = 27.
2
Y Y
l Z
Þ d sin q = + d sin a 1
2 M 1

>
j M
> 8Ö3
+
l 0.5 1 1 3 3i
sin q = + sin a = + sin 30° = + = 6Ö

>
\

>
8Ö3 j 3j O X
2d 2 ´1 4 2 4 +8Ö

>
3i
O 6Ö3i
X 6Ö
>

>
From equation (i), y = 1.15m M -10K
For first minima on the other side A=6Ö
3i +8

>
M Ö3j -

>
l 10k

>
d sin a + d sin q = Þ sin q = -1
2 4 (Fig. 1) (Fig. 2)
\ From (i), y = -0.26 m
Figure 1 shows vector OM ' = 6 3i + 8 3 $j
26. (a) Let the central maxima is obtained at a distance x below ur
O. [This is because a glass sheet is present in front of Figure 2 shows vector A = 6 3$i + 8 3 $j - 10k$
S2 which increases its path length to the screen. The perpendicular to line MOM' is Z-axis which has a unit
Therefore the path length of ray from S1 to the screen
vector of k$ .
should also increase]. uuur uuur
Here, Angle between vector MP and OP can be found by dot
S 1
product.
d/2
O x uuur uuur
P MP . OP = (MP) (OP) cos i
xd æ m g ö d/2
Þ =ç
D è mm ø
- 1÷ t S2
t
(6 )
3 $i + 8 3 $j - 10k$ .( - k$ )
= cos i
2 2 2 2
(6 3) + (8 3) + ( -10) + ( -1)
æ mg ö D æ 1.5 ö (10.4 ´ 10-6 ) (1.5) Þ i = 60°
Þ x= ç - 1÷ t ´ = ç - 1÷ × Unit vector in the direction of MOM' from fig. (1) is
è mm ø d è 4/3 ø 0.45 ´ 10-3
= 4.33 × 10–3 m 6 3i$ + 8 3 $j
n$ = 3 4$
, n$ = i$ + j
2 2 1/ 2
æ mg ö [(6 3) + (8 3) ] 5 5
(b) For O, path difference = ç - 1÷ t
è mm ø To find the angle of refraction, we use Snell's law
\ Phase difference 3 sin i sin 60°
= = Þ r = 45°
2 sin r sin r
Ray and Wave Optics P-S- 257

Incident ray 1 1 1 1 1 1
- = Þ - =
A (sin r) n^ v u fl v - 20 15
i x-y plane
^
M –(cos r)k 1 1 1 1 v 60
O M' ^r Þ = - = Þ v = 60 cm, m = = = –3
r v 15 20 60 u -20
The image is formed to the left of the lens, real, inverted and
Refracted ray three times the actual size (3.6 cm in height below PQ).
For the mirror,
Now, r$ = (sin r ) n$ - (cos r ) k$ 1 1 1 2
1 1 1
+ = Þ = - =-
v ' u ' fm v ' -30 30 30
é3 4 ù
= ( sin 45°) ê $i + $j ú - (cos 45°) k$
ë5 5 û Þ v' = – 15 cm

1 v' -15 1
= [3i$ + 4 $j - 5k$ ] m= -
u'
=-
30
=
2
5 2
28. First of all, we consider the refraction at plane surface. Here 1
size of image = ´ 3.6 = 1.8 cm.
the image of P will form at I' after refraction from I surface. 2
This image will be inverted w.r.t. the original image and its
m1 position will be 0.3 cm above RS and 1.5 cm below RS. The
position of the image is 15 cm to the right of the mirror.
P (b) The path difference between the two rays reflected
from the upper surface AB (shown by ray 1, single arrow
m2 upwards) and lower surface CD (shown by ray 2 double
arrow pointing upwards) is
I' u = mR 1 2
A B
For plane surface : n=1.8
l t C D
Object distance u = – mR D x = mn × 2t +
2
Radius of curvature of the plane surface = ¥
The ray is coming from air and incident on the glas. n=1.5
Here µ1 = 1, µ2 = 1.5. l
Here is the path difference as the ray 1 suffer reflection
2
m 2 m1 m 2 - m1 m 2 m1
Apply - = ; = (as R = ¥) from a denser medium on surface AB
v u R v u We known that for constructive interference
m1 1.5 Path difference = ml where m is 1, 2,....
\ Image distance v = u= (– mR) = – 1.5 mR
m2 1.0 l æ 1ö
\ mn × 2t + = ml Þ 2 m nt = ç m - ÷ l
Now we consider refraction at the curved surface. 2 è 2ø
Object distance, u = – (1.5 mR + R)
Here, µ2 = 1, µ1 = 1.5, Image distance, v = ¥, l 648
when m = 1, t = = = 90 nm.
Radius of curvature = – R 4 mn 4 ´ 1.8

1 1.5 1 - 1.5 4 30. For no deviation condition FLINT(µ¢)


Here, + = \ m=
¥ (1.5m + 1) R -R 3 é µ -1ù B
A' = ê úA
29. (a) For the lens ë µ '- 1 û

A R R R
P t
igh
Q teL B
B" B 0.6m Wh
i
B
R B' 1.5 - 1
S Þ A' = ´ 6° = 4°
1.75 - 1 CROWN(µ)
A'
A"
Now, the angular dispersion produced by crown glass prism
30cm db – dr = A (µb – µr)
30cm 20cm
EBD_7036
P-S- 258 Topic-wise Solved Papers - PHYSICS
Also the angular dispersion produced by flint glass prism The liquid is poured into vessel upto OQ.
db' – dr' = A' (µb' – µr') The central bright fringe is formed at Q.
\ Net deviation in blue light For central bright fringe net path difference = 0.
(m – 1)t = DX1 or (m – 1)(100) = 0.16
db = (mb1 - 1) A1 – (mb2 - 1) A2 or m – 1 = 0.0016 or m = 1.0016
= (1.51 – 1) 6° – (1.77 – 1) 4° = – 0.02° 32. The lens maker formula is
Similarly Net deviation of red light 1 æ 1 1ö
= (m - 1) ç - ÷
dr = (m r1 - 1) A1 – (m r2 - 1) A2 f è R1 R2 ø
= (1.49 – 1) 6° – (1.73 – 1) 4° = 0.02 When the space between the lens and the mirror is filled
\ Net dispersion = db – dr = –0.04° with water, a system of two lenses is formed.
\ The magnitude of the net angular dispersion = 0.04 (i) a glass lens
31. (i) O is the middle point of two slits S1 and S2. (ii) a plano concave water lens
For glass lens Here R1 = + R and R2 = – R
S1S2 = d = 0.8 mm
A C 1 æ1 1 ö 1
= (1.5 - 1) ç - =
R fl è R - R ÷ø R
S1
y2 O
I
P O q
a Q
40 cm y1
S2 15 cm
S B D R1 = + R
2m 10 cm
D1 D2
R 2= –R
y 40 1
tan a = 1 = = For water lens
D1 200 5
1 æ 1 1 ö - 0.33
= (1.33 - 1) ç - =
\ sin a =
1
=
1
»
1
» tan a fw è - R -¥ ÷ø R
26 5.1 1.5 R1 = –R
Path difference DX1 = SS1 – SS2
æ 1ö
or DX1 = d sin a = ( 0.8 mm ) ç ÷ = 0.16 mm ... (i) R2 = ¥
è 5ø The focal length of the combination of two lenses will be
Let R denotes the position of central bright fringe. Net path 1 1 1 1 0.33 0.67
difference will be zero. = + = - = ... (i)
f f1 f 2 R R R
Now DX 2 = S2 R - S1R or DX 2 = d sin q ... (ii)
For central bright fringe A convex lens placed on a plane mirror behaves like a
concave mirror. The image is formed at the object itself if the
DX 2 - DX1 = 0 or d sin q - DX1 = 0
object is placed at centre of curvature of concave mirror.
or d sin q = DX 1 = 0.16 mm After refraction through lens, the rays fall on the plane mirror
0.16 1 normally and retrace their path to form image at the object
or (0.8)sin q = 0.16 or sin q = = itself.
0.8 5 \ Focal length of system (f) = 15 cm ... (ii)
1 1 1 From (i) and (ii)
\ tan q = = » = sin q \ tan q = y2
24 4.9 5 D2 1 0.67
= Þ R = 10.05 cm
1 y2 D 15 R
10
or 5 = D or y2 = 2 = = 2 cm The same situation is repeated with two differences
2 5 5 (a) The object and image distance are now 25 cm and
Hence position of central bright fringe is 2 cm above point
(b) In place of water there is a new liquid of refraction
Q on side CD.
(ii) m of liquid poured if central fringe is at Q: index µ
A C 1 1 1 -(m - 1)
Again = and = where f ' is the focal
S1 fl R f' R
length of new liquid lens.
P O \ New combined lens,
Q
y1 1 1 1 1 + (m - 1) 1 - m + 1 2 - m
S2 = + = - = =
S F fl f ' R R R R .... (i)
B D
t For new combined lens,
100 mm
Ray and Wave Optics P-S- 259

1 1 35. (a) Using Snell's law at surface AB


\ = ... (ii)
F 25 3
µair sin 60° = µp sin r Þ = 3 sin r Þ r = 30°
From (i) and (ii) 2
2 -µ 1 Now, NN' is the normal to surface AB.
= \ µ = 1.6 \ Ð AMN = 90°
10.02 25 But Ð QMN = 30° Þ Ð AMQ = 60°
33. (a) Because S is a point source, fringes will be circular.
A
2 2
I min æ I - 0.36 I ö æ 0.4 ö 1
(b) =ç ÷ = çè ÷ = 30o
I max è I + 0.36 I ø 1.6 ø 16
[Q If intensity of light falling on P directly from S is I, then N'
Q
o
the intensity of light falling at P after reflection from AB is 60
0.36 I ] 60o 30
o
M
(c) For maximum at P, path difference = nl N
If AB is shifted by a distance x, it will cause an additional
path difference of 2x.
P
Screen B C
In DAMQ
Ð AQM = 180° – (60° + 30°) = 90°
The refracted ray inside the prism hits the other face at 90°;
hence deviation produced by this face is zero and hence
D angle of emergence is zero.
(b) Multiple reflections occur in the film for minimum
thickness.
The intensity of emergent ray will be maximum if transmitted
waves undergo constructive interference.
h S \ For minimum thickness,
A B Dx = l
Þ D x = 2mt = l,
l l
2x = l (for minimum value of x) Þ x = = 300 nm where t = thickness Þ t = = 125 nm
2 2µ
34. For an object placed at infinity the image after first refraction 36. Use Snell's law
will be formed at a distance v1 n1 sin i = n2 sin r
m 2 m1 m 2 - m1 Here, n1 = 1, n2 = 2 , i = 45°, r = ?
- = ... (i)
v1 - ¥ +R
1 ´ sin 45° 1
Image afte second refraction will be formed at a distance v2 Þ sin r = =
Þ r = 30°
2 2
m3 m 2 m3 - m 2 The angle made by refracted ray at B with normal is 30°.
- = ... (ii)
v2 v1 +R \ Angle made by the first surface with refracted ray BC
is 60°.
Hence the refracted ray at B is parallel to horizontal
arrow.
I2 I1
m1 m2 m3 B
45° C
30°
v2 O
P
A n1 = 1 n2 = 2 n=1.514
v1
Adding (i) and (ii),
60°
µ3 µ3 - µ1 m3 R
- Þ v2 =
v2 R m 3 - m1 \ For refraction at spherical surface, u = ¥
Final image is formed at the focus when incident rays are n2 n1 n2 - n1
Now, - =
parallel. v u R
µ3 R 1.514 1.514 - 1.414
Therefore, focal length will be Þ = or v = 6.056 m
µ3 - µ1 v 0.4
\ OP = 6.056 m
EBD_7036
P-S- 260 Topic-wise Solved Papers - PHYSICS
37. At the place where maxima for both the wavelengths 40. (a) For minimum deviation of emergent ray from the first
coincide, y will be same for both the maxima, i.e., prism. MN is parallel to AC
n1l1D n2 l 2 D
\ Ð BMN = 60°
n1 l1 700 7
= Þ = = = Þ Ð r = 30°
d d n2 l 2 500 5 Applying Snell's law at M
Minimum integral value of n2 is 5. B
sin i
\ Minimum distance of maxima of the two wavelengths µ=
from central fringe sin r
sin i = µ sin r P o
60
n l D i N
= 2 2 = 5 × 700 × 10–9 × 103 = 3.5 mm. M
r
d 3 Q
sin i = 3 ´ sin 30° =
38. f = 0.3 m, u = – 0.4 m 2
Using lens formula 60o
Þ i = 60° A C
1 1 1 (b) When the prism DCE is rotated about C in
- = Þ v = 1.2 m
v - 0.4 0.3 anticlockwise direction by 60°, as shown in the figure,
then the final emergent ray SR becomes parallel to the
1 1 1 incident ray TM. Thus, the angle of deviation becomes
Now we have - = , differentiating w.r.t. t
v u f zero.

1 dv 1 du du B,D E
we have - 2 + 2 = 0 given = 0.01 m/s
v dt u dt dt R
M
T S
2
æ dv ö (1.20)
Þ çè ÷ø = ´ 0.01 = 0.09 m/s
dt (0.4) 2
So, rate of separation of the image (w.r.t. the lens) A C
= 0.09 m/s
F. Match the Following
udv vdu
v - 1. (A) ® (p).
dm
Now, m = Þ = dt 2 dt More the radius of aperture more is the amount of light
u dt u entering the telescope.
- (0.4) (0.09) - (1.2) (0.01) (B) ® (q).
= = -0.35s -1
(0.4) 2 f0
M=
Magnitude of rate of change of lateral magnification = 0.35 fe
s –1 . (C) ® (r).
39. For total internal reflection on interface AB L = f0 + fe
(D) ® (p), (q), (r).
m1 = 2 B Depends on dispersion of lens, spherical aberration and
A
radius of aperture.
i 2. A-p, q, r, s
m2 = 2 l When the object is at infinity, a real, inverted and
i
diminished image is formed at the focus of the concave
C D mirror.
m3 = 3
l As the object is brought closer to the mirror, the image
moves farther, remains real and inverted and increases
in size (but still it is diminished as compared to the object)
1 1µ 2 1
sin i = = = = Þ i = 45° l When the object is at C, a real, inverted and same size
2µ 2
1 2
µ image is formed at C.
2
l When the object is brought still closer, a real, inverted
For total internal reflection in interface CD.
and magnified image is formed beyond C.
1 m 3 l When the object is at focus (F), the image is highly
sin i = 3 = 3 = Þ i = 60° magnified, real and inverted and formed at infinity.
m 2m 2
2 l When the object is placed between pole and focus, a
Þ The minimum angle for total internal reflection for both virtual, erect and magnified image is formed behind the
the interface is 60°. mirror.
Ray and Wave Optics P-S- 261

3. A-p, s; B-q; C-t; D-r, s, t (C)

S2 P2
P2 P1
S2
P0
P1
P0 S1

S1
Here d ( P0 ) = -l / 2; d ( P1 ) = -l / 4, d ( P2 ) = -l / 6
I ( P0 ) = I1 + I 2 + 2 I1 I2 cos(– p)
For path difference l / 4 , phase difference is p / 2 .
= I1 + I 2 - 2 I1 I 2 = I 0 + I 0 - 2 I 0 = 0
For path difference l / 3 , phase difference is 2p / 3 .
I ( P1 ) = I1 + I 2 + 2 I1 I 2 cos(-p / 2)
Here, S1P0 - S2 P0 = 0
= I1 + I 2 = I 0 + I 0 = 2 I 0
\ d ( P0 ) = 0
Therefore, (p) matches with (A). æ pö
I ( P2 ) = I1 + I 2 + 2 I1 I 2 cos ç - ÷
The path difference for P1 and P2 will not be zero. The è 3ø
intensities at P0 is maximum.
= I1 + I 2 + I1 I 2 = I 0 + I 0 + I0 = 3I 0
I ( P0 ) = I1 + I 2 + 2 I1 I 2 cos 0°
\ I ( P2 ) > I ( P1 )
= ( I1 + I 2 ) 2 = ( I 0 + I 0 ) 2 = 4 I 0 (t) matches (C).
(D)
p
I ( P1 ) = I1 + I 2 + 2 I1 I 2 cos
2 S2 P2
= I1 + I 2 = I 0 + I 0 = 2 I 0 P1
P0
I ( P2 ) = I1 + I 2 + 2 I1 I 2 cos(2 p / 3)
S1
= I1 + I 2 - I1 I 2 = I0 + I 0 - I0 = I 0
\ I ( P0 ) > I ( P1 )
Therefore, (s) matches with (A). Here dP0 = 3l / 4; dP1 = -l / 2; dP2 = -5l /12
(B) æ -3p ö
I ( P0 ) = I1 + I 2 + 2 I1 I 2 cos ç
è 2 ÷ø
S2 P2
P1 = I1 + I 2 = I 0 + I 0 = 2 I 0
P0 I ( P1 ) = I1 + I 2 + 2 I1 I 2 cos(-p)

S1 = I1 + I 2 - 2 I1 I 2 = I0 + I 0 - 2 I 0 I 0 = 0

I ( P2 ) = I1 + I 2 + 2 I1 I 2 cos [ -5p / 6 ]

dP0 =
l
, dP1 = 0, dP2 =
l = I1 + I 2 - 3 I1 I 2 = 2 - 3 I 0( )
4 12 (r), (s), (t) matches (D).
4. A-p, r; B-q,s,t; C-p,r,t, D-q,s
I ( P0 ) = I1 + I 2 + 2 I1 I 2 cos p / 2 (a) When m1 < m2, the ray of light while entering the lens
will bend towards the normal. Therefore p, r are the
= I1 + I 2 = I 0 + I 0 = 2 I 0
correct options
I ( P1 ) = I1 + I 2 + 2 I1 I 2 = 4I 0 (B) When m1> m2, the ray of light while entering the lens
will bend away from the normal. Therefore q,s,t are the
I ( P2 ) = I1 + I 2 + 2 I1 I 2 cos p / 6 correct options.
(C) When m2 = m3, the ray of light while coming out from
= I1 + I 2 + 3 I1 I 2 the lens does not deviate from its path. Therefore p,r,t
are the correct option.
= I0 + I0 + 3 I 0 = (2 + 3)I 0 (D) m2> m3, the ray of light coming out of the lens deviates
Therefore, q match with (B) away from the normal. Therefore q,s are the correct
options.
EBD_7036
P-S- 262 Topic-wise Solved Papers - PHYSICS
5. (d) e ® f. For the ray to bend towards the normal at the 4. (c) As n is negative, therefore direction charges
prism surface m2 > m1. The ray then moves away n1 sin q1 = n2 sin q2
from the normal when it emerges out of the 5. (b) The physical characteristics remain unchanged.
rectangular block. Therefore m2 > m3. 6. (a, c)
e ® g. As there is no deviation of the ray as it emerges Applying Snell’s law at P; ns sin im = n1 sin (90° – C)
out of the prism, m2 = m1. ns = Refractive index of surrounding
Q n2
e ® h. As the ray emerges out of prism, it moves away
from the normal. Therefore m2 < m1 . As the ray ns n1
C
moves away from the normal as it emerges out P 90° – C
of the rectangular block, therefore m2 > m3.
im
e ® i. At the prism surface, total internal reflection
h2
has taken place. For this sin 45° >
h1
n2
\ m1 > Also sin C =
2 m2. (d) is the correct option. n1

1 é1 1 ù
6. (b) For (P) = ( m - 1) ê - ú n1
f ë R1 R2 û
n2
é2ù 1
= (1.5 - 1) ê ú = Þ f = r C
ër û r
Now
For the combination
n1 n1 n22
1 1
= +
1 1 1 2
= + = NA = sin im = n cos C = n 1 –
s s n12
F f1 f 2 r r r

r n12 – n22
\ F= \ NA =
2 ns
For S1 (in air) For S1 (in water)
1 é1 1 ù
For (Q) = ( m - 1) ê - ú 45 9 3 3 45 9 9
f ë R1 R2 û NA = – = NA = – =
16 4 4 4 16 4 16
é 1 1 ù 0.5 1 æ 6 ö
= (1.5 - 1) ê - ú = = For s1 ç in ns = ÷
ë ¥ -r û r 2r è 15 ø
\ f = 2r
15 45 9 3 15
For the combination NA = - =
6 16 4 24
1 1 1 1 1 2 1 For S2 (in water)
= + = + = =
F f1 f 2 2r 2r 2r r
3 64 49 3 15
NA = – =
\F=r 4 25 25 4 5
Similarly, we can either find or do not find the remaining For S2 (in air)
options (b) is the correct option.
64 49 15
NA = – =
G. Comprehension Based Questions 25 25 5
1. (a) For plane wave fronts the beam of light is parallel. 4
2. (c) Since points c and d are on the same wavefront, For S2 (in ns = )
15
therefore fd = fc
Similarly, fe = ff \ fd – ff = fc – ff 15 64 49 3
NA = – =
3. (b) The gap between consecutive wavefronts in medium 2 4 25 25 4
is less than that is medium 1. Therefore, wavelength of
æ 16 ö
light in medium 2 is less than that in medium 1. Therefore, For S2 ç in ns = ÷
speed of light is more in medium 1 and less in medium 2. è 3 15 ø
Ray and Wave Optics P-S- 263

3. 3 Using mirror formula for first position


3 15 64 49 9
NA = – =
16 25 25 16
(a), (c) are correct options
1
7. (d) NA = n12 – n22
ns object Image
Here
NA2 < NA1
\ the NA of combined structure is equal to the
smaller value of the two numerical apertures. 25
u1 = ?, v1 = cm, f = +10cm
(d) is the correct option. 3

H. Assertion & Reason Type Questions 1 1 1 3 1 1


+ = + =
1. (c) Statement 1 : v1 u1 f , 25 u1 10 \ u1 = – 50 m
Using mirror formula for the second position
1 1 1
NOTE : The mirror (spherical) formula + = is
u v f 1 1 1 7 1 1 1 1 7
+ = Þ + = Þ = -
valid only for mirrors of small apertures where the size v2 u2 f 50 u2 10 u2 10 50
of aperture is very small as compared to the radius of u2 = – 25m
curvature of the mirror. This statement is true. Change in position of object = 25 m
Statement 2 : 25 18
NOTE : Laws of mirror are valid for plane as well as Speed of object = ´ = 3 km h -1
30 5
large spherical surfaces.
4. 2 For the convex spherical refracting surface of oil we
Therefore, statement 2 is wrong.
apply
I. Integer Value Correct Type -m1 m 2 m 2 - m1
+ =
u v R
f
1. 6 Given f = + 20cm Also m = 7
f +u -1
-1 7/4 4 0
\ + =
( -24) v 6 24 cm
20
\ v = 21 cm
\
m25
= 20 - 25 = -30 = 6
For water-oil interface oil
m50 20 -5
20 - 50 -7 4

2. 6 In the figure, C represents the critical angle 4 + 3 =0
+21 V ¢ water
I
\ V¢ = 16 cm.
P R
This is the image distance from water-oil interface.
Therefore the distance of the image from the bottom of
C C the tank is 2 cm.
8 cm 5. 3 For maxima
C Path defference = ml
C \ S2A – S1A = ml
S1
R O
M
d x 2 + d2
1 3 3
\ sin C = = \ tan C =
m 5 4 x A

OM R
In D POM, tan C = =
PM 8

3 S2
\R = ´ 8 = 6 cm
4
EBD_7036
P-S- 264 Topic-wise Solved Papers - PHYSICS

é (n - 1) d 2 + x 2 + d 2 + x 2 ù – d 2 - x 2 1 1 1
\ = ml \ – = Þ v = 140 cm.
ëê ûú v –20 17.5

\ (n – 1) (d 2 + x 2 ) = ml v 140
M l¢ = Magnification by lens = = = –7
u –20
æ4 ö 2 2
\ ç 3 –1÷ d + x = ml M2 Mmirror ´ M l¢
è ø Now M = Mmirror ´ M = 7
1 l
\ d 2 + x 2 = 3ml 7. 2 Here ÐMPQ + ÐMQP = 60°. If ÐMPQ = r then ÐMQP
\ d2 + x2 = 9m2l2 = 60 – r
\ x2 = 9m2l2 – d2 Applying Snell’s law at P
\ p2 = 9 Þ p=3 sin60° = n sin r ...(i)
6. 7 Applying mirror formula Differentiating w.r.t ‘n’ we get
1 1 1 dr
+ =
v u f O = sin r + n cos r × ...(ii)
dn
1 1 1 1 1
= – = +
v f u –10 15

1 –15 + 10 –5 –1 60°
\ = = =
v 150 150 30
P Q
\ v = –30cm 60° r q
60°–rr
fm = –10cm fl = +10cm M

Applying Snell’s law at Q


sin q = n sin (60° – r) ...(iii)
15cm
Differentiating the above equation w.r.t ‘n’ we get
dq é dr ù
30cm cos q
dn
= sin (60° – r) + n cos (60° – r) êë – dn úû
50cm
dq é tan r ù
\cos q = sin (60° – r) – n cos (60° – r) ê –
For convex lens u = |2fl| dn ë n úû
Therefore image will have a magnification of 1.
[from (ii)]
When the set – up is kept in a medium
The focal length of the lens will change dq 1
\ = [sin (60° – r) + cos (60° – r) tan r] ...(iv)
dn cos q
1 æ nl ö é1.5 ù
ç – 1÷ From eq. (i), substituting n = 3 we get r = 30°
fl è ns ø f l¢ ê 1 – 1ûú
ë
= Þ =
1 æ nl ö 10 é 1.5 ù From eq (iii), substituting n = 3 , r = 30° we get q =
f l¢ ç
ç n¢
– 1÷
÷ êë 7 / 6 – 1úû 60°
è s ø
On substituting the values of r and q in eq (iv) we get
Þ fl¢ = 17.5 cm. dq 1
=
dn cos 60° [sin 30° + cos 30° tan 30°] = 2
1 1 1
Applying lens formula – =
v u f l¢
Ray and Wave Optics P-S- 265

Section-B JEE Main/ AIEEE


1. (b) KEY CONCEPT : The resolving power of a telescope 11. (c) KEY CONCEPT : The focal length(F) of the final mirror
D 1 2 1
R.P = is F = f + f
1.22 l where D = diameter of the objective lens l m

l = wavelength of light. 1 æ 1 1ö
Clearly, larger the aperture, larger is the value of D, Here = ( µ - 1) ç - ÷
fl è R1 R2 ø
more is the resolving power or resolution.
2. (a) KEY CONCEPT : When two plane mirrors are inclined
é1 1 ù 1
at each other at an angle q then the number of the = (1.5 - 1) ê - =
images of a point object placed between the plane ë a -30 úû 60

360° 360° 1 1 1 1
mirrors is - 1 , if is even \ = 2´ + = \ F = 10 cm
q q F 60 30 / 2 10
The combination acts as a converging mirror. For the
360°
\ Number of images formed = -1 = 5 object to be of the same size of mirror,
60º u = 2F = 20 cm
3. (a) The phenomenon of polarisation is shown only by 12. (d) The angle of incidence for total polarization is given
transverse waves.
by tan q = n Þ q = tan -1 n
( R.P )1 l 2 5 Where n is the refractive index of the glass.
4. (d) = =
( R.P )2 l1 4 13. (b) For constructive interference d sin q = nl
5. (a) In an optical fibre, light is sent through the fibre without n
any loss by the phenomenon of total internal reflection given d = 2 l Þ sin q =
2
as shown in the figure. n = 0, 1, – 1, 2, – 2 hence five maxima are possible
6. (b) Optical fibres form a dielectric wave guide and are free 14. (a) Frequency remains constant during refraction
from electromagnetic interference or radio frequency
interference. 1 c
vmed = =
7. (d) For the phenomenon of interference we require two µ0 Î0 ´4 2
sources of light of same frequency and having a definite
phase relationship (a phase relationship that does not l med vmed c / 2 1
= = =
change with time) l air vair c 2
8. (c) A real, inverted and enlarged image of the object is \ wavelength is halved and frequency remains
formed by th e objective lens of a compound unchanged
microscope.
1 3
360 360 15. (a) sin qc = =
9. (b) When q = 90° then = = 4 m 4
q 90
is an even number. The number of images formed is 3 3 R
or tan qc = = =
given by 16 - 9 7 12
360 360
n= -1 = -1 = 4 - 1 = 3
q 90 R
10. (b) The incident angle is 45° .
Incident angle > critical angle, i > ic
qc qc
12 cm
1
\ sin i > sin ic or sin 45 > sin ic sin ic =
n

1 1 1
\ sin 45° > or > Þn> 2
n 2 n
36
Þ R= cm
7
EBD_7036
P-S- 266 Topic-wise Solved Papers - PHYSICS
22. (a) The intensity of light at any point of the screen where
16. (b) y l
³ 1.22 the phase difference due to light coming from the two
D d
slits is f is given by
-3 -3
Þ D£ yd 10 ´ 3 ´ 10 = 30 » 5m
= æ fö
(1.22) l (1.22) ´ 5 ´ 10 - 7 6.1 I = Io cos2 ç ÷ where I is the maximum intensity..
è 2ø 0

\ Dmax = 5m NOTE : This formula is applicable when I1= I2. Here

1 æ 1.5 ö æ 1 1ö f = p3
17. (b) =ç - 1÷ ç - ÷ .... (i)
fa è 1 ø è R1 R2 ø
2
I p æ 3ö 3
\ = cos2 = ç ÷ =
1 æ mg öæ 1 1ö I0 6 è 2 ø 4
=ç - 1÷ ç - ÷
f m è m m ø è R1 R2 ø 23. (c) Power of combination is given by
P = P1 + P2 = (– 15 + 5) D = – 10 D.
1 æ 1.5 ö æ 1 1ö .... (ii)
=ç - 1÷ ç - ÷ 1
f m è 1.6 ø è R1 R2 ø 1 1
Now, P = Þ f = = metre
f P -10
æ ö æ1 ö
f m ç 1.5 - 1 ÷ \ f = - ç ´ 100÷ cm = -10 cm.
Dividing (i) by (ii), = =–8 è 10 ø
f a ç 1.5 ÷
ç - 1÷ 24. (d) The electron beam will be diffracted and the maxima is
è 1.6 ø
obtained at y = 0. Also the distance between the first
minima on both side will be greater than d.
1 1
Pa = - 5 = Þ fa = - 25. (c) This graph obeys the lens equation
fa 5
1 1 1
1 8 - =
Þ f m = -8 ´ f a = - 8 ´ - = v u f
5 5
where f is a positive constant for a given convex lens.
m 1.6 26. (a) To find the refractive index of glass using a travelling
Pm = = ´ 5 = 1D microscope, a vernier scale is provided on the
fm 8
microscope
18. (d) The shape of interference fringes formed on a screen 27. (b) Third bright fringe of known light coincides with the
in case of a monochromatic source is a straight line. 4th bright fringe of the unknown light.
Remember for double hole experiment a hyperbola is
3(590) D 4lD 3
generated. \ = Þ l = ´ 590 = 442.5 nm
d d 4
2
æ sin f ö p
19. (a) I = I 0 çç ÷÷ and f = ( b sin q) Q
è f ø l
n
C
When the slit width is doubled, the amplitude of the
90 – C
wave at the centre of the screen is doubled, so the 28. (c) q P
intensity at the centre is increased by a factor 4.

20. (b) I = I 0 cos 2 q


Applying Snell’s law at Q
I0
Intensity of polarized light = sin 90° 1
2 n= =
sin C sinC
I0 I 0
Þ Intensity of untransmitted light = I 0 - =
1 3
2 2
\ sin C = =
21. (a) For a thin prism, D = (m – 1) A n 2
Since lb < lr Þ mr < mb Þ D1 < D2 \ C = 60º
Applying Snell’s Law at P
Ray and Wave Optics P-S- 267

sin q
n= ; from (1)
sin(90 - C ) Þ sin q = n ´ sin(90 - C )
Ð i = 60 °
Þ sin q = n cos
é 2 ù
\ q = sin–1 ê ´ cos 600 ú
ë 3 û From Snell's law, 2 sin i = 3 sin r
Ð r = 45°
-1 æ 1 ö
or q = sin ç ÷ 34. (b) A phase change of p rad appears when the ray reflects
è 3ø at the glass-air interface. Also, the centre of the
interference pattern is dark.
Experimental
|v| curve 35. (a) From mirror formula

1 1 1 dv v 2 æ du ö
Straight + = so, =- 2ç ÷
line v u f dt u è dt ø

29. (d) (2f, 2f) 2


P dv æ f ö du dv 1
Þ = -ç Þ = m/s
45° dt è u - f ø÷ dt dt 15

|u| 36. (b) Q The E.M. wave are transverse in nature i.e.,
Here u = -2 f , v = 2 f r r
k ´E r
As |u| increases, v decreases for | u |> f . The graph = =H …(i)
mw
between |v| and |u| is shown in the figure. A straight line
r
passing through the origin and making an angle of r
45°with the x-axis meets the experimental curve at where H = B
m
P (2f, 2f ).
30. (b) In the medium, the refractive index will decrease from r r
k ´H r
the axis towards the periphery of the beam. and = -E … (ii)
Therefore, the beam will move as one move from the we
axis to the periphery and hence the beam will converge. r r r r
k is ^ H and k is also ^ to E
r r r r r
or In other words X || E and k || E ´ B
Decreasing m
37. (d) Let a1 = a, I1 = a12 = a2
Axis
a2 = 2a, I2 = a22 = 4a2. Therefore I2 = 4I1
Ir = I1 + I 2 + 2 I1 I2 cos f

31. (d) Initially the parallel beam is cylindrical . Therefore, the Ir = I1 + 4 I1 + 2 4 I12 cos f
wavefront will be planar. Þ Ir = 5I1 + 4 I1 cos f … (1)
32. (a) The speed of light (c) in a medium of refractive index
(m) is given by Now, Imax = ( a1 + a2 )2 = (a + 2 a)2 = 9a2
c0 Imax
m= , where c0 is the speed of light in vacuum Imax = 9I1 Þ I1 =
c 9
c0 c0 Substituting in equation (1)
\c= = m + m (I )
m 0 2 5 Imax 4 I max
Ir = + cos f
As I is decreasing with increasing radius, it is maximum 9 9
on the axis of the beam. Therefore, c is minimum on the Imax
axis of the beam. Ir = [ 5 + 4 cos f]
33. (a) Angle of incidence is given by 9

cos (p–i) =
(6 )
3iˆ + 8 3 ˆj - 10kˆ .kˆ Ir =
Imax
9
é 2f ù
êë 5 + 8 cos 2 - 4 úû
20

1 Imax é 2 fù
– cos i = – Ir = êë1 + 8 cos 2 úû
2 9
EBD_7036
P-S- 268 Topic-wise Solved Papers - PHYSICS
38. (d) The focal length of the lens 44. (b) For critical angle qc,
1 1 1
= - = 1 + 1 = 20 + 1 = 21
1
sin qc = air
f v u 12 240 240 240 m
For greater wavelength or q water
240
f = cm lesser frequency m is less.
21
So, critical angle would be more. So, they will not suffer
æ 1ö æ 1 ö 1 reflection and come out at angles less then 90°.
Shift = t ç 1 - ÷ Þ 1 ç 1 - ÷ = 1´
è mø è 3/2 ø 3 45. (d) According to malus law, intensity of emerging beam is
given by,
1 35 I = I0cos2q
Now v' = 12 - = cm
3 3 Now, IA' = I A cos230º
Now the object distance u. I B ' = I B cos 2 60º
1 3 21 1 é 3 21 ù
= - = - As I A ' = I B '
u 35 240 5 êë 7 48 úû
3 1 IA 1
1 1 é 48 - 49 ù Þ IA ´ = IB ´ \ =
= 4 4 IB 3
u 5 êë 7 ´ 16 úû
0.25 1
u = –7 ×16 × 5 = – 560 cm = – 5.6 m 46. (d) sin q = =
25 100
Velocity of light in vacuum
39. (c) \ n =
Velocity of light in medium
3 0.25cm q
\ n=
2 3cm
32 + (R – 3mm)2 = R2
R
Þ 32 + R2 – 2R(3mm) + (3mm)2 = R2 3mm
Þ R » 15 cm R– 3mm 25cm
1.22 l
1 æ 3 öæ 1 ö R Resolving power = 2 m sin q = 30 mm.
= ç –1 ÷ ç ÷ Þ f = 30 cm
f è 2 ø è 15 ø
40. (c) Relation between intensities Plane WF
47. (b) (Light bends µ increases
45° B Vel decreases
I0 (I0/2) upwards)
IR Refracted
(unpolarised) WF
A 48. (c) When r2 = C, ÐN2Rc = 90°
Where C = critical angle
æI ö I 1 I 1
I r = ç 0 ÷ cos 2 (45°) = 0 ´ = 0 As sin C = = sin r2
è 2ø 2 2 4 v
A
41. (d) It will be concentric circles.
42. (c) For the prism as the angle of incidence (i) increases, N1
the angle of deviation (d) first decreases goes to
q N2
minimum value and then increases.
43. (b) By Lens maker's formula for convex lens Q r1
r2 R
P
1 æ m öæ 2 ö
=ç - 1÷ ç ÷
f è mL øè R ø B C

4
for, m L1 = , f1 = 4 R Applying snell's law at ‘R’
3
µ sin r2 = 1 sin90° ...(i)
5 Applying snell's law at ‘Q’
for m L 2 = , f 2 = -5 R 1 × sin q = µ sin r1 ...(ii)
3
Þ f2 = (–) ve But r1 = A – r2
Ray and Wave Optics P-S- 269

So, sin q = µ sin (A – r 2)


db d æ lL ö
sin q = µ sin A cos r 2 – cos A ...(iii) [using (i)] For b to be minimum =0 ça + ÷=0
From (1) da da è a ø

2 1 a = lL
cos r2 = 1 – sin r2 = 1 – 2 ...(iv) b min = lL + lL = 2 lL = 4lL
µ
By eq. (iii) and (iv) 50. (b) A telescope magnifies by making the object appearing
1 closer.
sin q = µsin A 1 - - cos A 51. (c) We know that i + e - A = d
µ2
on further solving we can show for ray not to transmitted 35° + 79° – A = 40° \ A = 74°
through face AC
æ A + dm ö æ 74 + d m ö
é sin ç ÷ sin ç
–1 æ 1 ö ù 2 2 ÷
q = sin–1 ê u sin(A – sin çè µ ÷ø ú è ø= è ø
But m =
ë û sinA / 2 74
sin
So, for transmission through face AC 2
é –1 æ 1 ö ù
q > sin–1 ê u sin(A – sin çè µ ÷ø ú 5 æ d ö
ë û = sin ç 37° + m ÷
3 è 2 ø
49. (a) Given geometrical spread = a
l lL 5 5
Diffraction spread = ´L= m max can be . That is m max is less than = 1.67
a a 3 3

lL But dm will be less than 40° so


The sum b = a +
a 5 5
m< sin 57° < sin 60° Þ m = 1.45
3 3
EBD_7036
17
P-S- 270 Topic-wise Solved Papers - PHYSICS

Modern Physics

Section-A : JEE Advanced/ IIT-JEE

A 1. 30,000, 30,000 2. 500 disintegration/sec, 125 disintegration/sec


3. frequency 4. eight, six
5. intensity, decreases 6. lithium, 7 7. atomic number, mass number 8. 23.6 MeV
9. positive, p-part, n-part 10. reverse, negative terminal 11. 0.27Å 12. 3.81Å
13. B and D, A and C 14. 3 × 10 8 15. –1 16. neutrino 17. reverse
18. 0.62Å 19. 41 20. fusion, 24.03
B 1. F 2. F 3. T 4. F
C 1. (b) 2. (b) 3. (c) 4. (c) 5. (a) 6. (d) 7. (c) 8. (b)
9. (a) 10. (d) 11. (a) 12. (b) 13. (b) 14. (d) 15. (b) 16. (c)
17. (b) 18. (b) 19. (c) 20. (a) 21. (b) 22. (b) 23. (c) 24. (c)
25. (a) 26. (d) 27. (d) 28. (c) 29. (d) 30. (a) 31. (d) 32. (a)
33. (b) 34. (a) 35. (c) 36. (a) 37. (d) 38. (a) 39. (b) 40. (d)
41. (b) 42. (d) 43. (a) 44. (a) 45. (c) 46. (c) 47. (c) 48. (a)
49. (b) 50. (a) 51. (b) 52. (a) 53. (a) 54. (a) 55. (b) 56. (b)
57. (a) 58. (b) 59. (c) 60. (c)
D 1. (b, d) 2. (c, d) 3. (b, c) 4. (a, c, d) 5. (a) 6. (d) 7. (c) 8. (c, d)
9. (a) 10. (a, b, c) 11. (c) 12. (d) 13. (c, d) 14. (b) 15. (b, d) 16. (b)
17. (b) 18. (b, d) 19. (b, c) 20. (c) 21. (a, b, c) 22. (a, d) 23. (a, c) 24. (a, c)
25. (c, d) 26. (a, d) 27. (d) 28. (c) 29. (b) 30. (c) 31. (c) 32. (a, c)
33. (b, d) 34. (a, c) 35. (a, c) 36. (a) 37. (a,b,d) 38. (c)
E 1. (i) 5 (ii) 16.53 eV (iii) 3.65 × 10–9 m (iv) 340 eV, –680 eV, 1.05 × 10–34 J.s (v) 1.05 × 10–11m 2. 6, 18.835 × 10–7 m
3. 0, 11, 13 4. (i) 300Å (ii) 2.5 × 10–11 m 5. (i) 1.142 × 10–8 (ii) 3 6. 8 mW, 12.5 × 10–3 s,

n2 h 2 Î0
100 7. (i) r = (ii) 25 (iii) 5.478 × 10–11 m 8. (i) 2 (ii) 14.46 eV (iii) 13.5 eV, 0.7 eV
624pme 2
9. 2 eV, 0.754 V 10. 119.6 gm 12. (a) 232, 90 (b) 5.34 MeV, 1823 MeV

13. (a) 2.55 eV (b) 4 ® 2 (c) - h (d) 0.814 m/s 14. 5.95 l 15. 6, 3 16. (i) 3.4 eV (ii) 0.66 × 10–9 m
p
17. (i) 14.43 sec. (ii) 40 sec. 18. 151 eV, 0.5 Å 19. 3.32 × 10–5 W
1é 3
20. (a) a - (a - lN0 )e -lt ù (b) N0 , 2 N0 21. 3.4 eV, 3.84 eV and 2.64 eV 22. 2, 4, 10.5 eV
lë û 2
23. 6.25 × 1011, 0 eV, 5 eV 24. 38451 Kg 25. 10–12 J, 227.62 amu
20
10 20 æ 3 3 - 4ö
26. 15 loge 3, , 10 ç ÷ 27. (a) 3 (b) 4052 nm
3 3 è 3 3 ø

2
28. (a) 5 × 107 (b) 2000 N/C (c) 23 eV 29. 42 30. log (4 / 3) 32. 0.26
e

33. 0.55 eV 34. 2 35. (a) 56 (b) 1.546 × 1018 Hz. 36. 24
Modern Physics P-S- 271

F 1. A-p; B-t; C-u; D-r 2. A-p, q; B-p, r; C-p, s; D-p, q, r


3. A-p, r; B-q, s; C-p; D-q 4. A-p, q, t; B-q; C-s; D-s 5. (c)
6. A-r, t; B-p, s; C-p, q, r, t; D-p, q, r, t
G 1. (c) 2. (c) 3. (a) 4. (d) 5. (a) 6. (b)
7. (a) 8. (b) 9. (d) 10. (d) 11. (b) 12. (c)
13. (d) 14. (c) 15. (a) 16. (c)
H 1. (b)
I 1. (3) 2. (8) 3. (1) 4. (7) 5. (7) 6. (1) 7. (4)
8. (3) 9. (2) 10. (2) 11. (2) 12. (9) 13. (6)

Section-B : JEE Main/ AIEEE

1. (c) 2. (c) 3. (a) 4. (c) 5. (a) 6. (a) 7. (c) 8. (c) 9. (a) 10. (c) 11. (c) 12. (a)
13. (c) 14. (b) 15. (a) 16. (b) 17. (a) 18. (a) 19. (d) 20. (b) 21. (c) 22. (a) 23. (a) 24. (b)
25. (d) 26. (a) 27. (b) 28. (a) 29. (a) 30. (c) 31. (d) 32. (d) 33. (b) 34. (a) 35. (b) 36. (d)
37. (a) 38. (d) 39. (c) 40. (d) 41. (b) 42. (d) 43. (a) 44. (d) 45. (a) 46. (a) 47. (c) 48. (b)
49. (c) 50. (c) 51. (b) 52. (c) 53. (b) 54. (d) 55. (b) 56. (b) 57. (c) 58. (c) 59. (a) 60. (c)
61. (c) 62. (a) 63. (a) 64. (c) 65. (a) 66. (d) 67. (b) 68. (b) 69. (d) 70. (a) 71. (b) 72. (d)
73. (c) 74. (a) 75. (d) 76. (d) 77. (b) 78. (d) 79. (c) 80. (b) 81. (b) 82. (a) 83. (a) 84. (b)
85. (b) 86. (b) 87. (c) 88. (d) 89. (a) 90. (a) 91. (a) 92. (d) 93. (b) 94. (b) 95. (d) 96. (a)
97. (d) 98. (a) 99. (c) 100. (b) 101. (c) 102. (a) 103. (d) 104. (d) 105. (a) 106. (c) 107. (a) 108. (b, d)
109. (a) 110. (c) 111. (c) 112. (b) 113. (c)

Section-A JEE Advanced/ IIT-JEE


A. Fill in the Blanks \ Minimum accelerating voltage,
1. For minimum accelerating voltage, the electron should jump E
from n = 2 to n = 1 level. Vmin = 1 = 30, 000 V
e
For characteristic X-rays
n
é 1ù E æ 1ö
1 2. A = A0 ç ÷ where A0 = Initial activity = 1000 dps (given)
= Ra (Z – 1)2 ê1 - 2 ú = = è 2ø
l ë n û hc
A = Activity after n half lives
E1 2é 1ù
\ = Ra ( Z – 1) ê1 – ú ......(i) æ1ö
1
hc ë 22 û At t = 1, n = 1 \ A = 1000 ç ÷ = 500 dps
è2ø
The binding energy of innermost electron = 40 keV
3
\ Ionisation potential of tungsten = 40 kV = 40 × 103 V æ 1ö
At t = 3, n = 3 \ A = 1000 çè ÷ø = 125 dps
2
E2 é 1 ù
Þ = Ra ( Z - 1) 2 ê1 - ú .....(ii) 3. Note : According to law of photoelectric effect
hc ë ¥2 û (K.E.)max = hn – hn0
i.e., the maximum kinetic energy of electrons emitted in the
é 1ù photoelectric effect is linearly dependent on the frequency
ê1 – 2 ú of incident radiation.
E1 ë 2 û
\ = 238 206 4
E2 é 1 ù 4. 92 U ® 82 Pb + x 2 He + y –10 e
ê1 – 2 ú First we find the number of a- particles. The change in mass
ë ¥ û
number during the decay from uranium to lead = 238 – 206 =
3 3 32. Therefore, the number of a-particles (with mass no. 4)
Þ E1 = E2 = ´ 40, 000 eV = 30, 000 eV 32
4 4 = =8
4
EBD_7036
P-S- 272 Topic-wise Solved Papers - PHYSICS
The change in atomic number (i.e, number of protons) taking 11 +
16. 6 C ® 11 11 11 0
5 B + b + X Þ 6 C ® 5 B + +1e + n (neutrino)
place when 8 a-particles are emitted and lead is formed is
= 92 – (82 + 2 × 8) = 92 – (82 + 16) = 92 – 98 = – 6 The balancing of atomic number and mass number is correct.
This change will take place by emitting of six b-particles. Therefore, X stands for neutrino.
5. Note : More the number of electrons striking the anode, 17. Reverse
more is the intensity of X-rays. –34
´ 3 ´ 108
hc 6.63 ´ 10
When the speed of the striking electrons on anode is 18. lmin = = = 0.62Å
increased, the emitted X-rays have greater energy. We know eV 1.6 ´ 10 –19 ´ 20 ´ 103

hc 1 é1 1ù
that energy, E = . Therefore, when E increases then l 19. = R( Z – 1)2 ê 2 – 2 ú
l l
decreases. ëê n1 n2 úû
Since for Ka, n2 = 2 and n1 = 1
10 1 4
6. 5 B + 0n ¾¾ ® 2 He + 37 Li 1 é1 1 ù
The resulting nucleus is of element lithium and mass number \ –10 = 1.097 (Z – 1)2 ê 2 – 2 ú
0.76 ´ 10 ë1 2 û
is 7.
7. Atomic number, mass number Þ z – 1 = 40 Þ Z = 41
2 2 n=2
8. 1 H + 1H ¾¾ ® 42 He
X-ray
Binding energy of two deuterons
= 2 [1.1 × 2] = 4.4 MeV
Binding energy of helium nucleus = 4 × 7.0 = 28 MeV n=1
The energy released = 28 – 4.4 = 23.6 MeV 20. This is a nuclear fusion reaction
9. Positive, p-part, n-part Energy released = (Dm) [931.5 MeV/u]
10. Reverse, negative terminal. = [2 × 2.0141 – 4.0024 ] × 931.5 MeV
11. We know that = 24.03 MeV
B. True/ False
1 é1 1 ù
For Ka , = C ê – ú , where C is a constant 1. For photoelectric effect
l 2 2
êë n1 n2 úû hn – hn0 = (K.E.)max
where h = Planck’s constt.
1 é1 1 ù 3C n0 = Threshold frequency
Þ =Cê 2 – 2ú = ....(i)
0.32Å ë1 2 û 4 Þ (K.E.)max µ n
K.E. does not depend on the intensity of incident radiation.
1 é1 1 ù 8C 2. (K.E.)max = hn – hn0 Þ (K.E.)max µ n
For Kb, =Cê 2 – 2ú=
l ë1 3 û 9 Thus maximum kinetic energy is proportional to frequency
On dividing, we get l = 0.27 Å. and not intensity.
12. The fifth valence electron of phosphorous is in its third 3. Note : When the cathode temperature is higher, then more
shell, i.e., n = 3. For phosphorous, Z = 15. The Bohr’s radius number of electrons will be emitted which in turn will increase
for nth orbit the anode current.

æ n2 ö m A ´ 1.67 ´ 10 –27
32 4. Density = =
=ç e r ÷ r0 = ´ 12 ´ 0.529 Å = 3.81Å V 4 é 3
è Z ø 15 p R0 A1/ 3 ù
3 ë û
13. B and D is a.c. input and A and C is the d.c. output. 1.67 ´ 10 –27
Case (i) When B is –ve and D is +ve = = 3 × 1017 kg/m3
Current passes from D ® A ® C ® B 1.33 ´ 3.14 ´ (1.1 ´ 10 –15 )
Case (ii) When B is + ve and D is – ve where A = mass number.
Current passes from B ® A ® C ® B Note : The order of nuclear density is 1017 kg/m3.
Thus curve is always from A to C in output (a d.c. current) C. MCQs with ONE Correct Answer
14. The speed of X-rays is always 3 × 108 m/s in vacuum. It
does not depend on the potential differences through which 1. (b) KEY CONCEPT :
electrons are accelerated in an X-ray tube. We know that µ = gm × r0
Note : All electromagnetic waves propagate at 3 × 108 m/s in where µ = amplification factor,
vacuum. gm = mutual conductance
r0 = plate resistance
kZe2 \ µ = 3 × 103 × 1.5 × 10–3 = 4.5
15. K.E. = and
2r 2. (b) t1/2 = 3.8 day
0.693 0.693
– kZ e 2 K.E. \ l= = = 0.182
Total energy T.E. = \ = -1 t1/ 2 3.8
2r T.E.
Modern Physics P-S- 273

If the initial number of atom is a = A0 then after time t –dN


the number of atoms is a/20 = A. We have to find t. 11. (a) = l1 N + l 2 N
dt
2.303 A0 2.303 a
t = log = log = 16.46 days N
l A 0.182 a / 20 Þ log e = – ( l1 + l 2 ) t
No
3.
235
( )
(c) One point charge is 92 U uranium nucleus when No is initial number of atoms
\ q1 = 92e 0.693 0.693
Here l1 = and l 2 = ;
1620 810
+2e 92e N 1
= Þ loge = – æç
1 0.693 0.693ö
+ ÷t
d No 4 4 è 1620 810 ø
The other point charge is a particle \ q2= + 2e Þ t = 1 1080 years
Here the loss in K.E. = Gain in P.E. (till a-particle reaches 12. (b) KEY CONCEPT : For a semi conductor n = n0e–Eg/kT
the distance d) where n0 = no. of free electrons at absolute zero, n =
2q1q2 no. of free electrons at T kelvin, Eg = Energy gap, k =
1 2 qq
Þ mv = k 1 2 Þ r = k Boltzmann constant.
2 r 1
mv 2 As Eg increases, n decreases exponentially.
2 13. (b) As shown in the fig. (i) during one half cycle the polarity
of P and S are opposite such that diode (1) is reversed
9 ´ 109 ´ 2 ´ 1.6 ´ 10 –19 ´ 92 ´ 1.6 ´ 10 –19 biased and hence non conducting.
\ r=
5 ´ 1.6 ´ 10 –13 + I Forward biased Reversed biased

= 529.92 × 10–16 m p.n p.n
= 529.92 × 10–14 cm = 5.2992 × 10–12 cm –
S
+
S
4. (c) b-particles are charged particles emitted by the nucleus. P P
5. (a) KEY CONCEPT : The maximum number of electrons Output Output
+ –
in an orbit is 2n2. n > 4 is not allowed. p.n p.n
Therefore the number of maximum electron that can be – 1 + I 1
in first four orbits are Reverse bias Forward biased
2 (1)2 + 2 (2)2 + 2 (3)2 + 2 (4) 2 fig. (i) fig. (ii)
= 2 + 8 + 18 + 32 = 60 During the other half cycle, diode (1) gets forward
Therefore, possible element are 60. biased and is conducting. Thus diode (1) conducts in
6. (d) We know that one half cycle and does not conduct in the other so the
correct option is (b) (a and c.)
1 é1 1ù 1 2 14. (d) KEY CONCEPT :
= RZ 2 ê 2 – 2 ú Þ µ Z
l êë n2 n1 úû l
En = –13.6
( Z ) eV 2

1 (n2 )
l is shortest when is largest i.e., when Z has a higher
l Therefore, ground state energy of doubly ionized
value. Z is highest for lithium. lithium atom (Z = 3, n = 1) will be
7. (c) 411H + ® 42 He 2 + + 2e – + 26 MeV (3) 2
represent a fusion reaction. E1 = (–13.6) = –122.4 eV
(1) 2
8. (b) Fast neutrons can be easily slowed down by passing
them through water. \ Ionization energy of an electron in ground state of
9. (a) Note : The penetrating power is dependent on velocity. doubly ionized lithium atom will be 122.4eV.
For a given energy, the velocity of g radiation is highest 15. (b) In the circuit, diode D1 is forward biased, while D2 is
reverse biased. Therefore, current i (through D1 and
and a-particle is least.
100W resistance) will be
10. (d) When one e– is removed from neutral helium atom, it
becomes a one e– species. 6
i= = 0.02 A
For one e– species we know 50 + 100 + 150
–13.6Z 2 D1 150W
En = eV/atom
n2 D2 50W i
For helium ion, Z = 2 and for first orbit n = 1.
–13.6
\ E1 = ´ 22 = – 54.4 eV 100W
(1) 2
6V
\ Energy required to remove this e– = + 54.4 eV
Here, 50W is the resistance of D1 in forward biasing.
\ Total energy required = 54.4 + 24.6 = 79 eV
EBD_7036
P-S- 274 Topic-wise Solved Papers - PHYSICS
16. (c) In n-type semiconductors, electrons are the majority
charge carriers. é 1 1 kZe2 ù
The K.E. will increase êQ K.E. = | P.E. | = ú
17. (b) Note : ë 2 2 r úû
Stopping potential is the negative potential applied to
stop the electrons having maximum kinetic energy. é 1 kZe2 ù
Therefore, stopping potential will be 4 volt. The total energy decreases êT.E. = – ú
18. (b) KEY CONCEPT : ëê 2 r ûú
According to Doppler’s effect of light, the wavelength 26. (d) N1 = N0e–10lt and N2 = N0e–lt.
shift is given by
v N1 e –10 lt 1
Dl = ´ l \ = =
l
c N2 e – t
e lt
9

Dl ´ c (706 – 656)
Þ v= = ´ 3 ´ 108 » 2 × 107 m/s N1 1 1
= ; \ 9lt =
1
l 656 Given
N2 e e e
19. (c) Applying conservation of linear momentum,
Initial momentum = Final momentum æ 1ö
0 = m1v1 – m2v2 Þ m1v1 = m2v2 or, 9lt = 1 or t = ç ÷
è 9l ø
l1 h / m1v1 27. (d) KEY CONCEPT :
Now, = =1
l 2 h / m2 v2
hc
20. (a) Beta rays are same as cathode rays as both are stream lmin =
E
of electrons.
21. (b) Nuclear density of an atom of mass number A, 12400
–27 \ lmin = Å = 0.155Å
mass A(1.67 ´ 10 ) 80 ´ 103
d= =
volume 4 Energy of incident electrons is greater than the
p[1.25 ´ 10 –15 A1/ 3 ]3
3 ionization energy of electrons in K-shell, the K-shell
é 4 3 1/ 3 –15 ù electrons will be knocked off. Hence, characteristic
êQV = 3 pR , R = R0 A , R0 = 1.25 ´ 10 ú X-ray spectrum will be obtained.
ë û
\ d = 2 × 1017 kg/m3. 28. (c) Note : In a nucleus neutron converts into proton as
follows
22 4 4 14
22. (b) 10 Ne ® 2 He + 2 He + 6 X n ® p+ + e–1
The new element X has atomic number 6. Therefore, it Thus, decay of neutron is responsible for b-radiation
is carbon atom. origination
23. (c) KEY CONCEPT : Energy is released when stability 29. (d) For 2 to 1, 3 to 2 and 4 to 2 we get energy that n = 4 to
increases. This will happen when binding energy per n = 3,
nucleon increases. I.R. radiation has less energy than U.V. radiation.
Reactant Product 30. (a) KEY CONCEPT :
Reaction (a) 60 × 8.5MeV = 510MeV 2 × 30 × 5 = 300 MeV In case of Coolidge tube
Reaction (b) 120 × 7.5 = 900 MeV (90 × 8 + 30 × 5) = 870 MeV
hc
Reaction (c) 120 × 7.5 = 900 MeV 2 × 60 × 8.5 = 1020 MeV lmin = = l (as given here)
Reaction (d) 90 × 8 = 720 MeV (60 × 8.5 + 30 × 5) = 600 MeV eV
24. (c) KEY CONCEPT : Thus the cut off wavelength is inversely proportional
to accelerating voltage. As V increases, lc decreases.
1 lk is the wavelength of Kµ line which is a characteristic

m of an atom and does not depend on accelerating voltage
For ordinary hydrogen atom, longest wavelength of bombarding electron since lk always refers to a
photon wavelength of transition of e– from the target
1 é1 1 ù 5R 36 element from 2 ® 1.
= Rê – ú = or l =
l ë 2 2
3 2
û 36 5R The above two facts lead to the conclusion that
With hypothetical particle, required wavelength lk – lc increases as accelerating voltage is increased.
1 36 18 t

l' = ´ = 31. (d) N1 = N 0 e – l1t
= N 0e t ....(i)
2 5R 5R
25. (a) NOTE : 1
As the electron comes nearer to the nucleus the as t =
l1
potential energy decreases
t
æ – k .Ze 2 ö – 1
çQ = P.E. and r decreases÷ N2 = N0 e – l 2t = N0 e 5t ....(i) as 5t =
è r ø l2
Modern Physics P-S- 275

Adding (i) and (ii) we get 38. (a) KEY CONCEPT : We know that radius of the nucleus
R = R0A1/3, where A is the mass number.
N = N1 + N2 = N0 (e – t / t + e – t / 5t )
(a) is NOT the correct option as there is a time t for \ R 3 = R03 A
which N is constant which means for time t there is no
4 3 4 3
process of radioactivity which does not makes sense. Þ pR = pR0 A Þ Volume µ mass.
(b) and (c) shows intermediate increase in the number 3 3
of radioactive atom which is IMPOSSIBLE as N will 39. (b) By conservation of momentum, p1 = p2
only decrease exponentially. 2 K1m1 = 2 K 2 m2
32. (a) KEY CONCEPT :
q ne Þ 2 K1 (216) = 2 K 2 (4)
I= =
t t Þ K2 = 54K1 ...(i)
No. of electrons striking the target per second Also, K1 + K2 = 5.5 MeV ...(ii)
Solve equation (i) and (ii)
I 40. (d) From the graph it is clear that A and B have the same
= = 2 ´ 1016
e stopping potential and therefore, the same frequency.
nh Also, B and C have the same intensity.
33. (b) l= ,| E |µ Z 2 / n2 ; n = 3 1
2p 41. (b) In two half lives, the activity will remain of its initial
4
Þ lH = lLi and |EH| < |EL i|
activity.
34. (a) A = A0 (1/2)n; n = number of half lives. 42. (d) For photon,
n 4 n
A0 æ 1ö æ 1ö æ 1ö hc
= A0 ç ÷ \ ç ÷ =ç ÷ l2 = ...(i)
16 è 2ø è 2ø è 2ø E
Þ n=4
For proton, p = 2mE
\ t = (4 ´ 100)ms = 400 µs
35. (c) In g-decay, the atomic number and mass number do not h h
l1 = = ...(ii)
change. p 2mE
36. (a) Given potential energy between electron and proton
l2 hc
r = µ E -1/ 2
= eV0 log [Q | U |= eV ] l1 h
r0 E´
2mE
d é r ù eV0 1
\ |F| = êeV0 loge ú = ´ 1
dr ë r0 û r0 r 43. (a) For Ka , µ (Z – 1) 2
l
But this force acts as centripetal force
mv 2 eV0 eV l 2 ( Z1 –1)2 4l (11 –1)2
\ = Þ mv 2 = 0 ...(i) From (i), - Þ =
r rr0 r0 l1 ( Z 2 –1)2 l ( Z 2 –1)2
nh 10
By Bohr’s postulate, mvr = ....(ii)
2p Þ Z2 – 1 = Þ Z2 = 6
2
From (i) and (ii),
44. (a) Initially a photon of energy 10.2eV collides inelastically
m2 v 2 r 2 n2 h2 r0 with a hydrogen atom in ground state. For hydrogen
=
mv 2 4p 2 ´ V0 e atom,
13.6
2 n 2 h2 r0 E1 = – 13.6 eV; E2 = – eV = – 3.4eV
Þr = Þrµn 4
4pV0 me \ E2 – E1= 10.2 eV
37. (d) KEY CONCEPT : For an atom following Bohr’s The electron of hydrogen atom will jump to second
model, the radius is given by orbit after absorbing the photon of energy 10.2 eV. The
electron jumps back to its original state in less than
r0 m 2
rm = where r0 = Bohr’s radius and m = orbit microsecond and release a photon of energy 10.2 eV.
Z Another photon of energy 15 eV strikes the hydrogen
number. atom inelastically. This energy is sufficient to knock
For Fm, m = 5 (Fifth orbit in which the outermost electron out the electron from the atom as ionisation energy is
is present) 13.6 eV. The remaining energy of 1.4 eV is left with
electron as its kinetic energy.
r0 52 1
45. (c) Note : Since electron shows wave nature, it will show
\ rm = = nr0 (given) Þ n = 4
100 the phenomenon of interference.
EBD_7036
P-S- 276 Topic-wise Solved Papers - PHYSICS

h From (i) and (ii),


For electron, l =
mv hc ´ 2mel 2 2mcl 2
When speed of electron increases, l will decrease. The l0 = =
eh 2 h
distance between two consecutive fringes
51. (b) The continuous spectrum depends on the accelerating
lD voltage. It has a definite minimum wavelength.
b=
d Greater the accelerating voltage for electrons, higher
As l decreases, b also decrease. will be the kinetic energy it attains before striking the
target, higher will be the frequency of X - rays and
16
46. (c) 4 42 He ¾¾ ® 8O smaller will be the wavelength. The wavelength of
B.E. = Dm × 931.5 MeV continuous X - rays is independent of the atomic
= (4 × 4.0026 – 15.9994) × 931.5 = 10.24 MeV number of target material.
47. (c) For a nucleus to disintegrate in two half life, the 52. (a) Sample S –1 Sample S – 2
Activity 5 mCi 10 mCi
3 No. of nuclei N1= 2N N2 = N
probability is as 75% of the nuclei will disintegrate
4 æ dN ö æ dN ö
in this time. –ç = l1 N1 –ç = l 2 N2
è dt ÷ø 1 è dt ÷ø 2
48. (a) Iodine and Yttrium are medium sized nuclei and
therefore, have more binding energy per nucleon as Þ – 5 = l1× 2N ...(i) – 10 = l2 × N ...(ii)
compared to Uranium which has a big nuclei and less From (i) and (ii)
B.E./nucleon. 5 l1 ´ 2N l1 1
In other words, Iodine and Yttrium are more stable and = Þ =
therefore possess less energy and less rest mass. Also 10 l2 ´ N l2 4
when Uranium nuclei explodes, it will convert into I
( T1/ 2 )2 1 é 1 ù
and Y nuclei having kinetic energies. Þ = ê\ l µ ú
49. (b) The smallest frequency and largest wavelength in ( T1/ 2 )1 4 ë T1/2 û
ultraviolet region will be for transition of electron from 53. (a) The energy possessed by photons of wavelength
orbit 2 to orbit 1.
1240
æ 1 550 nm is = 2.25 eV
1 1ö 550
\ = Rç 2 – 2÷
l è n1 n2 ø The energy possessed by photons of wavelength
1240
1 é1 1ù é 1 ù 3R 450 nm is = 2.76 eV
Þ = R ê – ú = R ê1 – ú = 450
–9 2 2 ë 4û 4
122 ´ 10 m ë1 2 û The energy possessed by photons of wavelength
4 1240
Þ R= m –1 350 nm is = 3.54 eV
–9 350
3 ´ 122 ´ 10
The highest frequency and smallest wavelength for For metal plate p :
infrared region will be for transition of electron from ¥ fp = 2 eV.
to 3rd orbit. All the wavelengths are capable of ejecting electrons.
Therefore, the current is maximum. Also as the work
1 æ 1 1ö function is lowest in p, the kinetic energy of ejected
\ = Rç 2 – 2÷ electron will be highest and therefore, the stopping
l è n1 n2 ø potential is highest.
For metal plate q :
1 4 æ 1 1ö
Þ = – fq = 2.5 eV.
l 3 ´ 122 ´ 10 çè 32 ¥ ÷ø
–9
Photons of wavelength 550 nm will not be able to eject
electrons and therefore, the current is smaller than p.
3 ´ 122 ´ 9 ´ 10 –9 The work function is greater than q therefore the
\ l= = 823.5nm
4 stopping potential is lower in comparison to p.
50. (a) The cut off wavelength is given by For metal plate r :
hc fr = 3 eV
l0 = ...(i) Only wavelength of 350 nm will be able to eject electrons
eV and therefore, current is minimum. Also the stopping
According to de Broglie equation potential is least.
h h é1
l= = 1 1ù
p 2meV 54. (a) We know that = RZ 2 ê 2 - 2 ú
l ëê n1 n2 ûú
h2 h2 ..(ii) The wave length of first spectral line in the
Þ l2 = ÞV = Balmer series of hydrogen atom is 6561Å . Here n2 = 3
2meV 2mel 2
and n1 = 2
Modern Physics P-S- 277

1 2æ1 1ö 5R e(V01 - V02 )l1l 2


\ 6561 = R(1) ç 4 - 9 ÷ = 36 ...(i) \ h=
è ø (l 2 – l1 )c
For the second spectral line in the Balmer series of From the first two values given in data
singly ionised helium ion n2 = 4 and n1 = 2 ; Z = 2
1.6 ´ 10-19 [2 - 1] ´ 0.4 ´ 0.3 ´10 -6
1 2 é1 1 ù 3R h=
\ = R (2) ê - ú = ...(ii) 0.1 ´ 3 ´ 108
l ë 4 16 û 4
Dividing equation (i) and equation (ii) we get h = 0.64 ´ 10 -33 = 6.4 ´ 10-34 J-s
l 5R 4 5 Similarly if we calculate h for the last two values of
= ´ = data h = 6.4 × 10–34J-s
6561 36 3 R 27
59. (c) Binding energy of nitrogen atom
\ l = 1215 Å = [8 × 1.008665 + 7 × 1.007825 – 15.000109] × 931
Binding energy of oxygen atom
E P ´ t 30 ´ 10-3 ´ 100 ´ 10 -9
55. (b) p= = = 8
= 10 -17 kg ms -1 = [7 × 1.008665 + 8 × 1.007825 – 15.003065] × 931
c c 3 ´ 10 \ Difference = 0.0037960 × 931 MeV ...(I)
option (b) is correct
3 8´7 e2 3 56
Also E O = ´ ´ = ´ ´ 1.44MeV
lCu ( Z Mo - 1)
2 2 2
æ 42 - 1ö æ 41ö 5 R 4p Î0 5 R
56. (b) = =ç ÷ = çè ÷ø = 2.14
l Mo ( Z - 1)2 è 29 - 1ø 28
Cu 3 7´6 e2 3 42
EN = ´ ´ = ´ ´1.44MeV
5 R 4pÎ0 5 R
é ù
êQ n = ( Z - b ) here b = 1ú 3 14
ê ú \ E O - E N = ´ ´ 1.44MeV ...(II)
5 R
ê n = ( Z - 1)
2
ú
ê 1 ú From (i) & (ii)
µ ( Z - 1)
ê 2 ú
ë l û 3 14
´ ´ 1.44 = 0.0037960 ´ 931
5 R
hC 1 \ R = 3.42 fm
57. (a) - W = mu12
l1 2 A 1
60. (c) =
A 0 2n
hC 1
and - W = mu22 n A 0 64
l2 2 \2 = = = 26 Þ n = 6
A 1
Dividing the above two equations, we get \ time = 6 ´ t½ = 6 ´ 18 = 108
hC D. MCQs with ONE or MORE THAN ONE Correct
-W
l1 u2
= 1 1. (b,d) Note : Shortest wavelength means highest frequency.
hC 2
- W u2 This means highest energy.
l2 The energy of X-rays depends on the accelerating
voltage provided in the X-ray tube.
1240 Also, according to Moseley’s law n = a( Z – b) .
-W
248 4
\ = Thus the frequency also depends on the atomic number.
1240 1 2. (c,d) The threshold wavelength is 5200Å. For ejection of
-W
310 electrons, the wavelength of the light should be less
than 5200 Å, so that frequency increases and hence
1240 4 ´ 1240 the energy of incident photon increases. U.V light has
\ -W = - 4W
248 310 less wavelength than 5200 Å.
\ W = 3.7 eV 3. (b,c) Nuclear fusion occurs when two or more lighter nuclei
combine to form a heavier nucleus with release of a
hc f hc f huge amount of energy.
58. (b) - = V01 and - = V02
el1 e el 2 e 4. (a,c,d) We know, rn µ n 2

hc é 1 1 ù -13.6 Z 2
\ e ê l - l ú = V01 - V02 En = eV
ë 1 2û n2
nh
Angular momentum, Ln =
2p
EBD_7036
P-S- 278 Topic-wise Solved Papers - PHYSICS
| P.E. | = 2 × | K.E. | 10
5. (a) A diode can be used as a rectifier. Ib = 10% of Ic = ´ 10 = 1mA
6. (d) is the correct option. The electrons emitted by emitter 100
are collected to the maximum by the plate in this case. Now, Ie = Ib + Ic = 1 + 10 = 11mA
7. (c) is the correct option. 20. (c) 1H2 + 1H2 ® 1H3 + p
2 3 4
hc 1H + 1H ® 2He + n
l min = Net Reaction 31H ® 2He4 + p + n
2
eV Dm = 3 (2.014) – [4.001 + 1.007 + 1.008] = 0.026
8. (c,d) In the case of hydrogen, atomic number = mass number 3 deuterons release 3.87 × 10–12 J
In the other atoms, atomic number < mass number.
9. (a) 3.87 ´ 10 –12 ´ 1040
10. (a,b,c) are correct options. \ 1040 deuterons release =
3
11. (c) is the correct option.
12. (d) is the correct option. = 1.29 × 1028J
13. (c,d) are correct options.
E E 1.29 ´ 1028
14. (b) Note: The intensity of radiation emitted is proportional P= Þt = = 16
= 1.29 ´ 1012 sec
to the rate of decay which in turn is proportional to t P 10
number of atoms left (radioactive). 21. (a,b,c) For metal A
2.303 N0 4.25 = WA + TA ...(i)
t= log10
l N 2 2 2
h2
1 2 1 m vA pA
Also TA = mv A = = = ...(ii)
2.303 N0 2 2 m 2m 2ml 2A
Þt= log10 N / 64 Þ t = 12 hours.
0.693 / 2 0
15. (b, d) are correct options. é hù
Boron and Aluminium are trivalent impurities. êQ l = p ú
ë û
16. (b) Since the p-n junction arrangement are in series,
For metal B
therefore the potential drop across a p-n junction will
be proportional to their resistances. When the 4.7 = (TA – 1.5) + WB ...(iii)
resistances will be equal, the potential drops will be
h2
equal. In circuit I, the two p-n junctions are attached Also TB = ...(iv) [as eq. (ii)]
such that one is forward biased (low resistance) and 2ml 2B
other is reverse biased (high resistance). Whereas in Dividing equation (iv) by (ii),
the other two circuits both are either forward biased or
reversed biased. TB h2 2ml 2A l 2A
= ´ = 2
ln 2 1 TA 2ml 2B h2 lB
17. (b) T1/ 2 = and Mean life, t =
l l
18. (b, d) Since the stopping potential depends on the frequency TA –1.5 l 2A l2 1
and not on the intensity and the source is same, the Þ = = A =
stopping potential remains unaffected. The saturation
TA ( 2l A ) 4l A 4
2 2

current depends on the intensity of incident light on


the cathode of the photocell which in turn depends on [Q l B = 2l A given]
the distance of the source from cathode. The intensity Þ 4TA – 6 =TA Þ TA = 2 eV
(I) of light is inversely proportional to the square of the From (i), WA = 2.25 eV
distance between the light source and photocell. From (iii), WB = 4.2 eV
1 Also TB = TA – 1.5 Þ TB = 0.5eV
Iµ and saturation current µ I 22. (a,d) are correct options.
r2
23. (a,c) Holes are electron vacancies which participate in
1 electrical conductivity. These are produced in
Þ Saturation Current µ
r2 semiconductors.
24. (a,c) The circuit for a p-n-p transistor used in the common
(Saturation Current)final r2
Þ = Initial
2
emitter mode as an amplifier is shown in figure. The
(Saturation Current)initial rfinal base emitter junction is forward-biased and the input
0.2 ´ 0.2 signal is connected in series with the voltage applied
Þ (Saturation Current)final = ´ 18 = 2mA to bias the base emitter junction.
0.6 ´ 0.6
19. (b,c) Ic = 10 mA
90% of electrons emitted produce a collector current of
10 mA. The base current
Modern Physics P-S- 279

29. (b) The continuous X-ray spectrum is shown in figure.


C
RB RC Output E
B
Input N
E

l
25. (c, d) KEY CONCEPT : Due to mass defect (which is finally l min
responsible for the binding energy of the nucleus), All wavelengths > lmin are found.
mass of a nucleus is always less than the sum of masses
of its constituent particles. 12400
where lmin = V (in volts) Å
20
10 Ne is made up of 10 protons plus 10 neutrons.
Here V is the applied voltage.
20
Therefore, mass of 10 Ne nucleus –34
M1 < 10 (mp + mn) hc 6.63 ´ 10 ´ 3 ´ 108
30. (c) lmin = = = 310 × 10–9m.
Note : W 4(1.6 ´ 10 –19 )
Heavier the nucleus, more is the mass defect. = 310 nm
20 (mn + mp) – M2 > 10 (mp + mn) – M1
Thus, 10 (mn + mp) > M2 – M1 0.693 1
31. (c) (t1/2)x = (tmean)Y Þ =
or M2 < M1 + 10 (mp + mn) lx lY
Now, since M1 < 10 (mp + mn)
\ lx = 0.693 lY
\ M2 < 2M1
lx < lY. Now, rate of decay = lN
26. (a, d) The time period of the electron in a Bohr orbit is given
Initially, number of atoms (N) of both are equal but
2 pr since lY < lx, therefore Y will decay at a faster rate than
by T =
v x.
Since for the nth Bohr orbit, mvr = n (h/2p), the time 32. (a, c) f1 : f2 : f3 = eV01 : eV02 : eV03
period becomes
V01 : V02 : V03 = 0.001: 0.002 : 0.004 = 1: 2 : 4
2pr æ 4p 2 m ö 2
T= =ç ÷r Therefore option (a) is correct
nh /(2pmr ) è nh ø
V0
Since the radius of the orbit r depends on n, we replace
r. Bohr radius of a hydrogen atom is Metal 1 Metal 2 Metal 3
æ 2 ö
2 h e0
f1 f2 f3
r= n ç 2÷
è pme ø q q q –1
1/l (nm )
0.001 0.002 0.004
æ 4p 2 m ö æ n4 h 4 e 20 ö æ 3 2ö
3 4h e 0
Hence, T = ç ÷ç ÷ = n ç hc
4 ÷ By Einstein’s photoelectric equation, – f = eV
è nh ø è p 2 m2 e4 ø è me ø l

T1 æ n1 ö
3 hc f
ÞV= – ...(i)
For two orbits, = el e
T2 çè n2 ÷ø
Comparing equation (i) by y = mx + c, we get the slope
It is given that T1 / T2 = 8, hence, n1/n2 = 2.
27. (d) The result follows from the formula based on laws of hc
of the line m = = tan q
radioactive decay N = N0e–lt e
The nucleus start decaying after time t = 0 Þ Option (c) is correct.
28. (c) At junction a potential barrier/depletion layer is formed From the graph it is clear that,
as shown, with n-side at higher potential and p-side at
1
= 0.001( nm) Þ l = 1 = 1000mn
lower potential. Therefore, there is an electric field at –1
the junction directed from the n-side to p-side l 01 01
0.001
p E n 1
= 0.002 ( nm ) Þ l 02 = 500nm
–1
– + Also
l 02
– +
and l 03 = 250nm
– +
Note : Violet colour light will have wavelength less
– +
than 400 nm.
EBD_7036
P-S- 280 Topic-wise Solved Papers - PHYSICS
Therefore, this light will be unable to show photoelectric Total energy loss = (236 × 7.5) – [140 × 8.5 + 94 × 8.5]
effect on plate 3 Þ Option (d) is wrong. = 219 MeV
33. (b,d) Note : When binding energy per nucleon increases for The energies of kx and ky together is 4MeV
a nuclear process, energy is released. The energy remain is distributed by Sr and Xe which is
• When two nuclei of mass numbers between 51 to 100 equal to 219 – 4 = 215 MeV
fuse, the mass number of the resulting nuclei will come \ A is the correct option
out to be between 100 to 200. The graph shows that in Also momentum is conserved
this process the binding energy per nucleon increases 1
and therefore energy is released. \ K .E. µ . Therefore K.Esr > K.Exe
• When nucleus of mass number 200 to 260 breaks; it m
will produce nuclei of mass numbers lying between The energies of kx and ky together is 4MeV
100 to 200 if we assume that the two daughter nuclei The energy remain is distributed by Sr and Xe which is
are of nearly same mass. This in fact happens practically equal to 219 – 4 = 215 MeV
that when a heavy nucleus splits into two parts during \ A is the correct option
nuclear fission, two moderate size nuclei are formed in Also momentum is conserved
. Therefore K.Esr > K.Exe
general. The graph shows that in this process also the
37. (a, b, d)
binding energy per nucleon increases. Therefore energy
2
is released. n2 E = - 13.6 Z nh
We know that r = r0 , n , Ln =
nh 3h z n 2 2p
34. (a, c)Angular momentum = = . Therefore n = 3. Relative change in the radii of two consecutive orbitals
2p 2p
rn – rn -1 r (n - 1) 2
a n2 = 1 - n -1 = 1 - does not depend on Z
Also rn = o = 4.5a o rn rn n2
z
2n - 1 2
n2 9 = »(Q n > > 1)
\ = 4.5 Þ = 4.5 Þ z = 2 n 2 n
z z
Relative change in the energy of two consecutive orbitals
we know that
En – En -1 E n2 -2 n + 1 - 2
1 é1 1 ù 1 é1 1 ù = 1 - n -1 = 1 - = »
= R z 2 ê 2 - 2 ú = = 4R ê 2 - 2 ú En En (n - 1) 2
(n - 1) 2 n
l ëê n1 n 2 ûú l ëê n1 n 2 ûú
Ln - Ln -1 L (n - 1) 1
9 9 = 1 - n -1 = 1 - =
For n2 = 3, n1 = 1 we get l = = Ln Ln n n
8 ´ 4R 32R 38. (c) The wavelength of emitted photoelectron as per de
Broglie is
36 9
For n2 = 3, n1 = 2 we get l = = h h
5 ´ 4R 5R le = =
p 2m ( K.E )
4 1
For n2 = 2, n1 = 1 we get l = = When f increases, K.E. decreases and therefore le
3´ 4R 3R increases
(a), (c) are correct options When lph increases, N ph decreases , K.E decreases and
35. (a, c) therefore le increases.
hC hc W le is independent of the distance d.
We know that – W = eV0 Þ – = V0
l el e hc h2 é h ù
Also l + eV - f = l =
2 ê e ú
ph 2mle ë 2 mk. E û
1
For V0 versus we should get a straight line with negative
l hc f h2
\ +V- = ...(1)
slope and positive intercept. el ph e 2mele2
For V0 vesus l, we will get a hyperbola. As l decreases V0
increases. f
For V >> , f << eV
(a) and (c) are the correct options e
236
® 140 94 hc
36. (a) 92 U 54 Xe + 38 Sr + x + y Also << V . Then from eq (1).
The number of proton in reactants is equal to the el Ph
products (leaving x and y) and mass number of product 1
(leaving x and y) is two less than reactants le µ
V
\ x = p, y = e– is ruled out [B] is incorrect
and x = p, y = n is ruled out [C] is incorrect Therefore if V is made our times, le is approximately
half.
Modern Physics P-S- 281

E. Subjective Problems Note : For longest wavelength, the frequency should be


smallest.
13.6 2 13.6 2 This corresponds to the transition from n = 4 to n = 3, the
1. (i) E2 = – Z , E3 = – Z
4 9 13.6 13.6
energy will be E4 = – ; E3 = -
2
æ 1 1ö 13.6 ´ 5 2 4 32
E3 – E2 = –13.6 Z 2 ç – ÷ = + Z
è 9 4ø 36 –13.6 æ -13.6 ö é1 1 ù
But E3 – E2 = 47.2 eV (Given) \ E4 – E3 = –ç ÷ = 13.6 ê – ú
42 è 32 ø ë 9 16 û
13.6 ´ 5 2 47.2 ´ 36 = 0.66 eV = 0.66 × 1.6 × 10 J= 1.056 × 10–19J
–19
\ Z = 47.2 \ Z = =5
36 13.6 ´5 12400
Now, E = eV \ l = 18787 Å
–13.6 2 l (inÅ)
(ii) E4 = Z
16 3. (i) In a nucleus, number of electrons = 0 ( Q electrons don't
reside in the nucleus of atom).
2é1 1ù 2 é 9 –16 ù
\ E4 – E3 = –13.6 Z ê – ú = –13.6Z ê (ii) number of protons = 11
ë16 9 û ë 9 ´ 16 úû (iii) number of neutrons = 24 – 11 = 13
+13.6 ´ 25 ´ 7 I.E.
= = 16.53eV 4. (i) En = – for Bohr’s hydrogen atom.
9 ´ 16 n2
13.6 – 4R
(iii) E1 = – ´ 25 = –340 eV Here, I.E. = 4R \ En =
1 n2
\ E = E ¥ – E1 = 340 eV = 340 × 1.6 × 10–19 J [E ¥ = 0 eV]
–4 R æ 4R ö
hc \ E2 – E1 = –ç– = 3R ..(i)
But E = 2 2 è 12 ÷ø
l
hc
hc 6.6 ´ 10 –34 ´ 3 ´ 108 E2 – E1 = hn = ...(ii)
\ l= = = 3.65 ´ 10 -19 m l
E 340 ´ 10 –19 ´ 1.6 From (i) and (ii)
(iv) Total Energy of 1st orbit = – 340 eV hc
We know that – (T.E). = K.E. [in case of electron revolving = 3R
around nucleus] l
and 2T.E. = P.E. hc 6.6 ´ 10 –34 ´ 3 ´ 108
\ K.E. = 340 eV ; P.E. = – 680 eV \ l= = = 300Å
KEY CONCEPT : 3R 2.2 ´ 10 –18 ´ 3
Angular momentum in 1st orbit : (ii) The radius of the first orbit
According to Bohr’s postulate, Bohr’s radius of hydrogen atom = 5 × 10–11 m (given)
| En| = + 0.22 × 10–17Z2 = 4R = 4 × 2.2 × 10–18
nh \Z =2
mvr =
2p
For n = 1, r0 5 ´ 10–11 5 ´ 10 –11
\ rn = = = = 2.5 × 10–11m
h 6.6 ´ 10 –34 Z Z 2
mvr = = = 1.05 × 10–34J-s. 13.6
2p 2p 5. (i) En = – Z 2 eV/atom
(v) Radius of first Bohr orbit n2
5.3 ´ 10 -11 5.3 ´ 10 -11 –13.6 ´ 9
r1 = = For Li2+, Z = 3 \ En = eV/atom
Z 5 n2
= 1.06 × 10–11 m
12400 12400 13.6 ´ 9 13.6 ´ 9
2. E= eV = = 12.75 eV ...(i) \ E1 = – and E3 = – = –13.6
l (inÅ) 975 1 9
Also DE = E3 – E1 = – 13.6 – (–13.6 × 9)
= 13.6 × 8 = 108.8 eV/atom
é1 1ù é ù 12400 12400
13.6 ê 2 – 2 ú = 12.75 Þ ê1 – 1 ú = 12.75 Þ n = 4 l= Å = = 114Å
2 108.8
ëê n1 n2 ûú 2
êë1 n2 úû 13.6 E (in eV)
For every possible transition one downward arrow is shown (ii) The spectral line observed will be three namely 3 ® 1,
therefore the possibilities are 6. 3 ® 2, 2 ® 1.
6. I = 0.125 V – 7.5
n=4
n=3 dV 1
Increasing

Þ dI = 0.125 dV or = =8
n=2 dI 0.125
Energy

dV
n=1 We know that plate resistance, rp = = 8mW
dI
EBD_7036
P-S- 282 Topic-wise Solved Papers - PHYSICS

é dI ù 1 é1 1ù
The transconductance, gm = ê ú (iii) = 208R ´ Z 2 ê 2 – 2 ú
ëê dVg ûúV = constt l ëê n1 n2 úû
At Vg = – 1 volt, V = 300 volt, the plate current 1 é1 1ù
I = [0.125 × 300 – 7.5] mA = 30 mA = 208 ´ 1.097 ´ 10 7 ´ 32 ê – ú
Þ
l ë12 32 û
Also it is given that Vg = – 3V, V = 300 V and I = 5mA
Þ l = 5.478 × 10–11m
é 30 – 5 ù 25 –3 –3 8. (i) The transition state of six different photon energies are
\ gm = ê
–1 – (–3) ú = 2 ´ 10 = 12.5 × 10 s shown.
ë û
The characteristics are given in the form of parallel lines.
n = 4 [– 0.85 eV]

Increasing Energy
Amplification factor [0.65 eV]
= rp gm = 8 × 103 × 12.5 × 10–3 = 100 n = 3 [– 1.5 eV]
7. (i) Let m be the mass of electron. Then the mass of mu- [+ 1.9 eV]
meson is 208 m. According to Bohr’s postulate, the n = 2 [– 3.4 eV]
angular momentum of mu-meson should be an integral [10.2 eV]
multiple of h/2p.
v n = 1 [– 13.6 eV]
e
Since after absorbing monochromatic light, some of the
r emitted photons have energy more and some have less than
2.7 eV, this indicates that the excited level B is n = 2. (This is
+3e because if n = 3 is the excited level then energy less than 2.7
eV is not possible).
(ii) For hydrogen like atoms we have
–13.6
nh En = Z 2 eV/atom
\ (208m) vr = n2
2p
nh nh –13.6 2 æ –13.6 ö 2
E4 – E2 = Z –ç Z = 2.7
\ v= =
2p ´ 208mr 416pmr
...(i) 16 è 4 ÷ø
Note: Since mu-meson is moving in a circular path, é1 1 ù
therefore, it needs centripetal force which is provided Þ Z 2 ´ 13.6 ê – ú = 2.7
by the electrostatic force between the nucleus and mu- ë 4 16 û
meson.
2.7 4 ´ 16 æ1 1 ö
2 Þ Z2 = ´ Þ I.E. = 13.6 Z 2 ç 2 – 2 ÷
(208m)v 1 3e ´ e 13.6 12 è1 ¥ ø
\ = ´
r 4pe 0 r2
2.7 4 ´ 16
2
= 13.6 ´ ´ = 14.46 eV
3e 13.6 12
\ r= (iii) Max. Energy
4pe 0 ´ 208mv 2
æ 1 1ö
Substituting the value of v from (1), we get E4 – E3 = –13.6 Z 2 ç 2 – 2 ÷
è4 1 ø
3e 2 ´ 416pmr ´ 416pmr 2.7 4 ´ 16 15
r= = 13.6 ´ ´ ´ = 13.5 eV
4pe 0 ´ 208mn 2 h2 13.6 12 16
Min. Energy
n2 h2 e 0 æ 1 1ö
Þ r= ....(i) E4 – E3 = –13.6 Z 2 ç 2 – 2 ÷
624pme2 è4 3 ø
(ii) The radius of the first orbit of the hydrogen atom 2.7 4 ´ 16 7
= 13.6 ´ ´ ´ = 0.7eV
e 0 h2 13.6 12 9 ´ 16
= ...(ii) 9. For hydrogen like atom energy of the nth orbit is
pme 2
13.6 2
To find the value of n for which the radius of the orbit En = – Z eV/atom
is approximately the same as that of the first Bohr orbit n2
for hydrogen atom, we equate eq. (i) and (ii) For transition from n = 5 to n = 4,
n2 h2 e 0 e 0 h2 é 1 1 ù 13.6 ´ 9 ´ 9
= Þ n = 624 » 25 hn = 13.6 ´ 9 ê – ú = = 2.754 eV
624pme 2 pme 2 ë16 25 û 16 ´ 25
Modern Physics P-S- 283

For transition from n = 4 to n = 3, Area of aperture is,


é 1 1 ù 13.6 ´ 9 ´ 7 p 2 p
hn' = 13.6 ´ 9 ê – ú = = 5.95eV S1 = d = (0.1) 2 = 7.85 ´ 10 -3 m 2
ë 9 16 û 9 ´ 16 4 4
For transition n = 4 to n = 3, the frequency is high and hence \ Total number of photons incident per unit time on the
aperture,
wavelength is short.
For photoelectric effect, hn' – W = eV0, where W = work n3 = n2 s1 = (1.34 ´ 1018 ) (7.85 ´ 10 -3 ) / s
function = 1.052 × 1016 / s
5.95 × 1.6 × 10–19 – W = 1.6 × 10–19 × 3.95 The aperture will become new source of light.
Þ W = 2 × 1.6 × 10–19 = 2 eV Now these photons are further distributed in all directions.
Again applying hn – W = eV'0 Hence, at the location of detector, photons incident per unit
We get, 2.754 × 1.6 × 10–19 – 2 × 1.6 × 10–19 = 1.6 × 10–19 V'0 area per unit time :
Þ V'0 = 0.754 V
10. Energy required per day n3 1.052 ´ 1016
n4 = =
E = P × t = 200 × 106 × 24 × 60 × 60 4 p(6 - 0.6) 2 4p(5.4) 2
= 1.728 × 1013J = 2.87 × 1013 s–1 m–2
Energy released per fusion reaction This is the photon flux at the centre of the screen. Area of
= [2 (2.0141) – 4.0026] × 931.5 MeV detector is 0.5 cm2 or 0.5 × 10–4 m2. Therefore, total number
= 23.85 MeV = 23.85 × 106 × 1.6 × 10–19 of photons incident on the detector per unit time :
= 38.15 × 10–13J
\ No. of fusion reactions required n5 = (0.5 ´ 10-4 ) (2.87 ´ 1013 d ) = 1.435 ´ 109 s -1
The efficiency of photoelectron generation is 0.9. Hence,
1.728 ´ 1013 total photoelectrons generated per unit time :
= = 0.045 × 1026
38.15 ´ 10–13 n6 = 0.9n5 = 1.2915 ´ 109 s -1
\ No. of deuterium atoms required or, photocurrent in the detector :
= 2 × 0.045 × 1026 = 0.09 × 1026
Number of moles of deuterium atoms i = (e)n6 = (1.6 ´ 10 -19 ) (1.2915 ´ 109 ) = 2.07 ´ 10 -10 A
(b) Using the lens formula :
0.09 ´ 1026
= = 14.95 1 1 1
6.02 ´ 1023 - = or v = – 0.3 m
v -0.6 -0.6
\ Mass in gram of deuterium atoms
i.e., image of source (say S', is formed at 0.3 m from the lens.)
= 14.95 × 2 = 29.9 g
But the efficiency is 25%.
Therefore, the actual mass required = 119.6 g

hc (6.6 ´ 10-34 ) (3.0 ´ 108 )


11. Energy of one photon, E = = S S'
l 6000 ´ 10 -10 L D
= 3.3 × 10–19 J
0.3 m

5.7 m

Total number of photons incident per unit time on the lens


0.6 m 5.4 m are still n3 or 1.052 × 1016/s. 80% of it transmits to second
S A D medium. Therefore, at a distance of 5.7 m from S' number of
photons incident per unit are per unit time will be :

(80 /100) (1.05 ´ 1016 )


n7 = = 2.06 ´ 1013 s -1m -2
2
(4 p) (5.7)
Power of the source is 2 W or 2 J/s. Therefore, number of
photons emitting per second, This is the photon flux at the detector.
New value of photocurrent is :
2
n1 = -19
= 6.06 ´ 1018 / s i = (2.06 ´ 1013 ) (0.5 ´ 10 -4 ) (0.9) (1.6 ´ 10 -19 )
3.3 ´ 10
At distance 0.6 m, number of photons incident per unit area = 1.483 × 10–10 A
per unit time : (c) For stopping potential
hc
n1 = ( EK )max + W = eV0 + W
n2 = = 1.34 ´ 1018 / m 2 / s l
4 p (0.6) 2
EBD_7036
P-S- 284 Topic-wise Solved Papers - PHYSICS
By hit and trial we get n2 = 4 and n1 = 2
hc 3.315 ´ 10 –19
\ eV0 = –W = –1 = 1.07eV nh
l 1.6 ´ 10 –19 [angular momentum mvr = ]
\ V0 = 1.07 Volt 2p
Note : The value of stopping potential is not affected by the (c) Change in angular momentum
presence of concave lens as it changes the intensity and n1h n2 h h h h
not the frequency of photons. The stopping potential = – = (2 – 4) = ´ (-2) = -
depends on the frequency of photons. 2p 2p 2 p 2p p
A
(d) The momentum of emitted photon can be found by de
12. (a) 92X ® 228 4
XY + 2He Broglie relationship
A = 228 + 4 = 232; 92 = Z + 2 Þ Z = 90
(b) Let v be the velocity with which a - particle is emitted. h h hn E 2.55 ´ 1.6 ´ 10 –19
l= Þ p= = = \ p=
Then p l c c 3 ´ 108
mv 2 qrB 2 ´ 1.6 ´ 10 –19 ´ 0.11 ´ 3 Note : The atom was initially at rest the recoil momentum of
= qvB Þ v = = the atom will be same as emitted photon (according to the
r m 4.003 ´ 10–27
7 –1 conservation of angular momentum).
Þ v = 1.59 × 10 ms .
Applying law of conservation of linear momentum during Let m be the mass and v be the recoil velocity of hydrogen
a-decay we get atom then
mY vY = mava ...(1)
2.55 ´ 1.6 ´ 10 –19
The total kinetic energy of a-particle and Y is m´v =
3 ´ 108
1 1
E = K.E.a + K.E.Y = ma va 2 + mY vY 2
2 2 2.55 ´ 1.6 ´ 10 –19
Þv = = 0.814 m/s
1 1 ém v ù 1
2
m2 v2 3 ´ 108 ´ 1.67 ´ 10 –27
= ma va2 + mY ê a a ú = ma va 2 + ma va2 + a a 14. t1/2 = 15 hours
2 2 ë mY û 2 2mY
Activity initially A0 = 10–6 Curie (in small quantity of solution
1 é ma ù of 24Na) = 3.7 × 104dps
= ma va2 ê1 + ú Observation of blood of volume 1 cm3
2 ëê mY ûú After 5 hours, A = 296 dpm
The initial activity can be found by the formula
1 é 4.003 ù
= ´ 4.033 ´ 1.6 ´ 10 –27 ´ (1.59 ´ 107 )2 ê1 + J
2 ë 228.03 úû 2.303 A0
Þ5=
2.303 A
´ log10 0
t= log10
= 8.55 × 10–13 J l A 0.693 /15 296
= 5.34 MeV
\ Mass equivalent of this energy A0 5 ´ 0.693 0.3010
Þ log10 = = = 0.10033
296 2.303 ´ 15 3
5.34
= = 0.0051 a.m.u. A0 373
931.5 Þ = 1.26 Þ A0 = 373 dpm = dps
Also, mx = mY + ma + mass equivalent of energy (E) 296 60
= 228.03 + 4.003 + 0.0057 = 232.0387 u. This is the activity level in 1 cm3. Comparing it with the
The number of nucleus = 92 protons + 140 neutron. initial activity level of 3.7 × 104 dps we find the volume of
\ Binding energy of nucleus X blood.
= [92 × 1.008 + 140 × 1.009] – 232.0387 = 1.9571 u
= 1.9571 × 931.5 = 1823 MeV. 3.7 ´ 104
V= = 5951.7 cm3 = 5.95l litre
13. (a) The energy of photon causing photoelectric emission 373 / 60
= Work function of sodium metal + KE of the fastest 15. For hydrogen like atoms
photoelectron
= 1.82 + 0.73 = 2.55 eV 13.6
En = – Z 2 eV/atom
–13.6 eV n2
(b) We know that En = for hydrogen atom.
n 2 atom Given En – E2 = 10.2 + 17 = 27.2 eV ..(i)
Let electron jump from n2 to n1 then En – E3 = 4.24 + 5.95 = 10.2 eV
\ E3 – E2 = 17
–13.6 æ –13.6 ö
En2 – En1 = –ç 2 ÷ 13.6 2 æ 13.6 2 ö
n22 è n1 ø But E3 – E2 = – Z –ç– Z ÷
9 è 4 ø
æ 1 1ö é1 1ù
Þ 2.55 = 13.6 ç 2 – 2 ÷ = –13.6Z 2 ê – ú
è n1 n2 ø ë9 4û
Modern Physics P-S- 285

é 4 – 9 ù 13.6 ´ 5 2 l
= –13.6 Z 2 ê = Z and 2.5 Å = (n + 1)
ë 36 úû 36 2
2.5 n + 1 5 n + 1
13.6 ´ 5 2 \ = , = or n = 4
\ Z = 17 Þ Z = 3 2 n 4 n
36
Hence, from equation (1),
13.6 2 é 13.6 2 ù l
En – E2 = – ´ 3 – ê– ´3 ú 2Å = 4 i.e., l = 1Å
n2 ë 22 û 2
Now, de broglie wavelength is given by
é 9 9ù é 4 – n2 ù
= –13.6 ê 2 – ú = –13.6 ´ 9 ê ú ...(ii) h h2
ën 4û 2
êë 4n úû l= or K =
2mK l 2 .2m
From eq. (i) and (ii),
(6.63 ´ 10 –34 ) 2
é 4 – n2 ù \ K= eV
–13.6 ´ 9 ê ú = 27.2 (1 ´ 10 –10 ) 2 ´ 2 ´ 9.1 ´ 10 –31 ´ 1.6 ´ 10 –19
2
ëê 4n ûú
Þ – 122.4 (4 – n2) = 108.8n2 (6.63)2
= ´ 102 eV = 151 eV
8 ´ 9.1 ´ 1.6
489.6 d will be minimum, when
Þ n2 = = 36 Þ n = 6
13.6
l 1Å
16. (i) En = – 3.4 eV n = 1, dmin == = 0.5Å
2 2
The kinetic energy is equal to the magnitude of total energy
19. The reaction involved in a-decay is
in this case.
248 244 4
\ K.E. = + 3.4 eV 96 Cm ® 94 Pu + 2 He
(ii) The de Broglie wavelength of electron Mass defect
Dm = Mass of 96 248 244 4
h 6.64 ´ 10 –34 Cm – Mass of 94 Pu – Mass of 2 He
l= = eV
2mK = (248.072220 – 244.064100 – 4.002603) u
2 ´ 9.1 ´ 10 –31 ´ 3.4 ´ 1.6 ´ 10 –19 = 0.005517u
= 0.66 × 10–9 m Therefore, energy released in a-decay will be
17. (i) From the given information, it is clear that half life of Ea = (0.005517 × 931) MeV = 5.136 MeV
the radioactive nuclei is 10 sec (since half the amount is Similarly, Efission = 200 MeV (given)
consumed in 10 second. 12.5% is half of 25% pls. note). 1
Mean life is given as tmean = 1013 s =
Mean life l
1 1 t 10 \ Disintegration constant l = 10–13 s–1
t= = = 1/ 2 = Rate of decay at the moment when number of nuclei are
l 0.693/ t1/ 2 0.693 0.693 = 14.43 sec. 1020 is
N 6.25 dN
= lN = (10 –13 )(1020 ) = 107 dps
(ii) N = N 0 e -lt Þ N = 100 dt
0
Of these distintegrations, 8% are in fission and 92% are in
l = 0.0693s -1 a-decay.
Therefore, energy released per second
6.25 100 = (0.08 × 107 × 200 + 0.92 × 107 × 5.136) MeV
= e -0.0693t Þ e +0.0693t = = 16
100 6.25 = 2.074 × 108 MeV
\ Power output (in watt) = Energy released per second (J/s)
2.733 = (2.074 × 108) (1.6 × 10–13)
0.0693t = ln 16 = 2.773 or t = = 40 sec.
0.0693 \ Power output = 3.32 × 10–5 watt.
18. As nodes are formed at each of the atomic sites, hence A
20. Rate of production = a Rate of decay = lN
æ lö
2Å = n ç ÷ ...(1)
è 2ø
[ Q Distance between successive nodes = l/2]
o
2A
N N N N N N
N N N N N o (n+1) loops
n loops 2.5A t = time
N = Number of radioactive nuclei
EBD_7036
P-S- 286 Topic-wise Solved Papers - PHYSICS
(a) Let at time ‘t’ number of radioactive nuclei are N. DE = E5 – E4 = [– 2.18 – (– 3.4)] eV = 1.28 eV
Net rate of formation of nuclei of A. n=5®n=3
dN dN DE = E5– E3 = [– 2.18 – (– 6.04)] eV = 3.84 eV
= a – l N or = dt n=5®n=2
dt a – lN
DE = E5 – E2 = [– 2.18 – (– 13.6)] eV = 11.4 eV
t
N dN n=4®n=3
or ò = ò dt DE = E4 – E3 = [– 3.4 – (– 6.04)] eV = 2.64 eV
N 0 a – lN
0 Hence, the photons that are likely to be emitted in the range
Solving this equation, we get of 2 eV to 4 eV are 3.4 eV, 3.84 eV and 2.64 eV.
1é 22. Energy for an orbit of hydrogen like atoms is
N= a - (a - lN 0 )e -lt ù ........(1)
lë û 13.6 Z 2
En = –
(b) Substituting a = 2lN0 and n2
ln(2) For transition from 2n orbit to 1 orbit
t = t1/ 2 = in equation (1),
l æ
2 1 1 ö
Maximum energy = 13.6Z ç – ÷
we get, N =
3
N0 è 1 (2n)2 ø
2
(ii) Substituting a = 2l N0 and t ® ¥ in equation (1), we get æ1 1 ö
Þ 204 = 13.6 Z 2 ç – 2 ÷ ...(i)
a è 1 4n ø
N= = 2 N0 .
l Also for transition 2n ® n.
21. The energy of the incident photon is æ 1 1 ö æ 3 ö
40.8 = 13.6 Z 2 ç – Þ 40.8 = 13.6 Z 2 ç
–15
hc (4.14 ´ 10 eVs)(3 ´ 10 m / s) 8 è n 2 4n 2 ÷ø è 4 n 2 ÷ø
E1 = = = 3.1eV
l (400 ´ 10 –9 m)
Z2
The maximum kinetic energy of the emitted electrons is Þ 40.8 = 40.8 Þ 4n2 = Z2 or 2n = Z ....(ii)
Emax = E1 – W = 3.1 eV – 1.9 eV = 1.2eV 4n2
It is given that, From (i) and (ii)
æ Emitted electrons ö 2+ æ 1 ö
çè of maximum energy÷ø + 2 He ¾¾® He+ 204 = 13.6 Z 2 ç1 – = 13.6 Z 2 – 13.6
in 4th excited state è Z 2 ÷ø
+ photon 13.6Z 2 = 204 + 13.6 = 217.6
The fourth excited state implies that the electron enters in
217.6 Z 4
the n =5 state. Z2 = = 16, Z = 4, n = = = 2
In this state its energy is 13.6 2 2
orbit no. = 2n = 4
(13.6eV) Z 2 (13.6eV)(2)2 For minimum energy = Transition from 4 to 3.
E5 = – =–
n2 52
æ 1 1ö æ 7 ö
= – 2.18 eV. E = 13.6 ´ 4 2 ç – ÷ = 13.6 ´ 42 ç
The energy of the emitted photon in the above combination è32
4 ø
2 è 9 ´ 16 ÷ø
reaction is = 10.5 eV.
E = Emax + (– E5) = 1.2 eV + 2.18 eV = 3.4 eV Hence n = 2, Z = 4, Emin = 10.5 eV
Note : After the recombination reaction, the electron may 23. No. of photons/sec
undergo transition from a higher level to a lower level
thereby emitting photons. Energy incident on platinum surface per sec ond
=
The energies in the electronic levels of He+ are Energy of one photon
( –13.6eV ) (22 ) = –3.4 eV No. of photons incident per second
E4 =
42 2 ´ 10 ´ 10-4
= –19
= 1.18 ´ 1014
( –13.6eV) (2 2
) 10.6 ´ 1.6 ´ 10
E3 = 2
= –6.04 eV As 0.53% of incident photon can eject photoelectrons
3
\ No. of photoelectrons ejected per second
( –13.6eV) (22 ) = –13.6 eV 0.53
E2 = = 1.18 ´ 1014 ´ = 6.25 ´ 1011
22 100
The possible transitions are Minimum energy = 0 eV,
n=5®n=4 Maximum energy = (10.6 – 5.6) eV = 5 eV
Modern Physics P-S- 287

24. The formula for h of power will be T1/ 2 = 10sec T1/ 2 = 30sec
26. X ¾¾¾¾¾¾ ® Y ¾¾¾¾¾¾ ®Z
P l x = 0.1s -1 1 -1
h = out ly = s
Pin 30
The rate of equation for the population of X, Y and Z will be
Pout 1000 ´ 106
\ Pin = = = 1010 W dN x
h 0.1 = –l x N x ...(i)
Energy required for this power is given by dt
E =P×t dN y
= 1010 × 86,400 × 365 × 10 = –l y N y + l x N x ...(ii)
dt
= 3.1536 × 1018 J
200 × 1.6 × 10–13 J of energy is released by 1 fission dN z
= –l y N y ...(iii)
\ 3.1536 × 1018 J of energy is released by dt
Þ On integration, we get
3.1536 ´ 1018
fission N x = N 0e – l xt ...(iv)
200 ´ 1.6 ´ 10–13
= 0.9855 × 1029 fission Given
= 0.985 × 1029 of U235 atoms. l x N 0 é –l y t
Ny = e – el x t ù
6.023 × 1023 atoms of Uranium has mass 235g l x – lY ëê ûú
\ 0.9855 × 1029 atoms of Uranium has
To determine the maximum NY, we find
29
235 ´ 0.9855 ´ 10 dNY
23 g = 38451 kg =0
6.023 ´ 10 dt
25. Let the reaction be From (ii)
A A–4 4 -l y N y + l x N x = 0
Z X ® Z –2Y + 2 He
Here, my = 223.61 amu and ma = 4.002 amu Þ l x Nx = l y N y .......(v)
We know that

( )úúû
é l x N0 -l t ù
h h2 Þ l x ( N 0 e -l xt ) = l y ê e y - e l xt
l= Þ m2 v 2 = 2 = p 2 l
êë x - l y
mv l
-l y t
p2 h2 lx - l y e - e -l xt lx ( l -l )t
= Þ = Þ =e x y
Þ But K.E. = . Therefore K.E. ...(i) ly
2m 2ml 2 ly e -l xt
Applying eq. (i) for Y and a, we get lx
(6.6 ´ 10 –34 )2
Þ log e
ly
(
= lx - ly t )
K.E.a =
2 ´ 4.002 ´ 1.67 ´ 10 –27 ´ 5.76 ´ 10 –15 ´ 5.76 ´ 10 –15
= 0.0982243 × 10–11 = 0.982 × 10–12J é æ 1 öù
log e ê0.1/ ç ÷ ú
Similarly (K.E.)Y = 0.0178 × 10–12 J log e ( l x / lg ) ë è 30 ø û
Þt= = = 15 log e 3
Total energy = 10–12 J l x – lg 1
0.1 –
We know that E = Dmc2 30
E 10 –12 \ N x = N 0 e –0.1(15 log e 3) = N 0 elog e (3
–1.5
)
\ Dm = 2
=
8 2
kg
c (3 ´ 10 )
1.65 × 10–27kg = 1 amu 1020
Þ N x = N0 3–1.5 =
3 3
10 –12 10 –12 amu
Q kg =
(3 ´ 108 )2 1.67 ´ 10 –27 ´ (3 ´ 108 ) 2 dN y l x N x 1020
Since, = 0 at t = 15 loge 3, \ N y = =
dt ly 3
10 –12 amu
= = 0.00665 amu
1.67 ´ 9 ´ 10 –27 ´ 1016 and Nz = N0 – Nx – Ny
The mass of the parent nucleus X will be æ 1020 ö 1020 æ 3 3 – 4ö
mx = my + ma + Dm = 1020 – ç ÷– = 1020 ç ÷
è3 3ø 3 è 3 3 ø
= 223.61 + 4.002 + 0.00665 = 227.62 amu
EBD_7036
P-S- 288 Topic-wise Solved Papers - PHYSICS
27. (a) If x is the difference in quantum number of the states For transition from L shell to K shell
than x+1C2 = 6 Þ x = 3 b = 1, n2 = 2, n1 = 1
n+3 é1 1 ù
\ ( Z –1)2 Rhc ê – ú = hn
ë1 4 û
On putting the value of R = 1.1 × 107 m –1 (given),
c = 3 × 108 m/s, we get
Z = 42
n
Smallest l A
log e 0
30. l= A = 1 log n
– z 2 (13.6eV) t 2 e 0.75n
Now, we have = – 0.85eV ...(i)
n2 1 2
Þ Mean Life = =
l loge 4 / 3
– z 2 (13.6eV)
and = –0.544 eV ...(ii) 31. (a) eV0 = hn – hn0 = 5 – 3 = 2 eV
( n + 3) 2
\ V0 = 2 volt
Solving (i) and (ii) we get n = 12 and z = 3 (b) Note : When the intensity is doubled, the saturation
(b) Smallest wavelength l is given by current is also doubled.
hc
= (0.85 – 0.544) eV
l 8µA –5 2
Solving, we get l = 4052 nm. (b) I = 2 × 10 w/m
28. (a) Number of electrons falling on the metal plate A 4µA –5 2
(a) I = 10 w/m
= 1016 × (5 × 10–4)
A B Vs = – 2V

32. a = Initial Uranium atom


(a – x) = Uranium atoms left
n
æ 1ö
d = 1 cm ( a – x) = a ç ÷
è 2ø
\ Number of photoelectrons emitted from metal plate A upto
10 seconds is t 1.5 ´ 109 1
and n = = 9
=
t1/ 2 4.5 ´ 10 3
(5 ´ 104 ) ´ 1016 7
ne = ´ 10 = 5 ´ 10 1
106 \ a – x = a( )1/ 3
2
(b) Charge on plate B at t = 10 sec
Qb = 33.7 × 10–12 – 5 × 107 × 1.6 × 10–19 = 25.7 × 10–12 C a 1 21/ 3
Þ = = = 1.26
also Qa = 8 × 10–12 C a – x (1/ 2)1/ 3 1
sB s A 1 x
E= – = (QB – QA ) Þ = 1.26 – 1 = 0.26
2e 0 2e0 2 Ae0 a–x
33. KEY CONCEPT :
17.7 ´ 10–12 The wavelength l, of photon for different lines of Balmer
= = 2000 N/C
5 ´ 10 –4 ´ 8.85 ´ 10–12 series is given by
(c) K.E. of most energetic particles
= (hn – f) + e (Ed) = 23 eV hc é1 1ù
= 13.6 ê – ú eV, where n = 3, 4, 5
Note : (hn – f) is energy of photoelectrons due to light e l ë2 2
n2 û
(Ed) is the energy of photoelectrons due to work done by Using above relation, we get the value of l = 657nm, 487 nm
photoelectrons between the plates. between 450 nm and 700 nm. Since 487 nm, is smaller than
29. According to Bohr’s model, the energy released during
657 nm, electron of max. K.E. will be emitted for photon
transition from n2 to n1 is given by
corresponding to wavelength 487 nm with
é1 1ù
DE = hn = Rhc( Z – b)2 ê 2 – 2 ú (K.E.) =
hc æ 1242 ö
–W = ç – 2÷ = 0.55 eV
ëê n1 n2 úû l è 487 ø
Modern Physics P-S- 289

34. The de Broglie wave length is given by Also we know that when electron jumps from (n + 1)th orbit
to 1st orbit.
h h
l= Þl=
mv 2mK 1 é1 1 ù é 1 ù
= RZ 2 ê 2 – ú = 1.09 ´ 10 7 2
Z ê 1– ú
Case (i) 0 £ x £ 1 l êë1 (n + 1) 2 úû êë (n + 1)2 úû
For this, potential energy is E0 (given) ...(ii)
Total energy = 2E0 (given) From (i) and (ii)
\ Kinetic energy = 2E0 – E0 = E0
Z 7 2
é 1 ù
h = 1.09 ´ 10 Z ê 1– 2ú
l1 = 2p (0.529 ´ 10 –10 )( n + 1) ëê (n + 1) ûú
2mE0 ...(i)
On solving, we get n = 24
Case (ii) x > 1
For this, potential energy = 0 (given) F. Match the Following
Here also total energy = 2E0 (given) 1. A ® p; B ® t; C ® u; D ® r
\ Kinetic energy = 2E0 The correct match is as follows :
(A) Energy of thermal neutrons (p) 0.025 eV
h (B) Energy of X-rays (t) 10 k eV
\ l2 = ....(ii)
2m(2 E0 ) (C) Binding energy per nucleon (u) 8 M eV
(D) Photoelectric threshold (r) 3 eV
Dividing (i) and (ii) of a metal
l1 2 E0 l 2. A ® p, q; B ® p, r; C ® p, s; D ® p, q, r
= Þ 1 = 2 In a nuclear fusion reaction matter is converted into energy
l2 E0 l2
and nuclei of low atomic number generally given this reaction.
35. (a) KEY CONCEPT : We know that radius of nucleus is In a nuclear fission reaction matter is converted into energy
given by formula and nuclei of high atomic number generally given this
r = r0A1/3 where r0 = constt, and A = mass number. reaction.
For the nucleus r1 = r041/3 3. A ® p, r
For unknown nucleus r2 = r0 (A)1/3 Reason : Characteristic X-ray are produced due to transition
of electrons from one energy level to another.
1/ 3 1/ 3 Similarly the lines in the hydrogen spectrum is obtained due
r2 æ A ö 1/ 3 æ A ö
\ = ç ÷ , (14) = çè ÷ø Þ A = 56 to transition of electrons from one energy level to another.
r1 è 4 ø 4
B ® q, s
\ No of proton = A – no. of neutrons = 56 – 30 = 26 Reason : In photoelectric effect electrons from the metal
\ Atomic number = 26 surface are emitted out upon the incidence of light of
appropriate frequency.
é1 1ù
(b) We know that n = Rc (Z – b)2 ê 2 – 2 ú Note : In b-decay, electrons are emitted from the nucleus of
ëê n1 n2 ûú an atom.
C®p
Here, R = 1.1 × 107 , c = 3 × 108, Z = 26 Moseley gave a law which related frequency of emitted X-
b = 1 (for Ka), n1 = 1, n2 = 2 ray with the atomic number of the target material
é1 1 ù n = a( Z – b)
\ n = 1.1 × 107 × 3 × 108 [26 – 1]2 ê – ú
ë1 4 û D®q
3 In photoelectric effect, energy of photons of incident ray
= 3.3 ´ 1015 ´ 25 ´ 25 ´= 1.546 ´ 1018 Hz gets converted into kinetic energy of emitted electrons.
4
4. A ® p, q, t; B ® q; C ® s; D ® s
36. Note : nth line of Lyman series means electron jumping
(p) When an uncharged capacitor is connected to a battery,
from (n + 1)th orbit to 1st orbit.
it becomes charged and energy is stored in the
For an electron to revolve in (n + 1)th orbit. capacitor. (A) is the correct option.
2pr = (n + 1)l (q) When a gas in an adiabatic container fitted with an
2p 2p 2
adiabatic piston is compressed by pushing the piston
Þ l= ´r = é 0.529 ´ 10 –10 ù (n + 1) (i) the internal energy of the system increases
(n + 1) (n + 1) ë û Z
DU = Q - W = 0 - (- PdV ) = + PdV
1 Z (ii) Mechanical energy is proceeded to the piston
Þ =
l 2p é0.529 ´ 10 –10 ù ( n + 1) ..(i) which is converted into kinetic energy of the gas
ë û molecules.
EBD_7036
P-S- 290 Topic-wise Solved Papers - PHYSICS
(r) None of the options in column I matches. As the gas in 2. (c) After collision with hydrogen atom the He+ ion is in its
a rigid container gets cooled, the internal energy of the third excited state (n = 4). After that the electron can
system will decrease. The average kinetic energy per jump into n = 3.
molecule will decrease. hc
DE = hn = = E4 - E3
(s) When a heavy nucleus initially at rest splits into two l
nuclei of nearly equal masses and some neutrons are é -13.6 ´ 4 æ -13.6 ´ 4 ö ù -19
emitted then =ê -ç ÷ø ú ´ 1.6 ´ 10
ë 16 è 9 û
(i) Internal energy of the system is converted into
-34
mechanical energy (precisely speaking kinetic \ 6.6 ´ 10 ´ 3 ´ 108 é 1 1ù
= -13.6 ´ 4 ê - ú
energy) and l ë16 9 û
(ii) Mass of the system decreases which converts
6.6 ´ 10-34 ´ 3 ´ 108 ´ 9 ´ 16
into energy. l= = 4.68 ´ 10-7 m
× × × 7 ´ 13.6 ´ 4 ´ 1.6 ´ 10-19
Since only one option is correct, we need not work out
the case of electron jumping from n = 4 to n = 2 .
× × ×
+13.6 ´ 12
3. (a) K. E. for hydrogen atom (for n = 2) = eV
4
× × ×
13.6 ´ 22
(t) When a resistive wire loops is placed in a time varying K. E for He+ (for n = 2) = = 13.6 eV
magnetic field perpendicular to its palne. 22
(i) Induced current shows in the loop due to which the 1
\Ratio =
energy of system is increased. 4
15
5. (c) 8 O ¾® 15
7 N+
0
1b 4. (d) The collection of 12 H nuclei and electron is known as
+
b particle plasma which is formed due to high temperature inside
238 234 4 the reactor core.
92 U ¾® 90 Th + 2 He 5. (a) Applying conservation of mechanical energy we get
a- particle
Loss of kinetic energy of two deuteron nuclei
185
¾® 184 1 = Gain in their potential energy.
83 Bi 82 Pb + 1 H
proton 1 e´e
2 ´ 1.5kT =
239 4pe0 r
94 Pu ¾® 140 99
57 La + 37 X
(c) is the correct option. æ eV ö (1.44 ´ 10-9 eVm)
Þ 2 ´ 1.5 ´ ç 8.6 ´ 10-5 ÷ ´ T =
19. A ® r, t; B ® p, s; C ® p, q, r, t; D ® p, q, r, t è k ø 4 ´ 10-15 m
Based on facts 1.44 ´ 10-9
Þ T= -15
= 1.4 × 109K
-5
G. Comprehension Based Questions 2 ´ 1.5 ´ 8.6 ´ 10 ´ 4 ´ 10
6. (b) For the reading B we get nto > 5 × 1014 which is the
-13.6 Z2 Lawson criterion for a reactor to work successfully.
1. (c) For hydrogen like atoms E n = eV / atom
n2 h p2
7. (a) l= and E =
For hydrogen atom p 2m
E1 = -13.6eV n loops
(Z = 1)
E2 = – 3.4eV x=0 x=a
\ DE = E2 – E1 = – 3.4 – (– 13.6) = 10.2 eV
i.e., when hydrogen comes to ground state it will release
10.2 eV of energy. h2
Þ E=
For He+ion 2ml 2
E1 = – 13.6 × 4 eV = – 54.4 eV The length in which the particle is restricted to move is
(Z = 2)
l
E2 = – 13.6 eV a. This length is a multiple of .
E3 = – 6.04 eV 2
E4 = – 3.4eV l 2a
Now, n = a Þ l =
Here He+ ion is in the first excited state i.e., possessing 2 n
energy – 13.6 eV. After receiving energy of + 10.2 eV
h 2 n2 n 2 h2
from excited hydrogen atom on collision, the energy of Þ E= =
electron will be (–13.6 + 10.2) eV = –3.4 eV. This means 2 m ´ 4a 2 8m a 2
that the electron will jump to n = 4.
Modern Physics P-S- 291

maximum energy of anti-neutrino is nearly 0.8 × 106 eV.


Þ E µ a -2 for a particular value of n.
210 206
8. (b) For ground state n = 1, 15. (a) 84 Po ¾® 82 Pb + 42 He
Given m = 1.0 × 10–30 kg, a = 6.6 × 10–9 m Here Dm = [209.982876– (205.974455 + 4.002603)]× 932 MeV
12 ´ (6.6 ´ 10 -34 )2 = 5.422 MeV = 5422 keV
\ E= J = 8 meV By conservation of linear momentum
8 ´1 ´10 -30 ´ (6.6 ´10-9 ) 2 Linear momentum of a-particle = linear momentum of
h h h lead
9. (d) l = Þ l= Þ mv =
p mv l pa = plead
nl 2a 2ma K.E a = 2m lead K.E lead
But =a Þ l=
2 n m ´ K.E lead 206 ´ K.E lead
nh \K.E = lead = ...(1)
\ mv = nh Þ v = Þ v µn ma 4
2a 2 am Also K.Ea + K.Elead = 5422 keV ...(2)
nh On solving the above two equations we get
10. (d) According to Bohr’s quantisation principle L =
2p K.Ea = 5319 keV
2
1 2 1 é Lù (a) is the correct option.
Rotational kinetic energy = Iw = I ê ú 16. (c) Only in case of c we have m3 + m4 > M¢
2 2 ëI û
(c) is the correct option.
[Q L = Iw]
In other cases of fission m1 + m2 < M and in the other
1 L2 1 n2 h 2 é h2 ù case of fusion m3 + m4 > M¢
= = ´ 2
= n 2 ê 2 ú ...(i)
2 I 2I 4p ë 8p I û
11. (b) Energy given = change in kinetic energy H. Assertion & Reason Type Questions
h2 1. (b) Statement 1 : The wavelength of characteristic X-rays
hn = K f - Ki = [22 - 12 ] [From (i)] depends on the type of atoms of which the target material is
8p 2 I made. It does not depend on the accelerating potential.
2
3h Therefore, statement I is true.
hn =
8p 2 I Statement 2 : When an electric beam strikes the target in an
3h 3 ´ 2 p ´ 10 -34 3 X-ray tube, part of the kinetic energy is converted into X-
ÞI= 2 = = ´ 10 -45 ray energy. This statement is true. But statement 2 does not
8p n 8p 2 ´ 4 ´ 1011 16
p explain statement 1.
= 1.87 × 10–46 kg m2
I. Integer Value Correct Type
12. (c) Centre of mass divides the distance between the point
masses in inverse ratio of their masses. h
1. (3) We know that, l =
m2 d m1d 2mqV
\r1 = and r2 =
m1 + m2 m1 + m2 l1 ma qa 4 2
\ = = ´ = 8 »3
Also the moment of inertia of the system is la mpqp 1 1
C
r1 r2 2. (8) We know that N = N 0e -lt
c.m dN
d \ = N 0 e –λt (–λ) = –N0 λ e –λt
dt
I = m1r12 + m2 r22
Taking log on both sides
2
é 5 -27 ù dN
ê16 ´ 3 ´ 10 ´ d ú log e = loge (- N0 l) - lt
5 ê ú dt
Þ 1.87 ´ 10 -46 = 12 ´ ´ 10 -27
3 ê 28 ´ 5 ´ 10-27 ú Comparing it with the graph line,
ë 3 û 1 -1 é AC 1 ù
é 5 ù
2 we get l = yr
2
êë BC = 2 úû
ê 12 ´ ´ 10-27 d ú
5 3
+ 16 ´ ´ 10-2 ê ú 6
3 ê 28 ´ 5 ´ 10-27 ú 5
ë 3 û A
4
Þ d = 1.3 × 10–10 m B
13. (d) K should be less than 0.8 × 106 eV as anti-neutrino will 3 C
have some energy. 2
14. (c) The energy shared between anti-neutrino and electron. 1
2 3 4 5 6 7 8
If the energy of electron is almost zero then the
EBD_7036
P-S- 292 Topic-wise Solved Papers - PHYSICS

0.693 0.693
\ T1/ 2 = = 0.693 ´ 2 = 1.386 years
N -N
- ´t
l \ 0 = 1- e
t1 2 = 1 - e -0.04 = 1 - (1 - 0.04)
t 4.16 N
N æ 1 ö T1/ 2 1 æ 1 ö 1.386 1
Now =ç ÷ Þ =ç ÷ =
N0 è 2 ø p è 2ø 8 » 0.04 = 4% [Q e- x = 1 - x x <<1]
3. (1)
8. (3) T No T No T No
dN 1 No ¾¾
® ¾¾
® ¾¾
®
We know that , = lN = N 2 4 8
dt Tmean 100% 50% 25% 12.5%
1 Three half life are required. Therefore n = 3
\ 1010 = ´N
109
hc 13.6
\ N = 1019 9. (2) = 2 + 10.2
i.e. 1019 radioactive atoms are present in the freshly prepared l n
sample.
1242 13.6
The mass of the sample = 1019 × 10–25 kg = 10–6kg = 1 mg \ = 2 + 10.2 \ n2 = 4 \ n = 2
4. (7) 90 n

Stopping potential =
1 é hc
e êë l
ù
- f ú where hc = 1240eV –nm
û 10. (2) R = –
dA
=–
d é dN ù d 2 N d No e
– = =
2 –l t
( )
dt dt êë dt úû dt 2 dt 2
1 é1240 ù 1
- 4.7 ú = [6.2 - 4.7 ] \ R = No l2 e–lt = (Nol) l e–lt = Aole–lt
e êë 200
=
û e
[Q Ao = Nol]
1
= ´ 1.5eV = 1.5V 2t
e l t
RP l P e – l Pt lP e Q 2t e 2 t 2
1 q 1 ne = = ´ = =
But V = = \ RQ l – lQ t l
lQ e P t t 2t e
4pe 0 r 4pe 0 r Q e
et
Vr (4pe0 ) 1.5 ´ 10 -2 \ n=2
\n= =
e 9 ´109 ´1.6 ´10 -19
3h
\ n = 1.04 × 107
11. (2) Given mvr = Þn=3
Comparing it with A × 10z we get, z = 7 2p
5. (7) Loss in K.E. of proton = Gain in potential energy
hr 3h é h ù
of the proton – nucleus system \
l
=
2p êQ l = mv ú
1 1 q1 q2 ë û
mv2 =
2 4 p e0 r 2pr 2 é n2 ù é n2 ù
2 \ l = 3 = 3 p ê a0 z ú êQ r = a0 ú
p 1 q1 q2 êë úû êë z úû
\ =
2m 4 p Î0 r
2 é3 ´ 3ù
1 æ h2 ö 1 q1 q2 \ l= p a0 ê 3 ú = 2pa0
\ ç 2÷ = 3 ë û
2m è l ø 4 p Î0 r \ p=2
2
4 p Î0 r . h
\ l= = 7 fm 12. (9) Maximum kinetic energy of b-particle
q1 q2 (2 m)
6. (1) For photoelectric effect Vo = [mass of 12 12
5 B – mass of 6 C ] × 931.5 – 4.041
hn f o = [12.014 – 12] × 931.5 – 4.041] = 9MeV
- = Vo
e e
The slope is hc 1.237 ´ 10-6
q n 13. (6) E= = eV = 12.75eV
h
tan θ = = constant
l 970 ´ 10 -10
e \ The energy of electron after absorbing this photon
\ The ratio will be 1. = –13.6 + 12.75 = – 0.85eV
7. (4) For a radioactive decay This corresponds to n = 4
N = N 0 e-lt n(n - 1) 4(4 - 1)
Number of spectral line = = =6
2 2
N N
\ = e - l t \1 - = 1 - e-l t
N0 N0
Modern Physics P-S- 293

Section-B JEE Main/ AIEEE


1. (c) KEY CONCEPT : 12. (a) The electromagnetic spectrum is as follows
The energy of nth orbit of hydrogen is given by ¾¾¾¾¾¾¾¾¾¾¾¾¾¾¾¾¾¾¾¾¾¾®
increasing wavelength
g - rays × x-rays × UV rays × visible rays × IR rays × microwaves × radiowaves
13.6
En = - eV /atom
n
2 \ g-rays has least wavelength
-13.6 13. (c) Here, conservation of linear momentum can be applied
For n = 2, En = = -3.4 eV
4
Therefore the energy required to remove electron from 4
n = 2 is + 3.4 eV. 234 2 He
Th
90
2. (c) Pure silicon, at absolute zero, will contain all the
electrons in bounded state. The conduction band will
be empty. So there will be no free electrons (in
conduction band) and holes (in valence band) due to
4
thermal agitation. Pure silicon will act as insulator. 238 × 0 = 4 u + 234 v Þ \ v = - u
3. (a) Charged particles are deflected in magnetic field. 234
4. (c) We know that work function is the energy required r 4
\ speed =| v |= u
and energy E = hu 234
E Na h u Na l Cu é 1 ù 14. (b) When the temperature increases, certain bounded
\ = = êëQ u µ l for light úû electrons become free which tend to promote
E Cu h uCu l Na
conductivity. Simultaneously number of collisions
l Na E Cu 4.5 2 between electrons and positive kernels increases
\ l = E = 2.3 » 1
Cu Na 1 A 1 5000
5. (a) Formation of covalent bond is best explained by 15. (a) l = log e o = log e = 0.4 log e 2
t A 5 1250
molecular orbital theory.
16. (b) The number of a - particles released =8
6. (a) After every half-life, the mass of the substance reduces
to half its initial value. Therefore the atomic number should decrease by 16

5 years N 5 years N 5 years N The number of b - -particles released = 4


N 0 ¾¾¾¾ ® 0 ¾¾¾¾ ® 0 ¾¾¾¾ ® 0
2 2 8
2 Therefore the atomic number should increase by 4.
7. (c) Specific resistance is resistivity which is given by
Also the number of b+ particles released is 2, which
m should decrease the atomic number by 2.
r=
ne 2 t Therefore the final atomic number is
where n = no. of free electrons per unit volume 92 –16 + 4 – 2 = 78
and t = average relaxation time 17. (a) For one photocathode
For a conductor with rise in temperature n increases
1 2
and t decreases. But decrease in t is more dominant hf1 - W = mv ....(i)
than increase in n resulting an increase in the value of 2 1
r. For another photo cathode
For a semiconductor with rise in temperature, n
1 2
increases and t decreases. But the increase in n is more hf 2 - W = mv ....(ii)
dominant than decrease in t resulting in a decrease in 2 2
the value of r. Subtracting (ii) from (i) we get
8. (c) The energy band gap is maximum in insulators. 1 2 1
9. (a) Emitter sends the majority charge carrriers towards the ( hf1 - W ) – ( hf 2 - W ) = mv - mv
2 1 2 2
2

collector. Therefore emitter is most heavily doped. m 2 2


\ h( f1 - f 2 ) = (v 1 - v 2 )
10. (c) b -rays are fast moving beam of electrons. 2
11. (c) The resistance of metal (like Cu) decreases with 2 2h 2
\ v1 - v 2 =(f - f )
decrease in temperature whereas the resistance of a m 1 2
semi-conductor (like Ge) increases with decrease in 18. (a) The radioactive substances emit a -particles (Helium
temperature.
nucleus), b – particles (electrons) and neutrinoes.
EBD_7036
P-S- 294 Topic-wise Solved Papers - PHYSICS
3 3
19. (d) The average kinetic energy per molecule = kT m 1 r1 1 æ r1 ö æ 1 ö 1/ 3
2 Þ 1 = Þ 3
= Þ ç r ÷ = çè 2 ÷ø
m2 2 r2 2 è 2ø
This kinetic energy should be able to provide the
repulsive potential energy
3 -14
28. (a) The nuclear reaction of process is 212 H ®42 He
\ kT = 7.7 ´ 10
2 Energy released = 4 × (7) – 4(1.1) = 23.6 MeV
29. (a) KEY CONCEPT :
2 ´ 7.7 ´ 10 -14 9
ÞT = -23
= 3.7 ´ 10 Distance of closest approach
3 ´ 1.38 ´ 10
20. (b) The ionisation potential increases from left to right in a Ze(2e)
r0 =
period and decreases from top to bottom in a group. 4pe 0 E
Therefore ceasium will have the lowest ionisation
potential. Energy, E = 5 ´ 106 ´ 1.6 ´ 10-19 J
21. (c) The wavelength of spectrum is given by 9 -19 -19
\ r0 = 9 ´ 10 ´ (92 ´ 1.6 ´ 10 ) (2 ´ 1.6 ´ 10 )
æ 7
1 2 1 1ö where R =
1.097 ´ 10 5 ´ 106 ´ 1.6 ´ 10 -19
= Rz ç - ÷ m
l çè n 2 n 2 ÷ø 1+
1 2
M Þ r = 5.2 ´ 10 -14 m = 5.3 × 10–12 cm
where m = mass of electron 30. (d) Electrons move from base to emmitter.
M = mass of nucleus. 31. (d) In common emitter configuration current gain
For different M, R is different and therefore l is different - hf e -50
22. (a) As in reverse bias, the current through the 0000 is zero Ai = = = –48.78
through the electric field is also zero.
1 + boe RL 1 + 25 ´ 10-6 ´ 1 ´ 103
32. (d) Copper is a conductor, so its resistance decreases on
23. (a) 13.6 2
En = - Z eV/atom decreasing temperature as thermal agitation decreases,;
n2 whereas germanium is semiconductor therefore on
For lithium ion Z = 3 ; n = 2 ( for first excited state) decreasing temperature resistance increases.
13.6 33. (b) Pauli’s exclusion principle.
En = - ´ 32 = -30.6 eV 34. (a) Both the depletion region and barrier height is reduced.
22
E 35. (b) KEY CONCEPT : R = R0 ( A)1/ 3
24. (b) Momentum of photon =
c 1/ 3 1/ 3
R1 æA ö æ 27 ö 3
2E \ = ç 1÷ =ç =
Change in momentum = R2 è A2 ø è 125 ÷ø 5
c
= momentum transferred to the surface 5
(the photon will reflect with same magnitude of R2 = ´ 3.6 = 6 fermi
3
momentum in opposite direction)
25. (d) From Equation K .E = hn - f 7
36. (d) of Cu decays in 15 minutes.
8
slope of graph of K.E & n is h (Plank's constant)
which is same for all metals 3
7 1 æ1ö
26. (a) For the longest wavelength to emit photo electron \ Cu undecayed = N = 1 – = = ç ÷
8 8 è2ø
hc hc \ No. of half lifes = 3
=fÞl =
l f t 15
n= or 3 =
T T
6.63 ´ 10-34 ´ 3 ´ 108
Þl= = 310 nm 15
40 ´ 1.6 ´ 10 -16 Þ T = half life period = = 5 minutes
3
27. (b) From conservation of momentum m1v1 = m2 v2
2
1 ær ö
I1 1
æ m ö æv ö v 37. (a) I µ ; = çç 2 ÷÷ =
2 I
Þ ç 1 ÷ = ç 2 ÷ given 1 = 2 r 2 è r1 ø 4
è m2 ø è v1 ø v2
I 2 ® 4 times I1
Modern Physics P-S- 295

When intensity becomes 4 times, no. of photoelectrons


1
emitted would increase by 4 times, since number of 44. (d) Input frequency, f = 50 Hz Þ T =
electrons emitted per second is directly proportional 50
to intensity. T 1
38. (d) Band gap = energy of photon of wavelength 2480 nm. For full wave rectifier, T1 =
= Þ f1 = 100 Hz.
2 100
So,
45. (a) IC = 5.488 mA, Ie = 5.6 mA, IB = IE – IC
hc æ 6.63 ´ 10 - 34 ´ 3 ´ 108 ö 1
DE = =ç ÷´ eV Ic 5.488
l ç 2480 ´ 10 -9 ÷ -19 b= = = 49
è ø 1.6 ´ 10
IB 5.6 - 5.485
= 0.5 eV
46. (a) f = 6.2 eV = 6.2 ´ 1.6 ´10 -19 J
39. (c) KEY CONCEPT : Intensity I = I 0 . e - md ,
Applying logarithm on both sides, hc
V = 5 volt, - f = eV0
l
æ I ö
- md = log ç ÷
6.6 ´ 10 - ´ 3 ´ 10
34 8
è I0 ø hc -7
Þl= = » 10 m
f + eV0 1.6 ´ 10 -19 (6.2 + 5)
æ I /8ö
- m ´ 36 = logç ÷ ..........(i)
è I ø This range lies in ultra violet range.
47. (c) Work done to stop the a particle is equal to K.E.
æ I / 2ö
-m ´ d = log ç ...........(ii) 1 2 K (Ze) 1 2
è I ÷ø \ qV = mv Þ q ´ = mv
2 r 2
Dividing (i) by (ii),
2
2(2e) K ( Ze) 4 KZe
æ 1ö æ 1ö Þr= =
log ç ÷ 3log ç ÷ mv
2
mv
2
36 è 8ø è 2ø 36
= = = 3 or d = = 12 mm
d æ 1ö æ 1ö 3 1 1
log ç ÷ log ç ÷ Þrµ and r µ .
è 2ø è 2ø 2 m
v
40. (d) Zero; In common base amplifier circuit, input and output 48. (b) The order of time is 10–105.
voltage are in the same phase.
7
49. (c) 3 Li + 11p ¾
¾® 8
4 Be + 0
0g
é 1 1 ù
41. (b) KEY CONCEPT : E = Rhc ê 2 - 2 ú 50. (c) The range of energy of b-particles is from zero to some
êë n1 n2 úû maximum value.
51. (b) Van der Waal's bonding is attributed to the attractive
E will be maximum for the transition for which forces between molecules of a liquid. The conductivity
é 1 of semiconductors (covalent bonding) and insulators
1 ù is maximum. Here n is the higher energy
ê - ú 2 (ionic bonding) increases with increase in temperature
2 2
êë n1 n2 úû while that of metals (metallic bonding) decreases.
level. Ie ne eAve 7 7 ve v 5
52. (c) = Þ = ´ Þ e =
é 1 1 ù Ih nh eAvh 4 5 vh vh 4
Clearly, ê - ú is maximum for the third
êë n12 n2 2 úû 53. (b) D2 is forward biased whereas D1 is reversed biased.
So effective resistance of the circuit
transition, i.e. 2 ® 1 . I transition represents the R = 4 + 2 = 6W
absorption of energy. 12
42. (d) de-Broglie wavelength, \i = =2A
6
h h 1 54. (d) p-side connected to low potential and n-side is
l= = \ lµ
p 2. m.(K.E) K. E connected to high potential.
55. (b) As l decreases, n increases and hence the speed of
l photoelectron increases. The chances of photo electron
If K.E is doubled, wavelength becomes to meet the anode increases and hence photo electric
2
current increases.
43. (a) A
+ 0 n1 ¾¾
® 3 Li7 + 2 He 4 56. (b) Let E be the energy of proton, then
ZX
On comparison, E + 7 ´ 5.6 = 2 ´ [4 ´ 7.06]
A = 7 + 4 – 1 = 10, z = 3 + 2 – 0 = 5 Þ E = 56.48 - 39.2 = 17.28MeV
It is boron 5B10
EBD_7036
P-S- 296 Topic-wise Solved Papers - PHYSICS
57. (c) The risk posed to a human being by any radiation 66. (d) We have to find the frequency of emitted photons. For
exposure depends partly upon the absorbed dose, the emission of photons the transition must take place from
amount of energy absorbed per gram of tissue. a higher energy level to a lower energy level which are
Absorbed dose is expressed in rad. A rad is equal to given only in options (c) and (d).
100 ergs of energy absorbed by 1 gram of tissue. The Frequency is given by
more modern, internationally adopted unit is the gray
(named after the English medical physicist L. H. Gray); æ 1 1ö
one gray equals 100 rad. hn = -13.6 ç - ÷
çè n 2 n 2 ÷ø
58. (c) A crystal structure is composed of a unit cell, a set of 2 1
atoms arranged in a particular way; which is periodically For transition from n = 6 to n = 2,
repeated in three dimensions on a lattice. The spacing
between unit cells in various directions is called its -13.6 æ 1 1 ö 2 æ 13.6 ö
lattice parameters or constants. Increasing these lattice n1 = ç - 2÷ = ´ç ÷
constants will increase or widen the band-gap (Eg),
h è6 2
2 ø 9 è h ø
which means more energy would be required by For transition from n = 2 to n = 1,
electrons to reach the conduction band from the
valence band. Automatically Ec and Ev decreases. -13.6 æ 1 1 ö 3 æ 13.6 ö
n2 = ç - 2÷ = ´ç ÷.
59. (a) Erms = 720 h è2 1 ø 4 è h ø
2

The average total energy density


\ n1 > n2
1 2 1 2 2 67. (b) Using Bragg's equation 2d sinq = nl
= Î E = Î0 [ 2 E rms ] =Î0 Erms
2 0 0 2 Here n = 1, q = 90 – i = 90 – 30 = 60°
-12 2 -6 3 \ 2d sin q = l .......(i)
= 8.85 ´ 10 ´ (720) = 4.58 ´ 10 J/m
60. (c) Binding energy
= [ZMP + (A – Z)MN – M]c2
= [8MP + (17 – 8)MN – M]c2
= [8MP + 9MN – M]c2
= [8MP + 9MN – Mo]c2 q
61. (c) There is no change in the proton number and the
neutron number as the g-emission takes place as a result
of excitation or de-excitation of nuclei. g-rays have no
charge or mass.
62. (a) The current will flow through RL when the diode is 12.27 -10
forward biased. Also, l = ´ 10 m .......(ii)
V
63. (a) Energy of a photon of frequency n is given by E = h n .
Also, E = mc2, mc2 = hn 12.27
From (i) & (ii) 2 ´ 10 -10 ´ sin 60o = ´ 10 -10
hn hn V
Þ mc = Þ p=
C c 2
(12.27)
64. (c) According to question, V= = 50 V
3
Half life of X, T1/2 = tav , average life of Y
68. (b) 2d cos i = nldB
0.693 1 69. (d) We know that energy is released when heavy nuclei
Þ = Þ l X = (0.693). l Y
lX lY undergo fission or light nuclei undergo fusion.
Therefore statement (1) is correct.
\ lX < lY . The second statement is false because for heavy nuclei
Now, the rate of decay is given by the binding energy per nucleon decreases with
increasing Z and for light nuclei, B.E/nucleon increases
dN
- = lN with increasing Z.
dt
70. (a) It is a n-p-n transistor with R as base.
\ Y will decay faster than X. [Q N is some]
2
65. (a) Si and Ge are semiconductors but C is an insulator. k k mv
Also, the conductivity of Si and Ge is more than C 71. (b) When F = = centripetal force, then =
r r r
because the valence electrons of Si, Ge and C lie in
Þ mv2 = constat Þ kinetic energy is constant
third, fourth and second orbit respectively.
Þ T is independent of n.
Modern Physics P-S- 297

72. (d) A A B C A A -12 4


1 1 1 81. (b) Z X ¾¾¾® z -8 Y + 3 2 X e + 0e
t
1 0 1
C
0 1 1
A – 12 – (Z – 8)
B 0 0 0 A- Z -Y
\ Required ratio =
The truth table for the above logic gate is : Z -8
This truth table follows the boolean algebra C = A + B 82. (a) The final boolean expression is,
which is for OR gate
73 (c) It is given that transition from the state n =4 to n = 3 in X = ( A . B ) = A + B = A + B Þ OR gate
a hydrogen like atom result in ultraviolet radiation. For
infrared radiation the energy gap should be less. The nhn
83. (a) Power, P =
only option is 5 ® 4 . t
74. (a) l = 400 nm, hc = 1240 eV.nm, K.E. =1.68 eV P ´t
Þ n=
hc hc nh
We know that - W = K .E Þ W = - K .E
l l
4 ´ 103 ´ 1
1240 = = 6 × 1016 Hz
Þ W= - 1.68 = 3.1 – 1.68 = 1.42 eV 10 20 ´ 6.63 ´ 10-34
400
84. (b) For long distance communication, sky wave signals
75. (d) For A + B ® C + e, e is positive. This is because Eb are used.
Also, the state of ionosphere varies every time. So,
for C is greater than the E b for A and B. both statements are correct.
Again for F ® D + E + e, e is positive. This is 85. (b) Energy of excitation,
because Eb for D and E is greater than Eb for F . æ 1 1 ö
D E = 13.6 Z2 çç - ÷ eV
2
è n1 n22 ÷ø
76. (d) Here y = ( A + B ) = A.B = A × B . Thus it is an AND gate
for which truth table is æ1 1ö
Þ DE = 13.6 (3)2 çè - ÷ = 108.8 eV
1 2
32 ø
A B y 86. (b) Number of undecayed atom after time t2 ;
0 0 0 N0
= N 0 e -lt2 ...(i)
0 1 0 3
1 0 0 Number of undecayed atom after time t1;
1 1 1 2N0
= N 0 e -lt1 ...(ii)
3
77. (b) We know that a single p-n junction diode connected to
an a-c source acts as a half wave rectifier [Forward -lt 1
From (i), e 2 =
biased in one half cycle and reverse biased in the other 3
half cycle]. æ 1ö
78. (d) We know that Þ –lt2 = loge çè ÷ø ...(iii)
3
eV0 = K max = hn - f 2
From (ii) – e -lt2 =
3
where, f is the work function .
Hence, as n increases (note that frequency of X-rays æ 2ö
Þ –lt1 = loge çè 3 ÷ø ...(iv)
is greater than that of U.V. rays), both V0 and Kmax
increase. So statement - 1 is correct Solving (iii) and (iv), we get
79. (c) In nuclear fission, the binding energy per nucleon of t2 – t1 = 20 min
daughter nuclei is greater than the parent nucleus. 87. (c) By Einstein photoelectric equation,
80. (b) By conservation of energy, Kmax = eV0 = hv – hv0
2.M 2 1 2M 2 When v is doubled, Kmax and V0 become more than
( M + Dm) c2 = c + . v ,
double.
2 2 2
where v is the speed of the daughter nuclei 88. (d) The possible number of the spectral lines is given

M 2 2 Dm n(n - 1) 4(4 - 1)
2
Þ Dmc = v \v=c = = =6
2 M 2 2
EBD_7036
P-S- 298 Topic-wise Solved Papers - PHYSICS
89. (a)
1 L2
E = (m1r12 + m2 r2 2 ) 2
A 2 I
Y2 = A.AB
nh
Y = A.AB B.AB L=n (According Bohr's Hypothesis)
Y1 = AB 2n

B 1 L2
Y3 = B.AB E = (m1r12 + m2 r22 )
2 (m1r12 + m2 r22 )2
By expanding this Boolen expression
1 L2 n 2 h2
Y = A.B + B.A E= =
2 (m1r12 + m2 r22 ) 8p2 (m1r12 + m2 r22 )
Thus the truth table for this expression should be (1).
90. (a) Let d is the maximum distance, upto it the objects
(m1 + m2 )n 2 h 2 é m2 r m2 r ù
From DAOC C E= 2 2 êQ r1 = ; r2 = ú
8 p r m1m2 ë m1 + m 2 m1 + m2 û
OC 2 = AC 2 + AO2
h 93. (b) Given : Resistance R = 100 kilo ohm = 100 × 10 3 W
(h + R)2 = d 2 + R 2 d Capacitance C = 250 picofarad = 250 × 10–12F
B
t = RC = 100 × 103 × 250 × 10–12 sec
Þ d 2 = ( h + R )2 - R 2 A q R = 2.5 × 107 × 10–12 sec
R = 2.5 × 10–5 sec
d = ( h + R )2 - R 2 O The higher frequency whcih can be detected with
tolerable distortion is
d = h2 + 2hR 1 1
f = = Hz
2pma RC 2p ´ 0.6 ´ 2.5 ´ 10 -5
d = 5002 + 2 ´ 6.4 ´ 10 6 = 80 km

91. (a) 1
® 11H + -1e 0 + n + Q
¾¾ 100 ´ 104
0n = Hz = 10.61 KHz
25 ´ 1.2p
The mass defect during the process
This condition is obtained by applying the condition
Dm = mn - mH - me that rate of decay of capacitor voltage must be equal or
= 1.6725 × 10–27 – (1.6725 × 10–27+ 9 × 10–31kg) less than the rate of decay modulated singnal voltage
= – 9 × 10–31 kg for proper detection of mdoulated signal.
The energy released during the process 94. (b) From question,
E = Dmc2 B0 = 20 nT = 20 × 10–9T
E = 9 × 10–31× 9 × 1016 = 81 × 10–15 Joules (Q velocity of light in vacuum C = 3 × 108 ms–1)
r r r
E0 = B0 ´ C
81 ´ 10 -15
E= = 0.511MeV r r r
1.6 ´ 10 –19 | E 0 |=| B | . | C |= 20 ´ 10 -9 ´ 3 ´ 108
92. (d) The energy of the system of two atoms of diatomic = 6 V/m.
1 2 95. (d) As l is increased, there will be a value of l above which
molecule E = Iw photoelectrons will be cease to come out so
2
photocurrent will become zero. Hence (d) is correct
where I = moment of inertia
answer.
L 96. (a) For same value of current higher value of voltage is
w = Angular velocity = ,
I required for higher frequency hence (1) is correct
L = Angular momentum answer.

1 n 1 é 1 1 ù
I = (m1r12 + m 2 r22 ) = = RZ2 ê -
2 97. (d) x l 2 2 ú
êë n 1 n 2 úû
1 2 2 2
Thus, E = (m1r1 + m2 r2 )w … (i) [Q n > > 1]
2
Modern Physics P-S- 299

102. (a) P n
é 1
2 1 ù
n = RCZ ê 2
- 2ú
ë (n - 1) n û For forward bias, p-side must be at higher potential
than n-side. DV = (+ )Ve
é 2n - 1 ù
2
RCZ ê 2 103. (d)

ë n (n - 1) û (1) Infrared rays are used to treat muscular strain because
98. (a) The current voltage relation of diode is these are heat rays.
(2) Radio waves are used for broadcasting because these
I = (e1000 V /T - 1) mA (given) waves have very long wavelength ranging from few
centimeters to few hundred kilometers
When, I = 5mA, e1000 V /T = 6mA
(3) X-rays are used to detect fracture of bones because
1000 they have high penetrating power but they can't
Also, dI = (e1000 V /T

T penetrate through denser medium like bones.
(By exponential function) (4) Ultraviolet rays are absorbed by ozone of the
1000 atmosphere.
= (6 mA) ´ ´ (0.01)
300 1
= 0.2 mA 104. (d) Using Uav = e0 E 2
2
1 P
99. (c) E0 = CB0 and C = m e But U av =
0 0 4 pr 2 ´ c
1 P 1
Electric energy density = e 0 E02 = m E \ 2
= e0 E 2 ´ c
2 4 pr 2

1 Bo 2 2P 2 ´ 0.1 ´ 9 ´ 109
Magnetic energy density = = mB E 20 = =
2 m0 4 pr 2 e 0 c 1 ´ 3 ´ 108
Thus, mE = mB \ E0 = 6 = 2.45V/m
Energy is equally divided between electric and magnetic
105. (a) Amplitude modulated wave consists of three
field
frequencies are wc + wm, w,wc – wm
100. (b) Radius of circular path followed by electron is given
i.e. 2005 KHz, 2000KHz, 1995 KHz
by,
ze 2 k ze 2
mu 2meV 1 2m 106. (c) U = –K ; T.E = –
r= = = V r 2 r
qB eB B e
k ze 2
2 2 K.E = . Here r decreases
B r e 2 r
Þ V= = 0.8V
2m 107. (a) Frank-Hertz experiment - Discrete energy levels of atom
For transition between 3 to 2. Photoelectric effect - Particle nature of light
Davison - Germer experiment - wave nature of electron.
æ 1 1 ö 13.6 ´ 5
E = 13.6 ç - ÷ = = 1.88eV
è 4 9ø 36
Ic Ic
Work function = 1.88 eV – 0.8 eV 108. (b, d)We know that a = and b =
Ie Ib
= 1.08 eV » 1.1eV
Also Ie = Ib + Ic
1 é 1 1 ù
101. (c) Wave number = RZ 2 ê - ú Ic
l êë n12 n 2 úû
Ic I b
\ a= = b =
Ib + Ic I 1+ b
1 1+ c
Þ lµ Ib
Z2
By question n = 1 and n 1 = 2 Option (b) and (d) are therefore correct.
109. (a) In case of an 'OR' gate the input is zero when all inputs
Then, l1 = l2 = 4l3 = 9l4
are zero. If any one input is ' 1', then the output is '1'.
EBD_7036
P-S- 300 Topic-wise Solved Papers - PHYSICS
110. (c) In amplitude modulation, the amplitude of the high No
frequency carrier wave made to vary in proportional to = N0 -
24
the amplitude of audio signal. For Bt½ = 40 min., t = 80 min, number of half lifes n = 2
Audio signal
No
\ Nuclei remaining = . Therefore nuclei decayed
Carrier wave 22

No
Amplitude modulated wave = N0 -
22

1 No1
111. (c) hn02 – hn0 = mv2 No - 4 1-
2 2 16 15 4 5
4 1 \Required ratio = No = 1 = ´ =
\ hn 0 - hn 0 = mv ' 2 No - 2 1- 16 3 4
3 2 2 4
4 4 113. (c) Graph (a) is for a simple diode.
n - n0 n - n0
v '2 3 Graph (b) is showing the V Break down used for zener
\ = \ v' = v 3
v2 n - n0 n - n0 diode.
Graph (c) is for solar cell which shows cut-off voltage
4
\ v' > v and open circuit current.
3 Graph (d) shows the variation of resistance h and
112. (b) For At½ = 20 min, t = 80 min, number of half lifes n = 4
hence current with intensity of light.
No
\ Nuclei remaining = 4 . Therefore nuclei decayed
2

You might also like